Download as pdf or txt
Download as pdf or txt
You are on page 1of 329

PREVIOUS YEAR QUESTIONS

(PYQ) WORKBOOK OF
INDIAN POLITY
AND
GOVERNANCE
For Civil Services Prelims Examination

(1100+ Topic Wise Solved Objective Questions from UPSC CSE Prelims (1991 to 2023),
CAPF & CDS EXAM (2014 to 2023) and STATE PCS Prelims Exams (1991 to 2023))

PYQ WORKBOOK

Forum Learning Centres


DELHI (ORN) MUKHERJEE PATNA HYDERABAD GURGAON
NAGAR
19, Pusa Road, 2nd 862, Banda 2nd floor, AG 1st Floor, SM Plaza, Property No. 894,
Floor, IAPL House, Bahadur Marg, Palace, E Boring RTC X Rd, Indira G.F., Saraswati
Opposite Metro Ist Floor, Dr. Canal Road, Patna, Park Road, Jawahar Vihar, Chakkarpur,
Pillar #95-96, Mukherji Nagar, Bihar - 800001 Nagar, Hyderabad, Near MG Rd
Karol Bagh, New Near Batra Telangana - Metro Station,
Delhi-05 Cinema, Delhi - 500020 Sector-28,
110009 Gurgaon, Haryana
https://blog.forumias.com | www.academy.forumias.com | 9311740400
PREVIOUS YEAR QUESTIONS (PYQ) WORKBOOK OF
INDIAN POLITY AND GOVERNANCE
For Civil Services Prelims Examination

First Edition (Print): 2024

No part of this publication may be reproduced or transmitted, in any form or by any means,
electronic, mechanical, photocopying, recording or otherwise, or stored in any retrieval system of
any nature without the written permission of the copyright holder and the publisher, application
for which shall be made to the publisher.
©Forum (Flaviant Network Pvt. Ltd.)

This book/material is sold subject to the conditions that it shall not, by way of trade or otherwise,
be lent, re-sold, hired out or otherwise circulated without the publisher's prior consent in any
form of binding or cover other than that in which it is published and without a similar condition
including this condition being imposed on the subsequent purchaser.

We are continuously working on improving our quality. If you are unhappy or happy with the
quality of this study material, want to report an error, or want to leave us feedback, you can email
us at ravi@forumias.com

© 2024 1st Print Edition | For feedback email us : ravi@forumias.com


The PYQ WORKBOOK
Dear Aspirants,

We at ForumIAS have come up with a PYQ Workbook series to provide the aspirants with a
holistic idea regarding challenges associated with the changing pattern and difficulty level
of the UPSC Prelims Exam. The PYQ Workbook includes the questions asked in different
objective-based exams conducted by Union Public Service Commission like UPSC CSE
Prelims Exam, CAPF Exam, and CDS Exam. It also includes the solved previous year's
questions from different State PCS Prelims Examinations to equip the aspirants with every
relevant fact and concept needed for their UPSC CSE Prelims exam preparation. These
workbooks are divided subject wise. The PYQ Workbook Series, along with ForumIAS's
Practicebooks and Full-Length Testbooks, will help in building conceptual clarity with
respect to different subjects as well as understanding the importance/relevance of each
topic.

Indian Polity and Governance is one of the pillars of UPSC CSE Prelims exam from
which there has been a significant number of questions each year which is evident from
the graph provided below:

POLITY
25
24
No. of Questions

21
20 19
17 17
14 15
15 17
14
10 12
10 10
6
5
2012 2014 2016 2018 2020 2022 2023
Year

Number of Indian Polity and Governance questions asked in UPSC CSE GS - 1 Prelims
from 2011 till 2023

Regarding the Indian Polity and Governance part, the focus has remained on the core topics
like Parliament, Executive, Judiciary, Fundamental Rights, DPSPs and Constitutional
Amendments etc. Though in last few years some shift has been seen:
 The Governance aspect being given more weightage like Questions on Tea board etc.
 Not just knowing the concept but a deeper understanding of the Constitution and its
provisions.
 A few laws related questions like judicial custody, parole, preventive dentation, Prevention
of Terrorism Act, 2002 and The Arms (amendment) Act, 2019 etc.
 Important judgements of Supreme Court (both in Mains as well as Prelims).
 1-2 Political Science type questions like due process of law and Constitutionalism etc.

But overall, the serious aspirants have been able to score more than 70% from this section
and have been able to clear UPSC CSE Prelims based upon that. Overall conceptual clarity as
well as basic reading of newspaper articles related to this section is simply needed.

The compiled previous year UPSC Questions matched with the official answer key would
help the aspirants in aligning their preparation as per the demand of the exam.

Happy Learning and Happy Testing!!!


Your team @ForumIAS
INDEX
UNIT 1. INTRODUCTION TO THE CONSTITUTION .....1 – 40
1.1. UPSC CSE Previous Years' Questions .....1
1.2. Other Examination Previous Years' Questions .....4
SOLUTIONS .....17

UNIT 2. FOUNDATIONS OF THE CONSTITUTION .....41 – 102


2.1. UPSC CSE Previous Years' Questions .....41
2.2. Other Examination Previous Years' Questions .....46
SOLUTIONS .....64

UNIT 3. SYSTEM OF GOVERNMENT .....103 – 123


3.1. UPSC CSE Previous Years' Questions .....103
3.2. Other Examination Previous Years' Questions .....104
SOLUTIONS .....111

UNIT 4. JUDICIARY .....124 – 150


4.1. UPSC CSE Previous Years' Questions .....124
4.2. Other Examination Previous Years' Questions .....128
SOLUTIONS .....135

UNIT 5. CENTRE AND STATE EXECUTIVES .....151 – 190


5.1. UPSC CSE Previous Years' Questions .....151
5.2. Other Examination Previous Years' Questions .....155
SOLUTIONS .....168

UNIT 6. UNION AND STATE LEGISLATURE .....191 – 253


6.1. UPSC CSE Previous Years' Questions .....191
6.2. Other Examination Previous Years' Questions .....196
SOLUTIONS .....216

UNIT 7. LOCAL GOVERNMENT – UNION TERRITORIES – .....254 – 270


SPECIAL STATUS AREAS
7.1. UPSC CSE Previous Years' Questions .....254
7.2. Other Examination Previous Years' Questions .....255
SOLUTIONS .....261
UNIT 8. CONSTITUTIONAL AND NON-CONSTITUTIONAL BODIES .....271 – 296
8.1. UPSC CSE Previous Years' Questions .....271
8.2. Other Examination Previous Years' Questions .....274
SOLUTIONS .....282

UNIT 9. MISCELLANEOUS .....297 – 323


9.1. UPSC CSE Previous Years' Questions .....297
9.2. Other Examination Previous Years' Questions .....300
SOLUTIONS .....308
INDIAN POLITY AND GOVERNANCE

INDIAN POLITY AND GOVERNANCE


INTRODUCTION TO THE CONSTITUTION
*This unit consists of questions from Historical Background and Colonial rule, Constitutent Assembly,
Salient features of the Constitution, Preamble, Schedules and Parts of the Constitution, Union and its
territory and Citizenship.

1.1. UPSC CSE Previous Years’ Questions 4. With reference to India, consider the
following statements:
1. Which one of the following statements 1. There is only one citizenship and one
best reflects the Chief purpose of the domicile.
‘Constitution’ of a country?
2. A Citizen by birth only can become the
[UPSC CSE Pre 2023]
Head of State.
(a) It determines the objective for the making
of necessary laws. 3. A foreigner once granted the citizenship
(b) It enables the creation of political offices cannot be deprived of it under any
and a government. circumstance.
(c) It defines and limits the powers of Which of the statements given above is/are
government. correct? [UPSC CSE Pre 2021]
(d) It secures social justice, social equality (a) 1 only
and social security. (b) 2 only
2. Consider the following statements in (c) 1 and 3
respect of the Constitution Day: (d) 2 and 3
Statement-I: 5. What was the exact constitutional status of
The Constitution Day is celebrated on India on 26th January, 1950
26th November every year to promote [UPSC CSE Pre 2021]
constitutional values among citizens.
(a) A Democratic Republic
Statement-II: (b) A Sovereign Democratic Republic
On 26th November, 1949, the Constituent (c) A sovereign Secular Democratic
Assembly of India set up a Drafting (d) A sovereign Socialist secular Democratic
Committee under the Chairmanship of Dr. B. Republic
R. Ambedkar to prepare a Draft Constitution
of India. 6. Constitutional government means
Which one of the following is correct in [UPSC CSE Pre 2021]
respect of the above statements? (a) A representative government of a nation
[UPSC CSE Pre 2023] with federal structure
(a) Both Statement-1 and Statement-II are (b) A government whose head enjoys
correct and Statement-1 is the correct nominal powers
explanation for Statement-I (c) A government who’s Head enjoys real
(b) Both Statement-1 and Statement-11 are powers
correct and Statement-II is not the correct (d) A government limited by the terms of the
explanation for Statement-l constitution
(c) Statement-l is correct but Statement-II is
incorrect 7. The Preamble to the Constitution of India,
(d) Statement-I is incorrect but Statement-II is [UPSC CSE Pre. 2020]
is correct (a) part of the Constitution but has no legal
3. Which one of the following factors effect
constitutes the best safeguard of liberty in (b) not a part of the Constitution and has no
a liberal democracy? [UPSC CSE Pre. 2021] legal effect either
(a) A committed judiciary (c) a part of the Constitution and has the
(b) Centralization of powers same legal effect as any other part
(c) Elected government (d) a part of the Constitution but has no legal
(d) Separation of powers effect independently of other parts

1 PYQ Workbook
INDIAN POLITY AND GOVERNANCE

8. Other than the Fundamental Rights, which 13. Consider the following statements:
of the following parts of the Constitution 1. The Parliament of India can place a
of India reflect/reflects the principles and particular law in the Ninth Schedule of
provisions of the Universal Declaration of the Constitution of India.
Human Rights (1948)? 2. The validity of a law placed in the Ninth
1. Preamble Schedule cannot be examined by any
court and no judgement can be made on
2. Directive Principles of State Policy it.
3. Fundamental Duties Which of the statements given above is/are
Select the correct answer using the code given correct? [UPSC CSE Pre. 2018]
below: [UPSC CSE Pre 2020] (a) 1 only
(a) 1 and 2 only (b) 2 only
(b) 2 only (c) Both 1 and 2
(c) 1 and 3 only (d) Neither 1 nor 2
(d) 1, 2 and 3 14. Democracy’s superior virtue lies in the fact
that it calls into activity
9. In the context of India, which one of the
[UPSC CSE Pre. 2017]
following is the characteristic appropriate
(a) the intelligence and character of ordinary
for bureaucracy? [UPSC CSE Pre. 2020] men and women.
(a) An agency for widening the scope of (b) the methods for strengthening executive
parliamentary democracy leadership.
(b) An agency for strengthening the structure (c) a superior individual with dynamism and
of federalism vision.
(c) An agency for facilitating political (d) a band of dedicated party workers.
stability and economic growth 15. The mind of the makers of the Constitution
(d) An Agency for the implementation of of India is reflected in which of the
public policy following? [UPSC CSE Pre. 2017]
(a) The Preamble
10. A constitutional government by definition
(b) The Fundamental Rights
is a [UPSC CSE Pre. 2020] (c) The Directive Principles of State Policy
(a) government by legislature (d) The Fundamental Duties
(b) popular government
16. Which one of the following objectives
(c) multi-party government is not embodied in the Preamble to the
(d) limited government Constitution of India?
11. The Ninth Schedule was introduced in the [UPSC CSE Pre. 2017]
Constitution of India during the prime (a) Liberty of thought
ministership of: [UPSC CSE Pre. 2019] (b) Economic liberty
(a) Jawaharlal Nehru (c) Liberty of expression
(d) Liberty of belief
(b) Lal Bahadur Shastri
(c) Indira Gandhi 17. One of the implications of equality in
(d) Morarji Desai society is the absence of
[UPSC CSE Pre. 2017]
12. Which of the following are regarded as the (a) Privileges
main features of the “Rule of Law”? (b) Restraints
1. Limitation of Powers (c) Competition
2. Equality before law (d) Ideology
3. People’s responsibility to the Government 18. The provisions of the fifth and Sixth
4. Liberty and civil rights Schedule in the Constitution of India are
Select the correct answer using the code given made to- [UPSC CSE Pre 2015]
below: [UPSC CSE Pre. 2018] (a) Protect the interests of Scheduled Tribes
(b) Determine the boundaries between States
(a) 1 and 3 only (c) Determine the powers, authority and
(b) 2 and 4 only responsible of Panchayats
(c) 1, 2 and 4 only (d) Protect the interests of all the border
(d) 1, 2, 3 and 4 States

PYQ Workbook 2
INDIAN POLITY AND GOVERNANCE

19. Which one of the following Schedules of the Select the correct answer using the codes
Constitution of India contains provisions given below: [UPSC CSE Pre. 2012]
regarding anti-defection? (a) 1 and 2 only
[UPSC CSE Pre. 2014] (b) 3, 4 and 5 only
(a) Second Schedule (c) 1, 2 and 5 only
(b) Fifth Schedule (d) 1, 2, 3, 4 and 5
(c) Eighth Schedule 24. Consider the following statements:
(d) Tenth Schedule 1. The discussions in the Third Round Table
20. The Government enacted, the Panchayat Conference eventually led to the passing
Extension to Scheduled Areas (PESA) Act of the Government of India Act, 1935.
in 1996. Which one of the following is not 2. The Government of India Act, 1935
identified as its objective? provided for the establishment of an All-
[UPSC CSE Pre. 2013] India Federation to be based on a Union
(a) To provided self-governance of the Provinces of British India and the
(b) To recognize traditional rights Princely States.
(c) To create autonomous regions in tribal Which of the statement(s) given above is/are
areas correct? [UPSC CSE Pre 2009]
(d) To free tribal people from exploitation (a) 1 only
(b) 2 only
21. ‘Economic Justice’ as one of the objectives
(c) Both 1 and 2
of the Indian Constitutional has been
(d) Neither 1 nor 2
provided in [UPSC CSE Pre. 2013]
(a) the Preamble and the Fundamental Rights 25. The right of the government to impose
(b) the Preamble and the Directive Principles taxes and fees is provided in which List of
of State Policy Constitution? [UPSC CSE Pre 2009]
(c) the Fundamental Rights and the Directive (a) VI Schedule
Principles of State Policy (b) VII Schedule
(d) None of the above (c) IX Schedule
(d) XI Schedule
22. In the context of India, which of the following
principles is/are, implied institutionally in 26. Which Schedule of the Constitution of
the parliamentary government? India contains special provisions for the
1. Members of the Cabinet are Members of administration and control of Scheduled
the Parliament. Areas in several states?
2. Ministers hold the office till they enjoy [UPSC CSE Pre 2008]
confidence in the Parliament. (a) Third
3. Cabinet is headed by the Head of the (b) Fifth
State. (c) Seventh
(d) Ninth
Select the correct answer using the codes
given below: [UPSC CSE Pre. 2013] 27. Consider the following:
(a) 1 and 2 only Assertion (A): According to the Wavell Plan,
(b) 3 only the member of Hindu and Muslim members
(c) 2 and 3 only in the Executive Council was to be equal.
(d) 1, 2 and 3 Reason (R): Wavell thought that this
arrangement would have avoided the partition
23. Which of the following provisions of the
of India.
Constitution of India have a bearing on
Education? Code: [UPSC CSE Pre 2007]
1. Directive Principles of State Policy. (a) Both A and R are individually true, and R
is the correct explanation of A.
2. Rural and Urban Local Bodies. (b) Both A and R are individually true, but R
3. Fifth Schedule is not the correct explanation of A.
4. Sixth Schedule (c) A is true, but R is false.
5. Seventh Schedule (d) A is false, but R is true.

3 PYQ Workbook
INDIAN POLITY AND GOVERNANCE

28. Consider the following statements: 32. In the Interim Government in 1946, the
1. Article 371-A to 371-I were inserted in Vice-President of the Executive Council was
the Constitution of India to meet regional [UPSC CSE Pre 1995]
demands of Nagaland, Assam, Manipur, (a) Jawaharlal Nehru
Andhra Pradesh, Sikkim, Mizoram, (b) Dr. S. Radhakrishnan
Arunachal Pradesh and Goa. (c) C. Rajagopalachari
2. Constitution of India and the United (d) Dr. Rajendra Prasad
States of America envisage a dual policy 33. Which of the following statement(s) is/are
(The Union and the States) but a single true in relation to Constituent Assembly?
citizenship. 1. Assembly was based on adult suffrage.
3. A Naturalized citizen of India can never 2. The assembly was a result of direct
be deprived of his citizenship. election.
Which of the statements given above is/are 3. Assembly was a multi-party body.
correct? [UPSC CSE Pre 2005] 4. Assembly worked through various
(a) 1, 2 and 3 committees.
(b) 1 and 3 only Choose the correct answer by using the code
(c) 3 only given below: [UPSC CSE Pre 1993]
(d) 1 only (a) 1 and 2 only
(b) 2 and 3 only
29. If a new state of the Indian Union is to
(c) 3 and 4 only
be created, which one of the following (d) 1, 2, 3 and 4
Schedules of the Constitution must be
amended? [UPSC CSE Pre 2001] 34. Which of the following characteristics is
(a) First common between the Federal system of
(b) Second India and America? [UPSC CSE Pre 1993]
(c) Third (a) A Single Citizenship
(d) Fourth (b) Three lists in the Constitution
(c) Dual Judicial System
30. In the following quotation: (d) A Federal Supreme Court for
“WE THE PEOPLE OF INDIA, having Interpretation of the Constitution
solemnly resolved to constitute India into a
(Sovereign, Socialist, Secular, Democratic, 1.2. Other Examination Previous Years’
Republic) and to secure to all its citizens. Questions
JUSTICE, social, economic and political; 35. Match list-I with list-II and select the
LIBERTY of thought, expression, belief, correct answer using the codes given below
faith, and worship; EQUALITY of status and the lists:
of opportunity: and to promote among them
all; FRATERNITY assuring the dignity of the List-I (Schedule) List-II (Subject)
Individual and the unity and integrity of the A. Third 1. Allocation of seats in
Nation. In our Constituent Assembly, this ‘X’ Schedule the Council of States
do hereby adopt, enact and give to ourselves B. Fourth 2. Forms of Oath or
this Constitution.” Schedule Affirmations
‘X’ stands for: [UPSC CSE Pre 1997] C. Seventh 3. Languages
(a) Twenty-sixth day of January 1950 Schedule
(b) Twenty-sixth day of November 1949 D. Eighth 4. List of subject matter
(c) Twenty-sixth day of January 1949 Schedule of laws to be made by
(d) None of the above the Parliament and
31. Who gave the idea of a Constituent Assembly by State Legislatures
firstly for the formation of Constitution for Code: [UPPCS Pre 2022]
India [UPSC CSE Pre 1996] A B C D
(a) Swaraj party in 1934 (a) 3 4 2 1
(b) Congress party in 1936 (b) 1 2 3 4
(c) Muslim League in 1942 (c) 4 3 1 2
(d) All Parties conference in 1946 (d) 2 1 4 3

PYQ Workbook 4
INDIAN POLITY AND GOVERNANCE

36. Consider the following subjects and arrange (a)Public Health State List
them into sequential order as mentioned in and Sanitation
the Constitution: (b) Census Union List
1. The Union and its territory
(c) Allocation of Second Schedule
2. Fundamental duties seats in the
3. Citizenship council of
4. Directive Principles of State Policy States
Select the correct answer from the code given (d)Anti- Tenth Schedule
below. UPPCS Pre 2022] Defection
(a) 4, 2, 3, 1 40. The concept of “A Union of States in the
(b) 1, 3, 4, 2 Indian Constitution” has been derived from
(c) 2, 4, 1, 3 [UPPCS Pre 2017]
(d) 3, 1, 2, 4 (a) The American Declaration of
37. Which one of the following is not correctly Independence
matched? [UPPCS Pre 2021] (b) The Australian Constitution
(c) The British North American Act
Provisions Sources (d) The Swiss Constitution
(a) Fundamental USA
Rights 41. Which of the following are enshrined in the
third schedule of the Indian Constitution?
(b) Directive Ireland
Principles of 1. Form of oath of office for a Minister for
State Policy the Union.
(c) Residual Australia 2. Form of oath to be made by the Chief
Powers of Justice of Supreme Court.
Center 3. Form of oath of office for the President
(c) Emergency Germany of India.
Powers 4. Form of oath to be made by a Member of
Parliament.
38. Match list-I with list-II and select the
correct answer using the codes given below Select the correct answer using the code given
the lists: below: [UPPCS Pre 2017]
(a) 1, 2 and 3 only
List-I (Article) List-II (Provision) (b) 2, 3 and 4 only
A. Article 61 1. Removal of Deputy (c) 1, 2 and 4 only
Chairman of Rajya (d) 1 2, 3 and 4
Sabha
42. Coelho case is related to which Schedule of
B. Article 67 (b) 2. Impeachment of
President the Constitution of India?
[UPPCS (Mains) 2017]
C. Article 94 3. Removal of Vice-
President (a) Seventh
D. Article 90 4. Removal of Speaker (b) Eight
(c) Ninth
Code: [UPPCS Pre 2020] (d) Tenth
A B C D
43. In which case did the Supreme Court first
(a) 2 4 3 1 declare that the Preamble is not a part of
(b) 3 2 4 1 the Constitution? [U.P. P.C.S. (Mains) 2017]
(c) 2 3 4 1
(a) Berubari
(d) 4 1 3 2
(b) Sajjan Singh
39. Which of the following is not correctly (c) Golak Nath
matched? [UPPCS Pre 2019] (d) Keshvanand Bharti

5 PYQ Workbook
INDIAN POLITY AND GOVERNANCE

44. A citizen of India will loose his or her 47. The Sixth Schedule of the Constitution of
citizenship if he or she India deals with the administration of tribal
1. renounces Indian citizenship. areas of which of the following States?
2. voluntarily acquires the citizenship of [UPPCS (Mains) 2016]
another country.
(a) Bihar, Chhattisgarh, Goa
3. marries a citizen of another country.
(b) Meghalaya, Tripura, Mizoram
4. criticizes the government.
(c) Uttarakhand, Manipur, Jharkhand
Select the correct answer using the codes (d) Nagaland, Arunachal Pradesh, Tripura
given below: [U.P. P.C.S. (Mains) 2017]
(a) 1, 2 and 3 only 48. The idea of the Preamble has been
(b) 2, 3 and 4 only borrowed in Indian Constitution from the
(c) 1 and 2 only Constitution of- [UPPCS Pre 2015]
(d) 1 and 4 only
(a) Italy
45. Match list-I with list-II and select the (b) Canada
correct answer using the codes given below (c) France
the lists (d) USA
List-I List-II 49. Match list-I with list-II and select the
A. Constitutional 1. Article 360 correct answer using the codes given below
Amendments the lists:
B. Finance 2. Article 312
Commission List-I List-II
C. Financial 3. Article 280 A. Article 76 1. Comptroller and
Emergency Auditor General of
India
D. All India 4. Article 368
Services B. Article 148 2. Jurisdiction of
Supreme Court
Code: [UPPCS Pre 2016]
C. Article 75 (1) 3. Attorney General of
A B C D India
(a) 2 3 4 1
(b) 4 3 1 2 D. Article 131 4. Appointment of
(c) 3 4 1 2 Union Ministers
(d) 1 2 3 4 Code: [UPPCS (Mains) 2015]
46. Match list-I with list-II and select the A B C D
correct answer using the codes given below (a) 1 2 3 4
the lists: (b) 3 1 4 2
List-I (Institution) List-II (c) 4 1 2 3
(Articles) (d) 2 3 4 1
A. Comptroller and 1. Article 315 50. Fourth Schedule of the Constitution of
Auditor General of India deals with UPPCS (Mains) 2015]
India
(a) Allocation of seats the Council of States
B. Finance Commission 2. Article 280 (b) Political defection
C. Administrative 3. Article 148 (c) Panchayat system
Tribunal (d) Languages
D. Union Public Service 4. Article 323 (A)
Commission 51. The Eleventh Schedule of the Constitution
is related to which of the following?
Code: [UPPCS Pre 2016]
[UPPCS Pre 2015]
A B C D
(a) 3 2 4 1 (a) Municipality
(b) 3 4 2 1 (b) Panchayati Raj
(c) 1 2 4 3 (c) Centre-State relationship
(d) 4 1 3 2 (d) Removal of corruption

PYQ Workbook 6
INDIAN POLITY AND GOVERNANCE

52. Which part of the Indian Constitution (a) The Supreme Court
has been described as the ‘Soul’ of the (b) The Constitution
Constitution? [U.P.P.C.S. (Mains) 2015] (c) The Parliament
(a) Fundamental Rights (d) Religion
(b) Directive Principles of State Policy 59. Which part of our Constitution envisages a
(c) The Preamble three-tier system of Panchayats?
(d) Right to Constitutional Remedies
[UPPCS Pre 2013]
53. Which one of the following features of (a) Part IX
citizenship in India is correct? (b) Part X
[U.P.P.C.S. Pre 2015] (c) Part XI
(a) Dual Citizenship of the State and Nation (d) Part XII
(b) Single Citizenship of a State 60. Which one of the following is not correctly
(c) Single Citizenship of whole of India matched? [UPPCS Pre 2013]
(d) Dual Citizenship of India and another
Country (a) Article 39A Equal Justice and Free
Legal Aid
54. By which one of the following Acts was the
Federal Court in India created? (b) Article 40 Organization of Village
[UPPCS Pre 2014] Panchayats
(a) Indian Council Act, 1861 (c) Article 44 Uniform Civil Code
(b) Government of India Act, 1909 (d) Article 48 Separation of Judiciary
(c) Government of India Act, 1919 from Executive
(d) None of the above
61. Who/which of the following is competent
55. Who among the following were the to prescribe conditions for acquisition of
members of the drafting committee of the citizenship? [U.P.P.C.S. (Mains) 2013]
Constitution?
(a) Election Commission
1. N. Gopalaswami (b) President
2. Jawaharlal Nehru (c) Parliament and State Legislatures jointly
3. Alladi Krishnaswamy Ayyar (d) Parliament
4. Sardar Patel
62. Which Act for the first time made it
Select the correct answer using the codes possible for Indians to take some share in
given below. [UPPCS Pre 2014] the administration of their country?
(a) 1, 3 and 4 [UPPCS Pre 2012]
(b) 1 and 4 only (a) Charter Act, 1833
(c) 1 and 3 only (b) Charter Act, 1853
(d) 2, 3 and 4
(c) Government of India Act, 1858
56. Who was the Constitution Advisor at the (d) Indian Councils Act, 1861
time of the formation of the Constitution?
63. Which part and chapter of the Indian
[UPPCS Pre 2014] Constitution deal with the legislative
(a) Dr. B. R. Ambedkar relation between the Union and the States?
(b) Dr. Rajendra Prasad [UPPCS (Mains) 2012]
(c) B. N. Rau
(a) Part XI and Chapter I
(d) K. M. Munshi
(b) Part XI and Chapter II
57. The first-day session of Indian Constituent (c) Part XII and Chapter I
Assembly was chaired by- (d) Part XII and Chapter II
[UPPCS (Mains) 2013] 64. In which of the following cases Supreme
(a) Dr. Rajendra Prasad Court held that ‘The Preamble forms part
(b) Dr. Sachchidanand Sinha of the Constitution? [U.P.P.C.S. Pre 2012]
(c) Dr. B. Ambedkar (a) Union of India Vs. Dr. Kohli
(d) Pt. Jawaharlal Nehru (b) Banarsidas Vs. State of U.P.
58. In Indian political system which one is (c) Bommai Vs. Union of India
supreme? [UPPCS (Mains) 2013] (d) Malak Singh Vs. State of Punjab

7 PYQ Workbook
INDIAN POLITY AND GOVERNANCE

65. Which one of the following Acts led to the 69. Under which of the following Acts, Dyarchy
separation of Burma from India? was introduced at Central level?
[UPPCS (Mains) 2011] [UPPCS Pre 2008]
(a) The Indian Councils Act, 1909 (a) Act of 1909
(b) Government of India Act, 1919
(c) Government of India Act, 1935 (b) Government of India Act, 1919
(d) Indian Independence Act, 1947 (c) Government of India Act, 1935
(d) Indian Independence Act, 1947
66. Who amongst the following advised that
the Indian National Congress should be 70. The Indian Legislature was made bicameral
disbanded as a political party after the for the first time by [UPPCS (Mains)2008]
independence of India?
(a) Indian Council Act of 1892
[UPPCS (Mains) 2011]
(b) Indian Council Act of 1909
(a) C. Rajagopalachari
(b) Jai Prakash Narayan (c) The Government of India Act of 1919
(c) Acharya Kripalani (d) The Government of India Act of 1935
(d) Mahatma Gandhi 71. The power of the President to issue
67. Match list-I with list-II and select the ordinance is a relic of
correct answer by using the codes given [UPPCS (Mains) 2008]
below the lists:
(a) GOI Act, 1919
List-I List-II (b) GOI Act, 1935
A. Part IX of 1. Union Territories (c) GOI Act, 1909
the Indian (d) Indian Independence Act, 1947
Constitution
72. The proposal for framing of the Constitution
B. Part VIII of 2. Municipalities of India by an elected Constituent Assembly
the Indian
was made by [UPPCS (Spl) (Mains) 2008]
Constitution
(a) Simon Commission
C. Part IV A of 3. Panchayats
the Indian (b) Government of India Act, 1935
Constitution (c) Crips Mission
(d) British Cabinet Delegation
D. Part IX A of 4. Fundamental Duties
the Indian 73. Who was the Chairman of the Provincial
Constitution Constitution Committee of the Constituent
Code: [UPPCS (Mains) 2009] Assembly? [UPPCS (Mains) 2008]
A B C D (a) Dr. B. R. Ambedkar
(a) 3 1 4 2 (b) Pt. Jawaharlal Nehru
(b) 1 2 3 4 (c) Dr. Rajendra Prasad
(c) 2 4 1 3 (d) Sardar Patel
(d) 4 3 2 1
68. Consider the following statements in regard 74. The feature of Federal system of the Indian
to the Preamble of Constitution and select Constitution is inspired by the constitution
the correct one using the code given below: of- [UPPCS (Mains) 2006]
1. The objectives Resolution by Jawaharlal (a) Canada
Nehru finally became Preamble. (b) United Kingdom
2. It is non-Justiciable. (c) USA
3. It can’t be amended. (d) Ireland
4. Preamble cannot override the specific
provision of the Constitution. 75. Which one of the following describes India
Code: [U.P.P.C.S. Pre 2009] as a Secular State? [U.P.P.C.S. (Mains) 2005]
(a) Only 1 and 2 (a) Fundamental Rights
(b) Only 1, 2 and 4 (b) Preamble to the Constitution
(c) Only 1, 2 and 3 (c) 9th Schedule
(d) Only 2, 3 and 4 (d) Directive Principles

PYQ Workbook 8
INDIAN POLITY AND GOVERNANCE

76. Which among the following is the correct D. Appointment 4. Charter Act, 1833
expression of the term ‘Secular’ in India? of Law Member
[U.P.P.C.S. (Mains) 2005] in Governor-
(a) India has many religions. General Council
(b) Indians have religious freedom. Code: [UPPCS Pre 2003]
(c) Following religion depends upon the will A B C D
of an individual. (a) 1 2 3 4
(d) There is no religion of the State in India. (b) 2 1 3 4
77. The ideals and objectives outlined in the (c) 1 2 4 3
Preamble of the Indian Constitution have (d) 2 4 1 3
been further elaborated in– 80. By which of the following Act, Legislative
[U.P. P.C.S. (Mains) 2004] Council of India received the power to
(a) The Chapter on Fundamental Rights discuss the budget?
(b) The Chapter on Directive Principles of [UPPCS Pre 2003]
State Policy (a) Indian Council Act, 1861
(c) The Chapter on Directive Principles of (b) Indian Council Act, 1892
State Policy, Fundamental Rights, and (c) Indian Council Act, 1909
Fundamental Duties (d) Indian Council Act, 1919
(d) Nowhere else in the text of the 81. To give constitutional protection, state land
Constitution. reforms laws have been included in the-
78. Match the following: [UPPCS (Mains) 2003]
List-I List-II (a) 7th Schedule
(b) 9th Schedule
A. Union List 1. Police and Public (c) 8th Schedule
Order (d) 10th Schedule
B. State List 2. Space Research
82. In giving representation to talent,
C. Concurrent 3. Census experience, and service in the composition
List of the Council of States, the constitution
D. Residuary 4. Population Control makers of India were influenced by the
Subject and Family Planning example of [UPPCS Pre 1998]
Code: [UPPCS (Mains) 2004] (a) Irish Republic
A B C D (b) Canada
(a) 3 1 4 2 (c) USA
(b) 2 3 1 4 (d) Australia
(c) 1 2 3 4 83. Who was the chairman of the National Flag
(d) 4 1 2 3 Committee? [UPPCS Pre 1991]
79. Match list-I with list-II and select the (a) C. Rajagopalachari
correct answer from the code given below (b) Dr. Rajendra Prasad
the lists: (c) J. B. Kripalani
(d) Dr. B. R. Ambedkar
List-I List-II
84. Prevention of cruelty to animals is listed in
A. Establishment of 1. Regulating Act, which list of the India Constitution?
Board of Control 1773
[66th BPSC (Pre, Re-Exam) 2020]
B. Establishment of 2. Pitt’s India Act, (a) Union List
Supreme Court 1784 (b) State List
C. Permission 3. Charter Act, 1813 (c) Concurrent List
to English (d) Preamble
Missionaries to (e) None of the above/More than one of the
work in India above

9 PYQ Workbook
INDIAN POLITY AND GOVERNANCE

85. January 26 was selected as the date for the (a) Cabinet Mission
inauguration of the Constitution, because (b) Simon Commission
[53rd to 55th BPSC Pre 2011] (c) Government of India Act, 1935
(a) The Congress had observed it as the (d) Government of India Act, 1919
Independence Day in 1930.
92. Which of the following was inserted in the
(b) On that day the Quit India Movement
was started in 1942. ‘Preamble’ of the Constitution of India,
(c) It was considered to be an auspicious day. through 42nd Constitutional Amendment
(d) None of the above. Act? [Jharkhand PCS (Mains) 2016]
(a) Sovereignty
86. Land Reform comes under the subjects of- (b) Dignity of person
[43rd BPSC Pre 1999] (c) Socialist, Secular
(a) Union List (d) Privacy
(b) Concurrent List
(c) State List 93. The provision that “no bill imposing tax
(d) None of these can be introduced in the Legislature except
on the recommendation of the President” is
87. Which one of the following is matched
correctly – [UP Lower Sub. Pre 2002] covered in the Constitution of India under-
(a) Article 17 : Equality before Law [Jharkhand PCS Pre 2013]
(b) Article 78 : Establishment of (a) Article 117
Parliament (b) Article 266
(c) Article 192 : Salaries and Allowances of (c) Article 306
Parliament Members (d) Article 307
(d) Article 352 :Proclamation of Emergency
94. Concurrent List included in Indian
88. The goal of Constitution is to secure/assure Constitution is given by:
all its citizens- [Jharkhand PCS Pre 2003]
1. Justice, Social and Economic (a) The Soviet Union
2. Liberty of Thought and Expression (b) Australia
3. Equality of Opportunity (c) Italy
4. Dignity of the Individual (d) Canada
Choose the correct answer by using code:
95. Which of the following Act for the first time
[U.P. Lower Sub. Pre 1998] provided for communal representation in
(a) 1 and 2 British India? [MPPCS Pre 2019]
(b) 1, 2 and 3
(c) 2, 3 and 4 (a) Indian Councils Act, 1892
(d) All of the above (b) Morley-Minto Reforms, 1909
(c) Montague-Chelmsford Reforms, 1919
89. Indian Constituent Assembly was (d) Govt. of India Act, 1935
established under-
[Uttarakhand PCS Pre 2012] 96. Which of the following Acts introduced the
(a) Government of India Act, 1935 ‘Principle of Constitutional Autocracy’?
(b) Cripps Mission, 1942 [MPPCS Pre 2017]
(c) Cabinet Mission, 1946 (a) The Indian Councils Act of 1909
(d) Indian Independence Act, 1947 (b) The Government of India Act of 1919
90. Under the Constitution of India, Economic (c) The Government of India Act of 1935
Planning is a subject: (d) The Indian Independence Act of 1947
[Uttarakhand PCS Pre 2002] 97. Who was the Chairman of Advisory
(a) In the State List Committee on Fundamental Rights and
(b) In the Union List Minorities constituted by the Constituent
(c) In the Concurrent List Assembly? [MPPCS Pre 2014]
(d) Not Specified any list (a) Pandit Nehru
91. Which among the following introduced the (b) Sardar Patel
Provincial Autonomy in British India? (c) B. N. Rao
[Jharkhand PCS Pre 2021] (d) B. R. Ambedkar

PYQ Workbook 10
INDIAN POLITY AND GOVERNANCE

98. The Constitution of India was adopted on A B C D


which date? (a) 1 4 2 3
[MPPCS Pre 2010, MPPCS Pre 1998] (b) 4 3 1 2
(a) 26 January, 1950 (c) 1 2 3 4
(b) 26 November, 1949 (d) 3 4 1 2
(c) 26 January, 1949 102. What is/are true in relation to Autonomous
(d) 15 July, 1947 Districts?
99. Which one of the following said, “A 1. Each Autonomous District Council has
Constitution, like a machine, is a lifeless 30 members
thing. It acquires life because of the men 2. 24 members of Autonomous District
who control it, and India needs today Council are elected via voting and rest 6
nothing more than a set of honest men who are nominated by Governor
will have the interest of the country before 3. Rights to direct the Acts passed by
them”? [RAS/RTS Pre 2021] Parliament of India in Autonomous
(a) Dr. Rajendra Prasad Districts of Assam lies with Governor
(b) Jawahar Lal Nehru Code: [Chhattisgarh PCS Pre 2021]
(c) Dr. B. R. Ambedkar (a) 1, 2 and 3
(d) Mahatma Gandhi (b) 1 and 3 only
100. One amongst the following was not a salient (c) 1 and 2 only
feature of the Government of India Act, (d) 1 only
1935: [RAS/RTS Pre 2016] 103. By which Amendment of the Constitution
(a) The Act provided for an All-India of India forests were transferred from the
Federation State list to the Concurrent List?
(b) Residuary subjects were allocated to [Chhattisgarh PCS Pre 2020]
provincial Legislatures. (a) 41st
(c) It marked the beginning of the Provincial (b) 42nd
Autonomy. (c) 43rd
(d) It abolished Diarchy at the Provincial (d) 45th
level and introduced it at the Centre.
104. Match list-I with list-II and select the
101. Match List-I with list-II and select the correct answer using the codes given below
correct answer by using the codes given the lists:
below:
List-I (Subjects) List-II (Parts of
List-I List-II the Constitution)
A. First Vice- 1. V. T. A. Abolition of Titles 1. Directive
President of Krishnamachari Principles of
Constituent State Policy
Assembly B. Organisation of 2. The Union and
B. Originally the 2. Jawaharlal Nehru village panchayats its Territories
only Congress C. To provide the 3. Fundamental
Member of opportunities Rights
Drafting for education to
Committee the child/ward
C. Member of 3. K. M. Munshi between 6 to 14
Constituent years of age
Assembly D. India, that is 4. Fundamental
representing Bharat, is the Duties
Rajasthan’s Union of States
Princely State Code: [Chhattisgarh PCS Pre 2019]
D. Chairman 4. H. C. Mukherjee A B C D
of Union (a) 1 2 3 4
Constitution (b) 3 1 4 2
Committee (c) 4 3 2 1
Code: [RAS/RTS Pre 2013] (d) 2 4 1 3

11 PYQ Workbook
INDIAN POLITY AND GOVERNANCE

105. In which list of the Indian Constitution the 2. The expression ‘unity and integrity of the
‘Panchayati Raj’ subject is included? Nation’ was not there in the Preamble
[Chhattisgarh PCS Pre 2011] since the beginning.
(a) Union List Select the correct answer using the code given
(b) State List below:- [CDS 2022 (I)]
(c) Concurrent List (a) 1 only
(d) Residuary List (b) 2 only
106. Which one of the following pairs (c) Both 1 and 2
(Institutions and Articles of the (d) Neither 1 nor 2
Constitution) is properly matched? 110. Which of the following terms were added to
[Chhattisgarh PCS Pre 2008] the Preamble of the Constitution of India
(a) Supreme Court of India :Article 318 by the Constitutional Amendment, 1976?
(b) Election Commission 1. Socialist
of India :Article 324 2. Secular
(c) Union Public Service 3. Integrity
Commission :Article 332
4. Fraternity
(d) Attorney General :Article 351
Select the correct answer using the codes
107. The Citizenship Act, 1955 deals with the given below. [CDS 2020 (II)]
determination of citizenship on or after
(a) 1 and 2
[CDS 2023 (I)] (b) 1, 2 and 3
(a) 26th January, 1950 (c) 2 and 4
(b) 26th November, 1949 (d) 1, 3 and 4
(c) 15th August, 1947
(d) 14th August, 1947 111. The Citizenship (Amendment) Act falls
under which one of the following parts of
108. Consider the following pairs of Schedule the Constitution of India? [CDS 2020 (II)]
and Content of the Constitution of India: (a) Part I
1. First Schedule Forms of Oaths or (b) Part II
Affirmations (c) Part IV
2. Third Allocation of seats in (d) Part VI
Schedule the Council of States 112. Which of the following statements about
3. Fifth Schedule Provisions related to Alladi Krishnaswami Ayyar, as a drafting
the administration of member of the Constitution of India, are
Scheduled Areas and correct?
Scheduled Tribes 1. He favoured the role of the Supreme Court
4. Ninth Provisions related to in taking important decisions related to
Schedule the administration of the interpretation of the Constitution of
tribal areas in certain India.
States 2. He felt that the Supreme Court had to
How many of the above pairs is/are correctly draw the line between liberty and social
matched? [CDS 2022 (II)] control.
(a) 1 3. He believed in the dominance of the
(b) 2 executive over the judiciary
(c) 3 4. He favoured a dictatorial form of
(d) 4 governance.
109. Which of the following statements with Select the correct answer using the codes
regard to the Preamble to the Constitution given below. [CDS Pre. 2019 I]
of India is/are correct? (a) 1 and 2
1. Equality of status and equality of (b) 1, 2 and 3
opportunity find mention in the (c) 3 and 4
Preamble. (d) 1.2 and 4

PYQ Workbook 12
INDIAN POLITY AND GOVERNANCE

113. Which one of the following Schedules to the Select the correct answer using the codes
Constitution of India provides for setting given below: [CDS Pre. 2017 II]
up of Autonomous District Councils? (a) 1 and 2
[CDS Pre. 2018 II] (b) 2, 3 and 4
(a) Third Schedule (c) 1 and 4
(b) Fourth Schedule (d) 1, 2 and 4
(c) Fifth Schedule 117. Which one of the following statements
(d) Sixth Schedule with regard to the Ninth Schedule of the
114. Which of the following statements relating Constitution of India is not correct?
to the Government of India Act, 1858 is/are [CDS Pre. 2016 II]
correct? (a) It was inserted by the Constitution (First
1. The British Crown assumed sovereignty Amendment) Act, 1951
over India from the East India Company. (b) The acts and regulations specified in the
Ninth Schedule shall become void on the
2. The British Parliament enacted the first
ground that it violates AFundamental
statute for the governance of India under
Right in Part III of the Constitution.
the direct rule of the British.
(c) The supreme court has power of judicial
3. This Act was dominated by the principle review of an act included in the Ninth
of absolute imperial control without Schedule on the doctrine of basic
any population participation in the structure.
administration of the country. (d) The appropriate legislature can repeal
Select the correct answer using the codes or amend an act specified in the Ninth
given below : [CDS Pre. 2018 II] Schedule.
(a) 1 and 2 118. On 26th November, 1949, which of the
(b) Only 2 following provisions of the Constitution of
(c) 1, 2 and 3 India came into effect?
(d) 1 and 3 1. Citizenship
115. Which of the following statements relating 2. Elections
to the Indian Councils Act, 1861 is/are 3. Provisional Parliament
correct? 4. Fundamental Rights
1. The Act introduced a grain of popular Select the correct answer using the codes
element by including non-official given below: [CDS Pre. 2016 II]
members in the Governor-General’s (a) 2, 3 and 4
Executive Council (b) 1, 2 and 3
2. The members were nominated, and their (c) 1 and 3
functions were confined exclusively to (d) 1 and 2
consideration of legislative proposals
119. Which one of the following languages is not
placed before it by the Governor-General.
recognised in the Eighth Schedule to the
3. The Governor-General did not have Constitution of India? [CDS Pre. 2016 I]
effective legislative power
(a) English
Select the correct answer using the codes (b) Sanskrit
given below: [CDS Pre. 2018 II] (c) Urdu
(a) 1 and 2 (d) Nepali
(b) 2 and 3
120. Which of the following is/are not central
(c) 1, 2 and 3 tenet(s) of the Constitution of India?
(d) Only 1
1. Prohibits discrimination on grounds of
116. Which of the following features were religion.
borrowed by the Constitution of India from 2. Gives official status to certain religions.
the British Constitution? 3. Provides freedom to profess any religion.
1. Rule of Law 4. Ensures equality of all citizens within
2. Law making Procedure religious communities.
3. Independence of Judiciary Select the correct answer using the codes
4. Parliamentary System given below: [CDS Pre. 2015 II]

13 PYQ Workbook
INDIAN POLITY AND GOVERNANCE

(a) 1, 2 and 3 4. The laws in the 9th Schedule are primarily


(b) 3 and 4 those which pertain to the matters of
(c) 2, 3 and 4 national security.
(d) Only 2 Select the correct answer using the codes
121. Which of the following statement(s) about given below: [CDS Pre. 2015 I]
the formation of the Constituent Assembly (a) 1 and 2
is/are correct? (b) 2 and 3
1. The members of the Constituent (c) 3 and 4
Assembly were chosen on the basis of the (d) Only 3
Provincial Elections of 1946. 124. Consider the following statements about
2. The Constituent Assembly did not include democracy
representatives of the princely states. 1. It consists with the formation of
3. The discussions within the Constituent government elected by the people
Assembly were not influenced by opinions 2. In democracy, those currently in power
expressed by the public. have a fair chance of losing
4. In order to create a sense of collective 3. Each vote has one value.
participation. submissions were solicited Which of the statement(s) given above is/are
from the public correct? [CDS Pre. 2014 (I)]
Select the correct answer using the codes (a) 1 and 2
given below: [CDS Pre. 2015 II] (b) 1, 2 and 3
(a) Only 1 (c) Only 1
(b) 2 and 3 (d) 2 and 3
(c) 3 and 4
125. The Draft Constitution as framed only
(d) 1 and 4
provides a machinery for the government of
122. Which of the following statements with the country. It is not a contrivance to install
regard to citizenship provisions of the any particular party in power as has been
Constitution of India is/are correct? done in some countries. Who should be in
1. No person shall be a citizen of India by power is left to be determined by the people,
virtue of Article 5, or be deemed to be as it must be, if the system is to satisfy the
a citizen of India by virtue of Article 6 test of democracy’.
or Article 8, if he/she has voluntarily The above passage from Constituent Assembly
acquired the citizenship of any foreign debates is attributed to: [CDS Pre. 2014 I]
state. (a) Pandit Jawaharlal Nehru
2. The Parliament has power to make any (b) Dr BR Ambedkar
provision with respect to the acquisition (c) Maulana Abdul Kalam Azad
and termination of citizenship and all (d) Acharya JB Kriplani
other matters relating to citizenship.
126. The citizenship means
Select the correct answer using the codes 1. full civil and political rights of the
given below : [CDS Pre. 2015 II] citizens.
(a) Only 1 2. the right of suffrage for election to the
(b) Only 2 House of the People (of the Union) and
(c) Both 1 and 2 the Legislative Assembly of every state.
(d) Neither 1 nor 2
3. the right to become a Member of the
123. Which of the statement(s) is/are not correct Parliament and Member of Legislative
for the 9th Schedule of the Constitution of Assemblies.
India? Select the correct answer using the codes
1. It was inserted by the First Amendment given below : [CDS Pre. 2014 I]
in 1951. (a) 1 and 2
2. It includes those laws which are beyond (b) 1 and 3
the purview of judicial review. (c) 2 and 3
3. It was inserted by the 42nd Amendment. (d) All of these

PYQ Workbook 14
INDIAN POLITY AND GOVERNANCE

127. Who among the following warned his 131. Which of the following pairs of list and
colleagues that the Constituent Assembly contents is/are correctly matched?
was British made and was “working the 1. State List: Public health and sanitation
British plan as the British should like it to 2. Union List: Citizenship, naturalization
be worked out”? [CAPF 2021] and aliens
(a) Somnath Lahiri 3. Concurrent List: Legal, medical and other
(b) B.R. Ambedkar professions
(c) Shyama Prasad Mukherjee
(d) B.N. Rau Select the correct answer using the code given
below: [CAPF 2019]
128. Which one of the following statements with (a) 1 only
regard to the preamble to the Constitution (b) 1, 2 and 3
of India is not correct? [CAPF 2021] (c) 2 and 3 only
(a) The term ‘Secular Democratic Republic’ (d) 3 only
was not there in the Preamble originally.
(b) The Preamble contains the basic structure 132. Which of the following statements about
of the Constitution of India. the Constitution of India is/are correct?
(c) By itself, the Preamble is not enforceable 1. The ‘Objectives Resolution’ was moved by
in a court of law. Sardar Vallabhbhai Patel.
(d) The Supreme Court has held that the 2. Dr. BR Ambedkar served as the Chairman
preamble cannot be considered as an of the Drafting Committee in his capacity
aid to the legal interpretation of the as the Law Minister.
Constitution, where the language is
ambiguous. Select the correct answer using the codes
given below. [CAPF 2018]
129. As per provisions of the Constitution of (a) 1 only
India which one of the following is correct? (b) 2 only
[CAPF 2020] (c) Both 1 and 2
(a) Only Parliament has the power to legislate (d) Neither 1 nor 2
over a subject under the Concurrent list
(b) Both Parliament and State Legislatures 133. Which one of the following statements
have the power to legislate over a subject about the Constitution of India is correct?
under the Concurrent list. [CAPF 2018]
(c) A legislation made under the Concurrent (a) It was framed between December 1946
list by Parliament should be ratified by and December 1949
half of the State Legislatures so as to (b) The drafts of the Constitution were
become a law of the country discussed in public meetings
(d) Only State Legislatures are empowered to (c) It is the shortest document of a
make laws under Concurrent list constitution in the world.
130. Which of the following statement(s) (d) It came fully into operation immediately
regarding passing a law under the State list after the drafting was completed in
is/are correct? December 1949
1. Even in the sphere (State list) exclusively 134. Which one of the following is not a
reserved for the States, Parliament can characteristic feature of the Independence
legislate under certain circumstances. Act, 1947? [CAPF 2018]
2. Parliament cannot legislate a subject under (a) The Dominion of India got the residuary
the State list under any circumstances. territory of India, excluding the provinces
3. A resolution supported by two-thirds of Sind, Baluchistan, West Punjab, East
of the members present and voting is Bengal and NWFP.
required to be passed by Rajya Sabha to (b) The Act sought to lay down a Constitution
pass a law under the State list by the Legislative will of the British
Select the correct answer using the codes Parliament
given below: [CAPF 2020] (c) The Act proposed to set up two
(a) 1, 2 and 3 independent Dominions
(b) 2 and 3 only (d) The Constituent Assembly of each
(c) 1 and 3 only Dominion was to have unlimited power
(d) 1 Only to frame and adopt any Constitution.

15 PYQ Workbook
INDIAN POLITY AND GOVERNANCE

135. Which one of the following was the mandate (a) Only 3
of the Dhar Commission (1948)? (b) 1, 2 and 4
[CAPF 2017] (c) 2 and 3
(a) To study the classification of States (d) 1, 2 and 3
(b) To recommend whether the States can be 139. Which of the following are true regarding
reorganized on linguistic basis the Preamble of the Constitution of India?
(c) To study the Centre-State relations 1. The Objective Resolution was proposed
(d) To examine whether Madras city can be by Jawaharlal Nehru and passed by the
transferred to Andhra Constituent Assembly which ultimately
136. Which of the following statements regarding became the preamble.
the Constituent Assembly are correct? 2. The idea of the preamble was borrowed
1. It was not based on an adult franchise. from the Constitution of the USA.
2. It resulted from direct election. 3. The words Socialist and Secular were
3. It was a multi-party body. adopted by the 42nd Amendment in 1976.
4. It worked through several committees. 4. The preamble incorporates the
Fundamental Rights of the citizens of
Select correct answer using the code given India.
below. [CAPF 2016]
Select the correct answer using the codes
(a) 1, 2 and 4 given below. [CAPF 2014]
(b) 2 and 3
(c) 1 and 4 Codes:
(d) 1, 3 and 4 (a) 1, 2 and 3
(b) 2, 3 and 4
137. According to Granville Austin, which of (c) 1, 3 and 4
the following was/were the most significant (d) 1, 2 and 4
contributions of India to constitution-
making? [CAPF 2015] 140. Consider the following statements about
Constitutional Government.
(a) Majority Rule and Minority Rights
(b) Consensus and Accommodation 1. It is a form of limited government.
(c) Social revolution and Political Freedom 2. One where the Constitution is the basis of
(d) Asymmetrical Federal Structure public authority.
3. One where the Monarch is the formal
138. According to Article 3 of the Constitution head of the state.
of India, the Parliament may by law 4. One where there is always a universal
1. declare war on any country. adult franchise.
2. alter the boundaries of any State. Which of the statements given above are
3. increase the area of any State correct? [CAPF 2014]
4. establish an Autonomous Council within (a) 1 and 2
any State. (b) 2 and 4
Select the correct answer using the code given (c) 1, 2 and 4
below. [CAPF 2015] (d) 1 and 3

PYQ Workbook 16
INDIAN POLITY AND GOVERNANCE

SOLUTIONS

4. Solution: (a)
1.1. UPSC CSE Previous Years’ Questions
Exp) Option a is the correct answer.
1. Solution: (c) Statement 1 is correct. The Indian Constitution provides for
Exp) Option c is the correct answer only a single citizenship, that is, the Indian citizenship. The
Option c is correct: The Constitution contains the basic citizens in India owe allegiance only to the Union. There is
principles and laws of a nation that determine the powers no separate state citizenship. Also, when an Indian citizen
and duties of the government. A constitution serves multiple voluntarily acquires the citizenship of another country, his
purposes. It provides a legitimate legal and political basis Indian citizenship automatically terminates.
for the Government to propose and enact laws, organising There is only one domicile allowed in India. Domicile
public service and settling disputes. Certificate can be made only in one State/UT. In 2010, the
Uttarakhand High Court declared that there is no separate
Although the Constitution determines the objectives (DPSP)
domicile for each State and there is only one domicile for the
for the making of necessary laws and aims to promote social,
entire country.
economic and political democracy, its chief purpose is to
limit the power of government. In fact the Constitutional Statement 2 is incorrect. The President is the Head of the
government is by definition limited government. Indian State. In India, both a citizen by birth as well as a
naturalised citizen are eligible for the office of President.
2. Solution: (c) Thus, a naturalized citizen can become the Head of the State
Exp) Option c is the correct answer in India.

The Constituent Assembly of India was constituted in This is different from USA, where only a citizen by birth
November 1946 under the scheme formulated by the Cabinet (and not a naturalised citizen) is eligible for the office of
President.
Mission Plan,1946. The assembly deliberated on the matters
related to formation and provisions of the Constitution of Statement 3 is incorrect. A foreigner, who has been earlier
India. granted Indian citizenship, can be deprived of his citizenship.

Statement 1 is correct: In India, the Constitution Day Important Tips


is celebrated in India on 26 November every year to The Indian citizenship of a person can be terminated by
commemorate the adoption of the Constitution of India the Central government, if:
and also it was celebrated to promote the constitutional • he has obtained the citizenship by fraud:
values among the citizens of India.
• he has shown disloyalty to the Constitution of India:
Statement 2 is incorrect: On 29 August 1947, the • he has unlawfully traded or communicated with the
Constituent Assembly set up a Drafting Committee under enemy during a war;
the Chairmanship of Dr. B.R. Ambedkar, to prepare a
• he has, within five years after registration or
Draft Constitution for India. On 26 November 1949, the
naturalisation, been imprisoned in any country for two
Constituent Assembly of India adopted the Constitution years; and
of India, and it came into effect on 26 January 1950.
• he has been ordinarily resident out of India for seven
3. Solution: (d) years continuously.
Exp) Option d is the correct answer. 5. Solution: (b)
The Separation of powers between the legislature, the Exp) Option b is the correct answer.
executive and the judiciary constitute an important
The original Preamble, adopted by the Constituent Assembly
safeguard of liberty in a liberal democracy. The doctrine of on 26th November, 1949 and came into force on 26th January,
Separation of powers entails the division of the legislative, 1950, declared India as a “Sovereign Democratic Republic”.
executive, and judicial functions of government among
By the 42nd Amendment of 1976, enacted during the
different organs. This separation minimizes the possibility
Emergency, the words “Socialist” and “Secular” were
of arbitrary excesses by the government, since all the three
inserted. The Preamble now reads “Sovereign Socialist
organs act as check and balance on the powers of each other. Secular Democratic Republic”.
Therefore, none of the three organs can usurp the essential
functions of other organs. 6. Solution: (d)
This demarcation prevents the concentration of excessive Exp) Option d is the correct answer.
power by any branch of the Government. It thus helps to Constitutional government seeks to limit and regulate
safeguard the liberty and rights of the people in a democracy. the exercise of political power by the government.

17 PYQ Workbook
INDIAN POLITY AND GOVERNANCE

Constitutional government is by definition limited • Articles 22-27 includes the right to work, the right to
government. It means government conducted according to rest and leisure, the right to a decent standard of living,
rules and principles, which are binding on all political actors. and the right to education.
Important Tips • Articles 28-30 remind us that rights come with
A constitutional government help to constrain the obligations and that none of the rights mentioned
unfettered exercise of power by separating or dividing it. in the UDHR can be used to violate the spirit of the
Constitutional government provides a framework in which United Nations.
the government can be both responsible and representative
9. Solution: (d)
- managing conflicts, Protecting the rights, promoting
participation and maintaining security of the citizens. Exp) Option d is the correct answer.
Option d is correct. Bureaucracy is the executive arm of
7. Solution: (d)
the government. In traditional classical literature on organs
Exp) Option d is the correct answer. of government, one studied the legislature, the executive
The Preamble to a Constitution embodies the fundamental and the judiciary with bureaucracy being subsumed under
values and the philosophy, on which the Constitution is based, the executive (which comprises the political establishment
and the aims and objectives, which the founding fathers of the and the bureaucracy). Now, one finds that bureaucracy is
Constitution enjoined the polity to strive to achieve. being treated separately and this is indicative of its growing
The Supreme Court in the Berubari Union case (1960) held importance.
that the “Preamble is not a part of the Constitution.” It Bureaucracy refers to an administrative policy-
further said the Preamble can be used as a guiding principle implementation group, which is a body of non-elected
if a term in any article of the Constitution is ambiguous or
government officials. The major objective of bureaucracy
has more than one meaning.
in India has not been Parliamentary democracy’s expansion
Later in Kesavanand Bharti case (1973), the Supreme Court but rather the implementation of the decisions taken by
reversed its ruling and held that the Preamble is a part of the the executive. With All India Services at the helm of major
Constitution and can also be amended under Article 368 of the bureaucratic structure, it cannot be said that strengthening
Constitution. Preamble, therefore, is a part of the Constitution.
of federal structure is its main characteristic.
The preamble states the objects of the Constitution and is not
India’s bureaucratic system’s most important function since
enforceable in court thus has no legal effect independently of
independence has been implementation of public policy,
other parts.
making the decisions of the government reach a conclusion
8. Solution: (d) on the ground level. Parliamentary democracy results often
Exp) Option d is the correct answer. in change at the helm of executive, the static and permanent
bureaucracy provides continuity of policies and execution
Indian constitution was greatly influenced by the Universal
rather than political stability.
Declaration of Human Rights, 1948. Part III which stands
for Fundamental Rights and Part IV for Directive Principles 10. Solution: (d)
of State Policy bear a close resemblance to the Universal
Exp) Option d is the correct answer.
Declaration of Human Rights. The Preamble also provides for
social, economic, political, justice, as enshrined in UDHR and A constitutional government by definition is a limited
Article 28, 29 of UHDR, resemble the Fundamental Duties. government. A constitutional government is the one that is
defined by the constitution of the country. Constitution of
Important Tips
a country is supreme law of the land; it is empowered with
Articles under Universal Declaration of Human Rights: the sovereign authority of the people (we the people) by the
• Articles 1 and 2 reaffirms human dignity, equality and framers and the consent of the legislatures of the states. It
brotherhood. is the source of all government powers, and also provides
• Articles 3-11 are the rights of the individual: the right important limitations on the government that protect the
to life, outlawing of slavery or torture, equality before fundamental rights of citizens.
the law, the right to a fair trial etc.
11. Solution: (a)
• Articles 12-17 includes freedom of movement, the
right to a nationality, the right to marry and found a Exp) Option a is the correct answer.
family, as well as the right to own property. First Constitutional (Amendment) Act of 1951, introduced
• Articles 18-21 are the spiritual and religious rights of the Ninth Schedule in the Constitution of India during the
individuals, such as freedom of thought and conscience prime-ministership of Jawaharlal Nehru.
(religion), the right to your own opinion, the right to This amendment made laws placed in Ninth Schedule
peaceful assembly and association, and the right to immune to judicial review, even if they violate any
vote and take part in government. Fundamental Rights.

PYQ Workbook 18
INDIAN POLITY AND GOVERNANCE

Important Tips can be thought of as “power of the people”: a way of


governing which depends on the will of the people. The most
The Ninth Schedule was brought in the Indian Constitution
obvious ways to participate in government are to vote, or to
on 18 June 1951 to abolish Zamindari system. The Ninth
stand for office and become a representative of the people.
Schedule contains a list of laws that cannot be challenged
Democracy, however, is about far more than just voting, and
in courts. Out of 284 such laws which are shielded from
there are numerous other ways of engaging with politics
judicial review, 90 per cent of laws are about agriculture
and government. The effective functioning of democracy, in
and landholding.
fact, depends on ordinary people using these other means as
12. Solution: (c) much as possible. The take-off point for a democracy is the
idea of consent, i.e., the desire, approval and participation of
Exp) Option c is the correct answer.
people. It is the decision of people that creates a democratic
In order to understand the concept of rule of law, it is to government and decides about its functioning. So, since
be comprehended that the state is governed not by the democracy requires voter’s decision making- hence
ruler or the nominated representatives of the people but intelligence and character of common people are called in.
by the law. The term ‘Rule of Law’ is nowhere defined in
the Indian Constitution but this term is often used by the 15. Solution: (a)
Indian judiciary in their judgments. Rule of law has been Exp) Option a is the correct answer.
declared by the Supreme Court as one of the basic features The mind of the makers of the Constitution of India is
of the Constitution so it cannot be amended even by the reflected in the Preamble. The Preamble to a Constitution
constitutional amendment. Rule of law is seen as an integral embodies the fundamental values and the philosophy, on
part of good governance. which the Constitution is based. It embodies the aims and
The “Rule of Law” is a principle of governance which objectives, which the founding fathers of the Constitution
implies that the creation of laws, their enforcement, and enjoined the polity to strive to achieve.
the relationships among legal rules are themselves legally The Preamble embodies the basic philosophy and fundamental
regulated, so that no one—including the most highly placed values–political, moral and religious–on which the Constitution is
official—is above the law. Thus, ensuring equality before law. based. It contains the grand and noble vision of the Constituent
(Option 2 is correct) Assembly, and reflects the dreams and aspirations of the founding
People’s responsibility to the Government cannot be fathers of the Constitution. In the words of Sir Alladi Krishnaswami
attributed to “Rule of Law”. (Option 3 is incorrect) Iyer, a member of the Constituent Assembly who played a significant
Limitation of powers ensures no man is punished except role in making the Constitution, ‘The Preamble to our Constitution
for a breach of law, thus restricting discretion of the expresses what we had thought or dreamt so long’.
government of the day. It would also help in safeguarding 16. Solution: (b)
rights of the individual. (Option 1 and 4 are correct)
Exp) Option b is the correct answer.
13. Solution: (a) The text of the Preamble of the Constitution of India reads
Exp) Option a is the correct answer. as –
Statement 1 is correct. The Ninth Schedule contains a list WE, THE PEOPLE OF INDIA, having solemnly resolved to
of central and state laws. The Schedule became a part of constitute India into a SOVEREIGN SOCIALIST SECULAR
the Constitution in 1951, when the document was amended DEMOCRATIC REPUBLIC and to secure to all its citizens:
for the first time. The Parliament has the power to place a JUSTICE, social, economic and political; LIBERTY of
particular law in the Ninth Schedule of the Constitution of thought, expression, belief, faith and worship;
India.
EQUALITY of status and of opportunity;
Statement 2 is incorrect. The mandate of ninth schedule is
and to promote among them all FRATERNITY assuring the
to prevent judicial scrutiny but in a landmark ruling in IR
dignity of the individual and the unity and integrity of the
Coelho versus State of Tamil Nadu, 2007, the Supreme Court
Nation;
of India ruled that all laws (including those in the Ninth
Schedule) would be open to Judicial Review if they violated IN OUR CONSTITUENT ASSEMBLY this twenty-sixth day
of November, 1949, do HEREBY ADOPT, ENACT AND
the basic structure of the constitution. The Supreme Court
GIVE TO OURSELVES THIS CONSTITUTION.
judgment laid that the laws placed under Ninth Schedule
after April 24, 1973 shall be open to challenge in court if Thus, Preamble to the Constitution of India embodies liberty
they violated fundamental rights guaranteed under Article of thought, expression, belief, faith and worship but not
14, 19, 20 and 21 of the Constitution. economic liberty.

14. Solution: (a) 17. Solution: (a)


Exp) Option a is the correct answer. Exp) Option a is the correct answer.
The word democracy comes from the Greek word “demos”, “Equality means equal rights for all the people and the
meaning people, and “kratos” meaning power; so, democracy abolition of all special rights and privileges”. -Barker

19 PYQ Workbook
INDIAN POLITY AND GOVERNANCE

Thus, negatively equality implies abolition of all special it was meant to provide institutions of local self-governance
privileges and facilities which may be available to some classes in the Scheduled Areas and to recognise the traditional
or some persons in society. It also stands for the abolition rights of the tribals. The provisions of this Act address
of all man-made inequalities and discriminations. Positively some of the most complex problems of exploitation and
Equality stands for equal rights, equitable distribution marginalisation of the tribals.
of resources, equal opportunities for development and The act did not have any provisions of creating any
relative equality with due recognition of merit, abilities and
autonomous regions.
capacities of various persons.
Important Tips
Important Tips
The States with Fifth Schedule Areas that are covered
Privileges mean a special advantage to only a particular
under PESA are Andhra Pradesh, Chhattisgarh, Gujarat,
person/ group. Equality requires the state of being equal,
Himachal Pradesh, Jharkhand, Madhya Pradesh,
particularly in standing, rights, or opportunities and thus
Maharashtra, Odisha, Rajasthan and Telangana.
absence of any privileges.
In society there are present two types of inequalities: 21. Solution: (b)
1. Natural inequalities, and Exp) Option b is the correct answer.
2. Man-made unnatural inequalities. Economic justice denotes the non-discrimination between
The former means natural differences among human people on the basis of economic factors. It involves the
beings. These have to be accepted by all. The man-made elimination of glaring inequalities in wealth, income and
inequalities are those which are there because of some property. A combination of social justice and economic
social conditions and discriminations. These are of the justice denotes what is known as ‘distributive justice’.
nature of socioeconomic inequalities resulting from the The term ‘justice’ in the Preamble embraces three distinct
operation of social system. The discriminations practiced forms—social, economic and political, secured through
and inequalities maintained in the name of caste, colour,
various provisions of Fundamental Rights and Directive
creed, religion, sex, place of birth and the like are all
Principles.
unnatural man- made inequalities. Equality means end of
all such inequalities and discriminations. The concept of Economic Justice is also mentioned under the
Directive Principles of State Policy. Article 39 says that the
18. Solution: (a) State shall, in particular, direct its policy towards securing—
Exp) Option a is the correct answer. (a) that the citizens, men and women equally, have the right
Both the Fifth Schedule and the Sixth Schedule aim to an adequate means of livelihood; (b) that the ownership
to protect the interests of Scheduled Tribes. The Fifth and control of the material resources of the community are
Schedule of the Constitution of India contains special so distributed as best to subserve the common good; (c)
provisions for the administration and control of Scheduled that the operation of the economic system does not result
Areas in several states. Scheduled Areas are those areas where in the concentration of wealth and means of production to
a substantial number of Scheduled Tribes (STs) reside. The the common detriment; (d) that there is equal pay for equal
Sixth Schedule of the Constitution of India contains special work for both men and women.
provisions for the administration of tribal areas in the
northeastern states. 22. Solution: (a)

19. Solution: (d) Exp) Option a is the correct answer.


Exp) Option d is the correct answer. In the context of India, there are mainly two principles which
The Tenth Schedule of the Indian Constitution popularly can be implied institutionally in the Parliamentary form of
referred to as the ‘Anti-Defection Law’ was inserted by government:
the 52nd Amendment (1985) to the Constitution. It was Statement 1 is correct. A Member of the Cabinet must be a
formulated to bring stability in the Indian political system. Member of the Parliament. If a person who is not a member of
the parliament and is appointed as a minister then he/she has to
20. Solution: (c)
get elected to either of the two houses of parliament within six
Exp) Option c is the correct answer.
months.
PESA Act is a Central legislation that extends the Provisions
Statement 2 is correct. In a parliamentary form of government,
of the Panchayats, as given in Part IX of the Constitution
the council of ministers hold office till they enjoy the confidence
to the Fifth Schedule Areas with certain modifications
and exemptions. These areas have preponderance of tribal of the parliament (in India’s case that of Lok Sabha). Hence,
population. This Act is called “The Provisions of the Ministers hold the office till they enjoy confidence in the
Panchayats (Extension to the Scheduled Areas) Act, 1996”. Parliament.
In short form this is popular as “PESA”. Statement 3 is incorrect. The President is Head of the
The PESA Act was enacted to extend the provisions of 73 rd State in India, whereas the Cabinet is headed by the Prime
and 74 th Amendment Acts to the Fifth Schedule areas. Thus, Minister of India.

PYQ Workbook 20
INDIAN POLITY AND GOVERNANCE

23. Solution: (d)


3. Land revenue
Exp) Option d is the correct answer.
4. Stamp duty
All the above provisions of the Constitution of India, have a
5. Entertainment tax
bearing on Education.
6. GST (SGST)
Directive Principles of State Policy - Article 45: To provide
early childhood care and education for all children until 26. Solution: (b)
they complete the age of six years. 86th Amendment changed Exp) Option b is the correct answer.
the subject-matter of Article 45 and made elementary
The Schedule of the Constitution of India that contains
education a fundamental right under Article 21 A.
special provisions for the administration and control of
Rural and Urban Local Bodies – Education is one of the Scheduled Areas in several states is the Fifth Schedule.
subject matters under Schedules XI and XII. It empowers The Fifth Schedule is dedicated to the administration and
local bodies to take steps to improve educational standards governance of areas inhabited by Scheduled Tribes. It
in their regions. outlines the powers and functions of the tribal advisory
Fifth and Sixth Schedule – The Fifth and Sixth Schedule of councils, the governance structure, and the protection of
the constitution provide protection to the tribals on account tribal rights and customs.
of their economic disadvantages so that they could maintain
27. Solution: (c)
their tribal identity without any coercion or exploitation. This
includes promotion of education in their own language for their Exp) Option c is the correct answer.
upliftment. Assertion is correct: According to the Wavell Plan, the
Seventh Schedule - Under this, education is put under the number of Hindu and Muslim members in the Executive
concurrent list which allows both centre and states to make Council was to be equal. The Wavell Plan was a proposal
laws on the subject. initiated by the British government to resolve the deadlock
in India. It was prepared by Lord Wavell, who became the
24. Solution: (c) Viceroy of India in 1943. The plan proposed the complete
Exp) Option c is the correct answer. Indianization of the Executive Council, with seats reserved
for members on the basis of religion and caste.
Statement 1 is correct: The discussions in the Third Round
Table Conference, held in November-December 1932, Reason is incorrect: Wavell did not think that this
eventually led to the passing of the Government of India Act, arrangement would have avoided the partition of India.
1935 by the British Parliament. He was not concerned about the issue of partition of India.
Wavell’s most important task was to present a formula for
Statement 2 is correct: The Government of India Act, 1935
the solution of the Indian problem which was acceptable for
provided for the establishment of an All-India Federation
both the Congress and the Muslim League.
to be based on a Union of the Provinces of British India
and the Princely States. However, the federation never came 28. Solution: (d)
into being as the princely states did not join it. Exp) Option d is the correct answer.
25. Solution: (b) Statement 1 is correct- Article 371-A to 371-I were inserted
Exp) Option b is the correct answer. in the Constitution of India to give special status to the
North Eastern States and protect their cultural and linguistic
The right of the government to impose taxes and fees
identities.
is provided in Entry 82 of the Union List of the Seventh
Schedule of the Constitution of India. This means that the Statement 2 is incorrect- The Constitution of India follows
Parliament has exclusive power to make laws on this subject. a single citizenship principle, where every Indian citizen is
considered a citizen of the entire country. The United States,
Important Tips on the other hand, follows a dual citizenship system.
Taxes levied by the Central government: Statement 3 is incorrect- The Indian Constitution allows
1. Customs duty for the deprivation of citizenship for naturalized citizens
2. Excise duty under certain circumstances, such as acquiring citizenship
through fraud or false representation or acquiring
3. Income tax
citizenship of another country voluntarily.
4. Corporation tax
Important Tips
5. Service tax
Articles 371A to 371I of the Indian Constitution provide
6. GST (CGST)
special provisions and safeguards for certain states and
Taxes levied by the State government: regions. Here is a brief description of each article:
1. Sales tax 1. Article 371A: It grants special provisions for the state
2. Motor Vehicle tax of Nagaland.

21 PYQ Workbook
INDIAN POLITY AND GOVERNANCE

2. Article 371B: It provides special provisions for the 32. Solution: (a)
state of Assam. Exp) Option a is the correct answer.
3. Article 371C: It grants special provisions for the state Jawaharlal Nehru was the Vice-President of the Executive
of Manipur. Council in the Interim Government formed in 1946,
4. Article 371D: It provides special provisions for the under the provisions of the Cabinet Mission Plan of 1946.
state of Andhra Pradesh. The Interim Government was a provisional government
5. Article 371E: It grants special provisions for the state established on 2 September 1946, to assist the transition
of Sikkim. of India from British rule to independence. It consisted of
6. Article 371F: It provides special provisions for the 15 members, representing different political parties and
state of Mizoram. interests, headed by the Viceroy of India as the President of
7. Article 371G: It grants special provisions for the state the Executive Council. Jawaharlal Nehru was designated as
of Arunachal Pradesh. the Vice-President of the Executive Council and also held
8. Article 371H: It provides special provisions for the the portfolios of External Affairs and Commonwealth
state of Goa. Relations. He acted as the de facto Prime Minister of India
9. Article 371I: It grants special provisions for the state and led the Interim Government until 15 August 1947, when
of Jammu and Kashmir. India became independent.
29. Solution: (a) Important Tips
Exp) Option a is the correct answer. • Dr. S. Radhakrishnan was not a member of the Interim
Any change or creation of a new state would require amending Government, but he was a member of the Constituent
the First Schedule to reflect the addition or alteration in the Assembly and later became the first Vice-President and
list. The First Schedule contains the list of states and union second President of India.
territories with their territories and capitals. Articles 1 to • C. Rajagopalachari was a member of the Interim
4 are covered in the first schedule. Government and held the portfolio of Education, Arts,
Health and Lands. He later became the last Governor-
Important Tips
General of India from 1948 to 1950.
1. Article 1 of the Indian Constitution declares India as
• Dr. Rajendra Prasad was not a member of the Interim
a Union of States.
Government, but he was the President of the Constituent
2. Article 2 allows for the admission or establishment Assembly and later became the first President of India.
of new states within the Indian Union,
3. Article 3 provides the power to form new states or 33. Solution: (c)
alter existing state boundaries. Exp) Option c is the correct answer.
4. Article 4 allows for the creation or abolition of Statement 1 and 2 are incorrect: Constituent Assembly was
Legislative Councils in states. not based on adult suffrage. The members of the Constituent
30. Solution: (b) Assembly were elected by the provincial assemblies by a single,
transferable-vote system of proportional representation. The
Exp) Option b is the correct answer.
provincial assemblies themselves were elected on a limited
The word “X” in the Preamble stands for the date on which
franchise, based on property, tax or education qualifications.
the Preamble was adopted, which is November 26, 1949. The
Constituent Assembly was not a result of direct election.
Constitution of India came into force on January 26, 1950.
The members of the Constituent Assembly were elected
31. Solution: (a) indirectly by the provincial assemblies.
Exp) Option a is the correct answer. Statement 3 is correct: Constituent Assembly was a multi-
At a conference held in May 1934, the Swaraj Party passed party body. It consisted of members from various political
a resolution reiterating that the only course should be to parties and groups, such as the Indian National Congress
convene a Constituent Assembly representing all sections (INC), the All-India Muslim League (AIML), the Communist
of the Indian people to frame an “acceptable Constitution”.
Party of India (CPI), the Akhil Bharatiya Hindu Mahasabha
This was the first time that the Indian masses heard about
(ABHM), the Jana Sangh (JS), the Sikh Akali Dal (SAD), and
the demand for a Constituent Assembly
some independents
Note: In several digital sources, the Congress party in 1936
is mentioned as the correct answer. But it must be noted Statement 4 is correct: Constituent Assembly worked
that the Congress party accepted the idea of a Constituent through various committees. The Constituent Assembly
Assembly as its official demand in its session at Lucknow appointed a number of committees to deal with different
in April 1936, presided by Jawaharlal Nehru. This was after aspects of constitution-making. Out of these, eight were
the demand made by Swaraj Party in 1934. major committees and 22 were minor committees.

PYQ Workbook 22
INDIAN POLITY AND GOVERNANCE

Important Tips D. Eighth Schedule- (3) This schedule is specifically


related to Languages. Altogether there are 22 scheduled
Features of Constituent Assembly:
languages in India.
• It was formed in 1946 based on the Cabinet Mission
Plan, which proposed a three-tier federation for India, Important Tips
consisting of the provinces, the princely states and the Brief overview of the Schedules of the Indian
centre. Constitution:
• It had 389 members initially, of which 296 were elected 1. Schedule 1: The States and the Union Territories of
by the provincial assemblies and 93 were nominated India.
by the princely states. After partition, the number of
2. Schedule 2: Provisions related to emoluments,
members reduced to 299.
allowances, privileges and so on as to the President,
• It was presided over by Dr. Rajendra Prasad, who was
Vice-President, Governors, Speakers and Deputy
elected as its president. Its vice president was H.C.
Speakers of Legislatures and Judges of the Supreme
Mookerjee and its constitutional advisor was B.N. Rau
Court and High Courts.
• The Constituent Assembly consisted of members
from various political parties and groups, such as the 3. Schedule 5: The Scheduled Areas and Scheduled
Indian National Congress (INC), the All-India Muslim Tribes.
League (AIML), the Communist Party of India (CPI), 4. Schedule 6: Administration of Tribal Areas in the
the Akhil Bharatiya Hindu Mahasabha (ABHM), the States of Assam, Meghalaya, Tripura and Mizoram.
Jana Sangh (JS), the Sikh Akali Dal (SAD), and some 5. Schedule 9: It contains a list of central and state
independents
laws shielded from judicial scrutiny, added by
34. Solution: (d) the Constitution (First Amendment) Act, 1951,
Exp) Option d is the correct answer to protect agrarian reform and abolishing the
Zamindari system
The Supreme Court of the United States has power over the
entire country, but it can only pass federal laws. In India, 6. Schedule 10: Disqualification on Ground of
there is only one court system. The Supreme Court is in the Defections (Added via 52 nd Act, 1985).
Supreme Court and the higher courts in different states are 7. Schedule 11: Panchayats (Added via 73 rd Act, 1992).
in the lower courts. Both parties have the power to rule on 8. Schedule 12: Municipalities (Added via 74 th Act,
federal and state legislation. 1992).
Important Tips
36. Solution: (b)
1. Dual citizenship is offered in the USA while not in
India. Exp) Option b is the correct answer
2. In India, states are proportionately represented. Part I of the Indian Constitution is titled The Union and
whereas in the USA equal representation to all states its Territory. It includes Articles 1- 4. This part of the
so both are different. constitution contains the law in the establishment, renaming,
3. The system of Judicial Review is prevalent both in merging or altering of the borders of the states. Part II of the
India and U.S.A Constitution of India deals with the Citizenship of India. It
includes Articles 5-11. Part IV of the Indian Constitution
1.2. Other Examination Previous Years’ deals with Directive Principles of our State Policy (DPSP). It
Questions includes Articles 36-51. Part IVA of the Indian Constitution
35. Solution: (d) deals with Fundamental Duties. It includes Article 51A.

Exp) Option d is the correct answer. Important Tips


A. Third Schedule- (2) This schedule contains the text • Part I of the Indian Constitution contains the law in
of the oath or affirmation that is to be taken by the the establishment, renaming, merging or altering of
President, Vice-President, Governors, Speakers and the borders of the states.
Deputy Speakers of Legislatures, and Judges of the
Supreme Court and High Courts. • Part II of the Indian Constitution contains is related
with the citizenship.
B. Fourth Schedule- (1) This schedule is associated with
the allocation of seats in the Rajya Sabha to the states • Part IV DPSP cannot be enforced by any court, but
and the union territories. these principles are fundamental in the governance
of the country and it shall be the duty of the State to
C. Seventh Schedule- (4) The Union List (98 subjects),
the State List (59 subjects) and the Concurrent List (52
apply. The concept of Directive Principles of State
subjects). Policy was borrowed from the Irish Constitution.

23 PYQ Workbook
INDIAN POLITY AND GOVERNANCE

The concept of “A Union of States in the Indian Constitution”


• Part IV-A demonstrates the fundamental duties
has been derived from The British North-American Act.
of Indian citizens. Although, Article 51 –A of
Federation with a strong Centre is what we call A Union of
this part is where all the fundamental duties are
States in the Indian Constitution hence it is taken from the
defined. These duties are introduced under the 42nd
Canadian constitution i.e The British North-American Act.
Amendment of the constitution.
41. Solution: (c)
37. Solution: (c)
Exp) Option c is the correct answer.
Exp) Option c is the correct answer
The idea of Residuary Powers has been taken from the Statement 1 is correct- Form of oath of office for a Minister
Canadian Constitution and not Australian Constitution. for the Union is included in the third schedule. It outlines the
Other features that are borrowed from the Canadian oath that a Minister for the Union must take upon assuming
Constitution are as follows: office.
• Federation with a strong Centre Statement 2 is correct- Form of oath to be made by the Chief
• Appointment of state governors by the Centre, Justice of Supreme Court is included in the third schedule. It
specifies the oath that the Chief Justice of the Supreme Court
• Advisory jurisdiction of the Supreme Court.
of India takes while entering office.
Important Tips Statement 3 is incorrect- The oath of office for the President
• Fundamental Rights in India has been taken from the of India is mentioned separately in Article 60 of the
Constitution of the U.S. Bill of Rights. Constitution.
• Directive Principles of State Policy was borrowed from Statement 4 is correct- Form of oath to be made by a Member
the Irish Constitution. of Parliament is included in the third schedule. It provides
• The Cabinet system of the Indian Constitution is taken the oath that Members of Parliament must take when they
from the British Constitution. assume their positions.
• The concept of “A Union of States in the Indian 42. Solution: (c)
Constitution” has been derived from Canadian
Constitution. Exp) Option c is the correct answer.
In Ines Ribeiro Coelho (Dead) By L.Rs. vs. State of
38. Solution: (c) Tamil Nadu and Others on 11 January 2007, the Supre
Exp) Option c is the correct answer me Court held that laws placed in the Ninth Schedule of
Option A matches with Option 2: Article 61 of the the Constitution are not immune to judicial review. The
Constitution of India prescribes the procedure for Supreme Court held that the Kerala Land Reforms Act was
impeachment of the President. not immune to judicial review, and that it could be struck
Option B matches with Option 3: Article 67(b) deals with down if it was found to be unconstitutional.
removal of Vice President from his office by a resolution
43. Solution: (a)
of the council of States passed by a majority of all the then
members of the council and agreed to by the House of the Exp) Option a is the correct answer.
People. In the case of Berubari Union and Exchange of Enclaves, the
Option C matches with Option 4: Article 94 deals with the Supreme Court of India first declared that the Preamble is
vacation and resignation of, and removal from, the offices not a part of the Constitution. This case, which was decided
of Speaker and Deputy Speaker. in 1960, involved a dispute regarding the boundary between
Option D matches with Option 1: Article 90 deals with India and East Pakistan (now Bangladesh). The Supreme
Vacation and resignation of, and removal from, the office Court’s ruling clarified that the Preamble is not enforceable
of deputy chairman. and does not have legally binding effect.

39. Solution: (c) Important Tips


Exp) Option c is the correct answer. Very brief overview of other landmark cases:
The Second Schedule of the Constitution of India p 1. Sajjan Singh- The Seventeenth Amendment to the
rovisions related to emoluments, allowances, privileges Constitution of India was challenged in the Sajjan
and so on as to the President, Vice-President, Governors, Singh case, which introduced Article 31B granting
Speakers and Deputy Speakers of Legislatures and Judges Parliament the authority to amend any law that
of the Supreme Court and High Courts. The allocation of affected fundamental rights guaranteed by Part III of
seats in the Council of States or the Rajya Sabha is listed in the Constitution, except Articles 14, 19, and 21.
the Fourth Schedule of the Constitution of India. 2. Golak Nath- The Golak Nath case established
40. Solution: (c) limitations on Parliament’s power to amend
fundamental rights.
Exp) Option c is the correct answer

PYQ Workbook 24
INDIAN POLITY AND GOVERNANCE

Option A matches with Option 4: Article 368 is related


3. Kesavananda Bharati- The Kesavananda Bharati case
with the Power of Parliament to amend the Constitution
is a landmark case that established the doctrine of
and procedure thereof.
basic structure and affirmed the power of Parliament
to amend the Constitution. Option B matches with Option 3: Article 280 of the Indian
Constitution provides for the establishment of a Finance
44. Solution: (c) Commission as needed.
Exp) Option c is the correct answer. Option C matches with Option 1: Article 360 of the Indian
Statement 1 is correct- Any adult citizen of India can Constitution empowers the President to invoke financial
renounce their citizenship by making a declaration, resulting emergency.
in the termination of Indian citizenship upon registration. Option D matches with Option 2: Article 312 of the Indian
However, during times of war, the Central Government may Constitution explains the genesis of All India Services along
withhold the registration of such a declaration. with the genesis of the Indian Constitution.
Statement 2 is correct- A person who voluntarily acquires 46. Solution: (a)
the citizenship of another country automatically ceases to
Exp) Option a is the correct answer
be a citizen of India except during the times of war.
Option A matches with Option 3: Article 148 provides that,
Statement 3 is incorrect- Marriage to a citizen of another there shall be a Comptroller and Auditor-General of India
country does not automatically lead to the loss of citizenship who shall be appointed by the President by warrant under
of India. However, if a person marries a citizen of another his hand and seal and shall only be removed from office in
country and then voluntarily acquires the citizenship of like manner and on like grounds as a Judge of the Supreme
that country, then he or she will lose his or her citizenship Court.
of India Option B matches with Option 2: Article 280 of the Indian
Statement 4 is incorrect- Criticizing the government does Constitution provides for the establishment of a Finance
not lead to the loss of citizenship of India. A person is free Commission as needed.
to criticize the government as long as he or she does not Option C matches with Option 4: Article 323(A)provide
engage in any activities that are seditious or subversive. for the establishment of an administrative tribunal for the
Union and a separate administrative tribunal for each State
Important Tips
or for two or more States.
The Citizenship Act (1955) specifies three ways of losing
Option D matches with Option 1: Article 315 deals with
Indian citizenship: renunciation, termination, and
Public Service Commissions for the Union and for the States.
deprivation.
1. Renunciation: Any adult citizen can renounce their 47. Solution: (b)
Indian citizenship by making a declaration, resulting Exp) Option b is the correct answer.
in the loss of Indian citizenship. Minor children of
The Sixth Schedule of the Constitution of India deals with
the person also lose citizenship, but they can regain it
the administration of tribal areas in the northeastern states
upon turning eighteen.
of Assam, Meghalaya, Tripura, and Mizoram. The Sixth
2. Termination: When an Indian citizen voluntarily Schedule provides for the establishment of Autonomous
acquires citizenship of another country, their Indian District Councils (ADCs) in these areas. The ADCs have
citizenship automatically terminates, except during a degree of self-government and autonomy, and they are
times of war. responsible for matters such as education, health, and
3. Deprivation: The Central government can agriculture.
compulsorily terminate Indian citizenship if
48. Solution: (d)
acquired through:
Exp) Option d is the correct answer
fraud,
The Preamble to the Indian Constitution is based on the
• if the citizen shows disloyalty to the Indian
‘Objectives Resolution’, drafted and moved by Pandit Nehru,
Constitution,
and adopted by the Constituent Assembly. It is a borrowed
• engages in unlawful trade or communication with
feature from the Constitution of the USA. It has been
the enemy during war,
amended by the 42nd Constitutional Amendment Act
• is imprisoned in any country for two years within five (1976), which added three new words–Socialist, Secular and
years of registration or naturalization, or Integrity.
• has been continuously residing outside India for
49. Solution: (b)
seven years.
Exp) Option b is the correct answer.
45. Solution: (b) Option A matches with Option 3: Article 76 of the
Exp) Option b is the correct answer Constitution of India, 1950 carves an advisory role for

25 PYQ Workbook
INDIAN POLITY AND GOVERNANCE

Attorney General to counsel the Government in legal


10. Rural housing.
matters.
11. Drinking water.
Option B matches with Option 1: Article 148 provides for 12. Fuel and fodder.
a Comptroller and Auditor-General of India who shall be 13. Roads, culverts, bridges, ferries, waterways and other
appointed by the President by warrant under his hand and means of communication.
seal and shall only be removed from office in like manner 14. Rural electrification, including distribution of
and on like grounds as a Judge of the Supreme Court. electricity.
Option C matches with Option 4: Article 75(1) in The 15. Non-conventional energy sources.
Constitution Of India 1949 deals with (1) The Prime Minister 16. Poverty alleviation programme.
shall be appointed by the President and the other Ministers 17. Education, including primary and secondary schools.
shall be appointed by the President on the advice of the 18. Technical training and vocational education.
Prime Minister. 19. Adult and non-formal education.
Option D matches with Option 2: Article 131 of the 20. Libraries.
Constitution gives exclusive and original jurisdiction to the 21. Cultural activities
Supreme Court in matters of law between states or between 22. Markets and fairs.
states and the Union. 23. Health and sanitation, including hospitals, primary
health centers and dispensaries.
50. Solution: (a)
24. Family welfare.
Exp) Option a is the correct answer. 25. Women and child development.
The Fourth Schedule of the Indian Constitution deals 26. Social welfare, including welfare of the handicapped
with the allocation of seats in the Rajya Sabha, which is and mentally retarded.
the Council of States or the upper house of Parliament. The 27. Welfare of the weaker sections, and in particular, of
number of seats is based on the population of the state or the Scheduled Castes and the Scheduled Tribes.
union territory. The states with the largest populations have 28. Public distribution system.
the most seats in the Council of States. 29. Maintenance of community assets.

51. Solution: (b) 52. Solution: (d)


Exp) Option b is the correct answer. Exp) Option d is the correct answer.
Article 32 of the Constitution is considered very important,
The Eleventh Schedule of the Constitution of India contains
often called “the soul of the constitution.” It provides
a list of 29 matters that are to be entrusted to Panchayats
remedies to protect our fundamental rights. The Supreme
at the district, intermediate and village levels. The Eleventh Court has the power to issue orders, known as writs, like
Schedule was added to the Constitution by the 73rd habeas corpus, mandamus, certiorari, Prohibition, and
Amendment Act of 1992, which was enacted to strengthen Quo-Warranto to enforce these rights. The Supreme Court
Panchayati Raj institutions in India acts as a guardian and protector of our fundamental rights.

Important Tips 53. Solution: (c)


29 Matters listed in Eleventh Schedule. Exp) Option c is the correct answer.
1. Agriculture, including agricultural extension. While the Indian Constitution adopts a federal structure
2. Land improvement, implementation of land reforms, with dual governance (central and state), it establishes a
land consolidation and soil conservation. single citizenship, namely Indian citizenship. In India,
citizens pledge allegiance solely to the Union, and there is
3. Minor irrigation, water management and watershed
no distinct concept of state citizenship. In contrast, federal
development.
states like the USA and Switzerland have embraced the
4. Animal husbandry, dairying and poultry. concept of dual citizenship, allowing individuals to possess
5. Fisheries. citizenship at both the national and state levels.
6. Social forestry and farm forestry.
54. Solution: (d)
7. Minor forest produces.
Exp) Option d is the correct answer.
8. Small scale industries, including food processing
The Federal Court of India was created by the Government
industries.
of India Act 1935, which was an Act of the British Parliament
9. Khadi, village and cottage industries.
that introduced a federal system of governance in India. The

PYQ Workbook 26
INDIAN POLITY AND GOVERNANCE

Federal Court of India had original, appellate and advisory Important Tips
jurisdiction, and functioned until the Supreme Court of
• The constitution was made by the drafting committee
India which was established in 1950.
headed by Dr. B. R. Ambedkar.
55. Solution: (c) • The committee took 2 years, 11 months, and 17 days
Exp) Option c is the correct answer. to write.
N. Gopalaswami and Alladi Krishnaswami Ayyar were • It was ratified on 26th November 1949 and enacted on
the members of the drafting committee of the Constitution, 26th January 1950.
which was responsible for preparing a draft of the new 59. Solution: (a)
Constitution of India based on the decisions taken by the
Constituent Assembly. Exp) Option a is the correct answer
• Jawaharlal Nehru was the Chairman of both the The 73rd constitutional amendment act added Part IX to
Union Powers Committee and the Union Constitution the Indian Constitution, “The Panchayats” and it also added
Committee, which were responsible for determining the Eleventh Schedule which consists of the 29 functional
the main principles of the Union Constitution and its items of the panchayats. Part IX of the Constitution contains
structure and form. Article 243 and Article 243A to 243O and envisages a three-
• Sardar Patel was the Chairman of both the Provincial tier system of Panchayats. With the Act, Panchayati Raj
Constitution Committee and the Advisory Committee on systems come under the purview of the justiciable part of the
Fundamental Rights, Minorities and Tribal and Excluded Constitution and mandates states to adopt the system.
Areas, which were responsible for providing a model
Provincial Constitution and dealing with the issues of Important Tips
fundamental rights, minorities and tribal and excluded The 73rd constitutional amendment act has two parts:
areas in the new Constitution compulsory and voluntary.
• Compulsory provisions must be added to state laws,
56. Solution: (c) which include the creation of the new Panchayati Raj
Exp) Option c is the correct answer. systems.
B. N. Rau was a jurist and a civil servant who was appointed • Voluntary provisions, on the other hand, are the
as the Constitutional Advisor to the Constituent Assembly discretion of the state government
in 1946. He prepared the initial draft of the Constitution
based on the recommendations of various committees and 60. Solution: (d)
his study of other constitutions. He also consulted eminent Exp) Option d is the correct answer
jurists and scholars such as Sir Ivor Jennings, Sir Alladi
Article 48 of the Constitution of India directs the state to
Krishnaswamy Iyer, and K. M. Munshi for their opinions
make efforts for banning animal slaughtering of cows and
and suggestions. He played a vital role in shaping the
calves and other milch and draught cattle. Article 39A is
Constitution of India and was later appointed as a judge of
the International Court of Justice in The Hague. related to the rights of citizens to have free access to the legal
process, without any discrimination on account of class, sex,
57. Solution: (b) race and creed. Article 40 of the Indian constitution deals
Exp) Option b is the correct answer. with the organisation of village panchayat. According to
Dr. Sachchidanand Sinha was the one who chaired the Article 44, the State shall endeavour to secure for the citizens
first-day session of the Indian Constituent Assembly, which a uniform civil code throughout the territory of India.
was held on 9 December 1946. Dr. Sachchidananda Sinha,
the oldest member of the Assembly, was elected as the 61. Solution: (d)
temporary Chairman of the Assembly, as per the French Exp) Option d is the correct answer.
practice. He presided over the first-day session and delivered Article 11 of the Constitution of India gives the Parliament
his inaugural address. Dr. Rajendra Prasad was elected as the power to make laws with respect to the acquisition and
the permanent President of the Constituent Assembly on 11
termination of citizenship. This includes the power to
December 1946 and presided over its subsequent sessions.
prescribe conditions for the acquisition of citizenship.
He later became the first President of India.
62. Solution: (d)
58. Solution: (b)
Exp) Option b is the correct answer Exp) Option d is the correct answer.

In Indian political system the Constitution of India is The Indian Councils Act of 1861 was an important milestone
supreme because it enables the government to fulfill the in India’s journey towards self-governance. It introduced for
aspirations of a society and create conditions for a just the first time the concept of representation of Indians in
society. It provides a set of basic rules that allow for minimal the administration of their country. The act expanded the
coordination amongst members of society. It sets some limits legislative councils and allowed the appointment of Indians
on what a government can impose on its citizens. as non-official members.

27 PYQ Workbook
INDIAN POLITY AND GOVERNANCE

63. Solution: (a) February 2, 1948, that the Congress “has outlived its use”
Exp) Option a is the correct answer and advised “to disband the existing Congress organisation
and flower into a Lok Sevak Sangh (Society to Serve the
India is a “Union of States” according to the Constitution
People)”. He made this suggestion in the context of a debate
of India. Center-state relation is mentioned in Part XI of
on the future role of the party in the post-independence
the constitution. Chapter I of Part XI (Article 245-254)
period.
of the Indian Constitution specified two-fold division of
Legislative powers between the Union and the States: 67. Solution: (a)
1. with respect of territory Exp) Option a is the correct answer
2. with respect of subject matter Option A matches with Option 3: Part IX of the Constitution
was inserted by the Constitution (Seventy-third Amendment)
64. Solution: (c)
Act, 1992. It contains provisions for local self-government at
Exp) Option c is the correct answer. the rural level or Panchayats.
In the Bommai case, the Supreme Court held that the Option B matches with Option 1: Part VIII of the
Preamble is a part of the Constitution and that it must be Constitution contains provisions for the administration
read along with the other provisions of the Constitution. The of the Union Territories, including the National Capital
Court held that the Preamble sets out the goals and values Territory of Delhi. The Union Territories are small regions
that the Constitution is based on. The other provisions of the of strategic or historical significance which are governed
Constitution must be interpreted in a way that is consistent directly by the Union government
with the Preamble.
Option C matches with Option 4: Part IVA of the Indian
Important Tips Constitution deals with Fundamental Duties. As of now,
1. Union of India Vs. Dr. Kohli: This case pertains to there are 11 Fundamental duties.
service matters and the interpretation of constitutional Option D matches with Option 2: Part IXA of the
provisions related to promotions and seniority in Constitution was inserted by the Constitution (Seventy-
government services. fourth Amendment) Act, 1992. It contains provisions for
2. Banarsidas Vs. State of U.P.: This case involves local self-government at the urban level or Municipalities.
a dispute regarding the validity of certain land 68. Solution: (b)
acquisition proceedings.
Exp) Option b is the correct answer.
3. Bommai Vs. Union of India: This landmark case dealt
Statement 1 is correct- The Preamble of the Indian
with the issue of the dismissal of state governments
under Article 356 of the Constitution. The Constitution is based on the “Objectives Resolution”
Supreme Court held that the power to dismiss state drafted by Jawaharlal Nehru and adopted by the Constituent
governments should be exercised sparingly and within Assembly of India. It served as the foundation for the final
the constitutional limits. Preamble.

4. Malak Singh Vs. State of Punjab: This case involves Statement 2 is correct- The Preamble is non-justiciable,
issues related to the transfer of land and the validity meaning that it cannot be enforced in a court of law. It is
of certain state laws. not legally binding and does not grant enforceable rights
or remedies.
65. Solution: (c) Statement 3 is incorrect- The Preamble can be amended,
Exp) Option c is the correct answer. but it requires a special majority in Parliament as prescribed
Government of India Act, 1935 led to the separation of under Article 368 of the Constitution.
Burma from India, as it divided the previous legislation Statement 4 is correct- The Preamble cannot override the
into two separate Acts: The Government of India Act, specific provisions such as Article 13, Article 14, Article 16,
1935 and the Government of Burma Act, 1935. The Act Article 32, Article 368, etc of the Constitution. If there is a
recognized Burma as a separate colony, with its own conflict between the Preamble and a specific provision, the
governor, legislature, and administration. The separation latter takes precedence.
of Burma from India was a result of various factors, such
69. Solution: (c)
as the growing resentment of the Burmese population
towards Indian immigration and domination, the desire of Exp) Option c is the correct answer.
the British and other foreign interests to exploit Burma’s Government of India Act, 1935 introduced the system of
resources independently, and the recognition of the cultural dyarchy at the central level, which meant that some subjects
and historical differences between Burma and India. such as defence, foreign affairs, and ecclesiastical affairs were
reserved for the Governor-General and his executive council,
66. Solution: (d)
while other subjects such as finance, commerce, health, and
Exp) Option d is the correct answer. education were transferred to the Federal Legislature and the
Mahatma Gandhi wrote in an article published in Harijan on Council of Ministers. This was the first time that dyarchy

PYQ Workbook 28
INDIAN POLITY AND GOVERNANCE

was applied at the central level in British India. However,


• It provided for the establishment of a public service
this provision of the Act did not come into operation at
commission. Hence, a Central Public Service
all. The act also abolished dyarchy in the provinces and Commission was set up in 1926 for recruiting civil
introduced ‘provincial autonomy’ in its place servants
Important Tips
71. Solution: (b)
Features of Government of India Act, 1935:
Exp) Option b is the correct answer.
• It provided for the establishment of an All-India
Federation consisting of provinces and princely states The President’s power to promulgate ordinances was
as units. inspired from section 42 of the Government of India Act,
• It introduced bicameralism in six out of eleven 1935. The Act gave the President the authority to promulgate
provinces. Thus, the legislatures of Bengal, Bombay, ordinances during the recess of Parliament, if he was satisfied
Madras, Bihar, Assam and the United Provinces were that immediate action was necessary.
made bicameral.
72. Solution: (c)
• It further extended the principle of communal
representation by providing separate electorates for Exp) Option c is the correct answer.
depressed classes (scheduled castes), women and The demand for Constituent Assembly was finally accepted
labour (workers). in principle by the British Government in what is known
• It extended franchise. About 10 per cent of the total as the ‘August Offer’ of 1940. In 1942, Sir Stafford Cripps, a
population got the voting right. member of the cabinet, came to India with a draft proposal
• It provided for the establishment of a Reserve Bank of of the British Government on the framing of an independent
India to control the currency and credit of the country. Constitution to be adopted after the World War II.
Cripps Mission proposed that after the end of the war, a
70. Solution: (c)
constituent assembly would be convened to frame a new
Exp) Option c is the correct answer. constitution. Members of this assembly would be partly
The Government of India Act of 1919 was thus enacted, elected by the provincial assemblies through proportional
which came into force in 1921. This Act is also known as representation and partly nominated by the princes.
Montagu-Chelmsford Reforms (Montagu was the Secretary
73. Solution: (d)
of State for India and Lord Chelmsford was the Viceroy of
India). Exp) Option d is the correct answer.
The act introduced, for the first time, bicameralism and Sardar Patel was the Chairman of the Provincial
direct elections in the country. Thus, the Indian Legislative Constitution Committee of the Constituent Assembly, which
Council was replaced by a bicameral legislature consisting was set up to discuss and provide for a model Provincial
of an Upper House (Council of State) and a Lower House Constitution, which would determine the system and form
(Legislative Assembly). The majority of members of both the of government at the provincial level. The Committee held
Houses were chosen by direct election. several meetings in 1947 and discussed various issues such
as the role and functions of the Governor, the composition
Important Tips
of the provincial legislatures, and the division of powers
Features of Government of India Act of 1919: between the Centre and the provinces.
• It relaxed the central control over the provinces by
demarcating and separating the central and provincial Important Tips
subjects. • Dr. B. R. Ambedkar was the Chairman of the Drafting
• It further divided the provincial subjects into two Committee, which was responsible for preparing a
parts—transferred and reserved. draft of the new Constitution of India.
• It required that the three of the six members of • Pt. Jawaharlal Nehru was the Chairman of both the
the Viceroy’s executive Council (other than the Union Powers Committee and the Union Constitution
commander-in-chief ) were to be Indian. Committee, which were responsible for determining
• It extended the principle of communal representation the main principles of the Union Constitution and its
by providing separate electorates for Sikhs, Indian structure and form.
Christians, Anglo-Indians and Europeans. • Dr. Rajendra Prasad was not the Chairman of any
• It created a new office of the High Commissioner for major committee, but he was the President of the
India in London and transferred to him some of the Constituent Assembly and also chaired two minor
functions hitherto performed by the Secretary of State committees: the Rules of Procedure Committee and
for India. the Steering Committee.

29 PYQ Workbook
INDIAN POLITY AND GOVERNANCE

74. Solution: (a) B. State List- (1) Police and Public Order fall under the
Exp) Option a is the correct answer State List as the state governments have the authority to
legislate on the given subject.
The Constitution of India is the lengthiest written
constitution in the world. Its provisions were borrowed from C. Concurrent List- (4) Population Control and Family
the Government of India Act 1935 and the Constitutions Planning falls under the concurrent list as both the
of the US, Ireland, Britain, Canada, Australia, Germany, Central government and the state government have the
the USSR, France, South Africa, Japan, and other countries authority to legislate on the given subject.
The federal nature of the Indian constitution was adopted D. Residuary Subject- (2) Space Research falls under
from the Constitution of Canada. the residuary subject. It includes subjects that are not
specifically mentioned in any of the lists and are within
75. Solution: (b)
the domain of the central government.
Exp) Option b is the correct answer.
The Preamble to the Constitution of India states that India 79. Solution: (b)
is a secular state. The Preamble to the Constitution of India Exp) Option b is the correct answer.
begins with the words “WE, THE PEOPLE OF INDIA, having • Establishment of Board of Control: This was done by
solemnly resolved to constitute India into a SOVEREIGN, the Pitt’s India Act, 1784, which was the attempt by the
SOCIALIST, SECULAR, DEMOCRATIC REPUBLIC”. British Parliament to regulate the affairs of the East India
The word “secular” means that the state does not have an Company in India. The act created a Board of Control
official religion and does not favor any particular religion. consisting of six members appointed by the Crown, who
had the power to supervise and direct all civil, military,
Important Tips
and revenue matters in India.
Indian secularism is based on the principle of Sarva
• Establishment of Supreme Court: This was also done by
Dharma Sambhava, which means “equal respect for
the Regulating Act of 1773, which established a Supreme
all religions.” This principle is reflected in the Indian
Court at Calcutta, consisting of one chief justice and
Constitution, which guarantees the freedom of religion
three other judges appointed by the Crown. The court
to all citizens. Western secularism, on the other hand, is had jurisdiction over all British subjects in Bengal, Bihar,
based on the principle of separation of church and state. and Orissa, and could hear appeals from the provincial
This principle means that the state should not be involved courts.
in religious matters, and that religion should not be used
• Permission to English Missionaries to work in India:
to justify state policies.
The Charter Act, 1813 allowed English missionaries to
76. Solution: (d) propagate Christianity in India, subject to the approval
Exp) Option d is the correct answer. of the local governments.

The term ‘Secular’ in India means that the State does not • Appointment of Law Member in Governor-General
have an official or designated religion. It implies that the Council: The Charter Act, 1833 added a fourth member
government treats all religions equally and does not favor or to the Governor-General’s Council, who was a law
promote any particular religion. This principle of secularism member appointed by the Crown. The first law member
allows individuals to practice their own religion freely and was Thomas Babington Macaulay.
grants religious freedom to all citizens. Important Tips
77. Solution: (c) Features of Charter Act, 1813:
Exp) Option c is the correct answer. • The Act declared that the British territories in India
The Preamble to the Constitution of India sets out the were held by the East India Company “in trust for His
ideals and objectives of the Constitution. These ideals Majesty, His heirs and successors”.
and objectives are further elaborated in the Chapter on • The act empowered the local governments to impose
Directive Principles of State Policy, Fundamental Rights, taxes on the persons subject to the jurisdiction of the
and Fundamental Duties. The Directive Principles of State Supreme Court.
Policy are a set of guidelines that the state should follow
in order to achieve the goals of the Constitution. The • Financial provision was made to encourage a revival
Fundamental Rights are a set of rights that are guaranteed in Indian literature and for the promotion of science.
to all citizens of India. The Fundamental Duties are a set of
80. Solution: (b)
responsibilities that all citizens of India have to the state
and to each other. Exp) Option b is the correct answer.
Indian Council Act, 1892 increased the functions of
78. Solution: (a)
legislative councils and gave them the power of discussing
Exp) Option a is the correct answer. the budget and addressing questions to the executive. It
A. Union List- (3) Census falls under the Union List as the increased the number of additional (non-official) members
central government has the authority to legislate on the in the Central and provincial legislative councils, but
given subject. maintained the official majority in them.

PYQ Workbook 30
INDIAN POLITY AND GOVERNANCE

Rajendra Prasad. The other members were Abul Kalam


Important Tips
Azad, K.M. Panikkar, Sarojini Naidu, C. Rajagopalachari,
• For the first time, Indian Council Act, 1892 allowed K.M. Munshi and B.R. Ambedkar.
the council to discuss the budget and address questions
to the executive. However, these powers were very 84. Solution: (c)
limited and restricted. Exp) Option c is the correct answer.
• The council could not vote on the budget or move Prevention of cruelty to animals is listed in the Concurrent
any resolution on it. It could only ask questions for List of the Indian Constitution.
information and not for criticism.
85. Solution: (a)
• The questions had to be submitted six days in advance
and could not be asked orally or supplemented by Exp) Option a is the correct answer.
further questions. January 26 was selected as the date for the inauguration of
• The executive had the right to refuse to answer any the Constitution, because it was on that day in 1930 when
question if it deemed it inexpedient or injurious to the Indian National Congress proclaimed the Declaration
public interest. of Indian Independence or Purna Swaraj at its Lahore
session. This declaration rejected the Dominion status
81. Solution: (b) offered by the British and demanded complete independence
from British rule. The date was chosen to honour this historic
Exp) Option b is the correct answer.
event and to mark India’s transition from a dominion to a
The Ninth Schedule currently contains 284 laws , including republic.
land reforms laws, and laws relating to the Scheduled
Castes and Scheduled Tribes. The Ninth Schedule was 86. Solution: (c)
added to the Constitution in 1951, in order to protect land Exp) Option c is the correct answer.
reforms laws from being struck down by the Supreme Court. Land reform comes under the subjects of the State List in
Important Tips the Indian Constitution. The State List contains subjects
over which the state governments have the authority to
• The Ninth Schedule contains a list of central and state legislate and implement policies. Land reform involves
laws that are immune to court challenges. measures aimed at redistributing land ownership, promoting
• It was introduced by the Constitution (First agricultural productivity, and improving the socio-economic
Amendment) Act, 1951. conditions of farmers.
• Article 31B of the Indian Constitution established Important Tips
the Ninth Schedule, which shields specific laws
Some of the important subjects under the State List include:
or enactments (unlike Article 31A, which extends
protection to ‘classes’ of laws). 1. Police and public order.

• Article 31B operates retrospectively, meaning that 2. Public health and sanitation
if laws are added to the Ninth Schedule after being 3. Agriculture and irrigation
declared unconstitutional, they are considered valid 4. Local government
from their commencement. 5. Public works
• Although Article 31B excludes judicial review, the 6. Education
Supreme Court has asserted that laws under the Ninth
7. Land reforms
Schedule can still be subject to scrutiny if they violate
Fundamental Rights or the basic structure of the 8. Water supply and sanitation
Constitution. 9. State taxation

82. Solution: (a) 87. Solution: (d)


Exp) Option a is the correct answer Exp) Option d is the correct answer
The makers of the Indian Constitution were influenced by the Article 352 of the Constitution provides for the provision
Irish Republic in representing talent, experience, and service of National Emergency which can be applied if any
in the formation of the Rajya Sabha or Council of state. extraordinary situation arises that may threaten the security,
peace, stability and governance of the country.
83. Solution: (b)
Important Tips
Exp) Option b is the correct answer.
1. Article 17 of the Indian Constitution states that
Dr. Rajendra Prasad was the chairman of the National Flag
“Untouchability” is abolished and its practice in any
Committee, which was an ad hoc committee constituted by form is forbidden.
the Constituent Assembly to finalize the design of the Indian
2. Article 78 lays down a few duties and responsibilities
National Flag. The National Flag Committee was appointed
for the Prime Minister as stated:
on 23 June 1947 and had seven members, including Dr.

31 PYQ Workbook
INDIAN POLITY AND GOVERNANCE

of plans for the economic development of the country,


• To inform the President of all significant decisions made
which requires coordination between the central and state
by the Council of Ministers regarding the management of
Union affairs and other legislative proposals. governments.

• To share information regarding the Union’s Important Tips


management of its affairs and legislative proposals as Union List- It has 98 subjects like defence, banking,
the President may require. foreign affairs, currency, atomic energy, insurance,
• Any matter on which a Minister has made a decision communication, inter-state trade and commerce, census,
but the Council has not yet considered it must be audit and so on.
submitted, if the President so requests, for the Council State List- It has 59 subjects like public order, police, public
of Minister’s consideration. health and sanitation, agriculture, prisons, local government,
3. Article 192 deals with the decision on questions as to fisheries, markets, theaters, gambling and so on.
disqualifications of members Concurrent List- It has 52 subjects like criminal law
and procedure, civil procedure, marriage and divorce,
88. Solution: (d)
population control and family planning, electricity, labour
Exp) Option d is the correct answer. welfare, economic and social planning, drugs, newspapers,
Statement 1 is correct- The Constitution promises justice - books and printing press, and others.
social, economic, and political - to all. This means that all
people should have equal opportunities in society, regardless 91. Solution: (c)
of their social status or economic background. Exp) Option c is the correct answer.
Statement 2 is correct- The Constitution of India guarantees Government of India Act of 1935 abolished dyarchy in
Liberty of thought, expression, belief, faith and worship. the provinces and introduced ‘provincial autonomy’ in
This means that people should be free to think and express its place. The provinces were allowed to act as autonomous
their thoughts without fear of censorship or reprisal. units of administration in their defined spheres.
Statement 3 is correct- The Constitution of India strive to
92. Solution: (c)
provide Equality of status and of opportunity. This means
that all people should be treated equally under the law, Exp) Option c is the correct answer.
regardless of their social status or economic background. The 42nd Constitutional Amendment Act, 1976 inserted
Statement 4 is correct- The Constitution of India aims to the words “socialist” and “secular” in the Preamble to the
establish Fraternity assuring the dignity of the individual Constitution of India. The word “socialist” was inserted
and the unity and integrity of the nation. This means that to emphasize the commitment of the Indian state to the
all people should feel a sense of brotherhood or sisterhood, principles of social justice and equality. The word “secular”
and that the nation should be united and integrated. was inserted to emphasize the commitment of the Indian
state to the principle of religious neutrality.
89. Solution: (c)
93. Solution: (a)
Exp) Option c is the correct answer.
Exp) Option a is the correct answer
The Indian Constituent Assembly was established under
the Cabinet Mission Plan of 1946, which was a constitutional Under Article 117 of the Indian Constitution, no bill
scheme proposed by the British Cabinet Delegation that imposing tax can be introduced in the legislature except on
visited India in 1946 to find a solution to the constitutional the recommendation of the President.
deadlock and the demand for independence. The Cabinet
Important Tips
Mission Plan provided for the election of a Constituent
Assembly by indirect vote through provincial assemblies. • Article 266 is related with Consolidated Funds and
The Constituent Assembly was to frame a constitution for public accounts of India and of the States.
independent India, with provisions for a federal structure • Article 306 is related to the Power of certain States
and safeguards for minorities. The Cabinet Mission Plan in Part B of the First Schedule to impose restrictions
was accepted by both the Congress and the Muslim League, on trade and commerce. It was Repealed by the
although with some reservations Constitution (Seventh Amendment) Act, 1956.

90. Solution: (c) 94. Solution: (b)


Exp) Option c is the correct answer. Exp) Option b is the correct answer
Under the Constitution of India, Economic Planning is a Concurrent List in the Indian Constitution has been
subject included in the Concurrent List. The Concurrent adopted from the Constitution of Australia. Central
List comprises of 66 subjects on which both the central and Government and State Government both can make laws on
state governments have the authority to legislate. Economic the subjects under this list. In case of any conflict, the law
planning involves the formulation and implementation made by the Central Government will prevail.

PYQ Workbook 32
INDIAN POLITY AND GOVERNANCE

95. Solution: (b) 99. Solution: (a)


Exp) Option b is the correct answer. Exp) Option a is the correct answer.
Minto-Morley Reforms, 1909 introduced the system Dr. Rajendra Prasad said “A Constitution, like a machine,
of separate electorates for Muslims, which meant that is a lifeless thing. It acquires life because of the men who
Muslims could vote only for Muslim candidates in certain control it, and India needs today nothing more than a set of
constituencies. This was the first time that communal honest men who will have the interest of the country before
representation was provided for in British India. them” in his speech on 26th November 1949, when the
It also provided for the separate representation of presidency Constituent Assembly adopted the Constitution of India. He
corporations, chambers of commerce, universities and was also the first President of India and played an important
zamindars. role in drafting the Constitution
96. Solution: (c) 100. Solution: (b)
Exp) Option c is the correct answer. Exp) Option b is the correct answer.
The Government of India Act of 1935 introduced the Residuary subjects were allocated to provincial Legislatures
principle of constitutional autocracy, which meant that the was not a salient feature of the Government of India Act,
executive authority of the provinces and the centre was vested 1935. On the contrary, the Act vested the residuary powers
in the Governors and the Governor-General respectively, on in the Governor-General, who could exercise them on behalf
behalf of the British Crown. The Act gave them absolute of the British Crown. The Act did not give any autonomy to
power over all matters of administration and legislation, and the provinces in matters of residuary subjects.
they were not accountable to any elected body or the people
101. Solution: (b)
97. Solution: (b)
Exp) Option b is the correct answer.
Exp) Option b is the correct answer.
• H. C. Mukherjee was a Christian leader and a member
Sardar Patel was the Chairman of the Advisory Committee
of the Bengal Legislative Assembly who was elected as
on Fundamental Rights and Minorities constituted by
the first Vice-President of the Constituent Assembly on
the Constituent Assembly, which was set up to deal with
11 December 1946. He later resigned from this post and
the issues of fundamental rights, minorities and tribal
and excluded areas in the new Constitution of India. The became the Governor of West Bengal in 1952.
Advisory Committee was appointed on 24 January 1947 and • K.M. Munshi was a lawyer, writer and politician who
had 64 members, including Sardar Patel. was elected to the Constituent Assembly from Bombay
B. N. Rao was not the Chairman of any committee, but he was Presidency as a Congress member. He was originally
the Constitutional Advisor to the Constituent Assembly, the only Congress member in the Drafting Committee,
who prepared a draft of the text of the Constitution based on which was chaired by B. R. Ambedkar. He also served as
various sources and suggestions a member of several other committees such as the Union
Powers Committee, the Union Constitution Committee,
98. Solution: (b) and the Advisory Committee on Fundamental Rights.
Exp) Option b is the correct answer. • V. T. Krishnamachari was a businessman and politician
The Constitution of India was adopted by the Constituent who was nominated to the Constituent Assembly by
Assembly of India on 26 November, 1949, after almost the Maharaja of Bikaner, one of the princely states of
three years of deliberations and drafting. The Constituent Rajasthan. He later became a member of the Provisional
Assembly was a body of elected representatives from the Parliament and served as a minister in various portfolios
provinces and princely states of India, which was formed such as finance, commerce and industry, and defence
in December 1946 to frame a constitution for independent production.
India. The Constitution of India came into effect on 26 • Jawaharlal Nehru was the first Prime Minister of India
January 1950, making India a sovereign, democratic republic. and a prominent leader of the Indian National Congress.
Important Tips He was elected to the Constituent Assembly from
the United Provinces and served as the Chairman of
• The resolution for giving a Constitution to India was
several important committees such as the Union Powers
adopted by the Constituent Assembly on January 22,
Committee, the Union Constitution Committee, and
1947.
the States Committee. He also moved the Objectives
• This resolution, also known as the Objective Resolution, Resolution in the Assembly, which laid down the guiding
was moved by Jawaharlal Nehru on December 13, principles of the Constitution.
1946, and it laid down the basic principles and values
of the Constitution of India. 102. Solution: (b)
• 24 January 1950: Last meeting of Constituent Exp) Option b is the correct answer.
Assembly. The Constitution was signed and accepted Statement 1 is correct- Each Autonomous District Council
by 284 members of the assembly has 30 members. An Autonomous District Council is a local

33 PYQ Workbook
INDIAN POLITY AND GOVERNANCE

self-government body established in tribal areas of India to Option C matches with Option 4: 86th Constitutional
provide autonomy and govern the affairs of the region. Amendment Act amended article 51A and it inserted a
Statement 2 is incorrect- Out of the 30 members in the fundamental duty that states “ parent or guardian to provide
Autonomous District Council, 26 are elected on the basis of opportunities for education to his child or, as the case may
adult franchise and rest 4 are nominated by the Governor. be, ward between the age of six and fourteen years.”

Statement 3 is correct- In the case of Autonomous Districts in Option D matches with Option 2: Article 1 provides for
Assam, the Governor has the power to direct the Acts passed name and Territory of the Union. It talks about India, that is
Bharat, is the Union of States It comes under Part 1 which is
by the Parliament of India specifically for these districts.
related with Union and its Territories.
The Governor has the authority to reorganize and arrange
the Autonomous Districts, which includes altering their 105. Solution: (b)
boundaries, changing their names, or adjusting their areas.
Exp) Option b is the correct answer.
Important Tips The subject of “Panchayati Raj” is included in the State
As per Article 244, the Autonomous District Councils are List of the Indian Constitution. The State List contains
established for Sixth Schedule States (Assam, Tripura, subjects over which only the state governments have the
Meghalaya, Mizoram) of the Indian Constitution. The authority to legislate. Panchayati Raj refers to the system of
Governor is empowered to increase, decrease the area, or local self-government at the village or rural level, and its
change the name of Autonomous Districts. establishment, organization, and functioning are primarily
There are a total of 10 ADCs in India, located in the governed by laws made by the state governments.
following areas: 106. Solution: (b)
• Assam: Karbi Anglong Autonomous Council, Dima Exp) Option b is the correct answer
Hasao Autonomous District Council, Bodoland
Article 324 is related to superintendence, direction,
Territorial Council.
and control of elections to be vested in an Election
• Meghalaya: Khasi Hills Autonomous District Council, Commission. Article 351 of the Constitution provides that
Jaintia Hills Autonomous District Council, Garo Hills it shall be the duty of the Union to promote the spread of
Autonomous District Council. the Hindi language. Article 332 talks about the Reservation
• Mizoram: Chakma District Council, Lai District of seats for Scheduled Castes and Scheduled Tribes in the
Council, Mara District Council, Pawi District Council, Legislative Assemblies of the States. Article 318 is related to
Riang District Council. the power to make regulations as to conditions of service
• Tripura: Tripura Tribal Areas Autonomous District of members and staff of the Commission.
Council 107. Solution: (a)
103. Solution: (b) Exp) Option a is the correct answer
Exp) Option b is the correct answer. The Citizenship Act, 1955 deals with the determination of
citizenship on or after 26th January, 1950, which marks
Forests were transferred from the State list to the
the commencement of the Act and the establishment of the
Concurrent List of the Constitution of India through the
Republic of India. Prior to the Citizenship Act, 1955, the
42nd Amendment of 1976. The Act made changes to the
legal provisions regarding citizenship in India were governed
distribution of powers between the central government
by the Indian Independence Act, 1947, and the Constitution
and the state governments. It shifted the subject of forests
of India, which came into effect on 26th November, 1949.
from the State list to the Concurrent list, allowing both
However, the Citizenship Act, 1955 consolidated and codified
the central and state governments to legislate and exercise the various laws and provisions related to citizenship into a
control over forest-related matters comprehensive legislation.
104. Solution: (b) 108. Solution: (a)
Exp) Option b is the correct answer Exp) Option a is the correct answer
Option A matches with Option 3: Article 18 of the Indian 1 st pair is incorrectly matched: The First Schedule of the
Constitution, titled “Abolition of Titles,” forbids the State Constitution of India contains details about the states and
from bestowing titles on anyone, whether a citizen or not. It union territories of India, along with their territories and
comes under Fundamental Rights. languages.
Option B matches with Option 1: Article 40 deals with 2 nd pair is incorrectly matched: The Third Schedule of
Organization of village panchayats.It requires the State to the Constitution of India y contains the forms of oaths or
take steps to organise village panchayats and endow them affirmations for elected officials such as the President, the
with such powers and authority as may be necessary to enable Vice-President, governors, judges, and other authorities.
them to function as units of self-government. It comes under 3rd pair is correctly matched: The Fifth Schedule of the
Directive Principles of State Policies. Constitution of India indeed contains provisions related to the

PYQ Workbook 34
INDIAN POLITY AND GOVERNANCE

administration of Scheduled Areas and Scheduled Tribes. It Important Tip


outlines the powers and functions of the tribal advisory councils,
• The Citizenship Act, 1955 was amended 6 times in
safeguards for the welfare and advancement of Scheduled Tribes,
1986, 1992, 2003, 2005, 2015 and 2019.
and other related provisions.
• The CAA 2019 provides citizenship on the basis
4 th pair is incorrectly matched: The Ninth Schedule of the of religion to six undocumented non-Muslim
Constitution of India a special provision that was added to communities (Hindus, Sikhs, Buddhists, Jains,
protect certain laws from judicial scrutiny on the grounds of Parsis and Christians) from Pakistan, Afghanistan
violating fundamental rights. It includes laws related to land and Bangladesh who entered India on or before 31st
reforms, the abolition of the zamindari system, and other December, 2014.
social and economic reforms. • It exempts the members of the six communities from
any criminal case under the Foreigners Act, 1946 and
109. Solution: (c)
the Passport Act, 1920.
Exp) Option c is the correct answer.
Statement 1 is correct- Equality entails the absence of 112. Solution: (a)
privileges or discrimination within society, and the Exp) Option a is the correct answer.
Preamble emphasizes equality of status and opportunity for • The doctrine of independence is not to be raised to the
all citizens. level of a dogma so as to enable the judiciary to function
Statement 2 is correct-Through the 42nd Amendment Act as a kind of super-legislature or super-executive.
of 1976, the phrase was added to the Preamble to emphasize • The judiciary is there to interpret the Constitution or
the importance of maintaining the unity and integrity of adjudicate upon the rights. Option 1 is correct.
India as a sovereign, socialist, secular, and democratic • It is the great tribunal which has to draw the line between
republic. liberty and social control. Option 2 is correct
Important Tips 113. Solution: (d)
The Indian Constitution’s Preamble is derived from the Exp) Option d is the correct answer.
US Constitution and is often referred to as the “identity 1. Sixth schedule to the constitution of India provides for
card” of the Constitution.
setting up of autonomous district council (option d is
It is based on the Objective Resolution proposed by correct)
Pandit Nehru and was amended once through the 42nd
2. The Constitution of India makes special provisions for the
Constitutional Amendment Act, 1972, to include the
administration of the tribal dominated areas in four states
words “Socialist, Secular and Integrity.”
viz Assam, Meghalaya Tripura and Mizoram.
The Preamble is non-justiciable and does not grant
3. As per Article 244 and sixth Schedule these areas are called
legislative power or restrict it. It is an integral part of the
“Tribal Areas
Constitution and can be amended by a simple majority
under Article 368. It was added by the Constituent 4. The Sixth Schedule envisages establishment of Autonomous
Assembly after the enactment of the rest of the District Councils (ADC)
Constitution.
Tribal areas in the four states
110. Solution: (b) Assam North Cachar Hills District
Exp) Option b is the correct answer The Karbi Anglong District
The Preamble of the Constitution of India was amended in 1976 The Bodoland Territorial Areas District
by the 42nd Constitutional Amendment Act. This amendment Meghalaya Khasi Hills District
introduced three terms to the Preamble: Socialist, Secular, Jaintia Hills District
and Integrity. The term “Fraternity” was already a part of the
The Garo Hills District
original Preamble and was not introduced or amended in
Tripura Tripura Tribal Areas District
1976.
Mizoram The Chakma District
111. Solution: (b)
The Mara District
Exp) Option b is the correct answer
114. Solution: (d)
Article 11 of the Indian Constitution grants the Parliament
the authority to regulate citizenship rights, and it is under this Exp) Option d is the correct answer.
provision that the Citizenship (Amendment) Act (CAA) was • The government of India act 1858 is also known as the
enacted. As Article 11 is part of Part II of the Constitution of “act for better government of India”.
India, the CAA can be classified within the framework of Part II • It changed the designation of governor general of India
of the constitution. to that of viceroy of India.

35 PYQ Workbook
INDIAN POLITY AND GOVERNANCE

• The administration of India was taken from the East 116. Solution: (d)
India Company and placed directly under the British Exp) Option d is the correct answer
Crown. (option 1 is correct) • The Indian Constitution is unique in its contents and
• Thus the country was kept under the direct rule of the spirit.
British and the Cabinet. • Though borrowed from almost every Constitution of the
• A new post called Secretary of State was added who world, the Constitution of India has several features that
would be directly responsible to British cabinet and also distinguish it from the Constitutions of other countries.
a part of it. • Independence of Judiciary has been taken from the
Constitution of USA.
• The act also ended the double government which was
started by the Pitts India act 1784. • Following features have been borrowed by the British
Constitution:
• A 15 member advisory council to the secretary of state
was formed. Britain 1. Parliamentary System of government
• This act was dominated by the principle of absolute 2. Rule of Law
imperial control without any popular participation in 3. Legislative procedure
administration of the country. (Option 3 is correct) 4. Single citizenship
• Under this act, all the powers of Crown were exercised by 5. Cabinet system
Secretary of State of India and was assisted by Council of
6. Prerogative writs
India (Council of fifteen members).
7. Parliamentary privileges
• Under the Pitts India Act of 1784 for the first time,
the British Government was given supreme control 8. Bicameralism
over Company’s affairs and its administration in India. United 1. Impeachment of the president
States of 2. Functions of president and vice-president
(Option 2 is incorrect)
America
3. Removal of Supreme Court and High court
115. Solution: (d)
judges
Exp) Option d is the correct answer.
4. Fundamental Rights
• Indian Councils Act of 1861 made a beginning of
5. Judicial review
representative institutions by associating Indians with
the law-making process. 6. Independence of judiciary
7. The preamble of the constitution
• It thus provided that the viceroy should nominate some
Indians as non-official members of his expanded council. 117. Solution: (b)
(Statement 1 is correct) Exp) Option b is the correct answer.
• In 1862, Lord canning the then viceroy nominated three • Ninth Schedule says that the article mentioned in this are
Indians to his Legislative council immune from judicial review it is for the larger good of
• It empowered the viceroy to make rule and orders for the the society, the government cannot be challenged in the
more convenient transaction of business in the council. court for his actions.
• Ninth Schedule was added to the Constitution by First
• It also gave recognition to the Portfolio system introduced
Constitutional Amendment, 1951.
by Lord canning in 1859
• In a landmark ruling on 11th January 2007, the
• Under this a member of the viceroy’s council was made
Supreme Court ruled that all laws (including those in
in charge of one or more departments of the government the Ninth Schedule) would be open to judicial review,
and was authorized to issue final orders on behalf of the if they violated the basic structure of the Constitution.
council on matters of his department(s). (Statement b is incorrect)
• The Governor-General also had the power to promulgate
118. Solution: (b)
ordinances without the council’s concurrence during
emergencies (Statement 3 is incorrect). Exp) Option b is the correct answer.
• On 26th November 1949, 3 provisions i.e.., Elections,
• The non official members of the Viceroys Executive
citizenship and provisional Parliament came into force
council did not take part in the discussions and their
and rest of the provisions only on 26th January 1950.
role was only advisory.
• Fundamental Rights of the citizen came into force on
The non-official members of the Executive Council were 26th january 1950.
not interested in attending the meetings of the Council,
moreover, under this Act they were not bound to attend 119. Solution: (a)
them either.The Indian members were not eligible to Exp) Option a is the correct answer.
oppose any bill and most often the bills were passed in one • As per Article-344(1) and 351 of Indian Constitution,
sitting without discussion. (Statement 2 is incorrect) the 8th Schedule includes 22 language.

PYQ Workbook 36
INDIAN POLITY AND GOVERNANCE

• The Eighth Schedule to the Constitution consists of the by virtue of article 6 or article 8, if he has voluntarily
following 22 languages: acquired the citizenship of any foreign State.
• Of these languages, 14 were initially included in the • As per article 11 The Parliament has the right to
Constitution. make any provision concerning the acquisition and
• Sindhi language was added by the 21 st Amendment Act termination of citizenship and any other matter relating
of 1967. to citizenship.
• Konkani, Manipuri, and Nepali were included by the 123. Solution: (c)
71 st Amendment Act of 1992.
Exp) Option c is the correct answer.
• Bodo, Dogri, Maithili, and Santhali were added by the
• The 9th schedule was added to the constitution by the
92 nd Amendment Act of 2003 which came into force in
1st amendt. to the constitution in 1951 (Statement 3 is
2004.
incorrect)
120. Solution: (d) • The Ninth Schedule of the Constitution contains a list of
Exp) Option d is the correct answer. central and state laws which cannot be challenged in
courts. (Statement 4 is incorrect)
• Article 25 says that all persons are equally entitled to
freedom of Conscience and the right to freely profess, 124. Solution: (b)
practice and propagate religion.
Exp) Option b is the correct answer.
• The Indian Constitution embodies the positive concept
• Democracy is a form of government in which the rulers
of secularism ie, all religions in our country (irrespective
are elected by the people.
of their strength) and no official status was given to
certain religions. • It is based on a free and fair election where these
currently in a power having a free chance of loosing.
• 42nd amendment act of 1976, the Preamble to the
constitution of India asserted that India is a secular • In a democracy, each adult citizen have one vote and
nation. each vote have one value and the decision making
power rests with those elected by the people.
• Secularism is a belief system which rejects religion, or
the belief that religion should not be part of the affairs of • Democracy also enhances the dignity of citizens.
the state. (Statement 2 is incorrect) • Abraham Lincoln had said “Democracy is the
government of the people, by the people and for the
121. Solution: (d) people”.
Exp) Option d is the correct answer.
125. Solution: (b)
The members of constituent assembly were chosen indirectly
by members of provincial assemblies, who in turn were Exp) Option b is the correct answer.
elected through Provincial Election of 1946. (Statement 1 • BR Ambedkar has emerged as the epitome personality of
is correct) modern India.
The constituent assembly had representative of princely • He was appointed as the Chairman of the constitution
states , who were nominated by the princes. (Statement 2 is drafting committee in 1947.
incorrect) • He is recognised as the “Father of the Constitution of
The constituent assembly had representation from all parts India”.
of India , every community, every section of the society was • Major contributions of Ambedkar to modern India :
represented in the assembly and hence the discussions in
a) Education
the constituent assembly was influenced by public opinion
over an issue, although rational decision was taken after fair b) Changing hierarchical Structures of Indian society
discussions. (Statement 3 is incorrect) c) Bringing untouchables to the mainstream of Indian
To create a sense of collective participation the public was society.
asked to send in their views . Many of the linguistic minorities d) Framing an inclusive and secular Constitution.
wanted the protection of their mother tongue, religious e) Upliftment of marginalised
minorities asked for special safeguards, while dalits demanded
f ) Fundamental Rights
an end to all caste oppression and reservation of seats in
government bodies. Important issues of cultural rights and g) Economic Contributions
social justice raised in these public discussions were debated 126. Solution: (d)
on the floor of the Assembly. (Statement 4 is correct)
Exp) Option d is the correct answer.
122. Solution: (c) • In Part II of Constitution under Article 5-11, citizenship
Exp: Option c is the correct answer. clause has been explained.
• As per article 9 No person shall be a citizen of India by • Full civil and political rights are entitled to the citizen
virtue of article 5, or be deemed to be a citizen of India of India.

37 PYQ Workbook
INDIAN POLITY AND GOVERNANCE

• Also, they are entitled to be a Member of Parliament or


1. The Preamble is based on ‘Objective Resolution’ by
State Legislature but one has to also satisfy the age limit
Nehru and was adopted and enacted by the Constituent
criterion in order to be an MP or MLA.
Assembly of India on 26th November 1949.
• Citizens of India has right of suffrage for election to the 2. It has been amended only once through the 42nd
House of People (of the Union) and legislative assembly Amendment Act of 1976, which added the words
of every state. “Secular” and “Socialist” to the Preamble.
Important Tips 3. The Preamble serves as a guiding light for the
1. Citizenship is listed in the Union List. government, lawmakers, and citizens in shaping the
nation’s policies and actions.
2. The term ‘Citizen’ is not defined in the constitution.
3. Citizenship of India can be acquired in the following 129. Solution: (b)
ways: Exp) Option b is the correct answer.
• Citizenship at the commencement of the Constitution The Constitution of India provides for the division of
• Citizenship by birth powers between central and state government. The seventh
• Citizenship by descent schedule provides for three list- State List, Central List, and
Concurrent List. Out of the three list, the Concurrent list
• Citizenship by registration
includes subjects on which both the Parliament (Central
• Citizenship of naturalization government) and State Legislature (state governments)
• By incorporation of territory (by the Government of can legislate independently and concurrently.
India)
Important Tips
4. Termination of the Citizenship is possible in three
Few of the subjects under the Central List:
ways:
1. Defense and armed forces
Renunciation, Termination, Deprivation
2. Foreign affairs and international relations
127. Solution: (a) 3. Atomic energy
Exp) Option a is the correct answer. 4. Currency, coinage, and legal tender
Somnath Lahiri was a member of the Constituent Assembly 5. Railways
of India and a prominent leader of the Communist Party 6. Airports and airways
of India. He was a vocal critic of the British rule in India 7. Interstate trade and commerce
and he believed that the Constituent Assembly was not
8. Banking and financial institutions
truly representative of the Indian people. In a speech in
the Constituent Assembly on 19 December 1946, Lahiri said 9. Taxes on income other than agricultural income
that the Constituent Assembly was “British made” and that it Few of the subjects under the State List:
was “working the British plan as the British should like it to 1. Police and public order
be worked out”. He argued that the Assembly was dominated 2. Public health and sanitation
by the Indian National Congress, which was a party that had 3. Agriculture and irrigation
close ties to the British government
4. Fisheries
128. Solution: (d) 5. Local government
Exp) Option d is the correct answer. 6. Public works
The preamble to the Constitution of India is a short 7. Public entertainment
introductory statement that sets out the basic principles 8. Land and land revenue
and objectives of the Constitution. It is not enforceable in 9. Education at the state level
a court of law, but it is an important source of guidance for
Few of the subjects under the Concurrent List:
interpreting the Constitution. The Supreme Court has held
that the preamble can be used to interpret the Constitution 1. Criminal law and procedure
in cases where the language is ambiguous. 2. Marriage and divorce
3. Adoption
Important Tip
4. Bankruptcy and insolvency
Important points about Preamble:
5. Education, including technical education
Key elements of Preamble:
• Sovereign, socialist, secular, and democratic and 6. Population control and family planning
republic. 7. Protection of wildlife and animals
• Justice, liberty, equality, and fraternity. 8. Trusts and trusteeship
• Unity and integrity of the nation. 9. Price control and essential commodities

PYQ Workbook 38
INDIAN POLITY AND GOVERNANCE

130. Solution: (c) 134. Solution: (b)


Exp) Option c is the correct answer. Exp) Option b is the correct answer
Statement 1 is correct- Parliament can legislate a subject The Independence Act, 1947, was a British Act of Parliament
under the State list in certain conditions. These conditions that partitioned British India into two independent
include during a national emergency under Article dominions, India and Pakistan. The Act did not lay down
250, when a state makes a request under Article 252, to a constitution for either dominion. Instead, it provided
implement international agreements under Article 253, and for the establishment of constituent assemblies in each
dominion, which would be responsible for framing their
during President’s Rule under Article 356.
own constitutions.
Statement 2 is incorrect- Parliament can legislate a subject
under the State list under certain circumstances during 135. Solution: (b)
National Emergency and President’s rule. So, the statement Exp) Option b is the correct answer
that Parliament cannot legislate a subject under the State list The Dhar Commission was appointed by the Government
under any circumstances is not correct. of India in 1948 to study the feasibility of reorganizing the
Statement 3 is correct- As per Article 249 of the Constitution, states of India on a linguistic basis.
if the Rajya Sabha passes a resolution supported by not less Important Tips
than two-thirds of the members present and voting, stating
Dhar Commission-Recommendations
that it is necessary or expedient in the national interest,
1. It advocated for state reorganization based on
Parliament can make laws on any matter enumerated in the
administrative convenience rather than linguistic
State list.
considerations.
131. Solution: (b) 2. The Commission recommended the reorganization of
Exp) Option b is the correct answer States on the basis of the following criteria:

Pair 1 matched correctly: Public health and sanitation are • Geographical contiguity
among the subjects listed in the State List. • Financial self-reliance
Pair 2 matched correctly: Citizenship, naturalization, and • Administrative viability
aliens are among the subjects listed in the Union List. • Potential for development
Pair 3 matched correctly: Legal, medical, and other 136. Solution: (d)
professions are among the subjects listed in the Concurrent
Exp) Option d is the correct answer.
List.
The Constituent Assembly of India was the body that framed
132. Solution: (b) the Constitution of India. It was established under the
Exp) Option b is the correct answer Cabinet Mission Plan of 1946 and consisted of 389 members,
of which 296 were elected by the provincial assemblies and
Statement 1 is incorrect: The ‘Objectives Resolution’ was
93 were nominated by the princely states
actually moved by Jawaharlal Nehru on December 13, 1946,
Statement 1 is correct: The Constituent Assembly was not
in the Constituent Assembly of India. This resolution laid
based on an adult franchise, as the provincial assemblies
down the fundamental principles and objectives that would
themselves were elected on a limited franchise
guide the framing of the Constitution.
Statement 2 is incorrect: It was also not a result of direct
Statement 2 is correct: Upon India’s independence on 15 election, as the members were elected indirectly by the
August 1947, the new prime minister Jawaharlal Nehru provincial assemblies or nominated by the princely states.
invited Ambedkar to serve as the Dominion of India’s Law
Statement 3 is correct: It was a multi-party body, as it
Minister; two weeks later, he was appointed Chairman of
included representatives from various political parties
the Drafting Committee of the Constitution for the future such as the Indian National Congress, the Muslim League,
Republic of India. the Communist Party of India, etc.
133. Solution: (a) Statement 4 is correct: It worked through several committees,
such as the Drafting Committee, the Fundamental Rights
Exp) Option a is the correct answer
Committee, the Union Powers Committee, etc., to deal
The Constitution of India was framed between December with different aspects of the constitution-making process .
1946 and December 1949. The Constituent Assembly, which
was elected in 1946, met for the first time on December 137. Solution: (b)
9, 1946. The Assembly appointed a Drafting Committee in Exp) Option b is the correct answer.
1947, and the final draft of the Constitution was adopted According to Granville Austin, one of the most significant
on November 26, 1949. The Constitution came into force on contributions of India to constitution-making was
January 26, 1950. Consensus and Accommodation. This is because he argued

39 PYQ Workbook
INDIAN POLITY AND GOVERNANCE

that the Indian Constitution was a product of a remarkable • diminish the area of any State;
process of consensus-building and accommodation among
• alter the boundaries of any State;
various groups and interests in the Constituent Assembly and
outside. He highlighted how the framers of the Constitution • alter the name of any State.
sought to reconcile the conflicting demands of democracy, 139. Solution: (a)
nationalism, social justice, federalism, secularism, minority
Exp) Option a is the correct answer.
rights, and individual liberties in a diverse and complex
society. Statement 1 is correct: The Preamble of the Constitution
of India is a brief introductory statement that sets out the
Important Tips guiding principles and purpose of the Constitution. It is
• Granville Austin (1927-2014) was an American based on the Objective Resolution, which was moved by
historian and a leading authority on the Indian Jawaharlal Nehru in the Constituent Assembly on 13
Constitution. December 1946 and adopted on 22 January 1947.
• He wrote two influential books on the making and Statement 2 is correct: The idea of the preamble was
working of the Indian Constitution, namely The borrowed from the Constitution of the USA.
Indian Constitution: Cornerstone of a Nation (1966) Statement 3 is correct: The Preamble declares India to be a
and Working a Democratic Constitution: The Indian sovereign, socialist, secular, democratic republic and secures
Experience (1999). to all its citizens justice, liberty, equality and fraternity.
• He was awarded the Padma Shri by the Government The words socialist and secular were added by the 42nd
of India in 2011 for his contribution to constitutional Amendment in 1976.
studies. Statement 4 is incorrect: While the Preamble of the
Constitution of India sets out the ideals and objectives
138. Solution: (c)
of the Constitution, it does not explicitly mention the
Exp) Option c is the correct answer. Fundamental Rights. The Fundamental Rights are enshrined
According to Article 3 of the Constitution of India, the in Part III of the Constitution.
Parliament may by law:
140. Solution: (a)
• Alter the boundaries of any State: The Parliament has
Exp) Option a is the correct answer.
the power to alter the boundaries of any State in India.
This can be done through the process of reorganization Statement 1 is correct: Constitutional government is a form
of states, which requires the introduction and passing of of limited government. The constitution sets the boundaries
a law by Parliament. and limits of the powers of the government and prevents it
from becoming arbitrary or tyrannical. The constitution also
• Increase the area of any State: The Parliament also has
ensures that the government is accountable to the people and
the authority to increase the area of any existing state by
respects their rights and freedoms.
annexing or merging territories with it. This can be done
Statement 2 is correct: A constitutional government is one
through the enactment of a law by Parliament.
where the constitution is the basis of public authority.
Statement 1 is not related to Article 3, but to Article 253, The constitution establishes the structure, functions, and
which empowers Parliament to make laws for implementing procedures of the government and defines its relationship
treaties, agreements or conventions with foreign countries or with the people. The constitution also lays down the
international organizations. principles and values that guide the actions and decisions of
Statement 4 is not related to Article 3, but to Article 244A, the government and its officials.
which provides for the creation of an autonomous state Statement 3 is incorrect: A constitutional government does
comprising certain tribal areas in Assam and for the creation of not necessarily have a monarch as the formal head of the
local legislature or council of ministers or both for such state. state. A constitutional government can have different forms of
Important Tips executive leadership, such as a president, a prime minister, or
a chancellor.
Article 3 of the Constitution of India deals with the
formation of new States and alteration of areas, boundaries Statement 4 is incorrect: A constitutional government does
or names of existing States. In this regard, the Parliament not always have a universal adult franchise. A constitutional
may by law: government can have different forms of electoral systems,
such as proportional representation, first-past-the-post, or
• form a new State by separation of territory from any
mixed-member proportional. A universal adult franchise is
State or by uniting two or more States or parts of States,
one of the possible ways of ensuring political participation
or by uniting any territory to a part of any State;
and representation in a constitutional democracy, but it is
• increase the area of any State; not a necessary condition for a constitutional government.

PYQ Workbook 40
INDIAN POLITY AND GOVERNANCE

INDIAN POLITY AND GOVERNANCE


FOUNDATIONS OF THE CONSTITUTION
*This unit consists of questions from Fundamental Rights, Directive Principles of State Policy,
Fundamental Duties, Basic Structure, Emergency Provisions & Amendment to the Constitution.

2.1. UPSC CSE Previous Years’ Questions (a) Article 14


(b) Article 28
1. In India, which one of the following (c) Article 32
Constitutional Amendments was widely (d) Article 44
believed to be enacted to overcome the
5. Under the Indian constitution,
judicial interpretations of the Fundamental
Rights? [UPSC CSE Pre 2023] concentration of wealth violates
(a) 1st Amendment [UPSC CSE Pre 2021]
(b) 42nd Amendment (a) The Right to equality
(c) 44th Amendment (b) The Directive principles of state policy
(d) 86th Amendment (c) The right to freedom
(d) The concept of welfare
2. Consider the following Statements:
[UPSC CSE Pre. 2022] 6. What is the position of the right to property
1. A bill amending the Constitution requires in India? [UPSC CSE Pre 2021]
a prior recommendation of the Président (a) Legal right available to citizens only
of India. (b) Legal right available to any person
2. When a Constitution Amendment Bill is (c) Fundamental Right available to citizens
presented to the President of India, it is only
obligatory for the President of India to (d) Neither Fundamental Right nor legal
give his/her assent. right
3. A Constitution Amendment Bill must 7. Which one of the following categories
be passed by both the Lok Sabha and the of Fundamental Rights incorporate
Rajya Sabha by a special majority and protection against untouchability as a form
there is no provision for joint sitting. of discrimination? [UPSC CSE Pre. 2020]
Which of the statements given above are (a) Right against Exploitation
correct? (b) Right to Freedom
(a) 1 and 2 only (c) Right to Constitutional Remedies
(b) 2 and 3 only (d) Right to Equality
(c) 1 and 3 only 8. With reference to the provisions contained
(d) 1, 2 and 3 in Part IV of the Constitution of India,
3. ‘Right to Privacy’ is protected under which which of the following statements is/are
Article of the Constitution of India? correct? [UPSC CSE Pre. 2020]
[UPSC CSE Pre 2021] 1. They shall be enforceable by courts.
(a) Article 15 2. They shall not be enforceable by any
(b) Article 19 court.
(c) Article 21 3. The principles laid down in this part are
(d) Article 29 to influence the making of laws by the
4. A legislation which confers on the executive State.
or administrative authority an unguided Select the correct answer using the code given
and uncontrolled discretionary power in below:
the matter of application of law violates (a) 1 only
which one of the following Articles of the (b) 2 only
Constitution of India? (c) 1 and 3 only
[UPSC CSE Pre 2021] (d) 2 and 3 only

41 PYQ Workbook
INDIAN POLITY AND GOVERNANCE

9. Which Article of the Constitution of India 1. Dissolution of the State Legislative


safeguards one’s right to marry the person Assembly
of one’s choice? [UPSC CSE Pre. 2019] 2. Removal of the Council of Ministers in
(a) Article 19 the State
(b) Article 21 3. Dissolution of the local bodies
(c) Article 25
(d) Article 29 Select the correct answer using the code given
below:
10. In the context of polity, which one of the
following would you accept as the most (a) 1 and 2 only
appropriate definition of liberty? (b) 1 and 3 only
[UPSC CSE Pre. 2019] (c) 2 and 3 only
(a) Protection against the tyranny of political (d) 1, 2 and 3
rulers 15. Right to vote and to be elected in India is a:
(b) Absence of restraint [UPSC CSE Pre. 2017]
(c) Opportunity to do whatever one likes
(d) Opportunity to develop oneself fully (a) Fundamental Right
(b) Natural Right
11. Right to Privacy is protected as an intrinsic (c) Constitutional Right
part of Right to Life and Personal Liberty. (d) Legal Right
Which of the following in the Constitution
of India correctly and appropriately imply 16. Which of the following are envisaged
the above statement?[UPSC CSE Pre. 2018] by the Right against Exploitation in the
(a) Article 14 and the provisions under the Constitution of India?
42nd Amendment to the Constitution [UPSC CSE Pre. 2017]
(b) Article 17 and the Directive Principles of 1. Prohibition of traffic in human beings
State Policy in Part IV
and forced labour
(c) Article 21 and the freedoms guaranteed
in Part III 2. Abolition of untouchability
(d) Article 24 and the provisions under the 3. Protection of the interests of minorities
44th Amendment to the Constitution 4. Prohibition of employment of children in
12. Which one of the following reflects the factories and mines
most appropriate relationship between law Select the correct answer using the code given
and liberty? [UPSC CSE Pre. 2018] below:
(a) If there are more laws, there is less liberty. (a) 1, 2 and 4 only
(b) If there are no laws, there is no liberty. (b) 2, 3 and 4 only
(c) If there is liberty, laws have to be made by (c) 1 and 4 only
the people. (d) 1, 2, 3 and 4
(d) If laws are changed too often, liberty is in
danger. 17. Consider the following statements:
13. Which of the following statements is/are [UPSC CSE Pre. 2017]
true of the Fundamental Duties of an Indian With reference to the Constitution of India,
citizen? [UPSC CSE Pre. 2017] the Directive Principles of State Policy
1. A legislative process has been provided to constitute limitations upon:
enforce these duties. 1. legislative function.
2. They are correlative to legal duties. 2. executive function.
Select the correct answer using the code given
Which of the above statements is/are correct?
below:
(a) 1 only (a) 1 only
(b) 2 only (b) 2 only
(c) Both 1 and 2 (c) Both 1 and 2
(d) Neither 1 nor 2 (d) Neither 1 nor 2
14. Which of the following are not necessarily 18. Which principle among the following was
the consequences of the proclamation of the added to the Directive Principles of State
President’s rule in a State? Policy by the 42nd Amendment to the
[UPSC CSE Pre. 2017] Constitution? [UPSC CSE Pre. 2017]

PYQ Workbook 42
INDIAN POLITY AND GOVERNANCE

(a) Equal pay for equal work for both men 3. Promoting cottage industries in rural
and women areas.
(b) Participation of workers in the 4. Securing for all the workers reasonable
management of industries leisure and cultural opportunities.
(c) Right to work, education and public Which of the above Directive Principles of
assistance State Policy reflect Gandhian principles?
(d) Securing living wage and human (a) 1, 2 and 4 only
conditions of work to workers (b) 2 and 3 only
19. In the context of India, which one of (c) 1, 3 and 4 only
the following is the correct relationship (d) 1, 2, 3 and 4
between Rights and Duties? 23. With reference to the Constitution of India,
[UPSC CSE Pre. 2017] consider the following:
(a) Rights are correlative with Duties. 1. Fundamental Rights
(b) Rights are personal and hence 2. Fundamental Duties
independent of society and Duties.
3. Directive Principles of State Policy
(c) Rights, not Duties, are important for the
advancement of the personality of the Which of the above provisions of the
citizen. Constitution of India fulfilled the National
(d) Duties, not Rights, are important for the Social Assistance Programme launched by
stability of the State. the Government of India?
[UPSC CSE Pre 2010]
20. The ideal of ‘Welfare State’ in the Indian (a) Only 1
Constitution is enshrined in its (b) Only 3
[UPSC CSE Pre. 2015] (c) Only 1 and 3
(a) Preamble (d) 1, 2 and 3
(b) Directive Principles of State Policy
(c) Fundamental Rights 24. How are the Directive Principles of State
(d) Seventh Schedule Policy different from Fundamental Rights?
[UPSC CSE (Pre) 2009]
21. Which of the following is/are among the (a) Former is for Union Government and
Fundamental Duties of citizens laid down later is for state Government.
in the Indian Constitution? (b) Former is a part of Constitution whereas
[UPSC CSE Pre. 2012] later is not.
1. To preserve the rich heritage of our (c) Directive Principles are not enforceable
composite culture. whereas Fundamental Right can be
2. To protect the weaker sections from enforced.
social injustice. (d) None of these.
3. To develop the scientific temper and 25. Which of the following is/are included in
spirit of inquiry. the Directive Principles of State Policy
4. To strive towards excellence in all spheres 1. Prohibition of Trafficking in human
of individual and collective activity. beings and forced labour.
Select the correct answer using the codes 2. Prohibition of consumption except for
given below: medicinal purposes of intoxicating drinks
(a) 1 and 2 only and of other drugs which are injurious to
(b) 2 only health.
(c) 1, 3 and 4 only Select the correct answer using the code given
(d) 1, 2, 3 and 4 below? [UPSC CSE (Pre) 2008]
22. Consider the following provisions under (a) 1 only
the Directive Principles of State Policy as (b) 2 only
enshrined in the Constitution of India: (c) Both 1 and 2
[UPSC CSE Pre. 2012] (d) Neither nor 2
1. Securing for citizens of India a uniform 26. Consider the following statements in respect
civil code. of financial emergency under Article 360 of
2. Organizing village Panchayats. the Constitution of India:

43 PYQ Workbook
INDIAN POLITY AND GOVERNANCE

1. A Proclamation of financial emergency with the promotion of international peace


issued shall cease to operate at the and security? [UPSC CSE (Pre) 2002]
expiration of two months, unless, before (a) 51
the expiration of that period, it has been (b) 48 A
approved by the resolutions of both (c) 43 A
Houses of Parliament. (d) 41
2. If any Proclamation of financial emergency
is in operation, it is competent for the 31. Which one of the following amendments
President of India to issue directions for to the Indian Constitution empowers
the reduction of salaries and allowances the President to send back any matter
of all or any class of persons, serving in for reconsideration by the Council of
connection with the affairs of the Union Ministers? [UPSC CSE (Pre) 2002]
but excluding the Judges of the Supreme (a) 39th
Court and the High Courts. (b) 40th
Which of the statements given above is/are (c) 42nd
correct? [UPSC CSE (Pre) 2007] (d) 44th
(a) 1 only 32. The 93rd Constitution Amendment Bill
(b) 2 only deals with the: [UPSC CSE (Pre) 2002]
(c) Both 1 and 2 (a) Continuation of reservation for backward
(d) Neither 1 nor 2 classes in government jobs
27. Consider the following statements – (b) Free and compulsory education for all
1. Article 301 is related to Right to Property. children between the age of 6 and 14 years
2. Right to Property is a legal right but not a (c) Reservation of 30 percent posts for
Fundamental Right. women in government recruitments
3. Article 300-A was inserted in Indian (d) Allocation of more number of
Constitution by 44th Amendment during Parliamentary seats for recently formed
the period of Congress Government. States
Which of aforesaid statements is/are correct? 33. In the Indian Constitution, the right to
[UPSC CSE (Pre) 2005] equality is granted by five Articles. They
(a) Only 2 are: [UPSC. CSE (Pre) 2002]
(b) 2 and 3 (a) Article 16 to Article 20
(c) 1 and 3 (b) Article 15 to Article 19
(d) 1, 2 and 3 (c) Article 14 to Article 18
28. Which one of following Articles of the (d) Article 13 to Article 17
Indian constitution provide that it shall be 34. Match List-I (Article of Indian Constitution)
the duty of the Union to protect every state with List-II (Provisions) and select the
against external aggression and internal correct answer using the code given below
disturbance? [UPSC CSE (Pre) 2003] the lists:
(a) Article 215
(b) Article 275 List-I List-II
(c) Article 325 (Article (Provisions)
(d) Article 355 of Indian
Constitution)
29. The purpose of the inclusion of Directive
Principles of State Policy in the Indian A. Article 16 1. No person shall be
Constitution is to establish (2) deprived of his property
[UPSC CSE (Pre) 2002] by the authority of law.
(a) Political Democracy B. Article 29 2. No person can be
(b) Social Democracy (2) discriminated against
(c) Gandhian Democracy in the matter of public
(d) Social and Economic Democracy appointment on the
30. Which one of the following Articles of the grounds of race, religion
Directive Principles of State Policy deals or caste.

PYQ Workbook 44
INDIAN POLITY AND GOVERNANCE

C. Article 30 3. All minorities whether (a) 1, 2 and 3


(1) based on religion or (b) 1, 2 and 4
language shall have the (c) 1, 3 and 4
right to establish and (d) 2, 3 and 4
administer educational 38. Match list-I with list-II and select the
institutions of their correct answer using the codes given below
choice.
the lists:
D. Article 31 4. No citizen shall be
(1) denied admission List-I List-II
into any educational A. 13th 1. Nagaland
institution maintained Amendment
by the State, or receiving B. 18th 2. Anti-Defection Law
State aid, on the grounds Amendment
of religion, race, caste,
C. 39th 3. Clarified the ‘state’
language or any of them. Amendment
Code: [UPSC CSE (Pre) 2002]
D. 52nd 4. The election of
A B C D Amendment President Speaker
(a) 2 4 3 1 and Prime Minister
(b) 3 1 2 4 cannot be challenged
(c) 2 1 3 4 Code [UPSC CSE (Pre) 1995]
(d) 3 4 2 1
A B C D
35. A British citizen staying in India cannot (a) 2 4 1 3
claim right to– [UPSC CSE (Pre) 1999] (b) 4 3 2 1
(a) Freedom of Trade and Profession (c) 1 3 4 2
(b) Equality before the Law (d) 1 3 2 4
(c) Protection of life and Personal Liberty
(d) Freedom of Religion 39. Prohibition of discrimination on the
ground of religion etc. (Article 15 of the
36. An amendment to the Constitution of India
Constitution of India) is a Fundamental
can be initiated by the:
Right classifiable under:
1. Lok Sabha
[UPSC CSE (Pre) 1995]
2. Rajya Sabha
3. State Legislatures (a) The Right to Freedom of Religion
(b) The Right against Exploitation
4. President
(c) The Cultural and Educational Rights
Select the correct answer using the codes
(d) The Right to Equality
given below: [UPSC CSE (Pre) 1999]
(a) 1 only 40. The basic structure theory of the
(b) 1, 2 and 3 Constitution of India Implies that-
(c) 2, 3 and 4 [UPSC CSE (Pre) 1994]
(d) 1 and 2 (a) Certain features of the Constitution
37. Which of the following are the matters are so essential to it that they cannot be
on which a constitutional amendment is abrogated.
possible only with the ratification of the (b) Fundamental rights cannot be abridged
legislatures of not less than one-half of the or taken away.
states? (c) The Constitution cannot be amended
1. Election of the President except in accordance with the procedure
2. Representation of the States in Parliament laid down in Article 368.
3. Any of the Lists in the 7th Schedule (d) The Preamble of the Constitution
4. Abolition of the Legislature Council of a cannot be amended for it is not a part
State of the Constitution and at the same time
Code: [UPSC CSE (Pre) 1995] represents its real spirit.

45 PYQ Workbook
INDIAN POLITY AND GOVERNANCE

41. Freedom of News Papers in India- (b) The promotion of 2. Article 46


[UPSC CSE (Pre) 1994] educational and
(a) is specifically provided in Article 19 (1) economic interests of
(a) of the Constitution Scheduled Castes and
(b) is implied in the wider freedom of Scheduled Tribes
expression guaranteed by Article 19(1)(a)
(c) The promotion of Co- 3. Article 43A
of the Constitution
operative Societies
(c) is guaranteed under the provisions of
Article 361 A of the Constitution (d) The organization 4. Article 40
(d) emanates from the operation of the Rule of village panchayat
of Law In the country and enable them to
function as the units of
2.2. Other Examination Previous Years’ self-governace
Questions
46. Which Constitution Amendment Act
42. Which of the following is not a fundamental
duty? [UPPCS (Pre) 2022] reduced the voting age from 21 years to 18
(a) To protect and improve the natural years? [U.P.P.C.S. (Pre) 2021]
environment (a) 66th Amendment
(b) To respect the National Anthem (b) 62nd Amendment
(c) To protect monuments and places of (c) 61st Amendment
national importance (d) 63rd Amendment
(d) To safeguard public property
43. Right to Education has been included into 47. Consider the following and arrange these in
the Fundamental Rights by the- chronological order:
[U.P.P.C.S. (Pre) 2022] I. Golak Nath Case
(a) Constitution (93rd Amendment) Act, II. Keshavanand Bharti Case
2005
(b) Constitution (71st Amendment) Act, III. 24th Constitution Amendment Act
1992 IV. 42nd Constitution Amendment Act
(c) Constitution (103rd Amendment) Act,
2019 Select the correct answer from the codes
(d) Constitution (86th Amendment) Act, given below. [U.P.P.C.S. (Pre) 2021]
2002 (a) I, III, II, IV
44. The concept of ‘Basic Structure of the (b) I, II, III, IV
Constitution’ was propounded by the (c) III, I, II, IV
Supreme Court in the case of- (d) III, I, IV, II
[U.P.P.C.S. (Pre) 2022]
48. Which of the following Article was inserted
List-I List-II by the 42nd Constitution Amendment to
A. Golaknath vs State of Punjab 1. 1967 provide for participation of workers in
B. Yusuf vs State of Bombay 2. 1954 management? [UPPCS (Pre) 2019]
C. Keshvanand Bharti vs State of 3. 1973 (a) Article 38
Kerala (b) Article 39A
D. Chitralekha vs State of 4. 1964 (c) Article 45
Mysore (d) Article 43A
45. Which one of the following is not correctly 49. Given below are two statements, one is
matched? [U.P.P.C.S. (Pre) 2021] labelled as Assertion (A) and the other as
(Subject) (Related Reason (R):
Articles) Assertion (A): Article 30 of the Constitution
(a) The separation 1. Article 50 of India does not define the term’ minorities.
of judiciary from Reason (R): The Constitution recognizes
executive only linguistic and religious minorities.

PYQ Workbook 46
INDIAN POLITY AND GOVERNANCE

Select the correct answer from the codes 53. Which one of the following does not fall
given below: [U.P.P.C.S. (Pre) 2019] within the purview of article 21 of the
(a) Both (A) and (R) are true and (R) is the Constitution? [U.P.P.C.S. (Pre) 2017]
correct explanation of (A).
(a) Medical aid to injure by a doctor
(b) Both (A) and (R) are true, but (R) is not
the correct explanation of (A). (b) Sexual Harassment of Women at
(c) (A) is true, but (R) is false. workplace.
(d) (A) is false, but (R) is true. (c) Pollution of the quality of water.
50. In which of the following ‘the provisions (d) Capital punishment
for the protection and improvement of 54. Given below are two statements, one is
environment and safeguarding of forests labelled as Assertion (A) and the other as
and wildlife’ are found in the Constitution
Reason (R):
of India? [U.P.P.C.S. (Pre) 2019]
(a) Only in the Directive Principles of State Assertion (A): The State can treat unequal
Policy differently with the objective of creating a
(b) Only in the Fundamental Duties level playing field in the social, economic and
(c) Both (a) and (b) political spheres.
(d) None of the above
Reason (R): Among equals the law should be
51. The Directive principles of State Policy may equal and equally administered.
be classified into which of the following
parts for the sake of convenient study? In the context of the above statements select
[U.P.P.C.S. (Pre) 2018] the correct answer [U.P.P.C.S. (Mains) 2016]
(a) Socialist (a) (A) is true, but (R) is false.
(b) Liberal intellectualistic (b) (A) is false, but (R) is true.
(c) Gandhian (c) Both (A) and (R) are correct, but (R) is
(d) All of the above not the correct explanation of (A).
52. Match list-I with list-II and select the (d) Both (A) and (R) are true and (R) is the
correct answer using the codes given below correct explanation of (A).
the lists:
55. In the Indian Constitution the ‘Right to
List-I (Article) List-II (Provision) Freedom’ is granted by four Articles which
A. Curtailment of 1. 61st Amendment are– [U.P.P.C.S. (Mains) 2016]
the power of
judicial review (a) Article – 19 to Article – 22
(b) Article – 16 to Article – 19
B. Abolition 2. 42nd Amendment
of Right to (c) Article – 17 to Article – 20
property as a (d) Article – 18 to Article – 21
fundamental
right 56. Given below are two statements, one is
labelled as Assertion and the other as
C. Lowering the 3. 38th Amendment
voting age from Reason (R):
21 to 18 Assertion (A): The state shall provide free
th
D. Addition of the 4. 44 Amendment and compulsory education to all children of
word ‘Secular’ the age group of six to fourteen years.
in the Preamble
Reason (R): In a democratic society, the
Code: [UPPCS (Pre) 2017]
right to education is indispensable in the
A B C D
interpretation of the right to development as
(a) 1 2 4 3
a human right.
(b) 2 4 1 3
(c) 3 4 1 2 In the context of the above statements select
(d) 4 1 3 2 the correct answer. [U.P.P.C.S. (Mains) 2016]

47 PYQ Workbook
INDIAN POLITY AND GOVERNANCE

(a) Both (A) and (R) are true, and (R) is the 60. Which one of the following is not a part
correct explanation of (A). of Fundamental Duties under Indian
(b) Both (A) and (R) are true, but (R) is not Constitution?
the correct explanation of (A). (a) To defend the country and render national
(c) (A) is true, but (R) is false. service
(d) (R) is true, but (A) is true. (b) To value and preserve the rich heritage of
our composite culture
57. Given below are two statements, one is (c) To help in organizing Village Panchayats
labelled as Assertion (A) and the other as (d) To safeguard public property and to
Reason (R): abjure violence
Assertion (A): Dr. Ambedkar had described 61. Consider the following:
Article 32 of the Constitution as the very soul Assertion (A): Under Article 368, the
of it. Parliament can amend any part of the
Constitution.
Reason (R): Article 32 provides an effective
remedy against the violation of fundamental Reason (R): The Parliament is the Supreme
Legislative body elected by the People of
rights. India.
Select the correct answer using the codes Code: [U.P.P.C.S. (Pre) 2015]
given below: [U.P.P.C.S. (Pre) 2016] (a) Both (A) and (R) are correct, and (R) is
(a) (A) and (R) both are true, and (R) is the the correct explanation of (A).
correct explanation of (A) (b) Both (A) and (R) are correct, but (R) is
not the correct explanation of (A).
(b) (A) and (R) both are true, but (R) is not (c) (A) is true, but (R) is false.
the correct explanation of (A). (d) (A) is false, but (R) is true.
(c) (A) is true, but (R) is false.
62. The word ‘Hindu’ in Article 25 of the
(d) (A) is false, but (R) is true. Constitution of India does not include–
58. Which one of the following pairs is not [U.P.P.C.S. (Mains) 2014]
correctly matched? (a) Buddhists
[U.P.P.C.S. (Mains) 2015] (b) Jains
(c) Parsis
Provisions Sources (d) Sikhs
(a) Prohibition of traffic in Article 23 63. In which Article of Indian Constitution
human beings and forced Doctrine of Due Process of Law is included?
labour
[U.P.P.C.S. (Mains) 2014]
(b) Protection of interests of Article 29 (a) 11
minorities (b) 16
(c) Right to constitutional Article 32 (c) 21
remedies (d) 26
(d) Right of minorities to Article 31 64. The President of India may declare the
establish and administer National Emergency if there is the threat of:
educational institutions 1. External aggression
59. Under which Articles of Indian Constitution, 2. Internal disturbances
provision for Fundamental Duties has 3. Armed rebellion
been made by the 42nd Constitutional 4. Communal clashes
Amendment Act? [U.P.P.C.S. (Pre) 2015] Select the correct answer from the code given
below: [U.P.P.C.S. (Mains) 2010]
(a) Article 50
(a) 1 and 2 only
(b) Article 51A (b) 2 and 3 only
(c) Article 52 (c) 3 and 4 only
(d) Article 53 (d) 1 and 3 only

PYQ Workbook 48
INDIAN POLITY AND GOVERNANCE

65. Under Article 356 of the Constitution of 70. By Which one of the following
India, President’s Rule was imposed for the Constitutional Amendments Delhi became
first time in- [U.P.P.C.S. (Mains) 2010] ‘National Capital Region’?
(a) Uttar Pradesh [U.P.P.C.S. (Mains) 2009
(b) Travancore-Cochin (a) 61st Amendment
(c) PEPSU (b) 69th Amendment
(d) Bihar (c) 71st Amendment
(d) 79th Amendment
66. Which of the following is correct?
[U.P.P.C.S. (Mains) 2009] 71. Which Amendment of the Constitution
provides that no law passed to give effect
(a) The Nehru Report (1928) had advocated
to Directive Principles of State Policy
the inclusion of Fundamental Rights in
contained in Articles 39 (b) & (c) shall be
the Constitution of India. deemed to be void on the ground that it
(b) The Government of India Act, 1935 abridges the rights conferred by Articles 14
referred to Fundamental Rights. and 19? [U.P.P.C.S. (Mains) 2009]
(c) The August Offer, 1940, included the (a) 25th Amendment
Fundamental Rights. (b) 28th Amendment
(d) The Cripps Mission 1942 referred to (c) 42nd Amendment
Fundamental Rights. (d) 44th Amendment
67. Which of the following Articles of the 72. Which one of the following is not included
Indian Constitution guarantees equality in the fundamental right to equality as
of opportunity to all citizens of India in enshrined in the Indian Constitution?
matters relating to public employment? [U.P.P.C.S. (Spl) (Mains) 2008]
U.P.P.C.S. (Mains) 2009] (a) Equality before law
(a) Article 15 (b) Social equality
(b) Article 16 (1) and 16 (2) (c) Equal opportunity
(c) Article 16 (3) (d) Economic equality
(d) Article 16 (3), (4) and (5) 73. Which one of the following rights cannot
68. Which one of the following is incorrect? be suspended or restricted even during
[U.P.P.C.S. (Mains) 2009] National Emergency?
[U.P.P.C.S. (Mains) 2008]
(a) Fundamental Duties are part of
Fundamental Rights. (a) Right to reside and settle in any part of
the country.
(b) Fundamental Duties are not a part of
(b) Right to life and personal liberty.
Fundamental Rights.
(c) Right to move freely throughout the
(c) Fundamental Duties are enumerated in
territory of India.
Part-IV A of the Indian Constitution.
(d) Right to carry on any profession or
(d) Article 51 A explains the duties of every
business.
citizen of India.
74. Under which of the following articles
69. The First Constitution Amendment Bill the President of India can suspend the
passed in 1951 was related to enforcement of Fundamental Rights (except
[U.P.P.C.S. (Mains) 2009] Articles 20, 21)?
(a) Security of the country [U.P.P.C.S. (Spl) (Mains) 2008]
(b) Security of the Prime Minister (a) Article 358
(c) Protection of agrarian reforms in certain (b) Article 359
States (c) Article 13
(d) Scheduled castes and Scheduled tribes (d) Article 356

49 PYQ Workbook
INDIAN POLITY AND GOVERNANCE

75. The 79th Amendment of the Indian (c) Right to privacy


Constitution is related to- (d) Right to information
[U.P.P.C.S. (Mains) 2008] 80. Which part of Constitution of India
(a) Centre-State Relations describes the model of Welfare State?
(b) Establishment of two political parties [U.P.P.C.S. (Mains) 2004]
(c) Fundamental Rights (a) Fundamental Rights
(d) Reservation of SCs and STs in the Lok (b) Fundamental Duties
Sabha and State Legislative Assemblies (c) The Preamble
76. Match List-I with List-II and select the (d) Directive Principles of State Policy
correct answer using the code given below 81. According to the Indian Constitution, the
the lists: Constitutional amendment of the following
List-I (Article) List-II (Provision) topics requires consent of at least half of
State legislatures-
A. Fundamental 1. Minerva Mills Case
Duties 1. Federal provision of the Constitution
B. Parliament 2. Article 23 of the 2. Rights of the Supreme Court
can amend Constitution 3. The process of Constitutional Amendment
Fundamental 4. Formation of new States or transformation
Rights in the borders and names of the states
C. Doctrine of 3. 42nd Amendment Code: [U.P.P.C.S. Lower (Pre) 2004]
Basic Structure of the Constitution (a) 1, 2, 3
D. Prohibition of 4. Keshvanand Bharti (b) 1, 2, 4
Trafficking in Case (c) 1, 3, 4
Human Beings (d) 2, 3, 4
Code: [U.P.P.C.S. (Mains) 2007]
82. In the event of the Upper House rejecting
A B C D a Constitutional Amendment Bill passed by
(a) 3 1 4 2 the Lower House- [U.P.P.C.S (Mains) 2004]
(b) 4 3 2 1
(a) The Bill will have to be dropped
(c) 2 1 4 3
(b) A joint sitting of both Houses of
(d) 1 2 3 4
Parliament may be convened to consider
77. Which one of the following Bills must and pass the Bill
be passed by each House of the Indian (c) The Bill shall be deemed to have been
Parliament separately by the special passed by both the Houses of the
majority? [U.P.P.C.S. (Mains) 2007]
Parliament if the Lok Sabha passes it
(a) Ordinary Bill again by a two-third majority
(b) Money Bill
(d) The Bill shall be deemed to have been
(c) Finance Bill
(d) Constitution Amendment Bill passed by both the Houses of the
Parliament despite the disapproval of the
78. Right to education to all children between Bill by the Upper House
the age group of 6 to 14 years is-
U.P.P.C.S. (Pre) 2006] 83. Which of the following statements regarding
(a) Included in the Directive Principles of Fundamental Duties is not true:
State Policy [U.P.P.C.S. (Pre) 2003]
(b) A Fundamental Right (a) They can be enforced by writs
(c) A Statutory Right (b) They can be promoted only by
(d) None of the above constitutional methods
79. Which one of the following rights is (c) They can be used for interpreting
available under Article 19(1)read with ambiguous statutes
Article 21: [U.P.P.C.S. (Mains) 2004] (d) The performance of any duty comes
(a) Right to travel abroad within the sphere of constitutional law
(b) Right to Shelter which court has to decide.

PYQ Workbook 50
INDIAN POLITY AND GOVERNANCE

84. Which of the following rights are not 4. They are unlike Bill of Rights in the U.S.A.
available to all persons in India? Select the correct answer from the code given
Select your answer by using the code given below: [U.P.P.C.S. (Pre) 1998]
below: (a) 1 and 2 are correct
1. Equality Before the Law (b) 2 and 3 are correct
2. Right Against Discrimination (c) 1, 2 and 3 are correct
3. Freedom to Move Freely throughout the (d) 2, 3 and 4 are correct
Country 89. Which of the following has been recognized
4. Right to Contest Election as a fundamental right by the Supreme
Code: [U.P.P.C.S. (Mains) 2002] Court of India?
(a) 1, 3, 4 1. Right to Shelter
(b) 1, 2, 4 2. Right to Travel Abroad
(c) 1, 2, 3
(d) 2, 3, 4 3. Right to Equal Pay for Equal Work
Select the correct answer from the code given
85. Identify the items included in the Directive below: [U.P.P.C.S. (Pre) 1998]
Principles of State Policy:
(a) 1 and 2
1. Prohibition of Consumption of Liquor
(b) 2 and 3
2. Prohibition of employment of children in (c) 1 and 3
factories or mines (d) 1, 2 and 3
3. Prohibition of beggar or forced labour
4. Prohibition of untouchability 90. Which of the following was not added
to the Directive Principles by the 42nd
Select the correct answer with the help of the
code given below: [U.P.P.C.S. (Mains) 2002] Amendment of the Constitution?
(a) 1 and 2 [U.P.P.C.S. (Pre) 1998]
(b) 1, 2 and 3 (a) Protection of Children and youth from
(c) Only 1 exploitation
(d) 2, 3 and 4 (b) Equal justice and free legal aid
(c) Uniform civil code
86. In which of the following cases the Supreme (d) Participation of workers in the
Court limited the power of Parliament to management of industries
amend the Constitution for the first time?
[U.P.P.C.S. (Mains) 2002] 91. Which one of the following writs is regarded
(a) A.K. Gopalan Vs. The State of Madras as the greatest safeguard for the liberty of a
(b) Golaknath Vs. The State of Punjab person? [U.P. Lower Sub. (Pre) 2015]
(c) Keshavananda Bharati Vs. The State of (a) Mandamus
Kerala (b) Habeas Corpus
(d) Minerva Mills Vs. The Union of India (c) Certiorari
(d) Prohibition
87. Which of the following provisions under the
Constitution of India enabled the Supreme 92. Which one of the following statements is
Court of India to deduce a fundamental not correct? [U.P. Lower Sub. (Pre) 2013]
right to equal pay for equal work? (a) K.M. Munshi was one of the members
[U.P.P.C.S. (Pre) 1999] of the drafting committee of the
(a) The word ‘socialist’ used in the Preamble Constitution.
to the Constitution (b) The Constitution of India was adopted by
(b) (a) read with Article 14 of the Constitution the Constituent Assembly.
(c) (a) read with Article 16 of the Constitution (c) The Panchayati Raj was recommended
(d) (a), (b) and (c) all read together by Balwant Rai Mehta Committee
88. Consider the following statements about Report-1957.
the Fundamental Rights in India: (d) The President of India is the Guardian
1. They are a guarantee against state action. of Fundamental Rights under the
2. They are enumerated in Part III of the Constitution.
Constitution. 93. Given below are two statements, one
3. They ensure social, economic and labelled as Assertion (A) and the other as
political justice. Reason (R):

51 PYQ Workbook
INDIAN POLITY AND GOVERNANCE

Assertion (A): MGNREGA is providing (c) II and III


employment to at least one member of the (d) I, II and IV
eligible household for a minimum period of (e) None of the above/More than one of the
100 days in a year. above
Reason (R): Right to employment is 98. The Supreme Court has held that hoisting
enumerated in Part III of the Constitution. the National Flag atop the private buildings
Select the correct answer from the code given is a fundamental right of every citizen
below: [U.P. Lower Sub. (Mains) 2013] under: [60th to 62nd B.P.S.C. (Pre) 2016]
(a) Both (A) and (R) are true, and (R) is the (a) Article 14 of the Constitution
correct explanation of (A). (b) Article 19 (1) (a) of the Constitution
(b) Both (A) and (R) are true, but (R) is not a (c) Article 21 of the Constitution
correct explanation of (A). (d) Article 25 of the Constitution
(c) (A) is true, but (R) is false. (e) None of the above/More than one of the
(d) (A) is false, but (R) is true. above
94. The Fundamental Duties of a citizen do not 99. Which of the following rights conferred by
include duty – [U.P. Lower Sub. (Pre) 2008] the Constitution of India is also available to
(a) To protect and improve the natural non-citizens?
environment [53rd to 55th B.P.S.C. (Pre) 2011]
(b) To cherish and follow the noble ideals (a) Right to Constitutional Remedies
which inspired our national struggle for (b) Freedom of Speech
freedom (c) Freedom to Move and Settle in any Part
(c) To strive towards abolition of of the Country
untouchability (d) Freedom to Acquire Property
(d) To develop scientific temper, humanism
100. According to which Article National
and the spirit of enquiry and reform
Emergency may be declared by the president
95. Fundamental Rights– of India? [53rd to 55th B.P.S.C. (Pre) 2011]
[U.P. Lower Sub. (Pre) 2002] (a) Article 352
(a) Cannot be suspended (b) Article 370
(b) Can be suspended by order of Prime (c) Article 371
Minister (d) Article 395
(c) Can be suspended on the will of President 101. Which of the following is given the power
(d) Can be suspended during Emergency to enforce the Fundamental Rights by the
96. What is meant by ‘Rule of Law’? Constitution? [47th B.P.S.C. (Pre) 2005]
[66th B.P.S.C. (Pre) 2020] (a) All Courts in India
(a) One act for all and one judiciary for all (b) The Parliament
(b) One act for all and one State for all (c) The President
(c) One State for all and one judiciary for all (d) The Supreme Court and High Courts
(d) All acts for one and one judiciary for all 102. Which of the following Constitutional
(e) None of the above / More than one of the Amendment Acts was termed as Mini
above Constitution? [47th B.P.S.C. (Pre) 2005]
97. Which of the following is/are listed among (a) 42nd
the Directive Principles in Part-IV of the (b) 44th
Constitution of India? (c) 46th
(d) 50th
I. Equal Pay for Equal Work
II. Uniform Civil Code 103. Which Article of the Constitution gives
precedence to Constitutional provisions
III. Small family norm over the laws made by the Union Parliament/
IV. Education through mother tongue at State Legislatures? [45th B.P.S.C. (Pre) 2001]
primary level. (a) 13
Code: [60th to 62nd B.P.S.C. (Pre) 2016] (b) 32
(a) I, II and III (c) 245
(b) I and II (d) 326

PYQ Workbook 52
INDIAN POLITY AND GOVERNANCE

104. Mainly on whose advice is the President’s 110. The ‘Directive Principles’ are
Rule imposed in the State? [M.P.P.C.S. (Pre) 2013]
[45th B.P.S.C. (Pre) 1997] (a) Justifiable
(a) Chief Minister (b) Non-justifiable
(b) Legislative Assembly (c) Rigid
(c) Governor (d) Flexible
(d) Chief Justice of High Court 111. Consider the following:
105. Which of the following statement is correct Statement: Constitution of India is basically
regarding Article 368 of the Constitution of Federal in nature.
India? Reason: Article 352 has provisions to deal
(i) Article 368 was amended by the 24th with the failure of constitutional machinery
Constitution Amendment Act, 1971. in a state.
(ii) Article 368 was amended by the 101st Code: [M.P.P.C.S.] (Pre) 2008]
Constitution Amendment Act, 2016. (a) Statement and Reason both are correct,
Codes: [M.P.P.C.S. (Pre) 2020] and the Statement is the correct
(a) (i) is true and (ii) is false explanation of the reason.
(b) (i) is false and (ii) is true (b) Statement and Reason both are correct,
(c) Both (i) and (ii) are true but the statement is not the correct
(d) Both (i) and (ii) are false explanation of reason.
(c) The statement is correct, but Reason is
106. When was the special rights and privy false.
purse enjoyed by the Indian Princely States (d) The statement is false, but Reason is
abolished? [M.P.P.C.S. (Pre) 2020] correct.
(a) 1950 112. By which of the following case Parliament
(b) 1949 got the right to amend Fundamental Rights?
(c) 1962
[M.P.P.C.S. (Pre) 1991]
(d) 1971
(a) Keshvananda Bharati Case
107. Article 24 of the Constitution of India (b) Rajnarayan Vs. Indira Gandhi Case
prohibits the employment of child in (c) Golaknath’s Case
factories related to hazardous works. Such (d) Sajjan Singh Case
prohibition is [M.P.P.C.S. (Pre) 2020]
113. Identify the correct pair mentioning the
(a) Absolute prohibition Directive Principles of State Policy in the
(b) Partial prohibition Constitution of India-
(c) Reasonable prohibition
[R.A.S./R.T.S. (Pre) 2021]
(d) Moral prohibition
(a) Equal justice and free legal Article
108. The scope of the Article 21 of the
aid 42
Constitution was expanded to include
Right to Education, because of the decision (b) Organization of agriculture Article
of Supreme Court in the case and animal husbandry 43
[M.P.P.C.S. (Pre) 2019] (c) Organization of village Article
(a) Unnikrishnan v/s Andhra Pradesh Panchayat 41
(b) Govind v/s State of Madhya Pradesh (c) That there is equal pay for Article
(c) Parmanand Katara v/s Union of India equal work for both men and 39
(d) Chameli Singh v/s State of Uttar Pradesh women
109. When were the Fundamental Duties 114. Fundamental Duties were incorporated in
mentioned in the Constitution? the Constitution on the recommendation of
[M.P.P.C.S. (Pre) 2017] [R.A.S./R.T.S. (Pre) 2021]
(a) At the time of framing of Constitution (a) Swaran Singh Committee
(b) On 26th January 1950 (b) Shah Commission
(c) In the 42nd Constitutional Amendment (c) Administrative Reforms Commission
(d) In the 41st Constitutional Amendment (d) Santhanam Committee

53 PYQ Workbook
INDIAN POLITY AND GOVERNANCE

115. Select the correct chronological order of B. Protection of SC/ 2. 91st


the following judgements dealing with ST reservation in Amendment,
Fundamental Rights: filling backlog of 2003
(A) Golaknath v/s State of Punjab vacancies
(B) Keshvananda Bharati v/s State of Kerala C. Formation of 3. 97th
(C) Minerva Mills v/s Union of India National Judicial Amendment,
Appointments 2011
(D) A.K Gopalan v/s State of Madras Commission
Select the correct answer: D. Limiting the size 4. 99th
[R.A.S/R.T.S. (Pre) 2018] of the Council of Amendment,
(a) (D), (B), (C), (A) Ministers 2014
(b) (A), (B), (C), (D) Code: [R.A.S./R.T.S. (Pre) (Re-Exam)
(c) (D), (A), (B), (C) 2013]
(d) (D), (C), (B), (A) A B C D
116. Under the provisions of the Article 368 (a) 3 2 4 1
(2) of the Constitution of India, if a bill (b) 1 4 2 3
comes to Rajasthan Legislative Assembly (c) 3 1 4 2
after passing by parliament of India for (d) 2 3 1 4
resolution to rectify, then- 119. Consider the following statements
1. Resolution can be passed by Assembly. regarding Article 356 of the Constitution:
2. Resolution can be rejected by Assembly. (i) Failure of Constitutional Machinery in
3. No Amendment shall be proposed to States is an objective reality.
such a resolution (ii) A Proclamation under this Article can be
Select the correct answer using code given reviewed by the Supreme Court.
below: [R.A.S./R.T.S. (Pre) 2018] (iii) Along with this Proclamation, the State
(a) 1 only Legislative Assembly can be dissolved.
(b) 2 and 3 (iv) This Proclamation has to be approved
(c) 1 and 3 by each House of Parliament within two
(d) 1, 2 and 3 months.
117. “The age of a Judge of the Supreme Court of Which of the statements given above is/are
India shall be determined by such authority correct? [R.A.S./R.T.S.(Pre) 2013]
and in such manner as Parliament may by (a) (i), (ii) and (iv) only
laws provide” inserted by- (b) (i), (ii) and (iii) only
[R.A.S./R.T.S. (Pre) 2018] (c) (iii) and (iv) only
(d) (ii), (iii) and (iv) only
(a) 15th Constitution Amendment
(b) 16th Constitution Amendment 120. The President’s power to veto a Bill for
(c) 17th Constitution Amendment amendment of the Constitution has been
(d) 18th Constitution Amendment taken away by substituting the word ‘shall
give his assent’ by which amendment?
118. Match list-I with list-II and select the
[R.A.S./R.T.S. (Pre) (Re-Exam) 2013]
correct answer using the codes given below
(a) 44th Amendment
the lists:
(b) 23rd Amendment
List-I List-II (c) 24th Amendment
(Provision of (Number of (d) 42nd Amendment
Constitutional Constitutional 121. Consider the following statements:
Amendment) Amendment)
I. Through the 44th Constitutional
A. Right to form 1. 81st Amendment, 1978 all the Directive
Cooperative Amendment, Principles of State Policy have been given
societies under 2000 preference over Fundamental Rights
Article 19 (1) (c) mentioned in Articles 14 and 19.

PYQ Workbook 54
INDIAN POLITY AND GOVERNANCE

II. Supreme Court has laid down in its (c) Article 132
verdict in Minerva Mills Case, 1980 that (d) Article 226
the preference given only to Directive 125. Which Constitutional Amendment
Principles of State Policy mentioned in bifurcated the Combined National
Article 39 (b) and 39 (c) over Fundamental Commission for Scheduled Castes and
Rights mentioned in Articles 14 and 19 is Scheduled Tribes into two separate bodies?
Constitutional. [Jharkhand P.C.S. (Pre) 2016]
Code: [R.A.S./R.TS. (Pre) 2013] (a) 42nd Amendment
(a) Neither I nor II is correct. (b) 44th Amendment
(b) Only I is correct. (c) 89th Amendment
(c) Only II is correct. (d) 93rd Amendment
(d) Both I and II are correct. 126. As a part of the Right to Freedom, which
122. 97th Constitutional Amendment Act of one of the following does not form part
2012 is concerned with- of “Freedom to Assemble Peaceably and
Without Arms”?
[R.A.S./R.T.S. (Pre) 2012]
[Jharkhand P.C.S. (Pre) 2013]
(a) Free and compulsory education for (a) Gherao officials not performing their
children upto the age of 14 years duties
(b) Organization and working of cooperative (b) Peaceful assembly
societies (c) People who assemble should not bear
(c) Stringent measures to deal with terrorism arms
(d) Provision of Lokpal to prevent corruption (d) State can make a law imposing reasonable
restrictions on the exercise of this Right
123. Which of the following is not correctly
matched? [Jharkhand P.C.S. (Pre) 2021] 127. Which of the following constitution
(Amendment) Act provides for goods and
Provisions Sources Services Tax (G.S.T.)?
(a) Article 22(1) Right to be defended by [Chhattisgarh P.C.S. (Pre) 2019]
a legal practitioner of her (a) The Constitutions (Ninety Eighth)
or his choice. Amendment Act 2012
(b) Article 22(4) No law providing (b) The Constitutions (Ninety Eighth)
preventive detention shall Amendment Act 2014
(c) The Constitutions (One-Hundredth and
authorize the detention
First) Amendment Act 2015
for a period longer than (d) The Constitutions (One Hundred and
three months. First) Amendment Act 2016
(c) Article 22(2) Parliament may by
128. What are the restrictions in the freedom to
law prescribe the
manage religious affairs in the Article-26 of
circumstances under
the constitution?
which a person may be
detained for a period (i) Public Order
longer than three (ii) National Security
months. (iii) Education
(d) Article 22(1) No person who is (iv) Morality
arrested shall be in (v) Health
custody without being
(vi) Secularism
informed, as soon as may
be, of the grounds for Code: [Chhattisgarh P.C.S. (Pre) 2018]
such an arrest. (a) (i), (ii), (iii)
(b) (ii), (iii), (v)
124. Under which of the following Article of (c) (ii), (iv), (vi)
thei Supreme Court is empowered to issue (d) (i), (iv), (v)
different ‘Writs’ for the enforcement of
Fundamental Rights of the citizens? 129. The rights are called Fundamental Rights
[Jharkhand PCS (Mains) 2016] because
(a) Article 32 i. It is written in the Constitution.
(b) Article 33 ii. It is democratic.

55 PYQ Workbook
INDIAN POLITY AND GOVERNANCE

iii. It is public welfare. 133. By which Constitutional Amendment OBCs


iv. It is essential for personality development. have been given 27 percent reservation in
the admission to educational institutions
v. Parliament can’t make law against it.
[Uttarakhand P.C.S. (Pre) 2012]
Code: [Chhattisgarh P.C.S. (Pre) 2017]
(a) 92nd
(a) i, ii, iii (b) 93rd
(b) i, iii, v (c) 94th
(c) i, iv, v (d) 96th
(d) ii, iii, v
(e) None of the Above 134. Which Article of the Constitution provides
protection to the accused regarding
130. What is correct about external emergency conviction from double-bar and self-
as per Article 352? incrimination?
i. It is proclaimed by the President. [Uttarakhand P.C.S. (Pre) 2010]
ii. It should be approved by Parliament within (a) Article 19
two months. (b) Article 22
iii. It should be approved by Parliament (c) Article 21
within one month. (d) Article 20
iv. It should be approved by Parliament by a 135. The Constitution of India does not
simple majority. guarantee which one of the following as a
v. It should be approved by Parliament by a fundamental right? [CAPF 2021]
2/3 majority. (a) Right to equality
vi. It can be revoked by a simple majority of (b) Right against exploitation
Parliament. (c) Right to property
(d) Right to carry on trade and business
vii. It can be revoked by a 2/3 majority of
Parliament. 136. Which one of the following statements with
viii. It can be approved in a joint session of regard to the Directive Principles of State
Policy is not correct? [CAPF 2021]
Parliament.
(a) They are not enforceable in a court of law.
Code: [Chhattisgarh P.C.S. (Pre) 2017]
(b) They can override all the fundamental
(a) i, ii, iv, viii rights.
(b) i, iii, vi, viii (c) They can be related to social and
(c) i, iii, v, vi economic justice.
(d) i, iv, vii, viii (d) They can be related to giving free legal
(e) None of the above aid.
131. By which of the following grounds, a 137. Which one of the following fundamental
citizen’s freedom of expression may not be rights has not been provided to a person?
subjected to restriction? [CAPF 2021]
[Chhattisgarh P.C.S (Pre) 2013] (a) Protection against prosecution and
(a) Sovereignty of India punishment for the same offence more
(b) Public order than once.
(c) Contempt of Court (b) To refuse to give his/her sample of
(d) Unbecoming criticism handwriting as evidence to support a
(e) All of the above prosecution against him/her.
132. The Provisions regarding division of taxes (c) To act as a witness against himself/herself.
between the Union and the States- (d) Right not to be convicted of any offence
except for violation of a law in force at the
[Chhattisgarh P.C.S. (Pre) 2011]
time of commission of the act charged as
(a) Can be suspended during Financial an offence.
Emergency
(b) Can be suspended during National 138. In relation to the fundamental duties
Emergency enshrined in the Constitution of India,
(c) Cannot be suspended under any which one of the following is NOT correct?
circumstances [CAPF 2020]
(d) Can be suspended only with the consent (a) The fundamental duties were not
of a majority of the State Legislatures originally a part of the Constitution

PYQ Workbook 56
INDIAN POLITY AND GOVERNANCE

(b) To safeguard public property and abjure (b) Anti-defection provisions of 52nd
violence are fundamental duties Amendment
(c) They are not enforceable through writs (c) Repealing of many of the provisions of
(d) They may not be used for determining 42nd Amendment by 44th Amendment
constitutionality of laws (d) 73rd Amendment that conferred extensive
139. “To cherish and follow the noble ideas the powers on Panchayat Bodies
which inspired our national struggle for 144. Which one of the following concerns has
freedom” is a provision of which one of the found a place in both Fundamental Duties
following of the Constitution of India? and Directive Principles of State Policy?
[CAPF 2019] [CAPF 2018]
(a) Directive Principles of the State Policy (a) Safeguard of Public Property
(b) Fundamental Duties (b) Protection of improvement of
(c) Fundamental Rights environment, forests and wildlife
(d) Preamble (c) Respect for the rich heritage of our
140. Which of the following is NOT provided composite culture
for under the Directive Principles of State (d) Promotion of cooperative societies
Policy? [CAPF 2019] 145. Which one of the following is not a part of
(a) Right to work, ‘to education and to public the Directive Principles of State Policy?
assistance in certain cases [CAPF 2018]
(b) Uniform Civil Code for the citizens
(c) Separation of Judiciary from the executive (a) Uniform Civil Code for all citizens
(d) Participation of workers in the (b) Separation of Judiciary from Executive
management of local-self government (c) Duty of the State to raise the level of
nutrition and standard of living
141. Which one of the following statements (d) Promotion of scientific temper, humanity
regarding the Directive Principles of State and the spirit of inquiry and reform
Policy is NOT correct? [CAPF 2019]
(a) State shall follow the Directive Principles 146. The power of the President of India to refer
of State Policy both in the matter of a matter back to the Council of Ministers
administration as well as in the making for reconsideration was inserted in the
of laws Constitution by [CAPF 2018]
(b) The Directive Principles of State Policy (a) 44th Amendment
embody the object of the State under the (b) 42nd Amendment
Republican Constitution (c) 43rd Amendment
(c) The Directive Principles of State Policy (d) 35th Amendment
have precedence over the Fundamental 147. The provision under Article-51A of the
Rights in case of conflict between the two Constitution of India relates to the
(d) The Directive Principles of State Policy
[CAPF 2016]
are not enforceable in the Courts.
(a) uniform civil code for the citizens
142. Sikkim became an Associate State of the (b) organization of village panchayats
Indian Union through [CAPF 2018] (c) right to education
(a) the Constitution (36th Amendment) Act, (d) fundamental duties
1975
(b) the Constitution (7th Amendment) Act, 148. The amending power of the Parliament set
1956 forth in which one of the following Articles
(c) the Constitution (35th Amendment) Act, of the Constitution of India? [CAPF 2016]
1974 (a) Article-368
(d) the Constitution (5th Amendment) Act, (b) Article-360
1955 (c) Article-13(2)
(d) Article-370
143. Which one of the following Constitutional
Amendments has enormously strengthened 149. With regard to a Constitution Amendment
the powers of the Speaker/Chairman Bill, which one of the following statements
of the Houses of the Parliament/State is not correct? [CAPF 2016]
Legislatures? [CAPF 2018] (a) The Speaker of the Lok Sabha can call a
(a) 61st Amendment which reduced the joint sitting of both the Houses to pass
voting age from 21 to 18 years the Bill

57 PYQ Workbook
INDIAN POLITY AND GOVERNANCE

(b) Each House needs to pass the Bill 3. Improvement of Public health and
separately by a prescribed special majority prohibition of intoxicating drinks.
end 4. Promotion of the educational and
(c) The Bill can be introduced in either economic interests of the weaker sections
House of Parliament of the people, especially the Scheduled
(d) The Bill can be sponsored by a Private Castes and Scheduled Tribes.
Member
Select the correct answer using the codes
150. Who among the following are entitled to get given below.
the benefits of equality before the law and Codes [CAPF 2014]
the equal protection of the laws as enshrined (a) 1 and 2
under Article-14 of the Constitution of (b) 2 and 4
India? (c) Only 3
1. All Indian citizens living in India. (d) 3 and 4
2. All Indian citizens living abroad.
153. Which of the following are related to Right
3. Foreigners living within the territory of
to Equality under the provisions of Articles
India.
14-18 of the Constitution of India?
4. All citizens born in India.
1. Prohibition of discrimination on grounds
Select the correct answer using the code given of religion, race, caste, sex or place of
below: [CAPF 2017] birth.
(a) 1 and 2 only 2. Equality of opportunity in matters of
(b) 1, 2 and 3 public employment.
(c) 1 and 3 only
3. Educational and cultural rights to
(d) Only 4
minorities.
151. Which of the following are provided in 4. Abolition of untouchability.
India by the Right to Education Act?
Select the correct answer using the codes
1. Right of children to free and compulsory given below.
education till completion of elementary
Codes: [CAPF 2014]
education in a neighborhood school.
2. Prohibition of deployment of teachers (a) 1, 2 and 3
for non-educational works, other (b) 3 and 4
than decennial census, elections to (c) 1, 2 and 4
local authority like State Legislatures, (d) 1 and 2
Parliament and disaster relief. 154. The major cleavage in global human rights
3. Right of minorities to establish and discourse has been between: [CAPF 2014]
administer education institution. (a) religious rights vs. civil rights
4. No citizen shall be denied admission into (b) national rights vs. minority rights
any educational institution maintained (c) gender rights vs. equal rights
by the State or receiving aid out of State (d) civil and political rights vs. economic,
funds on grounds only of religion, race, social and cultural rights
caste, language or any of them.
Select the correct answer using the code given 155. Which one of the following is not a part of
below. [CAPF 2015] Article 51 of the Constitution of India?
(a) 1, 2 and 4 [CDS 2023 (I)]
(b) 1 and 3 (a) Promote international peace and security
(c) 2 and 4 (b) Maintain just and honourable relations
(d) 1 and 2 between nations
(c) Use of UN peacekeeping force for
152. Which of the following does not relate to
resolution of international disputes
the Fundamental Rights as enshrined in the
Constitution of India? (d) Encourage settlement of inter- national
disputes by arbitration
1. Free and compulsory education for all
children of the age of 6-14 yr. 156. Which one of the following is not mentioned
2. Prohibition of trafficking in human as a form of Emergency in the Constitution
beings and forced labour. of India? [CDS 2022 (I)]

PYQ Workbook 58
INDIAN POLITY AND GOVERNANCE

(a) National Emergency 161. Which one of the following statements is


(b) State Emergency in terms of Proclamation correct? [CDS 2022 (II)]
of President’s Rule in a State (a) Under Article 21A of the Constitution
(c) Financial Emergency of India, the State shall provide free and
(d) Health Emergency compulsory education to all children up
157. Article 19(1) of the Constitution of India, to the age of eleven years.
(b) Every religious denomination has got
as it stands amended, includes which of the
absolute power under Article 26 of the
following?
Constitution of India to manage its own
1. Freedom of speech and expression affairs in matters of religion.
2. Assemble peaceably and without arms (c) Only religious and linguistic minorities
3. To acquire and dispose property find mention in Article 30 of the
4. To move freely throughout the territory Constitution of India in reference to the
of India right to establish educational institutions
Select the correct answer using the code given of their choice.
below: - [CDS 2022 (I)] (d) Parliament cannot empower, even by law,
any court other than the Supreme Court
(a) 1 and 2 only
of India the power to issue writs within
(b) 1, 2, 3 and 4
local limits of its jurisdiction.
(c) 4 only
(d) 1, 2 and 4 only 162. By which one of the following Constitutional
Amendment Acts, was the Article 21A
158. In Part-IVA of the Constitution of India,
(Right to Education) inserted into the
which one of the following is not mentioned
Constitution of India? [CDS 2022 (II)]
as a duty of every citizen of India?
(a) 83rd Amendment Act
CDS 2022 (I)] (b) 84th Amendment Act
(a) To render national service when called (c) 85th Amendment Act
upon to do so (d) 86th Amendment Act
(b) To renounce practices derogatory to the
dignity of women 163. Equality before the law or equal protection
(c) To value and preserve a unitary national of the laws within the territory of India
culture is guaranteed under which one of the
(d) To develop the spirit of inquiry and following Articles of the Constitution of
reform India? [CDS 2021(I)]
(a) Article 14
159. Which one of the following terms represents (b) Article 15
the maxim, ‘no man/person shall be (c) Article 1
condemned unheard’? [CDS 2022 (I)] (d) Article 22
(a) Autrefois acquit
(b) Non bis in idem 164. Which one among the following was
(c) Autrefois convict eliminated by the 44th Amendment Act of
(d) Audi Alteram Partem the Parliament? [CDS 2021(I)]
(a) Right against Exploitation
160. Which one of the following statements about
a Bill for Amendment of the Constitution (b) Right to Constitutional Remedies
of India is not correct? [CDS 2022 (I)] (c) Right to Property
(d) Right to Education
(a) It is governed by Article 368(2) of the
Constitution of India. 165. Which among the following fundamental
(b) Joint sitting can be resorted to for Passing rights is/are available to non-citizens?
a Bill amending the Constitution of India. 1. Freedom of speech
(c) The State Legislatures cannot initiate any 2. Protection against self-incrimination
Bill or proposal for amendment of the
3. Freedom of conscience
Constitution of India.
(d) The previous sanction of the President of 4. Non-discrimination in matters of
India is not required for introducing any employment
Bill in the Parliament for amendment of Select the correct answer using the codes
the Constitution of India. given below: [CDS 2021(I)]

59 PYQ Workbook
INDIAN POLITY AND GOVERNANCE

(a) 1, 2 and 3 only Select the correct answer using the codes
(b) 1, 3 and 4 only given below: - [CDS 2021 (II)]
(c) 2 and 3 only (a) Only 1
(d) 2 only (b) 1, 2 and 3
166. The provision, “the State shall take (c) 2 and 3
steps to separate the Judiciary from the (d) 1 and 2
Executive in the public services of the 170. Which one of the following is not a
State” is incorporated in which part of the circumstance for proclamation of
Constitution of India? [CDS 2021 (II)] Emergency by the President of India under
(a) Part-IV Article 352 of the Constitution of India?
(b) Part-V [CDS 2021 (II)]
(c) Part-VI (a) War
(d) Part-VII (b) External aggression
167. Which one of the following is the correct (c) Internal disturbance
meaning of the term ‘State’, as defined in (d) Armed rebellion
Article 12 of the Constitution of India? 171. The Constitution of India guarantees
[CDS 2021 (II)] freedom of speech and expression. But the
(a) It refers only to the Government of India. freedom is subject to certain reasonable
(b) It refers only to the Government of India restrictions imposed by the state. These
and the Parliament of India. restrictions may relate to which of the
(c) It refers only to the Government of India, following?
Parliament of India, and Governments 1. Defamation
and Legislatures of each State. 2. Decency or morality
(d) It refers to the Government of India, 3. Incitement to an offence
Parliament of India, Governments and Select the correct answer using the codes
Legislatures of each State, and all local or given below. [CDS 2021 (II)]
other authorities within the Territory of (a) Only 1
India. (b) 2 and 3
168. Which one of the following is not a correct (c) 1 and 3
statement with reference to the Constitution (d) 1, 2 and 3
of India? [CDS 2021 (II)] 172. Which of the following Articles in the
(a) The Constitution (Eighty-Fifth) Constitution of India are exceptions to the
Amendment Act has inserted the ‘Right Fundamental Right enumerated in Article
to Education under Article 21-A. 14 and Article-19? [CDS 2020 (II)]
(b) Article 22 talks about preventive (a) Article 31A and Article 31C
detention. (b) Article 31B and Article 31D
(c) Right to constitutional remedies is in Part (c) Article 1 and Article 13
– III of the constitution. (d) Article 16 and Article 17
(d) Writ jurisdiction of High courts is wider
than the Supreme Court of India. 173. Which one of the following was added as a
Fundamental duty through the Constitution
169. Which of the following statements regarding (86th Amendment) Act, 2002?
constitutional amendment is/are correct? [CDS 2020 (I)]
1. The procedure for amendment to the (a) To strive towards excellence in individual
Constitution is provided in Article 368. and collective activity
2. A Bill to amend the Constitution can (b) To provide opportunities for education to
be introduced in either House of the one’s child between the age of 6 and 14
Parliament. years
3. The special procedure in Article 368 vests (c) To work for the welfare of women and
constituent powers upon the ordinary children
legislation. (d) To promote peace and harmony

PYQ Workbook 60
INDIAN POLITY AND GOVERNANCE

174. Which one of the following Articles of the 179. Which of the following statements
Constitution of India protects a person regarding Article-21 of the Constitution of
against double jeopardy? [CDS 2020 (I)] India is/are correct?
(a) Article-20 1. Article 21 is violated when the under-
(b) Article-21 trial prisoners are detained under judicial
(c) Article-22 custody for an indefinite period.
(d) Article-23 2. Right to life is one of the basic human
rights and not even the State has the
175. Which one of the following Articles was authority to violate that right,
defended by Dr. BR Ambedkar on the plea 3. Under Article-21, the right of a woman
that it would be used as ‘a matter of last to make reproductive choices is not a
resort’? [CDS 2020 (I)] dimension of personal liberty
(a) Article-352 Select the correct answer using the codes
(b) Article-359 given below [CDS Pre. 2017 I]
(c) Article-356 (a) 1, 2 and 3
(d) Article 368 (b) 1 and 2
(c) 1 and 3
176. What is the ground on which the Supreme
(d) Only 2
Court can refuse relief under Article-32?
[CSDS 2020 (I)] 180. Which one of the following statements
regarding freedom to manage religious
(a) The aggrieved person can get remedy
affairs as per the Constitution of India is
from another court.
not correct? [CDS Pre. 2017 I]
(b) That disputed facts have to be investigated.
(a) Every religious denomination shall have
(c) That no Fundamental Right has been the right to manage its own affairs in
infringed. matters of religion except some minor
(d) That the petitioner has not asked for the communities.
proper writ applicable to his/her case. (b) Every religion or any section there of
177. Which one of the following Articles of the shall have the right to own and acquire
Constitution of India lays down that no movable and immovable property.
citizen can be denied the use of wells, tanks (c) Every religious community has the right
to establish and maintain institutions for
and bathing ghats maintained out of state
religious and charitable purposes
funds? [CDS Pre. 2019 I] (d) Every community has the right to manage
(a) Article-14 its own affairs in matters of religion.
(b) Article-15
(c) Article-16 181. Which one of the following statements
relating to cultural and educational rights
(d) Article-17
in India is not correct? [CDS Pre. 2017 I]
178. Which among the following statements (a) Every section of the citizens has the FRS
about the power to change the basic right to conserve its language. script or
structure of the Constitution of India is/are culture.
correct? (b) No citizen shall be denied admission into
1. It falls outside the scope of the amending any educational Institution maintained
powers of the Parliament. by the State or receiving aid out of State
2. It can be exercised by the people through funds on grounds of religion, race or
representative in a Constituent Assembly. language.
(c) The State shall, in granting aid educational
3. It falls within the constituent powers of
institutions, discriminate against any
the Parliament.
educational institution on the ground that
Select the correct answer using the codes it is under the management of a majority
given below: [CDS Pre.2019 I] community
(a) 1 and 3 (d) All minorities whether based on religion
(b) 1 and 2 or language, shall have the right to
(c) Only 1 establish and administer educational
(d) 2 and 3 institutions of their choice.

61 PYQ Workbook
INDIAN POLITY AND GOVERNANCE

182. The Fundamental Rights guaranteed in the 4. the judgement in Golaknath case (1967).
Constitution of India can be suspended Which of the statement(s) given above is/are
only by [CDS Pre.2017 II] correct? [CDS Pre.2016 II]
(a) a proclamation of National Emergency (a) 1, 2 and 3
(b) an Act passed by the Parliament (b) 2 and 4
(c) an amendment to the Constitution of (c) 1 and 3
India (d) All of these
(d) the judicial decisions of the Supreme
Court 187. Which one of the following statements is
not correct with respect to protection of
183. Article 21 of the Constitution of India individuals being tried for offences?
includes [CDS Pre.2016 II]
1. Rights of transgenders (a) A confession can never be used as
2. Rights of craniopagus twin evidence against the accused
3. Rights of mentally retarded women to (b) The accused must have violated an
bear a child existing law
Select the correct answer using the codes (c) An accused cannot be tried and punished
given [CDS Pre.2017 II] for the same offence again
(a) Only 1 (d) The quantum of punishment must be
(b) 1 and 2 provided in law as it existed on the date
(c) 2 and 3 of commission of an offence
(d) 1, 2 and 3 188. Which of the following Fundamental
184. Which one of the following statements Right(s) is/are available to non-citizens?
relating to protection against arrest and 1. Equality Before Law
detention of individuals under article 22 is 2. Right Against Discrimination
not correct? [ CDS Pre.2017 I] 3. Equality of Opportunity
(a) No person who is arrested shall be 4. Protection of Life and Personal Liberty
detained in custody without being Select the correct answer using the codes
informed of the grounds for such arrest given below : [CDS Pre. 2015 II]
(b) No person shall be denied the right (a) Only 1
to consult, and be defended by, a legal (b) 1 and 4
practitioner of his/her choice. (c) 1, 2 and 4
(c) Every person, who is arrested and (d) 2 and 3
detained in custody, shall be produced
before the nearest Magistrate within a 189. Which of the following is not true of
period of one week of such arrest. Article-32 of the Indian Constitution?
(d) The right to protection against arrest is [CDS 2015 II ]
not available to a person in jall pursuant (a) It gives the Supreme Court and the High
to a judicial order. Courts the power to issue writs for the
enforcement of Fundamental Rights.
185. The right to form association and unions is (b) It is included in Part III of the Indian
a right [CDS 2016 II] Constitution and is therefore itself a
(a) guaranteed to everybody Fundamental Right
(b) to freedom guaranteed to citizens only (c) Dr Ambedkar called it the ‘very soul of
(c) to equality before law the Indian Constitution.
(d) to life and personal liberty (d) An aggrieved person has no right to
186. The basic structure doctrine with regard to complain under Article-32 where a
the Constitution of India relates to Fundamental Right has not been violated.
1. the power of judicial review. 190. Freedom of conscience under the
2. the judgement in Kesavananda Bharati Constitution of India is subject to
case (1973). 1. public order, morality and health.
3. the constraints on Article-368 of the 2. a law providing for social welfare and
Constitution of India. reform.

PYQ Workbook 62
INDIAN POLITY AND GOVERNANCE

3. opening Hindu religious institutions of a (a) 1, 2 and 3


public character to all Hindus. (b) 2 and 3
4. defamation or incitement to an offence. (c) Only 1
Which of the statement(s) given above is/are (d) 1 and 2
correct? [CDS Pre.2015 II]
192. Which one of the following categories
(a) 1 and 2
of persons is not treated at par so far as
(b) 1.2 and 3
(c) 3 and 4 the availability of Fundamental Rights is
(d) 1 and 2 concerned? [CDS Pre.2015 I]
(a) Members of the armed forces
191. A writ of Habeas Corpus for the release of a
person can be issued: (b) Members of the forces charged with the
1. where the arrest or detention has taken responsibility of maintenance of public
place in contravention of the procedure order
established by law. (c) Members of the forces employed in
2. to secure the release of a person connection with the communications
imprisoned on a criminal charge. systems set-up in the country
3. where the arrest has taken place for (d) Members of the forces employed in
contempt of court or the Parliament connection with the communication
Select the correct answer using the codes systems set-up for maintenance of public
given below: [CDS Pre.2015 II] order.

63 PYQ Workbook
INDIAN POLITY AND GOVERNANCE

SOLUTIONS

running government’s majority in Lok Sabha undermine the


2.1. UPSC CSE Previous Years’ Questions Rajya Sabha in a joint sitting.
1. Solution: (a) 3. Solution: (c)
Exp) Option a is the correct answer Exp) Option c is the correct answer.
Option a is correct: The First Amendment Act, 1951, The nine –judge Constitutional Bench of the Supreme Court
added the fourth clause to Article 15 that empowered the in Justice K. S. Puttaswamy (Retd.) and Anr. vs Union Of India
government to make any law for the upliftment of socially And Ors (2017) has ruled that individual privacy is intrinsic
and educationally backward classes of citizens or for the to life and liberty and an inherent part of the fundamental
Scheduled Castes and Scheduled Tribes. It reads, “Nothing rights enshrined in the Article 21 of the Constitution.
in this article or in clause (2) of article 29 shall prevent the
State from making any special provision for the advancement Important Tipss
of any socially and educationally backward classes of citizens What are the various features of Right to Privacy?
or for the Scheduled Castes and the Scheduled Tribes.”. The • Privacy is a constitutionally protected right emerging
added clause elucidates that in case such special provisions primarily from the guarantee of life and liberty in
are introduced, they cannot be said to be breaching Article Article 21 of the Constitution.
15 and Article 29(2) of the Constitution. • It includes the preservation of personal intimacies,
The need to insert this clause was felt after the decision sanctity of family life, marriage, procreation, the home
of the Supreme Court in the State of Madras v. Srimathi and sexual orientation.
Champakam (1951). According to the facts of this case, the • Privacy connotes a right to be left alone. It safeguards
Madras government issued an Order that provided reservation individual autonomy and recognizes one’s ability to
on the grounds of religion, race, and caste. This Order was control vital aspects of his/her life.
contended to be in breach of Article 15(1) of the Indian
• Privacy is not an absolute right, but any invasion must
Constitution. The Court also gave a literal interpretation
be based on legality, need and proportionality.
to the constitutional provisions and held that reserving
seats in public institutions for backward classes violates • Informational privacy is a facet of this right. Dangers to
Articles 15(1) and 29(2). Therefore, in order to nullify the this can originate from both state and non-state actors.
effect of similar judicial pronouncements, Article 15 was 4. Solution: (a)
amended.
Exp) Option a is the correct answer.
Similarly, Article 19(1)(a) grants the right to free speech
Article 14 of Indian Constitution says that the State shall
and expression to Indian citizens. This right is considered
not deny to any person equality before the law or the
an essential feature of democracy. However, Article 19(2)
equal protection of the laws within the territory of India.
specifies the restrictions that can curtail this freedom. The
The concept of ‘equality before law’ is an element of the
First Amendment to the Indian Constitution altered these
concept of ‘Rule of Law’. This concept has the following three
restrictions by widening their ambit. The second change, via
elements or aspects:
the Amendment Act of 1951, was made to Clause 6 of Article
19. • Absence of arbitrary power
• Equality before the law, that is, equal subjection of all
2. Solution: (b) citizens to the ordinary law of the land
Exp) Option b is the correct answer • The constitution is the result of the rights of the individual
Statement 1 is incorrect: Constitution Amendment Bill can The first and the second elements are applicable to India.
be introduced either by a minister or by a private member This means ‘equality before law’ under Article 14 connotes
and does not require prior permission from the President. the absence of arbitrary power with the authorities. Thus,
Statement 2 is correct: The 24th Amendment of the Indian when a legislation confers uncontrolled discretionary
Constitution amended Article 368 to provide expressly powers on any authority, it tends to violate the article 14 of
that Parliament has the power to amend any provision of the the constitution.
Constitution. The amendment further made it obligatory
Important Tipss
for the President to give his assent when a Constitution
Amendment Bill was presented to him. Article 28 is related to the right to freedom of religion.
It provides that no religious instruction shall be provided
Statement 3 is correct: Article 368 of the Indian constitution in any educational institution wholly maintained out of
requires that the constitution of India can be amended State funds. Further, no person attending any educational
by both houses of parliament by a 2/3 majority(special institution recognised by the State or receiving aid out
majority). In case of disagreement between both houses, of State funds shall be required to attend any religious
there is no provision to summon a joint session of instruction or worship in that institution without his
consent.
parliament. The reasoning behind this is so as to not let the

PYQ Workbook 64
INDIAN POLITY AND GOVERNANCE

Article 32 confers the right to remedies for the enforcement Right to Equality (Article 14 -18) protects the citizens against
of the fundamental rights of an aggrieved citizen. Article any discrimination by the State on the basis of religion, caste,
32 affirms the right to move the Supreme Court if a race, sex, or place of birth.
fundamental right is violated. Under this article, the Under Article 17 of the Constitution of India, Untouchability
Supreme Court can issue writs for the enforcement of any
is abolished and its practice in any form is forbidden. The
of the fundamental rights of the citizens.
enforcement of any disability arising out of untouchability
Article 44 provides for one of the Directive Principles of
State Policy. Article 44 says that the “State shall endeavour shall be an offence punishable in accordance with law.
to secure for the citizens a uniform civil code throughout In 1976, the Untouchability (Offences) Act, 1955 has been
the territory of India”. comprehensively amended and renamed as the Protection of
Civil Rights Act, 1955 to enlarge the scope and make penal
5. Solution: (b)
provisions more stringent. The act defines civil right as
Exp) Option b is the correct answer. any right accruing to a person by reason of the abolition of
The concentration of wealth violates the Directive principles untouchability by Article 17 of the Constitution.
of state policy. Under Article 39 of the Constitution of India, However, the term ‘untouchability’ has not been defined
the State shall direct its policy towards securing that the either in the Constitution or in the Act.
operation of the economic system does not result in the
concentration of wealth and means of production to the 8. Solution: (d)
common detriment. Exp) Option d is the correct answer.
Other principles of policy to be followed by the State under Directive Principles of State Policy (DPSP) are enshrined in
the Article 39 are: Part-IV (Article 36 to 51) of the Constitution.
• The citizens, men and women equally, have the right to Statement 1 is incorrect and statement 2 is correct. Article
an adequate means to livelihood; 37 of the Constitution states that the provisions contained in
this Part (IV) shall not be enforceable by any court.
• The ownership and control of the material resources of
Statement 3 is correct. The principles therein laid down are
the community are so distributed as best to subserve the
nevertheless fundamental in the governance of the country
common good;
and it shall be the duty of the State to apply these principles
• There is equal pay for equal work for both men and in making laws. Hence, they impose a moral obligation on
women; the state authorities for their application.
• The health and strength of workers, men and women, Limitations of Part IV of the Indian Constitution:
and the tender age of children are not abused and that 1) No Legal Force: The DPSP are non-justiciable in nature
citizens are not forced by economic necessity to enter i.e. they are not legally enforceable by the courts for
avocations unsuited to their age or strength; their violation.
• Children are given opportunities and facilities to develop 2) Constitutional Conflict: DPSP leads to constitutional
in a healthy manner and in conditions of freedom and conflict between Centre and states, Centre and
dignity and that childhood and youth are protected President, Chief Minister and governor.
against exploitation and against moral and material 3) Conflict with Fundamental rights: They can be amended
abandonment to implement the fundamental rights.
6. Solution: (b) 4) Constitutional validity: A law cannot be struck down by
courts for violating DPSP.
Exp) Option b is the correct answer.
The Right to Property is a legal right available to any 9. Solution: (b)
person. Article 300-A provides that no person shall be Exp) Option b is the correct answer.
deprived of his property save by authority of law. Article 21 of the Constitution of India safeguards one’s right
Right to Property was a Fundamental Right in the original to marry the person of one’s choice.
constitution under Article 31, which provided that the In 2014, the Supreme Court took suo-motu cognizance of
Constitution not only guarantees the right of private newspaper reports of the gang rape of a 20-year-old Indian
ownership but also the right to enjoy and dispose of property woman on the orders of a village court. In this case, Supreme
free from restrictions other than reasonable restriction. It is Court held that “an inherent aspect of Article 21 of the
also mentioned that compensation would be paid to a person Constitution would be the freedom of choice in marriage”.
whose property has been taken for public purposes. This was reiterated by Supreme court in 2017. The choice of
The Constitution (Forty-fourth Amendment) Act, 1978 a partner whether within or outside marriage lies within the
deleted the right to property from the Fundamental exclusive domain of each individual. Intimacies of marriage
Rights. The Amendment omitted Article 31 and Article 19 lie within a core zone of privacy, which is inviolable. The
(1) (f ) and inserted in their place Article 300-A. absolute right of an individual to choose a life partner is not
in the least affected by matters of faith. The Constitution
7. Solution: (d) guarantees to each individual the right freely to practise,
Exp) Option d is the correct answer. profess and propagate religion.

65 PYQ Workbook
INDIAN POLITY AND GOVERNANCE

10. Solution: (d) 14. Solution: (b)


Exp) Option d is the correct answer Exp) Option b is the correct answer.
The term ‘liberty’ means the absence of restraints on the Statements 1 and 3 are incorrect. When President’s Rule
activities of individuals, and at the same time, providing is imposed in a state, the President dismisses Council of
opportunities for the development of individual personalities. Ministers in the State. Dissolution of the State Legislative
The Preamble of Constitution of India provides for liberty Assembly and local bodies is not necessarily a consequence
of thought, expression, belief, faith and worship. Thus, of the proclamation of the President’s rule in a State.
in context of polity, liberty provides for an opportunity to The President either suspends or dissolves the State
develop oneself fully. Legislative Assembly.
11. Solution: (c) The President acquires the following extraordinary powers
when the President’s Rule is imposed in a state:
Exp) Option c is the correct answer.
1) He can take up the functions of the state government
In Justice K. S. Puttaswamy (Retd) and others vs Union of
and powers vested in the governor or any other executive
India (2017), the Supreme Court observed that ‘Right to
authority in the state.
Privacy’ is an integral part of Right to Life and Personal
Liberty guaranteed in Article 21 of the Constitution. A nine- 2) He can declare that the powers of the state legislature are
judge bench of the Supreme Court declared that the right to be exercised by the Parliament
to privacy is a Fundamental Right protected under Part III 3) Therefore, when the President’s Rule is imposed in
of the Constitution of India. While primarily focused on a state, the President dismisses the state council of
the individual’s right against the State for violations of their ministers headed by the chief minister. The state
privacy, this landmark judgement had repercussions across governor, on behalf of the President, carries on the state
both State and non-State actors and is likely to result in the administration with the help of the chief secretary of the
enactment of a comprehensive law on privacy. state or the advisors appointed by the President. Hence,
Statement 2 is correct.
12. Solution: (b)
15. Solution: (c)
Exp) Option b is the correct answer.
John Locke said “Where there is no law, there is no freedom…” Exp) Option c is the correct answer.
Liberty is protected by law, ‘liberty’ means absence of The right to vote has been recognised under Article 326
restraints on the activities of individuals, and at the same of the Indian Constitution and thus it is a Constitutional
time, providing opportunities for the development of Right. India is a democratic country and right to vote and
individuals. Law promotes liberty. The more law there is the, to be elected is granted to every citizen of India under the
more liberty there is. And as soon as law ends, liberty also Constitution of India. Under Article 326 of the Constitution
ceases to exist. Where there are no laws to protect or provide of India it is provided that “The elections to the House of the
freedom and liberty, violations of the same take place. People and to the Legislative Assembly of every State shall be
Law protects liberty is in several ways. Firstly, the law helps on the basic of adult suffrage; that is to say every person who
in establishing a civilized society, thereby easing the way of is a citizen of India and who is not less than eighteen years of
life for individuals. This is done by the creation of a provision age and is not disqualified on the ground of non-residence,
for punishment to the criminals. Secondly, it guarantees unsoundness of mind, crime or corrupt or illegal practice
rights and duties to the citizens and individuals. The state shall be entitled to be registered as a voter”.
intervenes whenever the rights of an individual are violated Under Section 62 of the Representation of the People Act,
by another. Thirdly, the constitution acts as the custodian of 1951 it is provided that “every person who is, for the time
liberty. It is the constitution that provides the authority of being entered in the electoral roll of any constituency shall
the state and protects the fundamental rights of the people. be entitled to vote in that constituency”.
13. Solution: (d) In simple words “every Citizen of India who is not less than
eighteen years of age is entitled to get enrolled into electoral
Exp) Option d is the correct answer.
roll and is entitled to vote in the constituency in which his
Statement 1 is incorrect. There is no provision in name is entered in the electoral roll irrespective of caste,
the Constitution for direct enforcement of any of the creed, religion or gender unless he is disqualified under
Fundamental Duties nor for any sanction to prevent their the Constitution or any other law on the ground of non-
violation. All the Fundamental Duties under Article 51 A are residence, unsoundness of mind, crime or corrupt or illegal
‘non-justiciable’. practice.
Statement 2 is incorrect. There is no legislative process to
enforce citizens to do their fundamental duties. The legal 16. Solution: (c)
duty is one which is required to be carried out as per law. So, Exp) Option c is the correct answer.
there is no correlation between both duties. It has been held The Right against Exploitation under Articles 23 and 24
that these duties being duties of individual citizens cannot be of the Indian Constitution guarantees the dignity of the
enforced through mandamus. individual. Under these -

PYQ Workbook 66
INDIAN POLITY AND GOVERNANCE

Article 23 prohibits traffic in human beings, begar (forced has a corresponding duty. Therefore, there can be no duty
labour) and other similar forms of forced labour. Any unless there is someone to whom it is due. There can be
contravention of this provision shall be an offence punishable no right without a corresponding duty or a duty without a
in accordance with law. This right is available to both citizens corresponding right. Likewise, every right is right against
and non-citizens. It protects the individual not only against some person or persons upon whom a correlative duty is
the State but also against private persons imposed. Every right or duty involves a legal obligation by
Article 24 prohibits the employment of children below the which two or more persons are bound together.
age of 14 years in any factory, mine or other hazardous Under Indian Constitution, Part III deals with the
activities like construction work or railway. But it does not Fundamental Rights which are certain basic human rights
prohibit their employment in any harmless or innocent work. to its people. Our Constitution as originally enacted did not
Abolition of untouchability and protection of the interests of expressly lay down any Fundamental Duties to be performed
minorities are envisaged under Article 17 and Article 29 of by citizens. It was only in 1976 that a specific Chapter IV-A
the Constitution respectively. was incorporated in the Constitution by a constitutional
amendment and Article 51-A was enacted. The idea
17. Solution: (d)
behind the incorporation of the Fundamental Duties was
Exp) Option d is the correct answer. to emphasize the obligation of the citizen in exchange of
As Directive Principles of State Policy are not justiciable, they Fundamental Rights enjoyed by them.
do not constitute limitations upon legislative and executive
20. Solution: (b)
function of the state. The phrase ‘Directive Principles of
State Policy’ denotes the ideals that the State should keep Exp) Option b is the correct answer.
in mind while formulating policies and enacting laws. These Part IV of the Constitution of India (Article 36–51) contains
are the constitutional instructions or recommendations to the Directive Principles of State Policy (DPSP). These
the State in legislative, executive and administrative matters. principles aim at ensuring socio-economic justice to the
According to Article 36, the term ‘State’ in Part IV has the people and establishing India as a Welfare State. The Directive
same meaning as in Part III dealing with Fundamental Rights. Principles constitute a very comprehensive economic, social
Therefore, it includes the legislative and executive organs of and political programme for a modern democratic State.
the central and state governments, all local authorities and They aim at realising the high ideals of justice, liberty,
all other public authorities in the country. equality and fraternity as outlined in the Preamble to the
Constitution. They embody the concept of a ‘welfare state’
18. Solution: (b) and not that of a ‘police state’, which existed during the
Exp) Option b is the correct answer. colonial era. In brief, they seek to establish economic and
The Directive Principles of State Policy are enumerated in social democracy in the country.
Part IV of the Constitution from Articles 36 to 51.
21. Solution: (c)
The 42nd Amendment Act of 1976 added four new Directive
Exp) Option c is the correct answer.
Principles to the original list. They require the State:
Article 51A of the Constitution enlists Fundamental duties.
• To secure opportunities for healthy development of It shall be the duty of every citizen of India—
children (Article 39)
1) to abide by the Constitution and respect its ideals and
• To promote equal justice and to provide free legal aid to institutions, the National Flag and the National Anthem;
the poor (Article 39 A) 2) to cherish and follow the noble ideals which inspired
• To take steps to secure the participation of workers in the our national struggle for freedom;
management of industries (Article 43 A) 3) to uphold and protect the sovereignty, unity and
• To protect and improve the environment and to safeguard integrity of India;
forests and wild life (Article 48 A) 4) to defend the country and render national service when
The 44th Amendment Act of 1978 added one more Directive called upon to do so;
Principle, which requires the State to minimise inequalities 5) to promote harmony and the spirit of common
in income, status, facilities and opportunities (Article 38). brotherhood amongst all the people of India
The 86th Amendment Act of 2002 changed the subject-matter transcending religious, linguistic and regional or
of Article 45 and made elementary education a fundamental sectional diversities; to renounce practices derogatory
right under Article 21 A. to the dignity of women;
The 97th Amendment Act of 2011 added a new Directive 6) to value and preserve the rich heritage of our composite
Principle relating to cooperative societies culture;
7) to protect and improve the natural environment
19. Solution: (a) including forests, lakes, rivers and wild life, and to have
Exp) Option a is the correct answer. compassion for living creatures;
With reference to the Constitution of India, it can be said 8) to develop the scientific temper, humanism and the
that the Rights are correlative with Duties. Every right spirit of inquiry and reform;

67 PYQ Workbook
INDIAN POLITY AND GOVERNANCE

9) to safeguard public property and to abjure violence; • Indira Gandhi National Widow Pension Scheme
10) to strive towards excellence in all spheres of individual (IGNWPS);
and collective activity so that the nation constantly rises
• Indira Gandhi National Disability Pension Scheme
to higher levels of endeavour and achievement;
(IGNDPS);
11) who is a parent or guardian to provide opportunities
• National Family Benefit Scheme (NFBS);
for education to his child or, as the case may be, ward
between the age of six and fourteen years. • Annapurna
‘To protect the weaker sections from social injustice’ is not
24. Solution: (c)
listed as a Fundamental Duty.
Exp) Option c is the correct answer.
22. Solution: (b)
Directive Principles of State Policy and Fundamental
Exp) Option b is the correct answer
Rights are not divided on the basis of level of government -
The Constitution doesn’t differentiate between types
Union Government and State Governments. Both the Union
of Directive Principles of State Policy but for better
government and State governments shall protect fundamental
understanding of the terms, these can be classified into three
rights of the citizen and frame policies or laws enshrined
broad categories namely, Socialistic, Gandhian and Liberal-
intellectual. Gandhian Principles are based on the ideology in the DPSPs. Both are the part of the constitution. But
of Gandhi and strive to fulfil the dreams of Gandhi and to there is a fundamental difference between two. Directive
achieve a Gandhian State. They are reflected in following Principles are not enforceable whereas Fundamental Right
provisions - can be enforced. Therefore, the government (Central, state
Article 40: Steps will be taken by the states to organize village and local) cannot be compelled to implement them, unlike
panchayats and there will be decentralisation of power and Fundamental rights.
authority, in order to form self-government.
25. Solution: (b)
Article 43: State will ensure a living wage to industrial,
agriculture workers and ensure a decent workplace to work Exp) Option b is the correct answer.
and to promote cottage industries on an individual or co- Statement 1 is incorrect: Prohibition of Trafficking in
operative basis in rural areas. human beings and forced labour is given in Article 23 of the
23. Solution: (b) Fundamental Rights (not in DPSP).
Exp) Option b is the correct answer. Statement 2 is correct: According to Article 47 of the DPSP,
Directive Principles of State Policy fulfilled the National “The State shall also endeavour to bring about prohibition
Social Assistance Programme (NSAP) launched by the of the consumption except for medicinal purposes of
Government of India. The NSAP provides social assistance intoxicating drinks and of drugs which are injurious to
benefits to poor households in the case of old age, death health.”
of the breadwinner and maternity. The NSAP aims at
ensuring minimum national standards, in addition to the 26. Solution: (a)
benefits that the States are currently providing or might Exp) Option a is the correct answer
provide in future. DPSP also aims to establish social and According to Article 360(2) of the Indian Constitution, the
economic democracy through a welfare state. The ideas of
declaration of financial emergency must be ratified by both
Fundamental Rights and Duties are not explicitly related to
Houses of Parliament within two months of its issuance.
the welfare of the people.
A financial emergency, if approved by both Houses of
Important Tipss Parliament, lasts indefinitely until it is repealed.
National Social Assistance Programme (NSAP): After the Parliament approves the state of emergency, the
• It was launched on 15th August 1995. Union assumes complete control of the country’s financial
• NSAP is a Centrally Sponsored Scheme of the GoI that affairs. The Union Government may provide financial
provides financial assistance to the elderly, widows and directives to any of the states, and the President may ask the
persons with disabilities in the form of social pensions. states:
• The scheme represents a significant step towards the • To cut the pay and benefits of all or any class of
fulfilment of the Directive Principles in Article 41. government employees.
• Article 41 of the Constitution of India directs the State • To hold all money bills in reserve for consideration by
to provide public assistance to its citizens in case of
Parliament once they have been enacted by the State
unemployment, old age, sickness and disablement.
Legislature.
• Presently NSAP comprises of five schemes, namely –
• To reduce the salaries and allowances of central
• Indira Gandhi National Old Age Pension Scheme
government officials, including those of Supreme Court
(IGNOAPS);
and High Court judges.

PYQ Workbook 68
INDIAN POLITY AND GOVERNANCE

Important Tipss Important Tipss


• Article 360 empowers the president to proclaim a • According to Article 215 of Indian Constitution, every
Financial Emergency if he is satisfied that a situation High Court shall be a court of record and shall have
has arisen due to which the financial stability or credit
of India or any part of its territory is threatened. all the powers of such a court including the power to
• Once approved by both the Houses of Parliament, the punish for contempt of itself.
Financial Emergency continues indefinitely till it is • Article 275 provides for Grants from the Union to
revoked. This implies two things: certain States.
• There is no maximum period prescribed for its
operation; and • Article 325 of the Indian Constitution says that No
person to be ineligible for inclusion in, or to claim to
• Repeated parliamentary approval is not required
for its continuation. A resolution approving the be included in a special, electoral roll on grounds of
proclamation of financial emergency can be passed religion, race, caste or sex
by either House of Parliament only by a simple
majority 29. Solution: (d)
Exp) Option d is the correct answer.
27. Solution: (a)
The Directive Principles constitute a very comprehensive
Exp) Option a is the correct answer.
economic, social and political programme for a modern
Statement 1 is incorrect: Article 301 pertains to freedom democratic State. They aim at realising the high ideals of
of trade, commerce and intercourse within the territory of justice, liberty, equality and fraternity as outlined in the
India. It is not related to right to property. Preamble to the Constitution. They embody the concept of a
Statement 2 is correct: Right to property is a legal right but ‘welfare state’ and not that of a ‘police state’, which existed
not a fundamental right. It was originally a fundamental during the colonial era. In brief, they seek to establish
right under Article 19 (1) (f ) and Article 31, but these ‘economic and social democracy’ in the country.
articles were repealed by the 44th Amendment Act of 1978.
30. Solution: (a)
A new article, Article 300-A, was inserted in Part XII of
the Constitution, which provides that no person shall be Exp) Option a is the correct answer.
deprived of his property save by authority of law. Article 51 of the of the Directive Principles of State Policy
Statement 3 is incorrect: Article 300-A was inserted in the serves as a beacon, reflecting a sincere commitment to
Indian Constitution by the 44th Amendment Act of 1978 maintain world peace and security by avoiding disputes and
during the period of Janta Party Government. The Janta wars for the sake of the entire human species.
Party came to power in 1977 after defeating the Congress
Important Tipss
Party led by Indira Gandhi. The Janta Party government
introduced several constitutional amendments to undo Under Article 51, the State shall endeavour to
the changes made by the previous government during the • promote international peace and security;
Emergency period of 1975-77. • maintain just and honourable relations between nations
Important Tipss • foster respect for international law and treaty
obligations,
• The right to property as a fundamental right was
abolished by the 44th Amendment Act in 1978, which • encourage settlement of international disputes by
added Article 300A as a new provision in Part XII of arbitration
the constitution.
31. Solution: (d)
• The reason for this amendment was to facilitate land
Exp) Option d is the correct answer.
reforms and social justice by allowing the state to
acquire surplus land from landowners and distribute it One of the key provisions of the 44th Amendment is the
to the landless farmers. empowerment of the President to send back any matter for
reconsideration by the Council of Ministers. This means
• The amendment also removed the obligation of
that if the President is not satisfied with a decision taken
the state to pay compensation for acquiring private
by the Council of Ministers, he/she can send it back for
property, although the Supreme Court has held that
reconsideration.
compensation can be inferred from Article 300A
Important Tipss
28. Solution: (d)
Features of 44th Constitutional Amendment Act:
Exp) Option d is the correct answer
• It restored the term of the Lok Sabha and the state
According to Article 355 of Indian Constitution, it shall be
legislative assemblies from six to five years.
the duty of the Union to protect every State against external
aggression and internal disturbance and to ensure that the • It deleted the right to property from the list of
government of every State is carried on in accordance with fundamental rights and made it a legal right under
the provisions of this Constitution. Article 300A.

69 PYQ Workbook
INDIAN POLITY AND GOVERNANCE

• It empowered the President to send back any matter • Right to Freedom (Article 19 – 22): This right protects
for reconsideration by the Council of Ministers once, six freedoms regarding speech, expression, assembly,
but if the same advice is given again, he/she has to association, movement, residence, and profession. It
accept it. also provides for protection against arbitrary arrest
• It reduced the period of operation of a proclamation and detention, and safeguards against conviction for
of national emergency from one year to six months. offences in certain cases.
• It provided that a proclamation of emergency can • Right against Exploitation (Article 23 – 24): This
be issued only on the basis of written advice by the right prohibits trafficking of human beings and forced
Cabinet and not by individual ministers. labour, and bans employment of children under 14
years in hazardous occupations.
• It restored the power of judicial review to the Supreme
Court and the High Courts which was curtailed by the • Right to Freedom of Religion (Article 25 – 28): This
42nd Amendment. right grant freedom of conscience and free profession,
practice and propagation of religion, freedom to
• It provided safeguards against the misuse of Article
manage religious affairs, freedom from payment of
356 (President’s rule) by laying down some conditions
taxes for promotion of any religion, and freedom from
for its invocation and continuation
attending religious instruction or worship in certain
32. Solution: (b) educational institutions.
• Cultural and Educational Rights (Article 29 – 30):
Exp) Option b is the correct answer.
This right protects the interests of minorities by
The 93rd Amendment of the Constitution of India, preserving their right to conserve their language, script
officially known as The Constitution (Ninety-third and culture, and granting them the right to establish
Amendment) Act, 2005, added clause (5) in Article 15 and administer educational institutions of their choice.
which enables the government to make any special provision • Right to Constitutional Remedies (Article 32): This
for the advancement of any socially and educationally right empowers the Supreme Court and the High Courts
to issue writs for the enforcement of Fundamental
backward classes of citizens or for the Scheduled Castes or
Rights. It also gives the right to move the Supreme Court
the Scheduled Tribes in so far as such special provisions
directly for violation of Fundamental Rights.
relate to their admission to educational institutions,
including private educational institutions, whether aided 34. Solution: (a)
or unaided by the State, other than the minority educational Exp) Option a is the correct answer.
institutions. A. Article 16 (2) - 2. No person can be discriminated against
in the matter of public appointment on the grounds of
33. Solution: (c) race, religion or caste
Exp) Option c is the correct answer. B. Article 29 (2) - 4. No citizen shall be denied admission
into any educational institution maintained by the State,
The right to equality in the Indian Constitution is granted
or receiving State aid, on the grounds of religion, race,
by five Articles, which are:
caste, language or any of them
Article 14: Right to equality before law C. Article 30 (1) - 3. All minorities whether based on
Article 15: Prohibition of discrimination on grounds of religion or language shall have the right to establish and
religion, race, caste, sex, or place of birth administer educational institutions of their choice
D. Article 31 (1) - 1. No person shall be deprived of his
Article 16: Equality of opportunity in matters of public
property by the authority of law
employment
Article 17: Abolition of untouchability 35. Solution: (a)
Exp) Option a is the correct answer.
Article 18: Abolition of titles
A British citizen staying in India cannot claim right to:
Important Tipss Freedom of trade and profession (Article 19 (1) (g)) - This
The six Fundamental Rights under Indian Constitution right allows citizens of India to practice any profession, or to
are: carry on any occupation, trade or business. It is one of the six
• Right to Equality (Article 14 – 18): This right freedoms guaranteed under Article 19, which are available
provides for equality before law and equal protection only to the citizens of India.
of laws, prohibition of discrimination on grounds of The rights that are available to a British citizen staying in
religion, race, caste, sex or place of birth, equality of India are:
opportunity in matters of public employment, abolition • Equality before the law (Article 14) - This right ensures
of untouchability and titles. that the State shall not deny to any person equality before

PYQ Workbook 70
INDIAN POLITY AND GOVERNANCE

the law or the equal protection of the laws within the


territory of India. It is available to all persons, whether • Article 29: Protection • Article 26: Freedom
citizens or foreigners. of language, script and to manage religious
• Protection of life and personal liberty (Article 21) - This culture of minorities. affairs.
right guarantees that no person shall be deprived of his Article 21A: Right to • Article 27: Freedom
life or personal liberty except according to procedure elementary education. from payment of taxes
established by law. It is available to all persons, whether • Article 30: Right for promotion of any
citizens or foreigners. of minorities to religion.
• Freedom of religion (Article 25-28) - These rights establish and • Article 28: Freedom
ensure that all persons are equally entitled to freedom administer educational from attending religious
of conscience and the right to freely profess, practice institutions. instruction or worship
and propagate religion. They also protect the rights of in certain educational
religious minorities and prohibit religious instruction in institutions.
State-funded educational institutions. They are available
to all persons, whether citizens or foreigners 36. Solution: (d)
Important Tips Exp) Option d is the correct answer.
Fundamental Rights Fundamental rights available An amendment to the Constitution of India can be initiated
available only to citizens to both citizens & foreigners by either the Lok Sabha (1) or the Rajya Sabha (2). Both
and not to foreigners (except enemy aliens) Houses of Parliament have the power to propose and pass the
Constitution Amendment Bills.
• Article 15: Prohibition • Article 14: Equality
of discrimination on before law and equal Important Tipss
grounds of religion, protection of laws. • An amendment of the Constitution may be initiated
race, caste, sex or place • Article 20: Protection only by the introduction of a bill for the purpose
of birth.Article 14: in respect of conviction in either House of Parliament and not in the state
Equality before law for offences. legislature.
and equal protection of • Article 21: Protection • The bill can be introduced either by a minister or by a
laws. of life and personal private member and does not require prior permission
• Article 16: Equality of liberty. of the President.
opportunity in matters • Article 21A: Right to • The bill must be passed by both Houses by a special
of public employment. elementary education. majority of not less than two-thirds of members
Article 20: Protection • Article 22: Protection present and voting and more than half of the total
in respect of conviction against arrest and membership of each House.
for offences. detention in certain • The President must give his assent to the bill. He has
• Article 15: Prohibition cases. no power to withhold or return the bill.
of discrimination on • Article 23: Prohibition
grounds of religion, of traffic and human 37. Solution: (a)
race, caste, sex or place beings and forced Exp) Option a is the correct answer.
of birth.Article 14: labour.
Equality before law Article 368 of the Constitution of India states that some
• Article 24: Prohibition amendments also require the ratification by not less than
and equal protection of
of employment of
laws. half of the states by resolution passed by their legislatures
children in factories
• Article 16: Equality of before they are presented to the President for assent.
etc.
opportunity in matters Therefore, the matters that require consent of at least half
• Article 25: Freedom
of public employment. of the state legislatures for constitutional amendment are:
Article 20: Protection of conscience and free
profession, practice 1. Election of the President. This is true, as it involves
in respect of conviction
for offences. and propagation of changes in Article 54 and Article 55.
• Article 19: Protection religion. 2. Representation of the States in Parliament. This is true,
of six rights regarding as it involves changes in the representation of States in
freedom of (speech Parliament.
and expression, 3. Any of the Lists in the 7th Schedule. This is true, as
(ii) assembly (iii)
it involves changes in any of the Lists in the Seventh
association, (iv)
Schedule.
movement (v) residence
and profession.Article 4. Abolition of the Legislature Council of a State. This is
21: Protection of life false, as it does not require the ratification of at least one-
and personal liberty. half of the state legislatures. Parliament can abolish or
create a Legislative Council in a State only if the State
Legislative Assembly passes a resolution with a special

71 PYQ Workbook
INDIAN POLITY AND GOVERNANCE

majority. The law passed by the Parliament for this • The judiciary has interpreted Article 15 in a broad and
purpose shall also amend the Constitution accordingly liberal manner to protect the rights of the marginalized
and shall not be considered as an amendment under and vulnerable sections of society. For example, the
article 368. Supreme Court has held that sexual harassment of
women at workplace is a violation of Article 15. The
38. Solution: (d) Supreme Court has also held that reservation within
Exp) Option d is the correct answer. reservation based on area or region is permissible
• The 13th Amendment of the Indian Constitution gave the under Article 15(4) to address the regional disparities
among the backward classes.
status of a state to Nagaland and made special provisions
for it. • The judiciary has also balanced the implementation of
Article 15 with the principle of equality under Article
• The 18th Amendment of the Indian Constitution 14. For example, the Supreme Court has held that
amended Article 3 to include Union territories in the reservation cannot exceed 50% of the total seats or
definition of ‘state’ and to empower Parliament to form a posts, except in extraordinary circumstances.
new State or Union territory by uniting a part of any State
40. Solution: (a)
or Union territory to any other State or Union territory.
Exp) Option a is the correct answer.
• The 39th Amendment of the Indian Constitution placed
the election of the President, the Vice President, the The basic structure doctrine is a judicial principle
established by the Supreme Court of India. According to
Prime Minister and the Speaker of the Lok Sabha beyond
this doctrine, certain features or elements of the Indian
the scrutiny of the Indian courts.
Constitution are considered so fundamental and essential
• The 52nd Amendment of the Indian Constitution added that they form the basic structure of the Constitution.
the Tenth Schedule to provide for the disqualification of These features cannot be abrogated or altered even through
the members of Parliament and the state legislatures on the process of Constitutional amendment under Article 368.
the grounds of defection The Supreme Court has held that while Parliament
has the power to amend the Constitution, it cannot use
39. Solution: (d)
this power to destroy or alter the basic structure of the
Exp) Option d is the correct answer. Constitution. The Court has not provided an exhaustive
Article 15 of the Constitution of India forbids discrimination list of elements that constitute the basic structure, but some
on grounds only of religion, race, caste, sex, or place of of the features considered as part of the basic structure
birth. It applies the general principle of equality in specific include the supremacy of the Constitution, the rule of law,
situations by forbidding classifications made on protected the independence of the judiciary, the secular nature of the
grounds. It also empowers the state to make special provisions state, federalism, and the basic rights guaranteed to citizens.
for the advancement of any socially and educationally Important Tipss
backward classes of citizens or for the Scheduled Castes and The basic structure doctrine has been invoked by
the Scheduled Tribes. The prohibition of discrimination the Supreme Court in several cases to strike down
on the ground of religion etc. is a fundamental right that is constitutional amendments that were found to be violative
classifiable under the Right to Equality. of the basic structure. Some examples are:
• Indira Nehru Gandhi v Raj Narain (1975), where clause
Important Tipss (4) of article 329A, which immunized the election of
Article 15 of the Constitution of India has been Prime Minister from judicial scrutiny, was struck down
implemented in various ways by the government and the as it violated democracy and rule of law.
judiciary to ensure equality and non-discrimination for • Minerva Mills v Union of India (1980), where clauses
all citizens. Some of the ways are: (4) and (5) of article 368, which gave unlimited
• The government has enacted laws such as the Hindu amending power to parliament and excluded judicial
Succession Act, 1956 to remove discrimination based review, were struck down as they destroyed harmony
on gender and caste in matters of inheritance and between fundamental rights and directive principles.
social status. • S.R. Bommai v Union of India (1994), where arbitrary
• The government has also made special provisions for use of article 356, which empowers President to impose
women, children, socially and educationally backward President’s rule in states, was held to be violative of
classes, scheduled castes and scheduled tribes under federalism and democracy.
Article 15(3), (4) and (5) to promote their advancement • I.R. Coelho v State of Tamil Nadu (2007), where it was
and welfare. For example, the government has provided held that laws placed in Ninth Schedule after April 24,
reservation of seats in educational institutions and 1973 (the date of Kesavananda Bharati judgement) are
public employment for these groups. open to judicial review if they violate basic structure.

PYQ Workbook 72
INDIAN POLITY AND GOVERNANCE

41. Solution: (b) 44. Solution: (c)


Exp) Option b is the correct answer. Exp) Option c is the correct answer.
Freedom of newspapers in India is a part of the freedom Keshvanand Bharti vs State of Kerala case was decided in 1973
of speech and expression, which is guaranteed by Article and it was the first time that the Supreme Court propounded the
19 (1) (a) of the Indian Constitution. This article states that concept of ‘Basic Structure of the Constitution’. The court held that
“All citizens shall have the right to freedom of speech and the Parliament has the power to amend any part of the Constitution,
expression.” including the fundamental rights, but it cannot alter or destroy the
basic structure or essential features of the Constitution.
Freedom of newspapers in India is not explicitly
mentioned or specially provided in Article 19 (1) (a) of 45. Solution: (c)
the Constitution. It is implied or derived from the wider Exp) Option c is the correct answer.
freedom of expression guaranteed by this article. The
The promotion of Co-operative Societies is given in Article
Supreme Court has interpreted this article to include the
43B (not Article 43A). This article reads “The State shall
freedom of press as an integral part of the freedom of speech
endeavour to promote voluntary formation, autonomous
and expression functioning, democratic control, and professional
2.2. Other Examination Previous Years’ management of cooperative societies”. Whereas, Article
43A of the Constitution of India mentions about the
Questions
participation of workers in management of industries.
42. Solution: (c)
46. Solution: (c)
Exp) Option c is the correct answer.
Exp) Option c is the correct answer.
To protect monuments and places of national importance
The 61st Amendment of the Constitution of India, officially
is not a fundamental duty under the Indian Constitution. known as The Constitution (Sixty-first Amendment) Act,
It is a part of the Directive Principles of State Policy 1988, lowered the voting age of elections to the Lok Sabha
(DPSP) under the Indian Constitution. Article 49 of the and to the Legislative Assemblies of States from 21 years to
Constitution says that it shall be the obligation of the State 18 years.
to protect every monument or place or object of artistic The amendment aimed to provide an opportunity to the
or historic interest, declared by or under law made by unrepresented youth of the country to participate in the
Parliament, to be of national importance, from spoliation, political process and express their views. It also brought
disfigurement, destruction, removal, disposal or export, as India in line with many other countries that had specified 18
the case may be. years as the voting age.

43. Solution: (d) 47. Solution: (a)


Exp) Option d is the correct answer. Exp) Option a is the correct answer.
The 86th Amendment of the Constitution of India, officially The correct chronological order is as follows:
known as The Constitution (Eighty-sixth Amendment) Act, 2002, • Golak Nath Case: 1967. This case held that the Parliament
inserted Article 21A in the Constitution which makes education cannot amend the fundamental rights enshrined in Part
a fundamental right for all children between the age of 6 and 14 III of the Constitution.
years. • 24th Constitution Amendment Act: 1971. This act was
Important Tipss enacted to override the Golak Nath case and restore the
Features of 86th Constitutional Amendment Act: Parliament’s power to amend any part of the Constitution,
including Part III.
• It inserted Article 21A in the Constitution which
• Keshavanand Bharti Case: 1973. This case upheld the
makes education a fundamental right for all children
validity of the 24th Amendment Act, but also introduced
between the age of 6 and 14 years and requires the
the doctrine of basic structure, which limits the
state to provide free and compulsory education to them
Parliament’s amending power to preserve the essential
• It amended Article 45 of the Constitution, which
features of the Constitution.
deals with the directive principle of state policy
• 42nd Constitution Amendment Act: 1976. This act was
on providing free and compulsory education to all
enacted during the Emergency period and made several
children up to the age of 14 years, and changed it to
changes to the Constitution, such as expanding the scope
provide for early childhood care and education for all
of Article 31C, adding new Directive Principles and
children until they complete the age of six years.
Fundamental Duties, and changing the preamble.
• It inserted a new clause (k) in Article 51A of the
Constitution, which makes it a fundamental duty of 48. Solution: (d)
every citizen who is a parent or guardian to provide Exp) Option d is the correct answer.
opportunities for education to his/her child or ward Article 43A deals with Participation of workers in
between the age of 6 and 14 years management of industries. It mentions that ‘The State shall

73 PYQ Workbook
INDIAN POLITY AND GOVERNANCE

take steps, by suitable legislation or in any other way, to Gandhian Principles


secure the participation of workers in the management of
• These principles are based on Gandhian ideology. They
undertakings, establishments or other organisations engaged
represent the programme of reconstruction enunciated
in any industry’. This article was inserted in the Constitution
by Gandhi during the national movement. In order to
by the 42nd Amendment 1976.
fulfil the dreams of Gandhi, some of his ideas were
49. Solution: (b) included as Directive Principles. They require the
Exp) Option b is the correct answer. State: To organise village panchayats and endow them
with necessary powers and authority to enable them
Assertion (A) states that Article 30 of the Constitution of to function as units of self-government, to promote
India does not define the term ‘minorities’. This statement cottage industries on an individual or cooperation
is true. Article 30 of the Indian Constitution provides certain basis in rural areas, etc.
rights to minorities to establish and administer educational
Liberal-Intellectual Principles
institutions. However, it does not specifically define the term
‘minorities’. • The principles included in this category represent the
ideology of liberalism. They direct the state: To secure
Reason (R) states that the Constitution recognizes only
for all citizens a uniform civil code throughout the
linguistic and religious minorities. This statement is also country, to provide early childhood care and education
true. The Constitution of India recognizes two types of for all children until they complete the age of six years,
minorities - linguistic minorities and religious minorities. It to organise agriculture and animal husbandry on
ensures certain protections and rights to these minorities to modern and scientific lines, and to protect and improve
preserve and promote their distinct language and culture or the environment and to safeguard forests and wild life.
to manage their religious institutions. But, it does state the
reason as to why Constitution of India does not define the 52. Solution: (c)
term. Exp) Option c is the correct answer.

50. Solution: (c) • According to 38th Constitutional Amendment Act,


any declarations made by the President declaring a state
Exp) Option c is the correct answer.
of emergency are no longer subject to legal review
Article 48A under DPSP lays down the principle or judicial review under the 38th Amendment Act.
for protection and improvement of environment and According to this rule, the President may restrict a citizen’s
safeguarding of forests and wildlife. It reads as: “The State fundamental rights depending on the circumstances
shall endeavor to protect and improve the environment surrounding the declaration of an emergency.
and to safeguard the forests and wildlife of the country.”
• According to 44th Amendment to the Indian
Similarly, according to Article 51A (g), it shall be the duty
Constitution, Right to Property (Article 31) was
of every citizen of India to protect and improve the natural
removed as a fundamental right and reclassified
environment including forests, lakes, rivers and wildlife and
it as a legal right. It also added Article 300-A to the
to have compassion for living creatures. Hence, Provision
Constitution, which deals with the right to compensation
related to the protection of the environment is described
in case of acquisition of property.
in both - as Fundamental Duty and as DPSP.
• The 61st Amendment to the Indian Constitution was
51. Solution: (d) passed in 1989. It lowered the voting age from 21 to 18
Exp) Option d is the correct answer. years, thereby allowing young adults aged 18 to 21 years
On the basis of content and direction given in Directive to exercise their right to vote in elections.
principles of State Policy, they can be classified into three • The 42nd Amendment of the Indian Constitution,
broad categories, viz, socialistic, Gandhian and liberal- enacted in 1976, added the words ‘socialist’, ‘secular’
intellectual. However, the Constitution does not contain any and ‘integrity’ to the preamble. It also made several
classification of Directive Principles. other changes to the Constitution that affected the basic
structure, fundamental rights, fundamental duties,
Important Tipss
emergency provisions, etc6
Socialistic Principles
• These principles reflect the ideology of socialism. They 53. Solution: (d)
lay down the framework of a democratic socialist state, Exp) Option d is the correct answer.
aim at providing social and economic justice, and set Capital punishment does not fall within the purview of
the path towards welfare state. They direct the state: Article 21 as it is not related to the right to life and personal
To Minimise inequalities in income, status, facilities liberty. Capital punishment means the death penalty
and opportunities, to provide opportunities for healthy awarded by a court of law for certain heinous crimes. Capital
development of children, to ensure equal pay for equal punishment is not a violation of Article 21 as it is imposed
work for men and women, etc. according to the procedure established by law.

PYQ Workbook 74
INDIAN POLITY AND GOVERNANCE

• Medical aid to injure by a doctor: The Supreme Court • Equality before law does not mean that all laws must be
has held that the right to medical aid is a part of the right general in character and universal in application. Equal
to life under Article 21. It means that every person has the protection of laws does not mean that all laws must be
right to receive timely and adequate medical treatment in alike for all people without any distinction.
case of injury or illness. • Equality before law deals with equal subjection of all
• Sexual Harassment of Women at workplace: The persons to the ordinary law of the land administered
Supreme Court has held that the right to work with by ordinary law courts. Equal protection of laws deals
dignity is a part of the right to life under Article 21. It with similar application of the same laws to all persons
means that every woman has the right to work in a safe who are similarly situated
and respectful environment free from any form of sexual
55. Solution: (a)
harassment or abuse.
Exp) Option a is the correct answer.
• Pollution of the quality of water: The Supreme Court
The ‘Right to Freedom’ in the Indian Constitution is granted
has held that the right to clean environment is a part
by four articles, which are Article 19 to Article 22.
of the right to life under Article 21. It means that every
• Article 19 guarantees six freedoms to citizens, which
person has the right to enjoy a healthy and pollution-free
are freedom of speech and expression, freedom to
environment, including water, air, and land.
assemble peacefully and without arms, freedom to
54. Solution: (d) form associations or unions, freedom to move freely
throughout the territory of India, freedom to reside and
Exp) Option d is the correct answer.
settle in any part of the country, and freedom to practice
Assertion (A) is correct: State can treat unequal differently any profession, occupation, trade, or business.
with the objective of creating a level playing field in the
• Article 20 provides protection against ex post facto laws
social, economic and political spheres. This is based on the
and double jeopardy. It also ensures that no person can
principle of equal protection of laws, which means that the be compelled to be a witness against himself/herself.
law gives equal opportunities to all people who are in similar
• Article 21 guarantees the right to life and personal liberty.
circumstances or situations. This principle allows the State
It ensures that no person shall be deprived of their life
to make reasonable classifications among people and provide
or personal liberty except according to the procedure
special provisions or reservations for the advancement established by law.
of any backward class of citizens, which is not adequately
• Article 22 provides protection against arrest and
represented in the services under the State. This is done
detention in certain cases. It lays down provisions for
to ensure substantive equality and not merely formal
the protection of individuals arrested or detained under
equality. preventive detention laws.
Reason (R) is correct statement: Among equals the law
56. Solution: (a)
should be equal and equally administered. This is based on
the principle of equality before law, which means that no one Exp) Option a is the correct answer.
is above the rule of law. This principle implies that all people Assertion (A) states that the state shall provide free and
are subject to the same laws and the same judicial system, compulsory education to all children of the age group of
regardless of their status or power. six to fourteen years. This statement is true. The Right to
Education (RTE) Act, enacted in India in 2009, mandates
Reason (R) is also the correct explanation of Assertion (A)
free and compulsory education for children in the specified
because it implies that the State can treat unequal differently
age group.
only with the objective of creating equality among them.
Reason (R) states that in a democratic society, the right
The State cannot discriminate among equals or favour any
to education is indispensable in the interpretation of the
group without a reasonable basis. The special provisions for
right to development as a human right. This statement
the backward classes and the scheduled castes and tribes
is also true. Education is considered a fundamental right
are based on the reasonable classification of these groups
and an essential aspect of human development. It enables
as being under-represented and deserving of preferential individuals to exercise their other rights effectively and
treatment to achieve substantive equality. contributes to their overall well-being and progress.
Important Tipss 57. Solution: (a)
• Equality before law is a negative concept that restricts Exp) Option a is the correct answer.
the State from engaging in any arbitrary discrimination
Assertion (A) states that Dr. Ambedkar described Article
between individuals.
32 of the Constitution as the very soul of it. This statement
• Equal protection of laws is a positive concept that is true. Dr. B.R. Ambedkar, the principal architect of the
puts a positive obligation on the State to prevent the Indian Constitution, indeed emphasized the significance of
violation of rights. Article 32 as a fundamental right itself and as a means to

75 PYQ Workbook
INDIAN POLITY AND GOVERNANCE

protect and enforce other fundamental rights. He considered Important Tipss


it an essential provision for the effective functioning of the
The fundamental duties are listed in Article 51A of the
Constitution. Indian Constitution and they are as follows:
Reason (R) states that Article 32 provides an effective • To abide by the Constitution and respect its ideals
remedy against the violation of fundamental rights. This and institutions, the National Flag and the National
statement is also true. Article 32 of the Indian Constitution Anthem
empowers individuals to directly approach the Supreme • To cherish and follow the noble ideals which inspired
Court for the enforcement of their fundamental rights. our national struggle for freedom
It provides a significant safeguard and remedy to protect • To uphold and protect the sovereignty, unity, and
citizens’ fundamental rights by allowing them to seek judicial integrity of India
intervention against any violation. • To defend the country and render national service
when called upon to do so
The Reason (R) provides the correct explanation for
• To promote harmony and the spirit of common
Assertion (A) as it explains why Dr. Ambedkar considered brotherhood amongst all the people of India
Article 32 as the very soul of the Constitution. Article 32 transcending religious, linguistic and regional or
plays a crucial role in upholding and protecting fundamental sectional diversities; to renounce practices derogatory
rights and serves as a mechanism for ensuring their to the dignity of women
enforcement. • To value and preserve the rich heritage of our composite
culture
58. Solution: (d) • To protect and improve the natural environment
Exp) Option d is the correct answer. including forests, lakes, rivers and wildlife, and to have
compassion for living creatures
Article 31 of the Constitution of India was a fundamental
• To develop the scientific temper, humanism and the
right that guaranteed the right to property to all citizens.
spirit of inquiry and reform
It provided that no person shall be deprived of his property
• To safeguard public property and to abjure violence
except by authority of law, and that no property shall be
• To strive towards excellence in all spheres of individual
compulsorily acquired or requisitioned except for a public
and collective activity so that the nation constantly
purpose and on payment of just compensation. However, rises to higher levels of endeavour and achievement
Article 31 was repealed by the Constitution (Forty-fourth
• To provide opportunities for education to his child
Amendment) Act, 1978, which removed the right to property or, as the case may be, ward between the age of six
from the list of fundamental rights and inserted it as a legal and fourteen years (This duty was added by the 86th
right under Article 300A Constitutional Amendment Act, 2002)
The right of minorities to establish and administer 61. Solution: (a)
educational institutions is not guaranteed by Article 31,
but by Article 30. Article 30 is a fundamental right that Exp) Option a is the correct answer.
protects the cultural and educational rights of minorities. Assertion (A) states that under Article 368 of the Indian
Constitution, the Parliament can amend any part of the
59. Solution: (b)
Constitution. This is correct as Article 368 grants the power
Exp) Option b is the correct answer. of amendment to the Parliament.
Article 51A of the Indian Constitution lists the fundamental Reason (R) states that the Parliament is the Supreme
duties of every citizen of India. These fundamental duties Legislative body elected by the People of India. This is also
are the basic norms to be observed by the citizens towards
correct as the Parliament is the highest legislative body
their State. Fundamental Duties are mentioned under Part
in India and is elected by the people through democratic
IVA of the Indian Constitution. They were added through
elections.
42nd Amendment 1976 upon the recommendations of
Swaran Singh Committee to pay respect towards the Nation. Reason (R) is the correct explanation of Assertion
(A) because the fact that the Parliament is the supreme
60. Solution: (c) legislative body elected by the people of India gives it the
Exp) Option c is the correct answer. authority to amend the Constitution through Article 368.
To help in organizing Village Panchayats is not a part The Constitution vests this power in the Parliament as a
of the fundamental duties under the Indian Constitution. reflection of the democratic principles and representative
It is a part of the Directive Principles of State Policy nature of the Indian polity.
(DPSP) under the Indian Constitution. Article 40 of the
Important Tipss
Constitution says that the State shall take steps to organize
village panchayats and endow them with such powers and • Article 368 is a provision in the Constitution of India
authority as may be necessary to enable them to function as that deals with the power of Parliament to amend the
units of self-government Constitution and its procedure.

PYQ Workbook 76
INDIAN POLITY AND GOVERNANCE

• It is in Part XX of the Constitution and states • Article 347: It empowers the President to direct
that Parliament can amend any provision of the any State to recognize any language spoken by a
Constitution by way of addition, variation or repeal, substantial proportion of the population of that State
subject to some conditions and limitations. as an official language of that State
• Some amendments require the ratification of not
63. Solution: (c)
less than one half of the States by resolution passed
by their Legislatures before they are presented to the Exp) Option c is the correct answer.
President for assent. The phrase “due process of law” is not included in any
• Article 368 also declares that nothing in Article 13, of the articles of the Indian Constitution. However, the
which prohibits laws that abridge or take away any of concept has been enshrined in Article 21 of the Indian
the fundamental rights, shall apply to any amendment Constitution, which states that “No person shall be deprived
made under this article. of his life or personal liberty except according to the
procedure established by law”.
62. Solution: (c)
Over time, the Supreme Court of India (Maneka Gandhi vs
Exp) Option c is the correct answer. Union of India case (1978)) has interpreted Article 21 to
Article 25 of the Constitution of India is a fundamental include the doctrine of “due process of law”, which means
that not only there must be a law to deprive a person of his
right that guarantees the freedom of conscience and the
life or personal liberty, but also that the law must be fair, just,
right to freely profess, practice, and propagate religion to
and reasonable. The Supreme Court has held that Article
all persons. 21’s reference to “procedure established by law” has been
Explanation II to Article 25 (2) (b) states that “In sub- understood as “due process of law”
clause (b) of clause (2), the reference to Hindus shall be
64. Solution: (d)
construed as including a reference to persons professing
Exp) Option d is the correct answer
the Sikh, Jaina or Buddhist religion, and the reference to
National Emergency can only be declared on grounds
Hindu religious institutions shall be construed accordingly.”
of “External aggression or war”, also called as External
Therefore, among the given options, only Parsis are Emergency & on the ground of “armed rebellion”. Article
not included in the word ‘Hindu’ in Article 25 of the 352 of the Indian Constitution allows the president to
Constitution of India. declare a national emergency. Only after obtaining a written
recommendation from the cabinet may the president declare
Important Tipss a national emergency
• The Constitution does not define the term “minority”
but it recognizes religious and linguistic minorities 65. Solution: (c)
under Article 30(1). Exp) Option c is the correct answer
• The Constitution also provides various rights and The President’s Rule under Article 356 of the Constitution
safeguards for the protection and promotion of the of India was for the first time imposed in the year 1951 in
interests of minorities. Some of them are: the former state of Punjab and East Punjab States Union
• Article 29: It grants any section of citizens having (PEPSU).
a distinct language, script or culture the right to
Important Tipss
conserve the same and the right to be protected against
• Article 355 imposes a duty on the Centre to ensure
discrimination on any ground.
that the government of every state is carried on in
• Article 30: It grants any religious or linguistic accordance with the provisions of the Constitution.
minority the right to establish and administer • It is this duty in the performance of which the Centre
educational institutions of their choice and the right takes over the government of a state under Article 356
to receive aid from the State without discrimination. in case of failure of constitutional machinery in state.
• Article 350A: It imposes a duty on every State and • This is popularly known as ‘President’s Rule’. It is
local authority to provide adequate facilities for also known as ‘State Emergency’ or ‘Constitutional
instruction in the mother tongue at the primary Emergency’.
stage of education to children belonging to linguistic • The President’s Rule can be proclaimed under Article
minority groups. 356 on two grounds–one mentioned in Article 356
itself and another in Article 365:
• Article 350B: It provides for a Special Officer for
• Article 356 empowers the President to issue a
Linguistic Minorities appointed by the President to proclamation, if he is satisfied that a situation has
investigate all matters relating to the safeguards for arisen in which the government of a state cannot be
linguistic minorities and to report to the President carried on in accordance with the provisions of the
upon those matters. Constitution.

77 PYQ Workbook
INDIAN POLITY AND GOVERNANCE

• Article 365 says that whenever a state fails to Important Tipss


comply with or to give effect to any direction from Article 16 of the Indian Constitution has six clauses.
the Centre, it will be lawful for the president to hold They are:
that a situation has arisen in which the government
• Clause (1): Equal opportunity for all citizens in public
of the state cannot be carried on in accordance with
employment or office.
the provisions of the Constitution
• Clause (2): No discrimination on grounds of religion,
66. Solution: (a) race, caste, sex, descent, place of birth, residence or any
of them in public employment or office.
Exp) Option a is the correct answer.
• Clause (3): Parliament can make laws on residence
The Nehru Report was a memorandum drafted by a requirements for certain public posts within a State or
committee chaired by Motilal Nehru, with Jawaharlal Nehru Union territory.
acting as the secretary, to appeal for a new dominion status • Clause (4): State can make provisions for reservation
and a federal set-up of government for the constitution of appointments or posts for backward classes who are
of India5. It also proposed for the joint electorates with under-represented in the services under the State.
reservation of seats for minorities in the legislatures. The • Clause (4A): State can make provisions for reservation
report demanded inalienable fundamental rights for the in promotion with consequential seniority for
people of India, such as equal rights for women, right to scheduled castes and scheduled tribes who are under-
form unions, and universal adult suffrage. represented in the services under the State.
Option b is incorrect: The Government of India Act, 1935 • Clause (4B): State can treat unfilled reserved vacancies
did not refer to Fundamental Rights. It only provided for of a year as a separate class of vacancies to be filled up
in any succeeding year or years without exceeding the
provincial autonomy and a federal structure of government,
50% reservation limit.
but it did not guarantee any civil liberties or rights to the
people of India. 68. Solution: (a)
Option c is incorrect: The August Offer, 1940, did not include Exp) Option a is the correct answer.
the Fundamental Rights. It was a proposal made by the British
Fundamental duties are not part of the fundamental rights.
government to the Indian leaders to seek their cooperation during the
The fundamental rights are the basic rights guaranteed to the
Second World War. It offered some concessions such as expansion of
citizens and the aliens by the Indian Constitution under Part III.
the executive council, establishment of a constituent assembly after
the war, and recognition of the principle of federation. • They are enforceable by the courts and can be suspended
during an emergency.
Option d is incorrect: The Cripps Mission 1942 did not
refer to Fundamental Rights. It was an attempt by the • The fundamental duties are the moral obligations of the
British government to negotiate with the Indian leaders and citizens to promote the spirit of patriotism and preserve
persuade them to support the British war effort. It offered India’s unity.
some proposals such as granting dominion status to India • They are listed in Article 51A under Part IV-A of the
after the war, setting up a constituent assembly to frame a Indian Constitution.
new constitution
• They are not enforceable by any court and cannot be
67. Solution: (b) suspended at any time
Exp) Option b is the correct answer. 69. Solution: (c)
Article 16 of the Indian Constitution is a fundamental right Exp) Option c is the correct answer.
that guarantees equality of opportunity for all citizens in
matters relating to employment or appointment to any First Constitution Amendment Bill passed in 1951 was
office under the State. It prohibits discrimination on the related to several changes to the Fundamental Rights
grounds of religion, race, caste, sex, descent, place of birth, provisions of the Indian constitution, including the
residence or any of them. validation of zamindari abolition laws in certain States.
The amendment sought to secure the constitutional validity
• Article 16 (1) states the general principle of equality
of these laws, which were challenged in courts by landowners
of opportunity for all citizens in matters relating to
on the ground of being discriminatory or violating the right
employment or appointment to any office under the State.
to property.
• Article 16 (2) specifies the grounds on which
discrimination is prohibited, namely religion, race, caste, Important Tipss
sex, descent, place of birth, residence or any of them. • The First Constitution Amendment added a new clause
• Article 16 (1) and 16 (2) together guarantee equality of (4) to Article 31, which stated that no law made by the
opportunity to all citizens of India in matters relating to State for the acquisition of any estate or of any rights
public employment therein or for the extinguishment or modification
• Article 16 (3) deals with residence requirement, Article of any such rights shall be deemed to be void on
16 (4) deals with reservation for backward classes, and the ground that it is inconsistent with or takes away
Article 16 (5) deals with religious or denominational or abridges any of the rights conferred by Article 14,
institutions. Article 19 or Article 31

PYQ Workbook 78
INDIAN POLITY AND GOVERNANCE

70. Solution: (b) • Article 31 deals with the right to property, which
Exp) Option b is the correct answer. was later deleted from the list of fundamental rights
The 69th Amendment of the Constitution of India, officially by the 44th Amendment in 1978.
known as The Constitution (Sixty-ninth Amendment) Act, • The 25th Amendment was enacted to give primacy to
1991, designated the Union Territory of Delhi as the National the Directive Principles over the Fundamental Rights
Capital Territory of Delhi and provided for a legislative in matters of social and economic justice.
assembly and a council of ministers for Delhi.
72. Solution: (d)
Important Tipss
Exp) Option d is the correct answer.
Features of 69th Constitutional Amendment Act:
Economic equality is not mentioned in any of the articles
• It conferred a special status to Delhi by declaring related to the right to equality. Economic equality is more
the Union Territory of Delhi as the National Capital
related to the directive principles of state policy, which are
Territory of Delhi and provided for a legislative
assembly and a council of ministers for Delhi. not enforceable by courts but are guidelines for the State to
follow while making policies and laws
• It inserted two new articles, Article 239AA and
Article 239AB, in the Constitution, which deal with • Equality before law: Article 14 ensures that all
the special provisions for Delhi as the National Capital individuals are equal before the law and are entitled to
Territory and the provisions in case of failure of equal protection of the laws.
constitutional machiner.
• Social equality: Article 15 prohibits discrimination on
• It designated the administrator of Delhi appointed
the grounds of religion, race, caste, sex, or place of birth
under Article 239 as the Lieutenant Governor and
made him/her bound by the aid and advice of the and promotes social equality.
council of ministers except in matters where he/she is • Equal opportunity: Article 16 guarantees equality of
required to act in his/her discretion. opportunity in matters of public employment.
• It conferred on the legislative assembly of Delhi the
power to make laws on all matters in the state list 73. Solution: (b)
and the concurrent list except public order, police Exp) Option b is the correct answer.
and land.
A national emergency is a situation of crisis that threatens
• It provided that the laws made by Parliament and the security, unity, and integrity of the country. It can be
by the legislative assembly of Delhi shall have effect
declared by the President of India under Article 352 of the
concurrently subject to certain conditions.
Constitution on the grounds of war, external aggression, or
71. Solution: (a) armed rebellion.
Exp) Option a is the correct answer. According to Article 359, when a national emergency is
The 25th Amendment of the Constitution, enacted in declared, the President can suspend the right to move any
1971, provides that no law passed to give effect to Directive court for the enforcement of any of the fundamental rights
except Articles 20 and 21.
Principles of State Policy contained in Articles 39 (b) & (c)
shall be deemed to be void on the ground that it abridges the Important Tipss
rights conferred by Articles 14 and 19. • Article 20: Protection in respect of conviction for
• Article 39 (b) & (c) deal with the principles of offences. It prohibits ex post facto laws, double
securing equitable distribution of material resources jeopardy, and self-incrimination.
and preventing concentration of wealth and means of • Article 21: Protection of life and personal liberty.
production. It states that no person shall be deprived of his life
• Article 14 guarantees equality before law and equal or personal liberty except according to procedure
protection of laws. established by law.
• Article 19 guarantees six freedoms, such as freedom of 74. Solution: (b)
speech and expression, freedom of assembly, freedom of
Exp) Option b is the correct answer
association, freedom of movement, freedom of residence
and settlement, and freedom of profession. Article 359 of the Constitution authorizes the President
of India to suspend the right to move any court for the
Important Tipss enforcement of Fundamental Rights during National
• The 25th Amendment also added a new clause (4) to Emergency except Article 20 and Article 21.
Article 31, which states that no law giving effect to the
Important Tipss
policy of securing the principles specified in Article 39
(b) or (c) shall be called in question in any court on • Article 358 deals with regulations that give rights to
the ground that it does not provide for compensation the state government to suspend the Fundamental
or that it provides for inadequate compensation for Rights during war or external aggression but not
acquisition or requisition of property. armed rebellion.

79 PYQ Workbook
INDIAN POLITY AND GOVERNANCE

• Article 13 deals with the laws inconsistent with or in Important Tipss


derogation of fundamental rights. • An Ordinary Bill is concerned with any matter other
• Under Article 356 of the Constitution of India, if than financial matters. It can be introduced in either
a state government is unable to function according House of Parliament by any member with a seven-day
to Constitutional provisions, the Union government notice. It has to be passed by both Houses by a simple
can take direct control of the state machinery. majority of members present and voting.
• A Money Bill is concerned with financial matters
75. Solution: (d) like taxation, expenditure, borrowing, etc. It can be
Exp) Option d is the correct answer. introduced only in the Lok Sabha by a minister with the
The 79th Amendment of the Constitution of India, officially prior permission of the President. It has to be passed by
known as The Constitution (Seventy-ninth Amendment) the Lok Sabha by a simple majority of members present
Act, 1999, extended the period of reservation of seats for and voting.
the Scheduled Castes and Scheduled Tribes in the Lok • A Financial Bill is also concerned with financial
Sabha and the State Legislative Assemblies for another ten matters. It has to be passed by both Houses by a simple
years, i.e., up to 26 January 2010. majority of members present and voting.
This was done by amending Article 334 of the Constitution, 78. Solution: (b)
which provides for the reservation of seats for the SCs and
Exp) Option b is the correct answer.
STs and the representation of the Anglo-Indian community
by nomination in the Lok Sabha and the State Legislative Right to education is included in the right to life enshrined
Assemblies. in Article 21 of the Constitution of India, which provides
the right to live with dignity. The Supreme Court of India has
76. Solution: (a) interpreted Article 21 to include the right to education as a
Exp) Option a is the correct answer. part of the right to life and personal liberty.
• The 42nd Amendment Act of 1976 added a new Part Right to education is also a fundamental right for children
IV-A to the Constitution which consists of Article 51-A, of age 6 to 14 years under Article 21A of the Constitution
enlisting ten fundamental duties of the citizens. of India, which was inserted by the Constitution (Eighty-
sixth Amendment) Act, 2002. It provides that “The state
• The Minerva Mills Case of 1980 was a landmark judgment
shall provide free and compulsory education to all children
of the Supreme Court that upheld the doctrine of basic
of age of 6 to 14 years in such manner as the state may, by
structure and struck down two clauses of the 42nd
law, determine.”
Amendment Act that gave power to Parliament to amend
any part of the Constitution, including the fundamental 79. Solution: (a)
rights. Exp) Option a is the correct answer.
• The Kesavananda Bharti Case of 1973 was a historic Article 19 (1) (d) of the Constitution of India guarantees
verdict of the Supreme Court that propounded the the right to freedom of movement to all citizens. It means
doctrine of basic structure for the first time. It held that every citizen has the right to move freely throughout
that Parliament has the power to amend any part of the the territory of India, without any unreasonable restriction
Constitution, but it cannot alter or destroy the basic or or interference by the State. This right is essential for the
essential features or framework of the Constitution. development of personality, social and cultural interaction,
• Article 23 of the Constitution prohibits trafficking in and national integration.
human beings and other forms of forced labour, and Article 21 of the Constitution of India guarantees the right
declares them as offences punishable by law. It also to life and personal liberty to all persons. It means that no
empowers Parliament to enact laws to prevent such evils person shall be deprived of his life or personal liberty except
and provide for their punishment. according to the procedure established by law. This right is
the most fundamental of all rights and includes various other
77. Solution: (d) rights that are necessary for a dignified human existence.
Exp) Option d is the correct answer. The Supreme Court of India (Maneka Gandhi v. Union of
The Constitution Amendment Bill must be passed by each India, 1978) has held that the right to travel abroad is a part
House of the Indian Parliament separately by a special of the right to freedom of movement under Article 19 (1) (d)
majority. A special majority means that the bill must be as well as the right to life and personal liberty under Article
approved by a majority of the total membership of each 21 of the Constitution.
House and also by a majority of not less than two-thirds of
80. Solution: (d)
the members present and voting in each House. This special
majority requirement ensures that any amendment to the Exp) Option d is the correct answer.
Indian Constitution is subjected to a rigorous and broad- Directive Principles of State Policy aim to create social and
based process of consideration and approval. economic conditions under which the citizens can lead a

PYQ Workbook 80
INDIAN POLITY AND GOVERNANCE

good life. They also aim to establish social and economic 82. Solution: (a)
democracy through a welfare state. The idea of welfare Exp) Option a is the correct answer.
states arises from the philosophy of compassion towards the
The Bill will have to be dropped. This is because a
weaker sections. For example, the government provides food
Constitutional Amendment Bill requires the approval of
grains to people at a very subsidised rate under the National
Food Security Act, 2013. both Houses of Parliament by a special majority of not
less than two-thirds of the members present and voting.
Important Tipss Therefore, if the Upper House rejects such a Bill, it cannot
Key features of DPSPs which describes the model of be passed by a joint sitting or by a re-passage by the Lower
Welfare State: House. The Bill will have to be dropped or reintroduced with
• To Minimise inequalities in income, status, facilities changes acceptable to both Houses.
and opportunities (Art. 38).
83. Solution: (a)
• To provide opportunities for healthy development of
children (Art. 38). Exp) Option a is the correct answer.
• To ensure equal pay for equal work for men and women Fundamental duties are not enforceable by any court of law.
(Art. 39A). They are only the moral obligations of the citizens and there
• To secure the right to work, to education and to public is no legal sanction for their violation or non-compliance.
assistance in cases of unemployment, old age, sickness Option b is correct: They can be promoted only by
and disablement (Art. 41). constitutional methods: This means that the citizens should
• To make provision for just and humane conditions of respect the constitutional provisions and procedures while
work and maternity relief (Art. 42). performing their duties and not resort to any unlawful or
• To secure a living wage, a decent standard of life and violent means.
social and cultural opportunities for all workers (Art. Option c is correct: They can be used for interpreting
43). ambiguous statutes: This means that the courts can use
• To provide early childhood care and education for all the fundamental duties as a guide or a tool to resolve any
children until they complete the age of six years (Art. ambiguity or doubt in the meaning or scope of any law or
45). statute.
• To promote the educational and economic interests of Option d is correct: The performance of any duty comes
SCs, STs, and other weaker sections of the society and within the sphere of constitutional law which court has to
to protect them from social injustice and exploitation
decide: This means that the courts have the power and
(Art. 46).
authority to adjudicate any matter or dispute related to the
• To raise the level of nutrition and the standard of living performance or non-performance of any duty by the citizens
of people and to improve public health (Art. 47).
or the state.
81. Solution: (a) Important Tipss
Exp) Option a is the correct answer. Features of Fundamental Duties of India:
The provisions of the Constitution which are related to the • Some of them are moral duties while others are
federal structure of the polity can be amended by a special civic duties. For instance, cherishing noble ideals of
majority of the Parliament and also with the consent of half freedom struggle is a moral precept and respecting the
of the state legislatures by a simple majority. Constitution, National Flag and National Anthem is a
The following provisions can be amended in this way: civic duty.
1. Election of the President and its manner. • They refer to such values which have been a part of the
2. Extent of the executive power of the Union and the states. Indian tradition, mythology, religions and practices. In
other words, they essentially contain just a codification
3. Supreme Court and high courts.
of tasks integral to the Indian way of life.
4. Distribution of legislative powers between the Union and
• Unlike some of the Fundamental Rights which extend
the states.
to all persons whether citizens or foreigners, the
5. Any of the lists in the Seventh Schedule. Fundamental Duties are confined to citizens only and
6. Representation of states in Parliament. do not extend to foreigners
7. Power of Parliament to amend the Constitution and its • Like the Directive Principles, the fundamental duties
procedure (Article 368 itself ). are also nonjusticiable. The Constitution does not
Statement 4 is incorrect: Formation of new States or provide for their direct enforcement by the courts.
transformation in the borders and names of the states can Moreover, there is no legal sanction against their
be amended by a simple majority of the two Houses of violation. However, Parliament is free to enforce them
Parliament outside the scope of Article 368. with suitable legislation.

81 PYQ Workbook
INDIAN POLITY AND GOVERNANCE

• Like the Directive Principles, the fundamental duties held that the Parliament cannot amend the fundamental
are also nonjusticiable. The Constitution does not rights enshrined in Part III of the Constitution and that any
provide for their direct enforcement by the courts. amendment made in contravention of this principle would
Moreover, there is no legal sanction against their be void.
violation. However, Parliament is free to enforce them Important Tipss
with suitable legislation.
• Golaknath Vs. The State of Punjab case overruled
84. Solution: (d) the earlier decisions of A.K. Gopalan Vs. The State of
Exp) Option d is the correct answer. Madras (1950) and Sajjan Singh Vs. State of Rajasthan
(1965), which upheld the unlimited amending power
The rights that are not available to all persons in India are:
of Parliament.
• Right against discrimination (Article 15) - This right
• However, the Golaknath case was itself overruled by
prohibits discrimination on grounds of religion, race,
Keshavananda Bharati Vs. The State of Kerala (1973),
caste, sex or place of birth. It is available only to the which introduced the doctrine of basic structure, which
citizens of India. limits the Parliament’s amending power to preserve the
• Freedom to move freely throughout the country (Article essential features of the Constitution.
19 (1) (d)) - This right allows citizens to move freely • The Minerva Mills Vs. The Union of India (1980)
throughout the territory of India. It is one of the six case applied the basic structure doctrine and struck
freedoms guaranteed under Article 19, which are down some provisions of the 42nd Amendment Act,
available only to the citizens of India. 1976, which sought to curtail the judicial review of
• Right to contest election - This right is not explicitly constitutional amendments
mentioned in the Constitution, but it is implied
from Article 84 and Article 173, which prescribe the 87. Solution: (d)
qualifications for being a member of Parliament and Exp) Option d is the correct answer.
State Legislature respectively. One of the qualifications is Supreme Court of India has deduced a fundamental right
that the person must be a citizen of India to equal pay for equal work from the word ‘socialist’ used
85. Solution: (c) in the Preamble to the Constitution as well as from Article
14, 15 and 16 of the Constitution.
Exp) Option c is the correct answer.
Equal pay for equal work is not a constitutional right
Option 1 is correct: According to Article 47 of the DPSP,
or a fundamental right. It can be described through the
“The State shall also endeavour to bring about prohibition
of the consumption except for medicinal purposes of interpretations of Article 14, 15 and 16 which guarantee
intoxicating drinks and of drugs which are injurious to fundamental rights of equality before law, protection against
health.” any kind of discrimination and equal opportunities in the
matters of public employment.
Option 2 is incorrect: Prohibition of employment of
children in factories or mines is a Fundamental Right The word ‘socialist’ used in the Preamble to the Constitution
(Not DPSP) under article 24. Article 24 says that “No child also enabled the Supreme Court of India to deduce a
below the age of fourteen years shall be employed to work fundamental right to equal pay for equal work. The Preamble
in any factory or mine or engaged in any other hazardous declares India to be a sovereign, socialist, secular, democratic
employment.” republic. The term ‘socialist’ implies social and economic
justice for all citizens. It also implies that the State shall
Option 3 is incorrect: Prohibition of beggar or forced labour
is a Fundamental Right (Not DPSP) under article 23. strive to reduce inequalities in income, status, facilities and
Article 23 prohibits the trafficking of human beings, begar, opportunities among individuals and groups
and any similar form of forced labor. It also states that any 88. Solution: (a)
contravention of this provision is punishable by the law.
Exp) Option a is the correct answer.
Option 4 is incorrect: Prohibition of untouchability is a
Statement 1 is correct: Fundamental Rights are enshrined in
Fundamental Right (Not DPSP) under article 17. It reads
that “Untouchability is abolished and its practice in any form Part III of the Constitution of India and are enforceable by
is forbidden. The enforcement of any disability arising out of the courts, subject to certain restrictions. They protect the
Untouchability shall be an offence punishable in accordance citizens from arbitrary or discriminatory state action that
with law.” violates or abridges their rights.
Statement 2 is correct: Fundamental Rights are listed in
86. Solution: (b) Articles 12 to 35 of Part III of the Constitution of India.
Exp) Option b is the correct answer. There are six Fundamental Rights, namely, Right to Equality,
Golaknath Vs. The State of Punjab case was decided in 1967 Right to Freedom, right against Exploitation, Right to
and it was the first time that the Supreme Court limited the Freedom of Religion, Cultural and Educational Rights, and
power of Parliament to amend the Constitution. The court Right to Constitutional Remedies.

PYQ Workbook 82
INDIAN POLITY AND GOVERNANCE

Statement 3 is incorrect: Fundamental Rights are more integrity, and security of India, friendly relations with
concerned with individual rights and liberties that protect foreign states, public order, or morality under Article 19
the citizens from arbitrary state action or discrimination. (5) of the Constitution.
They do not directly ensure social, economic and political • Right to Equal Pay for Equal Work: The Supreme Court
justice, which is more of a goal of the Directive Principles has held that the right to equal pay for equal work is
of State Policy (Part IV of the Constitution) a part of the right to equality under Article 14 and 16
Statement 4 is incorrect: Fundamental Rights in India are of the Constitution. It means that every citizen has the
heavily inspired by the Bill of Rights in the U.S.A., which right to receive equal remuneration for work of equal
is the first ten amendments to the U.S. Constitution that value without any discrimination on the grounds of
guarantee civil liberties to the people. The Fundamental sex, caste, religion, etc . The Supreme Court has also
Rights in India have their origins in many sources, including held that this right is a constitutional goal that is to be
England’s Bill of Rights, the United States Bill of Rights and attained by the State in accordance with Article 39 (d) of
France’s Declaration of the Rights of Man. the Constitution, which is a directive principle of state
policy.
Important Tipss
90. Solution: (c)
Features of Fundamental Rights in India:
Exp) Option c is the correct answer.
• They are universal, which means they apply to all
citizens of India, irrespective of their race, place of Uniform civil code is given in the original constitution
birth, religion, caste, creed, color, or sex. under Article 44. It states that “The State shall endeavour
to secure the citizen a Uniform Civil Code throughout the
• They are justiciable, which means they can be enforced
territory of India”.
by the courts, subject to certain restrictions. The
Supreme Court and the High Courts can issue writs for Important Tipss
the protection of Fundamental Rights under Article 32 Some Directive Principles are added in the original
and Article 226 respectively. Constitution as per the needs arise with the time.
• They are not absolute, which means they can be • The 42nd Amendment Act of 1976 added four new
restricted or suspended by the Parliament or the Directive Principles to the original list.
President under certain circumstances, such as during
• They require the State:
a national emergency, martial law, or public interest.
• To secure opportunities for healthy development of
• They are amendable, which means they can be changed
children (Article 39).
or repealed by a constitutional amendment, if it does
not alter the basic structure of the Constitution. • To promote equal justice and to provide free legal aid
to the poor (Article 39 A).
• They are dynamic, which means they can be
interpreted and expanded by the judiciary according to • To take steps to secure the participation of workers in
the changing needs and aspirations of society. the management of industries (Article 43 A).
• To protect and improve the environment and to
89. Solution: (d) safeguard forests and wild life (Article 48 A).
Exp) Option d is the correct answer. • The 44th Amendment Act of 1978 added one more Directive
The Supreme Court of India has recognized all the three Principle, which requires the State to minimise inequalities in
options as fundamental rights of the citizens under the income, status, facilities and opportunities (Article 38).
Constitution of India. These are: • The 86th Amendment Act of 2002 changed the subject-
• Right to Shelter: The Supreme Court has held that the matter of Article 45 and made elementary education a
right to shelter is a part of the right to life and personal fundamental right under Article 21 A. The amended
liberty under Article 21 of the Constitution. It means directive requires the State to provide early childhood
that every citizen has the right to adequate housing care and education for all children until they complete
that ensures human dignity, privacy, and security. The the age of six years.
Supreme Court has also held that the State has a duty to • The 97th Amendment Act of 2011 added a new
provide shelter to the poor, homeless, and marginalized Directive Principle relating to cooperative societies.
sections of society. It requires the state to promote voluntary formation,
• Right to Travel Abroad: The Supreme Court has held that autonomous functioning, democratic control and
the right to travel abroad is a part of the right to freedom professional management of co-operative societies
of movement under Article 19 (1) (d) of the Constitution. (Article 43B).
It means that every citizen has the right to go outside
91. Solution: (b)
India and return to India without any unreasonable
restriction or interference by the State. The Supreme Exp) Option b is the correct answer.
Court has also held that the State can impose reasonable The writ of Habeas Corpus is regarded as the greatest
restrictions on this right in the interests of sovereignty, safeguard for the liberty of a person. Habeas Corpus is a

83 PYQ Workbook
INDIAN POLITY AND GOVERNANCE

Latin term that means “to have the body.” It is a legal action Constitution. It is given in Part IV of the Constitution.
through which a person can challenge the lawfulness of Hence, correct answer is: (A) is true, but (R) is false. The
their detention or imprisonment. The writ is used to seek Part III of the Constitution deal with Fundamental Rights
the release of a person who is unlawfully detained or held (Article 12–35), while Part IV of the Constitution of India
against their will. contains the DPSP (Article 36–51). Article 41 of the Indian
The purpose of the writ of Habeas Corpus is to protect Constitution directs the state to secure the right to work,
individual liberty and ensure that a person is not education and public assistance in certain cases such as
unlawfully deprived of their freedom. It is considered a unemployment, old age, sickness and disablement.
fundamental safeguard against arbitrary or illegal detention, Important Tipss
providing a mechanism for individuals to challenge the
Mahatma Gandhi National Rural Employment Guarantee
lawfulness of their detention before a court of law.
Act (MGNREGA):
Important Tipss • The Indian Parliament passed NREGA in August
• Mandamus: This writ means ‘we command’. It is used to 2005. It came into force on February 2, 2006. On
order a public official, a public body, a corporation, an 2nd October 2009, the act NREGA was renamed
inferior court, a tribunal or a government to perform MGNREGA.
a duty that they are obliged to perform but have failed • It is a poverty alleviation programme of the
or refused to do so. Government of India, which provides the legal
• Prohibition: This writ means ‘to forbid’. It is used to Right to Work.
stop an inferior court, a tribunal or an authority from • It aims to enhance livelihood security in rural areas
exceeding its jurisdiction or acting contrary to the rules by providing at least 100 days of guaranteed wage
of natural justice. It prevents them from continuing a employment in a financial year to every household
proceeding that is ultra vires or invalid. whose adult members volunteer to do unskilled
• Certiorari: This writ means ‘to be certified’. It is used manual work.
to transfer a case pending before an inferior court, a • Funding:
tribunal or an authority to a higher court for review.
• It is shared between the Centre and the States.
It can also be used to quash or nullify an order already
passed by an inferior court, a tribunal or an authority • The Central Government bears 100 per cent of the
that is ultra vires, unconstitutional, illegal, erroneous cost of unskilled labour, 75 percent of the cost of
or without jurisdiction. semi-skilled and skilled labour, 75 percent of the
cost of materials and 6 percent of the administrative
• Quo Warranto: This writ means ‘by what authority’. It
costs.
is used to challenge the legality of a person’s claim to
hold a public office. It prevents usurpation of public • Time-Bound Guarantee of Work as employment must
office by an ineligible or unqualified person. It can also be provided within 15 days of being demanded. In
be used to oust such person from the office if he or she case of failure, an ‘unemployment allowance’ must be
is already holding it given.

92. Solution: (d) 94. Solution: (c)


Exp) Option d is the correct answer. Exp) Option c is the correct answer.
The President of India is not the Guardian of Fundamental The fundamental duties list is given in Article 51A of the
Rights under the Constitution. The Supreme Court and Indian Constitution. These are the moral obligations of
High Courts are the guardians of Fundamental Rights as every citizen of India to promote the spirit of patriotism and
they have the power to issue writs for their enforcement. preserve India’s unity.
The President of India, on the other hand, is the head of the The duty “To strive towards the abolition of untouchability”
executive branch of the government and performs various is not included in the Fundamental Duties of a citizen in
ceremonial, constitutional, and diplomatic functions. India.
While the President has a significant role in upholding and The other options, (a) To protect and improve the natural
defending the Constitution, the direct responsibility for the environment, (b) To cherish and follow the noble ideals
protection and enforcement of fundamental rights lies with which inspired our national struggle for freedom, and (d)
the judiciary. To develop scientific temper, humanism and the spirit of
enquiry and reform, are included as Fundamental Duties of
93. Solution: (c)
a citizen in the Indian Constitution.
Exp) Option c is the correct answer.
95. Solution: (d)
MGNREGA is providing employment to at least one
member of the eligible household for a minimum period of Exp) Option d is the correct answer.
100 days in a year. Hence, Assertion (A) is correct. Whereas, Fundamental rights of citizens of India can be suspended
Right to employment is not enumerated in Part III of the during a state of emergency declared by the President of

PYQ Workbook 84
INDIAN POLITY AND GOVERNANCE

India on the recommendation of the Union Cabinet under Hoisting the National Flag atop the private buildings
Article 352 of the Constitution. During a state of emergency: is a fundamental right of every citizen under Article 19
• All the freedoms guaranteed in Article 19 of the (1) (a) of the Constitution, which guarantees the right to
Constitution shall be automatically suspended. There is freedom of speech and expression. This right includes the
no need for a separate order to suspend them. right to express one’s views and opinions through any mode
of communication, such as words, signs, symbols, gestures,
• The suspension of the right to move to any court for the
pictures, etc. It also includes the right to receive information
enforcement of fundamental rights can only be done by from any source and disseminate it to others.
the order of the President of India under Article 359 of
However, hoisting the National Flag is not explicitly
the Constitution. The President in his order will mention
mentioned or specially provided in Article 19 (1) (a) of
the right whose enforcement will be suspended. The
the Constitution. It is implied or derived from the wider
order of the President may extend to the whole or any
freedom of expression guaranteed by this article. The
part of the territory of India.
Supreme Court has interpreted this article to include the
• The suspension of fundamental rights will exist as long as freedom of press as an integral part of the freedom of speech
the emergency period. and expression
96. Solution: (a) 99. Solution: (a)
Exp) Option a is the correct answer. Exp) Option a is the correct answer.
The rule of law is a political and legal principle that means The right under Article 32 is available to anyone whose
all people and institutions are subject to and accountable rights are being violated, not just citizens – the distinction
to law that is fairly applied and enforced. It also implies is important because, for instance, while the right to freedom
that no one is above the law, that the government is not of speech in Article 19(1)(a) is provided to citizens, the right
arbitrary, and that the laws are consistent and impartial. to life and personal liberty is guaranteed to all persons in
In India, the rule of law is enshrined in the Constitution, India.
which prescribes the ideas of justice, liberty and equality for The right of a foreign national (i.e. a natural person)
all citizens. The Constitution also provides for separation of to file a writ petition to enforce the fundamental rights
powers, judicial review, fundamental rights, and directive guaranteed to him by the Constitution of India, was upheld
principles of state policy to ensure that the rule of law is by the apex Court in the case of Louis De Raedt vs. Union
upheld in the country. of India wherein, a petition was filed to enforce Article 21 of
The most appropriate answer is- One act for all and one the Constitution.
judiciary for all. This option reflects the idea that everyone 100. Solution: (a)
is subject to the same laws and the same judicial system,
Exp) Option a is the correct answer
regardless of their status or power.
Under Article 352, the president can declare a national
97. Solution: (b) emergency when the security of India or a part of it is
Exp) Option b is the correct answer. threatened by war or external aggression or armed rebellion.
The President can declare a national emergency even before
Option I is correct: In Article 39 under DPSP, it is given
the actual occurrence of war or armed rebellion or external
that there shall be equal pay for equal work for both men
aggression
and women.
Option II is correct: In Article 44 under DPSP, reference Important Tipss
related to Uniform civil code is given in the constitution. • Article 370 of the Indian constitution is an article that
Article 44 states that “The State shall endeavour to secure granted special autonomous status to the state of Jammu
the citizen a Uniform Civil Code throughout the territory and Kashmir. On 5 August 2019, the government of
of India”. India revoked the special constitutional status of the
erstwhile state of Jammu and Kashmir under Article
Option III is incorrect: Small family norm is not given in
370 of the Constitution,
the Constitution of India explicitly.
• Under Article 371, the President is authorised to
Option IV is incorrect: Education through mother tongue at
provide that the Governor of Maharashtra and that
primary level is not given in the DPSP in the Constitution.
of Gujarat would have special responsibility for: The
Article 350 A describes that “It shall be the endeavor of every
establishment of separate development boards for
State and of every local authority within the State to provide
Vidarbha, Marathwada and the rest of Maharashtra,
adequate facilities for instruction in the mother tongue
Saurashtra, Kutch and the rest of Gujarat.
at the primary stage of education to children belonging to
linguistic minority groups.” • Article 395 repeals the Indian Independence Act,
1947, and the Government of India Act, 1935, but not
98. Solution: (b) including the Abolition of Privy Council Jurisdiction
Exp) Option b is the correct answer. Act, 1949..

85 PYQ Workbook
INDIAN POLITY AND GOVERNANCE

101. Solution: (d) this clause shall, to the extent of the contravention, be void.
Exp) Option d is the correct answer. It also defines law and laws in force for the purpose of this
article. It also provides that nothing in this article shall apply
According to the Constitution of India, the Supreme Court
to any amendment of this Constitution made under Article
and High Courts are given the power to enforce the
368.
Fundamental Rights by the Constitution. Article 32 of the
Constitution grants the right to remedies for the enforcement Article 13 of the Constitution gives precedence to
of the fundamental rights of a citizen of India who has constitutional provisions over the laws made by the
suffered. This makes the fundamental rights enforceable in Union Parliament/State Legislatures because it declares
the court of law. The aggrieved party can directly move to that any law that is inconsistent with or derogates from
the Supreme Court or the High Court for the protection of the fundamental rights (Part III) shall be null and void. It
their rights. also empowers the courts to strike down any such law that
violates or infringes the fundamental rights.
102. Solution: (a)
104. Solution: (c)
Exp) Option a is the correct answer.
Exp) Option c is the correct answer
The 42nd Constitutional Amendment Act of 1976 is
often referred to as the “Mini Constitution” because of its The President of India imposes President’s rule in any state on
extensive scope and significant changes to various parts of the recommendation of the Governor of the concerned state.
the Indian Constitution. It was passed during the period of President’s rule means the suspension of state government
the Emergency and introduced numerous amendments that and the direct rule of the center. Presidential rule or state
aimed to strengthen the position of the government and emergency can be imposed in any state under Art 356 of the
centralize power. constitution of India.

Important Tipss Important Tipss


Features of 42nd Constitutional Amendment Act: • The proclamation of the presidential rule shall be
approved by the Parliament within two months from
• It added the words socialist, secular and integrity to
the date of its issue by a simple majority.
the Preamble of the Constitution.
• After the proclamation of Presidential rule, state
• It laid down the Fundamental Duties of Indian
assembly get dissolved.
citizens in a new Part IV-A of the Constitution.
• This article was used for the first time in the state of
• It extended the term of the Lok Sabha and the state
Punjab.
legislative assemblies from five to six years.
• It transferred five subjects from the state list to the 105. Solution: (c)
concurrent list, namely education, forests, weights and Exp) Option c is the correct answer.
measures, protection of wild animals and birds, and Statement I is correct: Article 368 was amended by the 24th
administration of justice. Constitution Amendment Act, 1971 to affirm the power of
• It curtailed the power of the Supreme Court and the Parliament to amend any part of the Constitution and to
High Courts to review the constitutional validity of make it mandatory for the President to give assent to any
laws and gave more power to the Parliament to amend Constitutional Amendment Bill.
the Constitution. Statement II is correct: Article 368 was also amended by the 101st
• It changed the relationship between the Centre and Constitution Amendment Act, 2016 to introduce a new clause
the states by giving more authority to the Centre in (2A) that provides for ratification by at least half of the State
matters of deployment of armed forces, property taxes, Legislatures for amendments related to Goods and Services Tax
elections, inter-state trade and commerce, etc. (GST).
• It gave more power to the Prime Minister’s Office and
106. Solution: (d)
reduced the role of the President and the Governors
to mere formal heads Exp) Option d is the correct answer.
The special rights and privy purse enjoyed by the Indian
103. Solution: (a)
Princely States were abolished in 1971 by the 26th
Exp) Option a is the correct answer. Amendment of the Indian Constitution.
Article 13 of the Constitution of India describes the means
Important Tipss
for judicial review. It states that all laws in force in the
territory of India immediately before the commencement • The privy purse was a specific amount of money that
of this Constitution, in so far as they are inconsistent with was paid annually by the Indian government to the
the provisions of this Part (Part III), shall, to the extent of rulers of princely states and their successors who had
such inconsistency, be void. It also states that the State shall acceded to India in 1947 and 1949.
not make any law which takes away or abridges the rights • The amount of privy purse was determined by many factors
conferred by this Part and any law made in contravention of like state’s revenue, state’s status under British rule, etc.

PYQ Workbook 86
INDIAN POLITY AND GOVERNANCE

107. Solution: (a) The Directive Principles are non-justiciable in nature, that
Exp) Option a is the correct answer. is, they are not legally enforceable by the courts for their
violation. Therefore, the government (Central, state and
Article 24 of the Constitution of India is a fundamental
local) cannot be compelled to implement them.
right that prohibits the employment of children below the
age of fourteen years in any factory, mine, or any other 111. Solution: (c)
hazardous occupation. The article aims to protect the health
Exp) Option c is the correct answer
and education of children and to prevent their exploitation.
The Indian constitution is both federal and unitary in nature,
The prohibition of child labour under Article 24 is absolute
as it is a combination of federal and unitary features. Under
and unconditional. It means that no child below the age
Article 352, the president can declare a national emergency
of fourteen years can be employed in any factory, mine, or
on the basis of war, external aggression or armed rebellion.
any other hazardous occupation, irrespective of the nature,
According to Article 356, President’s Rule can be imposed
duration, or condition of work. The Supreme Court of India
on any state of India on the grounds of the failure of the
has upheld the absolute prohibition of child labour under
Article 24 and has issued various directions and guidelines constitutional machinery.
to ensure its effective implementation. 112. Solution: (a)
108. Solution: (a) Exp) Option a is the correct answer.
Exp) Option a is the correct answer. The case that gave Parliament the right to amend fundamental
The Supreme Court of India has expanded the scope of rights was the Kesavananda Bharati Case of 1973. In this landmark
Article 21 to include the right to education as a part of the judgment, the Supreme Court propounded the doctrine of basic
right to life and personal liberty. The landmark case that structure, which holds that there is a basic structure or essential
initiated this expansion was Unnikrishnan v/s Andhra features of the Constitution that cannot be altered or destroyed by
Pradesh (1993). In this case, the Supreme Court held that the any amendment. The court held that Parliament has the power to
right to education is implicit in the right to life, as education amend any part of the Constitution, including the fundamental
is essential for the development of human personality and rights, but it cannot violate the basic structure. The court overruled
potential. The Supreme Court also held that the right to the previous decision of Golaknath Case of 1967, which had held
education is a fundamental right for children up to the age of that Parliament cannot amend the fundamental rights at all.
14 years, as it is enshrined in Article 45 of the Constitution,
113. Solution: (d)
which is a directive principle of state policy.
Exp) Option d is the correct answer.
Important Tipss
The correct Pairs are:
• Govind v/s State of Madhya Pradesh (1975) is a case
that expanded the scope of Article 21 to include the (a) Organization of Article 39 A (not Article 42)
right to privacy. agriculture and animal
husbandry
• Parmanand Katara v/s Union of India (1989) is a case
that expanded the scope of Article 21 to include the (b) Organization of village Article 48 (not Article 43)
right to medical aid. Panchayat
• Chameli Singh v/s State of Uttar Pradesh (1996) is a (c) Organization of village Article 40 (not Article 41)
case that expanded the scope of Article 21 to include Panchayat
the right to shelter. (d) There is equal pay for Article 39
equal work for both
109. Solution: (c)
men and women
Exp) Option c is the correct answer.
The Fundamental Duties of citizens were added to the Important Tipss
Constitution by the 42nd Amendment in 1976, upon the Other Articles given in the question:
recommendations of the Swaran Singh Committee that was Article 41:
constituted by the government earlier that year. Fundamental
• To secure the right to work, to education and to public
Duties help to regulate the behaviour of the citizens and to
assistance in cases of unemployment, old age, sickness
bring about excellence in all the spheres of the citizens. The
and disablement
42nd Amendment also made various other changes to the
Constitution, such as adding the terms socialist, secular Article 42:
and integrity to the Preamble, curtailing the power of the • To make provision for just and humane conditions of
Supreme Court and High Courts, and giving more power work and maternity relief.
to the Prime Minister Article 43:
110. Solution: (b) • To secure a living wage, a decent standard of life and
Exp) Option b is the correct answer. social and cultural opportunities for all workers.

87 PYQ Workbook
INDIAN POLITY AND GOVERNANCE

114. Solution: (a) to amend any part of the Constitution, including the
Exp) Option a is the correct answer. Fundamental Rights, but it cannot alter or destroy the
basic structure or essential features of the Constitution.
The Swaran Singh Committee was set up by the Congress
government in 1976 to make recommendations about (C) Minerva Mills v/s Union of India (1980) - This case
fundamental duties, the need and necessity of which was felt dealt with the validity of the 42nd Amendment Act,
during the operation of the internal emergency (1975–1977). which inserted two clauses in Article 368, giving power
to Parliament to amend any part of the Constitution,
The committee recommended the inclusion of a chapter on
including the basic structure, and making such
fundamental duties in the Constitution. It stressed that the
amendments immune from judicial review. The Supreme
citizens should become conscious that in addition to the
Court struck down these clauses as unconstitutional and
enjoyment of rights, they also have certain duties to perform
violative of the basic structure. It reaffirmed the doctrine
as well.
of basic structure and held that there is a harmony
Important Tipss and balance between the Fundamental Rights and the
• Shah Commission: The Shah Commission, officially Directive Principles, which constitute the core of the
basic structure.
known as the Commission of Inquiry into the Affairs
of the Indian National Congress, was established 116. Solution: (d)
in 1977 to investigate allegations of corruption and
Exp) Option d is the correct answer.
malpractices by the government during the period of
the state of emergency in India from 1975 to 1977. Article 368 (2) of the Constitution of India states that some
amendments also require the ratification by not less than
• Administrative Reforms Commission: The half of the states by resolution passed by their legislatures
Administrative Reforms Commission (ARC) is a before they are presented to the President for assent.
committee set up by the government of India to
If a bill comes to Rajasthan Legislative Assembly after
recommend measures to improve the efficiency and
passing by Parliament of India for resolution to ratify such
effectiveness of the public administration system.
an amendment, then:
• Santhanam Committee: The Santhanam Committee,
• Resolution can be passed by Assembly. This is true, as
officially known as the Committee on Prevention of
the Assembly has the power to ratify the amendment by a
Corruption, was appointed in 1962 to investigate and
simple majority of members present and voting.
suggest measures to combat corruption in India.
• Resolution can be rejected by Assembly. This is true, as
115. Solution: (c) the Assembly has the power to reject the amendment by a
Exp) Option c is the correct answer. simple majority of members present and voting.

The correct chronological order of the judgements dealing • No Amendment shall be proposed to such a resolution.
This is true, as the Assembly cannot propose any
with Fundamental Rights is:
amendment to the bill passed by Parliament.
(D) A.K Gopalan v/s State of Madras (1950) - This case
dealt with the validity of preventive detention and the 117. Solution: (a)
scope of Article 21 of the Constitution. The Supreme Exp) Option a is the correct answer.
Court held that the Fundamental Rights are not absolute “The age of a Judge of the Supreme Court of India shall
and can be restricted by law. It also held that each be determined by such authority and in such manner, as
Fundamental Right is separate and distinct, and there is Parliament may by-laws provide” this feature was inserted
no inter-relationship between them. by the 15th Constitution Amendment. This amendment
(A) Golaknath v/s State of Punjab (1967) - This case empowered Parliament to prescribe the age of retirement for
dealt with the power of Parliament to amend the judges of the Supreme Court of India through legislation.
Fundamental Rights. The Supreme Court overruled
118. Solution: (c)
its previous decisions and held that Parliament cannot
Exp) Option c is the correct answer.
amend the Fundamental Rights, as they are sacrosanct
and transcendental. It also held that Article 368 only lays • The 97th Amendment of the Constitution of India added
down the procedure for amendment, and any amendment the right to form cooperative societies under Article 19
is a law under Article 13, which cannot abridge or take (1)(c) in 2011
away the Fundamental Rights. • The 81st Amendment of the Constitution of India
(B) Keshvananda Bharati v/s State of Kerala (1973) - This provided for the protection of SC/ST reservation in
filling backlog of vacancies in 2000
case dealt with the validity of the 24th, 25th and 29th
Amendment Acts, which curtailed the Fundamental • The 99th Amendment of the Constitution of India
Rights relating to property and introduced the doctrine established the National Judicial Appointments
of basic structure. The Supreme Court overruled Commission in 2014
the Golaknath case and upheld the validity of these • The 91st Amendment of the Constitution of India limited
amendments. It held that Parliament has the power the size of the Council of Ministers to 15% of the total

PYQ Workbook 88
INDIAN POLITY AND GOVERNANCE

number of members of the House of the People or the Statement II is correct: Supreme Court has laid down in its
Legislative Assembly in 2003 verdict in Minerva Mills Case, 1980 that the preference given
only to Directive Principles of State Policy mentioned in
119. Solution: (a)
Article 39 (b) and (c) over Fundamental Rights mentioned
Exp) Option a is the correct answer in Articles 14 and 19 is Constitutional.
According to Article 356, President’s Rule can be imposed
on any state of India on the grounds of the failure of the Important Tipss
constitutional machinery. The proclamation of President’s Minerva Mill Case:
Rule should be approved in both Houses of Parliament within • The case was filed by Minerva Mills, a textile company
two months of its issue. The approval is through a simple that was nationalized by the government under the Sick
majority. A Proclamation under this Article can be reviewed Textile Undertakings Act, 1974.
by the Supreme Court. Along with this Proclamation, the
• The case challenged the validity of clauses 4
State Legislative Assembly cannot be dissolved.
and 5 of Article 368, which were inserted by the
120. Solution: (c) 42nd Amendment Act, 1976. These clauses gave
Exp) Option c is the correct answer. unlimited power to Parliament to amend any part of
the Constitution, including the basic structure, and
The President’s power to veto a Bill for the amendment of
excluded judicial review of such amendments.
the Constitution was taken away by the 24th Amendment
to the Indian Constitution. Before the 24th Amendment, • The Supreme Court struck down these clauses as
the President had the discretion to either give or withhold unconstitutional and violative of the basic structure
his assent to a Constitution Amendment Bill passed by doctrine. The court held that Parliament’s power to
Parliament. However, the 24th Amendment, which was amend is limited by the Constitution and cannot be
enacted in 1971, substituted the phrase ‘shall give his assent’ used to destroy its identity and spirit.
for the word ‘may,’ thereby making it mandatory for the • The court also upheld the validity of clause 1 of Article
President to give his assent to a Constitution Amendment Bill 31C, which gave preference to Directive Principles of
once it has been duly passed by both Houses of Parliament. State Policy contained in Article 39 (b) and © over
Important Tipss Fundamental Rights mentioned in Articles 14 and 19.
However, the court struck down clause 2 of Article 31C,
• The 23rd Amendment of the Constitution of India,
which immunized any law from judicial scrutiny even
enacted in 1969, extended the reservation of seats
if it does not give effect to the Directive Principles.
for Scheduled Castes and Scheduled Tribes and
representation of Anglo-Indians by nomination in the • The court reaffirmed the doctrine of basic structure
Lok Sabha and the State Legislative Assemblies for and harmonious construction between Fundamental
another ten years, i.e., up to 1980 Rights and Directive Principles.
• The 42nd Amendment of the Constitution of India, • The court also clarified that the basic structure
enacted in 1976, gave primacy to the Directive includes the limited amending power of Parliament,
Principles over Fundamental Rights, curtailed the the supremacy of the Constitution, the democratic
power of judicial review, strengthened the power and secular character of the polity, the separation of
of Parliament and the executive, declared India as powers, and the individual freedom.
a “sovereign socialist secular democratic republic”,
added new fundamental duties and a new chapter on 122. Solution: (b)
emergency provisions. Exp) Option b is the correct answer.
• The 44th Amendment of the Constitution of India, The 97th Amendment of the Constitution of India, officially
enacted in 1978, restored the civil liberties and known as The Constitution (Ninety-seventh Amendment)
democratic rights that were curtailed by the 42nd Act, 2011, introduced clauses dealing with the organization
Amendment during the Emergency. It also reduced
and working of cooperative societies in the country.
the duration of President’s rule, introduced safeguards
against misuse of emergency provisions, and deleted the Important Tipss
right to property from the list of fundamental rights. Features of 97th Constitutional Amendment Act:
121. Solution: (c) • It inserted a new Part IXB in the Constitution, which contains
Exp) Option c is the correct answer. Articles 243ZH to 243ZT, which deal with various aspects of
cooperative societies such as incorporation, membership,
Statement I is incorrect: 44th Constitutional Amendment, elections, audit, supervision, offences and penalties.
1978 did not give preference to all the Directive Principles
of State Policy over Fundamental Rights mentioned in • The amendment also added the words “or co-operative
Articles 14 and 19. It only retained the preference given to societies” to Article 19(1)© of the Constitution to
Directive Principles contained in Article 39 (b) and © by expand the fundamental right to form associations or
the 25th Amendment, 1971. unions to cover cooperative societies too.

89 PYQ Workbook
INDIAN POLITY AND GOVERNANCE

• It also added a new Directive Principle under Article 125. Solution: (c)
43B, which says: “The State shall endeavour to promote Exp) Option c is the correct answer.
voluntary formation, autonomous functioning, The 89th Amendment of the Constitution of India, officially
democratic control and professional management of known as The Constitution (Eighty-ninth Amendment) Act,
co-operative societies” 2003, bifurcated the combined National Commission for
SCs and STs into two separate bodies, namely, National
123. Solution: (c)
Commission for Scheduled Castes (under Article 338) and
Exp) Option c is the correct answer. National Commission for Scheduled Tribes (under Article
Option (c) Article 22(2) is not correctly matched. The 338-A).
correct match for Article 22(2) is: Every person who is 126. Solution: (a)
arrested and detained in custody shall be produced before
Exp) Option a is the correct answer.
the nearest magistrate within a period of twenty-four hours
Gherao is a form of protest or agitation in which a group
of such arrest excluding the time necessary for the journey
of people surround an official or a building and prevent
from the place of arrest to the court of the magistrate and
them from leaving or entering until their demands are met.
no such person shall be detained in custody beyond the said
Gherao is not protected by Article 19 (1) (b) of the
period without the authority of a magistrate.
Constitution. It is punishable under various sections of the
Important Tipss Indian Penal Code, such as Section 141 (unlawful assembly),
Article 22 is divided into six clauses, which are: Section 143 (punishment for unlawful assembly), Section
147 (rioting), Section 149 (common object of unlawful
• Clause (1): Right to be informed of the grounds for
assembly), Section 341 (wrongful restraint), Section 342
arrest and the right to consult and be defended by a
(wrongful confinement), Section 353 (assault or criminal
legal practitioner of choice.
force to deter public servant from discharge of his duty), etc.
• Clause (2): Detained individuals must be produced
before the nearest magistrate within 24 hours, and 127. Solution: (d)
their detention cannot exceed this period without the Exp) Option d is the correct answer.
authority of a magistrate.
The Constitutions (One Hundred and First) Amendment
• Clause (3): Exemptions to the rights of arrest and Act 2016 introduced a national Goods and Services Tax
detention under clauses (1) and (2) for enemy aliens (GST) in India from 1 July 2017. The amendment gave
and those detained under laws providing for preventive concurrent taxing powers to the Union and the States to levy
detention. GST on every transaction of supply of goods or services or
• Clause (4): Preventive detention laws cannot authorize both. It also created the GST Council, a joint forum of the
detention for more than three months unless an Centre and States, to recommend matters related to GST.
Advisory Board of High Court judges reports sufficient
cause for the continued detention. 128. Solution: (d)
• Clause (5): Detained individuals must be provided with Exp) Option d is the correct answer.
the grounds for the order of preventive detention and Article 26 of the Constitution of India is a fundamental
given an opportunity to make a representation against right that grants the freedom to manage religious affairs
it. to every religious denomination or any section thereof. It
• Clause (6): The authority making the order of provides that subject to public order, morality and health,
preventive detention is not obligated to disclose facts every religious denomination or any section thereof shall
that it deems against the public interest. have the right:
• to establish and maintain institutions for religious and
124. Solution: (a)
charitable purposes;
Exp) Option a is the correct answer. • to manage its own affairs in the matter of religion;
Article 32 of the Constitution of India empowers the • to own and acquire movable and immovable property;
Supreme Court to issue different writs for the enforcement and
of fundamental rights of the citizens. A writ is a written • to administer such property in accordance with law
order from the court that commands constitutional remedies
National security, education, and secularism are not
for the violation or protection of the right. The Supreme mentioned in Article 26 as grounds for limiting the
Court can issue five types of writs, namely, habeas corpus, religious rights of any denomination or section.
mandamus, prohibition, certiorari and quo warranto.
Article 32 is considered as a part of the basic structure of 129. Solution: (c)
the Constitution and cannot be suspended except during an Exp) Option c is the correct answer.
emergency. The rights are called Fundamental Rights because:

PYQ Workbook 90
INDIAN POLITY AND GOVERNANCE

i. It is written in the Constitution. The Fundamental Rights Freedom of expression is a fundamental right guaranteed by
are enshrined in Part III of the Constitution of India from Article 19 (1) (a) of the Constitution of India. It includes
Article 12 to 35 and are guaranteed to all citizens. the right to express one’s views and opinions through any
iv. It is essential for personality development. The mode of communication, such as words, signs, symbols,
Fundamental Rights are defined as basic human freedoms gestures, pictures, etc. It also includes the right to receive
where every Indian citizen has the right to enjoy for a proper and impart information from any source and disseminate it
and harmonious development of personality and life. to others.
However, freedom of expression is not absolute or unlimited.
v. Parliament can’t make law against it. The Fundamental
It is subject to reasonable restrictions imposed by the State
Rights are enforceable by the courts, subject to certain
under Article 19 (2) of the Constitution. These restrictions
restrictions, and any law made by the Parliament that
can be imposed in the interests of the sovereignty and
violates or abridges the Fundamental Rights can be declared
integrity of India, the security of the State, friendly
unconstitutional by the judiciary relations with foreign States, public order, decency or
• Although the Fundamental Rights are essential for a morality, contempt of court, defamation or incitement to
democratic system, they are not exclusively democratic an offence.
in nature. There are other forms of government that may Unbecoming criticism is not one of the grounds on which
also grant some civil liberties to their citizens, such as freedom of expression can be restricted. Unbecoming
constitutional monarchy or republicanism. criticism means criticism that is inappropriate, improper, or
• The Fundamental Rights are not necessarily related to disrespectful. However, such criticism may not necessarily
public welfare, which is more of a goal of the Directive violate any of the grounds mentioned in Article 19 (2) of the
Principles of State Policy (Part IV of the Constitution). Constitution.
The Fundamental Rights are more concerned with 132. Solution: (b)
individual rights and liberties that protect the citizens
Exp) Option b is the correct answer
from arbitrary state action or discrimination.
During National Emergency the President by order can direct
130. Solution: (c) that all provisions regarding division of taxes between Union
Exp) Option c is the correct answer and States and grants-in-aids remain suspended. However,
such suspension shall not go beyond the expiration of the
When a national emergency is declared on the ground of ‘war’
financial year in which the Proclamation ceases to operate.
or ‘external aggression’, it is known as ‘External Emergency’.
On the other hand, when it is declared on the ground of 133. Solution: (b)
‘armed rebellion’, it is known as ‘Internal Emergency’. As per Exp) Option b is the correct answer.
Article 352, the President can declare a national emergency
The 93rd Amendment of the Constitution of India, officially
when the security of India is threatened by war/external known as The Constitution (Ninety-third Amendment)
aggression/armed rebellion. The proclamation of Emergency Act, 2005, added clause (5) in Article 15 which enables
must be approved by both the Houses of Parliament within the government to make any special provision for the
one month. If approved by both the Houses of Parliament, the advancement of any socially and educationally backward
emergency continues for six months. For the proclamation classes of citizens or for the Scheduled Castes or the
of emergency or its continuance, the resolution must be Scheduled Tribes in so far as such special provisions relate
passed by either House of Parliament by a special majority. to their admission to educational institutions, including
A revocation of emergency does not require parliamentary private educational institutions, whether aided or unaided
approval. It can be revoked by the President at any time by a by the State, other than the minority educational institutions.
subsequent proclamation. The emergency must be revoked The amendment enabled the implementation of the
if the Lok Sabha passes a resolution by a simple majority 27% reservation for OBCs in admission to educational
disapproving of its continuation. institutions as recommended by the Mandal Commission in
1992.
Important Tipss
• President can declare a national emergency even 134. Solution: (d)
before the actual occurrence of these situations if he is Exp) Option d is the correct answer.
satisfied that there is a probable danger. Article 20 of the Constitution of India is a fundamental
• The emergency can be extended to an indefinite period right that provides protection to the accused regarding
with an approval of Parliament every six months. conviction from double-bar and self-incrimination. The
• The President can proclaim a national emergency only article is divided into three clauses, which are:
after receiving a written recommendation from the • Clause (1): Protection against ex post facto laws. It means
cabinet that no person shall be convicted of any offence except for
violation of a law in force at the time of the commission
131. Solution: (d) of the act, nor be subjected to a penalty greater than that
Exp) Option d is the correct answer. prescribed by such law.

91 PYQ Workbook
INDIAN POLITY AND GOVERNANCE

• Clause (2): Protection against double jeopardy. It means • No ex-post-facto law: It ensures that a person cannot
that no person shall be prosecuted and punished for the be convicted of any offence except for the violation of a
same offence more than once. law in force at the time of committing the act.
• Clause (3): Protection against self-incrimination. It • No double jeopardy: No person can be prosecuted and
means that no person accused of any offence shall be punished for the same offence more than once.
compelled to be a witness against himself.
• No self-incrimination: No person accused of any
135. Solution: (c) offence can be compelled to be a witness against
themselves.
Exp) Option c is the correct answer.
• An ex-post-facto law is one that imposes penalties
The Constitution of India does not guarantee the Right to
retrospectively, which means it applies to acts that were
Property as a fundamental right. Originally, the right to
committed before the law was enacted or increases the
property was included as a fundamental right under Article
penalties for such acts.
19(1)(f ) and Article 31 of the Constitution. However, the
44th Amendment Act of 1978 removed the right to property • Article 20 prohibits the enactment of ex-post-facto
from the list of fundamental rights and made it a legal right criminal laws, but this limitation does not apply to
instead. civil laws or tax laws. Civil liability or taxes can be
imposed retrospectively.
136. Solution: (b)
138. Solution: (d)
Exp) Option b is the correct answer.
Exp) Option d is the correct answer.
The Directive Principles of State Policy cannot override
fundamental rights. Fundamental rights, enshrined in Part While fundamental duties do not directly determine the
III of the Indian Constitution, are the individual rights and constitutionality of laws, they are often considered by the
liberties that are guaranteed to every citizen. If a conflict courts while interpreting and deciding cases related to the
arises between a fundamental right and a directive principle, fundamental rights of citizens. Fundamental duties provide
the courts generally strive to interpret the laws in a way that guidance and context in interpreting the fundamental
reconciles both, but in cases where reconciliation is not rights, and they can influence the court’s decision-making
possible, the fundamental rights take precedence. process.

Important Tips Important Tips


• DPSP are a set of guidelines or principles provided in • The fundamental duties were added to the Constitution
Part IV of the Constitution of India. of India through the 42nd Amendment Act in 1976
• They serve as moral and political directives for the through Article 51A of the Constitution of India.
government in formulating policies and making laws. • The fundamental duties are not just applicable to
• DPSP are not legally enforceable in a court of law. citizens but also to the State and every individual
• They are non-justiciable, meaning they cannot be residing in India.
directly enforced through the courts. • The fundamental duties aim to promote patriotism,
• The DPSP aim to establish social and economic justice unity, and the spirit of common brotherhood among
in society. citizens.
• They provide a framework for the government to • Some of the fundamental duties include respecting the
promote the welfare of the people, secure a just and Constitution and its ideals, promoting harmony and
humane society, and reduce inequalities. the spirit of inquiry, safeguarding public property,
protecting the environment, and striving towards
137. Solution: (c)
excellence in all spheres of individual and collective
Exp) Option c is the correct answer. activity.
According to the Article 20 (3) of the Indian Constitution, • The fundamental duties are non-enforceable through
“No person accused of any offence shall be compelled to writs.
be a witness against himself.” This indicates that a person • The fundamental duties are not justiciable, meaning
has the right to not be compelled to act as a witness against individuals cannot be legally prosecuted for failing to
themselves. fulfill their fundamental duties.
Important Tips 139. Solution: (b)
Some points about Article 20 of the Indian Constitution: Exp) Option b is the correct answer
• It safeguards against arbitrary and excessive The provision “To cherish and follow the noble ideas which
punishment for accused persons, including citizens, inspired our national struggle for freedom” is a part of
foreigners, and legal entities such as companies or the Fundamental Duties (Art 51A-b) mentioned in the
corporations. Constitution of India.

PYQ Workbook 92
INDIAN POLITY AND GOVERNANCE

140. Solution: (d) Fundamental Duties and not DPSP. Article 51A(h) of the
Exp) Option d is the correct answer Indian Constitution states that it shall be the duty of every
citizen to develop scientific temper, humanism, and the
Participation of workers has been provided for under DPSPs
spirit of inquiry and reform.
(Art 43A), but it is for the management of industries and
not for local self-government. 146. Solution: (a)

141. Solution: (c) Exp) Option a is the correct answer

Exp) Option c is the correct answer The 44th Amendment Act added a new clause to Article
74 of the Constitution, which gives the President the
In the Minerva Mills case (1980), the Supreme Court of
power to refer a matter back to the Council of Ministers
India examined the relationship between the Directive
for reconsideration. This power can be exercised by the
Principles of State Policy (DPSP) and the Fundamental
President only once on a matter. If the Council of Ministers
Rights enshrined in the Indian Constitution. The court sends the same advice again to the President, the President
held that the DPSP and the Fundamental Rights are both must accept it.
equally important and essential for the governance of the
country. They are complementary to each other and must 147. Solution: (d)
be interpreted harmoniously. Exp) Option d is the correct answer.
142. Solution: (a) Article 51A of the Constitution of India deals with the
fundamental duties of citizens. It lists a set of moral, civic,
Exp) Option a is the correct answer
and ethical obligations that citizens are expected to fulfill
The Constitution (36th Amendment) Act, 1975, was for the betterment of society and the nation as a whole. The
passed by the Parliament of India to incorporate Sikkim fundamental duties were added to the Constitution by the
into the Indian Union as an Associate State. The Act was 42nd Amendment Act in 1976.
passed on 26 April 1975, and Sikkim became an Associate
State on 16 May 1975. 148. Solution: (a)
Exp) Option a is the correct answer.
143. Solution: (b)
The amending power of the Parliament is set forth in Article
Exp) Option b is the correct answer
368 of the Constitution of India. Article 368 provides the
The Anti-Defection Law empowers the Speaker/Chairman procedure for amending the Constitution, including the
to make decisions on disqualification and to maintain the types of amendments, the majority required for their passage,
stability and integrity of political parties and legislative and the involvement of the President in the amendment
bodies by curbing defections. Thus, it can be said that process.
the 52nd Amendment has enormously strengthened the
powers of the Speaker/Chairman of the Parliament and State 149. Solution: (a)
Legislatures. Exp) Option a is the correct answer.
144. Solution: (b) Option a is incorrect: The Speaker of Lok Sabha cannot
call a joint sitting of both Houses to pass a Constitution
Exp) Option b is the correct answer
Amendment Bill, as such a bill must be passed separately by
Both the Fundamental Duties and Directive Principles of each House with the required majority.
State Policy recognize and emphasize the concern for the
Option b is correct: A Constitution Amendment Bill must be
protection and improvement of the environment, forests,
passed separately by both the Houses of Parliament with a
and wildlife.
special majority. This means that it should be supported by
Important Tipss a majority of the total membership of each House, as well
• Fundamental Duty: Article 51A(g) of the Constitution as by a majority of not less than two-thirds of the members
of India states that it shall be the fundamental duty present and voting.
of every citizen of India to protect and improve the Option c is correct: A Constitution Amendment Bill can be
natural environment including forests, lakes, rivers introduced in either House of Parliament by a minister or
and wildlife. a private member, and does not require the prior approval
• Directive Principle: Article 48A of the Constitution of the President
of India states that the State shall endeavour to protect Option d is correct: A Constitution Amendment Bill can be
and improve the environment and to safeguard the sponsored by a private member, but it has very little chance
forests and wildlife of the country. of being passed by Parliament.

145. Solution: (d) 150. Solution: (c)


Exp) Option d is the correct answer Exp) Option c is the correct answer
“Promotion of scientific temper, humanity and the Article 14 of the Constitution of India guarantees equality
spirit of inquiry and reform” is actually a part of the before the law and equal protection of the laws to all persons

93 PYQ Workbook
INDIAN POLITY AND GOVERNANCE

within the territory of India. This means that both citizens Statement 2 is correct: The right to equality includes the
and foreigners are entitled to the protection of Article 14 principle of equality of opportunity in matters of public
and it does not apply to Indian citizens living abroad and employment (Article 16). It ensures that all individuals
citizens born in India but living abroad. have an equal opportunity to be considered for public
employment based on their qualifications and abilities,
151. Solution: (d)
without discrimination.
Exp) Option d is the correct answer.
Statement 3 is incorrect: The right to equality, under the
Statement 1 is correct: The Right to Education Act guarantees provisions of Articles 14-18, does not specifically grant
the right of children to receive free and compulsory educational and cultural rights to minorities. They are
education until the completion of elementary education. part of the Cultural and Educational Rights under Articles
It emphasizes the importance of providing education to all 29-30 of the Constitution.
children and ensures that they have access to education in
Statement 4 is correct: The right to equality includes the
their neighborhood.
abolition of untouchability. Untouchability, which was a
Statement 2 is correct: The Act prohibits the deployment practice associated with the caste system in India, was
of teachers for non-educational works, except for specific abolished under Article 17 of the Constitution. It ensures
purposes such as the decennial census, elections, and disaster that every individual is treated with dignity and without any
relief. This provision aims to ensure that teachers are focused form of untouchability.
on their primary role of imparting education.
Statement 3 is incorrect: The Right to Education Act 154. Solution: (d)
does not specifically provide for the right of minorities Exp) Option d is the correct answer.
to establish and administer educational institutions. This The major cleavage in global human rights discourse is often
provision is covered under Article 30 of the Constitution of seen between civil and political rights on one hand, and
India, which guarantees the right of minorities to establish economic, social, and cultural rights on the other hand.
and administer educational institutions of their choice.
There is often a debate and tension between these two
Statement 4 is incorrect: Article 29(2) of the Constitution categories of rights, as they require different approaches
of India and not RTE Act prohibits discrimination on and resource allocation. Some argue that civil and political
grounds of religion, race, caste, language or any of them in rights are fundamental and must be prioritized, while others
admission to educational institutions. emphasize the importance of economic, social, and cultural
152. Solution: (d) rights for achieving social justice and equality.

Exp) Option d is the correct answer. 155. Solution: (c)


Statement 1 is correct: Free and compulsory education for all Exp) Option c is the correct answer
children of the age of 6-14 years, relates to the Fundamental Article 51 of the Constitution of India outlines the directive
Right to Education. This right is provided under Article principles of state policy related to international relations.
21A of the Constitution of India. However, Article 51 does not explicitly mention the use
Statement 2 is correct: The prohibition of trafficking in of UN peacekeeping forces as a directive principle of state
human beings and forced labor, relates to the Fundamental policy.
Right against exploitation. This right is provided under
Article 23 of the Constitution of India. Important Tips:

Statement 3 is incorrect: Improvement of public health Article 51 of the Constitution of India deals with the
and prohibition of intoxicating drinks, does not directly Promotion of international peace and security. It states
relate to any specific Fundamental Right as enshrined in that the State shall:
the Constitution of India. However, the Constitution does • Promote international peace and security
provide for the promotion of public health as a directive • Maintain just and honourable relations between nations
principle of state policy under Article 47.
• Foster respect for international law and treaty
Statement 4 is incorrect: Promotion of the educational and obligations in the dealings of organised people with
economic interests of the weaker sections of the people, one another
especially the Scheduled Castes and Scheduled is provided
• Encourage settlement of international disputes by
under Article 46 of DPSP.
arbitration
153. Solution: (c) • Adhere to the principles of the United Nations
Exp) Option c is the correct answer. • Strive for international co-operation in solving
Statement 1 is correct: The right to equality, as enshrined international problems of an economic, social,
in Articles 14-18 of the Constitution of India, prohibits cultural, or humanitarian character, and in promoting
discrimination on the basis of religion, race, caste, sex, or and encouraging respect for human rights and for
place of birth (Article 15). It ensures that every citizen of fundamental freedoms for all without distinction as to
India is treated equally before the law. race, sex, language, or religion.

PYQ Workbook 94
INDIAN POLITY AND GOVERNANCE

156. Solution: (d)


Important Tipss
Exp) Option d is the correct answer. • The Indian Constitution’s Fundamental Duties draw
inspiration from the former USSR’s Constitution,
Health Emergency is not mentioned as a form of Emergency added through the 42nd Constitutional Amendment
in the Constitution of India. Act, 1976, based on the Swaran Singh Committee’s
recommendation, and are non-justiciable obligations
The Constitution provides for three types of emergencies: applicable to citizens only.
National Emergency (Article 352), State Emergency • List of ten Fundamental Duties:
(President’s Rule- Article 356), and Financial Emergency • To abide by the Constitution and respect its ideals
and institutions.
(Article 360). While the government can take measures
• To cherish and follow the noble ideals that inspired
under existing laws and provisions to address public health the national struggle for freedom.
crises, a specific provision for a “Health Emergency” is not • To uphold and protect the sovereignty, unity, and
integrity of India.
included in the Constitution.
• To defend the country and render national service
157. Solution: (d) when required.
• To promote harmony and the spirit of common
Exp) Option d is the correct answer. brotherhood among all citizens.
Article 19(1) of the Constitution of India includes the rights: • To value and preserve the rich heritage of India’s
composite culture.
(a) to freedom of speech and expression; • To protect and improve the natural environment,
including forests, lakes, rivers, and wildlife.
(b) to assemble peaceably and without arms;
• To develop scientific temper, humanism, and the
(c) to form associations or unions; spirit of inquiry and reform.
• To safeguard public property and abjure violence.
(d) to move freely throughout the territory of India;
• To strive for excellence in all spheres of individual
(e) to reside and settle in any part of the territory of India; and collective activity.
and omitted
159. Solution: (d)
(f ) to practise any profession, or to carry on any occupation, Exp) Option d is the correct answer.
trade or business The term “Audi Alteram Partem” is a Latin phrase that
Important Tips translates to “hear the other side.” It is a fundamental
principle of natural justice, which states that no person
Reasonable restrictions by the state in India include:
should be condemned or punished without being given a
• Freedom of speech and expression can be restricted fair opportunity to present their case. It is a crucial aspect
for reasons such as national security, public order, of due process and ensures fairness and impartiality in legal
defamation, and incitement to offense. proceedings.
• Restrictions on the freedom of assembly can be imposed
to maintain public order and protect the integrity of 160. Solution: (b)
the nation. Exp) Option b is the correct answer.
• Freedom of association can be limited to ensure public A joint sitting of both Houses of Parliament is not allowed
order, national integrity, and morality. in the case of a deadlock over the passage of a constitutional
• The freedom of movement and residence can be amendment bill. However, in the case of an ordinary bill,
restricted in the interest of public welfare and there is a provision for a joint sitting if needed.
protection of marginalized groups. Important Tipss
• Certain professions involving immoral or dangerous • The procedure for amending the Indian Constitution
activities may face restrictions to maintain social order was borrowed from the South Africa Constitution.
and public safety.
• Article 368 in Part XX of the Constitution deals with
158. Solution: (c) the powers of Parliament to amend the Constitution
and its procedure.
Exp) Option c is the correct answer.
• “Parliament can amend the constitution provided that
To value and preserve a unitary national culture is not Basic Structure remains intact”- Kesvananda Bharti
a Fundamental Duty as mentioned in Part-IVA of the Case 1973.
Constitution of India. However, the other options, are all • Parliament can only initiate constitutional amendment
mentioned as duties of every citizen of India. These duties bill.
aim to promote the welfare of society, gender equality, and • It can be introduced by either minister or private
the pursuit of knowledge and progress. member- does not require president approval.

95 PYQ Workbook
INDIAN POLITY AND GOVERNANCE

• The bill must be passed by each house by Special 164. Solution: (c)
Majority. Exp) Option c is the correct answer
• The President must give his assent- Can not return or The Right to Property was originally included as a
withhold assent of the constitutional amendment bill. fundamental right under Article 19(1)(f ) of the Indian
• Three ways in which constitution can be amended: Constitution, which guaranteed the right to acquire,
• Simple Majority of the parliament. hold, and dispose of property. However, through the 44th
• Special Majority of the Parliament. Amendment Act, passed in 1978, the Right to Property was
• Special Majority of the Parliament and the removed as a fundamental right and was instead reclassified
ratification of half of the state legislatures as a legal right.

161. Solution: (c) Important Tipss


Some other changes under 44th Amendment Act:
Exp) Option c is the correct answer
• Proclamation of Emergency: Proclamation of
Option a is incorrect: Article 21A guarantees free and
Emergency can be issued only when the security of
compulsory education to all children in the age group of six
India or any part of its territory is threatened by war
to fourteen years, not eleven years.
or external aggression or by armed rebellion. Internal
Option b is incorrect: While Article 26 grants religious disturbance not amounting to armed rebellion would
denominations the right to manage their own affairs in not be a ground for the issue of a Proclamation. An
matters of religion, this power is not absolute. It is subject emergency can be proclaimed only on the basis of
to public order, morality, and health, and is also subject to written advice tendered to the President by the Cabinet.
other provisions of the Constitution, including fundamental • Judicial Review: The amendment reinforced the power
rights. of judicial review by stating that Parliament cannot
Option c is correct: Article 30 of the Constitution of India amend the Constitution in a way that would take away
guarantees the right to establish and administer educational or abridge the basic structure of the Constitution.
institutions to religious and linguistic minorities. It states • Fundamental Rights during Emergency: The
that all religious and linguistic minorities have the right to amendment clarified that certain fundamental rights,
establish and administer educational institutions of their including the right to life and personal liberty, cannot
choice, whether aided or unaided by the state. be suspended even during a state of emergency.
• Restored the original terms of Lok sabha and the state
Option d is incorrect: Parliament has the power to empower
legislative assemblies (i.e. 5 years).
other courts, besides the Supreme Court, to issue writs. The
power to issue writs is not exclusive to the Supreme Court 165. Solution: (c)
and can be conferred on high courts or other courts by law.
Exp) Option c is the correct answer
162. Solution: (d) Statement 1 is incorrect: Freedom of speech, as guaranteed
Exp) Option d is the correct answer by the Constitution of India, is available only to citizens of
The Article 21A, which guarantees the Right to Education, India under Article 19.
was inserted into the Constitution of India through the Statement 2 is correct: Protection against self-
86th Amendment Act. This amendment added Article 21A, incrimination is available to both citizens and non-citizens
which states that the State shall provide free and compulsory under Article 20.
education to all children in the age group of six to fourteen Statement 3 is correct: Freedom of conscience is available
years.
to both citizens and non-citizens under Article 25. Freedom
163. Solution: (a) of conscience protects an individual’s right to profess,
Exp) Option a is the correct answer practice, or propagate any religion or belief.

The guarantee of equality before the law or equal protection Statement 4 is incorrect: Non-discrimination in matters
of the laws within the territory of India is provided under of employment is specifically guaranteed only to citizens of
Article 14 of the Constitution of India. Article 14 of the India under Article 16. .
Indian Constitution states that “The State shall not deny to Important Tips:
any person equality before the law or the equal protection
Fundamental Rights Available Only to Citizens of India
of the laws within the territory of India.” This article
• Article 15 – Prohibition of discrimination on grounds
forms a fundamental right and ensures that all individuals,
of religion, race, caste, sex or place of birth.
irrespective of their caste, religion, gender, race, or place of
birth, are treated equally and without discrimination under • Article 16 – Equality of opportunity in matters of
the law. public employment.

PYQ Workbook 96
INDIAN POLITY AND GOVERNANCE

• Article 19 – Protection of six rights related to freedom – 168. Solution: (a)


(a) of speech and expression; (b) to assemble peaceably Exp) Option a is the correct answer.
and without arms; (c) to form associations or unions; The Constitution of India was amended by the Eighty-
(d) to move freely throughout the territory of India; (e) sixth Amendment Act of 2002 (and not the Eighty-fifth
to reside and settle in any part of the territory of India; Amendment Act) to include Article 21A, which mandates
and (f ) to practice any profession, or to carry on any
that every state must ensure free and compulsory education
occupation, trade or business.
for children aged six to fourteen. This amendment recognizes
• Article 29 – Protection of language, script and culture education as a Fundamental Right protected under Part III
of minorities. of the Constitution.
• Article 30 – Right of minorities to establish and
administer educational institutions. 169. Solution: (b)
Exp) Option b is the correct answer.
166. Solution: (a)
Statement 1 is correct- The procedure for amending the
Exp) Option a is the correct answer.
Constitution is provided in Article 368 of the Constitution of
The Constitution of India includes provisions in Part IV India. According to article 368, constitution can be amended
(Article 36-51) known as the Directive Principles of State
in three ways- Simple Majority, Special Majority, Special
Policy (DPSP).
Majority of the Parliament and the ratification of half of the
One such principle is highlighted in Article 50, which state legislatures.
emphasizes the importance of separating the judiciary
Statement 2 is correct- A bill to amend the Constitution
from the executive in the public services of the State. This
can be introduced in either House of Parliament. It can be
provision reflects the commitment to maintaining the
introduced as a Constitutional Amendment Bill and must go
independence and impartiality of the judiciary, ensuring a
through the prescribed legislative process before it can be
fair and just system of governance.
enacted into law.
Important Tipss
Statement 3 is correct- The special procedure outlined in
Classification of Directive Principles of State Policy:
Article 368 of the Constitution vests constituent powers upon
• Socialistic Principles - Article 38, 39, 39A, 41, 42, 43,
43A, 47 the ordinary legislation. This means that the Parliament,
• Gandhian Principles- Article 40, 43, 43B, 46, 47, 48 while acting in its legislative capacity, has the power to amend
• Liberal-Intellectual Principles- Article 44, 45, 48, or modify the provisions of the Constitution provided that
48A, 49, 50, and 51. Basic Structure of the Indian Constitution remains intact.
The Constitution does not contain any classification of
Directive Principles. It is classified on the basis of their 170. Solution: (c)
content and direction Exp) Option c is the correct answer.
167. Solution: (d) A national emergency in India can be declared based on the
Exp) Option d is the correct answer. grounds of war, external aggression, or armed rebellion. The
term “armed rebellion” was introduced through the 44th
Article 12 of the Constitution of India defines the term
amendment to the Constitution. Prior to this amendment, it
‘State’ for the purpose of interpreting fundamental rights.
was referred to as “internal disturbance.”
According to Article 12, the term ‘State’ includes not only
the Government of India and the Parliament of India but also Important Tips:
the Governments and Legislatures of each State. • A national emergency in India can be proclaimed
Important Tips: under Article 352 of the Constitution during situations
of war, external aggression, or armed rebellion that
According to Article 12, ‘State’ includes:
poses a threat to the country’s security and integrity.
• Government and Parliament of India- executive and
• The President of India has the authority to declare a
legislative organs of the Union government.
national emergency based on the advice of the Union
• Government and legislature of states- executive and Cabinet.
legislative organs of state government.
• Once a national emergency is proclaimed, the
• All Local Authorities- municipalities, panchayats, central government assumes additional powers, and
district boards, improvement trusts, etc. the federal structure of the country undergoes a
• All Other Authorities- statutory or non-statutory temporary change.
authorities like LIC, ONGC, SAIL, etc. • Fundamental rights guaranteed by the Constitution can
• A private body or an agency working as an instrument be suspended or curtailed during a national emergency,
of the State falls within the meaning of the ‘State’ under except for certain rights mentioned in Article 20 and
Article 12. Article 21.

97 PYQ Workbook
INDIAN POLITY AND GOVERNANCE

• The proclamation of a national emergency must be and ensure that every child in India has access to quality
approved by both houses of Parliament within one education during the crucial formative years of their life.
month and is valid for six months. It can be extended Important Tips:
with the approval of Parliament in subsequent intervals
• Article 21 A (Fundamental Right)- The State shall
of six months.
provide free and compulsory education to all children
171. Solution: (d) of the age of six to fourteen years in such manner as
the State may, by law, determine.
Exp) Option d is the correct answer.
• Article 45 (Directive Principle of State Policy)- The
Freedom of speech and expression in India is not absolute and
State shall endeavor to provide early childhood care
has reasonable restrictions. Restrictions include defamation,
and education for all children until they complete the
indecency, morality, and incitement to offense. These
age of six years.
limitations aim to protect reputation, maintain decency, and
• Article 51 A (Fundamental Duties) - Parent or guardian to
prevent illegal activities. The Constitution guarantees this
provide opportunities for education to his child or, as the case
fundamental right while allowing necessary limitations for
may be, ward between the age of six and fourteen years.
public safety.
174. Solution: (a)
172. Solution: (a)
Exp) Option a is the correct answer.
Exp) Option a is the correct answer
Article-20 of the Constitution of India protects a person
Articles 31A and 31C are exceptions to the Fundamental
against double jeopardy. Double jeopardy refers to the
Rights enumerated in Article 14 and Article 19, as they allow
principle that an individual cannot be prosecuted or
for certain restrictions on equality and property rights in
punished more than once for the same offense. The article
specific contexts for the purpose of agrarian reforms and
states three specific protections:
socioeconomic welfare.
• No Double Jeopardy.
Important Tips: • No ex-post facto law.
Article 31A: It saves five categories of laws from being • No self-incrimination.
challenged and invalidated on the ground of contravention
of the fundamental rights conferred by Article 14 and Important Tips:
Article 19. It includes: Double Jeopardy- Double jeopardy protection is
• Acquisition of estates and related rights by the State; applicable only in legal proceedings before a court or
• Taking over the management of properties by the State; judicial tribunal. It does not apply to proceedings before
departmental or administrative authorities since they are
• Amalgamation of corporations; not of a judicial nature.
• Extinguishment or modification of rights of directors No ex-post facto law- An ex-post-facto law refers to a
or shareholders of corporations law that applies punishments or increases penalties for
• Extinguishment or modification of mining leases. actions that have already been committed in the past. In
It also provides the guaranteed right to compensation in other words, it is a law that is enforced retroactively. It is
case of acquisition or requisition of the private property applicable only to criminal laws.
by the state. No Self-Incrimination- It means no person accused of any
Article 31C: It contained two provisions: offence shall be compelled to be a witness against himself.
• It says that no law that seeks to implement socialistic The protection against self-incrimination extends to both
directive principles specified in Articles 39 (b) and (c), oral evidence and documentary evidence. It extends only
shall be declared void on the grounds of contravention to criminal proceedings. However, it does not extend to
of the fundamental rights conferred by Article 14 or (i) compulsory production of material objects,
Article 19. (ii) compulsion to give thumb impression, specimen
• Moreover, no law containing a declaration that it is for signature, blood specimens, and
giving effect to such policy shall be questioned in any (iii) compulsory exhibition of the body.
court on the ground that it does not give effect to such
a policy. 175. Solution: (c)
Exp) Option c is the correct answer.
173. Solution: (b)
During the debates of the Constituent Assembly, Dr. B.R.
Exp) Option b is the correct answer. Ambedkar, the principal architect of the Indian Constitution,
The Constitution (86th Amendment) Act, 2002 added the defended Article-356 on the plea that it would be used as “a
fundamental duty of providing opportunities for education matter of last resort.” He argued that this provision would
to one’s child between the age of 6 and 14 years. This only be invoked in exceptional circumstances when all
amendment aimed to emphasize the importance of education other remedies and alternatives have been exhausted. The

PYQ Workbook 98
INDIAN POLITY AND GOVERNANCE

intention behind Article-356 was to ensure the maintenance • Defended and guaranteed by the Supreme Court.
of constitutional order and stability in the country by
• There are six fundamental rights recognised by the
providing a mechanism to address critical situations in the
Indian constitution:
states.
• Right to Equality (Article 14-18)
Important Tips: • Right to Freedom (Article 19-22)
• President’s Rule, also known as State Emergency, is the • Right against Exploitation (Article 23-24)
suspension of the state government and the imposition
• Right to Freedom of Religion (Article 25-28)
of direct central government rule in a state of India.
• Cultural and Educational Rights (Article 29-30)
• It is implemented under Article 356 of the Indian
Constitution when the President deems that the • Right to Constitutional Remedies (Article 32)
state’s governance cannot be carried out according to NOTE: Originally, there were 7 Fundamental Rights
constitutional provisions. enshrined in the constitution. However, Right to
• The decision to impose President’s Rule is taken by Property was deleted as a fundamental right through the
the President on the advice of the Governor of the 44th Amendment Act, 1978. It was made a legal right
concerned state or the Union Cabinet. (Article 300A).
• The state legislative assembly is either dissolved or 178. Solution: (c)
kept under suspended animation during President’s
Exp) Option c is the correct answer.
Rule.
The concept of basic structure was laid down by the Supreme
• President’s Rule can be imposed for a maximum period
Court in the Keshvananda Bharti Case (1973).
of six months, and its extension beyond six months
requires parliamentary approval every six months. The basic structure doctrine states that the Constitution
of India has certain basic features that cannot be altered or
• Currently, Manipur is the only state with highest
destroyed through amendments by the Parliament Article
number (i.e. 10) of President’s Rule so far. Uttar
368 does not give absolute powers to the Parliament to
Pradesh has 9 President Rule so far, Bihar and Punjab
amend any part of the Constitution.
both has 8 President Rule as of today.
It can’t be exercised by the people through representatives
176. Solution: (c) in a constituent assembly (Statement 2 is incorrect)
Exp) Option c is the correct answer. It falls outside the constituent powers of the parliament.
Under Article-32 of the Constitution of India, individuals (Statement 3 is incorrect)
have the right to move the Supreme Court for the enforcement 179. Solution: (b)
of their Fundamental Rights. However, there are certain
Exp: Option b is the correct answer.
grounds on which the Supreme Court can refuse relief under
Article-32. One such ground is when it is determined that no Under article 21 of the Indian constitution, women’s
reproductive choice falls under the right to personal
Fundamental Right has been infringed in the case.
liberty.
177. Solution: (b) The right to personal liberty also includes women’s
Exp) Option b is the correct answer reproductive choice the same was held in the case of suchitra
Article 15(2) of the constitution No citizen shall, on srivastava and ors V.chadi. (option 3 is not correct)
grounds only of religion, race, caste, sex, Article 21 of the constitution deals with the protection of life
and personal liberty.
place of birth or any of them, be subject to any disability,
liability, restriction or condition with regard to: No person shall be deprived of his life or personal liberty
according to the procedure established by law. (option 1 is
• Access to shops, public restaurants, hotels and palaces of
correct).
public entertainment; or
• The use of wells, tanks, bathing Ghats, roads and places 180. Solution: (a)
of public resort maintained wholly or partly out of Exp) Option a is the correct answer.
State funds or dedicated to the use of the general public • According to article 28 of the Indian constitution
(option b is correct) Freedom as to attendance at religious instruction or
religious worship in certain educational institutions.
Important Tips:
• No religion instruction shall be provided in any
• Referred to as the “Magna Carta” of India
educational institution wholly maintained out of State
• Fundamental Rights have been mentioned in the Part funds.
III of the constitution from Article 12 to 35.
• Nothing in clause ( 1 ) shall apply to an educational
• They are justiciable in nature. institution which is administered by the State but has

99 PYQ Workbook
INDIAN POLITY AND GOVERNANCE

been established under any endowment or trust which • Right to personal liberty
requires that religious instruction shall be imparted in
such institution. 184. Solution: (c)

• No person attending any educational institution Exp) Option c is the correct answer.
recognised by the State or receiving aid out of State funds Every person, who is arrested and detained in custody shall
shall be required to take part in any religious instruction be produced before the nearest Magistrate within a period of
that may be imparted in such institution or to attend twenty-four hours of such arrest excluding the time necessary
any religious worship that may be conducted in such for the journey from the place of arrest to the court of the
institution or in any premises attached thereto unless Magistrate and no such person shall be detained in custody
such person or, if such person is a minor, his guardian has beyond the said period without the authority of a Magistrate.
given his consent thereto. (statement a is not correct)
Important Tips:
181. Solution: (c) • Protection against arrest and detention in certain
Exp) Option c is the correct. cases: Article 22
According to article 30(2) in the Indian constitution the • Article 22 comes into play after arrest or detention, not
state shall not, in granting aid to educational institutions, before that.
discriminate against any educational institution on the • Detention is of two types, namely, punitive and
ground that it is under the management of a minority, preventive.
whether based on religion or language.
• Punitive detention is to punish a person for an offence
182. Solution: (a) committed by him after trial and conviction in a court.
Exp) Option a is the correct answer. • Preventive detention means detention of a person
• Fundamental rights can be suspended during a national without trial and conviction by a court, Its purpose is
emergency. not to punish a person for a past offense but to prevent
him from committing an offence in the near future.
• But, the rights guaranteed under Articles 20 and 21
cannot be suspended. 185. Solution: (b)
Important Tips: Exp) Option b is the correct answer.
• Fundamental Rights are justifiable, allowing persons Article 19(1) (c) Right to Form Associations
to move the courts for their enforcement, if and when All citizens have right to form associations or unions or co-
they are violated. operative societies (Included through 97th constitutional
• They are defended and guaranteed by the Supreme amendment, 2011).
Court. Hence, the aggrieved person can directly go to Subject to reasonable restrictions on grounds of sovereignty
the Supreme Court, not necessarily by way of appeal
and integrity of India, public order and morality.
against the judgment of the high courts.
However, right to obtain recognition of the association is not
• They are not sacrosanct or permanent. The Parliament
a fundamental right.
can curtail or repeal them but only by a constitutional
amendment act and not by an ordinary act Moreover, In view of Supreme Court, Right to strike is a statutory right
this can be done without affecting the ‘basic structure’ (controlled by industrial laws) and not a fundamental right.
of the Constitution. Article-19(4) states that the State may impose reasonable
• They can be suspended during the operation of a restrictions on this right in the interest of public order
National Emergency except the rights guaranteed by morality, sovereignty and integrity of India.
Articles 20 and 21. Article 21 No person shall be deprived of his life or personal
liberty except according to procedure established by law.
183. Solution: (d)
Exp) Option d is the correct answer. 186. Solution: (a)
• The article 21 includes all the three mentioned above: Exp) Option a is the correct answer.
• Rights of transgender The basic structure doctrine was given in the landmark
• Rights of craniopagus twins decision of Kesavananda Bharati v State of Kerala (1973).
• Rights of mentally retarded women to bear a child The basic structure doctrine states that the Constitution
of India has certain basic features that cannot be altered or
• Article 21 states that “Protection of life and personal
liberty No person shall be deprived of his life or personal destroyed through amendments by The Parliament.
liberty except according to procedure established by law” The Supreme Court can undertake judicial review for such
• Right to life: Everyone has the right to life, liberty and amendments.
security of a person according to Procedure established The Supreme Court declared that Article 368 did not enable
by law. Parliament to alter the basic structure or framework of the

PYQ Workbook 100


INDIAN POLITY AND GOVERNANCE

Constitution and Parliament could not use its amending • Fundamental Rights available to both citizens and
powers under Article 368 alter the basic structure of the foreigners except enemy alliens:
Constitution. Thus, statement 3 is correct.
• Equality before law and equal protection of laws
In the Golaknath case, the supreme court ruled that (Article 14).
Parliament couldn’t curtail any of the Fundamental • Protection in respect of conviction for offences
rights. (this case is not related to basic structure of the (Article 20).
constitution).Thus, statement 4 is incorrect
• Protection of life and personal liberty (Article 21).
187. Solution: (a) • Right to elementary education (Article 21A).
Exp) Option a is the correct answer. • Protection against arrest and detention in certain
Article 20 (part III) of the constitution contains protection cases (Article 22).
in respect for conviction for offences. • Prohibition of traffic in human beings and forced
(a) No person shall be convicted of any offence except for labour (Article 23).
violation of the law in force at the time of the commission • Prohibition of employment of children in factories
of the act charged as an offence, nor be subjected to a etc. (Article 24).
penalty greater than that which might have been inflicted • Freedom of conscience and free profession, practice
under the law in force at the time of the commission of and propagation of religion (Article 25).
the offence. • Freedom to manage religious affairs (Article 26).
(b) No person shall be prosecuted and punished for the same • Freedom from payment of taxes for promotion of
offence more than once any religion (Article 27).
(c) No person accused of any offence shall be compelled to • Freedom from attending religious instruction or
be a witness against himself. worship in certain educational institutions (Article
28).
Under section 25 of the Indian Evidence Act, a confession
to a Police officer is inadmissible in evidence. 189. Solution: (a)
Although Judicial Confession under Section 80 of the Exp) Option a is the correct answer.
Indian Evidence Act makes judicial confessions admissible, • It gives the Supreme Court and (not the High courts) the
stating that if a confession is made in the presence of a power to issue writs for the enforcement of Fundamental
magistrate or in a court and is registered by the magistrate Rights. (option a is not correct)
as required by law, the confession is presumed to be valid • Article-32 confers the right to remedies for the
and genuine, and the accused may be tried as per law. enforcement of the Fundamental Rights of an
(statement a is incorrect) aggrieved citizen.
188. Solution: (b) • Dr BR Ambedkar called Article-32 as the most
important Article of the Constitution: “It is the very
Exp) Option b is the correct answer.
Soul of the Constitution and the very heart of it”
The Fundamental Rights guaranteed by Articles 14,20,21,
• Only the Fundamental Rights guaranteed by the
21A, 22, 23, 24, 25, 26, 27 and 28 are available to all persons
Constitution can be enforced under Article 32 and not
whether citizens or foreigners. any other right like non-fundamental Constitutional
Important Tips rights, statutory rights and so on.
• Fundamental Rights available to only citizens and • Article-226 vests original powers in the High Court
not foreigners to issue directions, orders and writs of all kinds for the
• Prohibition of discrimination on grounds of enforcement of the Fundamental Rights.
religion, race, caste, sex or place of birth (Article 190. Solution: (b)
15).
Exp) Option b is the correct answer.
• Equality of opportunity in matters of public
• As per article 25(1) every person is guaranteed freedom
employment (Article 16).
of Conscience and freedom to profess practice,propagate
• Six basic freedoms subject to reasonable restrictions
in own religion subject to public order, morality and
(Article 19).
health
• Protection of language, script and culture of
• As per article 25(2) (b) It allows the state to provide
minorities (Article 29).
for social welfare and reform or the throwing open of
• Right of minorities to establish and administer Hindu religious institutions of public characters to all
educational institutions (Article 30). classes and sections of Hindus

101 PYQ Workbook


INDIAN POLITY AND GOVERNANCE

191. Solution: (c) • Habeas Corpus- It is a Latin term which literally means
Exp) : Option c is the correct answer. ‘to have the body of ’.
• Writ of habeas corpus is also known as a writ of liberty • Mandamus- It literally means ‘we command’.
• A writ of habeas corpus is issued for release of a person • Prohibition- It means ‘to forbid’.
who has been detained unlawfully by the State or by • Certiorari- It means ‘to be certified’ or ‘to be informed’
any private individual.
• Quo-Warranto- It means ‘by what authority or warrant’
• A writ of habeas corpus is not issued if the person
• Mandamus- It literally means ‘we command’.
concerned is not in unlawful detention.
• Prohibition- It means ‘to forbid’.
Important Tips:
• Certiorari- It means ‘to be certified’ or ‘to be informed’
• Article 32 confers the right to remedies for the
• Quo-Warranto- It means ‘by what authority or warrant’
enforcement of the fundamental rights of an aggrieved
citizen.
192. Solution: (a)
• Article 226 High Courts can also issue writs.
• The Parliament (under Article 32) can empower any Exp) Option a is the correct answer
other court to issue these writs. Members of Armed Forces are not treated at par so far as the
• Meaning of various Writs: availability of Fundamental Rights is concerned.

PYQ Workbook 102


INDIAN POLITY AND GOVERNANCE

INDIAN POLITY AND GOVERNANCE


SYSTEM OF GOVERNMENT
*This unit consists of questions from Parliamentary system, Federal system, Central-State relations
and Inter-State relations.

3.1. UPSC CSE Previous Years’ Questions (d) A society permanently living in a definite
territory with a central authority, an
1. We adopted parliamentary democracy executive responsible to the central
based on the British model, but how does authority and an independent judiciary.
our model differ from that model? 4. Which one of the following factors
1. As regards legislation, the British constitutes the best safeguard of liberty in
Parliament is supreme or sovereign but a liberal democracy? [UPSC CSE Pre. 2021]
in India, the power of the Parliament to (a) A committed judiciary
legislate is limited. (b) Centralization of powers
2. In India, matters related to the (c) Elected government
Amendment of an Act of the Parliament (d) Separation of powers
are referred to the Constitution Bench by 5. Which one of the following suggested that
the Supreme Court. the Governor should be an eminent person
Select the correct answer using the code given from outside the State and should be a
below: [UPSC CSE Pre 2021] detached figure without intense political
(a) 1 only links or should not have taken part in
(b) 2 only politics in the recent past?
(c) Both 1 and 2 [UPSC CSE Pre. 2019]
(d) Neither 1 nor 2 (a) First Administrative Reforms
2. Which one of the following in Indian polity Commission (1966)
(b) Rajamannar Committee (1969)
is an essential feature that indicates that it
(c) Sarkaria Commission (1983)
is federal in character? (d) National Commission to Review the
[UPSC CSE Pre. 2021] Working of the Constitution (2000)
(a) The independence of judiciary is
safeguarded. 6. If the President of India exercises his
power as provided under Article 356 of the
(b) The Union Legislature has elected
Constitution in respect of a particular state,
representatives from constituent units.
then [UPSC CSE Pre. 2018]
(c) The Union Cabinet can have elected
(a) the Assembly of the State is automatically
representatives from regional parties.
dissolved.
(d) The Fundamental Rights are enforceable
(b) the powers of the Legislature of that
by Courts of Law. state shall be exercisable by or under the
3. Which one of the following best defines the authority of the Parliament.
term ‘State’? [UPSC CSE Pre. 2021] (c) Article 19 is suspended in that State.
(a) A community of persons permanently (d) the President can make laws relating to
occupying a definite territory independent that state.
of external control and possessing an 7. The main advantage of the parliamentary
organized government. form of governments is that:
(b) A politically organized people of a definite [UPSC CSE Pre. 2017]
territory and possessing an authority to (a) the executive and legislature work
govern them, maintain law and order, independently.
protect their natural rights and safeguard (b) it provides continuity of policy and is
their means of sustenance. more efficient.
(c) A number of persons who have been (c) the executive remains responsible to the
living in a definite territory for a very long legislature.
time with their own culture, tradition and (d) the head of the government cannot be
government. changed without election.

103 PYQ Workbook


INDIAN POLITY AND GOVERNANCE

8. The Parliament of India acquires the power (a) Centre’s control of the state in the
to legislate on any item in the State List in legislative sphere.
the national interest if a resolution to that (b) Centre’s control of the state in financial
effect is passed by the matters.
[UPSC CSE Pre. 2016] (c) Centre’s control of the state in the
(a) Lok Sabha by a simple majority of its total administrative sector.
membership. (d) Centre’s control of the state in the
(b) Lok Sabha by a majority of not less than planning process.
two-thirds of its total membership. 14. Which one of the following determines that
(c) Rajya Sabha by a simple majority of its Indian Constitution is Federal?
total membership. [UPSC CSE Pre 1994]
(d) Rajya Sabha by a majority of not less than
(a) A written and rigid Constitution
two-thirds of its members present and
(b) An independent Judiciary
voting.
(c) Vesting of residuary powers with the
9. The Parliament can make any law for whole Centre.
or any part of India for implementing (d) Distribution of powers between union
international treaties and states.
[UPSC CSE Pre. 2013]
(a) with the consent of all the states 3.2. Other Examination Previous Years’
(b) with the consent of the majority of states Questions
(c) with the consent of the states concerned 15. Given below are two statements, one is
(d) without the consent of any state labelled as Assertion (A) and the other as
10. Consider the following statements: Reason (R):
In India, stamp duty on financial transactions Assertion (A): The nature of Indian Political
are System embodies the elements of ‘continuity
1. imposed and collected by state and change’.
government. Reason (R): The Indian Political System
2. regulated by the union government. contains the elements of the following
Which of the following is/are correct? political system such as the modern style, the
traditional style and saint style.
[UPSC CSE Pre 2003]
(a) only 1 Select the correct answer from the codes
(b) only 2 given below. [U.P.P.C.S. (Pre) 2020]
(c) both 1 and 2 (a) Both (A) and (R) are true and (R) is the
(d) none of these correct explanation of (A).
(b) Both (A) and (R) are true but (R) is not
11. The Indian Parliamentary System is the correct explanation of (A).
different from the British Parliamentary (c) (A) is true but (R) is false.
System in that India has: (d) (A) is false but (R) is true.
[UPSC CSE Pre 1998]
16. Given below are two statements, one is
(a) Both a real and a nominal executive
labelled as Assertion (A) and the other as
(b) A system of collective responsibility
Reason (R):
(c) Bicameral legislature
(d) The system of judicial review Assertion (A): Indian federalism is called
‘Quasi-federal’.
12. Agriculture Income Tax is assigned to the
Reason (R): India has an independent
State Government by [UPSC CSE Pre 1995]
judiciary with power of Judicial Review.
(a) The Finance Commission
(b) The National Development Council Select the correct answer from the codes
given below. [U.P.P.C.S. (Pre) 2020]
(c) The Inter-State Council
(d) The Constitution of India (a) Both (A) and (R) are true and (R) is the
correct explanation of (A).
13. In which respect have the Centre-state (b) Both (A) and (R) are true but (R) is not
relations been specifically termed as the correct explanation of (A).
municipal relation - (c) (A) is true but (R) is false.
[UPSC CSE Pre 1994] (d) (A) is false but (R) is true.

PYQ Workbook 104


INDIAN POLITY AND GOVERNANCE

17. With reference to the federal system in (c) Chandigarh although not a state, is
India, which of the statements is/are included in Zonal Council
correct? (d) It is an Advisory body
1. States have no right to secede from the 22. Which one of the following is not related to
Union under the constitution of India. Union-State relations in India?
2. Just advocacy of secession will have the [U.P.P.C.S. (Pre) 2015]
protection of freedom of expression. (a) Sarkaria Commission
Select the correct answer from the codes (b) Rajmannar Committee
given below: [U.P.P.C.S. (Pre) 2019] (c) Indrajit Gupta Committee
(a) 1 only (d) Punchhi Commission
(b) 2 only
(c) Both 1 and 2 23. Which of the following is correct?
(d) Neither 1 nor 2 [U.P.P.C.S. (Mains) 2014]
The main feature of a Presidential form of
18. Of the following political features, which Government is/are –
is/are common to India and the USA?
(a) The Head of the Executive is the President.
1. Residuary powers vested in the centre. (b) The President appoints his Council of
2. Residuary powers vested in the states. Ministers.
3. President has the power of pocket veto. (c) The President cannot dissolve the
4. The upper house has some nominated Legislature.
members. (d) All of the above.
Code: [U.P.P.C.S. (Pre) 2017] 24. Consider the following statements:
(a) 3 only 1. India is a democratic polity.
(b) 3 and 4 only
2. India is a sovereign state.
(c) 2, 3 and 4 only
(d) 1, 3 and 4 only 3. India has a democratic society.
4. India is a welfare state.
19. With reference to the Sovereignty of India,
Which of the above statements are true?
which of the following statements is/are
correct? [U.P.P.C.S. (Mains) 2012]
1. India is a member of Commonwealth of (a) 1 and 2 only
Nations. (b) 1, 2 and 3 only
(c) 2, 3 and 4 only
2. Sovereignty of India is curtailed due to (d) 1, 2, 3 and 4
its membership of Commonwealth of
Nations. 25. Is a State of the Indian Union empowered
Select the correct answer using the codes under the Constitution to take foreign
given below: [U.P.P.C.S. (Mains) 2017] loans directly? [U.P.P.C.S. (Mains) 2012]
(a) 1 only (a) Yes
(b) No
(b) 2 only
(c) Yes, but from international organizations
(c) Both 1 and 2
only
(d) Neither 1 nor 2 (d) Yes, but only with the permission of the
20. Who among the following is empowered to Reserve Bank of India
establish the Inter-State council? 26. Centre-State relations in India are
[U.P.P.C.S. (Pre) 2017] dependent upon-
(a) Parliament 1. Constitutional provisions
(b) President 2. Conventions and practices
(c) Speaker of Lok Sabha
3. Judicial interpretations
(d) Prime Minister
4. Mechanisms for dialogue
21. Which one of the following is not a feature Select the correct answer from the codes
of a zonal council? given below: [U.P.P.C.S. (Pre) 2009]
[U.P.P.C.S. (Pre) 2017] (a) 1 and 2
(a) It is a constitutional body (b) 1, 2 and 3
(b) Five Zonal Councils have set up under (c) 2, 3 and 4
the State Reorganization Act, 1956 (d) All the four

105 PYQ Workbook


INDIAN POLITY AND GOVERNANCE

27. Article 249 of the Indian Constitution deals (a) The relations between the President and
with [U.P.P.C.S. (Mains) 2009] the Prime Minister
(a) Emergency Power of the President (b) The relations between the Legislature and
(b) Dissolution of the Lok Sabha the Executive
(c) Administrative Power of the Parliament (c) The relations between the Executive and
(d) Legislative Powers of the Parliament with the Judiciary
respect to subjects in the State List (d) The relations between the Union and the
States
28. In a Parliamentary system of Government-
33. Consider the following statements-
[U.P.P.C.S. (Mains 2010)]
(a) Judiciary controls Executive In India taxes on transactions in Stock-
(b) Executive controls Judiciary Exchanges and Futures Markets are-
(c) Executive controls Legislature 1. Levied by Union
(d) Legislature controls Executive 2. Collected by the State Government
Code: [U.P.P.C.S. (Mains) 2006]
29. Here are two statements:
(a) Only 1
Assertion (A): Political Parties are lifeblood (b) Only 2
of democracy. (c) Both 1 and 2
Reason (R): Normally people blame political (d) Neither 1 nor 2
parties for bad governance. 34. Which of the following statements about
Which of the following is correct in the above the federal system in India are correct?
context? [U.P.P.C.S. (Pre) 2009] 1. The Constitution introduces a federal
(a) Both (A) and (R) are true, and (R) is the system as the basic structure of
correct explanation of (A). Government.
(b) Both (A) and (R) are true, but (R) is not a 2. There is a strong admixture of a unitary
correct explanation of (A). bias.
(c) (A) is true, but (R) is wrong. 3. Both the Union and State Legislatures are
(d) (A) and (R) both are wrong. sovereign.
30. Which one is the most prominent element 4. The legislative, financial and judicial
of a state? [U.P.P.C.S. (Pre) 2008] powers have been divided between the
(a) Flag Union and its units.
(b) Capital Select the correct answer from the codes
(c) Sovereignty given below: [U.P.P.C.S. (Mains) 2004]
(d) Head of State (a) 1 only
(b) 1 and 2 only
31. Given below are two statements, one is (c) 1, 2 and 3 only
labelled as Assertion (A) and the other as (d) All the four
Reason (R):
Assertion (A): Indian Constitution is quasi- 35. Under which of the following taxes is the
total amount payable by an individual,
federal.
limited by the Constitution?
Reason (R): Indian Constitution is neither [U.P.P.C.S. (Mains) 2004]
federal nor unitary.
(a) Corporation tax
Select the correct answer using the codes (b) Estate Duty
given below: [U.P.P.C.S. (Pre) 2007] (c) Succession Duty
(a) Both (A) and (R) are true, and (R) is the (d) Tax on profession, trade, and callings
correct explanation of (A).
36. What is the source of Inter-State councils
(b) Both (A) and (R) are true, but (R) is not
the correct explanation of (A). [U.P.P.C.S. (Pre) 2000]
(c) (A) is true, but (R) is false. (a) Constitution
(d) (A) is false, but (R) is true. (b) Parliamentary laws
(c) Recommendation of Planning
32. Sarkaria Commission was set up to review- Commission
[U.P.P.C.S. (Mains) 2006] (d) Approved Resolution by CM

PYQ Workbook 106


INDIAN POLITY AND GOVERNANCE

37. The cardinal features of political system in 43. Which of the following institutes are
India are– considered necessary to promote ‘Unity
1. It is a democratic republic. among diversity’ in the Indian Federalism?
2. It has a Parliamentary form of government. [Uttarakhand P.C.S. (Pre) 2016]
3. The supreme power vests in People of (a) Inter-State Councils & National
India. Development Council
(b) Finance Commission & Regional Council
4. It provides for a unified authority. (c) Unitary Judicial System & All India
Select the correct answer from the code given Services
below: [U.P. Lower Sub. (Pre) 2008] (d) All of the above
(a) 1 and 2 only 44. Given below are two statements, one is
(b) 1, 2 and 3 only labelled as Assertion (A) and the other as
(c) 2, 3 and 4 only Reason (R):
(d) All the four Assertion (A): Women, Dalits, Poor and
38. ‘India is a Republic’ means- Minority groups are the biggest stakeholders
[U.P.P.C.S. (Pre) 1997] of Democracy in India.
(a) It is the people who are the final authority Reason (R): Democracy in India has emerged
in all matters. as the carrier for the desire of Self Respect.
(b) There is no Parliamentary System of Select the correct answer from the codes
Government in India. given below. [R.A.S./R.T.S. (Pre) 2013]
(c) There are no hereditary rulers in India. (a) Both (A) and (R) are true and (R) is the
(d) India is the Union of States. correct explanation of (A).
(b) Both (A) and (R) are true but (R) is not
39. The Indian Constitution vests the residuary the correct explanation of (A).
powers with: [U.P.P.C.S. (Pre) 1995] (c) (A) is true but (R) is false.
(a) The Union Government (d) (A) is false but (R) is true.
(b) The State Governments
45. Given below are two statements one is
(c) Both the Union as well as State labelled as Assertion (A) and the other is
Governments labelled as Reason (R).
(d) Neither the Union nor the State
Assertion (A): Sarkaria Commission
Governments
recommended that Article 356 should be
40. Main agency to resolve the fiscal disputes sparingly used.
between the Centre and States is- Reason (R): The political parties that came to
[U.P.P.C.S. (Pre) 1995] form the Government at the Centre misused
(a) Supreme Court Article 356.
(b) Law Minister Code:
(c) Finance Minister [R.A.S./R.T.S. (Pre) (Re. Exam.) 2013]
(d) Finance Commission (a) (R) is true, but (A) is false.
(b) Both (A) and (R) are true, but (R) is not
41. Freedom of inter-state trade and commerce
the correct explanation of (A).
has been regulated by the Indian
(c) Both (A) and (R) are true and (R) is the
Constitution under Article: correct explanation of (A).
[U.P.P.C.S. (Pre) 1999] (d) (A) is true, but (R) is false.
(a) 19(1) (d-e)
46. Given below are two statements, one is
(b) 301
labelled as Assertion (A) and the other as
(c) 301 to 307 Reason (R):
(d) All of the above
Assertion (A): The primary objective of
42. Who among the following chairs the Zonal India’s federal design was to weave a nation
Council [Uttarakhand P.C.S. (Pre) 2021] out of its many diverse parts and protect
(a) Prime Minister of India national integration.
(b) Home Minister of India Reason (R): Accommodation of diversities
(c) Chief Ministers of the zones in rotations has built a stronger, not weaker, Indian
(d) Finance Minister of India nationhood.

107 PYQ Workbook


INDIAN POLITY AND GOVERNANCE

Code: [R.A.S./R.T.S. (Pre) 2013] 52. Which unitary elements are found in Indian
(a) (A) is false, but (R) is true. Federal System?
(b) Both (A) and (R) are individually true (i) Written Constitution
and (R) is the correct explanation of (A).
(ii) Rigid Constitution
(c) Both (A) and (R) are individually true,
but (R) is not the correct explanation of (iii) Supremacy of the Constitution
(A). (iv) Appointment of Governor
(d) (A) is true, but (R) is false. (v) Decentralization of Powers
47. In the Indian Fiscal federal system resources (vi) President’s approval on state bills
are allocated from the centre to the states, (vii) Constitutional Crisis
which of the following is not a means of
resource allocation at present (viii) One Supreme Court
[Jharkhand P.C.S. (Pre) 2021] Code: [Chhattisgarh P.C.S. (Pre) 2017]
(a) Grant-in-aid (a) i, iii, vii
(b) Centrally sponsored schemes (b) ii, v, viii
(c) Transfer for plan implementation under (c) i, ii, v
Gadgil formula (d) iv, vi, vii
(d) Tax sharing (e) None of the above
48. Under constitutional provision of tax 53. Which of the following States/UTs are
assignment in India, which of the following included in the Northern Zonal Council?
is a tax exclusively imposed and collected [CDS 2023 (I)]
by the states? [Jharkhand P.C.S. (Pre) 2021] (a) Uttarakhand, Uttar Pradesh, Haryana,
(a) Income Tax Punjab, Jammu and Kashmir
(b) Land Revenue (b) Haryana, Punjab, Himachal Pradesh,
(c) Corporation Tax Rajasthan and NCT of Delhi
(d) GST (c) Uttar Pradesh, Uttarakhand, Punjab,
49. Under which Article of the Constitution of Haryana and Rajasthan
India, Taxes are levied and collected by the (d) Uttarakhand, Uttar Pradesh, Himachal
centre but distributed between the centre Pradesh, Punjab and Haryana
and the States? 54. Which one of the following is not a
[Jharkhand P.C.S. (Pre) 2016] characteristic feature of Indian Federalism?
(a) Article 268 [CDS 2022 (I)]
(b) Article 269
(c) Article 270 (a) The federating units consented to form a
(d) Article 271 union
(b) Residuary powers vest with the Centre
50. Which Commission advocated the (c) Single citizenship
establishment of a Permanent Inter-State (d) An extensive Union and Concurrent list
Council, called ‘Inter-Governmental
Council’? [Jharkhand P.C.S. (Pre) 2013] 55. M.M. Punchhi Commission on Centre-State
(a) Punchhi Commission Relationship has recommended the disposal
(b) Sarkaria Commission of a bill reserved for the consideration of
(c) Radhakrishnan Commission the Union Executive within [CDS 2021 (I)]
(d) Moily Commission (a) four months
(b) Six months
51. Which Article of the Indian Constitution
(c) Eight months
describes “The Executive power of every
State shall be so exercised as not to impede (d) five months
or prejudice the exercise of the Executive 56. The Inter-State Council was set-up in 1990
power of the Union”? on the recommendation of
[Jharkhand P.C.S. (Pre) 2011] [CDS Pre.2019 I]
(a) Article 257 (a) Punchhi Commission
(b) Article 258 (b) Sarkaria Commission
(c) Article 355 (c) Rajamannar Commission
(d) Article 356 (d) Mungerilal Commission

PYQ Workbook 108


INDIAN POLITY AND GOVERNANCE

57. Which one of the following is not a feature 60. Which one of the following is a characteristic
of Indian federalism? [CAPF 2022] of Presidential form of government?
(a) Courts can interpret the Constitution and [CAPF 2016]
powers different levels of the Government. (a) President is not a part of legislative body
(b) Sources of revenue for the Union (b) It does not separate Legislative and
Government and the State Governments Executive functions
are specified. (c) President follows the principle of
(c) Powers of the Union and the States are collective responsibility
specified in the Constitution. (d) The tenure of the President depends on
(d) Indian federalism is based on the principle the legislature
of Separation of Powers.
61. Which one of the following is not a feature
58. Which of the following is/are source/ of Indian federalism? [CAPF 2016]
sources of Law in India? (a) Every State Government has powers of its
1. Constitution of India own
2. Statutes (b) Courts have the power to interpret the
3. Customary Law constitution and the powers of different
4. Case Law levels of government
(c) States are subordinate to the Central
Select the correct answer using the code given Government
below. [CAPF 2022] (d) Sources of revenue for each level of
(a) 1 only government are clearly specified.
(b) 1, 2 and 4 only
(c) 2 and 3 only 62. The setting up of the Inter-State Council in
(d) 1, 2, 3 and 4 1990 was meant to: [CAPF 2015]
(a) substitute the National Development
59. Match List I with List II and select the Council
correct answer using the codes given below (b) strengthen the federal provisions of the
the lists: Constitution
List I List II (c) be an institutional interface between the
(Concept) (Explanation/Aim) Judiciary and the Government
(d) provide membership to local customary
A. Pluralist 1. Power is always bodies
democracy exercised by a
privileged few 63. Which of the following statements
B. People’s 2. Highest and regarding the residuary powers under the
democracy harmonious Constitution of India is/are correct?
development 1. Residuary powers have been given to the
of individual Union Parliament.
capabilities 2. In the matter of residuary powers,
C. Developmental 3. Social equality the Constitution of India follows the
democracy through common Constitution of Australia.
ownership of 3. Schedule 7 of the Constitution of India
wealth provides a list of residuary powers.
D. Elitist 4. Capacity of 4. The Government of India Act, 1935
democracy groups to ensure placed residuary powers in the hands of
Governmental the Governor-General.
responsiveness Select the correct answer using the codes
Codes: [CAPF 2020] given below. [CAPF 2014]
A B C D Codes:
(a) 4 3 2 1 (a) 1 and 3
(b) 4 2 3 1 (b) 2 and 3
(c) 1 2 3 4 (c) 1 and 4
(d) 1 3 2 4 (d) Only 4

109 PYQ Workbook


INDIAN POLITY AND GOVERNANCE

64. Which of the following statements about (a) suits and proceedings
the Presidential system is/are correct? (b) government contracts
1. The Head of Government is also the Head (c) Attorney General
of State. (d) restriction on trade and commerce
2. The Executive can veto Legislative acts. (e) None of the above/More than one of the
Select the correct answer using the codes above
given below. 66. The Democracy of India is based on the fact
Codes: [CAPF 2014] that– [39th B.P.S.C. (Pre) 1994]
(a) Only 1 (a) The Constitution is written
(b) Only 2 (b) Fundamental rights have been provided
(c) Both 1 and 2 (c) Public enjoys the rights to choose and
(d) Neither 1 nor 2 change the Government
65. Article 300 of the Indian Constitution deals (d) Directive principles of state policy have
with- [67th B.P.S.C. (Pre) 2022] been provided here

PYQ Workbook 110


INDIAN POLITY AND GOVERNANCE

SOLUTIONS

3.1. UPSC CSE Previous Years’ Questions • Written Constitution


• Division of powers between the Centre and the states
1. Solution: (c)
(by Schedule VII)
Exp) Option c is the correct answer.
• Supremacy of the Constitution
Statement 1 is correct. Parliamentary sovereignty is a
• Rigidity of the constitution
principle of the UK constitution. It makes Parliament the
supreme legal authority in the UK, which can create or end • Bicameral legislature consisting of Rajya Sabha and
any law. Generally, the courts cannot overrule its legislation Lok Sabha
and no Parliament can pass laws that future Parliaments • Independency of judiciary
cannot change.
3. Solution: (a)
Indian Parliament is not a sovereign body like the British
Parliament. The Indian Parliament may, in exercise of its Exp) Option a is the correct answer.
constituent power, amend by way of addition, variation or State is “a community of persons permanently occupying
repeal any provision of the Constitution in accordance with a definite portion of territory, independent of external
the procedure laid down for the purpose. However, the control, and possessing an organized government”
Parliament cannot amend those provisions which form the According to Max Weber, a State is a human community that
‘basic structure’ of the Constitution. This was ruled by the
(successfully) claims the monopoly of the legitimate use of
Supreme Court in the Kesavananda Bharati case (1973).
physical force within a given territory.
Statement 2 is correct. In India, matters related to the
constitutionality of an amendment of an act of the Important Tips
Parliament are referred to the Constitution Bench by the The state possesses a government which has the authority
Supreme Court. to enforce a system of rules over the people living inside
it. That system of rules is commonly composed of a
Important Tips
constitution, statutes, regulations, and common law. It is
A Constitution Bench is a bench of the Supreme Court independent of external control.
having five or more judges on it. These benches are not a
Its mains functions can be maintaining law, order and
routine phenomenon. A vast majority of cases before the
stability, resolving various kinds of disputes through the
Supreme Court are heard and decided by a bench of two
legal system, providing common defence. It also looks
judges (called a Division Bench), and sometimes of three.
out for the welfare of the population in ways that are
Constitution Benches are set up when the case involves a
beyond the means of the individual, such as implementing
substantial question of law pertaining to the interpretation
public health measures, providing mass education and
of the Constitution (Article 145(3) of the Constitution,
underwriting expensive medical research.
which mandates that such matters be heard by a bench of
not less than five judges). 4. Solution: (d)
Presently, Constitution Benches are set up on an ad hoc Exp) Option d is the correct answer.
basis as and when the need arises. The idea behind a The Separation of powers between the legislature, the
Constitution Bench is clear: it is constituted in rare cases
executive and the judiciary constitute an important
to decide important questions of fact or legal and/or
safeguard of liberty in a liberal democracy. The doctrine of
constitutional interpretation.
Separation of powers entails the division of the legislative,
2. Solution: (a) executive, and judicial functions of government among
Exp) Option a is the correct answer. different organs. This separation minimizes the possibility
The establishment of an independent judiciary necessarily of arbitrary excesses by the government, since all the three
indicates the federal character of Indian polity. The organs act as check and balance on the powers of each other.
Constitution establishes an independent judiciary headed Therefore, none of the three organs can usurp the essential
by the Supreme Court to settle the disputes between the functions of other organs.
Centre and the states or between the states. This helps in This demarcation prevents the concentration of excessive
maintaining the separate jurisdiction and authority of both power by any branch of the Government. It thus helps to
the Centre and the States. safeguard the liberty and rights of the people in a democracy.
Important Tips 5. Solution: (c)
Following are the federal features of Indian Constitution: Exp) Option c is the correct answer.
• Dual polity consisting of the Union at the Centre and Sarkaria Commission was constituted by the Government
the states at the periphery.
of India in 1983 to look after the situation on various

111 PYQ Workbook


INDIAN POLITY AND GOVERNANCE

matters, the most important being centre-state relations. to make laws on behalf of the state Legislature. (Hence,
The important recommendation of the Sarkaria Commission option b is correct) – The Parliament can delegate the
with respect to Governor were - power to make laws for the state to the President or any
• Procedure of consulting the chief minister in the other body specified by him when the state legislature is
appointment of the state governor should be prescribed suspended or dissolved.
in the constitution itself 7. Solution: (c)
• The Governor should be an eminent person from outside Exp) Option c is the correct answer.
the State
In a Parliamentary form of Government, real executive
• Governor should be a detached figure without political power lies with the Council of Ministers headed by the Prime
links or should not have taken part in politics in the Minister which is responsible to the legislature. In other
recent past words, they hold office only if the majority in Lok Sabha
• Governors should be allowed to complete their term of supports them and must resign if they lose confidence of the
five years Lok Sabha.

Important Tips 8. Solution: (d)


The important recommendation of the Sarkaria Exp) Option d is the correct answer.
Commission were -
Article 249 of the Constitution of India deals with the power
• Setting up a permanent inter-state council of the Parliament to legislate with respect to a matter in the
• Article 356 should be used sparingly State List in national interest. It requires Rajya Sabha to pass
• Institution of all-India service should be strengthened a resolution supported by not less than two-thirds of the
• Residuary power should remain with the parliament members present and voting.

• Reasons should be communicated to the state when Important Tips


state bills are vetoed by the President If the Rajya Sabha declares that it is necessary in the
national interest that Parliament should make laws with
• Centre should have powers to deploy its armed forces, respect to a matter in the State List, then the Parliament
even without the consent of states. However, it is becomes competent to make laws on that matter.
desirable that the states should be consulted • Such a resolution must be supported by two-thirds of
• Commissioner for linguistic minorities should be the members present and voting.
activated • The resolution remains in force for one year;
6. Solution: (b) • it can be renewed any number of times but not
Exp) Option b is the correct answer. exceeding one year at a time.
Emergency Provisions are included in part XVIII from articles • The laws cease to have effect on the expiration of six
352 to 360 in the Constitution for dealing with extraordinary months after the resolution has ceased to be in force.
situations that may threaten the peace, security, stability and • This provision does not restrict the power of a state
governance of the country or a part thereof. legislature to make laws on the same matter. But,
According to the Constitution it is the duty of the Union in case of inconsistency between a state law and a
Government to ensure that governance of a State is carried parliamentary law, the latter is to prevail.
on in accordance with the provisions of the Constitution. 9. Solution: (d)
Under Article 356, the President may issue a proclamation
Exp) Option d is the correct answer.
to impose emergency in a state if he is satisfied on receipt
of a report from the Governor of the concerned State, or The Parliament can make laws on any matter in the State
otherwise, that a situation has arisen under which the List for implementing the international treaties, agreements
administration of the state cannot be carried on according to or conventions. It can be done without the consent of the
the provisions of the constitution. states. This provision enables the Central government to
fulfil its international obligations and commitments.
In such a situation, proclamation of emergency by the
President is on account of the failure (or breakdown) of Some examples of laws enacted under the above provision
constitutional machinery. Thus, it is known as “President’s are United Nations (Privileges and Immunities) Act, 1947;
Rule” or “State Emergency” or “Constitutional Emergency”. Geneva Convention Act, 1960; Anti-Hijacking Act, 1982 and
legislations relating to environment and TRIPS.
Effects of Imposition of President’s Rule in a state
– The President can assume to himself all or any of the 10. Solution: (d)
functions of the state government or he may vest all or any Exp) Option d is the correct answer
of those functions with the Governor or any other executive Statement 1 is incorrect: In India, stamp duty on financial
authority. goods deals, is imposed by the Union or Central Government.
– The President may dissolve the State Legislative Assembly However, the collection and appropriation of stamp duty are
or put it under suspension. He may authorize the Parliament done by the state governments.

PYQ Workbook 112


INDIAN POLITY AND GOVERNANCE

Statement 2 is incorrect: Stamp duty is not regulated by the • The members of Parliament alone are appointed as
Union or Central Government. Instead, it is regulated by the ministers in Britain. In India, a person who is not
respective state governments. The rates and rules related to a member of Parliament can also be appointed as
stamp duty may vary from state to state, as it falls under the minister, but for a maximum period of six months.
purview of the State List (Entry 63) of the Seventh Schedule
• Britain has the system of legal responsibility of
of the Indian Constitution.
the minister while India has no such system. Unlike
Important Tips in Britain, the ministers in India are not required to
Article 268 of the Indian Constitution deals with “Duties countersign the official acts of the Head of the State.
levied by the Union but collected and appropriated by Some of the common features of the constitutions of
the States.” India and the Britain are Parliamentary government,
1. Stamp Duties and Duties of Excise: The article states Bicameral legislature, Rule of Law, both a real and a
that certain stamp duties and duties of excise on nominal executive, a system of collective responsibility,
medicinal and toilet preparations, which are mentioned Single citizenship, Cabinet system, and Parliamentary
in the Union List, shall be levied by the Government of privileges, etc.
India.
12. Solution: (d)
2. Collection of Duties: While these duties are levied by
Exp) Option d is the correct answer
the Union, their collection is carried out differently
The Constitution of India empowers State Governments to
based on their applicability:
levy and collect Agriculture Income Tax through Article
a. If the duties are leviable within any Union territory, 246(3) read with Entry 46 of the State List in the Seventh
their collection is done by the Government of India. Schedule. It explicitly grants State Legislatures exclusive
b. If the duties are leviable in other areas, their collection authority to make laws regarding “Taxes on agricultural
is done by the States within which these duties are income.”
applicable.
13. Solution: (d)
3. Appropriation of Proceeds: The proceeds generated
Exp) Option d is the correct answer
from these duties in any financial year, when
leviable within any State, do not become part of the Centre-State relations, specifically in the planning process,
Consolidated Fund of India. Instead, they are assigned have been compared to municipal relations, treating states
to the respective State where the duties are collected. as subordinate entities. The centralized approach led by the
Planning Commission directed states to align with national
11. Solution: (d) objectives. After the Planning Commission’s abolition in
2015 and NITI Aayog’s establishment, efforts were made
Exp) Option d is the correct answer.
to promote cooperative federalism, granting states greater
India has system of judicial review unlike Britain. The involvement and collaborative planning.
British system is based on the doctrine of the sovereignty
of Parliament, while the Parliament is not supreme in 14. Solution: (d)
India and enjoys limited and restricted powers due to a Exp) Option d is the correct answer.
written Constitution, federal system, judicial review and Distribution of powers between union and states is most
fundamental rights. British judiciary cannot review or important prerequisite for becoming a federal state. The
declare the laws made by British Parliament invalid. seventh schedule of the Indian constitution provides the
Whereas, Indian judiciary can review or declare the laws division of powers between state and central government.
made by Indian Parliament invalid. The powers are divided between the national government
and the regional governments by the Constitution itself and
Important Tips
both operate in their respective jurisdictions independently.
The parliamentary system of government in India is largely
based on the British parliamentary system. However, it Important Tips
never became a replica of the British system and differs 1. Article 246 deals with the 7th Schedule of the Indian
in the following respects: Constitution that mentions three lists named as Union
• India has a republican system in place of British List, State List and Concurrent List which specify the
monarchical system. In other words, the Head of the divisions of power between Union and States.
State in India (that is, President) is elected, while the 2. Presently, the Union List contains 98 subjects
Head of the State in Britain (that is, King or Queen) (originally 97), the State List contains 59 subjects
enjoys a hereditary position. (originally 66) and the Concurrent List contains 52
• In Britain, the Prime Minister should be a member of subjects (originally 47).
the Lower House (House of Commons) of Parliament. 3. Through the 42nd Amendment Act of 1976, five
In India, the Prime Minister may be a member of any subjects were transferred from the State to the
of the two Houses of Parliament. Concurrent List. They are:

113 PYQ Workbook


INDIAN POLITY AND GOVERNANCE

A. Education; have the protection of the freedom of expression. Later


on, the advocacy of secession was also made officially
B. Forests;
unlawful in 1967, with the passage of the Unlawful Activities
C. Protection of wild animals and birds; (Prevention) Act.
D. Weights and measures and;
18. Solution: (a)
E. Administration of justice, constitution and organisation
Exp) Option a is the correct answer.
of all courts except the Supreme Court and the High
Courts. Options 1 and 2 are incorrect: In India, residuary powers
vested in the centre. While in USA, residuary powers vested
in the states. So both have different mechanisms to deal with
3.2. Other Examination Previous Years’ residuary powers.
Questions
Option 3 is correct: Presidents of both countries - India
15. Solution: (a) and USA- have the power to exercise pocket veto. In case
Exp) Option a is the correct answer. of Pocket veto, the President neither ratifies nor rejects
nor returns the bill, but simply keeps the bill pending.
The nature of the Indian Political System embodies the
The way they exercise this veto power is different in both
elements of ‘continuity and change’. Most provisions of
countries.
Constitution have been in continuity since its formation,
while many provisions have been changed as per the demand Option 4 is incorrect: In India, 12 members are nominated
of the time. For example, removal of “internal disturbance” by the president in the upper house. Whereas, there is
as ground for imposition of national emergency after its no provision of nominated member in US upper house
misuse in 1975. The reason for ‘continuity and change’ in (Senate). In India, the number of representatives varies
Indian Political System is due to the fact that it contains from state to state. For example, Uttar Pradesh has 31
elements of different nature such as the modern style (eg. members while Tripura has 1 member only. However, in
Universal Adult Frenchise) the traditional style ( Eg, USA, all states are given equal representation in the Senate
federal form of government) and saint style (eg, Freedom of irrespective of their population. USA has 50 states and the
conscience and free profession, practice, and propagation Senate has 100 members–2 from each state.
of religion). Hence, both (A) and (R) are true and (R) is the
19. Solution: (a)
correct explanation of (A).
Exp) Option a is the correct answer.
16. Solution: (b) Statement 1 is correct: India is a member of
Exp) Option b is the correct answer. ‘Commonwealth of Nations. The organisation comprises
India is regarded as a quasi-federal state as described by of 54 member nations. In 1949 after getting independence,
Prof. K.C. India is referred as a quasi federal state because India declared the continuation of her full membership of
of strong central machinery. In a Quasi federal system, the Commonwealth of Nations and accepted the British
distribution of powers between the Center and the state Crown as the head of the Commonwealth.
are not equal. Hence, Assertion (A) is correct. Statement 2 is incorrect: In 1949, India declared the
The Constitution establishes an independent judiciary continuation of her full membership of the Commonwealth
headed by the Supreme Court. It protects the supremacy of Nations and accepted the British Crown as the head of the
of the Constitution by exercising the power of judicial Commonwealth. This extraconstitutional declaration does
review. Independent judiciary also checks arbitrary use of not curtail India’s sovereignty in any manner.
power by executives. Hence, Reason (R) is correct. 20. Solution: (b)
But Assertion reflects that India has unitary features as Exp) Option b is the correct answer
well and Reason is talking about the federal feature of the
Constitution. So both are true but R is not the reason of A. According to Article 263 of The Constitution of India,
President is empowered to establish the Inter-State
Hence, both (A) and (R) are true but (R) is not the correct
council. The objective of the ISC is to discuss or investigate
explanation of (A).
policies, subjects of common interest, and disputes among
17. Solution: (a) states.
Exp) Option a is the correct answer. Important Tips
Statement 1 is correct: The states have no right to secede The Inter-State Council composes of the following
from the Union under the constitution of India. The members:
federation is an Union because it is indestructible. The
• Prime Minister, Chairman.
country is an integral whole and divided into different states
only for the convenience of administration. • Chief Ministers of all states.

Statement 2 is incorrect: 16 th Constitution Amendment • Chief Ministers of the union territories having
Act of 1963 maintains that advocacy of secession will not legislative assemblies.

PYQ Workbook 114


INDIAN POLITY AND GOVERNANCE

• Lieutenant Governors and Administrators of the union 23. Solution: (d)


territories not having legislative assemblies. Exp) Option d is the correct answer.
• 6 Union Cabinet Ministers, including Union Home The American Constitution provides for the presidential form of
Minister, to be nominated by the Prime Minister. government. Some of features of the American presidential system
• Governors of the states being administered under of government are - the Head of the Executive is the President; the
President’s rule. President appoints his Council of Ministers; the President cannot
Standing Committee dissolve the Legislature. Hence, option d is the correct answer.
• Union Home Minister, Chairman
Important Tips
• 5 Union Cabinet Ministers The Features of the American presidential system of
• 9 Chief Ministers government:

21. Solution: (a) • The American President is both the head of the
State and the head of government. As the head
Exp) Option a is the correct answer
of State, he occupies a ceremonial position. As the
Option a is not correct: A zonal council is not a constitutional head of government, he leads the executive organ of
body. It is a statutory body established under the States government.
Reorganization Act, 1956.
• The President is elected by an electoral college for
Option b is correct: There are five Zonal Councils in India,
a fixed tenure of four years. He cannot be removed
namely: Northern Zonal Council, Eastern Zonal Council,
by the Congress except by impeachment for a grave
Western Zonal Council, Southern Zonal Council, Central
Zonal Council and Southern Zonal Council. unconstitutional act.

Option c is correct: Chandigarh, the joint capital of Haryana • The President and his secretaries are not responsible
and Punjab, is included in the Northern Zonal Council. to the Congress for their acts.
Option d is correct: Zonal councils are advisory bodies and • The President and his secretaries neither possess
do not possess any executive or legislative authority. They membership in the Congress nor attend its sessions.
can only make recommendations to the Union Government
and to the state governments. • The President cannot dissolve the House of
Representatives–the lower house of the Congress.
22. Solution: (c)
• The doctrine of separation of powers is the basis of
Exp) Option c is the correct answer the American presidential system. The legislative,
The Indrajit Gupta Committee, established in 1998, executive and judicial powers of the government are
focused on recommending state funding for elections in separated and vested in the three independent organs
India to curb the influence of money power in politics, of the government.
ensuring transparency, and accountability. It studied funding
sources, eligibility criteria, funding amount, and distribution 24. Solution: (d)
mechanisms for political parties. Exp) Option d is the correct answer.
Important Tips India is a democratic polity as government is elected by the people.
Rajmannar Commission: Elections are held periodically. It provides opportunity to people
Rajmannar Commission was set up by the then DMK to remove any despotic government. India is a sovereign state
Government of Tamil Nadu in 1969 under the Chairmanship as it is free to conduct its own affairs (both internal and external).
of Dr P.V. Rajamanar. It was appointed to look into the question India has a democratic society as all people are treated equal and
regarding the relationship that should subsist between the provided equal opportunities Under Article 14, 15 or 16. India is a
Centre and the States in a federal set-up. It submitted its report in secular nation. It does not differentiate person based on caste, creed,
1971. Its important recommendations are given in the following:
race or gender. It takes care well marginalized, minorities, tribals,
• Readjustment of the VII schedule and residuary women, children, etc. It all makes Indian society democratic. The
power to the states.
Directive Principles of State Policy, enshrined in Part IV of the
• The setting of an Inter-State council immediately Indian Constitution reflects that India is a welfare state.
• Finance commission to be made a permanent body
25. Solution: (b)
• Deletion of Articles 356, 357, and 365 which dealt
with the President’s rule. Exp) Option b is the correct answer
• Abolition of All-India Services (lAS, IPS, and IFS). As per the provisions of Article 293, a State of the
• Planning Commission to be replaced by a statutory Indian Union cannot take foreign loans directly; they
body. can only borrow within India and require the consent of
The Central government completely ignored its the Government of India for any borrowing beyond the set
recommendations. limits.

115 PYQ Workbook


INDIAN POLITY AND GOVERNANCE

26. Solution: (d) 28. Solution: (d)


Exp) Option d is the correct answer Exp) Option d is the correct answer.
Constitutional Provisions: Centre-State relations in India In the Parliamentary form of Government, the executive
are primarily governed by the provisions laid out in the is responsible to the legislature and is under its control.
Indian Constitution. Articles 245 to 255 (Part XI) deal with The executive remains in office till it enjoys the confidence
the distribution of legislative powers between the Centre and of the legislature, especially the Lok Sabha. The ministers
the States. For instance, Article 246 delineates the subject (Executive) are collectively responsible to the Parliament
matters falling under the jurisdiction of the Union, States, in general and to the Lok Sabha in particular (Article 75).
and concurrent list. This ensures a clear demarcation of
powers and responsibilities. 29. Solution: (b)
Conventions and Practices: Over time, certain unwritten Exp) Option b is the correct answer.
conventions and practices have evolved, shaping Centre- Political Parties are the lifeblood of democracy. The
State relations. For instance, the Sarkaria Commission political parties are responsible for the wide range of domains
Report outlines various practices, like the appointment of including providing good governance, efficient delivery of
Governors and the use of Article 356 (President’s Rule), service, or required welfare measure. Any failure in the
which influence interactions between the Centre and States. system may mobilise people or other political parties to
Judicial Interpretations: The judiciary plays a vital role condemn political party/ies in government.
in interpreting and clarifying constitutional provisions However, Political Parties are the lifeblood of democracy
related to Centre-State relations. Landmark judgments, like not only because people can blame political parties in
the Kesavananda Bharati case, have helped to define the case of any failure. This can be one of the reason but not
limitations and extent of powers of both entities. the only reason for making political parties lifeblood of
Mechanisms for Dialogue: To facilitate cooperation and democracy. Hence, both (A) and (R) are true, and (R) is not
coordination, various mechanisms for dialogue exist, such the correct explanation of (A).
as the Inter-State Council and NITI Aayog. These platforms 30. Solution: (c)
provide a forum for both levels of government to discuss and
Exp) Option c is the correct answer.
resolve issues of mutual concern.
The word ‘sovereign’ implies that State is neither a dependency
27. Solution: (d) nor a dominion of any other State, but an independent state.
Exp) Option d is the correct answer Sovereignty is the most important element of the state. It
Article 249 of the Indian Constitution deals with the power of the confers legal status on the state’s actions. It’s sovereignty that
Parliament to legislate on matters in the State List under certain allows the state to exercise legitimate control over all the
circumstances. Article 249 provides a mechanism for the Union laws, rules, policies, and decisions. The idea of sovereignty
Parliament to intervene in State List subjects temporarily when there goes back to Aristotle, who spoke of the “Supreme power of
is a need for national interest. However, it is subject to certain checks the state.” There is no meaning of Flag, Capital or Head of
and balances to ensure that such intervention remains limited and State, if state is not sovereign.
does not undermine the federal structure of the country.
31. Solution: (a)
Important Tips Exp) Option a is the correct answer.
The key provisions of Article 249 are as follows: India is regarded as a quasi-federal state because more
• If the Council of States (Rajya Sabha) passes a power is given to Centre. In a Quasi federal system, the
resolution, supported by not less than two-thirds distribution of powers between the Center and the state are
of its members present and voting, declaring that it not equal. Hence, Assertion (A) is correct.
is necessary or expedient in the national interest that Indian Constitution is also neither federal nor unitary. In
Parliament should make laws on a specific matter in the words of D.D. Basu, the Constitution of India is neither
the State List, then Parliament can do so. purely federal nor unitary, but is a combination of both.
• The resolution remains in force for a specific Hence, Reason (R) is correct.
period (not exceeding one year) as mentioned in the Indian constituion is consider as quasi federal as it is
resolution itself.
neither federal nor unitary.
• If the resolution is approved to continue in force
through a similar process as mentioned in point 1, it 32. Solution: (d)
remains in force for an additional one-year period Exp) Option d is the correct answer
from the date it would have otherwise ceased.
The Sarkaria Commission, established in 1983, aimed to
• Any law made by Parliament during this period, review the state-center relationship in India and propose
which it would not have been competent to make but constitutional changes. Headed by Rajinder Singh Sarkaria, a
for the resolution, ceases to have effect to the extent retired Supreme Court judge, it submitted a comprehensive
of that incompetency after a period of six months 1600-page report with 247 specific recommendations in
from the resolution’s expiration. 1988.

PYQ Workbook 116


INDIAN POLITY AND GOVERNANCE

33. Solution: (a) 36. Solution: (a)


Exp) Option a is the correct answer Exp) Option a is the correct answer
Statement 1 is correct: Taxes on transactions in Stock- The Constitution of India in Article 263, provided that an
Exchanges and Futures Markets in India are levied by the Inter-State Council (ISC) may be established “if at any time
Union. This is listed in the Seventh Schedule (Article 246) it appears to the President that the public interests would
under List I-Union List, item no. 90, which grants the be served by the establishment of a Council”. This option
exclusive power to the Union to impose taxes other than was exercised in 1990. Therefore, the ISC was established as
stamp duties on transactions in stock exchanges and futures a permanent body on 28 May 1990 by a presidential order
markets. on recommendation of Sarkaria Commission

Statement 2 is incorrect: The collection of taxes on 37. Solution: (b)


transactions in Stock-Exchanges and Futures Markets is also Exp) Option b is the correct answer.
done by the Union, not by the State Government, as it is an Statement 1 is correct: India is called a democratic republic
exclusive domain of union government. country because head of the state is elected by the people.
All positions in the government are available to all
34. Solution: (b)
people, regardless of religion, caste, or gender. The Indian
Exp) Option b is the correct answer. Constitution provides for representative parliamentary
Statement 1 is correct: The Constitution introduces a federal democracy under which the executive is responsible to the
system as the basic structure of Government. A majority of legislature for all its policies and actions.
judges in the Kesavananda Bharati case in 1973 included Statement 2 is correct: India has a Parliamentary form of
federalism as part of the basic structure of the Constitution. government. The Parliamentary system is a democratic type
Later on, in the SR Bommai case in 1994 and in a number of of government in which the party (or coalition of parties)
cases since then, the court has held that federalism is part of with the most representation in Parliament (legislature)
the basic structure of our Constitution. forms the government, with its leader becoming Prime
Minister.
Statement 2 is correct: India has a strong admixture of
a unitary bias. India has some of unitary features such as Statement 3 is correct: The Preamble states that the
a strong Centre, single Constitution, single citizenship, Constitution derives its authority from the people of
flexibility of Constitution, etc. These unitary features India. In Parliamentary system, the supreme power vests
with the body of people’s representative called a Parliament.
coexist with many federal elements such as Written
Constitution, Rigid Constitution, Supremacy of the Statement 4 is incorrect: Indian Constitution does not
Constitution, independent judiciary, etc. provide a unified authority. The seventh schedule of
the Indian constitution provides the division of powers
Statement 3 is incorrect: Neither Union nor State
between state and central government. It has provided
Legislatures are sovereign in India. As both has to operate separate three lists - the Union List, the State List, and the
within jurisdiction earmarked for it by the Constitution. Concurrent List. India is a federal nation. The powers are
Moreover, the Supreme Court can declare a law passed by divided between the national government and the regional
Parliament unconstitutional if it contravenes the provision governments by the Constitution itself and both operate in
of the Constitution. Unlike, British parliament is sovereign their respective jurisdictions independently.
body.
38. Solution: (c)
Statement 4 is incorrect: Only legislative, executive and
Exp) Option c is the correct answer.
financial power have been divided between the Union and
its units. Judicial powers have not been divided. The Indian India is a Republic’ means there are no hereditary rulers
Constitution has established an integrated judicial system in India. A democratic polity can be classified into two
with the Supreme Court at the top and the state high courts categories– monarchy and republic. In a monarchy, the
head of the state (usually king or queen) enjoys a hereditary
below it. This single system of courts enforces both the
position, that is, he/she comes into office through succession,
Central laws as well as the state laws. In US, on the other
e.g., Britain. In a republic, on the other hand, the head of
hand, there is a double system of courts whereby the federal
the state is always elected directly or indirectly for a fixed
laws are enforced by the federal judiciary and the state laws
period, e.g., USA or India. Therefore, the term ‘republic’ in
by the state judiciary. our Preamble indicates that India has an elected head called
35. Solution: (d) the President. S/he is elected indirectly for a fixed period of
five years.
Exp) Option d is the correct answer
Article 276 of the Indian Constitution limits the tax on 39. Solution: (a)
professions, trades, and callings to ₹2,500 per year per Exp) Option a is the correct answer
individual, preventing an excessive burden on taxpayers Article 248 of the Indian Constitution grants Parliament
in these categories. exclusive power to make laws on any matter not listed

117 PYQ Workbook


INDIAN POLITY AND GOVERNANCE

in the Concurrent List or State List. This includes the unified framework. Jawaharlal Nehru used this famous
authority to impose taxes not specified in either list. This phrase ‘Unity in Diversity’ in his book ‘Discovery of India’.
provision ensures no legislative gaps and empowers the This phrase is used even today as an expression of harmony
Union Government with residuary powers. and unity amongst different regions and cultures. All
institutes i.e. Inter-State Councils, National Development
40. Solution: (a) Council, Finance Commission, Regional Council, Unitary
Exp) Option a is the correct answer Judicial System and All India Services institutes are
The Supreme Court of India holds original jurisdiction, as considered necessary to promote ‘Unity among diversity’
per Article 131 of the Indian Constitution, to resolve fiscal in the Indian Federalism.
disputes between the Government of India and one or more
44. Solution: (a)
states, between the Government of India and any state(s)
versus other state(s), and between two or more states. Exp) Option a is the correct answer.
Women, Dalits, Poor and Minority groups are the biggest
41. Solution: (c) stakeholders of Democracy in India. As the constitution
Exp) Option c is the correct answer protects their rights and has many provisions for their
The freedom of trade, commerce, and intercourse is welfare and empowerment such as Fundamental rights,
provided under Part XIII of the Indian Constitution Directive Principle of State Policy (DPSPs), and many
in Articles 301 to 307. Article 301 lays down the general political rights. The weaker section of society needs more
external support and assistance for their empowerment
principles of trade and commerce whereas Article 302 to
unlike the dominant section of society. So maximum
305 enunciates the restrictions which trade is subjected
benefits of a democratic system of governance are taken by
to. Article 306 has been repealed and Article 307 allows
these groups. The reason is that the democracy has emerged
Parliament to appoint authority for implementing Articles
as the carrier for the desire of Self Respect. Their identity
301 to 304. The source for adopting these provisions was the
and dignity are well protected by the constitution of India
Australian Constitution.
and through various parliamentary laws. Hence, both (A)
42. Solution: (b) and (R) are true and (R) is the correct explanation of (A).
Exp) Option b is the correct answer 45. Solution: (c)
Zonal Councils are advisory councils and are made up of Exp) Option c is the correct answer
the states of India that have been grouped into five zones to Assertion (A) is true: Article 356 deals with President’s
foster cooperation among them. These were set up vide Part- Rule in case of constitutional breakdown in a state. Sarkaria
III of the States Reorganization Act, 1956. The Union Home Commission recommended using it sparingly, maintaining
Minister is the common chairman of five zonal councils. federal structure, and avoiding undue Central Government
Each chief minister acts as a vice chairman of the council interference in state affairs.
by rotation, holding office for a period of one year at a
Reason (R) is true: The political parties in power at the
time.
Centre have frequently misused Article 356. Since India’s
Important Tips independence, it has been invoked over 100 times to impose
President’s Rule in various states. In most cases it was used for
• The Northeastern states are not covered by any of
political considerations rather than any genuine breakdown
the Zonal Councils and their special problems are
of constitutional machinery in the States.
addressed by another statutory body, the North
Eastern Council at Shillong, created by the North (R) is the correct explanation of (A): The reason behind
Eastern Council Act, 1971. the Sarkaria Commission’s recommendation to use Article
356 sparingly was precisely because it had been misused by
• This council originally comprised Arunachal Pradesh, the political parties that came to form the Government at
Assam, Manipur, Meghalaya, Mizoram, Nagaland
the Centre.
and Tripura; later the state of Sikkim was also added
vide North Eastern Council (Amendment) Act, 2002 46. Solution: (b)
notified on 23 December 2002. Exp) Option b is the correct answer.
• The union territories of Andaman and Nicobar The federal system was adopted in India to safeguard and
Islands and Lakshadweep are not members of any promote the unity of the country, and accommodate regional
of the Zonal Councils. However, they are presently diversity. Such accommodation of diversities has built a
special invitees to the Southern Zonal Council. stronger nation. Most of Indian States have different ethnicity,
culture, languages, etc, but still India as nation stood firm
43. Solution: (d)
and robust. Federal system was designed to facilitate many
Exp) Option d is the correct answer. diverse parts and protect national integration because
The Constitution establishes a federal system of government, accommodation of these diversities has built a stronger
where power is divided between the center and the states. Indian nationhood. Hence, both (A) and (R) are true, and
This allows for regional autonomy and diversity within a (R) is the correct explanation of (A).

PYQ Workbook 118


INDIAN POLITY AND GOVERNANCE

47. Solution: (c) • Clause (1) empowers the President, with the
Exp) Option c is the correct answer consent of the State Governor, to entrust functions,
The Gadgil-Mukherjee Formula, devised in 1969, allocated conditionally or unconditionally, to the State
central assistance to states based on population, per capita Government or its officers concerning matters falling
income, and special problems. It is no longer in use, and under the executive power of the Union.
allocation now occurs through grant-in-aid, centrally
• Clause (2) enables Parliament to make laws that
sponsored schemes, and tax sharing as per Finance
apply to a State, even if the State Legislature lacks
Commission recommendations. The Planning Commission
the authority to enact laws on that subject. It allows
was replaced by NITI Aayog in 2015, leading to changes
the conferment of powers and imposition of duties on
in resource allocation mechanisms in India’s fiscal federal
the State or its officers and authorities.
system.
• Clause (3) addresses the reimbursement of any
48. Solution: (b)
additional administrative costs incurred by the State
Exp) Option b is the correct answer due to the exercise of powers and duties conferred
Under Article 246 of the Indian Constitution, which deals upon it under this article. The Government of India
with the distribution of legislative powers between the Union pays an agreed amount, or in case of disagreement,
and the States, “Land Revenue” is exclusively imposed and an arbitrator appointed by the Chief Justice of India
collected by the States under the State List (List II). The determines the compensation.
State Governments have the authority to levy taxes on land
and agricultural income, and they exercise this power as part 52. Solution: (d)
of their revenue generation. Exp) Option d is the correct answer.

49. Solution: (c) A unitary government is one in which all the powers are
vested in the national government. The regional governments,
Exp) Option c is the correct answer
if exist, derive their authority from the national government.
Under Article 270 of the Indian Constitution, taxes on
income, excluding agricultural income, are levied and The Indian Constitution also contains a large number of
collected by the Government of India. These taxes’ net unitary or non-federal features such as a strong Centre,
proceeds, except those attributed to Union territories or single Constitution, single citizenship, flexibility of
taxes on Union emoluments, are not part of the Consolidated Constitution, integrated judiciary, appointment of state
Fund of India but are assigned to States where the tax is governor by the Centre, all-India services, emergency
leviable. These proceeds are distributed among the States provisions (Constitutional crisis), President’s approval on
as prescribed. state bills, no equality of state representation and so on.
50. Solution: (b) Whereas, some of federal features are, Written Constitution,
Exp) Option b is the correct answer Rigid Constitution, Supremacy of the Constitution,
Decentralization of Powers (7 th Schedule), independent
The Sarkaria Commission recommended establishing
a Permanent Inter-State Council (Inter-Governmental judiciary headed by the Supreme Court (which hears cases
Council) to foster cooperation and resolve disputes between related center and States), Bicameralism and so on.
the central government and states, promoting federal Federal Government Unitary Government
principles, dialogues, and consensus-building for governance
1. Dual Government (that 1. Single government,
and policy implementation.
is, national government that is, the national
51. Solution: (a) and regional government which
Exp) Option a is the correct answer government) may create regional
governments
Article 257(1) states that, “The executive power of every
State shall be so exercised as not to impede or prejudice 2. Written Constitution 2. Constitution may be
the exercise of the executive power of the Union, and the written (France) or
executive power of the Union shall extend to the giving of unwritten (Britain)
such directions to a State as may appear to the Government 3. Division of powers 3. No division of powers.
of India to be necessary for that purpose”. This provision between the national and All powers are vested in
enables the Central Government to intervene when necessary regional government the national government
to maintain the integrity and unity of the nation and to
4. Supremacy of the 4. Constitution may be
ensure the smooth functioning of the federal structure.
Constitution supreme (Japan) or may
Important Tips not be supreme (Britain)
Article 258 of the Constitution of India, deals with the 5. Rigid Constitution 5. Constitution may be
“Power of the Union to confer powers, etc, on States in rigid (France) or flexible
certain cases.” (Britain)

119 PYQ Workbook


INDIAN POLITY AND GOVERNANCE

6. Independent judiciary 6. Judiciary may be Characteristics features of Indian Federalism:


independent or may not • Distribution of Powers: Powers divided between
be independent central and state governments. Schedule 7 divides
7. Bicameral legislature 7. Legislature may be subjects between Centre and States and maintains an
bicameral (Britain) or extensive concurrent list over which both centre and
unicameral (China) states have jurisdiction. Residuary power rests with
Central Government.
53. Solution: (b)
• Dual government: India has a dual polity system with
Exp) Option b is the correct answer separate governments at the central and state levels.
The Northern Zonal Council is a zonal council that comprises • Written Constitution: Federal structure defined by
the states and union territories of Chandigarh, National written constitution.
Capital Territory of Delhi, Haryana, Himachal Pradesh, • Bicameralism: The Constitution establishes a
Jammu and Kashmir, Punjab, Rajasthan and Ladakh. bicameral legislature comprising the Rajya Sabha and
Important Tip: Lok Sabha.
Five Zonal Councils were set up vide Part-III of the States • Supremacy of Constitution: Constitution is supreme
Re-organisation Act, 1956. The present composition of law.
each of these Zonal Councils is as under: • Independent Judiciary: Judiciary interprets and
• The Northern Zonal Council, comprising the States enforces Constitution.
of Haryana, Himachal Pradesh, Punjab, Rajasthan, • Rigidity of Constitution: The Constitution’s
National Capital Territory of Delhi and Union Territory rigidity requires joint action of the Central and state
of Chandigarh, Jammu & Kashmir and Ladakh. governments to amend provisions related to the federal
structure.
• The Central Zonal Council, comprising the States of
Chhattisgarh, Uttarakhand, Uttar Pradesh and Madhya 55. Solution: (b)
Pradesh;
Exp) Option b is the correct answer
• The Eastern Zonal Council, comprising the States of
Bihar, Jharkhand, Orissa, Sikkim and West Bengal; The M.M. Punchhi Commission on Centre-State Relations
• The Western Zonal Council, comprising the States of recommended that a bill reserved for the consideration
Goa, Gujarat, Maharashtra and the Union Territories of the Union Executive should be disposed of within six
of Daman & Diu and Dadra & Nagar Haveli; months. The Punchhi Commission, officially known as the
• The Southern Zonal Council, comprising the States of Commission on Centre-State Relations, was established in
Andhra Pradesh, Karnataka, Kerala, Tamil Nadu and
2007 to examine and review the relationship between the
the Union Territory of Puducherry.
central government and state governments in India.
• The North Eastern States i.e. (i) Assam (ii) Arunachal
Pradesh (iii) Manipur (iv) Tripura (v) Mizoram (vi)
56. Solution: (b)
Meghalaya and (vii) Nagaland are not included in the
Zonal Councils and their special problems are looked Exp) Option b is the correct answer.
after by the North Eastern Council, set up under the • Article-263 in Part XI of the Constitution provides for
North Eastern Council Act, 1972.
the establishment of an Inter-State Counci for better
54. Solution: (a) coordination among the states and between Centre and
Exp) Option a is the correct answer. States.
The Indian Federation is not the result of an agreement • In 1990, the Inter-State Council was established on the
among the states, unlike the American Federation.
recommendation of Sarkaria Commission (1983-87)
Article 1 describes India as a ‘Union of States’ which implies
• The present composition of the Council is as follows:
two things: one, Indian Federation is not the result of an
agreement by the states; and two, no state has the right to (I) Prime Minister as the Chairman
secede from the federation.
(II) Chief Minister of all the States and Union Territories
Important Tips (III) Administrators of Union Territories not having
Indian Federation system is based on the Canadian Assemblies
Model, where a unitary state transforms into a federation
by granting provinces autonomy to promote regional (IV) Governor’s of States under President rule
interests. (V) Six Central Cabinet Ministers

PYQ Workbook 120


INDIAN POLITY AND GOVERNANCE

Important Tips subjects, such as criminal law, civil law, taxation, labor
• Punchhi Commission (2007) : is related to the laws, and more. Statutes are an important source of law in
Commission on Centre-State relations. India.
• Raiamannar Commission (1969) : is related to Option 3 is correct- Customary law refers to the practices,
examine all aspects of centre and state. customs, and traditions that are followed by a specific
• Mungerilal Commission (1977) : Identification community or group of people. Customary laws have
backward castes in Bihar.
legal recognition and are applicable in specific regions or
57. Solution: (d) communities, particularly in matters related to personal
laws, inheritance, marriage, and other cultural practices.
Exp) Option d is the correct answer.
Option 4 is correct- Case law, also known as judicial
Indian federalism rather follows the principle of a “blended”
precedent, refers to the decisions and interpretations made
or “cooperative” federalism, where there is a significant
by the courts while deciding legal disputes. The judgments
degree of cooperation and coordination between the Union delivered by higher courts, particularly the Supreme Court
Government and the State Governments. The Constitution of India, create precedents that serve as a source of law.
provides for a distribution of powers between the Union These precedents help in interpreting laws and establishing
and the States (through Union and States list), but it also legal principles for future cases.
allows for concurrent powers and shared responsibilities.
59. Solution: (a)
Important Tip
Exp) Option a is the correct answer.
Some of the features of Indian Federalism:
A. Pluralist democracy - (4) Capacity of groups to ensure
1. Division of Powers: Clear division of powers between Governmental responsiveness. Pluralist democracy refers
the Union Government and State Governments. to a system where power is dispersed among different
2. Written Constitution: Indian federalism is based on a interest groups, and these groups have the ability to
written Constitution. influence government decisions and policies.
3. Supremacy of the Constitution: The Constitution is B. People’s democracy - (3) Social equality through
the supreme law of the land. common ownership of wealth. People’s democracy
4. Independent Judiciary: Presence of an independent aims for social equality by advocating for common
judiciary to interpret and protect the Constitution. ownership of wealth and resources, ensuring equitable
distribution among all members of society.
5. Dual Polity: Recognition of a dual polity consisting of
the Union Government and State Governments. C. Developmental democracy - (2) Highest and
harmonious development of individual capabilities.
6. Rigid Constitution: The Indian Constitution requires Developmental democracy focuses on promoting the
special procedures for amendments. holistic development of individual capabilities,
7. Distribution of Powers: Powers allocated through allowing individuals to reach their full potential in a
Union List, State List, and Concurrent List. harmonious society.
8. Strong Center: The Union Government holds more D. Elitist democracy - (1) Power is always exercised by a
extensive powers compared to State Governments. privileged few. Elitist democracy refers to a system where
9. Cooperative Federalism: Emphasis on collaboration power is concentrated in the hands of a small, privileged
and coordination between Union and State group, often excluding the majority of the population
Governments. from decision-making processes.
10. Independent Election Commission: An independent 60. Solution: (a)
body responsible for conducting elections at national Exp) Option a is the correct answer.
and state levels.
In a presidential form of government, the President is
58. Solution: (d) separate from the legislative body, which means that the
President is not a member of the legislature. The President is
Exp) Option d is the correct answer.
the head of the executive branch and is elected independently
Option 1 is correct- The Constitution of India is the of the legislature. This separation of powers ensures a clear
supreme law of the land. It lays down the fundamental distinction between the executive and legislative functions.
principles, rights, and duties of citizens, and establishes Option b is incorrect: The Presidential form of government
the structure and functioning of the government. The does separate the legislative and executive functions,
Constitution serves as the foundation and source of all laws with the President being the head of the executive branch
in India. and the legislature having its own separate powers and
Option 2 is correct- Statutes, also known as legislation or responsibilities.
acts, are laws enacted by the Parliament of India or the Option c is incorrect: The principle of collective
State Legislative Assemblies. These laws are formulated and responsibility is usually associated with parliamentary
passed by the respective legislative bodies and cover various systems, where the executive (Prime Minister and

121 PYQ Workbook


INDIAN POLITY AND GOVERNANCE

Cabinet) is collectively responsible to the legislature. In 63. Solution: (c)


a presidential system, the President is not bound by the Exp) Option c is the correct answer.
principle of collective responsibility.
Statement 1 is correct: Residuary powers have been given
Option d is incorrect: In a presidential form of government, to the Union Parliament by Article 248 and Entry 97 of the
the tenure of the President is typically fixed and does not Union List. These provisions give Parliament exclusive power
depend on the legislature. The President serves a fixed term to make any law with respect to any matter not enumerated
as specified by the constitution and is not subject to removal in List II or List III, including any tax not mentioned in
by the legislature, except under specific circumstances either of those lists.
outlined in the constitution. Statement 2 is incorrect: In the matter of residuary powers,
the Constitution of India does not follow the Constitution
61. Solution: (c)
of Australia, but follows the Constitution of Canada. The
Exp) Option c is the correct answer. Constitution of Australia does not vest the residuary powers
Statement a is correct: Indian federalism grants powers to in any level of government, but leaves them to be determined
both the central government and the state governments. by judicial interpretation. The Constitution of Canada, on
Each state government has its own legislative and executive the other hand, vests the residuary powers in the federal
powers to govern their respective territories. government by Section 91 of its Constitution Act, 1867.
Statement b is correct: The judiciary in India, particularly Statement 3 is incorrect: Schedule 7 of the Constitution
the Supreme Court, has the authority to interpret the of India does not provide a list of residuary powers, but
constitution and determine the scope of powers and provides three lists of subjects on which Parliament and state
jurisdiction of different levels of government. The judiciary legislatures can make laws. These are List I (Union List), List
acts as a neutral arbiter in resolving disputes between the II (State List) and List III (Concurrent List). The residuary
central government and state governments. powers are those that are not covered by any of these lists.
Statement c is incorrect: States are subordinate to the Statement 4 is correct: Government of India Act, 1935 placed
Central Government: This statement is not a feature of residuary powers in the hands of the Governor-General.
This was done to ensure that no important subject was left
Indian federalism. In Indian federalism, both the central
out from legislative competence and to give flexibility to deal
government and state governments are sovereign in their
with unforeseen situations. The Governor-General could
respective spheres of power. While the central government
legislate on any matter that was not included in any of the
has certain powers and responsibilities that are applicable
seven lists provided by this Act
to the entire country, state governments also possess
independent powers to govern their respective states. 64. Solution: (c)
Statement d is correct: Indian federalism entails a division Exp) Option c is the correct answer.
of fiscal powers between the central government and Statement 1 is correct: The head of government is also
state governments. While the Constitution provides for the the head of state in the presidential system. The president
assignment of revenue sources to each level of government, represents the nation both internally and externally, and has
the distribution of fiscal resources can be subject to periodic the authority to appoint and dismiss cabinet members, issue
revisions and negotiations between the central government executive orders, veto legislative acts, negotiate treaties,
and state governments. command the armed forces, etc.

62. Solution: (b) Statement 2 is correct: Executive can veto legislative acts
in the presidential system. The president has the power to
Exp) Option b is the correct answer. reject or approve bills passed by the legislature, and can also
The setting up of the Inter-State Council in 1990 was propose legislation to the legislature. However, the legislature
aimed at strengthening the federal provisions of the can override the president’s veto by a supermajority vote in
Constitution of India. The Inter-State Council serves as some cases
a platform for coordination and cooperation between the
Union government and the state governments. It facilitates 65. Solution: (a)
the discussion and resolution of issues and disputes between Exp) Option a is the correct answer
the Union and states, promoting cooperative federalism. Article 300 of the Indian Constitution deals with “Suits
and proceedings.” It states that the Union Government
Important Tips
(Government of India) and State Governments (Government
• The Inter-State Council is a non-permanent of a State) can sue or be sued in their respective names.
constitutional body that was established by a
presidential order on 28 May 1990 based on Article 66. Solution: (c)
263 of the Constitution of India. Exp) Option c is the correct answer.
• The main purpose of the Inter-State Council is to The Democracy of India is based on the fact that public
enhance the coordination between the Centre and the enjoys the rights to choose and change the Government.
States and to discuss or investigate policies, subjects of Democracy refers to a system of government where the
common interest, and disputes among States citizens exercise power by voting.

PYQ Workbook 122


INDIAN POLITY AND GOVERNANCE

Important Tips
Key characteristics of Democracy:
1. Democracy is of two types–direct and indirect.
2. In direct democracy, the people exercise their supreme power directly as is the case in Switzerland. There are four devices
of direct democracy, namely, Referendum, Initiative, Recall and Plebiscite.
3. In indirect democracy, on the other hand, the representatives elected by the people exercise the supreme power and thus
carry on the government and make the laws.
• This type of democracy, also known as representative democracy, is of two kinds–parliamentary and presidential.
• The Indian Constitution provides for representative parliamentary democracy under which the executive is responsible
to the legislature for all its policies and actions.
4. The term ‘democratic’ is used in the Indian Preamble in the broader sense embracing not only political democracy but
also social and economic democracy. This dimension was stressed by Dr. Ambedkar in his concluding speech in the
Constituent Assembly on November 25, 1949, in the following way: “Political democracy cannot last unless there lies at
the base of it social democracy.

123 PYQ Workbook


INDIAN POLITY AND GOVERNANCE

INDIAN POLITY AND GOVERNANCE


JUDICIARY
*This unit consists of questions from Supreme Court, High Courts, Lower Judiciary and Important
Judgements of Higher Courts.

4.1. UPSC CSE Previous Years’ Questions 2. The Constitution of India empowers the
Supreme Court and the High Courts to
1. Consider the following statements: punish for contempt of themselves.
Statement-1: In India, prisons are managed by 3. The Constitution of India defines Civil
State Governments with their own rules and Contempt and Criminal Contempt.
regulations for the day-to-day administration 4. In India, the Parliament is vested with
of prisons the powers to make laws on Contempt of
Statement-II: In India, prisons are governed Court.
by the Prisons Act, 1894 which expressly Which of the statements given above is/are
kept the subject of prisons in the control of correct? [UPSC CSE Pre 2022]
Provincial Governments. (a) 1 and 2 only
Which one of the following is correct in (b) 1, 2 and 4
respect of the above statements? (c) 3 and 4 only
[UPSC CSE Pre 2023] (d) 3 only
(a) Both Statement-I and Statement-II are
4. With reference to India, consider the
correct and Statement-II is the correct
following statements:
explanation for Statement-1
(b) Both Statement-1 and Statement-II are 1. Government law officers and legal firms
correct and Statement-II is not the correct are recognised as advocates, but corporate
explanation for Statement-1 lawyers and patent attorneys are excluded
(c) Statement-1 1s correct but Statement-11 from recognition as advocates.
is incorrect 2. Bar Councils have the power to lay down
(d) Statement-1 Is incorrect but Statement-II the rules relating to legal education and
is correct recognition of law colleges.
2. With reference to the writs issued by the Which of the statements given above is/are
Courts in India, consider the following correct? [UPSC CSE Pre 2022]
statements: (a) 1 only
1. Mandamus will not lie against a private (b) 2 only
organisation unless it is entrusted with a (c) Both 1 and 2
public duty. (d) Neither 1 nor 2
2. Mandamus will not lie against a Company 5. With reference to India, consider the
even though it may be a Government following statements:
Company. 1. Judicial custody means an accused is in
3. Any public minded person can be a the custody of the concerned magistrate
petitioner to move the Court to obtain and such accused is locked up in police
the writ of Quo Warranto. station, not in jail.
Which of the statements given above are 2. During judicial custody, the police officer
correct? [UPSC CSE Pre 2022] in charge of the case is not allowed to
(a) 1 and 2 only interrogate the suspect without the
(b) 2 and 3 only approval of the court.
(c) 1 and 3 only Which of the statements given above is/are
(d) 1, 2 and 3
correct? [UPSC CSE Pre. 2021]
3. Consider the following statements: (a) 1 only
1. Pursuant to the report of H.N. Sanyal (b) 2 only
Committee, the Contempt of Courts Act, (c) Both 1 and 2
1971 was passed. (d) Neither 1 nor 2

PYQ Workbook 124


INDIAN POLITY AND GOVERNANCE

6. With reference to India, consider the (c) 3 and 4 only


following statements: (d) 1, 3 and 4 only
1. When a prisoner makes out a sufficient 9. In India, separation of judiciary from the
case, parole cannot be denied to such executive is enjoined by
prisoner because it becomes a matter of [UPSC CSE Pre. 2020]
his/her right. (a) the Preamble of the Constitution
2. State Governments have their own (b) a Directive Principle of State Policy
Prisoner Release on Parole Rules. (c) the Seventh Schedule
Which of the statements given above is/are (d) the conventional practice
correct? [UPSC CSE Pre. 2021] 10. Consider the following statements:
(a) 1 only 1. The Constitution of India defines its
(b) 2 only ‘Basic Structure’ in terms of federalism,
(c) Both 1 and 2 secularism, fundamental rights and
(d) Neither 1 nor 2 democracy.
7. With reference to Indian Judiciary, consider 2. The Constitution of India provides for
the following statements: ‘Judicial review’ to safeguard the citizens’
liberties and to preserve the ideals on
1. Any retired judge of the Supreme Court
which the Constitution is based.
of India can be called back to sit and act
as a Supreme Court judge by the Chief Which of the statements given above is are
Justice of India with prior permission of correct? [UPSC CSE Pre. 2020]
the president of India. (a) 1 only
(b) 2 only
2. A High Court in India has the power to
(c) Both 1 and 2
review its own judgement as the Supreme (d) Neither 1 nor 2
Court does.
Which of the statements given above is/are 11. In India, Legal Services Authorities provide
correct? [UPSC CSE Pre. 2021] free legal services to which of the following
type of citizens?
(a) 1 only
1. Person with an annual income of less
(b) 2 only than Rs. 1,00,000.
(c) Both 1 and 2
2. Transgender with an annual income of
(d) Neither 1 nor 2
less than Rs. 2,00,000.
8. Consider the following statements: 3. Member of Other Backward Classes
1. The motion to impeach a Judge of the (OBC) with an annual income of less than
Supreme Court of Indian cannot be Rs. 3,00,000.
rejected by the Speaker of the Lok Sabha 4. All Senior Citizens.
as per the Judges (Inquiry) Act, 1968. Select the correct answer using the code given
2. The Constitution of India defines below: [UPSC CSE Pre. 2020]
and gives details of what constitutes (a) 1 and 2 only
‘incapacity and proved misbehaviour’ of (b) 3 and 4 only
the Judges of the Supreme Court of India. (c) 2 and 3 only
3. The process of impeachment of the Judges (d) 1 and 4 only
of the Supreme Court of India is given in 12. With reference to the Constitution of India,
the Judges (Inquiry) Act, 1968. consider the following statements:
4. If the motion for the impeachment of 1. No High Court shall have the jurisdiction
a Judge is taken up for voting, the law to declare any central law to be
requires the motion to be backed by each constitutionally invalid.
House of the Parliament and supported 2. An amendment to the Constitution of
by a majority of total membership of that India cannot be called into question by
House and by not less than two-thirds of the Supreme Court of India.
total members of that House present and Which of the statements give above is/are
voting. correct? [UPSC CSE Pre. 2019]
Which of the statements given above is/are (a) 1 only
correct? [UPSC CSE Pre. 2020] (b) 2 only
(a) 1 and 2 only (c) Both 1 and 2
(b) 3 only (d) Neither 1 nor 2

125 PYQ Workbook


INDIAN POLITY AND GOVERNANCE

13. Consider the following statements: 1. While appointing the Supreme Court
1. The 44th Amendment to the Constitution Judges, the President of India has to
of India introduced an Article placing the consult the Chief Justice of India.
election of the Prime Minister beyond 2. The Supreme Court Judges can be
judicial review. removed by the Chief Justice of India
2. The Supreme Court of India struck down only.
the 99th Amendment to the Constitution 3. The salaries of the Judges are charged on
of India as being violative of the the Consolidated Fund of India to which
independence of judiciary. the legislature does not have to vote.
Which of the statements given above is/are 4. All appointments of officers and staffs of
correct? [UPSC CSE Pre. 2019] the Supreme Court of India are made by
the Government only after consulting the
(a) 1 only
Chief Justice of India.
(b) 2 only
(c) Both 1 and 2 Which of the statements given above is/are
(d) Neither 1 nor 2 correct? [UPSC CSE Pre. 2012]
(a) 1 and 3 only
14. In India, Judicial Review implies (b) 3 and 4 only
[UPSC CSE Pre. 2017] (c) 4 only
(a) The power of the Judiciary to pronounce (d) 1, 2, 3 and 4
upon the constitutionality of laws and 18. Which of the following are included in the
executive orders. original jurisdiction of the Supreme Court?
(b) The power of the Judiciary to question
1. A dispute between the Government of
the wisdom of the laws enacted by the
India and one or more States
Legislatures.
(c) The power of the Judiciary to review all 2. A dispute regarding elections to either
House of the Parliament or that of
the legislative enactments before they are
Legislature of a State
assented to by the President.
(d) The power of the Judiciary to review its 3. A dispute between the Government of
own judgements given earlier in similar India and a Union Territory
or different cases. 4. A dispute between two or more States
Select the correct answer using the codes
15. With reference to the ‘Gram Nyayalaya Act’, given below: [UPSC CSE Pre. 2012]
which of the following statements is/are
(a) 1 and 2 only
correct?
(b) 2 and 3 only
1. As per the Act, Gram Nyayalayas can hear (c) 1 and 4 only
only civil cases and not criminal cases. (d) 3 and 4 only
2. The Act allows local social activists as
mediators/ reconciliators. 19. How many High Courts in India have
jurisdiction over more than one State
Select the correct answer using the code given (Union Territories not included)?
below. [UPSC CSE Pre. 2016] [UPSC CSE Pre. 2012]
(a) 1 only (a) 2
(b) 2 only (b) 3
(c) Both 1 and 2 (c) 4
(d) Neither 1 nor 2 (d) 5
16. Who/Which of the following is the custodian 20. Consider the following statements:
of the Constitution of India?
The Supreme Court of India tenders advice
[UPSC CSE Pre. 2015] to the President of India on matters of law or
(a) The President of India fact
(b) The Prime Minister of India 1. On its own initiative (on any matter of
(c) The Lok Sabha Secretariat larger public interest).
(d) The Supreme Court of India 2. If he seeks such an advice.
17. What is the provision to safeguard the 3. Only if the matters relate to the
autonomy of the Supreme Court of India? Fundamental Rights of the citizens.

PYQ Workbook 126


INDIAN POLITY AND GOVERNANCE

Which of the statement given above is/are Which of the statements given above is/are
correct? [UPSC CSE Pre. 2011] correct? [UPSC CSE Pre 2007]
(a) 1 only (a) 1 only
(b) 2 only (b) 2 only
(c) 3 only (c) Both 1 and 2
(d) 1 and 2 (d) Neither 1 nor
21. With reference to Lok Adalat’s, which of the 25. Consider the following statements:
following statements is correct? 1. The Parliament cannot enlarge the
[UPSC CSE Pre 2010] Jurisdiction of the Supreme Court of
(a) Lok Adalat has the jurisdiction to settle India as its Jurisdiction is limited to that
the matters at the pre-litigation stage and conferred by the Constitution.
not those matters pending before any 2. The officers and servants of the Supreme
court. Court and High Courts are appointed
(b) Lok Adalat’s can deal with matters which by the concerned Chief Justice, and the
are civil and not criminal in nature. administrative expenses are charged on
(c) Every Lok Adalat consists of either the Consolidated Fund of India.
serving or retired judicial officers only Which of the statements given above is/are
and not any other person. correct? [UPSC CSE Pre 2005]
(d) None of the statements given above is
(a) 1 only
correct.
(b) 2 only
22. A Constitution (Amendment) Act may be (c) Both 1 and 2
declared unconstitutional by the Supreme (d) Neither 1 nor 2
Court of India if it- [UPSC CSE Pre 2009]
26. The power to enlarge the jurisdiction of
(a) Establishes three-tier federal set up in the Supreme Court of India with respect
place of the existing two tier-set up. to any matter included in the Union List of
(b) Removes Right of Equality before the law Legislative Powers rests with-
from Part III and places it elsewhere in
the Constitution. [UPSC CSE Pre 2003]
(c) Replaces Parliamentary System of (a) The President of India
Executive with the Presidential one. (b) The Chief Justice of India
(d) Establishes a Federal Court of Appeal to (c) The Parliament
lighten the burden of the Supreme Court. (d) The Union Ministry of Law, Justice and
Company Affairs
23. Consider the following statements:
1. Justice V.R. Krishna Iyer was the Chief 27. The concept of Public Interest Litigation
Justice of India. originated in: [UPSC CSE Pre 1997]
2. Justice V.R. Krishna Iyer is considered as (a) Australia
one of the progenitors of Public Interest (b) India
Litigation (PIL) in the Indian Judicial (c) The United States
System. (d) The United Kingdom
Which of the statement(s) given above is/are 28. Given below are two statements, one
correct? [UPSC CSE Pre 2008] labelled as Assertion (A) and the other
(a) 1 only labelled as Reason (R).
(b) 2 only Assertion (A): Willful disobedience or
(c) Both 1 and 2 non-compliance of Court orders and use of
(d) Neither 1 nor 2 derogatory language about Judicial behaviour
24. Consider the following statements: amount to Contempt of Court.
1. The mode of removal of a Judge of a High Reason (R): Judicial activism cannot be
Court in India is same as that of removal practiced without arming the Judiciary with
of a Judge of the Supreme Court. punitive powers to punish contemptuous
2. After retirement from the office, a behavior.
permanent Judge of a High Court cannot In the context of the above two statements
plead or act in any Court or before any which one of the following is correct?
authority in India. [UPSC CSE Pre 1997]

127 PYQ Workbook


INDIAN POLITY AND GOVERNANCE

(a) Both (A) and (R) are true and (R) is the Assertion (A): The Supreme Court of
correct explanation of (A). India has exclusive jurisdiction regarding
(b) Both (A) and (R) are true but (R) is not Constitutional Validity of Central laws.
the correct explanation of (A). Reason (R): The Supreme court is the
(c) (A) is true but (R) is false. Guardian of the Constitution of India.
(d) A) is false but (R) is true.
Select the correct answer from the codes
29. According to the Constitution of India the given below: [U.P.P.C.S. (Pre) 2019]
term ‘district judge’ shall not include: (a) Both (A) and (R) are true and (R) is the
[UPSC CSE Pre 1996] correct explanation of (A).
(a) Chief presidency magistrate (b) Both (A) and (R) are true, but (R) is not
(b) Sessions judges the correct explanation of (A).
(c) Tribunal judge (c) (A) is true, but (R) is false.
(d) Chief judge of a small cause court (d) (A) is false, but (R) is true.
30. When the Chief Justice of a High Court acts 33. Supreme Court judgement regarding
in an administrative capacity, he is subject homosexuality is related to which of the
to: [UPSC CSE Pre 1996] following? [U.P.P.C.S. (Pre) 2018]
(a) The writ jurisdiction of any of the other (a) Section 377 of IPC
judges of the High Court (b) Article 377 of the Indian Constitution
(b) Special control exercised by the Chief (c) Article 277 of the Indian Constitution
Justice of India (d) None of the Above
(c) Discretionary powers of the Governor of 34. In India, the power of Judicial Review is
the state enjoyed by? [U.P.P.C.S. (Mains) 2017]
(d) Special powers provided to the Chief
(a) The Supreme Court alone
Minister in this regard
(b) The Supreme Court as well as High
31. Which of the following statements Courts
regarding the Advisory Jurisdiction of the (c) All the Courts
Supreme Court is correct? (d) None of the above
1. It is binding on the Supreme Court to 35. Which of the following statements is not
give its opinion on any matter referred to true about the power of Supreme Court of
it by the President. India? [U.P. P.C.S. (Mains) 2017]
2. The full bench of the Supreme Court (a) It has the exclusive power to issue writs
hears any reference made to it under its to protect the fundamental rights of the
power of Advisory Jurisdiction. People.
3. The opinion given by the Supreme Court (b) It has original and exclusive jurisdiction
on a reference under Advisory Jurisdiction in inter-governmental disputes.
is not binding on the government. (c) It has advisory jurisdiction on a question
4. Not more than one reference at a time can of law or fact which may be referred to it
be made to the Supreme Court under its by the President of India.
power of Advisory Jurisdiction. (d) It has power to review its own judgment
Select the answer from the codes given below. or order.
[UPSC CSE Pre 1994] 36. Which among the following is not correct
(a) 1 and 2 about a High Court in India?
(b) 1 and 3 [U.P.P.C.S. (Mains) 2016]
(c) 2 and 3 (a) The second appeal to the High Court is
(d) 2 and 4 within its appellate jurisdiction.
(b) Every judge of a High Court is appointed
4.2. Other Examination Previous Years’
by the President.
Questions
(c) Governor of the State appoints the Judges
32. Given below are two statements, one is of a High Court.
labelled as Assertion (A) and the other as (d) High Courts may accept a petition related
Reason (R) to ‘Public interest litigation’.

PYQ Workbook 128


INDIAN POLITY AND GOVERNANCE

37. Curative Petition in India can he filed in 43. Which Article of the Constitution permits
Supreme Court under Article- the Supreme Court to review its judgement
[U.P.P.C.S. (Mains) 2014] or order? [U.P.P.C.S. (Mains) 2009]
(a) 138 (a) Article 137
(b) 140 (b) Article 130
(c) 142 (c) Article 139
(d) 146 (d) Article 138
44. The Supreme Court of India is a ‘Court of
38. A Judge of the Supreme Court may resign
Record’. It implies that-
his officeby writing a letter to-
[U.P.P.C.S. (Pre) 2008]
[U.P.P.C.S. (Pre) 2014]
(a) It has to keep a record of its decisions
(a) The Chief Justice (b) All its decisions have evidentiary value
(b) The President and cannot be questioned in any court
(c) The Prime Minister (c) It has the power to punish for its contempt
(d) The Law Minister (d) No appeal can be made against its
39. Consider the following statements and state decisions
which one of them is correct? 45. Which of the following Courts in India is/
[U.P.P.C.S. (Mains) 2013] are known as the Court (s) of Record?
(a) Supreme Court of India has only Original [U.P.P.C.S. (Mains) 2008]
Jurisdiction (a) The High Court’s only
(b) It has only Original and Appellate (b) The Supreme Court only
Jurisdiction (c) The High Courts and the Supreme Court
(c) It has only Advisory and Appellate (d) The District Courts
Jurisdiction 46. The Supreme Court holds its meetings in
(d) It has Original, Appellate as well as New Delhi, but it can meet elsewhere-
Advisory Jurisdiction [U.P.P.C.S. (Spl) (Mains) 2008]
40. Under which law is it prescribed that all (a) With the approval of President.
proceedings in the Supreme Court of India (b) If most of the Judges of Supreme Court
shall be in English language? decide.
(c) With the approval of Parliament.
[U.P.P.C.S. (Mains) 2013] (d) On the request of State Legislature.
(a) The Supreme Court rules, 1966
(b) Article 145 of the Constitution of India 47. Salaries of the Judges of the Supreme Court
(c) A Legislation made by Parliament are determined by-
(d) Article 348 of the Constitution of India [U.P.P.C.S. (Mains) 2008]
(a) Pay Commission appointed by the
41. The pension of a retired High Court Judge is President
charged to the- [U.P.P.C.S. (Mains) 2013] (b) Law Commission
(a) Consolidated Fund of India. (c) Parliament
(b) Consolidated Fund of the State where he (d) Council of Ministers
last served. 48. The second largest bench Constituted by
(c) Consolidated Fund of the different States the Supreme Court till date was in the-
where he has served. [U.P.P.C.S. (Spl) (Mains) 2004]
(d) Contingency Fund of India.
(a) Golaknath Case
42. In which of the following cases, the (b) Minerva Mills Case
Supreme Court held that, “Fundamental (c) Bank Nationalization Case
Rights enable a man to chalk out his life in (d) T.M.A. Pai Foundation Case
the manner he likes best”? 49. Consider the following statements and
[U.P.P.C.S.(Pre) 2012] choose the correct answer from the code
(a) Indira Gandhi Vs. Raj Narain given below:
(b) Golaknath Vs. The State of Punjab Assertion (A): Public interest litigation
(c) Bank Nationalization Case permits public-minded citizens to reach the
(d) Azhar Vs. Municipal Corporation Court of Law.

129 PYQ Workbook


INDIAN POLITY AND GOVERNANCE

Reason (R): The public-minded people may 1. Article 32


seek Justice for the person who is unable to 2. Article 226
reach the Court of Law for any reasons. 3. Article 227
Select the correct answer using the code given 4. Article 245
below: [U.P.P.C.S. (Spl) (Mains) 2004] Code: [U.P.P.C.S. (Pre) 1999]
(a) Both (A) and (R) are true, and (R) is the (a) 1, 2 and 3
correct explanation of (A). (b) 1, 3 and 4
(b) Both (A) and (R) are true, but (R) is not a (c) 1, 2 and 4
correct explanation of (A). (d) 1, 2, 3 and 4
(c) (A) is true, but (R) is false. 54. The Judges of the Supreme Court, after
(d) (A) is false, but (R) is true. retirement, are permitted to carry on
50. “I will bear true faith and allegiance to the practice before-
Constitution of India……... uphold the [U.P.P.C.S. (Pre) 1997]
Sovereignty and Integrity of India……... (a) Supreme Court only
Perform the duties of my office……... (b) High Courts only
uphold the Constitution and Law.” It is the (c) Both Supreme Court and High Court
form of Oath taken by- (d) None of the Courts
[UPPCS (Spl) (Mains) 2004] 55. The power of High Court to issue writ
(a) The President of India covers: [U.P.P.C.S. (Pre) 1997]
(b) The Chief Justice of India (a) Constitutional Rights
(c) The Member of Parliament (b) Statutory Rights
(d) The Governor (c) Fundamental right
(d) All of the above
51. Consider the following statements and
select the correct answer from the code 56. Who appoints the Judges of the Supreme
given: Court of India? [63rd B.P.S.C. (Pre) 2017]
Assertion (A): High Courts are in better (a) The Prime Minister
position to protect rights of Indian citizens (b) The President
than the Supreme Court. (c) The Chief Justice of India
(d) The Ombudsman
Reason (R): Supreme Court can issue writs to (e) None of the above / more than one of the
protect only Fundamental Rights. above
Code: [U.P.P.C.S. (Mains) 2004]
57. A High Court Judge addresses his letter of
(a) Both (A) and (R) are true and (R) is the
resignation to-
correct explanation of (A).
(b) Both (A) and (R) are true but (R) is not a [48th to 52nd B.P.S.C. (Pre) 2008]
correct explanation of (A). (a) The President
(c) (A) is true but, (R) is false. (b) The Chief Justice of India
(d) (A) is false but, (R) is true. (c) The Chief Justice of his High Court
(d) The Governor of the State
52. Acting Chief Justice of India is appointed
58. Supreme Court in India was established-
by the- [UPPCS (Pre) 2000]
[42nd B.P.S.C. (Pre) 1997]
(a) Chief Justice of India
(a) By an Act of Parliament in 1950
(b) Chief Justice of India with previous
(b) Under Indian Independence Act, 1947
consent of the President
(c) Under Indian Government Act, 1953
(c) President (d) By the Indian Constitution
(d) President on consultation with the Chief
Justice of India 59. Which one of the following is correct in
respect of the appointment of District
53. Which of the following Articles of the Judges? [CAPF 2022]
Constitution of India have been declared by (a) They are appointed by the Chief Justice of
the Supreme Court as the ‘Inviolable basic the concerned High Court.
structure’ of the Constitution: (b) They are appointed by the President of
Select the correct answer from the code given India on the advice of the Chief Justice of
below: the concerned High Court.

PYQ Workbook 130


INDIAN POLITY AND GOVERNANCE

(c) They are appointed by the Governor in (a) Kesavananda Bharati vs. State of Kerala
consultation with the High Court of the (b) SP Gupta vs. President of India
concerned State. (c) Maneka Gandhi vs. Union of India
(d) They are appointed by the Supreme Court (d) SR Bommai vs. Union of India
on the advice of the concerned High 65. Which one of the following judgements
Court. declared that the Parliament has NO power
60. Who can be appointed as a Judge of the to amend any of the provisions of Part III
Supreme Court? [CAPF 2021] of the Constitution of India? [CAPF 2017]
(a) An advocate who has practised for at least (a) Kesavananda Bharati Vs. State of Kerala
ten years in any court (b) Golak Nath Vs. State of Punjab
(b) A Judge of a High Court for at least three (c) Champakam Dorairajan Vs. State of
years Madras
(c) A distinguished jurist in the opinion of (d) Minerva Mills Limited Vs .Government
the President of India of India
(d) A person who has held a judicial office 66. Which one of the following cases in the
for at least fifteen years Supreme Court of India dealt with the
61. Which one of the following statements issue of ‘creamy layer’ among the backward
about the Supreme Court is not correct? classes? [CAPF 2017]
[CAPF 2021] (a) KM Nanavati Vs State of Bombay
(a) Under Article 129 and Article 144 of the (b) Indra Sawhney Vs Union of India
Constitution of India, the Supreme Court (c) Madhu Limaye Vs Ved Murti
has been vested with power to punish for (d) Sajjan Singh Vs State of Punjab
Contempt of court. 67. The Locus Standi rule to move the court
(b) The Supreme Court has been vested with was liberalized by the case of?
advisory jurisdiction. [CAPF 2015]
(c) The Supreme Court may refuse to provide (a) SR Bommai vs Union of India
opinion to the President, if so asked, in (b) Minerva Mills vs Union of India
certain circumstances. (c) SP Gupta vs Union of India
(d) Law declared by the Supreme Court is (d) Kesavananda Bharati vs State of Kerala
binding on all courts within the Territory
of India 68. Which of the following conditions is/are
necessary for the issue of a writ of certiorari
62. Which one of the following powers of the in India?
Supreme Court is also conferred on a High 1. There should be a tribunal or an officer
Court? [CAPF 2021] having legal authority to determine
(a) Advisory power questions affecting rights of subjects and
(b) Power of doing complete justice in any having a duty to act judicially.
cause or matter pending before the court 2. Such tribunal or officer has acted without
(c) Power of court of record jurisdiction.
(d) Power to allow/refuse Special Leave
Select the correct answer using the code given
Petitions
below. [CDS 2023 (I)]
63. Which one of the following authorities (a) 1 only
CANNOT organize ‘Lok Adalats”? (b) 2 only
[CAPF 2019] (c) Both 1 and 2
(a) Panchayat Committees (d) Neither 1 nor 2
(b) Taluk Legal Services Committee 69. Who among the following Chief Justices of
(c) District Authority India ordered the constitution of a Special
(d) High Court Legal Services Committee Bench called ‘Social Justice Bench’ ?
64. Which one of the following judgements is [CDS 2023 (I)]
associated with the primacy of the Chief (a) Justice H. L. Dattu
Justice of India and the Collegium of Judges (b) Justice K. G. Balakrishnan
in the appointment and transfer of the (c) Justice R. M. Lodha
higher judiciary? [CAPF 2018] (d) Justice Y. K. Sabharwal

131 PYQ Workbook


INDIAN POLITY AND GOVERNANCE

70. The issue, ‘whether Section 377 of the (a) The Office must be a Public Office.
Indian Penal Code is violative of Articles (b) The Office must be created by the Statute
21, 14 and 15 of the Constitution of India’ or by the Constitution itself.
was decided in which one of the following (c) The Office must not be a substantive one.
cases? [CDS 2022 (I)] (d) There has been a contravention of the
(a) Shreya Singhal vs. Union of India Constitution or a Statute in appropriating
(b) Vishakha vs. State of Rajasthan such person to that Office.
(c) Shayara Bano vs. Union of India 77. Which among the following statements
(d) Naz Foundation vs. Government of NCT about the power to change the basic
of Delhi and others structure of the Constitution of India is/are
correct?
71. Which one of the following writs means
‘you may have the body? [CDS 2022 (I)] 1. It falls outside the scope of the amending
powers of the Parliament.
(a) Habeas Corpus
2. It can be exercised by the people through
(b) Mandamus
representative in a Constituent Assembly.
(c) Certiorari
(d) Quo Warranto 3. It falls within the constituent powers of
the Parliament.
72. Article 231 of the Constitution of India Select the correct answer using the codes
grants power to establish a common High given below: [CDS Pre. 2019 I]
Court for two or more states to (a) 1 and 3
[CDS 2021 (I)] (b) 1 and 2
(a) the Parliament (c) Only 1
(b) the Supreme Court (d) 2 and 3
(c) the President of India 78. In which one of the following judgments of
(d) the Union Cabinet the Constitutional Bench of the Supreme
73. What is the ground on which the Supreme Court of India, the rarest of rare principle
Court can refuse relief under Article-32? in the award of death penalty was first laid
[CDS 2020 (I)] down? [CDS Pre. 2016 I]
(a) The aggrieved person can get 20s remedy (a) Bachan Singh Vs State of Punjab (1980)
from another court. (b) Gopalanachari Vs State of Kerala (1980)
(b) That disputed facts have to be investigated. (c) Dr. Upendra Baxi Vs State of Uttar
(c) That no Fundamental Right has been Pradesh (1983)
(d) Tukaram Vs State of Maharashtra (1979)
infringed.
(d) That the petitioner has not asked for the 79. In which one of the following cases, the
proper writ applicable to his/her case. constitutional validity of the Muslim
Women (Protection of Rights of Divorce)
74. The power of the Supreme Court to decide Act, 1986, was upheld by the Supreme Court
in the case of a dispute between two or more of India? [CDS Pre.2016 I]
States is called [CDS 2020 (I)] (a) Muhammad Ahmed Khan Vs Shah Bano
(a) original jurisdiction Begum
(b) inherent jurisdiction (b) Daniel Latifi Vs Union of India
(c) plenary jurisdiction (c) Mary Roy vs State of Kerala
(d) advisory jurisdiction (d) Shankari Prasad Vs Union of India
75. In the Indian judicial system, writs are 80. The basic structure doctrine with regard to
issued by [CDS 2020 (II)] the Constitution of India relates to
(a) the Supreme Court only 1. the power of judicial review.
(b) the High Courts only 2. the judgement in Kesavananda Bharati
(c) the Supreme Court and High Courts only case (1973).
(d) the Supreme Court, High Courts and 3. the constraints on Article-368 of the
Lower Courts Constitution of India.
76. Which one of the following is not the 4. the judgement in Golaknath case (1967).
necessary condition for the issue to a Writs Which of the statement(s) given above is/are
of Quo Warranto? [CDS 2020 (II)] correct? [CDS II- 2016]

PYQ Workbook 132


INDIAN POLITY AND GOVERNANCE

(a) 1, 2 and 3 84. Judicial review in the Indian Constitution is


(b) 2 and 4 based on- [Chhattisgarh P.C.S. (Pre) 2015]
(c) 1 and 3 (a) Due Process of Law
(d) All of these (b) Procedure established by Law
81. Which one of the following is not the powers (c) Rule of Law
of the Supreme Court? [CDS Pre. 2016 II] (d) Precedents and conventions
(a) The Supreme Court has original (e) None or the above
and exclusive jurisdiction in inter- 85. A Judge of the Supreme Court can be
governmental disputes. removed by the President of India after-
(b) The Supreme Court has advisory
[Chhattisgarh PCS (Pre) 2003]
jurisdiction on a question of law or
fact which may be referred to it by the (a) An enquiry by C.B.I.
President of India (b) An enquiry by Chief Justice of India
(c) The Supreme Court has the power to (c) A report by the Bar Council of India
review its own judgement or order (d) An impeachment by the Parliament
(d) The Supreme Court has the exclusive 86. Which of the following type of Judge/Judges
power to issue writs to protect the can be appointed in the Supreme Court of
fundamental rights of the people. India?
82. In which of the following cases did the (i) Ad hoc Judge
Supreme Court rule that Constitutional (ii) Additional Judge
Amendments were also laws under
Codes: [M.P.P.C.S. (Pre) 2020]
Article-13 of the Constitution of India,
which could be declared void for being (a) (i) is true and (ii) is false
inconsistent with Fundamental Rights? (b) (i) is false and (ii) is true
[CDS II - 2015] (c) Both (i) and (ii) are true
(a) Keshavanand Bharati Case (d) Both (i) and (ii) are false
(b) Golaknath Case 87. Who has the right to transfer any case
(c) Minerva Mills Case anywhere in India? [M.P.P.C.S. (Pre) 2010]
(d) Maneka Gandhi Case (a) President
83. In the following questions, there are two (b) Supreme Court
statements. One is labelled as Assertion (A) (c) High Court
and other is labelled as Reason (R): (d) None of these
Assertion (A): The District Judge is the 88. To become a Judge of Supreme Court, a
highest judicial authority in the district. person must be an advocate in High Court
Reason (R): The District Judge has both for at least how many years?
judicial and administrative powers. He [M.P.P.C.S. (Pre) 2002]
has also supervisory powers over all the (a) 20
subordinate courts in the district.
(b) 10
In the context of the above two statements (c) 8
which of the following is correct? (d) 25
[Chhattisgarh P.C.S. (Pre) 2021]
89. The case of Vishakha and others Vs. State of
(a) Both Assertion (A) and Reason (R)
are true, and Reason (R) is correct Rajasthan and others is related to-
explanation of Assertion (A) [R.A.S./R.T.S. (Pre) 2021]
(b) Both Assertion (A) and Reason (R) (a) Maternity leave in respect of working
are true, but Reason (R) is not correct women
explanation of Assertion (A) (b) Transfer policy for women
(c) Assertion (A) is true, but Reason (R) is (c) Prevention of the practice of dowry
false prevalent in the society
(d) Assertion (A) is false, but Reason (R) is (d) Prevention of sexual harassment of
true women at workplace

133 PYQ Workbook


INDIAN POLITY AND GOVERNANCE

90. Identify the incorrect pair of Case and 91. All the cases regarding interpretation of the
Ruling in that case by Supreme Court: Constitution can be brought to the Supreme
[R.A.S./R.T.S. (Pre) 2013] Court under its-
(a) Indira : Creamy Layer for Other [U.P. Lower Sub. (Mains) 2013]
Sawhney Backward Classes. (a) Original Jurisdiction
Case (b) Appellate Jurisdiction
(b) Vishakha : Protection of working (c) Advisory Jurisdiction
Case women against sexual (d) None of the above
harassment at their place
of work. 92. Where can the Public Interest Litigation
(c) Maneka : Articles 14, 19 and 21 are (PIL) be filed?
Gandhi not mutually exclusive. [Uttarakhand PCS (Pre) 2012]
Case (a) Only in Supreme Court of India
(d) Bella : The right to travel abroad (b) Only in High Courts of States
Banerjee is a part of personal (c) In Central Administrative Tribunals
Case liberty. (d) Both in High Courts & Supreme Court

PYQ Workbook 134


INDIAN POLITY AND GOVERNANCE

SOLUTIONS

Pursuant to the recommendations made by the H N Sanyal


4.1. UPSC CSE Previous Years’ Questions Committee, the Contempt of Courts Act, 1971 was passed.
1. Solution: (a) Statement 2 is correct: The Supreme Court and the High
Exp) Option a is the correct answer Courts of India have been empowered with the power to
The Prisons Act 1894 is one of the oldest pieces of legislation penalize for Contempt of Court under Articles 129 and 215
in India dealing with laws enacted in relation to prisons in of the Constitution of India.
India. Statement 3 is incorrect: The Constitution of India does
Statement 1 is correct: Prison is a State subject under List- not define Civil Contempt and/or Criminal Contempt.
II of the Seventh Schedule in the Constitution.. States have Statement 4 is correct: Article 142 (2) of the Indian
the primary role, responsibility and power to change the Constitution clearly states that “subject to the provisions of
current prison laws, rules and regulations. any law made on this behalf by Parliament” the Supreme
Hence it is true that prisons are managed by the state Court shall have all and every power to make any order on the
governments with their own rules and regulations. punishment of any contempt of itself. Thus, the parliament
is vested with powers to make laws on Contempt of Court.
Statement 2 is correct: The management and administration
of Prisons falls exclusively in the domain of the State 4. Solution: (b)
Governments and is governed by the Prisons Act, 1894 Exp) Option b is the correct answer
and the Prison Manuals of the respective State Governments.
Statement 1 is incorrect: Corporate Lawyers, as well as
Hence it is true that the Prisons Act 1894 kept the subject
patent attorneys, are too recognized as lawyers and there’s
of prisons in the Provincial government (State governments)
no prohibition on their recognition as lawyers.
and this explains why State governments have the power
to frame their own rules and regulations regarding prison Statement 2 is correct: The Bar Council of India visits and
management. inspects Universities/Law colleges in the country as part
of its statutory function of promoting legal education and
2. Solution: (d) laying down standards in consultation with the Universities
Exp) Option d is the correct answer in India and the State Bar Councils. The Bar Council of
Statement 1 is correct: Mandamus is a command issued India is a statutory body established under section 4 of
by the court to a public official asking him to perform his the Advocates Act 1961 that regulates the legal practice and
official duties that he has failed or refused to perform. It can legal education in India.
also be issued against any public body, a corporation, an 5. Solution: (b)
inferior court, a tribunal, or a government for the same
Exp) Option b is the correct answer.
purpose. It is usually not usable against a private entity.
The provisions for custody in India are governed by Section
Statement 2 is correct: Mandamus cannot be used against
167 of the Code of Criminal Procedure. A person may be
a government company which is incorporated under
held in the custody of the police or in judicial custody. The
the Companies Act as there is neither a statutory nor a
first thing that happens to a suspect on arrest is that he is
public duty imposed on it by a statute in respect of which
taken into police custody, following which he is taken before
enforcement could be sought by means of a mandamus.
a magistrate. From there, he may either be remanded to
Statement 3 is correct: Quo Warranto is issued by the judicial custody or be sent back into police custody.
court to enquire into the legality of the claim of a person
Statement 1 is incorrect. Police Custody means that police
to a public office. Hence, it prevents illegal usurpation of
has the physical custody of the accused while Judicial
public office by a person. Unlike the other writs, this can be
Custody means an accused is in the custody of the
sought by any interested person and not necessarily by the
concerned Magistrate. In Police custody, the accused is
aggrieved person
lodged in police station lockup while in Judicial Custody,
3. Solution: (b) he is locked up in the jail.
Exp) Option b is the correct answer Statement 2 is correct. During Judicial Custody, the police
Statement 1 is correct: A committee was set up in 1961 officer in charge of the case is not allowed to interrogate the
under the chairmanship of the late H N Sanyal, the suspect. However, the court may allow the interrogations
then additional solicitor general. The committee made to be conducted if it opines the interrogation being necessary
a comprehensive examination of the law and problems under the facts produced before the court.
relating to contempt of court in the light of the position However, during police custody, the police officer in charge
obtained in our own country and various foreign countries. of the case, may interrogate the suspect.

135 PYQ Workbook


INDIAN POLITY AND GOVERNANCE

6. Solution: (b) 8. Solution: (c)


Exp) Option b is the correct answer. Exp) Option c is the correct answer.
Statement 1 is incorrect. Parole is a system of releasing a Statement 1 is incorrect. According to provisions of Judges
prisoner with suspension of the sentence. Parole cannot (Inquiry) Act, 1968, the motion to impeach a judge of the
be claimed as a matter of right. Parole may be denied to a Supreme Court of India can be rejected by the Speaker of
prisoner even when he makes out sufficient cause for release the Lok Sabha.
on parole. Statement 2 is incorrect. The Constitution of India does
Statement 2 is correct. State governments have their not define or give details of what constitutes ‘incapacity and
own Prisoner Release on Parole Rules. ‘Prisons’ is a State proved misbehaviour’ of the Judges of the Supreme Court of
India.
subject under the State List of the Seventh Schedule of the
Constitution. Statement 3 is correct. The Judges Enquiry Act (1968)
regulates the procedure relating to the removal of a judge of
The management and administration of prisons fall
the Supreme Court by the process of impeachment. It states
exclusively in the domain of state governments and are
that a removal motion signed by 100 members (in the case of
governed by the Prisons Act, 1894, and the Prison Manuals
Lok Sabha) or 50 members (in the case of Rajya Sabha) is to
of the respective state governments.
be given to the Speaker/ Chairman.
The Prisons Act further says that only states can make rules Statement 4 is correct. If the motion for the impeachment
regarding the release of prisoners on furlough, parole and of a Judge is taken up for voting, the law requires the motion
remission, as part of the correctional process in jail reforms. to be backed by each House of the Parliament and supported
Important Tips by a majority of total membership of that House and by not
less than two-thirds of total members of that House present
Parole is to be granted only on a sufficient cause such
and voting. After the motion is passed by each House of
as cases of severe illness or death of any member of the
Parliament, an address is presented to the president for
prisoner’s family. The granting authority for parole is the
removal of the judge. Finally, the president passes an order
deputy home secretary of the state government.
removing the judge.
On the other hand, Furlough is seen as a matter of right
for a prisoner, to be granted periodically irrespective of 9. Solution: (b)
any reason. It is granted merely to enable the prisoner to Exp) Option b is the correct answer.
retain family and social ties, and to counter the ill-effects
Directive Principles of State Policy (DPSP) is enshrined in
of prolonged time spent in prison.
Part-IV (Article 36 to 51) of the constitution.
Both Parole and Furlough can be denied if the competent
According to Article 50, the State shall take steps to separate
authority is of the opinion that releasing the inmate would
the judiciary from the executive in the public services of the
not be in the interest of society.
State
7. Solution: (a)
Important Tips
Exp) Option a is the correct answer. The Criminal Procedure Code (1973) separated the
Statement 1 is correct. The Chief Justice of India can judiciary from the executive in the public services of
request a retired judge of the Supreme Court or a retired the state. Prior to this separation, the district authorities
judge of a high court (duly qualified for appointment as like the collector, the sub-divisional officer, the tehsildar
a Supreme Court judge) to act as a judge of the Supreme and so on used to exercise judicial powers along with the
Court for a temporary period. He can do so only with the traditional executive powers. After the separation, the
previous consent of the President and also of the person to judicial powers were taken away from these executive
be so appointed. Such a judge is entitled to such allowances authorities and vested in the hands of district judicial
as the president may determine. He will also enjoy all the magistrates who work under the direct control of the state
high court.
jurisdiction, powers and privileges of a judge of Supreme
Court. 10. Solution: (d)
Statement 2 is incorrect. The Supreme Court of India has Exp) Option d is the correct answer.
power to review its own judgement or order. Thus, it is not Statement 1 is incorrect. The basic structure doctrine is not
bound by its previous decision and can depart from it in the defined by the Constitution. It is rather a judicial invention
interest of justice or community welfare. As a court of record, that came through Kesavananda Bharati vs State of Kerala
a High Court also has the power to review and correct its case (1973). Supreme Court said that the Constitution of
own judgement or order. However, the power to review of India has certain basic features that cannot be altered or
High Court is not similar to the Supreme Court because destroyed through the provisions of amendment, by the
the Supreme Court’s power to review is consitutionally Parliament of India, as mentioned in the Constitution of
guaraneted but not in the case of High Court. India.

PYQ Workbook 136


INDIAN POLITY AND GOVERNANCE

Statement 2 is incorrect. The term judicial review is not (NJAC). This was struck down by the Supreme Court as
found explicitly anywhere in the Indian constitution. being violative of the independence of judiciary.
However, Article 13 of the Constitution empowers the SC
14. Solution: (a)
to review any law in India and strike it down if it violates
the Constitutional provisions, especially Part III (on Exp) Option a is the correct answer.
Fundamental Rights), thus providing the power of judicial Judicial Review is the power of the Courts to determine
review indirectly. the constitutionality of Legislative act and executions
orders. Judicial review is the power of the judiciary to
11. Solution: (a)
examine the constitutionality of legislative enactments and
Exp) Option a is the correct answer. executive orders of both the Central and State governments.
The National Legal Services Authority (NALSA) has been On examination, if they are found to be violative of the
constituted under the Legal Services Authorities Act, 1987 Constitution (ultra vires), they can be declared as illegal,
to provide free Legal Services to the weaker sections of the unconstitutional and invalid (null and void) by the judiciary.
society and to organize Lok Adalats for amicable settlement Consequently, they cannot be enforced by the Government.
of disputes. Important Tips
The Income Ceiling Limit prescribed under Section 12(h) The scope of judicial review before Indian courts has
of the Act for availing free legal services depend on the evolved in three dimensions:
limits set by the respective State Governments. Thus, Senior
citizens’ eligibility for free legal aid depends on the Rules
1. to ensure fairness in administrative action.
framed by the respective State Governments in this regard. 2. to protect the constitutionally guaranteed
The Income Ceiling Limit prescribed for availing free legal
fundamental rights of citizens
services in is: 3. to rule on questions of legislative competence
General –Rs. 1,00,000, Senior citizen- Rs. 2,00,000,
between the centre and the states
Transgender – Rs. 2,00,000. 15. Solution: (b)
Free legal aid is available to a member of a Scheduled Caste Exp) Option b is the correct answer.
or Scheduled Tribe, not OBC. Statement 1 is incorrect. Gram Nyayalayas are established
12. Solution: (d) under the Gram Nyayalayas Act, 2008 for speedy and easy
access to justice system in the rural areas. They can try
Exp) Option d is the correct answer.
criminal cases as well as civil suits, claims or disputes
Statement 1 is incorrect. With the power of judicial review, which are specified in the First Schedule and the Second
High Courts can strike down State law as well as Central Schedule of the Act.
laws. Though the phrase ‘judicial review’ has no where been
Statement 2 is correct. The Act explicitly provides for
used in the Constitution, the provisions of Articles 13 and
mediation by social workers. The Gram Nyayalaya shall not
226 explicitly confer the power of judicial review on a high
be bound by the rules of evidence provided in the Indian
court.
Evidence Act, 1872 but shall be guided by the principles of
Statement 2 is incorrect. The Supreme Court in Kesavananda natural justice and subject to any rule made by the High
Bharati case (1973) held that Constitutional amendment can Court.
be challenged on the ground that it violates a fundamental
right that forms a part of the ‘basic structure’ of the 16. Solution: (d)
Constitution and hence, can be declared as void. Therefore, Exp) Option d is the correct answer.
as it stands the constitutional amendment comes under a law The Constitution has conferred a very extensive jurisdiction
for judicial review if it violates any fundamental rights and and vast powers on the Supreme Court. It is not only a
Basic structure of our constitution. Federal Court like the American Supreme Court but also
a final court of appeal like the British House of Lords (the
13. Solution: (b)
Upper House of the British Parliament). It is also the final
Exp) Option b is the correct answer. interpreter and guardian/custodian of the Constitution
Statement 1 is incorrect. The 44 th Amendment to the and guarantor of the fundamental rights of the citizens.
Constitution of India had no provision placing the election of The Supreme Court ensures that the other branches of
the Prime Minister beyond judicial review. 44 th Amendment government perform their responsibilities in accordance
to the Constitution of India was enacted to nullify some of with the Constitution. The Supreme Court also ensures that
the amendments made by the 42 nd Amendment Act, 1976. the Constitution of India remains the law of the land and
Statement 2 is correct. 99th Constitutional Amendment the parliament doesn’t affect or in any way undo the soul of
Act, 2014 was enacted to reflect the change in the system of the Constitution through its basic structure doctrine. This
appointments of Judges from the collegium system. It sought also ensures that there aren’t any unreasonable restrictions
to establish the National Judicial Appointment Commission placed on fundamental rights of individuals by the state.

137 PYQ Workbook


INDIAN POLITY AND GOVERNANCE

17. Solution: (a) • Bombay High Court has the jurisdiction over
Exp) Option a is the correct answer. Maharasthra, Goa, Dada and Nagar Haveli and Daman
and Diu.
Statement 1 is correct. The judges of the Supreme Court
are appointed by the President in consultation with the • Guwahati High Court, which was earlier known as
Chief Justice of India. This provision curtails the absolute Assam High Court, has the jurisdiction over Assam,
discretion of the executive as well as ensures that the judicial Nagaland, Mizoram and Arunachal Pradesh.
appointments are not based on any political or practical • Punjab and Haryana High Court has the jurisdiction
considerations. over Punjab, Haryana and Chandigarh.
Statement 2 is incorrect. A judge of the Supreme Court
can be removed from his Office by an order of the 20. Solution: (b)
president. The President can issue the removal order only Exp) Option b is the correct answer.
after an address by Parliament has been presented to him The Supreme Court of India tenders advice to the President
in the same session for such removal. The address must be of India on matters of law or fact only if the President seeks
supported by a special majority of each House of Parliament.
such an advice (and not on its own initiative).
Statement 3 is correct. The salaries, allowances and
pensions of the judges and the staff as well as all the The Constitution (Article 143) authorises the president to
administrative expenses of the Supreme Court are charged seek the opinion of the Supreme Court in the two categories
on the Consolidated Fund of India. Thus, they are non- of matters:
votable by the Parliament (though they can be discussed by 1) On any question of law or fact of public importance which
it). has arisen or which is likely to arise. In this situation, the
Statement 4 is incorrect. All appointments of officers and Supreme Court may tender or may refuse to tender its
staffs of the Supreme Court of India are made by the Chief opinion to the president.
Justice of India. To maintain the independence of the 2) On any dispute arising out of any pre-constitution
judiciary the Chief Justice of India has been given powers to treaty, agreement, covenant, engagement, sanad or other
appoint officers and servants of the Supreme Court without similar. In this situation, the Supreme Court must tender
any interference from the executive. He can also prescribe
its opinion to the president.
their conditions of service.
In both the cases, the opinion expressed by the Supreme
Important Tips Court is only advisory and not a judicial pronouncement.
The power to increase the number of judges in the Hence, it is not binding on the president; he may follow
Supreme Court of India is vested in the parliament. At or may not follow the opinion. However, it facilitates the
present, the Supreme Court consists of thirty-four judges government to have an authoritative legal opinion on a
(one chief justice and thirty-three other judges). In 2019, matter to be decided by it.
the centre notified an increase in the number of Supreme The President can seek advice of Supreme Court on matters
Court judges from thirty-one to thirty-four, including related to Fundamental rights but it is not the only matter on
the Chief Justice of India. This followed the enactment of which President can seek advice.
the Supreme Court (Number of Judges) Amendment Act,
2019. Originally, the strength of the Supreme Court was 21. Solution: (d)
fixed at eight (one chief justice and seven other judges). Exp) Option d is the correct answer.

18. Solution: (c) Option a is incorrect-Lok Adalats have the jurisdiction to


settle both pre-litigation and pending cases. They are an
Exp) Option c is the correct answer.
alternative dispute resolution mechanism aimed at resolving
As a federal court, the Supreme Court decides the disputes disputes outside the regular court process. They can take up
between different units of the Indian Federation. More cases that are yet to be filed in court (pre-litigation) as well
elaborately, the Supreme Court has exclusive original as cases that are already pending before the courts.
jurisdiction in the cases where there is any dispute:
Option b is incorrect- While Lok Adalats are more
(a) Between the Centre and one or more states; or
commonly associated with settling civil disputes, they can
(b) Between the Centre and any state or states on one side also handle certain criminal cases that are compoundable
and one or more other states on the other side; or offenses. Compoundable offenses are those offenses where
(c) Between two or more states. the victim and the accused can reach a compromise, and the
Exclusive means, no other court can decide such disputes case can be withdrawn.
and original means, the power to hear such disputes in the Option c is incorrect- While Lok Adalats do have the
first instance, not by way of appeal. participation of judicial officers, they are not exclusively
composed of serving or retired judicial officers. Lok Adalats
19. Solution: (b)
typically involve a panel of members, which may include
Exp) Option b is the correct answer. serving or retired judicial officers, lawyers, social workers,
There are 21 High Courts in the country, out of these 3 have or any other person with legal expertise or experience in
jurisdiction over more than one state. dispute resolution.

PYQ Workbook 138


INDIAN POLITY AND GOVERNANCE

22. Solution: (b)


• Fixed service conditions: The salaries, allowances,
Exp) Option b is the correct answer privileges, leave and pension of the judges of a High
Right to equality forms part of basic structure of Constitution. Court are determined by Parliament. These cannot be
As per Keshavananda Bharati case basic structure of changed to their disadvantage after their appointment.
constitution cannot be amended. Hence removing Right to This ensures that judges can enjoy a reasonable
Equality from part III and placing it elsewhere in constitution standard of living and do not have to worry about their
may be declared as unconstitutional by Supreme Court of financial security.
India. • Expenses charged on Consolidated Fund: The
salaries and allowances of the judges of a High
23. Solution: (b) Court are charged on the Consolidated Fund of
Exp) Option b is the correct answer India. This means that they are not subject to the
Justice V. R. Krishna Iyer was a judge and Minister who annual budgetary process and cannot be reduced
reformed the Indian criminal justice system. He stood or abolished by Parliament. This ensures that the
financial independence of the judges is protected.
up for the poor and the underprivileged, and remained a
human rights champion, a crusader for social justice and • Prohibition of discussion in Parliament: The conduct
the environment, and a doyen of civil liberties, throughout of the judges of a High Court cannot be discussed in
his life. Justice V.R. Krishna Iyer and Justice P.N. Bhagwati Parliament. This ensures that the judges can discharge
are considered the pioneer/ progenitors of Public Interest their duties without fear of being subjected to political
Litigation (PIL). pressure

24. Solution: (a) 25. Solution: (d)

Exp) Option a is the correct answer. Exp) Option d is the correct answer

Statement 1 is correct- The mode of removal of a Judge of a Statement 1 is incorrect: Under Article 138 of the
High Court in India is same as that of removal of a Judge of Constitution of India, the Parliament by law can enlarge the
jurisdiction and power of the Supreme Court on any matter
the Supreme Court. Both judges can be removed by a process
in the Union List.
of impeachment, which is initiated by a motion -a removal
motion signed by 100 members (in the case of Lok Sabha) Statement 2 is incorrect: As per Article 146 and 229,
or 50 members (in the case of Rajya Sabha) and passed by appointments of officers and servants of the Supreme
Special majority once admitted by the Presiding officer of Court and High Courts are made by their respective
the House. The ground of removal is proved misbehavior Chief Justices. The conditions of service are governed
or incapacity. by rules made by the Chief Justices, with salary-related
rules requiring the President’s or Governor’s approval.
Statement 2 is incorrect- According to the constitution, a
The administrative expenses of the Supreme Court are
retired High Court judge is not allowed to practice law in any
charged on the Consolidated Fund of India, while the
court or before any authority in India, except the Supreme administrative expenses of High Courts are charged on the
Court and other High Courts. This restriction implies that Consolidated Fund of states.
the retired judge cannot practice in lower courts or in the
High Court from which he/she has retired. 26. Solution: (c)
Exp) Option c is the correct answer
Important Tips
Under Article 138 of the Constitution of India, the Parliament
Some of the provision for independence of Judge of High
by law can enlarge the jurisdiction and power of the Supreme
Court:
Court on any matter in the Union List.
• Mode of appointment: The judges of a High Court are
appointed by the President of India in consultation 27. Solution: (c)
with the Chief Justice of India and the Chief Justice Exp) Option c is the correct answer
of the High Court. This ensures that the appointment
The concept of Public Interest Litigation (PIL) originated
is made on the basis of merit and not on political
and developed in the USA in the 1960s. In the USA, it
considerations.
was designed to provide legal representation to previously
• Security of tenure: The judges of a High Court hold unrepresented groups or interests.
office until they attain the age of 62 years. They can
only be removed from office by the President on the 28. Solution: (b)
grounds of proved misbehaviour or incapacity, after Exp) Option b is the correct answer.
an inquiry into the matter by a Supreme Court judge.
Assertion (A) is true- Willful disobedience or non-
This ensures that judges can discharge their duties
compliance of Court orders and use of derogatory language
without fear of being removed from office for political
about Judicial behaviour amount to Contempt of Court. This
or other reasons
is because contempt of court is any act that tends to interfere

139 PYQ Workbook


INDIAN POLITY AND GOVERNANCE

with the administration of justice or to bring the authority judges to issue writs such as habeas corpus, mandamus,
and dignity of the court into disrespect. certiorari, prohibition, and quo warranto, which are used
Reason (R) is true- Judicial activism cannot be practiced to safeguard the fundamental rights of citizens and to ensure
without arming the Judiciary with punitive powers to punish that public authorities act within the bounds of law.
contemptuous behavior. This is because judicial activism is
31. Solution: (c)
the practice of judges. taking an active role in interpreting
the law and issuing rulings that have a significant impact on Exp) Option c is the correct answer
society. In order to be able to do this effectively, judges need Under Article 143 of the Constitution the President is
to have the power to punish people who try to interfere with authorised to seek the opinion of the Supreme Court on the
the administration of justice. following matters:

29. Solution: (c) 1. Any question of law or fact of public importance

Exp) Option c is the correct answer. 2. any dispute arising out of any pre constitution treaty,
agreement, covenant, agreement etc
As per Article 236(a) of the Constitution, the term “district
judge” includes various judicial positions, such as Judge of The supreme court may give or refuse to give its opinion
a city civil court, additional district judge, joint district in the first case whereas it must give its advisory opinion
judge, assistant district judge, chief judge of a small in the second case. The advice so given is not binding on
cause court, chief presidency magistrate, additional chief the President. The minimum number of judges to sit on the
presidency magistrate, session judge, additional session Constitutional Bench or on Bench which gives its advisory
judge, and assistant session judge. However, it does not opinion on the reference by the President must be one half of
encompass the position of a Tribunal Judge. Tribunal judges the total strength of the Supreme Court.
preside over specialized tribunals that deal with specific
areas of law and are not part of the regular district judiciary 4.2. Other Examination Previous Years’
Questions
Important Tips
Articles 233-237 of the Indian Constitution pertain to the 32. Solution: (d)
regulations governing subordinate courts.
Exp) Option d is the correct answer
1. Article 233- This article addresses the appointment
Under Article 131A the Supreme Court earlier had exclusive
of district judges. The Governor of the State,
in consultation with the High Court exercising jurisdiction to decide the constitutional validity of a Central
jurisdiction in that State, is responsible for appointing, law and thus deprived the High Courts of their jurisdiction
posting, and promoting district judges. in respect of the same. Forty-third Amendment) Act,
1977 omiited Article 131A from the constitution. Hence
2. Article 234- Article 234 deals with the recruitment
process for individuals who are not district judges but Assertion(A) is incorrect.
wish to enter the judicial service. The Supreme Court is the highest judicial court in India. It
3. Article 235- Article 235 grants the High Court the upholds and uplifts the rule of law and ensures and protects
authority to exercise control over district courts and citizens’ rights and liberties as given in the Constitution.
lower courts. This includes matters like posting, Therefore, the Supreme Court is also known as the Guardian
promotion, and granting leave to individuals in the of the Constitution. Hence Reason(R) is correct.
judicial service of a State who hold positions lower
than that of a district judge. 33. Solution: (a)

4. Article 236- This article provides the definition of the Exp) Option a is the correct answer
term “judicial service.” The Section 377 of the Indian Penal Code (IPC) is an act that
5. Article 237- Article 237 empowers the Governor criminalises homosexuality and was introduced in the ear
to extend the provisions outlined in this Chapter to 1861 during the British rule of India. referred to ‘unnatural
specific classes of magistrates within a State. offences’ and says whoever voluntarily has carnal intercourse
against the order of nature with any man, woman or animal,
30. Solution: (a) shall be punished with imprisonment for life.
Exp) Option a is the correct answer.
Important Tip
When the Chief Justice of a High Court acts in an
The Supreme Court of India on September 6, 2018,
administrative capacity, he is subject to the writ jurisdiction
decriminal decriminalised the Section 377 of the IPC and
of any of the other judges of the High Court. This means that
allowed gay sex among consenting adults in private.
if any action taken by the Chief Justice in his administrative
capacity is questioned, any other judge of the High Court The SC ruled that consensual adult gay sex is not a crime
can exercise their power of writ jurisdiction to review and saying sexual orientation is natural and people have no
potentially correct that action. Writ jurisdiction allows control over it.

PYQ Workbook 140


INDIAN POLITY AND GOVERNANCE

34. Solution: (b) 38. Solution: (b)


Exp) Option b is the correct answer Exp) Option b is the correct answer
Judicial review is the power of the judiciary to examine A Judge of the Supreme Court may resign his office by
the constitutionality of executive orders and legislative writing a letter to the President of India, as per Article
enactments of both the State and Central governments. 124(2).
The power of judicial review is itself conferred by the
39. Solution: (d)
Constitution on the judiciary (both the Supreme Court as
Exp) Option d is the correct answer
well as High Courts) in India.
The Supreme Court of India has three types of jurisdiction:
Important Tips
• Original Jurisdiction (Article 131): It can directly hear
Judicial review has two important functions, like, of
and decide disputes between the Government of India
legitimizing government action and the protection of
and one or more states, or between different states,
constitution against any undue encroachment by the gov-
involving questions of law or fact on the existence or
ernment.
extent of a legal right.
Judicial review is considered a basic structure of the
constitution. • Appellate Jurisdiction (Articles 132-134): It acts as
the highest court of appeal, hearing appeals in civil,
Judicial review is also called the interpretational and
criminal, and constitutional matters from high courts
observer roles of the Indian judiciary.
and tribunals.
35. Solution: (a) • Advisory Jurisdiction (Article 143): The President can
Exp) Option a is the correct answer seek the Supreme Court’s advisory opinion on questions
of law or public importance, which is not binding.
Both the Supreme Court and High Courts can issue
writs to protect fundamental rights under the Indian 40. Solution: (d)
Constitution. Supreme Court under Article 32 and High Exp) Option d is the correct answer
Courts under Article 226 have this power.
Article 348 (1) of the Constitution of India provides that
Important Tips all proceedings in the Supreme Court and every High court
• Articles 226 vests the High Courts with the power to shall be in the English Language until Parliament by law
issue writs in case of both - legal and fundamental otherwise provides.
rights. Supreme Court only issues writs in cases related Important Tip
to fundamental rights. Hence, the writ jurisdiction of
Part XVII of the Indian Constitution deals with the official
the High Courts is greater than that of the Supreme
languages in Articles 343 to 351. Article 343 (1) states that
Court.
the Official Language of the Union government shall be
• Supreme Court writ jurisdiction is nationwide while Hindi in Devanagari script.
High Court’s jurisdiction is limited to its respective
Article 351 gives power to the Union Government to issue
state/union territory.
a directive for the development of the Hindi language.
36. Solution: (c) The Hindi language is one of the 22 languages of the Eighth
Exp) Option c is the correct answer. Schedule of the Constitution of India.
According to the Article 217 of the Indian Constitution, 41. Solution: (a)
a Judge of a High Court is appointed by the President of
Exp) Option a is the correct answer.
India in consultation with the Chief Justice of India (CJI)
The money to pay the salaries and allowances of High
and the Governor of the respective state. The District Court
Court Judges comes from the Consolidated Fund of the
Judges are appointed by the Governor in consultation with
States where they serve. On the other hand, when these
the High Courts.
Judges retire and start receiving pensions, the money for
37. Solution: (c) their pensions comes from the Consolidated Fund of India,
which is managed by the central government.
Exp) Option c is the correct answer
Article 142 provides a unique power to the Supreme Court, Important Tips
to do “complete justice” between the parties, where, at times, Expenditures Charged on the Consolidated Fund of India:
the law or statute may not provide a remedy. Curative 1. Pensions of the judges of Supreme Court and High
Petition in India can be filed in the Supreme Court under Courts.
Article 142. It is a special remedy available after the review
2. Emoluments and allowances of the Chairman and
petition’s dismissal, meant to correct errors or injustices
Deputy Chairman of the Rajya Sabha.
during the judicial process, exercised in exceptional cases
3. Salaries and allowances of the President.
to prevent abuse of process or miscarriage of justice.

141 PYQ Workbook


INDIAN POLITY AND GOVERNANCE

46. Solution: (a)


4. Salaries and Allowances of the Supreme Court Judges.
5. Salaries and allowances of the Speaker and Deputy Exp) Option a is the correct answer
Speaker of the Lok Sabha. As per Article 130 of the Indian Constitution, the usual
place of the Supreme Court’s meetings is in New Delhi.
42. Solution: (b)
However, the Supreme Court has the flexibility to hold its
Exp) Option b is the correct answer
sessions or sittings at any other place outside New Delhi
In the case of Golaknath v. State of Punjab (1967),
if the President of India approves it. This provision allows
the Supreme Court of India held that “Fundamental
Rights enable a man to chalk out his life in the manner the court to conduct its proceedings in other locations when
he likes best.” This statement signifies the importance deemed necessary or in special circumstances with the
of fundamental rights in providing individuals with the President’s permission.
freedom and autonomy to live their lives according to their
own choices and preferences. 47. Solution: (c)
Exp) Option c is the correct answer
Important Tips
• Indira Gandhi vs. Raj Narain: Indira Gandhi’s Article 125 of the Indian Constitution states that the Judges
election to Lok Sabha was challenged by her opponent, of the Supreme Court shall be paid salaries as determined
Raj Narain, alleging electoral malpractices. The by Parliament or specified in the Second Schedule until a law
Allahabad High Court found her guilty and voided her is enacted. Judges shall be entitled to privileges, allowances,
election. The Supreme Court upheld her conviction. leave, and pension as determined by Parliament, or specified
The verdict was nullified by a 1976 amendment during
in the Second Schedule and these benefits cannot be reduced
the emergency.
to their disadvantage after appointment, except in case of
• Bank Nationalization Case: In R.C. Cooper Vs.
financial necessity.
Union of India (1970), the Supreme Court upheld
the nationalization of banks by the government as Important Tips
constitutionally valid. It emphasized that economic
1. The Salaries, Pension and Allowances of the Supreme
policies are within the scope of parliamentary powers,
Court Judges are charged upon the Consolidated
as long as they don’t violate fundamental rights.
Fund of India, whereas the Salaries and Allowances
43. Solution: (a) of the High Court Judges are charged upon the
Consolidated Fund of the States and the Pension is
Exp) Option a is the correct answer
charged on the Consolidated Fund of India.
Article 137 of the Indian Constitution permits the
2. The High Court Judges (Salaries and Conditions of
Supreme Court to review its judgments or orders. It allows
Service) Act, 1954, and the Supreme Court Judges
any person aggrieved by a Supreme Court judgment to
(Salaries and Conditions of Service) Act, 1958 regulate
seek a review of the same. The review petition is heard by a
the salaries and conditions of service of the judges of
larger bench of the Supreme Court, and it is based on limited
High Courts and the Supreme Court of India.
grounds such as the discovery of new and important matter
or evidence, a mistake or error apparent on the face of the 48. Solution: (a)
record, or any other sufficient reason.
Exp) Option a is the correct answer
44. Solution: (b)
The second largest bench constituted by the Supreme
Exp) Option b is the correct answer
Court till date was a 11-judge bench that was constituted
As a ‘Court of Record’ under Article 129 of the Indian in the case of Golaknath v. State of Punjab in 1967.
Constitution, the Supreme Court’s decisions are
authoritative, have evidentiary value, and can’t be Important Tips
questioned in other courts. Nevertheless, the court can • In the Golaknath Case, two brothers, Henry and
review and correct its judgments through Article 137, William Golaknath, challenged the Punjab Security
acknowledging its potential for errors. and Land Tenures Act that limited their land holdings
45. Solution: (c) to thirty acres each, declaring the rest as excess.

Exp) Option c is the correct answer • The case reached the Supreme Court in 1965 under
Article 32 of the Constitution, claiming violation of
The Supreme Court and High Courts, as Courts of Record
fundamental rights (Articles 19(f ), 19(g), and 14).
under Articles 129 and 215 respectively, record judgments,
proceedings, and acts for perpetual memory. These records • In 1967, the Supreme Court, in a historic judgment,
hold evidentiary value, cannot be questioned in lower courts, with 6:5 majority held that Parliament could not
and establish legal precedents. They also have the power to abridge any fundamental rights guaranteed by the
punish for contempt. Constitution.

PYQ Workbook 142


INDIAN POLITY AND GOVERNANCE

• Justice Subba Rao’s ruling declared the 17th 52. Solution: (c)
Amendment (which put the Punjab Act in the Ninth Exp) Option c is the correct answer
Schedule) unconstitutional regarding fundamental When the office of the Chief Justice of India (CJI) is
rights. The judgment established the doctrine of vacant or when the CJI is unable to perform their duties
basic structure, safeguarding essential features of the due to absence, illness, or any other reason, the President
Constitution. of India appoints the most senior judge of the Supreme
• In response, Parliament passed the 24th Amendment Court as the Acting Chief Justice of India. However, this
in 1971, asserting its power to amend any part of the appointment is made only after obtaining the previous
Constitution, including Fundamental Rights. consent of the outgoing Chief Justice of India. The Acting
• The largest-ever bench at the Supreme Court of India Chief Justice discharges the functions of the Chief Justice
has been constituted in 1973 in Kesavananda Bharati until a new Chief Justice is appointed or the incumbent Chief
v. State of Kerala. A bench of 13 judges was set up to Justice resumes their duties.
decide whether Parliament had the unfettered right
to amend the Constitution, which eventually gave 53. Solution: (a)
rise to the Basic Structure doctrine. Exp) Option a is the correct answer

49. Solution: (a) Articles 32, 226, and 227 of the Indian Constitution form
the inviolable basic structure.
Exp) Option a is the correct answer
• Article 32 grants the right to approach the Supreme Court
The Public Interest Litigation can be filed by any Indian directly for the enforcement of fundamental rights.
citizen in the court of Law. However, the only condition
• Article 226 empowers High Courts with the authority to
being that it must be recorded in the public interest rather
issue writs for fundamental rights enforcement.
than entertaining any private interest. It is not necessary,
• Article 227 confers the power of superintendence over
for the exercise of the court’s jurisdiction, that the person
courts and tribunals to High Courts.
who is the victim of the violation of his or her right should
personally approach the court. These articles ensure effective remedies for protecting
fundamental rights and maintain the independence
50. Solution: (b) and effectiveness of the judiciary, making them crucial
Exp) Option b is the correct answer components of the Constitution’s basic structure.
• Article 245 of the Indian Constitution is primarily
A person appointed as a judge of the Supreme Court or Chief
concerned with defining the territorial extent of laws
Justice of India, before entering upon his Office, has to make
made by the Parliament and the State Legislatures. It
and subscribe an oath or affirmation before the President, or
deals with the geographical scope within which the laws
some person appointed by him for this purpose. In his oath,
will apply. While it is an essential provision for the
a judge of the Supreme Court swears: functioning of the legal system, it is not considered
1. to bear true faith and allegiance to the Constitution of part of the “basic structure” of the Constitution.
India;
54. Solution: (d)
2. to uphold the sovereignty and integrity of India;
Exp) Option d is the correct answer
3. to duly and faithfully and to the best of his ability,
After retirement, Judges of the Supreme Court in India are
knowledge and judgement perform the duties of the
prohibited from practicing before any court, including the
Office without fear or favour, affection or ill-will; and Supreme Court and High Courts, as per the Supreme Court
4. to uphold the Constitution and the laws. Judges (Salaries and Conditions of Service) Act, 1958.
This restriction aims to preserve judicial independence,
51. Solution: (a) impartiality, and prevent potential conflicts of interest,
Exp) Option a is the correct answer. ensuring retired judges do not influence legal proceedings
Assertion is true- High Courts, being closer to the ground and maintain the integrity of the judiciary.
and having jurisdiction over specific states or union 55. Solution: (d)
territories, can more effectively address and protect the
Exp) Option d is the correct answer.
rights of Indian citizens within their territorial jurisdiction.
The power of the High Court to issue writs is a discretionary
They can swiftly respond to local issues and ensure that
power that is given to the High Court to protect the
justice is accessible to the people in their respective regions.
fundamental rights of the citizens and to ensure that
Reason (R) is true- The Supreme Court can only issue the government is functioning properly. The High Court
writs to protect fundamental rights under Article 32 of the can issue writs to protect both constitutional rights and
Indian Constitution. However, High Courts can issue writs statutory rights. Constitutional rights are rights that are
to protect both fundamental rights and other statutory guaranteed by the Constitution of India. Statutory rights are
rights under Article 226 of the Indian Constitution. rights that are granted by a statute or law. The High Court

143 PYQ Workbook


INDIAN POLITY AND GOVERNANCE

can also issue writs to protect the interests of the public at 60. Solution: (c)
large. This is known as the public interest litigation (PIL) Exp) Option c is the correct answer.
jurisdiction of the High Court. The PIL jurisdiction allows
the High Court to intervene in matters that are of public Article 124(3) of the Constitution of India states that a
importance, even if there is no individual who has been person can be appointed as a Judge of the Supreme Court
directly affected by the matter. if he or she is a distinguished jurist in the opinion of the
President of India.
Important Tips
The writs that can be issued by the High Court are: Important Tip:

1. Habeas corpus: This writ is used to ensure that a 1. Judges of the Supreme Court are appointed by the
person who is detained is not being held illegally. President of India according to Article 124(2) of the
Constitution.
2. Mandamus: This writ is used to command a person or
authority to perform a public duty. 2. When a judge of the Supreme Court wishes to resign,
they submit their resignation to the President.
3. Prohibition: This writ is used to prevent a lower court
or tribunal from exceeding its jurisdiction. 3. To be eligible for appointment as a judge of the
Supreme Court, a person must meet the following
4. Quo Warranto: This writ is used to challenge the right
qualifications under Article 124(3):
of a person to hold a public office.
• They should be a citizen of India.
5. Certiorari: This writ is used to quash the decision of
an inferior court or tribunal that has acted without or • They should have served as a judge of a High Court for
in excess of its jurisdiction. at least five years (or High courts in succession).
• They should have been an advocate of a High Court
56. Solution: (b) for at least ten years (or High courts in succession).
Exp) Option b is the correct answer • They should be recognized as a distinguished jurist in
Article 124(2) states that, “Every Judge of the Supreme the opinion of the President.
Court shall be appointed by the President by warrant 4. The Constitution does not specify a minimum age
under his hand and seal after consultation with such of the requirement for the appointment of a judge to the
Judges of the Supreme Court and of the High Courts in the Supreme Court.
States as the President may deem necessary for that purpose
and shall hold office until he attains the age of sixty-five 61. Solution: (a)
years” Exp) Option a is the correct answer.
57. Solution: (a) Article 129 establishes the Supreme Court as a court of
Exp) Option a is the correct answer. record with full powers, including the ability to punish
for contempt of itself whereas Article 144 advises civil
A High Court Judge addresses his letter of resignation to the
President of India. The President is the appointing authority and judicial authorities to act in aid of the Supreme Court.
for High Court Judges, and he is also the only authority that Article 143 of the Indian Constitution is related to the
can accept a High Court Judge’s resignation. advisory jurisdiction of the Supreme Court.

58. Solution: (d) 62. Solution: (c)


Exp) Option d is the correct answer Exp) Option c is the correct answer.
The Supreme Court of India was established by the The power of court of record is the power conferred on both
Constitution of India, which came into force on 26 the Supreme Court and High Courts. As courts of record,
January 1950. The Constitution of India, in Article 124(1), they have the authority to keep records of their proceedings,
provides for the establishment of a Supreme Court of judgments, and orders. The records of the courts are
India. The Supreme Court is the highest court in India and admissible as evidence in other courts. These records serve
has original, appellate and advisory jurisdiction.
as an important source of legal precedent and help maintain
59. Solution: (c) the integrity and accountability of the judiciary.
Exp) Option c is the correct answer. 63. Solution: (a)
The appointment of District Judges is governed by the
Exp) Option a is the correct answer
District Judges (Conditions of Service) Act, 1958. This
Act states that District Judges shall be appointed by the Lok Adalats are informal, speedy, and inexpensive alternative
Governor of the State in consultation with the High Court dispute resolution forums that are organized by the Legal
of the State. The Governor is free to consult any other Services Authorities at the national, state, and district levels.
person or authority before making an appointment, but the The Panchayat Committees are not part of the Legal
consultation with the High Court is mandatory. Services Authorities.

PYQ Workbook 144


INDIAN POLITY AND GOVERNANCE

Important Tips • However, the Kesavananda Bharati case later


• Lok Adalats have been given statutory status under overturned the Golak Nath judgment, stating that
the Legal Services Authorities Act, 1987. while Parliament can amend the Constitution, it
cannot destroy its basic structure.
• The Lok Adalat have the same powers as are vested
in a Civil Court under the Code of Civil Procedure 66. Solution: (b)
(1908). Exp) Option b is the correct answer
• Lok Adalat have the requisite powers to specify its own The case of Indra Sawhney Vs Union of India, commonly
procedure for the determination of any dispute coming known as the Mandal Commission case, dealt with the issue
before it. of the “creamy layer” among the backward classes in India.
• Under the said Act, the award (decision) made by the Important Tip:
Lok Adalats is deemed to be a decree of a civil court
• The Supreme Court, in its judgment, upheld the
and is final and binding on all parties and no appeal
validity of reservations for OBCs but introduced the
against such an award lies before any court of law.
concept of the “creamy layer.”
64. Solution: (b) • It defined the “creamy layer” as the relatively better-
Exp) Option b is the correct answer off and more privileged individuals within the OBC
In the SP Gupta vs. President of India case, which is also category who should be excluded from the benefits of
reservations.
known as the First Judges’ Case, the Supreme Court of
India held that the “primacy” of the CJI’s (Chief Justice • This exclusion was based on the rationale that
of India) recommendation on judicial appointments and reservations should reach those who genuinely
transfers can be refused for “cogent reasons.” The ruling need them and uplift the most socio-economically
gave the Executive primacy over the Judiciary in judicial disadvantaged sections.
appointments for the next 12 years. • KM Nanavati v. State of Bombay was a case about the
right to equality, Madhu Limaye v. Ved Murti was a
Important Tips
case about the right to freedom of speech, and Sajjan
1. Second Judges Case (1993): Singh v. State of Punjab was a case about the right to
• SC introduced the Collegium system, holding that property.
“consultation” really meant “concurrence”.
67. Solution: (c)
• It added that it was not the CJI’s individual opinion,
Exp) Option c is the correct answer.
but an institutional opinion formed in consultation
with the two senior-most judges in the SC. The locus standi rule to move the court was liberalized by
the case of SP Gupta vs Union of India. This case is also
2. Third Judges Case (1998): known as the Judges’ Transfer Case, as it dealt with the issue
• SC on the President’s reference (Article 143) expanded of transfer of judges from one High Court to another by the
the Collegium to a five-member body, comprising the President in consultation with the Chief Justice of India.
CJI and four of his senior-most colleagues.
Important Tip
65. Solution: (b) • SR Bommai vs Union of India was a case that dealt
Exp) Option b is the correct answer with the constitutional validity of President’s rule
imposed under Article 356 in various states. The
The Golak Nath case was a landmark judgment of the
court laid down certain safeguards and limitations on
Supreme Court of India in 1967 that dealt with the issue the exercise of this power by the Centre and upheld the
of the power of Parliament to amend the Constitution. federal structure of the Constitution.
The Court held that Parliament could not amend any of the
• Minerva Mills vs Union of India was a case that
provisions of Part III of the Constitution, which contains the
dealt with the constitutional validity of certain
Fundamental Rights. The Court held that these rights were amendments made to Article 31C and Article 368 by
fundamental and could not be taken away by Parliament. the 42nd Amendment Act, 1976. The court struck down
Important Tip these amendments as violative of the basic structure
doctrine and upheld the supremacy of judicial review
• In the Golak Nath case, the Supreme Court declared
and fundamental rights over directive principles.
the 24th Amendment, which amended Article 31
• Kesavananda Bharati vs State of Kerala was a case
regarding the right to property, as unconstitutional.
that dealt with the constitutional validity of certain
• The Court held that an amendment under Article amendments made to Article 31 by various land
368 is “law” within the meaning of Article 13 of the reform acts. The court propounded the basic structure
Constitution and therefore, if an amendment “takes doctrine and held that Parliament had no power to
away or abridges” a Fundamental Right conferred by amend any part of the Constitution that formed its
Part III, it is void. essential core or identity .

145 PYQ Workbook


INDIAN POLITY AND GOVERNANCE

68. Solution: (c) 71. Solution: (a)


Exp) Option c is the correct answer Exp) Option a is the correct answer.
Statement 1 is correct: The condition mentioned in statement The writ of Habeas Corpus is a legal remedy that allows
1 implies that the writ of certiorari can only be issued against an individual to seek relief from unlawful detention or
a tribunal or an officer who has the legal authority to decide imprisonment. The term “Habeas Corpus” is derived from
matters that impact the rights of individuals and is obligated Latin, which means “you may have the body.”
to act judicially. In other words, the tribunal or officer must
have the power to make decisions that affect the rights of Important Tip
individuals, and they must have a duty to act in a judicial or
1. Article 32 of the Indian Constitution provides for
quasi-judicial capacity.
the right to constitutional remedies to protect
Statement 2 is correct: This condition implies that the writ
Fundamental Rights and it is integral to Basic Structure
of certiorari can only be issued if the tribunal or officer has
doctrine.
acted beyond their legal authority or jurisdiction. If they
have acted without the requisite jurisdiction, the writ can be 2. Five Constitutional Writs:
issued to quash their decision or order. • Habeas Corpus: It used to ensure the release of a
For the issue of a writ of certiorari in India, both conditions person who is unlawfully detained or imprisoned
mentioned in options 1 and 2 are necessary. • Mandamus: It issued to compel public officials or
Important Tip authorities to perform their duties or functions.
Concept of Certiorari • Prohibition: It is used to prohibit lower courts or
• It literally means ‘to be certified’ or ‘to be informed.’ tribunals from exceeding their jurisdiction or acting
• It is issued by a higher court to a lower court or contrary to law.
tribunal to either transfer a case pending with the • Certiorari: It is issued to quash the orders or
latter or to overturn the latter’s order in a case.
decisions of lower courts or tribunals on grounds of
• It is issued due to an excess of jurisdiction, a lack of jurisdictional errors or violation of natural justice.
jurisdiction, or a legal error.
• Quo Warranto: It is used to inquire into the legality of
• As a result, unlike prohibition, which is only
preventative, certiorari is both preventive and a person holding a public office and to challenge their
curative. authority or credentials.
• Previously, the writ of certiorari could only be issued
72. Solution: (a)
against judicial and quasi-judicial authorities, not
administrative ones. Exp) Option a is the correct answer
• However, the Supreme Court ruled in 1991 that The power to establish a common High Court is vested in
certiorari can be issued even against administrative the Parliament, under Article 231 of the Constitution.
authorities affecting individual rights.
• Certiorari, like prohibition, is not available against 73. Solution: (c)
legislative bodies or private individuals or bodies. Exp) Option c is the correct answer.
69. Solution: (a) Under Article-32 of the Constitution of India, individuals
Exp) Option a is the correct answer have the right to move the Supreme Court for the enforcement
In December 2014, when Justice HL Dattu was the Chief of their Fundamental Rights. However, there are certain
Justice of India (CJI), the Supreme Court had issued a grounds on which the Supreme Court can refuse relief under
notification to set up a special Bench known as the Social Article-32. One such ground is when it is determined that no
Justice Bench. The Social Justice Bench was established to Fundamental Right has been infringed in the case.
exclusively hear cases related to social justice issues. It aimed
to expedite the hearing and disposal of cases pertaining to 74. Solution: (a)
matters such as gender inequality, child rights, disability Exp) Option a is the correct answer.
rights, environmental protection, and other socio-economic Under Article 131 of the Constitution of India, the Supreme
concerns. Court has original jurisdiction in matters related to disputes
70. Solution: (d) between states, or between the Government of India and one
Exp) Option d is the correct answer. or more states, or between two or more states on one side
and the Government of India on the other.
The case of “Naz Foundation vs. Government of NCT of
Delhi and others” addressed the constitutionality of Section Important Tip
377 of the Indian Penal Code, which criminalized same-
1. The Supreme Court has exclusive original jurisdiction
sex consensual activities. The court’s decision impacted
in federal disputes, which include disputes between
fundamental rights under Articles 21, 14, and 15 of the
the Centre and one or more states, or between
Indian Constitution, including the right to life, equality, and
different states.
prohibition of discrimination.

PYQ Workbook 146


INDIAN POLITY AND GOVERNANCE

into the authority by which the person claims to hold the


2. This jurisdiction is limited to disputes involving
questions of law or fact that determine the existence office and whether they have the legal right to do so. The
or extent of a legal right, excluding political questions. reason option (c) is not a necessary condition for issuing
a writ of quo warranto is that the eligibility or legality
3. The Supreme Court cannot entertain suits brought
by private citizens against the Centre or a state under of holding a substantive office is equally important to
this jurisdiction. ensure that the person holding the office has the necessary
qualifications and meets the requirements laid out by law
4. The Supreme Court’s jurisdiction does not extend
to disputes arising from pre-Constitution treaties, or the constitution.
agreements, or certain matters such as inter-state water Important Tip
disputes, matters referred to the Finance Commission,
1. ‘Quo Warranto’ means ‘by what warrant’. Through
commercial disputes, or recovery of damages by a state
this writ, the Court calls upon a person holding a public
against the Centre.
office to show under what authority he holds that
5. In 1961, the Supreme Court dismissed the first suit office. If it is found that the person is not entitled to
brought under its original jurisdiction, where West hold that office, he may be ousted from it. Its objective
Bengal challenged the Constitutional validity of the is to prevent a person from holding an office he is
Coal Bearing Areas (Acquisition and Development) not entitled to, therefore preventing usurpation of
Act, 1957, passed by the Parliament. any public office. It cannot be issued with respect to a
private office.
75. Solution: (c)
2. The writ can be issued only when the following
Exp) Option c is the correct answer
conditions are fulfilled:
Both the Supreme Court and High Courts have the
• The public office is wrongfully assumed by the
authority to issue writs. While the Supreme Court primarily
private person.
issues writs for the enforcement of fundamental rights, High
Courts can issue writs not only for fundamental rights but • The office was created by the constitution or law and
also for other purposes, such as the enforcement of legal the person holding the office is not qualified to hold
rights or statutory duties. the office under the constitution or law.

Important Tip • The term of the public office must be of a permanent


nature.
Article 32, Constitution of India 1950
• The nature of duties arising from the office must be
1. The right to move the Supreme Court by appropriate
public.
proceedings for the enforcement of the rights conferred
by Part III is guaranteed. 77. Solution: (c)
2. The Supreme Court shall have power to issue directions
Exp) Option c is the correct answer.
or orders or writs, including writs in the nature of
habeas corpus, mandamus, prohibition, quo warranto • The concept of basic structure was laid down by the
and certiorari, whichever may be appropriate, for the Supreme Court in the Keshvananda Bharti Case (1973).
enforcement of any of the rights conferred by this Part. • The basic structure doctrine states that the Constitution
3. Without prejudice to the powers conferred on the of India has certain basic features that cannot be altered
Supreme Court by clauses (1) and (2), Parliament or destroyed through amendments by the Parliament
may by law empower any other court to exercise Article 368 does not give absolute powers to the
within the local limits of its jurisdiction all or any of Parliament to amend any part of the Constitution.
the powers exercisable by the Supreme Court under
• It can’t be exercised by the people through representatives
clause (2).
The right guaranteed by this article shall not be suspended in a constituent assembly (Statement 2 is incorrect)
except as otherwise provided for by this Constitution. • It falls outside the constituent powers of the parliament.
Similarily, Under Art 226, Constitution of India - High (Statement 3 is incorrect)
Court have the power to issue writes.
The write jurisdiction of High Court is wider than that 78. Solution: (a)
of Supreme Court , as High Courts can issue writes with
respect to Fundamental as well as legal Rights , whereas Exp) Option a is the correct.
the supreme court can issue writes with respect to only • In Bachan Singh Vs State of Punjab (1980), Supreme
fundamental rights.
Court laid down that Capital Punishment in India would
76. Solution: (c) be given only in the rarest of rare cases.
Exp) Option c is the correct answer • It stated that the principle that the death penalty ought not
A writ of Quo Warranto is a legal action that challenges to be awarded unless the alternative of life imprisonment
a person’s right to hold a public office. It seeks to inquire is “unquestionably foreclosed”.

147 PYQ Workbook


INDIAN POLITY AND GOVERNANCE

Important Tip 82. Solution: (b)

• Gopalanachari Vs State of Kerala (1980) : Person shall Exp) Option b is the correct answer.
be given legal aid at the expense of the state. • Golak Nath case held that constitutional amendment
• Dr. Upendra Baxi Vs State of Uttar Pradesh (1983) : passed according to article 368 is a law within the
seeking enforcement of the constitutional rights. meaning of article 13.
• Tukaram Vs State of Maharashtra (1979) : There is a • 24th amendment act 1971 override this judgement and
word of difference between sexual intercourse & rape. declare that amendment under article 368 is not law
79. Solution: (b) within the meaning of article 13.

Exp) Option b is the correct answer Important Tip


• Daniel Latifi Vs Union of India : In this case a divorced • Keshavanand Bharati Case is related to the basic
Muslim women is entitled to reasonable and fair structure doctrine.
provision and maintenance from her former husband and • Minerva Mills Case is related to the interpretation of
this should be paid within the period of iddat. the basic structure doctrine.
• Muhammad Ahmed Khan Vs Shah Bano Begum : In • Maneka Gandhi Case is related to the Right of Personal
this case Supreme Court stated that divorced women had Liberty granted by Article 21 of the Constitution.
the right to maintenance even after the period of Iddat
was over. 83. Solution: (a)
• Mary Roy Vs State of Kerala : landmark case brought Exp) Option a is the correct answer.
equal rights for Syrian Christian women in India as Assertion is true- In the district, the district judge holds
their male siblings on matters of inheritance. the highest judicial authority, having both original and
• Shankari Prasad Vs. Union of India : In this case appellate jurisdiction over both civil and criminal matters.
Constitutional validity of the first amendment act(1951) Part VI (Articles 233 to 237) of the Indian Constitution
was challenged. deals with the District Courts.
80. Solution: (a) Reason (R) is true- The district judge exercises both
Exp) Option a is the correct answer. judicial and administrative powers, and they also possess
supervisory authority over all subordinate courts within the
• The basic structure doctrine was given in the landmark
district. The High Court hears appeals against the directives
decision of Kesavananda Bharati v State of Kerala (1973).
and judgments of the district judge.
• The basic structure doctrine states that the Constitution
of India has certain basic features that cannot be altered 84. Solution: (b)
or destroyed through amendments by The Parliament. Exp) Option b is the correct answer
• The Supreme Court can undertake judicial review for
Judicial Review is governed by the principle of “Procedure
such amendments.
established by law” Judicial review is the power of the
• The Supreme Court declared that Article 368 did judiciary to examine the constitutionality of executive orders
not enable Parliament to alter the basic structure or
and legislative enactments of both the State and Central
framework of the Constitution and Parliament could not
governments. If, after this examination, they are found to
use its amending powers under Article 368 alter the basic
be violative of the Constitution, they can be declared as
structure of the Constitution. Thus, statement 3 is correct.
unconstitutional, illegal, and invalid (null and void) by the
In the Golaknath case, the supreme court ruled that
Parliament couldn’t curtail any of the Fundamental judiciary.
rights. (this case is not related to basic structure of the
85. Solution: (d)
constitution).Thus, statement 4 is incorrect
Exp) Option d is the correct answer
81. Solution: (d) A Judge of the Supreme Court can be removed by
Exp) Option d is the correct answer. the President of India after an impeachment by the
• The Supreme Court is the guarantor and defender of Parliament. As per Article 124(4), A Judge of the Supreme
fundamental rights of the citizen . Court can be removed from office by the President of India
• In this regard, Supreme Court has original jurisdiction in only after an address by each House of Parliament supported
the sense that an aggrieved citizen can directly go to the by a majority of the total membership of that House and by a
Supreme Court, not necessarily by way of appeal. majority of not less than two-thirds of members present and
However, the writ jurisdiction of Supreme Court is not voting, and presented to the President in the same session
exclusive because High Court also empowered of issue for such removal on the ground of proved misbehaviour or
writ jurisdiction (Hence option d is incorrect) incapacity.

PYQ Workbook 148


INDIAN POLITY AND GOVERNANCE

Important Tips • Alternatively, must have been an advocate of a High


Article 124(4) and the Judges Inquiry Act 1968 determine Court for at least ten years or two or more such
the procedure of removal of the judges: courts in succession.
• A motion of impeachment addressed to the President is • Must be a distinguished jurist in the opinion of the
to be signed by at least 100 members of the Lok Sabha president.
or 50 members of the Rajya Sabha and then delivered
to the Speaker of Lok Sabha or the Chairman of Rajya 89. Solution: (d)
Sabha.
Exp) Option d is the correct answer
• The motion is to be investigated by a Committee of
3 judges of the Supreme Court and a distinguished Vishaka & ors. v/s state of Rajasthan is a case which deals
jurist. with the evil of Sexual Harassment of a women at her
• If the Committee finds the judge guilty of misbehavior workplace. It is a landmark judgment case in the history of
or that he suffers from incapacity, the motion along sexual harassment which as being decide by Supreme Court.
with the report of the committee is taken up for
consideration in the House where motion was moved. Important Tips
• The judge is then removed by the requisite majority, In 1997, the Supreme Court in India formulated the
i.e. majority of total and 2/3 of its members present Vishakha guidelines to protect women from sexual
and voting. harassment at workplaces.
The Supreme court made it mandatory for all institutions
86. Solution: (a)
to follow the Vishaka guidelines across the country.
Exp) Option a is the correct answer
The guidelines suggested that the compliance mechanism
Option (i) is True: As per Article 127(1), if there’s no should ensure the timebound treatment of complaints.
quorum of Judges available for a Supreme Court session,
The Vishakha guidelines gave rise to the foundation to
the Chief Justice of India, with the President’s prior consent
redress the Sexual Harassment of Women at Workplace
and consultation with the relevant High Court’s Chief Justice,
(Prevention, Prohibition, and Redressal) Act, 2013.
may request a qualified High Court Judge to serve as an ad
hoc Judge for the required period. 90. Solution: (d)
Option (ii) is False: As per Article 224(1), if there’s a
Exp) Option d is the correct answer
temporary increase in a High Court’s business or arrears
of work, the President may appoint qualified persons as Bella Banerjee case (1953) dealt with the constitutionality
additional Judges for up to two years. However, additional of the West Bengal Land Development and Planning Act,
judges are not appointed in the Supreme Court; they are 1948. The Supreme Court held certain provisions of the Act
appointed in High Courts.
unconstitutional and void. The judgment highlighted the
87. Solution: (b) importance of providing fair compensation to landowners
Exp) Option b is the correct answer during the process of land acquisition for public purposes
According to Article 139A of the Constitution of India the and established the principle that the government’s
Supreme Court has the right to transfer any case anywhere in declaration of public purpose should not be regarded as
the territory of India. The Supreme Court can also withdraw conclusive, leaving room for judicial review. It did not
the case or cases pending before the High Court and dispose concern the right to travel abroad as part of personal liberty.
of all the cases to itself.
Important Tips
88. Solution: (b)
• The Indira Sawhney case, also known as the Mandal
Exp) Option b is the correct answer Commission case, 1992 where the Supreme Court
According to Article 124(3) of the Constitution of India, upheld the implementation of reservations for Other
a person can be appointed as a Judge of the Supreme Court Backward Classes (OBCs) in educational institutions
if he/she has been an advocate of a High Court or of two or and public employment. However, the Court ruled
more such Courts in succession for at least 10 years. that the “creamy layer” within OBCs should be excluded
from reservation benefits to ensure that the benefits
Important Tips
reach the truly disadvantaged sections.
According to Article 124(3) of the Constitution, a person
• In the Vishakha case of 1997, the Supreme Court
can be appointed as a judge of the Supreme Court if he or
laid down guidelines to prevent sexual harassment
she:
of women at workplaces. These guidelines were later
• A person must be a citizen of India. incorporated into the Sexual Harassment of Women
• Must have served as a judge of a High Court for at at Workplace (Prevention, Prohibition, and Redressal)
least five years or two such courts in succession. Act, 2013.

149 PYQ Workbook


INDIAN POLITY AND GOVERNANCE

• In the Maneka Gandhi case (1978), the Supreme Court 92. Solution: (d)
held that the right to life and personal liberty (Article Exp) Option d is the correct answer
21) in the Indian Constitution is not limited to mere A Public Interest Litigation can be filed in the High Court
animal existence but includes a meaningful life with or Supreme Court. One can file a PIL under Article 226 of
dignity. The Court emphasized that Articles 14, 19, and the Constitution before the High Court and under Article 32
21 are not mutually exclusive and are interconnected to before the Supreme Court of India.
ensure fundamental rights’ comprehensive protection.
Important Tips
91. Solution: (b) Public interest Litigation (PIL) means litigation filed in a
court of law, for the protection of “Public Interest”, such as
Exp) Option b is the correct answer
Pollution, Terrorism, Road safety, Constructional hazards
The appellate jurisdiction of the Supreme Court can etc.
be invoked by a certificate granted by the High Court Any matter where the interest of public at large is affected
concerned under Article 132(1), 133(1) or 134 of the can be redressed by filing a Public Interest Litigation in a
Constitution in respect of any judgement, decree or final court of law.
order of a High Court in both civil and criminal cases, Public interest litigation is not defined in any statute or in
involving substantial questions of law as to the interpretation any act. It has been interpreted by judges to consider the
of the Constitution. intent of public at large.

PYQ Workbook 150


INDIAN POLITY AND GOVERNANCE

INDIAN POLITY AND GOVERNANCE


EXECUTIVE
*This unit consists of questions from Union Executive and State Executives

5.1. UPSC CSE Previous Years’ Questions Assemblies of States are also eligible to be
included in the Electoral College.
1. Consider the following statements: 2. Higher the number of elective Assembly
1. If the election of the President of India seats, higher is the value of vote of each
is declared void by the Supreme Court MLA of that State.
of India, all acts done by him/her in the 3. The value of vote of each MLA of Madhya
performance of duties of his/her office Pradesh is greater than that of Kerala.
of President before the date of decision 4. The value of vote of each MLA of
become invalid. Puducherry is higher than that of
2. Election for the post of the President of Arunachal Pradesh because the ratio
India can be postponed on the ground of total population to total number
that some Legislative Assemblies have of elective Puducherry is greater as
been dissolved and elections are yet to compared to Arunachal Pradesh.
take place.
How many of the above statements are
3. When a Bill is presented to the President
of India, the Constitution prescribes time correct? [UPSC CSE Pre 2023]
limits he/she has to declare his/her assent. (a) Only one
(b) Only two
How many of the above statements are
(c) Only three
correct? [UPSC CSE Pre 2023]
(d) All four
(a) Only one
(b) Only two 4. Consider the following statements:
(c) All three [UPSC CSE Pre. 2022]
(d) None 1. The Constitution of India classifies
2. Consider the following statements: the ministers into four ranks viz.
1. According to the Constitution of India, Cabinet Minister, Minister of State with
the Central Government has a duty to Independent Charge, Minister of State
protect States from internal disturbances. and Deputy Minister.
2. The Constitution of India exempts the 2. The total number of ministers in the
States from providing legal counsel Union Government, including the Prime
to a person being held for preventive Minister, shall not exceed 15 percent of
detention. the total number of members in the Lok
3. According to the Prevention of Terrorism Sabha.
Act, 2002, confession of the accused Which of the statements given above is/are
before the police cannot be used as correct?
evidence. (a) 1 only
How many of the above statements are (b) 2 only
correct? [UPSC CSE Pre 2023] (c) Both 1 and 2
(a) Only one (d) Neither 1 nor 2
(b) Only two 5. Consider the following statements:
(c) All three [UPSC CSE Pre. 2022]
(d) None
1. Attorney General of India and Solicitor
3. Consider the following statements in General of India are the only officers
respect of election to the President of India: of the Government who are allowed
1. The members nominated to either House to participate in the meetings of the
of the Parliament or the Legislative Parliament of India.

151 PYQ Workbook


INDIAN POLITY AND GOVERNANCE

2. According to the Constitution of India, Which of the statements given above is/are
the Attorney General of India submits his correct?
resignation when the Government which (a) 1 only
appointed him resigns. (b) 2 only
Which of the statements given above is/are (c) 1 and 3 only
correct? (d) 2 and 3 only
(a) 1 only
9. With reference to the Legislative Assembly
(b) 2 only
(c) Both 1 and 2 of a State in India, consider the following
(d) Neither 1 nor 2 statements: [UPSC CSE Pre. 2019]
1. The Governor makes a customary address
6. With reference to the Union Government, to Members of the state legislature at the
consider the following statements:
commencement of the first session of the
1. N. Gopala Swamy Iyengar Committee year.
suggested that a minister and a secretary
be designated solely for pursuing the 2. When a State Legislature does not have a
subject of administrative reform and rule on a particular matter, it follows the
promoting it. Lok Sabha rule on that matter.
2. In 1970, the Department of Personnel was Which of the statements given above is/are
constituted on the recommendation of correct?
the Administrative Reforms Commission, (a) 1 only
1966, and this was placed under the Prime (b) 2 only
Minister’s charge. (c) Both 1 and 2
Which of the statements given above is/are (d) Neither 1 nor 2
correct? [UPSC CSE Pre 2021]
10. Consider the following statements:
(a) 1 only
(b) 2 only [UPSC CSE Pre. 2018]
(c) Both 1 and 2 1. No criminal proceedings shall be
(d) Neither 1 nor 2 instituted against the Governor of a State
in any court during his term of office.
7. At the national level, which ministry
is the modal agency to ensure effective 2. The emoluments and allowances of
implementation of the scheduled Tribes the Governor of a State shall not be
and Other Traditional Forest Dwellers diminished during his term of office.
(Recognition of Forest Rights) Act, 2006? Which of the statements given above is/are
[UPSC CSE Pre 2021] correct?
(a) Ministry of Environment, Forest and (a) 1 only
Climate Change (b) 2 only
(b) Ministry of Panchayati Raj (c) Both 1 and 2
(c) Ministry of Rural Development (d) Neither 1 nor 2
(d) Ministry of Tribal Affairs
11. Consider the following statements:
8. Consider the following statements: [UPSC CSE Pre. 2016]
[UPSC CSE Pre. 2020] 1. The Chief Secretary in a State is appointed
1. The President of India can summon a by the Governor of that State.
session of the Parliament at such place as
2. The Chief Secretary in a State has a fixed
he/she thinks fit.
tenure.
2. The Constitution of India provides for
three sessions of the Parliament in a year, Which of the statements given above is/are
but it is not mandatory to conduct all correct?
three sessions. (a) 1 only
3. There is no minimum number of days (b) 2 only
that the Parliament is required to meet in (c) Both 1 and 2
a year. (d) Neither 1 nor 2

PYQ Workbook 152


INDIAN POLITY AND GOVERNANCE

12. Consider the following statements: Select the correct answer using the code given
[UPSC CSE Pre. 2015] below:
1. The Executive Power of the Union of (a) 1 and 2 only
India is vested in the Prime Minister. (b) 1 and 3 only
2. The Prime Minister is the ex Officio (c) 2, 3 and 4 only
Chairman of the Civil Services Board. (d) 1, 2, 3 and 4
Which of the statements given above is/are 16. With reference to Union Government,
correct? consider the following statements:
(a) 1 only 1. The Constitution of India provides that all
(b) 2 only Cabinet Ministers shall be compulsorily
(c) Both 1 and 2 the sitting members of Lok Sabha only.
(d) Neither 1 nor 2 2. The Union Cabinet Secretariat operates
13. Which of the following is/are the function/ under the direction of the Ministry of
functions of the Cabinet Secretariat? Parliamentary Affairs.
[UPSC CSE Pre. 2014] Which of the statement(s) given above is/are
1. Preparation of agenda for Cabinet correct? [U.P.S.C (Pre) 2013]
Meetings (a) 1 only
2. Secretariat assistance to Cabinet (b) 2 only
Committees (c) Both 1 and 2
3. Allocation of financial resources to the (d) Neither 1 nor 2
Ministries
17. To whom does the President of India address
Select the correct answer using the code given his resignation? [UPSC CSE Pre 2009]
below:
(a) Speaker of the Lok Sabha
(a) 1 only (b) Vice-President of India
(b) 2 and 3 only
(c) Chief Justice of India
(c) 1 and 2 only
(d) Prime Minister of India
(d) 1, 2 and 3
18. Consider the following statements:
14. Consider the following statements:
[UPSC CSE Pre. 2014] 1. The Advocate General of a State in India
is appointed by the President of India on
1. The President shall make rules for the
the recommendation of the Governor of
more convenient transaction of the
the concerned State.
business of the Government of India.
2. All executive actions of the Government 2. As provided in the Civil Procedure Code,
of India shall be expressed to be taken in High Courts have original, appellate and
the name of the Prime Minister. advisory jurisdiction at the State level.
Which of the statements given above is/are Which of the statement(s) given above is/are
correct? correct? [UPSC CSE Pre 2009]
(a) 1 only (a) 1 only
(b) 2 only (b) 2 only
(c) Both 1 and 2 (c) Both 1 and 2
(d) Neither 1 nor 2 (d) Neither I nor 2
15. Which of the following are the discretionary 19. With reference to Union Government,
powers given to the Governor of a State? consider the following statements:
[UPSC CSE Pre. 2014] 1. The Ministries/Departments of the
1. Sending a report to the President of India Government of India are created by
for imposing the President’s rule the Prime Minister on the advice of the
2. Appointing the Ministers Cabinet Secretary.
3. Reserving certain bills passed by the 2. Each of the Ministries is assigned to a
State Legislature for consideration of the Minister by the President of India on the
President of India advice of the Prime Minister.
4. Making the rules to conduct the business Which of the statement(s) given above is/are
of the State Government correct? [UPSC CSE Pre 2009]

153 PYQ Workbook


INDIAN POLITY AND GOVERNANCE

(a) 1 only 24. If the Prime Minister of India belongs to


(b) 2 only the Upper House of Parliament
(c) Both 1 and 2 [UPSC CSE Pre 1997]
(d) Neither 1 nor 2 (a) He will not be able to vote in his favour in
20. As per Indian Protocol, who among the the event of a no-confidence motion.
following ranks highest in the order of (b) He will not be able to speak on the budget
precedence? [UPSC CSE Pre 2003] in the Lower House.
(a) Deputy Prime Minister (c) He can make statements only in the
(b) Former President Upper House.
(c) Governor of a State within his State (d) He will have to become a member of the
(d) Speaker of Lok Sabha Lower House within six months after
being sworn in as the Prime Minister.
21. Consider the following statements-
25. Which one of the following statements is
Assertion(A): President of India is different
correct about the Prime Minister of India
from the British King.
[UPSC CSE Pre 1996]
Reason (R): Office of Indian President is
(a) He is free to choose his ministers only
similar to that of America.
among those who are members of either
Code: [UPSC CSE Pre 2000] House of Parliament.
(a) Both (A) and (R) are true, and (R) is the (b) He can choose his cabinet colleagues after
correct explanation of (A). due counselling by the President of India
(b) Both (A) and (R) are true but (R) is not in this regard.
the correct explanation of (A). (c) He has full discretion in the choice of
(c) (A) is true but (R) is false. persons who are to serve as ministers in
(d) (A) is false but (R) is true. his cabinet.
22. Consider the following statements about (d) He has only limited power in the choice
the Attorney General of India: of his cabinet colleagues because of
1. He is appointed by the President of India. the discretionary powers vested in the
President of India.
2. He must have the same qualifications as
are required for a Judge of the Supreme 26. Which one of the following is a part of the
Court. electoral college of the President but not of
3. He must be a member of either House of the process of impeachment?
Parliament. [UPSC CSE Pre 1996]
4. He can be removed by Impeachment by (a) Lok Sabha
Parliament. (b) Rajya Sabha
Which of these statements are correct? (c) State Legislative Councils
[UPSC CSE Pre 2000] (d) State Legislative Assemblies
(a) 1 and 2 27. Which one of the following is not explicitly
(b) 1 and 3 stated in the Constitution of India but
(c) 2, 3 and 4 followed as a convention?
(d) 3 and 4 only [UPSC CSE Pre 1995]
23. Consider the following functionaries: (a) The finance minister is to be a Member of
1. Cabinet Secretary the Lower House.
(b) The Prime Minister has to resign if he
2. Chief Election Commissioner
loses the majority in the Lower House.
3. Union Cabinet Minister
(c) All the parts of India are to be represented
4. Chief Justice of India in the Council of Ministers.
Their correct sequence in the Order of (d) In the event of both the President and
Precedence is: [UPSC CSE Pre 2000] the Vice-President demitting office
(a) 3, 4, 2, 1 simultaneously before the end of the
(b) 4, 3, 1, 2 tenure, the Speaker of the Lower House
(c) 4, 3, 2, 1 of the Parliament will officiate as the
(d) 3, 4, 1, 2 President.

PYQ Workbook 154


INDIAN POLITY AND GOVERNANCE

28. Article 156 of the Constitution of India (a) Neelam Sanjiva Reddy
provides that a Governor shall hold office (b) R. Venkatraman
for a term of five years from the date on (c) Dr. Shankar Dayal Sharma
which he enters upon his office. Which of (d) Dr. Zakir Hussain
the following can be deduced from this? 33. The National Disaster Management Division
1. No Governor can be removed from office in India is the nodal division in the Ministry
till the completion of his term. of Home Affairs for disaster management.
2. No Governor can continue in office Who is the Ex-officio Chairman of the
beyond the expiration of a period of five National Disaster Management Authority?
years. [U.P.P.C.S. (Pre) 2021]
Code: [UPSC CSE Pre 1995] (a) The Prime Minister
(a) 1 only (b) The Home Minister
(b) 2 only (c) The Defence Minister
(c) Both 1 and 2 (d) The Health and Family Welfare Minister
(d) Neither 1 nor 2
34. Which of the following gave legal basis
29. Which among the following is/are appointed to extraordinary powers exercised by the
by the President of India? Government of India during Lockdown?
1. Chairman of the Finance Commission [U.P.P.C.S. (Pre) 2020]
2. Vice-Chairman of the Planning 1. Mental Health Care Act, 2017
Commission 2. Disaster Management Act, 2005
3. Chief Ministers of the Union Territories 3. Enemy Property Act, 1968
Select the correct answer by using the 4. Epidemic Diseases Act, 1897
following code: [UPSC CSE Pre 1994] Select the correct answer from the codes
(a) 1 is correct given below.
(b) 1 and 2 are correct (a) 1, 2 and 3
(c) 1 and 3 are correct (b) 2, 3 and 4
(d) 2 and 3 are correct (c) 2 and 4
30. The Chief Minister of a State in India is not (d) 1, 3 and 4
eligible to vote in the Presidential election 35. Consider the following statements in the
if- [UPSC CSE Pre 1993] context of Attorney General of India:
(a) He is a candidate 1. The President shall appoint such a person
(b) He is yet to prove his majority on the as Attorney General of India who is
floor of the Lower House of the State qualified as to be appointed a Judge of
Legislature. Supreme Court.
(c) He is a member of the Upper House of the 2. The Attorney General shall receive such
State Legislature. remuneration as the Parliament may
(d) He is a member of the Lower House of the determine.
State Legislature. Select the correct answer using the codes
31. The only instance when the President of given below. [U.P.P.C.S. (Pre) 2020]
India exercised his power of veto, is related (a) Only 1 is correct
to- [UPSC CSE Pre 1993] (b) Only 2 is correct
(a) The Hindu Code Bill (c) Both 1 and 2 are correct
(b) The PEPSU Appropriation Bill (d) Neither 1 nor 2 is correct
(c) The Indian Post Office (Amendment) Bill 36. Consider the following statements and
(d) The Dowry Prohibition Bill select your answer from the codes given
below:
5.2. Other Examination Previous Years’ 1. The Governor does not have the power to
Questions appoint a judge of a state High Court.
32. Which one of the following did not serve as 2. He is not a part of the Legislature.
Vice-President before assuming the office 3. He has the power to nominate some
of the President of India? members of the Legislative Council.
[UPPCS (Pre) 2022] 4. He has no judicial powers.

155 PYQ Workbook


INDIAN POLITY AND GOVERNANCE

Code: [U.P.P.C.S. (Pre) 2017] 41. Given below are two statements:
(a) 1 and 2 are correct [U.P.P.C.S. (Mains) 2015]
(b) 1 and 3 are correct Assertion (A): The President of India is
(c) 2 and 4 are correct elected by indirect election.
(d) All are correct
Reason (R): There is a provision of Electoral
37. Which of the following is not true? College consisting of the elected members
[U.P.P.C.S. (Mains) 2016] of both Houses of Parliament and elected
(a) The President or Governor is immune members of the Legislative Assemblies of the
from legal action for officials acts. States.
(b) No Court can compel a Governor to In the context of above statements select the
perform any duty. correct answer.
(c) A two months’ notice in writing must be (a) (A) is true, but (R) is false.
delivered to the Governor for bringing (b) (A) is false, but (R) is true.
civil proceedings in respect of their (c) Both (A) and (R) are individually true
personal acts. and (R) is the correct explanation of (A).
(d) Courts are empowered to enquire the (d) Both (A) and (R) are individually true,
advice tendered by the Ministers to the but (R) is not the correct explanation of
President or Governor. (A).
38. When the same person is appointed as 42. In case the post of President of India falls
a Governor of two or more States, the vacant, and there is no Vice-President also,
emoluments and allowances payable to the who among the following would be the
Governor shall be- acting President?
[U.P.P.C.S. (Mains) 2016] [U.P.P.C.S. (Pre) (Re. Exam) 2015]
(a) As expressed, wish of the Governor (a) Vice-Chairman of Rajya Sabha
(b) As decided by the President (b) Attorney-General of India
(c) As decided by the Home Ministry (c) Lok Sabha Speaker
(d) It shall be allocated among the States in (d) Chief Justice of the Supreme Court
such proportion as the President may by
order determine 43. Under which one of the following Articles,
the procedure for removal of the President of
39. Which one of the following is not correct India through the process of impeachment
regarding the Presidential election? has been laid down?
[U.P.P.C.S. (Mains) 2016] [U.P.P.C.S. (Pre) (Re. Exam) 2015]
(a) Election of the successor must be held (a) Article-53
before the expiry of the term of the (b) Article-61
incumbent President. (c) Article-74
(b) The President shall hold office for a term (d) Article-13
of five years from the date on which he
enters upon his office. 44. Who among the following is the legal
(c) On the expiration of his term, he continues adviser of a State Government?
to hold office until his successor enters [U.P. Lower Sub. (Pre) 2015]
upon his office. (a) The Advocate-General
(d) President’s election may be postponed on (b) The Attorney-General
the ground that the electoral college is (c) The Solicitor-General
incomplete. (d) The Comptroller and Auditor General
40. Who among the following holds his/her 45. The Vice-President may be removed from
office during the pleasure of the President? his office by a resolution of –
[U.P.P.C.S. (Pre) 2016] [U.P.P.C.S. (Mains) 2013]
(a) Comptroller and Auditor General of India (a) Council of States
(b) Chief Election Commissioner (b) Lok Sabha
(c) Speaker of Lok Sabha (c) Cabinet
(d) Attorney General of India (d) Council of Ministers

PYQ Workbook 156


INDIAN POLITY AND GOVERNANCE

46. Which of the following statements is correct (a) Only elected members of both the Houses
about Prime Minister of India of Parliament.
[U.P.P.C.S. (Mains) 2013] (b) All the members of both the Houses of
(a) Prime Minister is the nominal head of the Parliament.
(c) Only elected members of Rajya Sabha and
Council of Ministers.
State Assemblies.
(b) He is not responsible to Parliament. (d) Only elected members of State Assemblies.
(c) He cannot dismiss any of his ministers.
(d) Prime Minister is the real head of the 52. Consider the following statements and
Council of Ministers. select the correct answer from the code
given below:
47. The number of Lok Sabha members 1. The executive power of the Union is
required to table a ‘No Confidence Motion’ vested in the Prime Minister.
against the Council of Ministers is- 2. The Prime Minister is appointed by the
[U.P.P.C.S. (Mains) 2013] President.
(a) 30 3. The Prime Minister is the head of the
(b) 40 Council of Ministers.
(c) 50 4. The Prime Minister, at the time of his
(d) 60 appointment, needs not be a member of
either House of Parliament.
48. The Policy of Liberalisation, Privatisation,
Code: [U.P.P.C.S. (Mains) 2011]
and Globalisation was announced as New
(a) Only 1, 2 and 4 are correct.
Economic Policy by Prime Minister-
(b) Only 1, 2 and 3 are correct.
[U.P. Lower Sub. (Pre) 2013] (c) Only 2, 3 and 4 are correct.
(a) Rajeev Gandhi (d) Only 1, 3 and 4 are correct.
(b) Vishwanath Pratap Singh
53. Which of the following statements is not
(c) P.V. Narsimha Rao
true regarding the Parliament of India?
(d) Atal Bihari Vajpayee
[U.P.P.C.S. (Pre) 2011]
49. Given below are two statements: (a) The Constitution provides for a
[U.P.P.C.S. (Mains) 2012] Parliamentary form of Government.
Assertion (A): The Union Executive is (b) The function of Parliament is to provide
headed by the President of India. for a Cabinet.
(c) The membership of the cabinet is
Reason (R): There is no limitation on the restricted to the Lower House.
power of the President. (d) The cabinet enjoys the confidence of the
In the context of the above which of the majority in the popular chamber of the
following is correct? house.
(a) Both (A) and (R) are true, and (R) is the 54. Who amongst the following is considered to
correct explanation of (A). be the first law officer of the Govt. of India?
(b) Both (A) and (R) are true, but (R) is not [U.P.P.S.C. (SpI) (Pre) 2010]
the correct explanation of (A). (a) Chief Justice of India
(c) (A) is true, but (R) is false. (b) Solicitor-General
(d) (A) is false, but (R) is true. (c) Attorney-General
(d) Advocate-General
50. Who among the following Presidents of
India was associated with Trade Union 55. Which one of the following statements
Movement? [U.P.P.C.S.(Pre) 2012] about the President of India is not correct?
(a) V. V. Giri [U.P.P.C.S. (Mains) 2010]
(b) N. Sanjiva Reddy (a) He is a constituent part of Parliament.
(c) K.R. Narayanan (b) He participates in the discussions in the
(d) Zakir Hussain both Houses.
(c) He addresses the joint meeting of both
51. The Vice-President of India is elected by the the Houses every year.
members of an electoral college comprising – (d) He can promulgate ordinances in certain
[U.P.P.C.S. (Mains) 2012] situations.

157 PYQ Workbook


INDIAN POLITY AND GOVERNANCE

56. Which one of the following motions, the 62. Given below are two statements:
Council of Ministers in India can move? Assertion (A): The President is the part of
[U.P.P.C.S. (Mains) 2010] the Parliament.
(a) No-Confidence Motion Reason (R): A Bill passed by the two Houses
(b) Censure Motion of Parliament cannot become law without the
(c) Adjournment Motion assent of the President.
(d) Confidence Motion In the context of the above, which one of the
57. The Office of the Deputy Prime Minister following is correct?
[U.P.P.C.S. (Mains) 2009] [U.P.P.C.S. (Mains) 2007]
(a) Was created under the original (a) Both (A) and (R) are true, and (R) is the
correct explanation of (A).
Constitution.
(b) Both (A) and (R) are true, but (R) is not
(b) Is an extra-Constitutional growth.
the correct explanation of (A).
(c) Was created by 44th Amendment. (c) (A) is true, but (R) is false.
(d) Was created by the 85th Amendment. (d) (A) is false, but (R) is true.
58. The approval of ordinance promulgated by 63. The Constitution authorizes the President
Governor is necessary- to make provisions for discharge of duties of
[U.P.P.C.S. (Spl) (Mains) 2008] Governors in extraordinary circumstances
(a) By President under [U.P.P.C.S. (Mains) 2007]
(b) By the State Legislature (a) Article 160
(c) By the Ministers of Council of State (b) Article 162
(d) None of the above (c) Article 165
(d) Article 310
59. A Member of Parliament or a State
Legislature can be elected as President but 64. Which one of the following is not
[U.P.P.C.S. (Spl) (Mains) 2008] Constitutional Prerogative of the President
of India? [U.P.P.C.S. (Mains) 2007]
(a) He will have to resign from his seat before
contesting the election (a) Returning an ordinary Bill for
(b) He will have to relinquish his seat within reconsideration
(b) Returning a financial bill for
6 months of his election
reconsideration
(c) He will have to relinquish his seat as soon
(c) Dissolving the Lok Sabha
as he is elected (d) Appointing the Prime Minister
(d) An MP can contest but an MLA can not
65. Which of the following types of authority
60. Which of the following is incorrect about are attributed to the President of the India?
the powers of the President of India? 1. Real and Popular
[U.P.P.C.S. (Spl) (Mains) 2008] 2. Titular (formal) and De jure
(a) The President should accept the advice of 3. Political and Nominal
Supreme Court 4. Constitutional and Nominal
(b) Emergency powers
Select the correct answer from the code given
(c) Power of extension of President’s Rule in
below: [U.P.P.C.S. (Mains) 2006]
States
(a) 1 and 3
(d) President can accept or reject the advice
(b) 2 and 3
of the Supreme Court
(c) 1 and 4
61. Which one of the following has the power to (d) 2 and 4
initiate the Motion of Impeachment of the 66. Who has the Constitutional Power to
President of India? declare a geographical area as a ‘Scheduled
[U.P.P.C.S. (Mains) 2007] Area’? [U.P.P.C.S. (Pre) 2006]
(a) Both the Houses of the Parliament (a) Governor
(b) Lok Sabha (b) Chief Minister
(c) Rajya Sabha (c) Prime Minister
(d) Speaker of the Lok Sabha (d) President

PYQ Workbook 158


INDIAN POLITY AND GOVERNANCE

67. Given below are two statements: 71. Which one of the following statements
Assertion (A): The Supreme Command of about the Governor of an Indian State is not
the Defence Forces is vested in the President. true? [U.P. Lower Sub. (Pre) 2004]
Reason (R): The President’s powers as (a) He is appointed by the President of India
Commander-in-Chief are independent of (b) He can be Governor of more than one
Legislative Control. Stale
(c) He holds office for a term of five years
In the context of the above, select the (d) He can be removed earlier if the
correct answer using the code given below: Legislature of the concerned State passes
[U.P.P.C.S. (Mains) 2005] a resolution for his removal
(a) Both (A) and (R) are true, and (R) is the
correct explanation of (A). 72. Under which of the following Article of
(b) Both (A) and (R) are true, but (R) is not a Indian Constitution the Governor may
reserve a Bill for the consideration of the
correct explanation of (A).
President? [U.P. Lower Sub. (Pre) 2004]
(c) (A) is true, but (R) is false.
(d) (A) is false, but (R) is true. (a) Article 169
(b) Article 200
68. Which of the following statements about (c) Article 201
the Vice-President of India are not correct? (d) Article 202
1. In order to be the Vice-President, a 73. Who among the following Chief Justices of
person must be qualified for election as a India also acted as the President of India?
member of the House of the People.
[U.P.P.C.S. (Mains) 2003]
2. A member of the State Legislature can be
(a) Justice M. C. Mahajan
a candidate for this office. (b) Justice M. Hidayatullah
3. The term of the Office of the Vice- (c) Justice P.N. Bhagwati
President is the same as that of the (d) Justice B.K. Mukherjee
President.
4. He can be removed through a formal 74. The original plan in the draft Constitution
impeachment. for having elected Governors was dropped
because-
Select the correct answer using the code given 1. It would have meant another election.
below: [U.P.P.C.S. (Mains) 2005]
2. The election would have been fought on
(a) 1 and 2 only major political issues.
(b) 2 and 3 only
3. The elected Governor might have
(c) 1 and 4 only
considered himself superior to the Chief
(d) 3 and 4 only
Minister.
69. Which one of the following is presided over 4. The Governor had to function under the
by one who is not its member? Parliamentary System.
[U.P.P.C.S. (Mains) 2004] Select the correct answer from the code given
(a) Lok Sabha below: [U.P.U.D.A./L.D.A. (Pre) 2002]
(b) Vidhan Sabha (a) 1 and 2
(c) Rajya Sabha (b) 2 and 3
(d) Council of Ministers (c) 1, 3 and 4
70. The Governor of a State- (d) 2, 3 and 4
1. Is appointed by the President. 75. Who of the following was elected President
2. Holds office during the pleasure of the of India unanimously?
President. [U.P.P.C.S. (Pre) 1998]
3. Is the head of the State’s Executive Power. (a) S. Radhakrishnan
4. Normally holds office for five years. (b) V. V. Giri
(c) N. Sanjiva Reddy
Select the correct answer from the codes
(d) Gyani Zail Singh
given below: U.P.P.C.S. (Mains) 2004]
(a) 1 and 2 76. Consider the following statements in
(b) 1, 2 and 3 relation to Governor of a State in India-
(c) 2 and 4 1. He must be a citizen of India by birth.
(d) 1, 2, 3 and 4 2. Must have completed the age of 35 years.

159 PYQ Workbook


INDIAN POLITY AND GOVERNANCE

3. He must be eligible to be a member of the 82. Consider the following and select the
Lok Sabha. correct answer by using the code given
4. He may be appointed as Governor for two below:
or more states. 1. The Governor shall take oath or
Which of these is/are correct? affirmation before entering upon his
[U.P. Lower Sub. (Pre) 1996] office.
(a) 1 and 2 are correct 2. The format of oath or affirmation is
(b) 1, 2 and 3 are correct given in the Third Schedule of Indian
(c) 3 and 4 are correct Constitution.
(d) 2 and 4 are correct 3. The Governor shall make and subscribe
77. The Solicitor General of India is: to the oath or affirmation in the presence
[U.P.P.C.S. (Pre) 1994] of Chief Justice of the High Court or in
(a) An Administrative Officer his absence senior most Judge of that
(b) A Legal Adviser Court.
(c) An adviser to Prime Minister 4. The process of oath and affirmation
(d) A law officer for giving advice to the is given in Article 159 of Indian
President of India Constitution.
78. Consider the following statements and Code: [R.A.S./R.T.S.(Pre) 2013]
choose the correct answer from the code (a) (1), (2) and (3)
given below: (b) (1), (3) and (4)
Assertion(A): Ministers make policy and (c) (1) (2) and (4)
civil servants adhere to that policy. (d) (1), (2), (3) and (4)
Reason(R): Parliamentary form of 83. Which Indian state had the first woman
government works on the principle of Chief Minister? [R.A.S./R.T.S.(Pre) 2003]
‘Cabinet Collective Responsibility’. (a) U.P.
Code: [U.P.P.C.S. (Pre) 1993] (b) Bihar
(a) Both (A) and (R) are true, and (A) is (c) Tamil Nadu
correct explanation of (R). (d) Delhi
(b) Both (A) and (R) are true, and (A) is not
84. In the order of precedence of the government
the correct explanation of (R).
(c) (A) is true, (R) is false. of India which of the following come/comes
(d) (A) is false, (R) is true. above the Chief Justice of India?
[R.A.S./R.T.S.(Pre) 1997]
79. To whom is the Governor of a State
(a) Attorney General of India
responsible? [U.P.P.C.S. (Pre) 1992]
(b) Former President
(a) Prime Minister (c) Chief of Staffs
(b) President
(d) Speaker of Lok Sabha
(c) Legislative Assembly
(d) Chief Minister 85. The President holds office for a term of 5
80. The procedure of Impeachment of the years from date on –
President of India is – [R.A.S./R.T.S. (Pre) 1996]
[R.A.S./R.T.S. (Pre) 2016] (a) Which he/she is elected
(a) Quasi-Judicial Procedure (b) Which is decided by Parliament
(b) Executive Procedure (c) Which he enters upon his office/hold his
(c) Judicial Procedure office
(d) Legislative Procedure (d) Which is decided by Election Commission
81. According to the Constitution, which among 86. Which of the following is required under
the following is literally not a ‘Power’ of the Article 108 of the Constitution of India to
President of India? summon a joint sitting of the two houses of
[R.A.S./R.T.S.(Pre) 2013] Parliament? [M.P.P.C.S. (Pre) 2021]
(a) Promulgation of Ordinances (a) Intention of the President
(b) To consult the Supreme Court (b) Satisfaction of the President
(c) To send messages to Houses of Parliament (c) Instruction of the President
(d) To grant Pardon (d) Consent of the President

PYQ Workbook 160


INDIAN POLITY AND GOVERNANCE

87. Match the following States and their 93. If Vice-President of India has to discharge
respective Chief Ministers, who became the duties of President of India is his
Prime Minister of India: absence due to some reasons, then what is/
are true for such duration?
List-I (Schedule) List-II (Subject)
1. In this duration, Vice-President holds the
A. P.V. Narasimha 1. Undivided Uttar post of Chairman of Council of States
Rao Pradesh 2. In this duration, Vice-President will get
B. Morarji Desai 2. Undivided the emolument of President
Andhra Pradesh 3. If in this duration, Vice-President resigns,
C. H.D. Devegowda 3. Gujarat then Chief Justice of India will perform
the duties of President of India.
D. Charan Singh 4. Karnataka Code: [Chhattisgarh PCS (Pre) 2021]
E. Narendra Modi 5. Bombay (a) 1, 2 and 3
Code: [M.P.P.C.S. (Pre) 2020] (b) 1 and 2 only
A B C D E (c) 2 and 3 only
(d) 1 only
(a) 4 5 2 1 3
(b) 4 2 5 1 3 94. Read the following statements related to
(c) 2 1 4 5 3 Cabinet Committees and choose the correct
(d) 2 5 4 1 3 option:
88. The President can dissolve Lok Sabha on– Statement I: Cabinet Committees are not
[M.P.P.C.S. (Pre) 1999] mentioned in the constitution.
(a) Recommendation of Chief Justice of India Statement II: Cabinet Committees are set
(b) Lok Sabha’s recommendation up by the Prime Minister according to the
(c) Recommendation of the Union Cabinet exigencies of the time and requirements of
(d) Rajya Sabha’s recommendation the situation.
Statement III: If Prime Minister is the
89. Who prepares the President’s address to
member of Committee, he may not necessarily
Parliament? [M.P.P.C.S. (Pre) 1998] be the Chairman of Committee.
(a) President himself
Statement IV: Parliamentary Affairs
(b) Speaker of Lok Sabha
Committee is chaired by the Prime Minister.
(c) Chairman of Rajya Sabha
(d) Central Cabinet Code: [Chhattisgarh P.C.S. (Pre) 2020]
(a) All statements are true
90. Who is the topmost civilian officer in (b) Statements I, II and III are true, but
Government of India? Statement IV is false
[M.P.P.C.S. (Pre) 1998] (c) Statements II, III and IV are true but
(a) Home Secretary Statement I is false
(b) Defence Secretary (d) Statements I and II are true, but Statement
(c) Cabinet Secretary III and IV are false
(d) Lt. Governor of Delhi 95. Which of the following doesn’t consists in
91. For how long can the Vice-President act the power ‘To Pardon’ of President?
as President when the President cannot i. Pardon
perform his duties for reasons of death ii. Commutation
resignation, removal or otherwise?
[Jharkhand P.C.S. (Pre) 2013] iii. Remission
(a) 5 years iv. Respite
(b) 1½ years v. Reprieve
(c) 1 year vi. Detention
(d) 6 months vii. Continuation
92. Governor’s salary and allowances are given Code: [Chhattisgarh P.C.S. (Pre) 2016]
by- [Jharkhand P.C.S. (Pre) 2003] (a) i
(a) Consolidated Fund of the State (b) ii, iii
(b) Consolidated Fund of India (c) iv, v
(c) Contingency Fund of the State (d) i, ii, iii, iv, v
(d) Both (a) and (b) (e) vi, vii

161 PYQ Workbook


INDIAN POLITY AND GOVERNANCE

96. On which matter, it is not essential for the (a) 1 only


President to take advice from the Council (b) 2 and 3 only
of Ministers? (c) 1 and 2 only
[Chhattisgarh P.C.S. (Pre) 2015] (d) 1, 2 and 3
(a) Appointment of Ambassadors 101. Which article of the Indian Constitution
(b) To assent to Bills prescribes eligibility conditions for re-
(c) To assent to Bills referred by Governors election to the post of the President of
(d) To dissolve Lok Sabha India? [Chhattisgarh P.C.S. (Pre) 2011]
(e) To assent on constitutional amendment (a) Article 52
bill (b) Article 54
97. Who advises the Government of India on (c) Article 55
legal matters? [49th B.P.S.C. (Pre) 2000] (d) Article 57
(a) Attorney General 102. Which of the following Constitutional
(b) Chief Justice of the Supreme Court Amendment Acts restricted the size of the
(c) Chairman, Law Commission Council of Ministers to 15 percent of the
(d) None of them total members of the Lok Sabha?
[66th B.P.S.C. (Pre) (Re-Exam), 2020]
98. After resigning, whose permission does a
(a) 95th Constitutional Amendment Act,
minister require to give a statement in Lok
2009
Sabha? [Chhattisgarh P.C.S. (Pre) 2015]
(b) 93rd Constitutional Amendment Act,
(a) Prime Minister 2005
(b) Speaker (c) 91st Constitutional Amendment Act,
(c) President 2003
(d) Supreme Court (d) 90th Constitutional Amendment Act,
(e) None of the above 2003
99. Which of the following are the discretionary (e) None of the above / More than one of the
powers of the President? above
(i) Appointment of Council of Ministers 103. In the Indian Order of Precedence, who
(ii) To return a Bill with objections amongst the following comes first?
[65th B.P.S.C. (Pre) 2019]
(iii) To withhold a Bill
(a) The Chairman of UPSC
(iv) To pardon (b) The Chief Election Commissioner
(v) To summon joint sessions (c) The Comptroller and Auditor General
(vi) To send message to Parliament (d) The Chief Justice of High Court
(e) None of the above/More than one of the
(vii) Appointment of Judges
above
Code: [Chhattisgarh P.C.S. (Pre) 2014]
(a) (i), (iii), (iv) 104. The Prime Minister of India is responsible
(b) (ii), (v), (vii) to whom? [63rd B.P.S.C. (Pre) 2017]
(c) (iii), (vi), (vii) (a) The Cabinet
(d) (ii), (iv), (vi) (b) The President
(e) (ii), (iii), (vi) (c) The Lok Sabha
(d) The Rajya Sabha
100. Which of the following is/are the function/ (e) More than one of the above
functions of the Cabinet Secretariat?
105. Who is the highest Civil Services Officers of
1. Preparation of agenda for Cabinet
the Central Government?
Meetings
[63rd B.P.S.C. (Pre) 2017]
2. Secretariat assistance to Cabinet
(a) The Attorney General of India
Committees
(b) The Cabinet Secretary
3. Allocation of financial resources to the (c) The Home Secretary
Ministries (d) The Finance Secretary
Select the correct answer using the code given (e) None of the above / more than one of the
below. [Chhattisgarh P.C.S. (Pre) 2013] above

PYQ Workbook 162


INDIAN POLITY AND GOVERNANCE

106. President of India exercises his powers (c) The National Authority Chemical
[60th to 62nd B.P.S.C. (Pre) 2016] Weapons Convention
(a) either directly or through officer’s (d) The Department of Jammu, Kashmir and
subordinate to him Ladakh Affairs
(b) through Ministers 113. Which one of the following statements
(c) through Prime Minister about the Solicitor General of India (SGI)
(d) through Cabinet is not correct? [CDS 2023 (I)]
(e) None of the above/More than one of the (a) The SGI is the Government’s chief legal
above advisor.
107. The President of India does not have the (b) The SGI is the secondary law officer of
right to – [41st B.P.S.C. (Pre) 1996] the country.
(c) The post of the SGI is a constitutional
(a) Give pardon post.
(b) Remove a judge of Supreme Court (d) The SGI is appointed by the Appointments
(c) Declare emergency Committee of the Cabinet.
(d) Pass ordinances
114. Which one of the following committees
108. The word ‘mantrimandal’ (cabinet) is does not have members from the Rajya
mentioned only once in Constitution, and Sabha? [CDS 2023 (I)]
that is in- [41st B.P.S.C. (Pre) 1996] (a) The Public Accounts Committee
(a) Article 352 (b) The Committee Undertakings on Public
(b) Article 74 (c) The Estimates Committee
(c) Article 356 (d) The Committee on the Welfare of
(d) Article 76 Scheduled Castes and Scheduled Tribes
109. The President of India can be removed from 115. Which one of the following statements
his post by – [39th B.P.S.C. (Pre) 1994] about the election of the President of India
(a) The Prime Minister of India is not correct? [CDS 2023 (I)]
(b) Lok Sabha (a) A Union Minister shall not be eligible for
(c) Chief Justice of India election as the President.
(d) Parliament (b) The President is elected by the elected
members of the Parliament and the
110. If there is any dispute regarding the
Legislative Assemblies of States and
Presidential election, it can be referred to – Union Territories.
[38th B.P.S.C. (Pre) 1992] (c) The Electoral College does not include
(a) Attorney General of India elected members of the Legislative
(b) Parliament Councils.
(c) Supreme Court of India (d) The system of proportional representation
(d) None of these by means of single transferable vote is
followed.
111. Which of the following statements is not
correct? [38th B.P.S.C. (Pre) 1992] 116. According to the Constitution of India,
(a) To hold the office of President, candidate which one of the following statements is not
should have the minimum age of 35 years correct? [CDS 2022 (I)]
(b) Vice-President is Chairman of Rajya (a) The executive powers of the Union shall
Sabha be vested in the President of India.
(c) Vice-President is appointed by the (b) Parliament, by law, can confer functions
President of India on authorities other than the President of
(d) First President of India was Dr. Rajendra India.
Prasad (c) The President of India is elected by an
electoral college consisting of all Members
112. Which one among among the following of Parliament and all Members of all State
Departments/Authorities is not under the Legislatures.
Ministry of Home Affairs? [CDS 2023 (I)] (d) A person, having held office as the
(a) The Department of States President, shall be eligible for election to
(b) The Department of Language Official that office for the second term.

163 PYQ Workbook


INDIAN POLITY AND GOVERNANCE

117. Which one of the following Articles of the 122. Which one of the following statements
Constitution of India requires the Prime about the Vice-President of India is not
Minister of India to furnish information correct? [CDS 2021(I)]
related to decisions of the Council of (a) He is not eligible for re-election.
Ministers to the President of India? (b) He must not be less than 35 years of age.
[CDS 2022 (I)] (c) He is elected by members of an electoral
(a) Article 78 college consisting of the members of both
(b) Article 74 Houses of the Parliament.
(c) Article 75 (d) His term of office is five years.
(d) Article 81 123. Which one among the following statements
118. Which one of the following statements is pertaining to the President’s term of Office
not correct? [CDS 2022 (I)] is not correct? [CDS 2020 (II)]
(a) All executive actions of the Government (a) The President holds Office for a term of
of India are taken in the name of the five years.
President of India. (b) The President may be removed from the
(b) The President of India appoints a person Office by way of impeachment.
as Attorney General of India provided (c) The President may resign before the
she/he is qualified to be a Judge of the expiration of his/her term by writing to
Supreme Court or a High Court. the Speaker of the Lok Sabha.
(c) The total number of Ministers in the (d) The President shall, notwithstanding the
Council of Ministers cannot exceed ten expiration of his/her term, continue to
percent of the total number of members hold Office unitl his/her successor enters
upon his/her Office.
of the House of People.
(d) The President of India is bound to act as 124. A special address by the Governor refers to
per the advice tendered by the Council of the address delivered by the Governor
Ministers. [CDS 2020 (II)]
119. Which one of the following methods is (a) when President’s Rule is called for
followed in electing the President of India? (b) when a national emergency necessitates
[CDS 2022 (I)] dissolution of Legislative Assembly
(c) at the commencement of the first session
(a) Proportional Representation only after general election and at the first
(b) First-Past-the-Post System only session of each year
(c) Proportional Representation and the (d) whenever he/she has concluded that such
Single Transferable Vote System is necessary
(d) Proportional Representation and the
First-Past-the-Post System 125. Which of the following is not related to the
powers of the Governor? [CDS Pre.2018 II]
120. National Disaster Management Authority is
(a) Diplomatic and Military powers
headed by [CDS 2021(I)]
(b) Power of appoint Advocate General
(a) the Prime Minister (c) Summoning, proroguing and dissolving
(b) the Home Minister State Legislature
(c) the President (d) Power to grant pardons, reprieves.
(d) the Health Minister respites or remission of punishments
121. Which one of the following is not a correct 126. Which one of the following regarding the
description of the Union Cabinet? ordinance making power of the Governor is
[CDS 2021(I)] not correct? [CDS Pre.2018 (II)]
(a) It is part of the Parliament. (a) It is not a discretionary power
(b) It is responsible to the Parliament. (b) The Governor may withdraw the
(c) It remains in power till it enjoys the ordinance anytime
confidence of the Parliament. (c) The ordinance power can be exercised
(d) A person from outside the Parliament when the Legislature is not in session
can never be appointed a member of the (d) The aid and advice of Ministers is not
Cabinet. required for declaring the ordinance

PYQ Workbook 164


INDIAN POLITY AND GOVERNANCE

127. Which of the following statements about the (a) 1 and 2


ordinance making power of the Governor (b) 2 and 3
is/are Correct? (c) 3 only
1. It is a discretionary power (d) 2 only
2. The Governor himself is not competent 131. Which one of the following powers has not
to withdraw the ordinance at any time. been conferred on the President of India?
Select the correct answer using the codes [CAPF 2021]
given below [CDS Pre.2018 (I)]
(a) Appointment of Prime Minister.
(a) Only 1 (b) To decide at his/her discretion any
(b) Only 2 question regarding disqualification of a
(c) Both 1 and 2
member of the Parliament.
(d) Neither 1 nor 2
(c) To grant pardon to a person who has been
128. Which of the following statements relating punished by a court-martial.
to the duties of the Governor is/are correct? (d) To remove a Judge of the High Court.
1. The duties of the Governor as a
132. The Governor of a state may exercise
Constitutional Head of the State do not
become the subject matter of questions or discretionary powers when
debate in the Parliament. 1. he or she is not convinced by the advice of
2. Where the Governor takes a decision the Council of Ministers.
independently of his Council of Ministers 2. the Constitution requires him/her to act
or where he acts as the Chief Executive in his/her discretion in certain matters.
of the State under President’s rule, his 3. he or she is appointed as the administrator
actions are subject to scrutiny by the of an adjoining Union Territory.
Parliament. Select the correct answer using the codes
Select the correct answer using the codes given below: [CAPF 2020]
given below: [CDS Pre.2018 (II)] (a) 1, 2 and 3
(a) Only 1 (b) 1 and 2 only
(b) Only 2 (c) 2 and 3 only
(c) Both 1 and 2 (d) 3 only
(d) Neither 1 nor 2
133. Which one of the following statements
129. Who among the following Prime Ministers about the Attorney-General of India is NOT
of India were defeated by a vote of No correct? [CAPF 2020]
Confidence?
(a) The person must be qualified to be
1. Morarji Desai appointed as a judge of the Supreme
2. Vishwanath Pratap Singh Court
3. HD Deve Gowda (b) He/she enjoys right of audience only in
4. Atal Bihari Vajpayee the Supreme Court and High Courts in
Select the correct answer using the codes India
given below : [CDS Pre.2016 (I)] (c) He/she holds office during the pleasure of
(a) 1, 2, 3 and 4 the President
(b) 1.2 and 3 (d) He/she receives remuneration as
(c) 2, 3 and 4 determined by the President
(d) 1 and 4
134. In relation to the powers of the Union
130. The Electoral College for electing the Government of India, which one of the
President of India does not include which following statements is NOT correct?
of the following? [CAPF 2020]
1. All elected members of the Rajya Sabha. (a) The Government of India may undertake
2. All elected members of the Legislative legislative functions of a territory
Assemblies of States. outside of India by agreement with the
3. All members of the Legislative Councils Government of that territory.
of States. (b) The Union Government may not entrust
Select the correct answer using the codes a State Government with functions over
given below. [CAPF 2021] matters where executive power of the

165 PYQ Workbook


INDIAN POLITY AND GOVERNANCE

Union extends. (a) The President has the power to grant


(c) The Union Government may direct pardon where punishment or sentence is
States for construction and maintenance by a Court Martial
of means of communication of military (b) The President can grant pardon in all
importance. cases where the sentence is a sentence of
(d) The Governor may, with consent of death
Government of India, entrust its officers (c) The Court’s power of judicial review is
with functions on matters which fall very limited in relation to the decision of
within the executive power of the State the President on mercy petition
(d) The power to grant pardon by the
135. Which of the following statement(s)
President is the power that the sovereign
regarding promulgation of ordinance is/are
never exercises against its own judicial
correct?
mandate
1. President can promulgate ordinances
which are of temporary duration. 140. Under Article 355 of the Constitution of
2. An ordinance promulgated by the India, it is the duty of the Union to
President has to be ratified by Parliament [CAPF 2015]
within six weeks of its re-assembly (a) protect Sates against external aggression
Select the correct answer using the codes and internal disturbance
given below: [CAPF 2020] (b) allow the extension of executive powers
(a) 1 only of the Union to any Sate
(b) 2 only (c) declare that the powers of the Legislature
(c) Both 1 and 2 of a State shall be exercisable by or under
(d) Neither 1 nor 2 the authority of the Parliament
(d) allow the Parliament to include or exclude
136. Who was the President of the Constituent any caste, race or tribe into the list of the
Assembly? [CAPF 2020] Scheduled Castes or Scheduled Tribes
(a) B R Ambedkar
141. The 2nd Administrative Reforms
(b) Rajendra Prasad
Commission has produced Reports on.
(c) Alladi Krishaswami Ayyar
(d) Sarvepalli Radhakrishnan 1. Ethics in Governance.
2. Local Governance.
137. Which one of the following statements 3. Combating Terrorism.
about the Attorney-General of India is NOT
4. Eradication of Corruption.
correct? [CAPF 2019]
(a) He shall have the right to speak in the Select the correct answer using the code given
Houses of Parliament. below. [CAPF 2015]
(b) He shall have a right of audience in all (a) 1, 3 and 4
courts in the territory of India in the (b) 2, 3 and 4
performance of his official duties. (c) 1 and 2
(c) He must have the same qualifications as (d) 1, 2 and 3
are required to be a Judge of the Supreme 142. Who is the Supreme Commander of the
Court. Indian Armed Forces? [CAPF 2015]
(d) He is a whole-time counsel for the (a) President
government. (b) Prime Minister
138. Which one of the following Commissions (c) Defence Minister
has not examined the issue of removal of (d) Longest serving Chief of Staff
the Governor of a State? [CAPF 2018] 143. Which of the following are true regarding
(a) Sarkaria Commission the Preamble of the Constitution of India?
(b) Thakkar Commission 1. The Objective Resolution was proposed
(c) Venkatachaliah Commission by Jawaharlal Nehru and passed by the
(d) Punchhi Commission Constituent Assembly which ultimately
139. Which one of the following statements became the preamble.
relating the power of the President of India 2. The idea of the preamble was borrowed
to grant pardon is not correct? [CAPF 2018] from the Constitution of the USA.

PYQ Workbook 166


INDIAN POLITY AND GOVERNANCE

3. The words Socialist and Secular were 1. One Governor can act as Governor of
adopted by the 42nd Amendment in 1976. more than one state.
4. The preamble incorporates the 2. The Governor has the power to grant
Fundamental Rights of the citizens of pardons, reprieves, remission of
India. Punishment to persons convicted under
Select the correct answer using the codes The State Law.
given below. 3. The Governor is appointed by the
Codes: [CAPF 2014] President on the recommendation of the
(a) 1, 2 and 3 Chief Minister of the respective state.
(b) 2, 3 and 4 Select the correct answer using the codes
(c) 3 and 4 given below.
(d) 1, 2 and 4 Codes: [CAPF 2014]
144. Which of the following statements are (a) 1 and 2
correct regarding the election of the (b) 1, 2 and 3
President of India? (c) 2 and 3
1. He is elected indirectly. (d) Only 1
2. He is elected through an electoral college 146. Consider the following statements about
consisting of all elected MPs and MLAs. Constitutional Government.
3. He is elected directly. 1. It is a form of limited government.
4. Members of the Legislative Council 2. One where the Constitution is the basis of
have no right to vote in the Presidential public authority.
election. 3. One where the Monarch is the formal
Select the correct answer using the codes head of the state.
given below. 4. One where there is always a universal
Codes: [CAPF 2014] adult franchise.
(a) 1 and 2 Which of the statements given above are
(b) 1, 2 and 4 correct? [CAPF 2014]
(c) 1 and 4 (a) 1 and 2
(d) 3 and 4 (b) 2 and 4
145. Which of the following is/are correct (c) 2 and 4
regarding the office of Governor in India? (d) 1 and 3

167 PYQ Workbook


INDIAN POLITY AND GOVERNANCE

SOLUTIONS

Hence, According to Clause (3) Article 22 of the Constitution


5.1. UPSC CSE Previous Years’ Questions of India, it is not mandatory for states to provide access to
1. Solution: (d) legal counsel to the person held for preventive detention.
Exp) Option d is the correct answer Statement 3 is incorrect. Section 32 (1) of the Prevention
The President is elected indirectly by the electoral college Of Terrorism Act (POTA) 2002 stated that “Notwithstanding
consisting of elected members of both the houses of anything in the Code 12 or in the Indian Evidence Act, 1872
Parliament, elected members of State legislative Assembly (1 of 1872), but subject to the provisions of this section, a
and elected members of the legislative assemblies of the confession made by a person before a police officer not
Union Territories of Delhi and Puducherry. lower in rank than a Superintendent of Police shall be
admissible in the trial of such person.”
Statement 1 is incorrect: If the election of a person as
President is declared void by the Supreme Court, acts done 3. Solution: (a)
by him before the date of such declaration of the Supreme
Exp) Option a is the correct answer.
Court are not invalidated and continue to remain in force.
Statement 1 is incorrect. The elected members of the
Statement 2 is incorrect: The Supreme court in 1974 held
Upper and Lower Houses of Parliament that is the Rajya
that the dissolution of state legislative assembly will not be
Sabha and the Lok Sabha as well as the elected Members of
a ground for preventing the holding of the election on the
the Legislative Assemblies of States and Union Territories
expiry of the term of the President. Nor can it be a ground
(MLAs) comprise the electoral college for the Presidential
to suggest that the election to office of the President could
Election in India.
be held only after the election to the state is held, where the
Legislative Assembly of a State is dissolved. Statement 2 is incorrect. The value of vote of each MLA
Statement 3 is incorrect: The Constitution of India does not depends on the ratio of the State’s population and the
prescribe any time-limit within which the President has to number of MLAs in its legislative Assembly. By dividing
take decision with respect to a bill presented to him/her for the State’s population by the number of MLAs in its
his/her assent. Thus the President of India can simply keep legislative Assembly, and then further dividing the quotient
the bills pending for an indefinite period. achieved by 1000, one may calculate the value of each MLA’s
vote. Based on a calculation that takes into account each
2. Solution: (b) State’s population in relation to the number of members in
Exp) Option b is the correct answer. its legislative Assembly, each MLA’s vote value varies from
Statement 1 is correct. Article 355 of the Indian Constitution State to State. According to the Constitution (Eighty-fourth
specifically states that “It shall be the duty of the Union Amendment) Act of 2001, the population of the States is
to protect every State against external aggression and currently calculated using data from the 1971 Census.
internal disturbance and to ensure that the government of Statement 3 is incorrect. The vote value of each MLA of
every State is carried on in accordance with the provisions of Madhya Pradesh is less than that of the vote value of each
this Constitution.” MLA of Kerala as the ratio of total population to total elective
Statement 2 is correct. According to Article 22 of Constitution seats in Kerala is greater relative to that in Madhya Pradesh.
of India (Protection against arrest and detention in certain Statement 4 is correct. The vote value of each MLA of
cases): Puducherry is higher than that of Arunachal Pradesh as the
1. No person who is arrested shall be detained in custody ratio of total population to total elective seats in Puducherry
without being informed, as soon as may be, of the is greater relative to that in Arunachal Pradesh. In 2022
grounds for such arrest nor shall he be denied the right Presidential election the vote value of each MLA from
to consult, and to be defended by, a legal practitioner of Puducherry was 16 whereas the vote value of each MLA
his choice from Arunachal Pradesh came out to be 8.
2. Every person who is arrested and detained in custody 4. Solution: (b)
shall be produced before the nearest magistrate within a
Exp) Option b is the correct answer
period of twenty-four hours of such arrest excluding the
time necessary for the journey from the place of arrest to Statement 1 is incorrect: There is no classification in the
the court of the magistrate and no such person shall be Constitution of India for the Council of Ministers. Article
detained in custody beyond the said period without the 74 provides that there shall be a Council of Ministers
authority of a magistrate with the Prime Minister at the head to aid and advise the
3. Nothing in clauses (1) and (2) shall apply (a) to any President who shall, in the exercise of his functions, act in
person who for the time being is an enemy alien; or (b) accordance with such advice.
to any person who is arrested or detained under any Statement 2 is correct: The Constitution (91st Amendment)
law providing for preventive detention Act, 2003 made Amendment in the Article 75. According to

PYQ Workbook 168


INDIAN POLITY AND GOVERNANCE

this amendment, the total number of ministers, including The period during which the House meets to transact its
the Prime Minister, in the COM shall not exceed 15% of business is called a session. There is no minimum number of
the total strength of the Lok Sabha. days that Parliament is required to meet in a year.

5. Solution: (d) 9. Solution: (c)


Exp) Option d is the correct answer Exp) Option c is the correct answer.
Statement 1 is incorrect: The Attorney General for India Statement 1 is correct. The Governor can summon
is appointed by the President of India under Article 76 of or prorogue the state legislature and dissolve the state
the Constitution and holds office during the pleasure of the legislative assembly. He can address the state legislature at
President. In the performance of his duties, he has the right the commencement of the first session after each general
to audience in all Courts in India as well as the right to take election and the first session of each year.
part in the proceedings of Parliament without the right to Statement 2 is correct. Article 208 states that, “A House
vote. Solicitor General of India has no such privilege. of the Legislature of a State may make rules for regulating
Statement 2 is incorrect: As per Article 76, the Attorney subject to the provisions of this Constitution, its procedure
General shall hold office during the pleasure of the and the conduct of its business.
President and shall receive such remuneration as the Until such rules are made, the rules of procedure and standing
President may determine Conduct of Government Business. orders in force immediately before the commencement
There is no mention in the Indian Constitution of the need of this Constitution with respect to the Legislature for the
for him to submit his resignation when the Government corresponding Province shall have effect in relation to
which appointed him resigns. legislature of state subject to modifications by Speaker of
6. Solution: (b) Legislative Assembly or Chairman of Legislative Council”.
Exp) Option b is the correct answer. 10. Solution: (c)
Statement 1 is incorrect. Sh. N. Gopala Swamy Ayyangar, Exp) Option c is the correct answer.
in his Report namely ‘Reorganization of the Machinery According to provisions made in the Constitution,
of Central Government’ in 1950 recommended for the Governor of a state has following immunities and office
grouping of ministries, improvement in the capabilities of conditions -
the personnel, and also in the working of the O&M Division.
Article 361(2): During his term of office, he is immune from
It nowhere mentions that a minister and a secretary be
any criminal proceedings, even in respect of his personal
designated solely for pursuing the subject of administrative
acts.
reform and promoting it.
Article 158(4): His emoluments and allowances cannot be
Statement 2 is correct. In 1970, on the basis of the
diminished during his term of office.
recommendations of the Administrative Reforms
Commission, the Department of Personnel was set up in the 11. Solution: (d)
Cabinet Secretariat. It was placed under the Prime Minister’s Exp) Option d is the correct answer.
Office. Neither 1 nor 2 is correct. The Chief Secretary of the state
7. Solution: (d) is appointed by the Chief Minister of the state. There is no
fixed tenure of a Chief Secretary in a state.
Exp) Option d is the correct answer.
However, at the central level, there is assured tenure of
The Union Ministry of Tribal Affairs (MoTA) is the nodal
two years for Cabinet Secretary, Home Secretary, Defence
agency for the implementation of the Scheduled Tribes
Secretary, Foreign Secretary as well as RAW and IB chief.
and Other Traditional Forest Dwellers (Recognition Of
Forest Rights) Act, 2006. The ministry deals with forest and The Administrative Reform Commission in 1969 had
livelihood objectives at the national level. recommended that a Chief secretary should have a minimum
tenure of three to four years.
8. Solution: (c)
12. Solution: (d)
Exp) Option c is the correct answer.
Exp) Option d is the correct answer.
Statement 1 is correct and statement 2 is incorrect.
According to Article 85(1) of the Constitution “The President Statement 1 is incorrect. All executive actions of the
shall from time to time summon each House of Parliament to Government of India are formally taken in the name of
meet at such time and place as he thinks fit, but six months President. He can make rules specifying the manner in which
shall not intervene between its last sitting in one session and the orders and other instruments made and executed in his
the date appointed for its first sitting in the next session.” name shall be authenticated. He also can make rules for more
Hence, Parliament must meet at least twice a year. convenient transaction of business of the Union government,
Statement 3 is correct. The parliament sits thrice in a year— and for allocation of the said business among the ministers.
Budget Session (February to May), Monsoon Session (July to Thus, the executive power of the Union of India is not
September) and Winter Session (November to December). vested in the Prime Minister, but in the President.

169 PYQ Workbook


INDIAN POLITY AND GOVERNANCE

Statement 2 is incorrect. Cabinet secretary (not Prime the Governor is by or under this Constitution required to
Minister) is the ex-officio chairman of the Civil Services act in his discretion. Hence, making the rules to conduct
Board. The Cabinet Secretariat functions directly under the the business of the State Government is not a discretionary
Prime Minister. The administrative head of the Secretariat power but a constitutional obligation. Also, the Governor
is the Cabinet Secretary who is also the ex-officio Chairman appoints Ministers on the advice of the Chief Minister.
of the Civil Services Board. The business allocated to
16. Solution: (d)
Cabinet Secretariat under Government of India (Allocation
of Business) Rules, 1961 includes (i) Secretarial assistance Exp) Option d is the correct answer
to the Cabinet and Cabinet Committees; and (ii) Rules of Statement 1 is incorrect: Cabinet ministers in India can be
Business. members of either house of Parliament (Lok Sabha or Rajya
Sabha) or not members of either house at the time of their
13. Solution: (c)
appointment. If a person who is not a member of either
Exp) Option c is the correct answer. house of Parliament is appointed as a Cabinet Minister, they
The Cabinet Secretariat is responsible for the administration are required to get elected to either the Lok Sabha or Rajya
of the Government of India (Transaction of Business) Rules, Sabha within six months from the date of their appointment.
1961 and the Government of India (Allocation of Business) Statement 2 is incorrect: The Cabinet Secretariat functions
Rules 1961. directly under the Prime Minister. The administrative head
This Secretariat provides Secretarial assistance to the Cabinet of the Secretariat is the Cabinet Secretary who is also the ex-
and its Committees, and also assists in decision-making in officio Chairman of the Civil Services Board.
Government by ensuring Inter-Ministerial coordination. It
also prepares agenda for Cabinet Meetings. 17. Solution: (b)
The Cabinet Secretariat ensures that the President, the Vice Exp) Option b is the correct answer
President and Ministers are kept informed of the major President of India addresses his resignation letter to Vice
activities of all Ministries/Departments by means of monthly President of India and in the absence of Vice President of
summary of their activities. India he will address it to the Chief Justice of India.
It does not allocate financial resources to the Ministries.
18. Solution: (d)
14. Solution: (a)
Exp) Option d is the correct answer.
Exp) Option a is the correct answer.
Statement 1 is incorrect: The Advocate General of a State
Statement 1 is correct. According to Article 77(3), in India is appointed by the Governor of the State under
the President shall make rules for the more convenient Article 165 of the Constitution, not by the President of India
transaction of the business of the Government of India and on the recommendation of the Governor.
for the allocation among Ministers of the said business.
Statement 2 is incorrect: High Courts have Original
Statement 2 is incorrect. According to Article 77(1),
Appellate and Writ jurisdiction not advisory jurisdiction.
all executive action of the Government of India shall be
High Courts primarily function as appellate courts and
expressed to be taken in the name of the President (and not
have the power to hear appeals from lower courts within
the Prime Minister).
their jurisdiction. High Courts also have writ jurisdiction to
15. Solution: (b) protect fundamental rights. but they do not have advisory
Exp) Option b is the correct answer. jurisdiction at the State level.
Only statements 1 and 3 are correct. 19. Solution: (b)
The governor can use discretionary powers: Exp) Option b is the correct answer
a) If no party gets an absolute majority, the governor can
Statement 1 is incorrect: The Ministries/Departments of
use his discretion in the selection of the chief minister;
the Government of India are created by the President on the
b) During an emergency he can override the advice of the advice of the Prime Minister under Government of India
council of ministers. At such times, he acts as an agent of (Allocation of Business) Rules, 1961.
the president and becomes the real ruler of the state;
Statement 2 is correct: The Prime Minister recommends
c) He uses his discretion in submitting a report to the the allocation of Ministries to different Ministers, and the
president regarding the affairs of the state; President formalizes these appointments.
d) He can withhold his assent to a bill and send it to the
president for his approval. 20. Solution: (c)

Options 2 and 4 are incorrect. Exp) Option c is the correct answer


Article 166(3) - The Governor shall make rules for the more In the order of precedence of the Government of India, the
convenient transaction of the business of the Government of Governors of States within their respective States
the State, and for the allocation among Ministers of the said comes above the other given Functionaries. The order of
business in so far as it is not business with respect to which precedence of important functionaries is given below.

PYQ Workbook 170


INDIAN POLITY AND GOVERNANCE

1. President
Important Tips
2. Vice-President
Order of precedence:
3. Prime Minister • The order of precedence is a list of dignitaries in an
4. Governors of States within their respective States order of their rank for ceremonial purposes and has no
5. Former Presidents role in the government functioning.
• It is neither an order of succession nor does it have any
6. Deputy Prime Minister
application in any parliamentary business.
7. Speaker of Lok Sabha and Chief Justice of India
• It is made by the President secretariat and maintained
21. Solution: (c) by the ministry of home affairs.
Exp) Option c is the correct answer • If in the table or order of precedence there are multiple
members of a similar rank, then they are listed in an
The Constitution of India has opted for the British alphabetical order and the order of precedence is based
Parliamentary System of Government rather than American on the date of entry to that particular office or rank .
Presidential System of Government. Even though the Indian
parliamentary system is largely based on the British pattern, 24. Solution: (a)
there are some fundamental differences between the two. For Exp) Option a is the correct answer
example, the Indian Parliament is not a sovereign body like If the Prime Minister of India belongs to the Rajya Sabha,
the British Parliament. Further, the Indian State has an elected he will not be able to vote in his favor in the event of a no-
head (republic) while the British State has hereditary head confidence motion. This is because the no-confidence
(monarchy). Hence Assertion (A) is correct. US president motion is moved and voted upon in the Lok Sabha, and as
is more or less directly elected whereas the Indian president a member of the Rajya Sabha, the Prime Minister does not
is indirectly elected. Hence office of Indain President is not have voting rights in the Lok Sabha.
similar to that of USA President. Therefore Reasoning (R)
25. Solution: (c)
is incorrect.
Exp) Option c is the correct answer.
22. Solution: (a) The Prime Minister of India has full discretion and
Exp) Option a is the correct answer. freedom in the choice of persons who are to serve as
ministers in his cabinet. The Prime Minister is the leader
Statement 1 is correct: The Attorney General of India
of the ruling party or coalition, and after being appointed by
is the highest law officer of the country and he/she is the
the President, he has the authority to select the members of
chief legal advisor to the Government of India. He/she is
his cabinet.
appointed by the President of India under Article 76 (1) of
the Constitution and holds office during the pleasure of the Important Tips
President. Important Articles in the Constitution related to Prime
Statement 2 is correct: The Attorney General of India must Minister
be a person qualified to be appointed as a Judge of the • ARTICLE 74: Council of Ministers aids and advises
Supreme Court. the President, who acts in accordance with the advice.
The President can ask for reconsideration but must
Statement 3 is incorrect: He/she need not be a member
follow the advice after that. Courts cannot inquire
of either House of Parliament. However, he/she has the
about advice.
right to attend and speak in any House of Parliament or any
committee thereof, but without a right to vote. • ARTICLE 75: The President appoints the Prime
Minister and other Ministers on the Prime Minister’s
Statement 4 is incorrect: He/she can be removed by the
advice. The total number of Ministers should not
President at any time without any reason or procedure. exceed 15% of the total House of the People members.
There is no provision for impeachment by Parliament for The article also mentions the collective responsibility
the Attorney General of India. of the Council of Ministers to the Lok Sabha.
23. Solution: (c) • ARTICLE 78: Prime Minister’s duties include
communicating Council of Ministers’ decisions and
Exp) Option c is the correct answer
legislation proposals to the President, providing
The protocol list or hierarchy of significant posts, known information upon request, and presenting individual
as the Order of Precedence of the Republic of India lists Minister’s decisions to the Council if required.
functionaries and officials in order of their rank and office
within the Indian government. Among the given options, 26. Solution: (d)
Chief Justice of India, Union Cabinet Minister, Chief Exp) Option d is the correct answer
Election Commissioner and Cabinet Secretary is the The elected members of the legislative assemblies of states
correct order of precedence. and the Union Territories of Delhi and Puducherry do not

171 PYQ Workbook


INDIAN POLITY AND GOVERNANCE

participate in the impeachment of the President though 29. Solution: (c)


they participate in his election. Exp) Option c is the correct answer
Important Tips According to Article -280 (1), the Chairman of the Finance
Commission is appointed by the President. According to
The President is elected not directly by the people but by
Article 239 (AA) (5) the Chief Minister of the Union Territory
members of electoral college consisting of:
is appointed by the President. The Vice-Chairman of the
• the elected members of both the Houses of Parliament; Planning Commission is not appointed by the President of
• the elected members of the legislative assemblies of the India.
states;
30. Solution: (c)
• the elected members of the legislative assemblies of the
Exp) Option c is the correct answer
Union Territories of Delhi and Puducherry1.
The Chief Minister of a State in India is not eligible to vote
The nominated members of both of Houses of Parliament,
in the Presidential election if he is a member of the Upper
the nominated members of the state legislative assemblies,
House of the State Legislature.
the members (both elected and nominated) of the state
legislative councils (in case of the bicameral legislature) Important Tip
and the nominated members of the Legislative Assemblies
The President is elected not directly by the people but by
of Delhi and Puducherry do not participate in the election
members of electoral college consisting of:
of the President.
• the elected members of both the Houses of Parliament;
27. Solution: (b) • the elected members of the legislative assemblies of the
Exp) Option b is the correct answer states;
While the Constitution outlines the process of forming a • the elected members of the legislative assemblies of the
government and the procedure for a vote of no confidence, Union Territories of Delhi and Puducherry.
it does not explicitly mention the requirement for the
31. Solution: (c)
Prime Minister to resign in case of losing the majority.
Instead, this practice has evolved as a convention in India’s Exp) Option c is the correct answer
parliamentary democracy. This convention is based on The Bill passed by the Parliament becomes an Act only when
the principle of responsible government, where the Prime President gives his assent to it. The President can withhold
Minister is accountable to the Parliament, particularly the or refuse to give assent on Bill presented. This power is
called “Veto Power” of the President. In 1986 President Zail
Lower House where the government needs the support to
Singh exercised the pocket veto with respect to the Indian
function effectively.
Post Office (Amendment) Bill. This Bill intended to impose
Important Tips restrictions on the freedom of the press.
• Under President discharge of functions act 1969,in the 5.2. Other Examination Previous Years’
event of the occurrence of vacancies in the office of Questions
the President and Vice President, by reason in each
case of death, registration or removal, or otherwise the 32. Solution: (a)
Chief Justice of India of the Supreme Court of India Exp) Option a is the correct answer
available shall discharge the functions of the President Neelam Sanjiva Reddy did not serve as Vice President
until a new President elected in accordance with the before assuming the office of the President of India. The
provisions of the constitution. only unopposed President of India, Neelam Sanjiva Reddy
• The Constitution does not mandate regional quotas became the President in 1977. He was elected on 21 July 1977
for Ministers. However, the convention of including and was sworn in as the sixth President of India on 25 July
representation from diverse regions in the Council of 1977. R. Venkatraman, Dr. Shankar Dayal Sharma, and Dr.
Ministers is followed to ensure inclusive governance Zakir Hussain serve as Vice President before assuming the
and consider regional interests. office of the President of India.

28. Solution: (d) Important Tips


Exp) Option d is the correct answer. • In India, Vice-President has the second-highest office
in the country.
Governor can be removed from office before the completion
• Article 63 of the Indian Constitution mentions the post
of his term by the President. There is not security of tenure
of Vice-President.
provided to the Governor. Article 156 also mentions that a
Governor shall continue to hold office until his successor • There is no direct election for the Vice-President
of India however, he/she is indirectly elected by an
enters upon his office. In such case, Governor can continue
Electoral College.
in office beyond the expiration of a period of five years.
Hence, both the statements are incorrect. • He acts as the ex-officio Chairman of Rajya Sabha

PYQ Workbook 172


INDIAN POLITY AND GOVERNANCE

33. Solution: (a) must be a person qualified to be appointed as a Judge of


Exp) Option a is the correct answer. the Supreme Court.
The Ex-officio Chairman of the National Disaster Statement 2 is incorrect: The Attorney General of India
Management Authority (NDMA) in India is the Prime shall receive such remuneration as the President may
Minister. The NDMA is the apex body responsible for determine. There is no provision in the Constitution or any
disaster management in the country. It sets policies, other law that empowers the Parliament to determine the
guidelines, and coordinates efforts for disaster preparedness, remuneration of the Attorney General of India.
mitigation, response, and recovery. As the Chairman of the 36. Solution: (b)
NDMA, the Prime Minister plays a crucial role in overseeing
Exp) Option b is the correct answer.
and guiding the country’s disaster management activities,
especially during major disasters or emergencies. Statement 1 is correct: The Governor does not have
the power to appoint a judge of a state High Court. The
Important Tips judges of a high court are appointed by the President in
• The NDMA was formally constituted on 27th consultation with the Chief Justice of India (CJI), the
September 2006, in accordance with the Disaster Governor of the State.
Management Act, 2005 with Prime Minister as its Statement 2 is incorrect: Governor is a part of the State
Chairperson and nine other members, and one such Legislature. In unicameral system, the state legislature
member to be designated as Vice-Chairperson. consists of the governor and the legislative assembly. In the
• Its primary purpose is to coordinate response to states having bicameral system, the state legislature consists
natural or man-made disasters and for capacity- of the governor, the legislative council and the legislative
building in disaster resiliency and crisis response. assembly.
• It is also the apex body to lay down policies, plans and Statement 3 is correct: Governor has the power to nominate
guidelines for Disaster Management to ensure timely some members of the Legislative Council. One sixth is
and effective response to disasters. nominated by the Governor from persons having knowledge
or practical experience in fields such as literature, science,
34. Solution: (c) arts, the co-operative movement and social services.
Exp) Option c is the correct answer. Statement 4 is incorrect: Governor has judicial powers
The legal basis for the extraordinary powers exercised by as well. He can grant pardons, reprives, respites and
the Government of India during the lockdown is provided remissions of punishment any person convicted of any
by the following Acts: offence against any law relating to a matter to which the
executive power of the state extends. He is consulted by the
• Disaster Management Act, 2005: The Disaster
president while appointing the judges of the concerned
Management Act, 2005, empowers the government
state high court.
to take necessary measures to prevent, mitigate, and
manage disasters, including public health emergencies. 37. Solution: (d)
It provides the legal framework for the government to Exp) Option d is the correct answer.
declare disasters, issue guidelines and orders, and take
Option a is correct: Article 361 provide protection to
actions to control the situation. President and Governors and Rajpramukhs. The President,
• Epidemic Diseases Act, 1897: The Epidemic Diseases Act, or the Governor shall not be answerable to any court for the
1897, gives the government the authority to take special exercise and performance of the powers and duties of his
measures and prescribe regulations to control the spread office or for any act done or purporting to be done by him
of epidemic diseases. This Act provides the legal basis in the exercise and performance of those powers and duties.
for the government to impose restrictions, quarantine Option b is correct: Writ of Mandamus issued by the court
measures, and other necessary actions during public to a public official asking him to perform his official duties
health emergencies. that he has failed or refused to perform. However, the writ
Therefore, the combination of the Disaster Management of mandamus cannot be issued against the president of
Act, 2005, and the Epidemic Diseases Act, 1897, gave the India or the state governors.
legal basis for the extraordinary powers exercised by the Option c is correct: According to Article 361, Governor
Government of India during the lockdown. is immune from criminal proceedings only. A civil
proceedings can be instituted against him during his term
35. Solution: (a)
of office in respect of his personal acts but only after giving
Exp) Option a is the correct answer. two months’ notice.)
Statement 1 is correct: The Attorney General of India is the Option d is incorrect: Article 74(2) barred courts from
highest law officer of the country and he/she is the chief legal inquiring into the advice given by Council of Ministers to
advisor to the Government of India. He/she is appointed by President. Similarly, Article 163 mentions that advice was
the President of India under Article 76 (1) of the Constitution tendered by Ministers to the Governor shall not be inquired
and holds office during the pleasure of the President. He/she into in any court.

173 PYQ Workbook


INDIAN POLITY AND GOVERNANCE

38. Solution: (d) Important Tips


Exp) Option d is the correct answer. The nominated members of both of Houses of Parliament,
Article 158 (3A) mentions that when the same person the nominated members of the state legislative assemblies,
is appointed as Governor of two or more States, the the members (both elected and nominated) of the state
emoluments and allowances payable to the Governor shall legislative councils (in case of the bicameral legislature)
be allocated among the States in such proportion as the and the nominated members of the Legislative Assemblies
President may by order determine. of Delhi and Puducherry do not participate in the election
of the President.
39. Solution: (d)
42. Solution: (d)
Exp) Option d is the correct answer
Exp) Option d is the correct answer
The election of a person as President cannot be challenged
When the office of President falls vacant due to his
or postponed on the ground that the electoral college was
resignation, removal, death or otherwise, the Vice-president
incomplete The President holds office for a term of five years
acts as the President until the new President is elected. If
from the date on which he enters upon his office. However,
the office of Vice- President is vacant, the Chief Justice of
he can resign from his office at any time by addressing the
India acts as the President and discharges the Function of
resignation letter to the Vice President. The President can
President.
hold office beyond his term of five years until his successor
assumes charge. When the vacancy is going to be caused by 43. Solution: (b)
the expiration of the term of the sitting President, an election Exp) Option b is the correct answer
to fill the vacancy must be held before the expiration of the Article 61 deals with the procedure for the impeachment
term. of the President. When a President is to be impeached for
40. Solution: (d) violation of the Constitution, the charge shall be preferred
by either House of Parliament. The proposal to prefer such
Exp) Option d is the correct answer.
charge is contained in a resolution which has been moved
The Attorney General of India is the highest law officer after at least fourteen days’ notice in writing signed by not
of the country and he/she is the chief legal advisor to the less than one-fourth of the total number of members of
Government of India. He/she is appointed by the President the House has been given of their intention to move the
of India under Article 76 of the Constitution and holds resolution, Such a resolution cannot be passed by a majority
office during the pleasure of the President. He/she can be of not less than two-thirds of the total number of members
removed by the President at any time without any reason. of that House.

Important Tips Important Tips


• The Comptroller and Auditor General of India can • Article 53 - The executive power of the Union shall be
be removed by the President only on an address from vested in the President and shall be exercised by him
both Houses of Parliament on the grounds of proved either directly or through officer’s subordinate to him
misbehavior or incapacity. in accordance with this Constitution.
• The Chief Election Commissioner can be removed by • Article 74 - mentions that the Council of Ministers
the president on the basis of a resolution passed to that is to aid and advise the President. But generally, the
effect by both the Houses of Parliament with special President asks the Council of Ministers to reconsider
majority, either on the ground of proved misbehavior the given advice, or otherwise, the President shall
or incapacity. act in accordance with the advice given after such
• The Speaker of Lok Sabha can be removed by a reconsideration.
resolution passed by a majority of all the then members • Article 13 - This article provides a constitutional basis
of Lok Sabha after giving him/her 14 days’ notice. for judicial review as it empowers the Supreme Court
and the High Courts to interpret pre-Constitution
41. Solution: (c)
laws and decide whether such laws are in line with the
Exp) Option c is the correct answer values and principles of the present Constitution.
The President is elected indirectly by the people but by
44. Solution: (a)
members of electoral college consisting of:
Exp) Option a is the correct answer.
• the elected members of both the Houses of Parliament;
The Advocate General of a State is the highest law officer
• the elected members of the legislative assemblies of the of a State and advises the State Government on legal
states; matters. He is appointed by the Governor of a State under
• the elected members of the legislative assemblies of the Article 165 of the Constitution and holds office during his
Union Territories of Delhi and Puducherry pleasure. He gives advice to the State Government upon such

PYQ Workbook 174


INDIAN POLITY AND GOVERNANCE

legal matters, which are referred or assigned to him by the 49. Solution: (c)
Governor. He also performs other duties as are assigned Exp) Option c is the correct answer
to him by the Governor or conferred upon him by the
The Union executive consists of the President, the Vice
Constitution or any other law. President, the Prime Minister, the council of ministers and
45. Solution: (a) the attorney general of India. All executive actions of the
Government of India are formally taken in the name of the
Exp) Option a is the correct answer.
President. He is the chief executive head of the Union.
Vice-President can also be removed from the office before Hence Assertion (A) is correct. The President of India has
completion of his term. A formal impeachment is not been made only a nominal executive; the real executive being
required for his removal. He can be removed by a resolution the Council of Ministers headed by the Prime Minister. There
of the Rajya Sabha (Council of States) passed by an absolute are limitations on the power of the President and does not
majority (ie, a majority of the total members of the House) enjoy unlimited power. Hence Reasoning (R) is incorrect.
and agreed to by the Lok Sabha. But, no such resolution
50. Solution: (a)
can be moved unless at least 14 days’ advance notice has
been given. Notably, no ground has been mentioned in the Exp) Option a is the correct answer
Constitution for his removal. V.V. Giri was closely associated with the labor and trade
union movement in India throughout his career. Giri was a
46. Solution: (d) founding member of the All-India Railway men’s Federation
Exp) Option d is the correct answer. which was formed in 1923 and served as its general secretary
The Prime Minister is the real head of the Council of Ministers, for over a decade. He was elected president of the All-India
not the nominal head. The Prime Minister is responsible to Trade Union Congress for the first time in 1926. In 1929, the
Indian Trade Union Federation (ITUF) was formed by Giri,
Parliament and can dismiss any of his ministers.
N. M. Joshi, and others with V. V. Giri as the president.
47. Solution: (c)
51. Solution: (b)
Exp) Option c is the correct answer
Exp) Option b is the correct answer.
In India, a motion of no confidence can be introduced
The Vice-President, like the President, is elected not directly
only in the Lok Sabha and after at least 50 members of
by the people but by the method of indirect election. S/he is
Lok Sabha support it, the Speaker may grant a leave and elected by the members of an electoral college consisting of
after considering the state of business in the House, allot a the members of both Houses of Parliament.
day or days or part of a day for the discussion of the motion
(under sub-rule (2) and (3) of rule 198 of Lok Sabha Rules, Important Tips
16th edition). • The original Constitution provided that the Vice-
President would be elected by the two Houses of
Important Tips
Parliament assembled at a joint meeting.
• Acharya Kripalani moved the first-ever no-confidence
• This cumbersome procedure was done away by the
motion on the floor of the Lok Sabha in August 1963,
11th Constitutional Amendment Act of 1961
immediately after the disastrous Sino-Indian War.
• As of July 2019, 27 no-confidence motions have been 52. Solution: (c)
moved. Exp) Option c is the correct answer
• Prime Minister Indira Gandhi faced the most no- Statement 1 is incorrect: Article 53 of the Constitution
confidence motions (15), followed by Lal Bahadur of India says that the executive power of the union shall be
Shastri and P. V. Narasimha Rao (three each), Morarji vested in President.
Desai (two) and Jawaharlal Nehru, Rajiv Gandhi, Atal Statement 2 is correct: The President appoints the Prime
Bihari Vajpayee, Narendra Modi (one each). Minister following the convention of inviting the leader
• The most recent no-confidence motion was against of the majority party or group in the Lok Sabha to form
the Narendra Modi government and accepted by the the government. The appointed leader becomes the Prime
Speaker but defeated by 325–126. Minister..
Statement 3 is correct: Article 75 provide for the Council
48. Solution: (c)
of Ministers with the Prime Minister at the head to aid and
Exp) Option c is the correct answer advice the President and he shall act in accordance with such
The New Economic Policy was announced by Prime advice while exercising his functions.
Minister P.V. Narsimha Rao in 1991. The policy was a major Statement 4 is correct: The person who is not a member
shift from the inward-looking and protectionist economic of either house of Parliament can be appointed as Prime
policies that had been followed by India since Independence. Minister. But within six months he should become a member
The New Economic Policy was based on the principles of of either house of Parliament otherwise he ceases to be the
liberalization, privatization, and globalization. Prime Minister.

175 PYQ Workbook


INDIAN POLITY AND GOVERNANCE

53. Solution: (c)


Important Tips
Exp) Option c is the correct answer
• No-Confidence Motion: This is a motion that is moved
In India’s parliamentary system, members of the cabinet can by the opposition parties in the Lok Sabha. It asks
be drawn from both houses of Parliament, i.e., the Lower the Lok Sabha to express its lack of confidence in the
House (Lok Sabha) and the Upper House (Rajya Sabha). This Council of Ministers. If the Lok Sabha passes the No-
provision allows for a broader representation and expertise Confidence Motion, the Council of Ministers must
to be included in the cabinet. resign.
• Censure Motion: This is a motion that is moved by the
54. Solution: (c) opposition parties in the Lok Sabha. It expresses the
Exp) Option c is the correct answer. disapproval of the Lok Sabha for the policies or actions
of the Council of Ministers. However, the Censure
The Attorney-General is considered to be the first law
Motion does not force the Council of Ministers to
officer of the Government of India. The Attorney-General
resign.
represents the Government of India in legal matters and
• Adjournment Motion: The adjournment motion is
provides legal advice to the government. The Solicitor-
introduced in the Parliament to draw the attention
General and Advocate-General are legal officers, but they do of the House to a definite matter of urgent public
not hold the position of the first law officer at the national importance and needs the support of 50 members to
level. be admitted. It is regarded as an extraordinary device
• The Solicitor General of India is the second-highest law because it interrupts the normal business of the House.
officer of the country and assists the Attorney General in 57. Solution: (b)
his/her functions.
Exp) Option b is the correct answer
• The Advocate General of a State is the highest law officer
The Constitution of India does not mention the office of
of a State and advises the State Government on legal the Deputy Prime Minister. It is an informal position that
matters. is created by the Prime Minister to assist him or her in
55. Solution: (b) the discharge of their duties. The Deputy Prime Minister
is not a constitutional functionary and does not have any
Exp) Option b is the correct answer special powers or privileges. It is thus an extra constitutional
The President is the Head of the executive as well as a growth.
constituent part of Parliament. Though the President of
Important Tips
India is a constituent part of Parliament, he does not sit or
Deputy Prime Ministers of India
participate in the discussions in either of the two Houses.
• India has had seven Deputy Prime Ministers since
Important Tips 1947.
Article 87 of the constitution provides two instances • Vallabhbhai Patel was the first, serving from 1947
until his death in 1950.
when the President specially addresses both Houses of
Parliament. The President of India addresses both the • Morarji Desai became the second in 1967 and
had the second-longest tenure. Charan Singh and
Rajya Sabha and the Lok Sabha at the beginning of the first
Jagjivan Ram succeeded him.
Session after each general election when the reconstituted
• Devi Lal represented two parties in the same post.
lower house meets for the first time. The President also
addresses both the houses at beginning of the first session • Lal Krishna Advani was the last Deputy Prime
Minister until the post became vacant intermittently.
of each year.
The President has been empowered to promulgate 58. Solution: (b)
Ordinances based on the advice of the central government Exp) Option b is the correct answer.
under Article 123 of the Constitution
The approval of ordinance promulgated by Governor is
56. Solution: (d) necessary by the State Legislature. An ordinance issued by
Governor should be laid before the legislative assembly or
Exp) Option d is the correct answer
both the Houses of the state legislature (in case of a bicameral
The Council of Ministers in India can move a Confidence legislature) when reassembles.
Motion in the Lok Sabha. This is a motion that asks the
Important Tips
Lok Sabha to express its confidence in the Council of
Ministers. If the Lok Sabha passes the Confidence Motion, Ordinance Making power of Governor:
the Council of Ministers remains in power. However, if the • Article 213 deals with the power of the Governor to
Lok Sabha does not pass the Confidence Motion, the Council legislate through ordinances. His power of ordinance
of Ministers must resign. making is quite similar to the President’s power.

PYQ Workbook 176


INDIAN POLITY AND GOVERNANCE

• He can promulgate ordinance when the Legislative the approval of both the Houses of Parliament and the assent
Assembly is not in session in case of the unicameral of the President of India. Hence the President is considered
legislature or when both Legislative assembly and as an integral part of the Parliament of India.
council are not in session in case of a bicameral 63. Solution: (a)
legislature
Exp) Option a is the correct answer
• His ordinance-making power is co-extensive with the
Article 160 in the Indian Constitution provides for the
legislative power of the state legislature. This means
discharge of the functions of the Governor in certain
that he can issue ordinances only on those subjects on
contingencies. It states that the President may make such
which the state legislature can make laws.
provision as he thinks fit for the discharge of the functions
• An ordinance issued by him has the same force and of the Governor of a State in any contingency not provided
effect as an act of the state legislature. Chapter II of the Constitution.
• The ordinance introduced by him can be withdrawn
Important Tips
anytime.
• Article 162 is related with the extent of executive power
• His power to promulgate ordinance is not a
of State.
discretionary power. Council of Ministers’ (headed by
CM) advice is a pre-requisite. • Article 165 talks about Advocate-General for the State.
The Governor of each State shall appoint a person who
• An ordinance issued by him ceases to operate on the
is qualified to be appointed as a Judge of a High Court
expiry of six weeks from the reassembly of the state
to be Advocate General for the State.
legislature. It may cease to operate even earlier than
the prescribed six weeks, if a resolution disapproving • Article 310 guarantees the right to hold a civil post in
it is passed by the legislative assembly and is agreed the Union or a state to any citizen of India, subject to
to by the legislative council (in case of a bicameral the provisions of the Constitution and any law made by
legislature). Parliament or the state legislature

59. Solution: (c) 64. Solution: (b)


Exp) Option b is the correct answer
Exp) Option c is the correct answer
Constitutional prerogative refers to the President’s
The President shall not be a member of either House of
constitutionally based authority to declare policy, take
Parliament or of a House of the Legislature of any state and if
action, and make law without congressional support or
any such member is elected President he shall be claimed to
in the face of inconsistent congressional legislation. The
have vacated his seat in that House on the date on which he
President may either give or withhold assent to a Money
enters upon office as President i.e., he will have to relinquish
Bill. Under the Constitution of India, a Money Bill cannot be
his seat as soon as he is elected.
returned to the House by the President for reconsideration.
60. Solution: (a) Hence, returning a money bill for reconsideration is not a
Exp) Option a is the correct answer constitutional prerogative of the president of India.

The President can seek advice from the Supreme Court on 65. Solution: (d)
any question of law or fact. However, the advice tendered by Exp) Option d is the correct answer
the Supreme Court is not binding on the President.
The president is the de jure head of country in contrast to
61. Solution: (a) the prime minister and the Prime Minister is referred to
as de facto head of state. President is called de jure head of
Exp) Option a is the correct answer
India because he is the head of the state not the head of the
The impeachment charges can be initiated by either House government. The President of India is referred to as a Nominal
of Parliament i.e. Lok Sabha or Rajya Sabha. These charges Head of State because India follows the parliamentary system
should be signed by one-fourth members of the House (that of the Government. In reality, the power is exercised by the
framed the charges), and a 14 days’ notice should be given to Council of Ministers and it is headed by the Prime Minister
the President. of India. The President of India takes decisions on behalf
of the Prime Minister (PM). Hence, Titular (formal) and
62. Solution: (a)
De jure; and Constitutional and Nominal authority are
Exp) Option a is the correct answer attributed to the President of the India
Assertion is correct. Legislature of the Union, which is
called Parliament, consists of the President and two Houses, 66. Solution: (d)
known as Council of States (Rajya Sabha) and House of the Exp) Option d is the correct answer
People (Lok Sabha). Thus the President is an integral part of The President of India is the head of the country. He can
the Parliament of India. declare any area as a scheduled area and has powers with
Reason is correct explanation for Assertion. A Bill is a respect to the administration of scheduled areas and tribal
statute in draft and cannot become law unless it has received areas

177 PYQ Workbook


INDIAN POLITY AND GOVERNANCE

maintains order, and ensures the smooth functioning of


Important Tips
the house, even though they are not a member of the Rajya
Executive Powers of President: Sabha.
• He appoints the attorney general of India and
70. Solution: (d)
determines his remuneration.
• He appoints the following people: Exp) Option d is the correct answer.

• Comptroller and Auditor General of India (CAG) Statement 1 is correct: According to Article 155, the
Governor of a State shall be appointed by the President by
• Chief Election Commissioner and other Election warrant under his hand and seal.
Commissioners
Statement 2 and 4 are correct: Article 156 of the Indian
• Chairman and members of the Union Public Service Constitution deals with the term of office of Governor.
Commission
The Governor shall hold office during the pleasure of the
• State Governors President. The Governor may, by writing under his hand
• Finance Commission of India chairman and addressed to the President, resign his office. A Governor
members shall hold for a term of five years from the date on which
• He appoints National Commissions of Scheduled he enters upon his office. Provided that a Governor shall
Castes, Scheduled Tribes, Other Backward Classes. continue to hold office until his successor enters upon his
office.
• He appoints administrators of union territories.
Statement 3 is correct: According to Article 154, the
67. Solution: (c) executive power of the State shall be vested in the
Exp) Option c is the correct answer Governor and shall be exercised by him either directly or
through officers subordinate to him in accordance with the
The President of India is the supreme commander of the
Constitution of India.
defence forces of India. In that capacity, he appoints the chiefs
of the Army, the Navy and the Air Force. Hence Assertion 71. Solution: (d)
(A) is correct. He can declare war or conclude peace, subject Exp) Option d is the correct answer.
to the approvalof the Parliament. The President’s powers as
The Constitution does not provide any power to state
Commander-in-Chief are not independent of Legislative
legislature related to removal of Governor. He cannot be
Control. Hence Reasoning (R) is incorrect.
removed earlier if the Legislature of the concerned State
68. Solution: (c) passes a resolution for his removal.
Exp) Option c is the correct answer. 72. Solution: (b)
Statement 1 is not correct: To be eligible for the office of Exp) Option b is the correct answer.
Vice-President of India, a person must be qualified for Under Article 200 of Indian Constitution, the Governor may
election as a member of the Rajya Sabha (the upper house reserve a Bill for the consideration of the President. After bill has
of Parliament), not the House of the People (Lok Sabha, the been passed by both Houses of the Legislature of the State, it shall
lower house of Parliament). be presented to the Governor and the Governor shall declare
Statement 2 is correct: A member of a State Legislature can either that he assents to the Bill or that he withholds assent
be a candidate for the office of Vice-President of India. therefrom or that he reserves the Bill for the consideration of
Statement 3 is correct: The term of the office of the Vice- the President.
President of India is the same as that of the President. Both Important Tips
hold office for a term of five years.
Other Articles given in the question:
Statement 4 is incorrect: The Vice-President of India
Article 169:
cannot be removed through a formal impeachment process.
Unlike the President, who can be impeached by Parliament • Article 169 of the Constitution allows Parliament to
for violation of the Constitution, there is no provision for either create or abolish a Legislative Council in a
impeachment of the Vice-President. The Vice-President can state if the Legislative Assembly of the State passes a
only be removed by a resolution passed by a majority of the resolution to that effect.
members of the Rajya Sabha present and voting Article 201:

69. Solution: (c) • Article 201 deals with procedure followed in case of
‘Bills Reserved for Consideration’ of the President.
Exp) Option c is the correct answer.
Article 202:
The Rajya Sabha is presided over by the Vice-President of
India, who is not a member of the Rajya Sabha. As per the • Under Article 202 of the Constitution of India, a
provisions of the Indian Constitution, the Vice-President statement of the estimated receipts and expenditure of
serves as the ex-officio Chairman of the Rajya Sabha. This the State for each financial year has to be laid before the
means that the Vice-President, in their capacity as the State Legislature, referred to as the “annual financial
Chairman, presides over the sessions of the Rajya Sabha, statement”.

PYQ Workbook 178


INDIAN POLITY AND GOVERNANCE

73. Solution: (b) 75. Solution: (c)


Exp) Option b is the correct answer Exp) Option c is the correct answer
In May 1969 when president Dr. Zakir Hussain died, the Neelam Sanjiva Reddy was one of the foremost political
then vice-president V.V. Giri was acting as the President. leaders from Andhra Pradesh. He participated in various
But he resigned from his post of Vice-president to contest freedom struggles against the British and was imprisoned
the presidential election. Then the Chief Justice of India M. several times. He was elected unopposed, the only President
Hidayatullah worked as the officiating president. to be elected thus, after being unanimously supported by all
political parties including the opposition Congress party.
Important Tips
• Mohammad Hidayatullah was the 11th Chief Justice of Important Tips
India. • When the Andhra Province was separated from Madras
• He had also served as the Acting President of India and T. Prakasam became the Chief Minister of Andhra,
from 20 July 1969 to 24 August 1969 and from 6 Sanjeeva Reddy was appointed Deputy Chief Minister.
October 1982 to 31 October 1982. • When the new state of Andhra Pradesh was constituted
74. Solution: (c) Sanjeeva Reddy became its first Chief Minister and
served in that capacity from 1956 to 1959.
Exp) Option c is the correct answer.
• He again became the Chief Minister of Andhra Pradesh
The Draft Constitution provided for the direct election of
in 1962 for one year
the governor on the basis of universal adult suffrage. But this
plan was dropped due to various reasons. Like, it would have 76. Solution: (d)
meant another election; the elected Governor might have Exp) Option d is the correct answer.
considered himself superior to the Chief Minister; and the
Statement 1 is incorrect: The Constitution lays down only
Governor had to function under the Parliamentary System.
two qualifications for the appointment of a person as a
However, ‘the election would have been fought on major
governor. One of them is: He should be a citizen of India.
political issues’ is not the reason for it. Hence, option c is
But it is not specified that he must be citizen of India by
correct answer.
birth.
Important Tips Statement 2 is correct: Second qualifications for the
The Constituent Assembly opted for the present system appointment of a person as a governor is: He must have
of appointment of governor by the president because of completed the age of 35 years.
the following reasons: Statement 3 is incorrect: ‘He must be eligible to be
• The direct election of the governor is incompatible a member of the Lok Sabha’ is not mentioned in the
with the parliamentary system established in the states. constitution. There is no such provision. However, this
• The mode of direct election is more likely to create statement stands true in case of President.
conflicts between the governor and the chief minister. Statement 4 is correct: Usually, there is a governor for
• The governor being only a constitutional (nominal) each state, but the 7th Constitutional Amendment Act of
head, there is no point in making elaborate arrangements 1956 facilitated the appoin.tment of the same person as a
for his election and spending huge amount of money. governor for two or more states.
• It is not in the national interest to involve a large 77. Solution: (b)
number of voters in such an election. Exp) Option b is the correct answer.
• An elected governor would naturally belong to a party The Solicitor General of India is the second-highest law
and would not be a neutral person and an impartial officer of the country and assists the Attorney General of
head. India, the highest law officer and chief legal adviser to the
• The election of governor would create separatist Government of India. The SGI advise the Government and
tendencies and thus affect the political stability and appear on behalf of the Union of India in terms of the Law
unity of the country. Officers (Conditions of Service) Rules, 1972.
• The system of presidential nomination enables the Important Tips
Centre to maintain its control over the states.
• The Appointments Committee of the Cabinet (ACC)
• The direct election of the governor creates a serious recommends the appointment and officially appoints
problem of leadership at the time of a general election the Solicitor General. Article 76 does not mention the
in the state. solicitor general and additional solicitor general.
• The chief minister would like his nominee to contest • The proposal for appointment is generally moved by
for governorship. Hence, a second rate man of the the Law Secretary in the Department of Legal Affairs
ruling party is elected as governor. and approved by the Minister of Law & Justice.

179 PYQ Workbook


INDIAN POLITY AND GOVERNANCE

• The Solicitor General is assisted by Additional Solicitors pending or otherwise. Thus it is literally not a power of
General of India, who are also statutory posts. President.
• The first Solicitor General of India was C. K. Daphtary, 82. Solution: (b)
who was appointed on 28 January 1950. He later
Exp) Option b is the correct answer.
became the Attorney General of India in 1963
Statement 1, 3 and 4 are correct: Article 159 deals with
78. Solution: (b) Oath or affirmation by Governor. Before entering upon his
Exp) Option b is the correct answer office, the governor has to make and subscribe to an oath
Assertion (A) is true: In a parliamentary form of or affirmation. The Governor shall make and subscribe
government, the ministers are responsible for formulating to the oath or affirmation in the presence of Chief Justice
and implementing government policies. The civil servants of the High Court or in his absence senior most Judge of
are responsible for providing advice and assistance to the that Court. In his oath, the governor swears: (a) to faithfully
ministers, but they do not have the authority to make policy execute the office; (b) to preserve, protect and defend the
decisions.
Constitution and the law; and (c) to devote himself to the
Reason (R) is true: Cabinet collective responsibility, service and well-being of the people of the state.
also known as collective ministerial responsibility, is
a constitutional convention in parliamentary systems Statement 2 is incorrect: The format of oath or affirmation
that members of the cabinet must publicly support all is not given in the Third Schedule of Indian Constitution.
governmental decisions made in Cabinet, even if they do not The third Schedule contains the “Forms of Oaths or
privately agree with them. Affirmations” of various constitutional posts. However,
(A) is not the correct explanation of (R): The fact that the oaths or Affirmations for the post of Governor or
ministers make policy decisions and civil servants adhere to President are not included in this schedule.
those policies is a general characteristic of how governments
Important Tips
function, not necessarily specific to the principle of ‘Cabinet
Collective Responsibility’. The third schedule of the Indian Constitution contains
the forms of oath and affirmation for:
79. Solution: (b) • Union Ministers of India
Exp) Option b is the correct answer. • The candidates for election to the Parliament
The Governor of a state is responsible to the President of • The Members of Parliament (MPs)
India. Under article 153, there shall be a governor of each
state. President appoints Governor by warrant under his • The Judges of Supreme Court
hand or seal. • The Comptroller and Auditor General of India
• State Ministers
80. Solution: (a)
• The candidates for election to the state legislature
Exp) Option a is the correct answer.
• The members of the state legislature
The President can be removed from office by a process of
impeachment for ‘violation of the Constitution’. However, • The judges of the High Courts
the Constitution does not define the meaning of the phrase
83. Solution: (a)
‘violation of the Constitution. The process of impeachment
is a quasi-judicial procedure of the Parliament. Exp) Option a is the correct answer.
The first woman Chief Minister of India was Sucheta
81. Solution: (c)
Kripalani. She was a freedom fighter and a politician who
Exp) Option c is the correct answer
served as the fourth Chief Minister of Uttar Pradesh from
The powers of the Indian President can be broadly classified
1963 to 1967.
under 8 headings.
• Legislative – Promulgation of Ordinances Important Tips
• Executive or Appointment powers About Sucheta Kripalani:
• Judicial powers – To consult Supreme Court • She was a member of the Constituent Assembly
that drafted the Indian Constitution and the founder
• Financial power
president of the All India Women Congress (AIWC).
• Diplomatic powers
• She was married to Acharya JB Kripalani, a former
• Military powers Congress president and a Gandhian leader.
• Pardoning Powers – To grant Pardon • She became the first woman Chief Minister of India
• Emergency powers on October 2, 1963, after Chandra Bhanu Gupta
Article 86 of the constitution states that the president can resigned as the Chief Minister of Uttar Pradesh under
send messages to the Houses of Parliament relating to bills K Kamaraj’s plan.

PYQ Workbook 180


INDIAN POLITY AND GOVERNANCE

• She led the Congress party to victory in the 1967 state • H.D. Devegowda was the Chief Minister of Karnataka
elections, but resigned shortly after due to internal from 1994 to 1996. He was the Prime Minister of India
conflicts in the party for a brief period in 1996-1997.
• Narendra Modi was the Chief Minister of Gujarat from
84. Solution: (b) 2001 to 2014. He is the Prime Minister of India since
Exp) Option b is the correct answer 2014.
In the order of precedence of the government of India, • Morarji Desai was the Chief Minister of Bombay (now
Former President’s comes above the Chief Justice of India. Mumbai) from 1952 to 1956. He was the Prime Minister
The order of precedence of important functionaries is given of India from 1977 to 1979.
below.
88. Solution: (c)
1. President
Exp) Option c is the correct answer
2. Vice-President
The president of India is not directly elected by the people of
3. Prime Minister India but is rather elected indirectly by the cabinet members.
4. Governors of States within their respective States Hence, the president does not exercise direct powers but has
subordinate powers. This means that the president needs the
• Former Presidents
approval of the Prime Minister and the cabinet members,
• 5A. Deputy Prime Minister except in very rare cases. As the Lok Sabha is run by the
• Chief Justice of India Cabinet members and the Prime Minister, the president can
dissolve the Lok Sabha on the recommendation of the Prime
85. Solution: (c)
Minister or the Union Cabinet.
Exp) Option c is the correct answer
89. Solution: (d)
The President holds office for a term of five years from
the date on which he enters upon his office. However, he Exp) Option d is the correct answer
can resign from his office at any time by addressing the The President’s Address to the Parliament is prepared
resignation letter to the Vice President. Further, he can also by the Union Cabinet because the real executive power is
be removed from the office before completion of his term by vested in the Union Cabinet. All executive power is vested
the process of impeachment. in the President of India. The council of minister headed by
Prime Minister aids and advises the President who exercises
86. Solution: (a) his powers in accordance with the advice.
Exp) Option a is the correct answer
90. Solution: (c)
Article 108 of the Constitution empowers the President to
Exp) Option c is the correct answer
summon a joint session of both houses “for the purpose
of deliberating and voting on the Bill”. The President may, The Cabinet Secretary is the top-most executive official and
unless the Bill has elapsed by reason of a dissolution of the senior-most civil servant of the Government of India. All
other functionaries like Defence Secretary, Home Secretary
House of the People, notify the Houses by message if they
and Lt. Governor comes below the Cabinet Secretary in the
are sitting or by public notification if they are not sitting, his
order of preference.
intention to summon them to meet in a joint sitting for the
purpose of deliberating and voting on the Bill.. Important Tips
Important Tips • The Cabinet secretary is the head of all civil services in
India and is the senior-most IAS officer of the country.
• Joint Sitting is presided over by the Speaker of the Lok
Sabha or in his absence by the Deputy Speaker of the • A cabinet secretary has many responsibilities, and the
person holding the position is also the ex-officio head
Lok Sabha or in his absence, the Deputy-Chairman of
of the civil services.
the Rajya Sabha.
• He is directly accountable to the prime ministers and
• In no case, the joint sitting is presided over by the
heads various country authorities.
chairman of the Rajya Sabha.
• The Cabinet secretary is answerable to the prime
87. Solution: (d) minister and the president and vice-president of
Exp) Option d is the correct answer the country where he should pass on all the major
information of the decisions made by the cabinet and
• Charan Singh was the Chief Minister of Undivided Uttar its activities to the head of the country.
Pradesh from 1967 to 1968, and again in 1970. He was the
Prime Minister of India for a brief period in 1979-1980. 91. Solution: (d)
• P.V. Narasimha Rao was the Chief Minister of Undivided Exp) Option d is the correct answer
Andhra Pradesh from 1971 to 1973. He was the Prime If the office of President falls vacant by resignation, removal,
Minister of India from 1991 to 1996. death or otherwise, then election to fill the vacancy should

181 PYQ Workbook


INDIAN POLITY AND GOVERNANCE

be held within six months from the date of the occurrence of • All committees except Cabinet Committee on
such a vacancy. So the Vice President can hold the office of Accommodation and Cabinet Committee on
President for not more than six months. Parliamentary Affairs are headed by the Prime
Minister.
92. Solution: (a)
• Cabinet Committe on Accomodation is chaired by
Exp) Option a is the correct answer. the Home Minister of India and Parliamentary Affairs
Governor’s salary and allowances are given by Consolidated Committee is headed by Defence Minister.
Fund of the State. When he is appointed as the Governor of
95. Solution: (e)
two or more state, then burden of salary is charged on the
Consolidated Fund of all such states. Exp) Option e is the correct answer
The pardoning power of the President includes the following:
93. Solution: (c)
Pardon: It removes both the sentence and the conviction
Exp) Option c is the correct answer. and completely absolves the convict from all the sentences,
Statement 1 is incorrect: While acting as President or punishments, and disqualifications.
discharging the functions of President, the Vice-President Commutation: Denotes the substitution of one form of
does not perform the duties of the office of the chairman of punishment for a lighter form.
Rajya Sabha. During this period, those duties are performed Remission: Reducing the period of a sentence without
by the Deputy Chairman of Rajya Sabha. changing its character.
Statement 2 is correct: During any period when the Vice- Respite: Delays in the imposition of a sentence but in no way
President acts as President or discharges the functions of modify a sentence or address questions of due process, guilt,
the President, he is not entitled to the salary or allowance or innocence, due to some special fact, such as the physical
payable to the Chairman of Rajya Sabha, but the salary and disability of a convict or the pregnancy of a woman offender.
allowance of the President. Reprieve: It implies a stay of the execution of a sentence
Statement 3 is correct: When a vacancy occurs in the office (especially that of death) for a temporary period.
of the President due to his resignation, removal, death or
Important Tips
otherwise, the Vice-President acts as the President until a
• Article 72 of the Constitution empowers the President
new President is elected. In case the office of Vice-President
to grant pardons to persons who have been tried and
is vacant, the Chief Justice of India acts as the President or convicted of any offense
discharges the functions of the President.
• He can grant pardon, reprieve, respite and remission
94. Solution: (d) of punishment, or suspend, remit or commute the
sentence of any person convicted of any offence:
Exp) Option d is the correct answer
• In all cases where the punishment or sentence is by
Statement I is true: The Constitution of India does not a c martial;
mention Cabinet Committees. The Rules of Business provide
• In all cases where the punishment or sentence is for
for the formation of Cabinet Committees.
an offer against a Union law; and
Statement II is true: The Prime Minister has the discretion • In all cases where the sentence is a sentence of death
to set up Cabinet Committees as and when he or she deems
necessary. 96. Solution: (b)
Statement III is false: If Prime Minister is the member of a Exp) Option b is the correct answer
Committee, he invariably presides over it. It is not essential for the President to take advice from the
Statement IV is false: Parliamentary Affairs Committee is Council of Minister while giving assent to bills. On other
not chaired by PM but by Defence Minister. matters like appointment of Ambassadors, to dissolve Lok
Sabha, to give assent on Constitutional Amendment bill and
Important Tips to give assent to bills referred by governors, it is essential for
• Eight Cabinet Committees: the President to take advice from Council of Minister.
• Appointments Committee of the Cabinet.
97. Solution: (a)
• Cabinet Committee on Accommodation.
Exp) Option a is the correct answer.
• Cabinet Committee on Economic Affairs.
The Attorney General of India is the highest law officer
• Cabinet Committee on Parliamentary Affairs. of the country and he/she is the chief legal advisor to the
• Cabinet Committee on Political Affairs. Government of India. He/she is appointed by the President
• Cabinet Committee on Security. of India under Article 76 of the Constitution and holds office
during the pleasure of the President. He/she gives advice to
• Cabinet Committee on Investment and Growth.
the Government of India upon such legal matters, which
• Cabinet Committee on Employment & Skill are referred or assigned to him/her by the President. He/she
Development. also performs other duties of a legal character assigned to

PYQ Workbook 182


INDIAN POLITY AND GOVERNANCE

him/her by the President or conferred upon him/her by the 102. Solution: (c)
Constitution or any other law. Exp) Option c is the correct answer
98. Solution: (b) The Constitution (91st Amendment) Act, 2003 inserted
Exp) Option b is the correct answer clause 1A in Article 75, which says “The total number of
A minister who has resigned from the government does not Ministers, including the Prime Minister, in the Council
lose his or her membership of the Lok Sabha. Therefore, he of Ministers shall not exceed fifteen per cent of the total
or she can still participate in the debates and discussions in number of members of the House of the People”
the Lok Sabha. However, he or she cannot make a statement
in the Lok Sabha without the permission of the Speaker, just Important Tips
like any other member. • The Constitution (91st Amendment) Act, 2003
99. Solution: (e) inserted clause 1A in Article 164, which says “the total
number of Ministers, including the Chief Minister, in
Exp) Option e is the correct answer
the Council of Ministers in a State shall not exceed
Though there is nothing called discretionary powers of the 15% of the total number of members of the Legislative
president in the constitution, the president enjoys certain Assembly of that State.
circumstantial powers and the president can exercise these
• It also provided that the number of Ministers, including
powers without the aid and advice of the council of ministers.
the Chief Minister in a State shall not be less than
There are certain Discretionary power with President like:
twelve.
• To return a Bill with objections
• It also provided that if the existing number exceeds
• To withhold a Bill
the limit, it must be reduced within six months of the
• To send message to Parliament President’s notification.
100. Solution: (c) 103. Solution: (e)
Exp) Option c is the correct answer
Exp) Option e is the correct answer
Statement 1 is correct: The Cabinet Secretariat is responsible
for preparing the agenda for Cabinet Meetings. The agenda The Chairman of UPSC, the Chief Election Commissioner,
includes various issues and policy matters that need to be and the Comptroller and Auditor General are at same order
discussed and decided upon by the Cabinet. of Precedence i.e 9A, and the Chief Justice of High Court is
Statement 2 is correct: The Cabinet Secretariat provides rank at number 14 in number of precedence.
administrative and secretarial assistance to various Cabinet
Committees. Cabinet Committees are smaller groups of 104. Solution: (c)
Cabinet Ministers formed to handle specific matters and Exp) Option c is the correct answer.
make recommendations to the Cabinet.
The Prime Minister and their cabinet are at all times
Statement 3 is incorrect: The allocation of financial responsible to the Lok Sabha. The Prime Minister is
resources to the Ministries is not a function of the Cabinet
appointed by the President of India; however the Prime
Secretariat. This task is typically carried out by the
Ministry of Finance and is based on the government’s Minister has to enjoy the confidence of the majority of Lok
budgetary process and priorities. Sabha members, who are directly elected every five years,
lest the prime minister shall resign.
101. Solution: (d)
Exp) Option d is the correct answer 105. Solution: (b)
Article 57 describes the eligibility for re-election to the post Exp) Option b is the correct answer
of the President of India. It provides ‘A person who holds, or
The Cabinet Secretary is the highest-ranking civil servant
who has held the office of President, shall subject to the other
in the Government of India. He or she is the head of the
provisions of this Constitution, be eligible for reelection to
that office’. Civil Services Board and the Cabinet Secretariat. The
Cabinet Secretary is responsible for providing secretarial
Important Tips
and administrative support to the Union Cabinet and the
• According to Article 52 of Constitution of India, the
Prime Minister.
President is the head of state of the Republic of India.
• According to Article 54 of the Constitution, the 106. Solution: (a)
President of India is elected by an Electoral College
Exp) Option a is the correct answer
comprised of elected members of both houses of
Parliament and Legislative Assemblies of all States, The executive power of the Union is vested in President and
the Union Territory of Puducherry, and the National it is to be exercised by him either directly or through officers
Capital of Delhi.
subordinate to him in accordance with this Constitution
• Article 55 deals with the manner of election of President (Article 53).

183 PYQ Workbook


INDIAN POLITY AND GOVERNANCE

Important Tips 109. Solution: (d)

• The Constitution of India has provided for a Exp) Option d is the correct answer
parliamentary form of government. Consequently, the The President can be removed from office by a process
President has been made only a nominal executive; the of impeachment for ‘violation of the Constitution’. The
real executive being the council of ministers headed by impeachment charges can be initiated by either House of
the prime minister. Parliament.
• The council of ministers shall be collectively responsible
110. Solution: (c)
to the Lok Sabha (Article 75). This provision is the
foundation of the parliamentary system of government. Exp) Option c is the correct answer
According to Article 71 of the Indian Constitution, all doubts
107. Solution: (b)
and disputes arising out of or in connection with the election
Exp) Option b is the correct answer of a President or Vice-President shall be inquired into and
The President of India can remove a judge but only after decided by the Supreme Court. The Presidential and Vice-
the procedure for removal of Judge is followed. It is not his Presidential Elections Act, 1952, an election petition can be
right to remove a judge of Supreme Court. The President filed before the Supreme Court
has power to Pardon, he can declare emergency, and pass
111. Solution: (c)
ordinances.
Exp) Option c is the correct answer.
Important Tips
The Vice-President of India is not appointed by the
• A Judge of the Supreme Court cannot be removed from President of India. The Vice-President is elected by an
office except by an order of the President passed after electoral college consisting of members of both houses of
an address in each House of Parliament supported by Parliament. The members of the electoral college include
a majority of the total membership of that House and elected members of Rajya Sabha (the upper house of
by a majority of not less than two-thirds of members
Parliament) and Lok Sabha (the lower house of Parliament).
present and voting, and presented to the President in
the same Session for such removal on the ground of Important Tips
proved misbehavior or incapacity.
• As per Article 58 of the Indian Constitution, a person
108. Solution: (a) must be at least 35 years old to be eligible for the office
of President of India.
Exp) Option a is the correct answer.
• The Vice-President of India serves as the ex-officio
The word ‘mantrimandal’ (cabinet) is mentioned only once
Chairman of the Rajya Sabha, which is the upper
in the Indian Constitution, and that is in Article 352(3).
house of Parliament.
This article deals with the proclamation of emergency. It
• The Vice-President presides over the sessions of the
states that the President shall not issue a proclamation of
Rajya Sabha, maintains order, and ensures the smooth
emergency unless the Union Cabinet communicates to him
functioning of the house.
in writing that such a proclamation may be issued. Union
Cabinet is defined as the Council consisting of the Prime 112. Solution: (c)
Minister and other Ministers of Cabinet rank under Article
Exp) Option c is the correct answer
75.
The National Authority for Chemical Weapons Convention
Important Tips (NACWC) has been set up as an office of the Cabinet
• The impeachment charges should be signed by one- Secretariat, Government of India in 1997 to fulfill, on
fourth members of the House (that framed the charges), behalf of the Government of India, the obligations under the
and a 14 days’ notice should be given to the President. Chemical Weapons Convention and to act as the national
• After the impeachment resolution is passed by a focal point for effective liaison with the Organization for the
majority of two-thirds of the total membership of that Prohibition of Chemical Weapons (OPCW) and other State
House, it is sent to the other House, which should Parties on matters relating to the Convention.
investigate the charges. 113. Solution: (c)
• The President has the right to appear and to be Exp) Option c is the correct answer
represented at such investigation.
The Solicitor General of India (SGI) is the second-highest
• If the other House also sustains the charges and passes the law officer of the country, after the Attorney General of
impeachment resolution by a majority of two-thirds of India. The SGI is not a constitutional post but a statutory
the total membership, then the President stands removed position. Solicitor general and Additional Solicitor
from his office from the date on which the resolution is so Generals’ office and duties are governed by Law Officers
passed. (Conditions of Service) Rules, 1987.

PYQ Workbook 184


INDIAN POLITY AND GOVERNANCE

114. Solution: (c) Delhi and Puducherry (excluding Jammu and Kashmir, as
Exp) Option c is the correct answer specified in the 70th Constitutional Amendment Act).
The Estimates Committee is a standing committee of the 117. Solution: (a)
Lok Sabha, the lower house of the Parliament of India. It Exp) Option a is the correct answer.
is responsible for scrutinizing the government’s budgetary
Article 78 of the Indian Constitution pertains to the duties
proposals and suggesting ways to improve their efficiency
of the Prime Minister. It states that it shall be the duty of the
and economy. The committee has 30 members, all of whom
Prime Minister to communicate to the President all decisions
are elected members of the Lok Sabha.
of the Council of Ministers relating to the administration
115. Solution: (a) of the Union and proposals for legislation. Additionally,
Exp) Option a is the correct answer the Prime Minister shall furnish such information to the
President as the President may call for. Article 78 outlines
According to Article 59 of the Constitution of India, the responsibilities of the Prime Minister in keeping the
The President shall not be a member of either House of President informed about the decisions and functioning of
Parliament or of a House of the Legislature of any State, and the Council of Ministers.
if a member of either House of Parliament or of a House of
the Legislature of any State be elected President, he shall 118. Solution: (c)
be deemed to have vacated his seat in that House on the Exp) Option c is the correct answer.
date on which he enters upon his office as President. Thus The 91st Constitutional Amendment Act, of 2003
any Indian citizen who is eligible to become a member of introduced provisions to address the size of the Council
Parliament can also be elected as the President, irrespective of Ministers, prevent defections, and strengthen the anti-
of whether they are a Union Minister or not. There is no defection law. One of the provisions states that the total
explicit restriction on Union Ministers from being eligible number of ministers, including the Prime Minister, in the
for the position of the President. Central Council of Ministers, should not exceed 15% of the
Important Tip: total strength of the Lok Sabha.
Facts regarding Election of President of India 119. Solution: (c)
• The Indian President is elected through an electoral Exp) Option c is the correct answer.
college system, by proportional representation by a As per Article 55(3) of the Constitution of India, the
single transferable vote. The Elections are conducted election of the President shall be held in accordance with
and overseen by the Election Commission of India. the system of proportional representation by means of
• The electoral college is based on all elected members single transferable vote and the voting at such election shall
of Parliament (MPs), and the elected members of be by secret ballot. This means that the voting process for
State Legislative Assemblies and Union Territories the election of the President involves the use of the single
(MLAs). transferable vote system, which allows voters to rank the
• Before voting, in the nomination stage, the candidate candidates in order of preference.
files nomination with a signed list of 50 proposers and 120. Solution: (a)
50 seconders. These can be anyone from the members of
Exp) Option a is the correct answer
the electoral college from the State or national level. This
rule was implemented by EC in 1974. An elector cannot The National Disaster Management Authority (NDMA) is
propose or second the nomination of more than one the apex statutory body for disaster management in India
candidate. and is headed by the Prime Minister. As the head of the
NDMA, the Prime Minister oversees the overall functioning
• A vote cast by each MP or MLA is not calculated as
of the organization and provides strategic direction for
one vote.
disaster management initiatives.
• The fixed value of each vote by an MP of the Rajya
Sabha and the Lok Sabha is 700. Important Tip
• Meanwhile, the vote value of each MLA differs from Mandate: Its primary purpose is to coordinate response to
State to State based on a calculation that factors in natural or man-made disasters and for capacity-building
its population vis-a-vis the number of members in its in disaster resiliency and crisis response. It is also the
legislative Assembly. apex body to lay down policies, plans and guidelines
for Disaster Management to ensure timely and effective
116. Solution: (c) response to disasters.
Exp) Option c is the correct answer. • Vision: To build a safer and disaster resilient India by
The President of India is elected indirectly by an electoral a holistic, proactive, technology driven and sustainable
college composed of the elected representatives (and not all development strategy that involves all stakeholders
members) from the Parliament of India and the Legislative and fosters a culture of prevention, preparedness and
Assemblies of the States, as well as the Union Territories of mitigation.

185 PYQ Workbook


INDIAN POLITY AND GOVERNANCE

• Divisions: NDMA has 5 major divisions viz. Policy & • But under the 7th Amendment Act, 1956, the same
Plans, Mitigation, Operations & Communications person can be appointed as Governor of one or more
& Information & Technology, Administration and States.
Finance. • Following are the powers and duties performed by
Governor in states, they are as follows:
121. Solution: (d)
(i) They have power to appoint Advocate General,
Exp) Option d is the correct answer.
(ii) They can summoning, proroguing and dissolving State
Article 75 of the Constitution states that the Prime Minister
Legislature,
has the authority to appoint ministers, including those who
(iii) They have power to grant pardons reprieves, respites
are not members of Parliament. However, non-member
or remission of punishments.
ministers are required to become members of either the
Lok Sabha or the Rajya Sabha within 6 months in order to 126. Solution: (d)
continue serving as ministers. Exp) : Option d is the correct answer
122. Solution: (a) • Article 213 empowers the Governor to promulgate
Ordinance, during recess of legislature.
Exp) Option a is the correct answer
The Vice-President of India is eligible for re-election. Important Tip
There is no restriction on the number of terms a Vice- • Governor of an Indian state draws ordinance making
President can serve. The Vice-President can be re-elected power from Article 213 of the Constitution.
for multiple terms if the person is chosen by the Electoral • The Governor can only issue ordinances when the
College again and meets the eligibility criteria. Legislative Assembly of a state or where there are two
houses in a state both houses are not in session.
123. Solution: (c)
• Governor’s ordinance-making power is not a
Exp) Option c is the correct answer discretionary power.
The President’s resignation is governed by Article 56 of the • This means that he can promulgate or withdraw an
Indian Constitution. According to this article, the President ordinance only on the advice of the council of ministers
can resign by writing a letter to the Vice President. headed by the chief minister.

124. Solution: (c) 127. Solution: (d)


Exp) Option c is the correct answer Exp) Option d is the correct answer
Article 176 states that at the commencement of the first • Ordinance making power of the governor is not a
session after each general election to the Legislative Assembly discretionary power, rather it comes under legislative
and at the commencement of the first session of each year, power of a governor (option 1 is incorrect)
the Governor shall address the Legislative Assembly or, in • The governor is himself competent to withdraw the
the case of a State having a Legislative Council, both Houses ordinance at any time. (option 2 is incorrect)
assembled together. This is also called as special address. 128. Solution: (b)
125. Solution: (a) Exp) Option b is the correct answer.
Exp) Option a is the correct answer • The duties of the Governor as a Constitutional Head of
the State do not become the subject matter of questions
• A governor possesses executive, legislative, financial, and
or debate in the Parliament, is not correct , As any
judicial powers
matter of utmost importance can be discussed in the
• However, he has no diplomatic, military or emergency parliament. (statement 1 is incorrect)
powers like the president • Parliament acts as a investigative body against all the
Important Tip decisions taken by the executive Legislature Ministry and
even the duties of governor.
• The Governor is the constitutional head of the State
Government. Important Tip
• He plays a two-fold function as the constitutional head • Article 153 to Article 162 of the Indian Constitution
of the State Government and as a link between the consists of the relevant provisions related to the office
Centre and the state government. of the Governor.
• The Governor of a State is appointed by the President • The Governor is the constitutional head of the State
by warrant under his hand and seal. Government
• Articles 153 says that three should be a Governor for • appointed by the President by warrant under his hand
each state. and seal.

PYQ Workbook 186


INDIAN POLITY AND GOVERNANCE

• Articles 153 says that three should be a Governor for • The Electoral College for electing the President of
each state. India consists of the following:
• under the 7th Amendment Act, 1956, the same person • All elected members of the Rajya Sabha
can be appointed as Governor of one or more States. • All elected members of the Lok Sabha
• Article 157: In order to be appointed as Governor, a • All elected members of the Legislative Assemblies
person: of States
• Must be a citizen of India; • All elected members of the Legislative Assemblies
• Must have competed the age of 35 years of Union Territories (except Jammu and Kashmir)
• In addition, three are two conventions that have come
131. Solution: (b)
to develop with regard to appointment of the Governor.
Exp) Option b is the correct answer.
• Must not belong to the state where he is appointed
and Consult the Chief Minister of the state where If any question arises regarding the disqualification of a
to be appointed member of Parliament, it shall be referred to the President
for decision. However, the President’s decision is not
129. Solution: (a) discretionary in this matter. The President is required to
Exp) Option a is the correct answer. obtain the opinion of the Election Commission and act
• The following Prime Ministers were defeated by a vote of according to that opinion. Therefore, this power does not
no confidence motion: come under the President’s discretion.
a) Morarji Desai:1978 132. Solution: (a)
b) Vishwanath Pratap Singh :1990 Exp) Option a is the correct answer.
c) HD Deve Gowda :1997 Statement 1 is correct- The governor may exercise
d) Atal Bihari Vajpayee :1999 discretionary powers, if he/she is not convinced by the advice
• No confidence motion: (Rule 198 Lok Sabha) of the Council of Ministers. For example, the Governor may
• Introduced only in Lok Sabha refuse to give assent to a bill passed by the state legislature if
he/she believes that the bill is unconstitutional or otherwise
• Supported by at least 50 members
harmful to the interests of the state.
• Debate
Statement 2 is correct- The Constitution of India specifies
• Not able to prove Majority Government has to resign certain matters in which the Governor has discretionary
(Simple Majority ) powers which includes appointing the Chief Minister of the
130. Solution: (c) state, approving the appointment of ministers, dissolving
the state legislature, and giving assent to bills passed by
Exp) Option c is the correct answer.
the state legislature
Statement 1 is correct- The Rajya Sabha, also known as
Statement 3 is correct- In cases where the Governor is
the Council of States, is the upper house of the Parliament
appointed as the administrator of an adjoining Union
of India. Elected members of the Rajya Sabha are part
of the Electoral College and have the right to vote in the Territory, they have the power to exercise discretionary
presidential election. powers in the administration of that territory.

Statement 2 is correct- All elected members of the 133. Solution: (b)


Legislative Assemblies of States are included in the Exp) Option b is the correct answer.
Electoral College. The Legislative Assembly of each state
The Attorney-General of India is the highest law officer of
in India consists of elected representatives who have the
authority to vote in the presidential election. the government and provides legal advice to the government.
The Attorney-General of India enjoys the right of audience
Statement 3 is incorrect- The Legislative Council, also
in not only the Supreme Court but also in any court
known as the Vidhan Parishad, is the upper house of
within the territory of India, including High Courts and
the state legislature in some states of India. However, the
subordinate courts.
members of the Legislative Councils are not part of the
Electoral College for electing the President of India. Important Tip
Important Tip Some other facts about Attorney-General of India:
Election of President of India: • The position of Attorney General is created by Article
• The election is held in accordance with the system 76 of the Indian Constitution.
of proportional representation by means of a single • The Attorney General is appointed by the President
transferable vote. Voting is done through Secret of India and holds office during the pleasure of the
ballot. President.

187 PYQ Workbook


INDIAN POLITY AND GOVERNANCE

December 1946 and served in that capacity until the


• To be eligible for the position, the person must be
qualified to be appointed as a judge of the Supreme Constituent Assembly was dissolved on 24 January 1950.
Court. Prasad was a prominent leader of the Indian independence
movement and a close associate of Mahatma Gandhi. He was
• The remuneration of the Attorney General is
also the first President of India from 1950 to 1962
determined by the President.
• The Attorney General can participate in the 137. Solution: (d)
proceedings of Parliament but does not have the right Exp) Option d is the correct answer
to vote.
The Attorney-General of India, as the highest law officer
• The Attorney General can be removed from office by
of the government under Article 76, is not exclusively
the President at any time.
engaged as a whole-time counsel for the government. The
134. Solution: (b) Attorney-General is free to engage in private legal practice
or pursue other professional responsibilities outside the
Exp) Option b is the correct answer.
realm of official duties.
According to Article 258(1) of the Indian Constitution,
the President of India has the power to entrust, with the Important Tips
consent of the Government of a State, certain functions to To avoid conflict of duty, there are a few limitations that
that Government or its officers in relation to any matter to are posted on the Attorney General which he should
which the executive power of the Union extends. keep in mind while performing his duties:
135. Solution: (c) • He should not advise or hold a brief against the
Exp) Option c is the correct answer. Government of India

Statement 1 is correct- According to Article 123, the • He should not advise or hold a brief in cases in
President has the power to promulgate an ordinance when which he is called upon to advise or appear for the
either one or both houses of Parliament are not in session. Government of India
These ordinances are temporary in nature and can be issued • He should not defend accused persons in criminal
to address urgent matters. prosecutions without the permission of the
Government of India
Statement 2 is correct- The approval of Parliament is
necessary for an ordinance to become a law. The ordinance • He should not accept appointment as a director in
must be laid before both houses of Parliament and must any company or corporation without the permission
be approved within six weeks from the reassembly of of the Government of India
Parliament. If not approved, the ordinance ceases to have
138. Solution: (b)
effect.
Exp) Option b is the correct answer
Important Tip
The Thakkar Commission was appointed in 1984 to
• Ordinances can only be introduced on subjects within investigate the assassination of Indira Gandhi, to find
the legislative competence of the Indian Parliament. lapses by the security and medical staff, and to determine
• The President’s ordinances have the same effect on possible involvement of foreign agencies.
policies as acts passed by Parliament.
139. Solution: (d)
• Article 213 deals with the power of the Governor to
legislate through ordinances. Exp) Option d is the correct answer
• The Governor’s ordinances have the same effect on The power of pardon is a discretionary power vested in
policies as state acts will have. the President, and it is independent of the judiciary. The
• If the Governor’s ordinance legislates on matters where President has the authority to exercise this power and grant
the state government has no power, the ordinance pardon even in cases where there is a judicial mandate or
becomes null and void. decision. The power of pardon is meant to provide relief,
• The power to promulgate ordinances by the President mercy, or mitigation in certain cases, and it is not restricted
and the Governor is not discretionary; it requires the by the judicial mandate.
advice of the Council of Ministers, headed by the
Important Tips
Chief Minister in the case of the state government
and Prime Minister in the case of Union government. • Under Article 72 of the Constitution, the President
shall have the power to grant pardons, reprieves,
136. Solution: (b) respites or remissions of punishment or to suspend,
Exp) Option b is the correct answer. remit or commute the sentence of any person
convicted of any offence where the sentence is a
Rajendra Prasad was the first President of the Constituent
sentence of death.
Assembly of India. He was elected as the President on 11

PYQ Workbook 188


INDIAN POLITY AND GOVERNANCE

143. Solution: (a)


• Limitation:
Exp) Option a is the correct answer.
• The President cannot exercise his power of pardon
independent of the government. Statement 1 is correct: The Preamble of the Constitution
of India is a brief introductory statement that sets out the
• In several cases, the Supreme Court has ruled that the
guiding principles and purpose of the Constitution. It is
President has to act on the advice of the Council of
based on the Objective Resolution, which was moved by
Ministers while deciding mercy pleas.
Jawaharlal Nehru in the Constituent Assembly on 13
• Although the President is bound by the Cabinet’s
December 1946 and adopted on 22 January 1947.
advice, Article74 (1) empowers him to return it for
reconsideration once. If the Council of Ministers Statement 2 is correct: The idea of the preamble was
decides against any change, the President has no borrowed from the Constitution of the USA.
option but to accept it. Statement 3 is correct: The Preamble declares India to be a
sovereign, socialist, secular, democratic republic and secures
140. Solution: (a) to all its citizens justice, liberty, equality and fraternity.
Exp) Option a is the correct answer. The words socialist and secular were added by the 42nd
Under Article 355 of the Constitution of India, it is the Amendment in 1976.
duty of the Union to protect every State against external Statement 4 is incorrect: While the Preamble of the
aggression and internal disturbance. This means that the Constitution of India sets out the ideals and objectives
Union government is responsible for ensuring the security of the Constitution, it does not explicitly mention the
and integrity of the States by defending them against any Fundamental Rights. The Fundamental Rights are enshrined
external threats or acts of aggression. in Part III of the Constitution.

141. Solution: (d) 144. Solution: (b)


Exp) Option d is the correct answer. Exp) Option b is the correct answer.

The 2nd Administrative Reforms Commission (ARC) in Statement 1 is correct: The President of India is elected
India has produced Reports on various subjects aimed at indirectly, which means that the citizens of India do not
improving governance and administrative processes. The directly vote for the President. Instead, the President is
Reports produced by the ARC are: elected by an electoral college.
Statement 2 is correct: The electoral college for the
• Ethics in Governance: The ARC has addressed the issue
Presidential election comprises all elected Members of
of ethics in governance, emphasizing the need for ethical
Parliament (MPs) from both the Lok Sabha and the Rajya
conduct and integrity in public administration.
Sabha, as well as Members of Legislative Assemblies
• Local Governance: The ARC has focused on the issue of (MLAs) from all the states and Union territories. These
local governance, recommending reforms to strengthen members collectively vote to elect the President of India.
decentralization and empower local bodies for effective
Statement 3 is incorrect: President of India is elected
service delivery and citizen participation.
indirectly, not through a direct vote by the citizens.
• Combating Terrorism: The ARC has examined the
Statement 4 is correct: Members of the Legislative Council,
challenges posed by terrorism and made recommendations
which is the upper house of the state legislature in certain
to enhance the capacity of security agencies, improve
states, do not participate in the Presidential election. Only
intelligence sharing, and strengthen counter-terrorism Members of Parliament (MPs) and Members of Legislative
measures. Assemblies (MLAs) have the right to vote in the Presidential
The 2nd ARC did not produce a report on eradication of election.
corruption specifically, though it touched upon the issue
in some of its reports such as ethics in governance and 145. Solution: (a)
refurbishing personnel administration. Exp) Option a is the correct answer.
Statement 1 is correct: According to Article 153 of the
142. Solution: (a)
Constitution of India, there can be a common Governor
Exp) Option a is the correct answer. for two or more states. This provision allows for the
According to Article 53 of the Indian Constitution, the appointment of a Governor who can act as the Governor of
President of India is the Supreme Commander of the multiple states.
Indian Armed Forces. The President holds the highest rank Statement 2 is correct: The Governor has the power to
and authority over the military forces of India, including the grant pardons, reprieves, and remission of punishment
Indian Army, Indian Navy, and Indian Air Force. to persons convicted under the State Law. This power is
The President exercises the powers and functions as the mentioned in Article 161 of the Constitution of India.
Supreme Commander in consultation with the Council of Statement 3 is incorrect: The Governor is not appointed
Ministers, headed by the Prime Minister. by the President on the recommendation of the Chief

189 PYQ Workbook


INDIAN POLITY AND GOVERNANCE

Minister. The Governor is appointed by the President of procedures of the government and defines its relationship
India, and the appointment is made by the President based with the people. The constitution also lays down the
on the advice of the Union Council of Ministers. The Chief principles and values that guide the actions and decisions of
Minister of the respective state does not have a role in the the government and its officials.
appointment of the Governor. Statement 3 is incorrect: A constitutional government does
146. Solution: (a) not necessarily have a monarch as the formal head of the
state. A constitutional government can have different forms
Exp) Option a is the correct answer.
of executive leadership, such as a president, a prime minister,
Statement 1 is correct: Constitutional government is a form or a chancellor.
of limited government. The constitution sets the boundaries
Statement 4 is incorrect: A constitutional government does
and limits of the powers of the government and prevents it
not always have a universal adult franchise. A constitutional
from becoming arbitrary or tyrannical. The constitution also
government can have different forms of electoral systems,
ensures that the government is accountable to the people and
such as proportional representation, first-past-the-post, or
respects their rights and freedoms.
mixed-member proportional. A universal adult franchise is
Statement 2 is correct: A constitutional government is one one of the possible ways of ensuring political participation
where the constitution is the basis of public authority. and representation in a constitutional democracy, but it is
The constitution establishes the structure, functions, and not a necessary condition for a constitutional government.

PYQ Workbook 190


INDIAN POLITY AND GOVERNANCE

INDIAN POLITY AND GOVERNANCE


UNION AND STATE LEGISLATURE
*This unit consists of questions from Union Legislature and State Legislatures.

6.1. UPSC CSE Previous Years’ Questions of Lok Sabha shall be from either the
principal opposition party or the ruling
1. With reference to Finance Bill and Money party.
Bill in the Indian Parliament, consider the 3. The Deputy Speaker has the same power
following statements: as of the Speaker when presiding over the
1. When the Lok Sabha transmits Finance sitting of the House and no appeal lies
Bill to the Rajya Sabha, it can amend or against his rulings.
reject the Bill. 4. The well-established parliamentary
2. When the Lok Sabha transmits Money practice regarding the appointment of
Bill to the Rajya Sabha, it cannot amend Deputy Speaker is that the motion is
or reject the Bill, it can only make moved by the Speaker and duly seconded
recommendations by the Prime Minister.
3. In the case of disagreement between the Which of the statements given above are
Lok Sabha and the Rajya Sabha, there is correct?
no joint sitting for money bill, but joint (a) 1 and 3 only
sitting becomes necessary for Finance (b) 1, 2 and 3
bill. (c) 3 and 4 only
How many of the above, statements are (d) 2 and 4 only
correct? [UPSC CSE Pre Pre 2023]
4. With reference to anti-defection law in
(a) Only one
India, consider the following statements:
(b) Only two
(c) All three 1. The law specifies that a nominated
(d) None legislator cannot join any political party
within six months of being appointed to
2. Which of the following is/are the exclusive the House.
power(s) of Lok Sabha? 2. The law does not provide any time-frame
[UPSC CSE Pre. 2022] within which the presiding officer has to
1. To ratify the declaration of Emergency decide a defection case.
2. To pass a motion of no-confidence against Which of the statements given above is/are
the Council of Ministers correct? [UPSC CSE Pre Pre 2022]
3. To impeach the President of India (a) 1 only
Select the correct answer using the code given (b) 2 only
below: (c) Both 1 and 2
(a) 1 and 2 (d) Neither 1 nor 2
(b) 2 only 5. Consider the following statements:
(c) 1 and 3
1. A bill amending the Constitution requires
(d) 3 only
a prior recommendation of the Président
3. With reference to Deputy Speaker of Lok of India.
Sabha, consider the following statements: 2. When a Constitution Amendment Bill is
[UPSC CSE Pre. 2022] presented to the President of India, it is
1. As per the Rules of Procedure and obligatory for the President of India to
Conduct of Business in Lok Sabha, the give his/her assent.
election of Deputy Speaker shall be held 3. A Constitution Amendment Bill must
on such date as the Speaker may fix. be passed by both the Lok Sabha and the
2. There is a mandatory provision that the Rajya Sabha by a special majority and
election of a candidate, as Deputy Speaker there is no provision for joint sitting.

191 PYQ Workbook


INDIAN POLITY AND GOVERNANCE

Which of the statements given above are 1. In the first Lok Sabha, the single largest
correct? [UPSC CSE Pre 2022] party in the opposition was the Swatantra
(a) 1 and 2 only Party
(b) 2 and 3 only 2. In the Lok Sabha, a “Leader of the
(c) 1 and 3 only Opposition” was recognised for the first
(d) 1, 2 and 3 time in 1969.
6. We adopted parliamentary democracy 3. In the Lok Sabha, if a party does not have
based on the British model, but how does a minimum of 75 members, its leader
our model differ from that model? cannot be recognised as the Leader of the
1. As regards legislation, the British Opposition.
Parliament is supreme or sovereign but Which of the statements given above is/are
in India, the power of the Parliament to correct?
legislate is limited. (a) 1 and 3 only
2. In India, matters related to the (b) 2 only
constitutionality of Amendment of an (c) 2 and 3 only
Act of the Parliament are referred to (d) 1, 2 and 3
the Constitution Bench by the Supreme 10. Regarding Money Bill, which of the
Court. following statements is not correct?
Select the correct answer using the code given [UPSC CSE Pre. 2018]
below. [UPSC CSE Pre 2021] (a) A bill shall be deemed to be a Money
(a) 1 only Bill if it contains only provisions relating
(b) 2 only to imposition, abolition, remission,
(c) Both 1 and 2 alteration or regulation of any tax.
(d) Neither 1 nor 2 (b) A Money Bill has provisions for the
7. Rajya Sabha has equal powers with Lok custody of the Consolidated Fund of
Sabha in [UPSC CSE Pre. 2020] India or the Contingency Fund of India.
(a) the matter of creating new All India (c) A Money Bill is concerned with the
Services appropriation of moneys out of the
(b) amending the Constitution Contingency Fund of India.
(c) the removal of the government (d) A Money Bill deals with the regulation
(d) making cut motions of borrowing of money or giving of any
guarantee by the Government of India.
8. In India, which of the following review
the independent regulators in sectors like 11. Consider the following statements:
telecommunications, insurance, electricity, [UPSC CSE Pre. 2018]
etc.? [UPSC CSE Pre. 2019] 1. The Speaker of the Legislative Assembly
1. Ad Hoc Committee set up by the shall vacate his/her office if he/she ceases
Parliament. to be a member of the assembly.
2. Parliamentary Department Related 2. Whenever the Legislative Assembly is
Standing Committee dissolved, the Speaker shall vacate his/her
3. Finance Commission office immediately.
4. Financial Sector Legislative Reforms Which of the statements given above is/are
Commission correct?
5. NITI Aayog (a) 1 only
(b) 2 only
Select the correct answer using the code given
(c) Both 1 and 2
below.
(d) Neither 1 nor 2
(a) 1 and 2 only
(b) 1, 3 and 4 only 12. With reference to the Parliament of India,
(c) 3, 4 and 5 only which of the following Parliamentary
(d) 2 and 5 only Committees scrutinizes and reports to
the House whether the powers to make
9. Consider the following statements: regulations, rules, sub-rules, by-laws, etc.
[UPSC CSE Pre. 2018] conferred by the Constitution or delegated

PYQ Workbook 192


INDIAN POLITY AND GOVERNANCE

by the Parliament are being properly (a) 1 only


exercised by the Executive and is within the (b) 2 only
scope of such delegation? (c) Both 1 and 2
[UPSC CSE Pre. 2018] (d) Neither 1 nor 2
(a) Committee on Government Assurances 16. The Parliament of India acquires the power
(b) Committee on Subordinate Legislation to legislate on any item in the State List in
(c) Rules Committee
the national interest if a resolution to that
(d) Business Advisory Committee
effect is passed by the
13. Out of the following statements, choose the [UPSC CSE Pre. 2016]
one that brings out the principle underlying (a) Lok Sabha by a simple majority of its total
the Cabinet form of Government: membership
[UPSC CSE Pre. 2017] (b) Lok Sabha by a majority of not less than
(a) An arrangement for minimizing the two-thirds of its total membership
criticism against the Government whose (c) Rajya Sabha by a simple majority of its
responsibilities are complex and hard to total membership
carry out the satisfaction of all. (d) Rajya Sabha by a majority of not less than
(b) A mechanism for speeding up the activities two-thirds of its members present and
of the Government whose responsibilities are voting
increasing day by day.
(c) A mechanism of parliamentary democracy 17. Consider the following statements:
for ensuring collective responsibility of [UPSC CSE Pre. 2015]
the Government to the people. 1. The Legislative Council of a State in
(d) A device for strengthening the hands of India can be larger in size than half of the
the head of the Government whose hold Legislative Assembly of that particular
over the people is in a state of decline. State.
14. Consider the following statements: 2. The Governor of a State nominates the
[UPSC CSE Pre. 2017] Chairman of Legislative Council of that
1. In the election for Lok Sabha or State particular State.
Assembly, the winning candidate must Which of the statements given above is/are
get at least 50 percent of the votes polled, correct?
to be declared elected. (a) 1 only
2. According to the provisions laid down in (b) 2 only
the Constitution of India, in Lok Sabha, (c) Both 1 and 2
the Speaker’s post goes to the majority (d) Neither 1 nor 2
party and the Deputy Speaker’s to the
Opposition. 18. With reference to the Union Government,
Which of the statements given above is/are consider the following statements:
correct? [UPSC CSE Pre. 2015]
(a) 1 only 1. The Department of Revenue is responsible
(b) 2 only for the preparation of Union Budget that
(c) Both 1 and 2 is presented to the Parliament.
(d) Neither 1 nor 2 2. No amount can be withdrawn from the
15. With reference to the Parliament of India, Consolidated Fund of India without the
consider the following statements: authorization from the Parliament of
[UPSC CSE Pre. 2017] India.
1. A private member’s bill is a bill presented 3. All the disbursements made from Public
by a Member of Parliament who is Account also need the authorization from
not elected but only nominated by the the Parliament of India.
President of India. Which of the statements given above is/are
2. Recently, a private member’s bill has been correct?
passed in the Parliament of India for the (a) 1 and 2 only
first time in its history. (b) 2 and 3 only
Which of the statements given above is/are (c) 2 only
correct? (d) 1, 2 and 3

193 PYQ Workbook


INDIAN POLITY AND GOVERNANCE

19. Consider the following statements: A 23. When the annual Union Budget is not
Constitutional Government is one which passed by the LokSabha
[UPSC CSE Pre. 2014] [UPSC CSE Pre. 2012]
1. Places effective restrictions on individual (a) the Budget is modified and presented
liberty in the interest of State Authority. again
2. Places effective restrictions on the (b) the Budget is referred to the RajyaSabha
Authority of the State in the interest of for suggestions
individual liberty. (c) the Union Finance Minister is asked to
resign
Which of the statements given above is/are
(d) the Prime Minister submits the
correct?
resignation of Council of Ministers
(a) 1 only
(b) 2 only 24. A deadlock between the LokSabha and the
(c) Both 1 and 2 RajyaSabha calls for a joint sitting of the
(d) Neither 1 nor 2 Parliament during the passage of
[UPSC CSE Pre. 2011]
20. Which one of the following is the largest
1. Ordinary Legislation
Committee of the Parliament?
2. Money Bill
[UPSC CSE Pre. 2014]
3. Constitution Amendment Bill
(a) The committee on Public Accounts
(b) The committee on Estimates Select the correct answer using the codes
(c) The committee on Public Undertakings given below:
(d) The committee on Petitions (a) 1 only
(b) 2 and 3 only
21. Consider the following statements (c) 1 and 3 only
regarding a No-Confidence Motion in (d) 1, 2 and 3
India: [UPSC CSE Pre. 2014]
1. There is no mention of a No-Confidence 25. Which of the following is responsible for
Motion in the Constitution of India. preparation and presentation of union
budget in the Parliament?
2. A motion of No-Confidence can be
introduced in the Lok Sabha only. [UPSC CSE Pre 2010]
(a) Department of Revenue
Which of the statements given above is/are
(b) Department of Economic Affairs
correct? (c) Department of Financial Services
(a) 1 only (d) Department of Expenditure
(b) 2 only
(c) Both 1 and 2 26. Which one of the following statements is not
(d) Neither 1 nor 2 correct with regard to control of Parliament
on the budget? [UPSC CSE Pre 2009]
22. Consider the following statements: (a) Parliament does not have any role in the
[UPSC CSE Pre. 2013] creation of the budget.
1. The Council of Ministers in the Centre (b) Parliament has the power to vote on the
shall be collectively responsible to the charged expenditure on the Consolidated
Parliament. Fund.
2. The Union Ministers shall hold the office (c) Parliament has no power to impose a
during the pleasure of the President of tax without the recommendation of the
India. President.
3. The Prime Minister shall communicate (d) Parliament has no power to increase any
to the President about the proposals for tax without the recommendation of the
legislation. President.
Which of the statements given above is/are 27. In India Lok Sabha Speaker is-
correct? [UPSC CSE Pre 2009]
(a) 1 only (a) Nominated
(b) 2 and 3 only (b) Chosen
(c) 1 and 3 only (c) Elected
(d) 1, 2 and 3 (d) Appointed

PYQ Workbook 194


INDIAN POLITY AND GOVERNANCE

28. Consider the following statements: (c) 7th and 31st


The Constitution of India provides that- (d) 11th and 42nd
1. The Legislative Assembly of each State 32. Consider the following statements:
shall consist of not more than 450 1. While members of the Rajya Sabha are
members chosen by direct election from associated with Committees on Public
territorial constituencies in the State. Accounts and Public Undertakings,
2. A person shall not be qualified to be members of Committee on Estimates are
chosen to fill a seat in the Legislative drawn entirely from Lok Sabha.
Assembly of a State if he/she is less than 2. The Ministry of Parliamentary Affairs
25 years of age. works under the overall direction of the
Which of the statements given above is/are Cabinet Committee on Parliamentary
correct? UPSC CSE Pre 2008] Affairs.
(a) 1 only 3. The Minister of Parliamentary Affairs
(b) 2 only nominates members of Parliament
(c) Both 1 and 2 on Committees, Councils, Boards,
(d) Neither nor 2 and Commissions, etc. set up by
29. Consider the following statements: the Government of India in various
1. The Chairman of the Committee on ministries.
Public Accounts is appointed by the Which of these statements are correct?
Speaker of the Lok Sabha. [UPSC CSE Pre 2007 2003]
2. The Committee on Public Accounts (a) 1 and 2
comprises Members of Lok Sabha, (b) 2 and 3
Members of Rajya Sabha and a few (c) 1 and 3
eminent persons of industry and trade. (d) 1, 2 and 3
Which of the statement(s) given above is/are 33. The term of the Lok Sabha:
correct? [UPSC CSE Pre 2007] [UPSC CSE Pre 2002]
(a) 1 only (a) Cannot be extended under any
(b) 2 only circumstances
(c) Both 1 and 2 (b) Can be extended by six months at a time
(d) Neither 1 nor 2 (c) Can be extended by one year at a time
30. Consider the following statements: during the proclamation of emergency
1. The joint sitting of both Houses of the (d) Can be extended for two years at a time
Parliament in India is sanctioned under during the proclamation of emergency
Article 108 of the Constitution. 34. Which one of the following statements
2. The first joint sitting of Lok Sabha and correctly describes the Fourth Schedule of
Rajya Sabha was held in the year 1961. the Constitution of India?
3. The second joint sitting of both Houses [UPSC CSE Pre 2001]
of Indian Parliament was held to pass the (a) It lists the distribution of powers between
Banking Service Commission (Repeal), the Union and the states.
Bill. (b) It contains the languages listed in the
Which of these statements is/are correct? Constitution.
[UPSC CSE Pre 2003] (c) It contains provisions for the
(a) 1 and 2 administration of Tribal areas.
(b) 2 and 3 (d) It allocates seats in the Council of States.
(c) 1 and 3 35. In what way does the Indian Parliament
(d) 1, 2 and 3 exercise control over the administration?
31. Which of the following Constitutional [UPSC CSE Pre 2001]
Amendments is related to raising the (a) Through Parliamentary Committees
number of Members of Lok Sabha to be (b) Through Advisory Committees of various
elected from the States? ministries
[UPSC CSE Pre 2003] (c) By making the administrators send
(a) 6th and 22nd periodic reports
(b) 13th and 38th (d) By compelling the executive to issue writs

195 PYQ Workbook


INDIAN POLITY AND GOVERNANCE

36. The Speaker can ask a member of the House (c) He loses his office if the house is dissolved
to stop speaking and let another member before the end of its normal tenure.
speak. This phenomenon is known as: (d) If he intends to resign the letter of his
[UPSC CSE Pre 2000] resignation is to be addressed to the
(a) Decorum Deputy Speaker.
(b) Crossing the floor
(c) Interpellation 6.2. Other Examination Previous Years’
(d) Yielding the floor Questions

37. Which of the following are/is stated in the 41. Which one of the following pairs is NOT
Constitution of India? correctly matched? [U.P.P.C.S. (Pre) 2022]
1. The President shall not be a member of State Members in the
either House of Parliament. Legislative Assembly
2. The Parliament shall consist of the (a) Goa 40
President and the two Houses. (b) Uttarakhand 70
Choose the correct answer from the codes (c) Manipur 62
given below: [UPSC CSE Pre 1997]
(d) Uttar Pradesh 403
(a) Neither 1 nor 2
(b) Both 1 and 2 42. The Public Accounts Committee of Indian
(c) 1 only Parliament scrutinizes the-
(d) 2 only [U.P. P.C.S. (Pre) 2021]
38. Which one of the following States of India (a) Report of the comptroller and auditor
does not have a Legislative Council so for general
even though the Constitution (Seventh (b) Consolidated fund of India
Amendment) Act, 1956 provides for it? (c) Public Account of India
[UPSC CSE Pre 1995] (d) Contingency fund of India
(a) Maharashtra 43. Which of the following Articles of the Indian
(b) Bihar Constitution is related to Contingency
(c) Karnataka Fund? [U.P.P.C.S. (Pre) 2021]
(d) Madhya Pradesh
(a) Article 265
39. Who among the following has the right to (b) Article 266
vote in the elections to both the Lok Sabha (c) Article 267
and the Rajya Sabha? [UPSC CSE Pre 1995] (d) Article 268
(a) Elected members of the Lower House of
44. Match list-I with list-II and select the
the Parliament
correct answer using the codes given below
(b) Elected members of the Upper House of
the lists:
the Parliament
(c) Elected members of the Upper House of List -I List-II
the state Legislature A. Indian Arms Act 1. 1876
(d) Elected members of the Lower House of
B. Royal Title Act 2. 1878
the state Legislature
C. Indian High Court 3. 1869
40. Which one of the following statements
D. Indian Divorce 4. 1861
regarding the office of the Speaker is Act
correct? [UPSC CSE Pre 1993]
Code: [UPPCS (Pre) 2020]
(a) He holds office during the pleasure of the
President. A B C D
(b) He need not be a member of the House at (a) 2 3 1 4
the time of his election but has to become (b) 3 1 2 4
a member of the House within six months (c) 1 2 3 4
from the date of his election. (d) 2 1 4 3

PYQ Workbook 196


INDIAN POLITY AND GOVERNANCE

45. Mazdoor Kisan Shakti Sangathan (MKSS), 51. A Bill which merely involves expenditure
a Social Movement has been associated with and does not include any of the matters
which of the following in India? specified in Article 110 can be:
[U.P.P.C.S. (Pre) 2019] [U.P.P.C.S. (Mains) 2016]
(a) RTE Act (a) Initiated only in Lok Sabha
(b) RTI Act (b) Initiated in either House of Parliament
(c) MGNREGA (c) Initiated only in Rajya Sabha
(d) National Health Mission (NHM) (d) Initiated only in joint session of both
House of Parliament
46. The Council of States has exclusive powers
in relation to which of the following? 52. In case the Deputy Speaker is presiding the
[U.P.P.C.S. (Pre) 2018] Lok Sabha he has right to
(a) To initiate proceedings for the removal of [U.P.P.C.S. (Mains) 2016]
the President (a) Take part in its deliberations
(b) To initiate proceedings for the removal of (b) Vote on any proposition before the house
a judges of the Supreme Court as a member
(c) To recommend creation of a new All (c) Vote in the event of equality of votes
India Service (d) No right to preside a joint sitting of the
(d) None of the above House of Parliament in case the Speaker
is absent
47. What can be the maximum representation
from the Union Territories to the Lok 53. Who among the following is the final
Sabha? [U.P.P.C.S. (Pre) 2018] authority to decide any question relating to
(a) 15 disqualification of a Member of a House of
(b) 20 Legislature of a State?
(c) 25 [U.P.P.C.S. (Mains) 2016]
(d) None of the above (a) Governor
48. After how many days of absence from (b) Speaker of the Legislative Assembly
Parliament without permission can an M.P. (c) Chief Minister
be disqualified? [U.P.P.C.S. (Pre) 2018] (d) High Court
(a) 30 days 54. Which one of the following Articles of the
(b) 60 days Constitution provides that the Speaker
(c) 90 days shall have and exercise a casting vote in the
(d) 120 days case of an equality of votes?
[U.P.P.C.S. (Pre) (Re-Exam) 2015]
49. What is the quorum to constitute a meeting
of Lok Sabha? [U.P.P.C.S. (Pre) 2017] (a) Article 99
(b) Article 103
(a) One sixth of the total members of the
(c) Article 100
House.
(d) Article 102
(b) One tenth of the total members of the
House. 55. Who can legislate on the subjects which are
(c) One fourth of the total members of the not included in any of the lists of subjects
House given in the Constitution?
(d) One third of the total members of the [U.P.P.C.S. (Pre) (Re-Exam) 2015]
House (a) Supreme Court
50. Which one of the following is not a collective (b) Parliament
privilege of the members of Parliament? (c) State Legislature
[U.P.P.C.S. (Pre) 2017] (d) Regional Councils
(a) Freedom of debates and proceedings. 56. The Union Territories get representation
(b) The Rights to regulate the internal matters in- [U.P.P.C.S. (Mains) 2015]
of the Parliament (a) Lok Sabha
(c) Freedom from attendance as Witness. (b) Rajya Sabha
(d) The privileges of excluding strangers (c) Both Houses of Parliament
from the house. (d) None of the above

197 PYQ Workbook


INDIAN POLITY AND GOVERNANCE

57. Which one of the following is authorized 63. No money Bill can be introduced in Assembly
to prorogue the session of the House of the of a State, except on the recommendations
People? [U.P.P.C.S. (Mains) 2015] of- [U.P.P.C.S. (Pre) 2012]
(a) Speaker (a) The Parliament
(b) Attorney General (b) The Governor of the State
(c) Minister of Parliamentary (c) The President of India
(d) President (d) A special Committee of Ministers
58. Which of the following is related to the 64. Which one of the following is not correct
Right to Information Act, 2005? with regard to the elections to the Lok
[U.P.P.C.S. (Pre) (Re-Exam) 2015] Sabha? [U.P.P.C.S. (Mains) 2011]
(a) Lily Thomas v/s Union of India (a) The elections are on the basis of adult
(b) Nandini Sundar v/s State of Chhattisgarh suffrage.
(c) Namit Sharma v/s Union of India (b) The Election Commission is required to
(d) None of the above conduct the elections.
(c) The electoral rolls are prepared under the
59. Which of the following are correct about direction and control of the President of
the Rajya Sabha? India.
1. It is not subject to dissolution. (d) On the request of the Election Commission,
2. It has a term of five years. the President is required to make available
3. One-third of its members retire after to the Election Commission such staff as
every two years. may be necessary for the discharge of its
4. Its members shall not be less than 25 functions.
years of age. 65. Which one of the following terms does not
Select the correct answer using the code given appear in the Constitution of India?
below: [U.P.P.C.S. (Pre) 2014] [U.P.P.C.S. (Mains) 2011]
(a) 1, 2 and 3 (a) Annual Financial Statement
(b) 2, 3 and 4 (b) Appropriation Bill
(c) 1 and 3 (c) Budget
(d) 2 and 4 (d) Consolidated Fund of India
60. Who among the following presides over a 66. Which one of the following statements
joint sitting of two houses of Parliament? about Rajya Sabha is correct?
[U.P.P.C.S. (Pre) 2014] [U.P.P.C.S. (Mains) 2011]
(a) President (a) It is not subject to dissolution.
(b) Prime Minister (b) Its members are chosen by direct election
(c) Chairman of the Rajya Sabha from territorial constituencies in the
(d) Speaker of the Lok Sabha States.
(c) It is required to choose, as soon as may
61. Parliament may legislate on any subject of be a member of the Rajya Sabha to be the
State List when- [U.P.P.C.S. (Pre) 2012] Chairman thereof.
(a) There is assent of President (d) The Attorney General of India does not
(b) Resolution thereof has been passed by have the right to speak in Rajya Sabha.
State
(c) In any situation 67. Which one of the following expenditures is
(d) General permission of State Legislative not charged on the Consolidated Fund of
Assembly has been received India? [U.P.P.C.S. (Mains) 2011]
(a) Salary and allowances of the Chief Justice
62. On which of the following, a Parliamentary of India
Committee has not been set up? (b) Salary and allowances of the Comptroller
[U.P.P.C.S. (Mains) 2012] and Auditor General of India
(a) Public Undertakings (c) Salary and allowances of the Prime
(b) Government Assurances Minister of India.
(c) Estimates (d) Salary and allowances of the Chairman of
(d) Welfare of Minorities the Union Public Service Commission.

PYQ Workbook 198


INDIAN POLITY AND GOVERNANCE

68. Which one of the following is not covered 74. Union Parliament may legislate on any
by the Anti-Defection Law? subject of State List-
[U.P.P.C.S. (Mains) 2010] [U.P.P.C.S. (Mains) 2005]
(a) An-bloc defections in a Party. 1. To Implement International Agreements
(b) Large defections in a Party in various 2. By consent of State
spells. 3. When Proclamation of Emergency is in
(c) Smaller defection in a single spell. operation.
(d) Party members in Parliament casting vote 4. In national interest and after a resolution
either to topple or support a government. passed by Council of states with 2/3
69. Which one of the following majority.
pairs is not correctly Choose the correct option using codes given
matched? [U.P.P.C.S. (Mains) 2009] below:
(a) Hindu Marriage Act: 1955 (a) 1, 2 and 3
(b) Hindu Succession Act: 1956 (b) 2, 3 and 4
(c) 73rd Constitutional Amendment Act: (c) 1, 2 and 4
Reservation of seats for women in the (d) All of the above
election of local bodies in urban areas
(d) Sati (Prevention) Act: 1987 75. Match list-I with list-II and select the
correct answer using the codes given below
70. Which is the only State in India to have the the lists:
Common Civil Code?
[U.P.P.C.S. (Mains) 2009] List -I List-II
(a) Jammu & Kashmir A. Public Accounts 1. Ad hoc
(b) Mizoram Committee Committee
(c) Nagaland B. Committee on 2. Standing
(d) Goa Petitions Committee
71. Members of the Rajya Sabha are not C. Joint Committee 3. Financial
associated with- on Stock Market Committee
[U.P.P.C.S. (Spl) (Mains) 2008] Scam
1. Public Accounts Committee D. Departmental 4. Functional
2. Estimates Committee Committees Committee
3. Committee on Public Undertakings Code: [U.P.P.C.S. (Mains) 2005]
Choose your answer from the given code. A B C D
(a) 1 and 2 (a) 1 4 3 2
(b) Only 3 (b) 2 3 4 1
(c) Only 2 (c) 3 4 1 2
(d) 1 and 3 (d) 4 2 1 3
72. Which of the Constitutional Provision lays 76. Which one of the following States has the
down that taxes can neither be levied nor highest number of reserve seats for the
collected without the authority of law? Scheduled Tribes in the Lok Sabha?
[U.P.P.C.S. (Mains) 2007] [U.P.P.C.S. (Mains) 2005]
(a) Article 265 (a) Bihar
(b) Article 266 (b) Uttar Pradesh
(c) Article 300 (c) Madhya Pradesh
(d) Article 368 (d) Gujarat
73. At which of the following stages general 77. Which one of the following is the period
discussion on a Bill takes place in the Lok for the Legislative Council to detain the
Sabha? [U.P.P.C.S. (Mains) 2006] ordinary bills? [U.P.P.C.S. (Mains) 2005]
(a) During introduction of the Bill (a) 3 months
(b) During second reading (b) 4 months
(c) During report stage (c) 6 months
(d) During third reading (d) 14 days

199 PYQ Workbook


INDIAN POLITY AND GOVERNANCE

78. Which of the following are exclusive powers 84. Which of the following is not a Social Act?
of Lok Sabha in comparison to the Rajya [U.P.P.C.S. (Pre) 1991]
Sabha? (a) MISA Act
1. A Money/Finance Bill can be introduced (b) Anti-Dowry Act
only in the House of the People. (c) Protection of Civil Rights Act
2. Regarding the rejection or amendment of (d) Prevention of Immoral Traffic Act
the Money Bill.
3. The accountability of the Council of 85. How can the President spend from
Ministers. Contingency Fund? [U.P.P.C.S. (Pre) 1991]
4. Concerning the creation of one or more (a) In time of natural calamity
All India Services. (b) After Authorization of parliament.
(c) Before Authorization of Parliament.
Select the correct answer from the code given
(d) Cannot spend.
below: [U.P.P.C.S. (Spl) (Mains) 2004]
(a) 1, 2, 3, 4 86. The members of Council of States are
(b) 1, 2, 3 elected by- [U.P.P.C.S. (Pre) 1990]
(c) 1, 3, 4 (a) Members of Legislative Assembly of
(d) 3, 4 States.
79. Economic Survey is presented in Parliament (b) Members of Legislative Council of States.
every year- [U.P.P.C.S. (Mains) 2004] (c) Members of both Legislative Assembly
(a) Before presentation of the Budget for the and Council.
coming year (d) Members of Municipalities and Gram
(b) After presentation of the Budget for the Panchayat.
coming year 87. Which of the following is a Parliamentary
(c) After presentation of Finance Bill Committee relating to delegation of power
(d) And has no relation with presentation of to make rules and regulations to the
the Budget Executive? [67th B.P.S.C. (Pre), 2021]
80. ‘Zero Hour’ is the contribution of which (a) Committee on Executive Legislation
country to the Parliamentary System? (b) Committee on Subordinate Legislation
[U.P.P.C.S. (Spl) (Mains) 2004] (c) Committee on Administrative Legislation
(a) India (d) Committee on Delegated Legislation
(b) America (e) None of the above/More than one of the
(c) Britain above
(d) Switzerland 88. What constitutes the definition of money
81. The sitting of House of the People may the bill under article 110 of the Indian
terminated by: [U.P.P.C.S. (Pre) 2000] Constitution?
(a) Adjournment sine die [66th B.P.S.C. (Pre) (Re-Exam), 2020]
(b) Prorogation (a) The imposition, abolition, alteration or
(c) Dissolution regulation of any tax
(d) All of the above (b) The regulation of borrowings of money
82. Who has the right to issue money from the (c) The appropriation of money out of the
Consolidated Fund of India? consolidated fund of India
[U.P.P.C.S. (Pre) 2000] (d) Declaration of any expenditure to be
(a) Comptroller and Auditor General charged on the consolidated fund of India
(b) Finance Minister of India (e) None of the above / More than one of the
(c) Authorized Minister above
(d) None of the above 89. In which Article of the Constitution of
83. Reservation of seats for Scheduled Tribes in India was the provision for reservation of
the Lok Sabha does not exist in the states of: scheduled castes in the Lok Sabha made?
[U.P.P.C.S. (Pre) 1998] [64th B.P.C.S. (Pre) 2018]
(a) Kerala and Tamil Nadu (a) Article 330
(b) Karnataka and Kerala (b) Article 331
(c) Tamil Nadu and Karnataka (c) Article 332
(d) All of the above (d) Article 333

PYQ Workbook 200


INDIAN POLITY AND GOVERNANCE

90. The Rajya Sabha consists of- 96. If the Speaker of Legislative Assembly
[53rd to 55th B.P.S.C. (Pre) 2011] of a State wants to resign, he will give his
(a) 280 members of which 20 members are resignation to- [40th B.P.S.C. (Pre) 1995]
nominated by the President of India. (a) Chief Minister
(b) 275 members of which 18 members are (b) Governor
nominated by the President of India. (c) Deputy Speaker
(c) 250 members of which 12 members are (d) President of India
nominated by the President of India. 97. Which of the following Articles of Indian
(d) 252 members of which 12 members are Constitution consists of the provision of the
nominated by the President of India. election of Legislative Assemblies of States-
91. The Public Accounts Committee submits [40th B.P.S.C. (Pre) 1995]
its reports to- (a) Article 170
[53rd to 55th B.P.S.C. (Pre) 2011] (b) Article 176
(a) The Comptroller and Auditor General (c) Article 178
(b) The Speaker of the Lok Sabha (d) None of the above
(c) The Minister of Parliamentary Affairs 98. Who is the leader of the Lok Sabha?
(d) The President of India
[42nd B.P.S.C. (Pre) 1995]
92. What is the minimum age laid down for a (a) President
person to seek election to the Lok Sabha? (b) Prime Minister
[45th B.P.S.C. (Pre) 2001] (c) Speaker
(a) 18 years (d) None of the above
(b) 21 years 99. Joint Session of both the Houses of
(c) 25 years Parliament is concerned-
(d) None of these
[40th B.P.S.C. (Pre) 1995]
93. Before completing its term, the Lok Sabha 1. For the election of the President of India
can be dissolved-[42nd B.P.C.S. (Pre) 1997] 2. For the election of the vice-President of
(a) By the discretion of President India
(b) By the discretion of Prime Minister 3. To pass the Bill relating to the amendment
(c) By the discretion of the Speaker of the of Constitution
Lok Sabha 4. To discuss and pass such a Bill where both
(d) By the President on the recommendation Houses have different opinion
of Prime Minister
Select your answer from the following code-
94. As per Indian Constitution, Legislature of (a) 1 and 4
States consists- [41st B.P.S.C. (Pre) 1996] (b) 3 and 4
1. Legislative Council and Governor (c) 1 and 2
2. Legislative Assembly and legislative (d) only 4
Council
100. The Representation of States in Council of
3. Legislative Assembly and Governor States is based on-[38th B.P.S.C. (Pre) 1992]
4. Governor, Legislative Assembly and (a) Equal for all States
Legislative Council where it exists (b) Ratio of their Population
Select your answer out of the following codes- (c) Ratio of their Area
(a) 3 Only (d) Ratio of their Revenue
(b) 2 and 3 Only
(c) 3 and 4 Only 101. Who among the following can impose the
(d) 4 Only penalty under Section 20 of the Right to
Information Act? [M.P.P.C.S. (Pre), 2021]
95. A candidate of Lok Sabha Poll loses his (a) Central Information Commission only
security amount if he/she does not get- (b) State Information Commission only
[40th B.P.S.C. (Pre) 1995] (c) Central Information Commission or State
(a) 1/3 of valid votes Information Commission
(b) 1/4 of valid votes (d) High Court or Central Information
(c) 1/5 of valid votes Commission or State Information
(d) None of the above Commission

201 PYQ Workbook


INDIAN POLITY AND GOVERNANCE

102. An appeal under Scheduled Castes and (c) Benefit of probation


Scheduled Tribes (Prevention of Atrocities) (d) All of the above
Act, 1989 shall lie from any Judgement,
109. The State Government can specify a Court
sentence or order to the high Court:
of Session to be a Special Court to try the
[M.P.P.C.S. (Pre) 2019] offences under the Scheduled Castes and
(a) Only on facts Scheduled Tribes (Prevention of Atrocities)
(b) Only on law Act, 1989 with the concurrence of the-
(c) Both on facts and law [M.P.P.C.S. (Pre) 2013]
(d) None of the above
(a) Governor
103. How many seats are reserved for Scheduled (b) Chief Justice of High Court
Castes in the Lok Sabha? (c) Session Judge of concerned district
[M.P. P.C.S. (Pre) 2018] (d) Ministry of Law
(a) 59 110. A court can presume that any act
(b) 69 constituting offence was committed on the
(c) 79 ground of untouchability - if such offence is
(d) 84 committed in relation to-
104. Who has/had the power to make rules [M.P.P.C.S. (Pre) 2013]
for carrying out the purposes of the (a) a member of Scheduled Castes
Scheduled Castes and the Scheduled (b) a member of Scheduled Tribes
Tribes (Prevention of Atrocities) Act, (c) a member of any community
1989? [M.P.P.C.S. (Pre) 2016] (d) None of the above
(a) The State Government 111. The Scheduled Castes and Scheduled Tribes
(b) The Central Government (Prevention of Atrocities) Act is a special
(c) Both and provision for the advancement of Scheduled
(d) The Supreme Court Castes and Scheduled Tribes which is based
105. Offences are tried under the Protection of on which one of the following doctrines?
Civil Rights Act, 1955 by the Courts of- [M.P.P.C.S. (Pre) 2012]
[M.P.P.C.S. (Pre) 2014] (a) Doctrine of Severability
(a) Sessions Court (b) Doctrine of Preferred Position
(b) Judicial Magistrate First Class (c) Doctrine of Protective discrimination
(c) Judicial Magistrate Second Class (d) Doctrine of Harmonious Construction
(d) Chief Judicial Magistrate 112. Who was the First Speaker of Lok Sabha?
106. Who is empowered for imposition and [M.P.P.C.S. (Pre) 2012]
realization of a collective fine under SC and (a) Mavalankar
ST (Prevention of Atrocities) Act, 1989 (b) Hukum Singh
[M.P.P.C.S. (Pre) 2014] (c) B.D. Jatti
(a) District Magistrate (d) V.V. Giri
(b) State Government 113. All offences punishable under the Protection
(c) Special Court of Civil Rights Act, 1955 are:
(d) High Court
[M.P.P.C.S. (Pre) 2012]
107. Which Article of the Indian Constitution (a) Cognizable and Non-bailable
provides the provision of Vidhan Parishad (b) Cognizable and Non-compoundable
in the State? [M.P.P.C.S. (Pre) 2014] (c) Non-cognizable and Bailable
(a) Article 170 (d) Non-cognizable and Compoundable
(b) Article 171
(c) Article 172 114. The Protection of Civil Rights Act extends
(d) Article 173 to: [M.P.P.C.S. (Pre) 2012]
(a) The whole of India
108. Under the Scheduled Castes and Scheduled (b) The whole of India except the Scheduled
Tribes (Prevention of Atrocities) Act 1989, areas
which of the following is barred? (c) The whole of India except the State of
[M.P.P.C.S. (Pre) 2013] Jammu and Kashmir
(a) Anticipatory Bail (d) The whole of India except the Union
(b) Post-arrest bail Territories of Goa, Daman and Diu

PYQ Workbook 202


INDIAN POLITY AND GOVERNANCE

115. Consider the following statements with (c) the Parliament as per Resolution passed
respect to qualification to become the by the Legislative Assembly of the State
member of Rajya Sabha and select the (d) The Governor on the recommendation of
correct answer from the code given below: the Council of Ministers
(1) Should be at least 30 years of age 121. The ‘Contingency Fund’ of the State is
(2) Should be registered as a voter on the operated by-
list of voters in the state from which the [U.P. Lower Sub. (Spl) (Pre) 2004]
candidate is to be elected (a) The Governor of the State
(3) Should not hold office of profit under the (b) The Chief Minister of the State
state (c) The State Finance Minister
Code: [M.P.P.C.S. (Pre) 2008] (d) None of the above
(a) (1) and (2) are correct
122. Which one of the following is not basic
(b) (1) and (3) are correct
element of the citizen charter?
(c) (2) and (3) are correct
(d) (1), (2) and (3) are correct [R.A.S./R.T.S. (Pre) 2021]
(a) Description of services being provided by
116. Who prorogues the Vidhan Sabha of a State? department of the agency
[M.P.P.C.S. (Pre) 2002] (b) Promotion of various methods to get
(a) Governor benefit from the services available
(b) Vidhan Sabha Speaker (c) To expect any public record
(c) Chief Minister (d) Provision for the inspection of the
(d) Law Minister agency’s work
117. How can the speaker of the Lok Sabha be 123. Which of the following is not true about the
removed? [M.P.P.C.S. (Mains) 1994] objectives of the Right to Information Act?
(a) By a resolution of Lok Sabha passed by a [R.A.S./R.T.S. (Pre) 2021]
two-third Majority of members present.
(a) To promote transparency and
(b) By a resolution of the Lok Sabha passed
by a majority of all the members of the accountability in the working of the
House. government
(c) By President on the advice of the Prime (b) To make our democracy work for the
Minister. people in real sense
(d) None of these (c) To empower the women and weaker
sections of the society
118. In which assembly of States, money Bills are (d) To empower citizens
proposed? [M.P.P.C.S. (Pre) 1993]
(a) Any one of the two 124. Which of the following statement about
(b) Together in both resolution adopted by a State Legislative
(c) Only in Legislative Assembly Assembly under Article 169 of the
(d) Only in upper chamber Constitution of India for the abolition of
the State Legislative Council is correct?
119. What is the consequence of the failure of
[R.A.S./R.T.S. (Pre) 2021]
Government in Rajya Sabha?
(a) It does impose an obligation on the
[M.P.P.C.S. (Pre) 1990]
Governor to reserve resolution for the
(a) Prime Minister gives his resignation consideration of the President.
(b) Parliament will dissolve
(b) It does not impose an obligation on the
(c) The operation of President Rule
Union Government to take action for
(d) None of the above
initiating legislative in Parliament.
120. In any State of India, Legislative Council (c) It does not impose an obligation on the
can be created or can be abolished by Governor to reserve resolution for the
[U.P. Lower (Spl) (Pre) 2008] consideration of the President.
(a) the President on the recommendation of (d) It does impose an obligation on the Union
the Governor of the State Government to take action for initiating
(b) the Parliament legislation in Parliament.

203 PYQ Workbook


INDIAN POLITY AND GOVERNANCE

125. The President of India addresses the (b) 1, 2 and 4


Parliament under Article 87 of the Indian (c) 1, 3 and 4
Constitution- [R.A.S./R.T.S. (Pre) 2018] (d) 1 and 2
(a) in a joint sitting of both Houses of 129. Which among the following group of States/
Parliament. Union Territories has only one seat in the
(b) in a joint meeting of both Houses of
Lok Sabha? [R.A.S./R.T.S. (Pre) 2016]
Parliament.
(c) in a joint session of both Houses of (a) Goa, Meghalaya, Nagaland
Parliament. (b) Manipur, Dadra and Nagar Haveli,
(d) both Houses of Parliament assembled Puducherry
together. (c) Arunachal Pradesh, Sikkim, Lakshadweep
(d) Chandigarh, Sikkim, Mizoram
126. Which of the following are Financial
Committees of Indian Parliament? 130. The objective of Right to Information Act is
1. Estimates Committee to- [R.A.S./R.T.S. (Pre) 2007]
2. Public Accounts Committee (a) Secure access to information from public
3. Committee on Public Undertakings authorities.
(b) Provide information to the public through
4. Joint Committee on Salaries and
a person who has the information.
Allowance for Members of Parliament
(c) Provide information by Government on a
Select the correct answer using code given public place.
below: [R.A.S./R.T.S. (Pre) 2018] (d) Secure information by police from
(a) 1, 3 and 4 culprits.
(b) 1, 2 and 4
(c) 1, 2 and 3 131. Which of the following statements is/
(d) 2, 3 and 4 are true in reference to the joint sitting of
Parliament for passage of bill?
127. Which of the following terminology
is/are mentioned in the Preamble (i) Provision for Joint sitting is applicable
of the Right to Information Act, for ordinary bills, financial bills and
2005? [R.A.S./R.T.S. (Pre) 2018] constitutional amendment bills
1. Transparency of Information (ii) In no situation, Deputy Chairman of
2. Revelation of Information Rajya Sabha can preside over the joint
3. Informed Citizenry sitting
4. Paramountcy of the democratic ideal (iii) Quorum of 1/6th of the total number of
members of both houses to be maintained
Select the correct answer using code given
for joint sitting
below:
Code: [Chhattisgarh P.C.S. (Pre) 2021]
(a) 1 only
(b) 1 and 2 (a) (i), (ii) and (iii)
(c) 1, 2 and 3 (b) (i) and (ii)
(d) 1, 2, 3 and 4 (c) (i) and (iii)
(d) None of these
128. Which of the following statements are
correct in reference to rule of procedure 132. Which of the following Committees does
for Motion for Adjournment on a matter of not come under the jurisdiction of Lok
public importance in Rajasthan Legislative Sabha? [Chhattisgarh P.C.S. (Pre) 2021]
Assembly? (a) Committee on Social Justice and
1. Speaker’s consent necessary to make Empowerment
motion. (b) Committee on Urban Development
2. The motion raises a question of privilege. (c) Committee on Labour
3. The motion shall be restricted to a specific (d) Committee on Health and Family Welfare
matter of recent occurrence. 133. In the following questions, there are two
4. Not more than one such motion shall be statements. One is labelled as Assertion and
made at the same sitting. other is labelled as Reason (R):
Select the correct answer using code given Assertion (A): The courts are barred from
below: [R.A.S./R.T.S. (Pre) 2018] enquiring into the advice rendered by the
(a) 1 and 3 ministers to the Governor.

PYQ Workbook 204


INDIAN POLITY AND GOVERNANCE

Reason (R): There is no provision in the (a) Joint session will be held in scheduled
Indian Constitution for the system of legal time
responsibility of the ministers in the States. (b) Joint session will be cancelled
In the context of the above two statements (c) Joint session will be held after composition
which of the following is correct? of New Lok Sabha
[Chhattisgarh P.C.S. (Pre) 2021] (d) Bill will be lapsed
(a) Both Assertion and Reason (R) are true, 137. Which of the following doesn’t take the
and Reason (R) is correct explanation of oath of the office?
Assertion
(b) Both Assertion and Reason (R) are true, [Chhattisgarh P.C.S. (Pre) 2017]
but Reason (R) is not correct explanation (a) President
of Assertion (b) Vice-President
(c) Assertion is true, but Reason (R) is false (c) Speaker
(d) Assertion is false, but Reason (R) is true (d) Prime Minister
134. Match the articles related to State (e) Members of Council of Ministers
Legislature: 138. What is correct about the State Legislative
List -I List-II Council?
A. Constitution of 1. Article (i) Its tenure is 6 years
legislature in States 178 (ii) It is a permanent House
B. The Speaker and Deputy 2. Article (iii) It cannot be dissolved
Speaker of the Legislative 199 (iv) 1/6 members are elected by local
Assembly institutions
C. Salaries and allowances 3. Article (v) 1/6 members are elected by legislative
of members 168 assembly
D. Definition of ‘Money 4. Article (vi) Every two years 1/3 members are retired
Bills’ 195 (vii) Deputy Governor is the Chairman of
Code: [Chhattisgarh P.C.S. (Pre) 2021] House
A B C D (viii) The tenure of its members is 6 years
(a) 1 2 3 4 Code: [Chhattisgarh P.C.S. (Pre) 2015]
(b) 4 2 1 3
(a) (i), (iii), (iv), (v)
(c) 3 1 4 2
(b) (iii), (vi), (vii), (viii)
(d) 2 3 1 4
(c) (ii), (iii), (vi), (viii)
135. The merger of a political party with another (d) (ii), (iv), (i), (viii)
political party in India is recognized in (e) (i), (iii), (v), (vii)
which of the following conditions?
[Chhattisgarh P.C.S. (Pre) 2019] 139. The tenure of which Lok Sabha was about 6
(a) When not less than one-third members of years? [Chhattisgarh P.C.S. (Pre) 2014]
the legislature party have agreed to such (a) 5th Lok Sabha
merger. (b) 7th Lok Sabha
(b) When not less than half members of the (c) 9th Lok Sabha
legislature party have agreed to such (d) 11th Lok Sabha
merger. (e) 13th Lok Sabha
(c) When not less than two-third members of
the legislature party have agreed to such 140. Who among the following can participate in
merger. the deliberations of Lok Sabha while being
(d) When not less than three-fourth members a member of Rajya Sabha?
of the legislature party have agreed to [Chhattisgarh P.C.S (Pre) 2013]
such merger. (a) Nominated member of Rajya Sabha who
136. What will be happened if President is an expert in a field
summons for the joint session of Parliament (b) Deputy Chairperson of Rajya Sabha
and in the meantime, Lok Sabha dissolved? (c) Minister who is a member of Rajya Sabha
[Chhattisgarh P.C.S. (Pre) 2018] (d) None of these

205 PYQ Workbook


INDIAN POLITY AND GOVERNANCE

141. Privilege motion can be raised against a IV. Members of Legislative Council
Minister when he- Select the correct answer:
[Chhattisgarh P.C.S. (Pre) 2011] [Uttarakhand P.C.S. (Pre) 2012]
(a) Loses confidence in the Government. (a) I, II and III
(b) Absents himself for a long time. (b) I, III and IV
(c) Withholds facts of a case or gives distorted (c) I and III
versions of Facts. (d) I, II and IV
(d) Limit value.
147. Which of the following statements about
142. Which of the following is not true about the the Committee on Public Undertakings is/
Anti-Defection Bill? are correct?
[Chhattisgarh P.C.S. (Pre) 2011] 1. There are more members from the
(a) An M.P. or an M.L.A. will be disqualified Rajya Sabha than the Lok Sabha in the
if he has voluntarily given up the Committee.
membership of the party to which he 2. The Chairperson of the Committee is
belonged. appointed by the Speaker of the Lok
(b) In the event of a member’s absence from Sabha.
voting in the House on a particular issue,
Select the correct answer using the code given
an M.P. or M.L.A. will be disqualified.
below. [CAPF 2022]
(c) A member shall not be disqualified if
there is a merger of political parties. (a) 1 only
(d) The Speaker of the Lok Sabha will not be (b) 2 only
disqualified if he becomes a member of (c) Both 1 and 2
another political party later on. (d) Neither 1 nor 2

143. Which one of the following provisions 148. Which one of the following statements is
deals with the privileges and immunities not correct? [CAPF 2022]
of the Members of Parliament under the (a) A censure motion can be moved. against
Constitution of India? a single Minister of the Council of
[Chhattisgarh P.C.S. (Pre) 2011] Ministers.
(b) A motion of no-confidence cannot be
(a) Article 104
moved against the Prime Minister alone.
(b) Article 105
(c) A censure motion cannot be moved
(c) Article 82
against the entire Council of Ministers.
(d) Article 117
(d) A censure motion can be moved against a
144. The state wise seat allocation in Lok Sabha group of Ministers.
is based on-
149. Which one of the following statements
[Chhattisgarh P.C.S. (Pre) 2005] about Public Accounts committee is
(a) Population correct? [CAPF 2021]
(b) Area (a) The committee Consists of not more than
(c) Poverty 15 members from the Lok Sabha and 10
(d) Language members from the Rajya Sabha.
145. The Lok Sabha Secretariat comes under the (b) The finance minister is an ex officio
direct supervision of: member of the Committee.
[Uttarakhand P.C.S. (Pre) 2012] (c) A member of the Committee is given a
(a) Ministry of Home Affairs term for one year.
(b) Ministry of Parliamentary Affairs (d) The term of the office of the members of
(c) President of India the Committee from the Rajya Sabha is
(d) Lok Sabha Speaker extended even when they retire from the
Rajya Sabha.
146. State Legislative Assembly can participate
in the election of which of the following: 150. Who among the following warned his
colleagues that the Constituent Assembly
I. President of India
was British made and was “working the
II. Vice-President of India British plan as the British should like it to
III. Members of Rajya Sabha be worked out”? [CAPF 2021]

PYQ Workbook 206


INDIAN POLITY AND GOVERNANCE

(a) Somnath Lahiri (c) 1 and 3 only


(b) B.R. Ambedkar (d) 1 Only
(c) Shyama Prasad Mukherjee
155. Which of the following statements are
(d) B.N. Rau
correct with regard to Speaker/Deputy
151. Who among the following fixes the date of Speaker of the Lok Sabha?
election of the Speaker of the Lok Sabha? 1. On dissolution of the Lok Sabha, both the
[CAPF 2021] Speaker and Deputy Speaker cease to be
(a) The President of India members of the House.
(b) The Minister of Parliamentary Affairs 2. Both Speaker and Deputy Speaker vacate
(c) The Secretary General of the Lok Sabha their office on dissolution of the Lok
(d) The Deputy Speaker of the Lok Sabha Sabha.
152. Which one of the following statements with 3. Only Deputy Speaker vacates his office
regard to Censure Motion is NOT correct? and Speaker shall not vacate his office
until immediately before the first meeting
[CAPF 2020]
of the Lok Sabha after dissolution.
(a) A leave of the House is required to move
it Select the correct answer using the codes
(b) The Government is free to fix time and given below: [CAPF 2020]
date for its discussion (a) 1 only
(c) It can also be moved against the entirety (b) 1, 2 and 3
of the Council of Ministers (c) 2 and 3 only
(d) The Speaker decides whether the motion (d) 1 and 3 only
is in order 156. With regard to Panel of Chairperson, Lok
153. As per provisions of the Constitution of Sabha, which of the following statements is/
India which one of the following is correct? are correct?
[CAPF 2020] 1. Panel of Chairperson is drawn from the
ruling party only.
(a) Only Parliament has the power to legislate
over a subject under the Concurrent list 2. Panel of Chairperson is nominated by
(b) Both Parliament and State Legislatures different political parties and appointed
have the power to legislate over a subject by Speaker, Lok Sabha.
under the Concurrent list. 3. Panel of Chairperson consists of 10
(c) A legislation made under the Concurrent members and one of them presides over
list by Parliament should be ratified by the House when both Speaker and Deputy
half of the State Legislatures so as to Speaker are not there.
become a law of the country Select the correct answer using the codes
(d) Only State Legislatures are empowered to given below: [CAPF 2020]
make laws under Concurrent list (a) 1 and 3
154. Which of the following statement(s) (b) 2 and 3
regarding passing a law under the State list (c) 3 only
is/are correct? (d) 2 only
1. Even in the sphere (State list) exclusively 157. A member giving notice of a resolution for
reserved for the States, Parliament can removal of Speaker, Lok Sabha should be
legislate under certain circumstances. addressed to the [CAPF 2020]
2. Parliament cannot legislate a subject under (a) Deputy Speaker, Lok Sabha
the State list under any circumstances. (b) Secretary General, Lok Sabha
3. A resolution supported by two-thirds (c) Prime Minister
of the members present and voting is (d) President
required to be passed by Rajya Sabha to 158. Which one of the following statements
pass a law under the State list regarding Public Accounts Committee is
Select the correct answer using the codes correct? [CAPF 2020]
given below: [CAPF 2020] (a) It consists of fifteen members from Lok
(a) 1, 2 and 3 Sabha and seven members from Rajya
(b) 2 and 3 only Sabha

207 PYQ Workbook


INDIAN POLITY AND GOVERNANCE

(b) It consists of twenty members from Lok (c) 2 and 3 only


sabha and ten members from Rajya Sabha (d) 1 and 3 only
(c) The term of members of Public Accounts 162. Which one of the following is NOT true
Committee is five years of the 10th Schedule of the Constitution
(d) Finance Minister is the Chairperson of of India pertaining to disqualification
the Public Accounts Committee of members of the Parliament and State
159. Which of the following statements with legislatures? [CAPF 2019]
regard to the Speaker, Lok Sabha is/are (a) The schedule lays down that elected
correct? members may be disqualified on the
1. The election of the Speaker of Lok Sabha grounds of defection.
is conducted by the Election Commission (b) Disqualification on grounds of defection
of India. does not apply in cases of merger with
another political party.
2. The Speaker on assuming his office is not (c) Cases of dispute are decided by the speaker
required to make and subscribe oath or or Chairman of the House concerned.
affirmation. (d) The Supreme Court of India is the final
3. The Speaker does not exercise a casting arbiter in cases which remain unresolved.
vote in case of equality of votes.
163. Which of the following pairs of list and
Select the correct answer using the codes contents is/are correctly matched?
given below: [CAPF 2020] 1. State List: Public health and sanitation
(a) 2 only 2. Union List: Citizenship, naturalization
(b) 1 and 3 and aliens
(c) 1 only 3. Concurrent List: Legal, medical and other
(d) 2 and 3 professions
160. Which one of the following does not fall Select the correct answer using the code given
under the definition of the Money bill? below: [CAPF 2019]
[CAPF 2019] (a) 1 only
(a) Amendment of law with respect to any (b) 1, 2 and 3
financial obligations undertaken by the (c) 2 and 3 only
government of India (d) 3 only
(b) The payment of money into the 164. The following item consists of two
Consolidated Fund of India statements, Statement I and Statement II.
(c) Any financial bill as per requirements of Examine these two statements carefully and
Article-117 select the correct answer using the codes
(d) Appropriation of money out of the given below
Consolidated Fund of India Codes: [CAPF 2018]
161. Consider the following statements relating Statement I: In India, a majority of the
to short notice questions asked in the members of the Rajya Sabha are elected by
Legislature the elected members of the State Legislatures.
1. These relate to matters of urgent public Statement II: Members of the Rajya Sabha
importance and can be asked for oral elected by the State Legislative Assemblies
answer at a notice less than 10 days. need to have their domicile in the concerned
2. Short notice questions can be admissible State.
if the Minister concerned agrees to (a) Both the statements are individually
answer to it. true and Statements II is the correct
explanation of Statement I
3. Short notice questions are asked during
(b) Both the statements are individually
question hour.
true but Statement II is not the correct
Which of the statements given above are explanation of Statement I
correct? [CAPF 2019] (c) Statement I is true but Statement II is false
(a) 1, 2 and 3 (d) Statement I is false but Statement II is
(b) 1 and 2 only true

PYQ Workbook 208


INDIAN POLITY AND GOVERNANCE

165. Which one of the following Constitutional 169. Which one of the following is the correct
Amendments has enormously strengthened sequence of different stages a budget has to
the powers of the Speaker/Chairman go through in the Parliament?
of the 2 Houses of the Parliament/State 1. Presentation of the Budget
Legislatures? [CAPF 2018] 2. Scrutiny by Departmental Committees
(a) 61st Amendment which reduced the 3. Passing of Finance Bill
voting age from 21 to 18 years 4. Passing of Appropriation Bill
(b) Anti-defection provisions of 52nd
Select the correct answer using the code
Amendment
given below: [CAPF 2017]
(c) Repealing of many of the provisions of
42nd Amendment by 44th Amendment (a) 1-2-4-3
(d) 73rd Amendment that conferred extensive (b) 1-3-2-4
the powers on Panchayat Bodies (c) 2-1-3-4
(d) 4-3-2-1
166. Which one of the following Committees of
the Parliament has no Members from the 170. Which one of the following is a characteristic
Rajya Sabha? [CAPF 2018] of Presidential form of government?
(a) Public Accounts Committee [CAPF 2016]
(b) Committee on Public Undertakings (a) President is not a part of legislative body
(c) Estimates Committee (b) It does not separate Legislative and
(d) Departmentally Related Standing Executive functions
Committee (DRSC) on Finance (c) President follows the principle of
collective responsibility
167. Over which of the following, the Lok Sabha (d) The tenure of the President depends on
and the Rajya Sabha held joint sittings to the legislature
resolve their differences?
1. The Dowry Prohibition Bill, 1959 171. Which one of the following articles of the
Constitution of India contains provisions
2. The Banking Service Commission
for the abolition and creation of Legislative
(Repeal) Bill, 1978
Councils? [CAPF 2016]
3. The Prevention of Terrorism Bill, 2002
(a) Article-171
4. The Land Acquisition, Rehabilitation and (b) Article-169
Resettlement Act, 2013 (c) Article-356
Select the correct answer using the codes (d) Article-182
given below: [CAPF 2018]
172. With regard to a Constitution Amendment
(a) 1, 2, 3 and 4
Bill, which one of the following statements
(b) 3 and 4 only
is not correct? [CAPF 2016]
(c) 1, 2 and 3 only
(d) 2 and 4 only (a) The Speaker of the Lok Sabha can call a
joint sitting of both the Houses to pass
168. Which of the following pair(s) is/are the Bill
correctly matched? (b) Each House needs to pass the Bill
1. Rule of lapse: Part of grant that can be separately by a prescribed special majority
carried over to next year. end
2. Supplementary grant: An advance grant (c) The Bill can be introduced in either
to meet expenditures. House of Parliament
3. Vote on account: Additional funds (d) The Bill can be sponsored by a Private
granted in the course of financial year. Member
Select the correct answer using the code given 173. ‘Cut Motion’ can be introduced after the
below: [CAPF 2017] presentation of [CAPF 2016]
(a) 1 only (a) any Bill introduced in the Parliament
(b) 1 and 2 (b) the Railway and General Budgets
(c) 2 and 3 (c) any Private Member’s Bill
(d) None of the above (d) a Constitution Amendment Bill

209 PYQ Workbook


INDIAN POLITY AND GOVERNANCE

174. According to Article 3 of the Constitution (a) It cannot be introduced in the Council of
of India, the Parliament may by law States.
1. declare war on any country. (b) If any question arises whether the Bill
2. alter the boundaries of any State. is Money Bill or not, the decision of the
3. increase the area of any State Speaker is final.
4. establish an Autonomous Council within (c) In case of deadlock over a Money Bill, the
any State. President can summon a joint sitting of
the Parliament.
Select the correct answer using the code given (d) A Money Bill cannot be introduced except
below. [CAPF 2015] on the recommendation of the President.
(a) Only 3
(b) 1, 2 and 4 179. Which of the following statements
(c) 2 and 3 regarding the residuary powers under the
(d) 1, 2 and 3 Constitution of India is/are correct?
1. Residuary powers have been given to the
175. The Rajya Sabha can withhold its consent to
Union Parliament.
Money Bill for: [CAPF 2015]
2. In the matter of residuary powers,
(a) 14 days
the Constitution of India follows the
(b) 15 days
Constitution of Australia.
(c) 30 days
(d) 18 day’ 3. Schedule 7 of the Constitution of India
provides a list of residuary powers.
176. The following item consists of two 4. The Government of India Act, 1935
Statements I and II. You are to examine placed residuary powers in the hands of
these two statements carefully and select the Governor-General.
the answer to these items using the code
given below. Select the correct answer using the codes
given below:
Statement I: The Rajya Sabha is not subject to
Codes: [CAPF 2014]
dissolution and the members enjoy a tenure
of six years. (a) 1 and 3
(b) 2 and 3
Statement II: According to Article 83 of the (c) 1 and 4
Constitution of India, one-third of members (d) Only 4
of Rajya Sabha retire every two years.
Codes: [CAPF 2014] 180. Which of the following statements about
(a) Both the statements are individually true the Presidential system is/are correct?
and Statement II is the correct explanation 1. The Head of Government is also the Head
of Statement I of State.
(b) Both the statements are individually 2. The Executive can veto Legislative acts.
true but Statement II is not the correct Select the correct answer using the codes
explanation of Statement I given below.
(c) Statement I is true and Statement II is Codes: [CAPF 2014]
false (a) Only 1
(d) Statement I is false but Statement II is (b) Only 2
true (c) Both 1 and 2
177. The issue whether a commitment made by a (d) Neither 1 nor 2
Minister on the floor of the House has been
181. Which one of the following is not a provision
fulfilled or not is decided by the
related to a Money Bill?- [CDS 2022
[CAPF 2014] (I)]
(a) Concerned Departmentally Related (a) Imposition, abolition, remission
Committee alteration or regulation of any tax.
(b) Business Advisory Committee (b) Appropriation of money out of the
(c) Committee on Public Assurances Consolidated Fund of India.
(d) Political Affairs Committee of the Union (c) Imposition of fines by local authority for
Cabinet local purpose.
178. Which of the following statements related (d) Custody of the Consolidated Fund of
to Money Bills is not correct? [CAPF 2014] India or the Contingency Fund of India.

PYQ Workbook 210


INDIAN POLITY AND GOVERNANCE

182. Which of the following statements is/are 186. Which one of the following statements
correct? about Money Bill is not correct?
1. Committee on Estimates is an ad hoc [CDS 2022 (II)]
committee of the Parliament. (a) Money Bill cannot be introduced in the
2. Committee on Railway Convention is a Council of States.
standing committee of the Parliament. (b) The Council of States has no power to
Select the correct answer using the code given reject or amend the Money Bill.
below:- [CDS 2022 (I)] (c) The Speaker of the Lok Sabha has the sole
(a) 1 only and final power in deciding whether a
(b) 2 only Bill is a Money Bill or otherwise.
(c) Both 1 and 2 (d) The Council of States has no power to
(d) Neither 1 nor 2 discuss the Money Bill.
183. Which one of the following statements 187. Which one of the following is not a
about the Speaker of Lok Sabha is not category under which the President of India
correct? - [CDS 2022 (I)] nominates Members of Parliament?
(a) He shall vacate his office if he ceases to [CDS 2022 (II)]
be a member of the House of the People. (a) Literature
(b) He may, at any time, resign by writing his (b) Science
resignation to the President of India. (c) Art
(c) He may be removed from his office by (d) State Service
a resolution of the House of the People
passed by majority of all the then 188. Which among the following is not the
members of the House. strength or chief benefit of Bicameralism?
(d) While the office of the Speaker is vacant, [CDS 2022 (II)]
the duties of the office shall be performed (a) Second chambers check the first chambers
by the Deputy Speaker. and prevent majoritarian rule.
(b) It checks the powers of the executive.
184. Which one of the following statements about (c) The second chambers can act as a
a Bill for Amendment of the Constitution constitutional safeguard.
of India is not correct?- [CDS 2022 (I)] (d) It often acts as a check on democratic
(a) It is governed by Article 368(2) of the rule, particularly when their members are
Constitution of India. non-elected or indirectly elected.
(b) Joint sitting can be resorted to for Passing
a Bill amending the Constitution of India. 189. Which of the following statements regarding
(c) The State Legislatures cannot initiate any the ‘casting vote’ in the Parliament is/are
Bill or proposal for amendment of the correct? [CDS 2021(I)]
Constitution of India. 1. It is cast by the speaker or a person acting
(d) The previous sanction of the President of as such.
India is not required for introducing any 2. It is cast in addition to voting in the first
Bill in the Parliament for amendment of instance.
the Constitution of India. 3. It is cast in the case of equality of votes.
185. Which one of the following statements 4. It is always cast to maintain the status
about the Public Accounts Committee quo.
(PAC) of the Parliament is not correct? Select the correct answer using the codes
[CDS 2022 (II)] given below:
(a) It examines the Finance Accounts of the (a) 1, 2 and 3 only
Government of India. (b) 1 and 3 only
(b) Fifteen members of the Committee are (c) 2 and 4 only
elected by the Lok Sabha from amongst (d) 3 only
its members.
(c) The Chairperson of the Committee is 190. Which one among the following motions
elected by its members. cannot be made while introducing an
(d) In case a member of any other Committee ordinary Bill in the Parliament?
constituted by the Government is elected [CDS 2021(I)]
to the PAC, the Speaker of the Lok Sabha (a) That the Bill be taken into consideration
decides whether he should continue to be (b) That the Bill be circulated for the purpose
a member of the former Committee. of eliciting public opinion

211 PYQ Workbook


INDIAN POLITY AND GOVERNANCE

(c) That the Bill be referred to a Select Select the correct answer using the codes
Committee given below: [CDS 2021 (II)]
(d) That the Bill be referred to a Joint (a) Only 1
Committee of the House without the (b) 1, 2 and 3
concurrence of the other House (c) 2 and 3
191. Which one of the following statements (d) 1 and 2
about the composition of the Parliament is 196. Which one of the following is not correct?
not correct? [CDS 2021(I)]
[CDS 2021 (II)]
(a) Representatives of the states in Rajya
Sabha are elected directly by the people. (a) Taxes on agricultural income is a subject
(b) Representatives from Union Territories in under the State List.
Lok Sabha are chosen by direct elections. (b) Price control is a subject under the
(c) Rajya Sabha has 12 nominated members. Concurrent List.
(d) Lok Sabha has seats reserved for SCs and (c) Insurance does not come under the Union
STs. List.
(d) ‘Forests’ is a subject under the Concurrent
192. A motion of no-confidence is moved against
List.
[CDS 2021(I)]
(a) an individual Minister. 197. Which among the following is not a
(b) the Council of Ministers. condition for the disqualification of a
(c) the Prime Minister. Member of Parliament? [CDS 2021 (II)]
(d) a political party. (a) Voluntary acquisition of citizenship of a
193. Which of the following statements are foreign country
correct? (b) Holding the office of the Chairperson of
1. Speaker may permit any Member to the National Commission for Women
address the House in his/her mother (c) The member abstains from voting in the
tongue, if he/she cannot adequately House without prior permission
express in either Hindi or English. (d) The Member holds the office of the
2. Business of the Parliamentary Committees Chairman of the Board of Directors of the
is transacted either in Hindi or in English. National Coal Development Corporation
3. The minutes of the Parliamentary Ltd
Committees are prepared invariably in 198. Overseas Indians can exercise franchise in
Hindi or English. an election to the Lok Sabha under which of
Select the correct answer using the codes the following conditions?
given below: [CDS 2021 (II)] 1. They must be citizens of India.
(a) 1, 2 and 3 2. Their names must figure in the electoral
(b) 1 and 2 roll.
(c) 2 and 3
(d) 1 and 3 3. They must be present in India to vote.
Select the correct answer using the codes
194. Which one among the following is the given below. [CDS 2020 (I)]
largest State in terms of seats in the Rajya
Sabha? [CDS 2021 (II)] (a) 1, 2 and 3
(a) Andhra Pradesh (b) 2 and 3
(b) Bihar (c) 1 and 2
(c) Rajasthan (d) Only 1
(d) Karnataka 199. Which one among the following is not
195. Which of the following statements regarding correct about the Secretary General of the
constitutional amendment is/are correct? Lok Sabha? [CDS 2020 (II)]
1. The procedure for amendment to the (a) The Secretary General is the advisor to
Constitution is provided in Article 368. the Speaker.
2. A Bill to amend the Constitution can (b) The Secretary General acts under the
be introduced in either House of the authority in the name of the Speaker.
Parliament. (c) The Secretary General works under the
3. The special procedure in Article 368 vests Speaker with delegated authority.
constituent powers upon the ordinary (d) The Secretary General passes orders in
legislation. the name of the Speaker.

PYQ Workbook 212


INDIAN POLITY AND GOVERNANCE

200. Which of the following statements with 204. Which one of the following regarding the
regard to the privileges of the Members of procedure and conduct of business in the
the Parliament are correct? Parliament is not correct?[CDS Pre.2018 II]
1. Privileges would not be fettered by the (a) To discuss state matters
Article-19(1)of the Constitution of India. (b) To discuss issues of the use of police force
2. Privileges must be read subject to the in suppressing the Scheduled Caste and
Articles 20-22 and Article-32 of the Scheduled Tribe communities
Constitution of India. (c) To discuss issues in dealing with violent
disturbances in an undertaking under the
3. Immunity is available in relation to both control of the Union Government
civil and criminal prosecution. (d) To discuss issues for putting down the
4. Immunity is available in relation to demands of the industrial labour
freedom of speech even in his/her private
or personal capacity. 205. Which one of the following is not considered
a part Legislature of States?
Select the correct answer using codes given
[CDS Pre.2018 II]
below [CDS 2020 (II)]
(a) The Governor
(a) 1, 2 and 4 only (b) The Legislative Assembly
(b) 1 and 2 only (c) The Legislative Council
(c) 2 and 3 only (d) The Chief Minister State
(d) 1 and 4 only
206. A joint sitting of the Parliament is resorted
201. Which one of the following statements with to, for resolving the deadlock between two
regard to the appointment of the Members Houses of the Parliament for passing which
of the Parliamentary Committees is correct? of the following Bills?
[CDS 2020 (II)] 1. Money Bill
(a) The Members are only appointed 2. Constitutional Amendment Bill
(b) The Members are only elected 3. Ordinary Bill
(c) The Members are only nominated. Select the correct answer using the codes
(d) The Members are appointed or elected on given below : [CDS Pre.2017 II]
a motion made and adopted or nominated (a) 1 and 2
by the Speaker of the Lok Sabha or the (b) 2 and 3
Chairman of the Rajya Sabha (c) 1, 2 and 3
202. Which one of the following is not a correct (d) Only 3
statement regarding the provision of 207. Which one of the following cannot be
Legislative Council in the State Legislature? introduced first in the Rajya Sabha?
[CDS Pre.2019 I] [CDS Pre.2017 II]
(a) The States of Bihar and Telangana have (a) Constitutional Amendment
Legislative Councils (b) CAG Report
(b) The total number of members in the (c) Annual Financial Statement
Legislative Council of a State shall not (d) Bill to alter the boundaries of any state
exceed one third of the total number of
208. Which one of the following Schedules of the
members in the Legislative Assembly Constitution of India has fixed the number
(c) One-twelfth of all members shall be of Members of the Rajya Sabha to be
elected by electorates consisting of local elected from each state? [CDS Pre.2017 II]
bodes and authorities
(a) Fifth Schedule
(d) One-twelfth of all members shall be (b) Third Schedule
elected by graduates residing in the state. (c) Sixth Schedule
203. Rajya Sabha has exclusive jurisdiction in (d) Fourth Schedule
[CDS Pre. 2019 II] 209. A Member of Lok Sabha does not become
(a) creation of new states disqualified to continue as a Member of the
(b) declaring a war House if the member [CDS Pre.2016 I]
(c) financial emergency (a) voluntarily gives up his/her membership
(d) authorising Parliament to legislate on a of the political party from which he/she
subject in the state list was elected

213 PYQ Workbook


INDIAN POLITY AND GOVERNANCE

(b) is expelled by the political party from 214. The subject matter of an adjournment
which he/she had been elected to the motion in the Parliament
House 1. must be directly related to the conduct of
(c) joins a political party after being elected the Union Government.
as an independent candidate 2. may involve failure of the Government of
(d) abstains from voting contrary to the India to perform its duties in accordance
direction by his/her political party with the Constitution.
210. After the general elections, the Protem Select the correct answer using the codes
Speaker is [CDS Pre. 2015 II] given below [CDS Pre.2014 (I)]
(a) elected by the Lok Sabha (a) Only 1
(b) appointed by the President of India (b) Only 2
(c) appointed by the Chief Justice of The (c) Both 1 and 2
Supreme Court (d) Neither 1 nor 2
(d) the senior most member of the Lok Sabha
215. Certain Bills cannot be introduced or
211. Which of the following statement(s) in the proceeded with unless the recommendation
context of structure of the Parliament is/are of the President is received.
correct?
However, no recommendation is required
1. The Parliament of India consists of the
President, the Council of States and the in some other cases. In which one of the
House of People. following cases such recommendation is
not required? [CDS Pre.2014 I]
2. The President of India is directly elected
by an electoral college consisting of the (a) For introduction of Bills and for moving
elected members of both the Houses of amendments relating to financial matters.
the Parliament only. (b) For introduction of a Bill relating to
formation of new states or of alternation
Select the correct answer using the codes of areas of existing states
given below: [CDS Pre.2014 (I)]
(c) For moving of an amendment making
(a) Only 1 provision for the reduction or abolition
(b) Only 2 of any tax
(c) Both 1 and 2 (d) For introduction of a Bill or moving of an
(d) Neither 1 nor 2
amendment affecting taxation in which
212. The Annual Financial Statement of the states are interested
Government of India in respect of each
financial year shall be presented to the 216. Which one of the following statements
House on such day as the [CDS Pre.2014 (I)] regarding the Departmental Committee of
the Parliament of India on the empowerment
(a) Speaker may direct
of women is correct? [CDS Pre.2014 I]
(b) President of India may direct
(c) Parliament may decide (a) The Committee will consist of members
(d) Finance Minister may decide of the Lok Sabha only
(b) A Cabinet Minister can be a member of
213. The principle of ‘collective responsibility’ the Committees
under parliamentary democracy implies (c) The term of office of the members of the
that Committee shall not exceed two years
1. a motion of no-confidence can be moved (d) It reports on the working of welfare
in the Council of Ministers as a whole as programmes for the women
well as an individual minister.
2. no person shall be nominated to the 217. The legislative power of the Parliament
cabinet except on the advice of the Prime includes making laws
Minister. 1. On matters not enumerated in the
3. no person shall be retained as a member Concurrent List and State list.
of the Cabinet if the Prime minister says 2. In respect of entries in State List if two
that he shall be dismissed. or more State Legislatures consider it
Select the correct answer using the codes desirable
given below : [CDS Pre.2014 (I) ] 3. For implementing any treaty agreement
(a) Only 1 or convention with any country even if it
(b) Only 2 falls in the State List.
(c) Only 3 Select the correct answer using the codes
(d) 2 and 3 given below : [CDS Pre.2014 I]

PYQ Workbook 214


INDIAN POLITY AND GOVERNANCE

(a) Only 2 Select the correct answer using the codes


(b) 1 and 2 given below: [CDS Pre.2014 I]
(c) 1 and 3 (a) Only 1
(d) All of these (b) Only 2
218. There are provisions in the Constitution (c) 1 and 2
of India which empower the Parliament to (d) All of these
modify or annul the operation of certain
provisions of the Constitution without 221. The functions of the committee on estimates,
actually amending them. They include as incorporated in the Constitution of
1. any law made under Article 2 (relating India, shall be to
to admission or establishment of new 1. report what economies, improvements in
states). organisation, efficiency or administrative
2. any law made under Article 3 (relating to reform may be effected.
formation of new states). 2. suggest alternative policies in order to
3. amendment of First Schedule and Fourth bring about efficiency and economy in
Schedule. administration.
Select the correct answer using the codes 3. examine whether the money is well laid
given below : [CDS Pre.2014 I] out within the limits of the policy implied
(a) 1 and 2 in the estimates.
(b) 2 and 3 4. examine the reports, if any, of the
(c) 1.2 and 3 Comptroller and Auditor General on the
(d) None of these public undertakings.
219. Which of the statements relating to the Select the correct answer using the codes
Deputy Speaker of the Lok Sabha is/are given below : [CDS Pre.2014 I]
correct? (a) 1 and 2
1. The office of the Deputy Speaker acquired (b) 2 and 3
a more prominent position after the
(c) 1, 2 and 3
enforcement of the Constitution of India
in 1950. (d) 3 and 4
2. He/She is elected from amongst the 222. Which of the following statements are
members. correct regarding Joint Session of the
3. He/She holds office until he/she ceases to Houses of the Parliament in India?
be a member of the House. 1. It is an enabling provision. empowering
Select the correct answer using the codes the President to take steps for resolving
given below : [CDS Pre.2014 I] deadlock between the two Houses,
(a) Only 1 2. It is not obligatory upon the President to
(b) 1 and 2 summon the Houses to meet in a joint
(c) 1, 2 and 3 sitting.
(d) 2 and 3
3. It is being notified by the President.
220. The Committee on Public Accounts under 4. It is frequently resorted to establish the
the Constitution of India is meant for : supremacy of the Lok Sabha
1. The examination of accounts showing
Select the correct answer using the codes
the appropriation of sums granted by
the House for the expenditure of the given below :
Government of India. [CDS Pre.2014 I]
2. scrutinising the report of the Comptroller (a) 1 and 2
and Auditor General. (b) 1, 2 and 3
3. suggesting the form in which estimates (c) 2 and 3
shall be presented to the Parliament. (d) 3 and 4

215 PYQ Workbook


INDIAN POLITY AND GOVERNANCE

SOLUTIONS
Deputy Speaker has the same power as the Speaker when
6.1. UPSC CSE Previous Years’ Questions presiding over the sitting of the House.
1. Solution (a) Statement 4 is incorrect: The Deputy Speaker is also
Exp) Option a is the correct answer elected by the Lok Sabha from amongst its members right
The Finance Bill is a part of the Union Budget, stipulating after the election of the Speaker has taken place. There is
all the legal amendments required for the changes in taxation no provision and or established practice of moving the
proposed by the finance minister. As per Article 110 of the motion for his election by the speaker and it is seconded by
Constitution of India, the Finance Bill is a Money Bill. the prime minister.

Statement 1 is incorrect: As the Finance Bill is a type of 4. Solution (b)


money bill, It (like the money bills) cannot be either Exp) Option b is the correct answer
rejected or amended by the Rajya Sabha.
Statement 1 is incorrect: A nominated member of a House
Statement 2 is correct: It is true that Money bills cannot be shall be disqualified for being a member of the House if
amended or rejected by the Rajya Sabha. The Rajya Sabha he joins any political party after the expiry of six months
should return the bill with or without recommendations, from the date on which he takes his seat. Thus he/she is free
which may be accepted or rejected by the Lok Sabha to join any political party within six months and would not
Statement 3 is incorrect: As the Finance bill is a type of be disqualified for the same.
money bill there is no provision of joint sitting for money Statement 2 is correct: The law does not specify a
as well as the Finance bill. time period for the Presiding Officer to decide on a
2. Solution: (b) disqualification plea under the Tenth Schedule.

Exp) Option b is the correct answer 5. Solution (b)


Statement 1 is incorrect: The proclamation of Emergency Exp) Option b is the correct answer
must be approved by both the Houses of Parliament within Statement 1 is incorrect: Constitution Amendment Bill can
one month from the date of its issue. If approved by both be introduced either by a minister or by a private member
the Houses of Parliament, the emergency continues for six and does not require prior permission from the President.
months, and can be extended to an indefinite period with an
Statement 2 is correct: The 24th Amendment of the Indian
approval of the Parliament for every six months.
Constitution amended Article 368 to provide expressly
Statement 2 is correct: Article 75 (3) provides clearly that that Parliament has the power to amend any provision of the
the Council of Ministers shall be collectively responsible to Constitution. The amendment further made it obligatory
the House of the People. So, only LokSabha can pass a no- for the President to give his assent when a Constitution
confidence motion against the Council of Ministers. Amendment Bill was presented to him.
Statement 3 is incorrect: As per Article 61, when a President Statement 3 is correct: Article 368 of the Indian constitution
is to be impeached for violation of the Constitution, the requires that the constitution of India can be amended
charges can be initiated by either House of Parliament. The by both houses of parliament by a 2/3 majority(special
impeachment resolution needs to be passed by a majority of majority). In case of disagreement between both houses,
two-thirds of the total membership in each house. Thus, it is there is no provision to summon a joint session of
not an exclusive power of the Lok Sabha. parliament. The reasoning behind this is so as to not let the
3. Solution: (a) running government’s majority in Lok Sabha undermine the
Rajya Sabha in a joint sitting.
Exp) Option a is the correct answer
Statement 1 is correct: The date of election of the Deputy 6. Solution (c)
Speaker is fixed by the Speaker. While on the other hand Exp) Option c is the correct answer.
date of election of the Speaker is fixed by the President. Statement 1 is correct. Parliamentary sovereignty is a
Statement 2 is incorrect: There is no mandatory provision principle of the UK constitution. It makes Parliament the
that the election of a candidate, as Deputy Speaker of Lok supreme legal authority in the UK, which can create or end
Sabha, shall be from either the principal opposition party or any law. Generally, the courts cannot overrule its legislation
the ruling party. It is only by convention that the position and no Parliament can pass laws that future Parliaments
of Deputy Speaker is offered to the opposition party in cannot change.
India. Indian Parliament is not a sovereign body like the British
Statement 3 is correct: In case of the absence of the Parliament. The Indian Parliament may, in exercise of its
Speaker, the Deputy Speaker presides over the sessions of constituent power, amend by way of addition, variation or
the Lok Sabha and conducts the business in the house. The repeal any provision of the Constitution in accordance with

PYQ Workbook 216


INDIAN POLITY AND GOVERNANCE

the procedure laid down for the purpose. However, the Ad Hoc Committee set up by the Parliament and Parliamentary
Parliament cannot amend those provisions which form the Department Related Standing Committee review the
‘basic structure’ of the Constitution. This was ruled by the independent regulators in sectors like telecommunications,
Supreme Court in the Kesavananda Bharati case (1973). insurance electricity, etc.
Statement 2 is correct. In India, matters related to the Finance Commission and NITI Aayog are advisory bodies
constitutionality of an amendment of an act of the and do not review the independent regulators in sectors
Parliament are referred to the Constitution Bench by the like telecommunications. Financial Sector Legislative
Supreme Court. A Constitution Bench is a bench of the Reforms Commission (FSLRC) also had no role in reviewing
Supreme Court having five or more judges on it. These independent regulators.
benches are not a routine phenomenon. A vast majority 9. Solution: (b)
of cases before the Supreme Court are heard and decided
Exp) Option b is the correct answer.
by a bench of two judges (called a Division Bench), and
Statement 1 is incorrect. The Communist Party of India
sometimes of three. Constitution Benches are set up when
(CPI) was the single largest opposition party in the First
the case involves a substantial question of law pertaining to
general Election held in 1951-52.
the interpretation of the Constitution (Article 145(3) of the
Constitution, which mandates that such matters be heard by Statement 2 is correct. Historically, the first officially
a bench of not less than five judges). designated Opposition party in Parliament emerged from
the break-up of the all-dominant Congress party in power.
Presently, Constitution Benches are set up on an ad hoc basis
In 1969, when Indira Gandhi was the Prime Minister,
as and when the need arises. The idea behind a Constitution
the Congress split to form the Indian National Congress
Bench is clear: it is constituted in rare cases to decide
(Requisitionists) and the Indian National Congress
important questions of fact or legal and/or constitutional
(Organisation). The Leader of INC(O), Ram Subhag Singh,
interpretation.
became the first person to be formally recognised as LoP in
7. Solution: (b) the Lok Sabha.
Exp) Option b is the correct answer. Statement 3 is incorrect. A party must have at least 10 per
cent of the strength of the House in order to qualify to be
Statement b is correct. Rajya Sabha has equal power with
designated as a parliamentary party and the leader of the
Lok Sabha under article 368 of the Constitution providing
largest such parliamentary party in the opposition ranks is
for Power of Parliament to amend the Constitution.
designated Leader of Opposition. Thus, a party must have
An amendment to the Constitution can be initiated only by 55 seats in Lok Sabha to designate its leader as the Leader of
the introduction of a Bill for the purpose in either House Opposition.
of Parliament, and when the Bill is passed in each House
by a majority of the total membership of that House and 10. Solution: (c)
by a majority of not less than two-thirds of the members of Exp) Option c is the correct answer.
that House present and voting, it is to be presented to the Statement c is incorrect. A Money Bill is concerned with
President who shall give his assent to the Bill and thereupon the appropriation of moneys out of the Consolidated Fund
the Constitution shall stand amended. of India.
Rajya Sabha cannot remove the government through motions Article 110 of the Constitution deals with the definition of
as the Council of Ministers is collectively responsible to the money bills. It states that a bill is deemed to be a money bill
House of the People and not to Rajya Sabha. if it contains ‘only’ provisions dealing with all or any of the
Under Article 312 of the Constitution, Rajya Sabha can following matters:
authorize the parliament to create a new All India Service 1. The imposition, abolition, remission, alteration or
by resolution supported by not less than two-thirds of the regulation of any tax; Option a is correct.
members present and voting. Lok Sabha has no such powers. 2. The regulation of the borrowing of money by the Union
A cut motion is a special power of the Lok Sabha members government; Option d is correct.
to oppose a demand being discussed for allocation by the 3. The custody of the Consolidated Fund of India or the
government in the Finance Bill as part of the Demand for contingency fund of India, the payment of moneys
Grants. into or the withdrawal of money from any such fund;
8. Solution: (a) Option b is correct.
Exp) Option a is the correct answer. 4. The appropriation of money out of the Consolidated
Fund of India;
Independent regularity authorities are agencies of modern
democratic governments. They are parts of the executive 5. Declaration of any expenditure charged on the
wing with a certain degree of statutory or constitutional Consolidated Fund of India or increasing the amount of
autonomy, reporting directly to the legislature. Like the any such expenditure;
general executive, they are accountable to the legislature and 6. The receipt of money on account of the Consolidated
subject to judicial review. Fund of India or the public account of India or the custody

217 PYQ Workbook


INDIAN POLITY AND GOVERNANCE

or issue of such money, or the audit of the accounts of the of individual liberty by a constitution. A constitutional
Union or of a state; or government puts limitations on the state to preserve individual
7. Any matter incidental to any of the matters specified liberty, so that the individual can develop to the best of his
above. ability. With reference to India, it is a system of Government
in which the executive is responsible to the legislature for its
However, a bill is not to be deemed to be a money bill by
policies and acts. This responsibility means that executive
reason only that it provides for: stay in power as long as it enjoys the confidence of the house.
1. the imposition of fines or other pecuniary penalties, or The Council of Ministers (executive) must resign when they
2. the demand or payment of fees for licenses or fees for lose confidence of the Parliament (Lok Sabha in particular).
services rendered; or 14. Solution: (d)
3. the imposition, abolition, remission, alteration or Exp) Option d is the correct answer.
regulation of any tax by any local authority or body for
Statement 1 is incorrect. As India follows the First Past the
local purposes
Post System, where a candidate who polls more votes than
Option c is incorrect. A Money Bill does not contain the any other candidate is declared elected.
provisions related with the appropriation of moneys out of Statement 2 is incorrect. As per the constitution, the
the Contingency Fund of India. speaker and deputy speaker in Lok Sabha are elected among
11. Solution: (a) its members. It is not a constitutional provision but a
convention that Speaker’s post goes to the majority party and
Exp) Option a is the correct answer.
the Deputy Speaker’s to the Opposition.
The Speaker is elected by the Lok Sabha from amongst its
members (as soon as may be, after its first sitting). Whenever 15. Solution: (d)
the office of the Speaker falls vacant, the Lok Sabha elects Exp) Option d is the correct answer.
another member to fill the vacancy. The date of election of Statement 1 is incorrect. Every Member of Parliament,
the Speaker is fixed by the President. who is not a Minister, is called a Private Member and bills
Under article 179 of the Indian Constitution: The speaker presented by them are known as private member’s bills. It
a. shall vacate his office if he ceases to be a member of the reflects the stand of opposition party on public matter. Its
Assembly. (Statement 1 is correct) introduction in the House requires one month’s notice.
b. whenever the Assembly is dissolved, the Speaker shall Statement 2 is incorrect. The last Private Member’s Bill
not vacate his office until immediately before the first passed by parliament was the Supreme Court (Enlargement
meeting of the Assembly after the dissolution. (Statement of Criminal Appellate Jurisdiction) Bill, 1968, which became
2 is incorrect) an act on August 9, 1970.
16. Solution: (d)
12. Solution: (b)
Exp) Option d is the correct answer.
Exp) Option b is the correct answer.
According to Article 249 of the Constitution of India if the
Option a is incorrect. Committee on government
Rajya Sabha declares that it is necessary in the national
assurances– checks the assurances, promises and
interest that Parliament should make laws with respect
undertakings given by ministers from time to time on the
matters in the State List, then the Parliament becomes
floor of the House and reports on the extent to which they
competent to make laws on that matter. Such a resolution
have been carried through.
must be supported by two-thirds of the members present and
Option b is correct. Committee on Subordinate legislation voting. The resolution remains in force for one year; it can be
examines and reports to the House whether the powers to renewed any number of times but not exceeding one year at
make regulations, rules, sub-rules and bye-laws delegated a time. The laws cease to have effect on the expiration of six
by the Parliament or conferred by the Constitution to the months after the resolution has ceased to be in force.
Executive are being properly exercised by it. This provision does not restrict the power of a state
Option c is incorrect. Rules committee considers the legislature to make laws on the same matter. But, in case of
matters of procedure and conduct of business in the House inconsistency between a state law and a parliamentary law,
and recommends necessary amendments or additions to the the latter is to prevail.
rules of the House.
17. Solution: (d)
Option d is incorrect. Business advisory committee
regulates the program and time table of the House. It Exp) Option d is the correct answer
allocates time for the transaction of legislative and other Statement 1 is incorrect. According to the Constitution of
business brought before the House by the government. India, the maximum strength of the legislative council is
fixed at one-third of the total strength of the assembly and
13. Solution: (b) the minimum strength is fixed at 40 (with some exceptions).
Exp) Option b is the correct answer. Statement 2 is incorrect. The Chairman of the Legislative
A constitutional government is one whose powers have been Council is elected by the council itself from amongst its
adapted to the interests of its people and to the maintenance members.

PYQ Workbook 218


INDIAN POLITY AND GOVERNANCE

18. Solution: (c) 21. Solution: (c)


Exp) Option c is the correct answer. Exp) Option c is the correct answer.
Statement 1 is incorrect. The Budget Division of the The principle of collective responsibility implies that the
Department of Economic Affairs (DEA) (and not the Lok Sabha can remove the ministry (i.e., council of ministers
department of revenue) in the finance ministry is the nodal headed by the prime minister) from office by passing a vote
body responsible for producing the Budget. of no confidence.
Statement 2 is correct. The Constitution (under article Statement 1 is correct. There is no mention of a No-
114) states that ‘no money shall be withdrawn from the Confidence Motion in the Constitution of India. However,
Consolidated Fund of India except under appropriation Rule 198 of the Rules of Procedure and conduct of Lok Sabha
made by law’. Accordingly, an appropriation bill is introduced specifies the procedure for moving a no-confidence motion.
in the parliament to provide for the appropriation, out of
Statement 2 is correct. A Council of Ministers is collectively
the Consolidated Fund of India. It should be noted in this
responsible to Lok Sabha and it remains in office till it enjoys
context that the voting of demands for grants is the exclusive
confidence of majority of the members in Lok Sabha. Thus,
privilege of the Lok Sabha, that is, the Rajya Sabha has no
power of voting the demands. A no-confidence motion can be moved only in the Lok Sabha
(or state assembly as the case may be).
Statement 3 is incorrect. Provident Fund deposits, savings
bank deposits, remittances etc. are credited to the Public A no-confidence motion can be moved by any member of
Account and this account is operated by the executive the House.
action i.e., payment from this account can be made without 22. Solution: (c)
Parliamentary appropriation.
Exp) Option c is the correct answer.
19. Solution: (b) Parliamentary democracy envisages a cabinet form of
Exp) Option b is the correct answer. government with a President as a titular head (and a
The committee on Estimates is the largest committee of the governor in the states) with responsibility of ministers to the
Parliament. It has total 30 members. Parliament.
The origin of this committee can be traced to the standing One of the key features of Parliamentary System of
financial committee set up in 1921. The first Estimates Government is that the Executive is responsible to the
Committee in the post-independence era was constituted Legislature. Thus, Under Article 75(3), In India the Council
in 1950 on the recommendation of John Mathai, the then of Ministers is collectively responsible to the House of People
finance minister. Originally, it had 25 members but in 1956 (Lok Sabha). The Council of Ministers remains in office as
its membership was raised to 30. All the thirty members are long as they enjoy the support and confidence of the Lok
from Lok Sabha only. The Rajya Sabha has no representation Sabha.
in this committee. These members are elected by the Lok The features of parliamentary government in India are:
Sabha every year from amongst its own members, according
(a) Presence of nominal and real executives;
to the principles of proportional representation by means of
a single transferable vote. The function of the committee is (b) Majority party rule,
to examine the estimates included in the budget and suggest (c) Collective responsibility of the executive to the
‘economies’ in public expenditure. Hence, it has been legislature,
described as a ‘continuous economy committee’. (d) Membership of the ministers in the legislature,
20. Solution: (b) (e) Leadership of the Prime Minister or the Chief Minister,
Exp) Option b is the correct answer. (f ) Dissolution of the lower House (Lok Sabha or Assembly).
The Constitution of India provides for a parliamentary
system of government modelled on the British pattern. Thus, 23. Solution: (d)
the council of ministers headed by the prime minister is Exp) Option d is the correct answer.
the real executive authority is our politico-administrative When the annual Union Budget is not passed by the Lok
system. Sabha, the Prime Minister submits the resignation of Council
Statement 1 is incorrect. Article 75 (3) of the Constitution of Ministers. The annual budget is a money bill and if the
states that the Council of Ministers shall be collectively budget is not passed in parliament, then it can be understood
responsible to the House of the People (Lok Sabha) and not that the ruling party has lost its majority in house. It means
parliament as a whole. the government has lost the confidence vote in the Lok Sabha
Statement 2 is correct. According to Article 75 (2), the and it has to resign.
Minister shall hold office during the pleasure of the President.
24. Solution: (a)
Statement 3 is correct. Article 78 (a) states that it shall
be the duty of the Prime Minister to communicate to the Exp) Option a is the correct answer.
President all decisions of the council of Ministers relating to A deadlock between the LokSabha and the RajyaSabha calls
the administration of the affairs of the union and proposals for a joint sitting of the Parliament during the passage of
for legislation. only ordinary legislation in the above situation.

219 PYQ Workbook


INDIAN POLITY AND GOVERNANCE

Joint sitting is an extraordinary machinery provided by the form set out for the purpose in the Third Schedule;
Constitution to resolve a deadlock between the two Houses • in the case of a seat in the Legislative Assembly, not less
over the passage of a bill. It must be noted here that the than twenty-five years of age and, in the case of a seat
provision of joint sitting is applicable to ordinary bills or in the Legislative Council, not less than thirty years of
financial bills only and not to money bills or Constitutional age; and
amendment bills. In the case of a money bill, the Lok Sabha
• possesses such other qualifications as may be prescribed
has overriding powers, while a Constitutional amendment
in that behalf by or under any law made by Parliament
bill must be passed by each House separately.
25. Solution (b) 29. Solution (a)

Exp) Option b is the correct answer. Exp) Option a is the correct answer

The Department of Economic Affairs (DEA) is responsible The chairman of the committee on Public Accounts is
for preparation and presentation to the Parliament of the appointed from amongst its members by the Speaker of Lok
Central Budget and the Budgets for the State Governments Sabha. The members are elected by the Parliament every
under President’s Rule and Union Territory Administration. year from amongst its members according to the principle
of proportional representation by means of the single
26. Solution (b) transferable vote. At present, it consists of 22 members (15
Exp) Option b is the correct answer. from the Lok Sabha and 7 from the Rajya Sabha).
Option a is correct: Parliament does not have any role in Important Tips
the creation of the budget. The Department of Economic
• Public Accounts Committee was set up first in 1921
Affairs (DEA) of Ministry of Finance is responsible for
under the provisions of the Government of India Act of
preparation and presentation Budget.
1919 and has since been in exist.
Option b is incorrect: Parliament has no power to vote
• A minister cannot be elected as a member of the
on the charged expenditure on the Consolidated Fund.
committee.
Expenditure charged upon the Consolidated Fund of India
can only be discussed in Parliament and It is not subject to • Since 1967 a convention has
the vote of Parliament. • developed whereby the chairman of the committee is
Option c is correct: No money bill imposing tax shall be selected
introduced in the Parliament except on the recommendation • invariably from the Opposition.
of the President, and such a bill shall not be introduced in
30. Solution (d)
the Rajya Sabha.
Exp) Option d is the correct answer.
Option d is correct: Parliament can reduce or abolish a tax
but cannot increase it. Statement 1 is correct: The joint sitting of both Houses of
the Parliament in India is sanctioned under Article 108 of
27. Solution (b) the Constitution.
Exp) Option b is the correct answer. Statement 2 is correct: The first joint sitting was held on 6
According to the Article 93 of the Indian Constitution, the May 1961 following a disagreement between the two Houses
Lok Sabha Speaker is chosen by the members of the Lok over certain amendments to the Dowry Prohibition Bill,
Sabha. The Speaker is the presiding officer of the Lok Sabha 1959. This was followed by another sitting on 9 May 1961
and is responsible for maintaining order and discipline in when the Bill, as amended, was finally passed.
the House. The Speaker also has the power to decide whether Statement 3 is correct: The second joint sitting was held f
a bill is a money bill or not. or the Banking Service Commission (Repeal) Bill, 1977. And
28. Solution (b) last joint sitting was held for the Prevention of Terrorism
Bill, 2002.
Exp) Option b is the correct answer.
Statement 1 is incorrect: According to Article 170 of the 31. Solution (c)
Constitution of India, the Legislative Assembly of each State Exp) Option c is the correct answer.
shall consist of not more than five hundred (500), and The 7th and 31st Constitutional Amendments raised the
not less than sixty, members chosen by direct election from number of Members of Lok Sabha to be elected from the
territorial constituencies in the State. States. The 7th Amendment increased the number of seats
Statement 2 is correct: According to Article 173 of the from 525 to 545, and the 31st Amendment increased the
Constitution of India, a person shall not be qualified to be number of seats to 552.
chosen to fill a seat in the Legislature of a State unless he or
she: 32. Solution (d)
• is a citizen of India, and makes and subscribes before Exp) Option d is the correct answer
some person authorised in that behalf by the Election Committees on Public Accounts and Public Undertakings
Commission an oath or affirmation according to the consists of 22 members (15 from the Lok Sabha and 7 from

PYQ Workbook 220


INDIAN POLITY AND GOVERNANCE

the Rajya Sabha). The Estimates Committee has currently 30 President’s Rule:
members and all the thirty members are from Lok Sabha.
• President’s Rule can be imposed in a state or Union
The Ministry of Parliamentary Affairs works under the
Territory when there is a breakdown of constitutional
overall direction of the Cabinet Committee on Parliamentary
machinery.
Affairs. The Minister of Parliamentary Affairs nominates
members of Parliament on Committees, Councils, Boards, • Under President’s Rule, the elected government of
and Commissions, etc. set up by the Government of India in the state or Union Territory is dissolved, and the
various ministries. governance comes under the direct control of the
President.
Important Tips • The President has the power to extend the term of the
Estimates Committee Lok Sabha for up to one year during President’s Rule.
• It has 30 members and all these members are from Lok Financial Emergency:
Sabha (Lower House). • A Financial Emergency can be proclaimed when there
• It is constituted for the purpose of scrutinizing the is a threat to the financial stability or credit of India
functioning of Government Ministries and departments or any part of its territory.
in terms of allocation of Funds. • The President can declare a Financial Emergency
• All the recommendations made by the committee based on the advice of the Council of Ministers.
are advisory in nature and stand non-binding for the • Similar to a National Emergency, the term of the Lok
Parliament. Sabha can be extended beyond the normal five-year
period during a Financial Emergency, for up to one
Public Accounts Committee
year at a time.
• It is formed every year with not more than 22 members.
• Out of 22 members, 15 members are from Lok Sabha 34. Solution (d)
and 7 members are from Rajya Sabha. Exp) Option d is the correct answer.
• None of the members of this committee are allowed to The Fourth Schedule of the Constitution of India allocates
be ministers in the Government. seats in the Council of States, which is the Rajya Sabha.
• The chairperson of this committee is appointed by Lok It outlines the distribution of seats among the states and
Sabha Speaker. union territories based on factors like population, ensuring
proportional representation.
Committee on Public Undertakings
• It also consists of 22 members from both the houses of 35. Solution (a)
the Parliament, 15 from Lok Sabha and 7 from Rajya Exp) Option a is the correct answer
Sabha. The Indian Parliament exercise control over the
• It is constituted by the Parliament of India for the administration through Parliamentary Committees.
purpose of examining the reports and accounts of the Parliamentary committees are of two kinds–Standing
Public Sector Undertakings. Committees and Ad Hoc Committees. The former are
permanent (constituted every year or periodically) and work
33. Solution (c)
on a continuous basis, while the latter are temporary and
Exp) Option c is the correct answer. cease to exist on completion of the task assigned to them.
The term of the Lok Sabha, under normal circumstances,
is five years from the date of its first meeting. However, 36. Solution (d)
during the proclamation of an emergency under Article 352 Exp) Option d is the correct answer.
of the Constitution of India, the term of the Lok Sabha can The speaker of the Lok sabha can ask a member of the house
be extended. This extension can be done by one year at a to stop speaking and let another member speak. It is known
time, allowing for the continuation of the Lok Sabha beyond as yielding the floor.
its regular five-year term. However, the extension cannot
continue beyond six months once the emergency has ceased Important Tips
to operate or has been revoked. Crossing the floor: Passing between the member
addressing the House and the Chair which is considered
Important Tips
breach of Parliamentary etiquette.
Proclamation of National Emergency:
Interpellation: The formal request made by a member of
• Under Article 352 of the Indian Constitution, the
the parliament to the government.
President has the authority to issue a proclamation of
National Emergency. Decorum: Generally, it is expected that a member must
• This can be done on the grounds of war, external maintain decorum of the Chair and the House. Presiding
aggression, or armed rebellion. officers maintain discipline and decorum in the house and
• The President can declare emergency rule for the entire can punish a member for unruly behaviour with respect to
country or for any specific part of Indian territory. law after suspending them.

221 PYQ Workbook


INDIAN POLITY AND GOVERNANCE

37. Solution (b) the confidence of the majority of the members.


Exp) Option b is the correct answer.
6.2. Other Examination Previous Years’
Article 79 describes the Constitution of Parliament. It Questions
says that there shall be a Parliament for the Union, which
shall consist of the President and two Houses, to be known 41. Solution (c)
respectively as the Council of States and the House of the Exp) Option c is the correct answer.
People.
The Legislative Assembly of Manipur is the unicameral
Important Tips legislature of the Indian state of Manipur. At present, it
Qualifications for Election as President: consists of 60 members, directly elected from 60 single-seat
constituencies. The elections to the Legislative Assembly are
• A person to be eligible for election as President should
held on the basis of adult suffrage, that is, every citizen of
fulfil the following qualifications:
India who is not less than 18 years of age and is not otherwise
• He should be a citizen of India. disqualified can vote.
• He should have completed 35 years of age. • Goa has 40 members in its Legislative Assembly.
• He should be qualified for election as a member of the • Uttarakhand has 70 members in its Legislative Assembly.
Lok Sabha ( should not a member of Lok Sabha).
• Uttar Pradesh has 403 members in its Legislative
• He should not hold any office of profit under the Union Assembly.
government or any state government or any local
authority or any other public authority. 42. Solution (a)
• A sitting President or VicePresident of the Union, the Exp) Option a is the correct answer
Governor of any state and a minister of the Union or The function of the Public Accounts Committee is to
any state is not deemed to hold any office of profit and examine the annual audit reports of the Comptroller and
hence qualified as a presidential candidate. Auditor General of India (CAG), which are laid before
the Parliament by the President. The CAG submits three
38. Solution (d)
audit reports to the President, namely, audit report on
Exp) Option d is the correct answer. appropriation accounts, audit report on finance accounts
Madhya Pradesh is one of the states that does not have a and audit report on public undertakings.
Legislative Council so far even though the Constitution
43. Solution (c)
(Seventh Amendment) Act, 1956 provides for it.
Exp) Option c is the correct answer.
The Constitution (Seventh Amendment) Act, 1956 was one
of the major amendments to the Constitution of India that The Constitution under article 267 authorised the Parliament
reorganized the states and union territories on linguistic and to establish a ‘Contingency Fund of India’, into which
administrative grounds. It also made some changes to the amounts determined by law are paid from time to time.
provisions related to the state legislatures, such as abolishing Accordingly, the Parliament enacted the contingency fund
the classification of Part A, Part B, Part C and Part D states of India Act in 1950.
and introducing the concept of bicameralism in some states. Important Tips
Important Tips The Constitution of India provides for the following three
• Currently, six states in India have a legislative council. kinds of funds for the Central government:
These states are Uttar Pradesh, Andhra Pradesh, Bihar, • Consolidated Fund of India (Article 266)
Maharashtra, Karnataka, and Telangana. • It is a fund to which all receipts are credited and all
payments are debited. It includes
39. Solution (d)
• all revenues received by the Government of India;
Exp) Option d is the correct answer.
• all loans raised by the Government by the issue of
Elected members of the Lower House of the state Legislature
treasury bills, loans or ways and means of advances;
have right to vote in the elections to both the Lok Sabha and
and
the Rajya Sabha. They vote as common voters in the election
of Lok Sabha, whereas, they elect the members of Rajya • all money received by the government in repayment of
Sabha from their respective state loans forms the Consolidated Fund of India.
• All the legally authorised payments on behalf of the
40. Solution (d)
Government of India are made out of this fund.
Exp) Option d is the correct answer.
• No money out of this fund can be appropriated (issued
If the Speaker of the Lok Sabha intends to resign, the or drawn) except in accordance with a parliamentary
correct procedure is to address the letter of resignation law.
to the Deputy Speaker. The Speaker, who is elected by the • Public Account of India (Article 266)
members of the Lok Sabha, holds office as long as they enjoy

PYQ Workbook 222


INDIAN POLITY AND GOVERNANCE

All other public money (other than those which are role in advocating for transparency and accountability in
credited to the Consolidated Fund of India) received by governance and played a crucial role in the campaign for the
or on behalf of the Government of India shall be credited enactment of the RTI Act, 2005.
to the Public Account of India. Mazdoor Kisan Shakti Sangathan (MKSS) is a social
• This includes provident fund deposits, judicial movement in India that works with workers and peasants in
deposits, savings bank deposits, departmental deposits, the villages of Central Rajasthan. It was set up by the people
remittances and so on. of the area in 1990 to strengthen participatory democratic
processes, so that ordinary citizens could live with dignity
• This account is operated by executive action, that is,
and have a say in governance
the payments from this account can be made without
parliamentary appropriation. 46. Solution (c)
• Contingency Fund of India (Article 267) Exp) Option c is the correct answer.
• The Constitution authorised the Parliament to The Council of States, also known as the Rajya Sabha, has
establish a ‘Contingency Fund of India’, into which exclusive powers in relation to the recommendation for the
amounts determined by law are paid from time to time. creation of a new All India Service. All India Services are
Accordingly, the Parliament enacted the contingency services created by the Union (Central) Government to serve
fund of India Act in 1950. both the Central Government and the State Governments.
• This fund is placed at the disposal of the president, These services include the Indian Administrative Service
and he can make advances out of it to meet unforeseen (IAS), the Indian Police Service (IPS), and the Indian Forest
expenditure pending its authorisation by the Service (IFS). The creation of All India Services requires the
Parliament. recommendation and approval of the Rajya Sabha.
• The fund is held by the finance secretary on behalf of Important Tips
the president.
The Council of States (Rajya Sabha) has exclusive
• Like the public account of India, it is also operated by powers in relation to the following:
executive action.
• To recommend the creation of a new All India Service.
44. Solution (d) • To enable the Parliament to make law on a matter of
Exp) Option d is the correct answer. state list.
• Indian Arms Act, 1878: During the 19 centuries, the • To enforce proclamation of emergency when Lok
British came up with the Arms Act to deal with the Sabha is dissolved
growing Indian nationalism. It was enacted in 1878.
Under this act, manufacture and sale of guns in the 47. Solution (b)
country were regulated. No Indian was allowed to Exp) Option b is the correct answer.
manufacture, sell, or even carry a weapon without a The maximum representation from the Union Territories
license from the government. to the Lok Sabha is 20. This is as per Article 81(2) of the
• The Royal Titles Act 1876 was an Act of the Parliament Constitution of India. The number of seats for the Union
of the United Kingdom which officially recognized Territories in the Lok Sabha is determined by the President
Queen Victoria as “Empress of India”. The act was passed of India.
with the understanding that the British imperial should
48. Solution (b)
be used only in India.
Exp) Option b is the correct answer.
• Indian High Courts Act 1861 was an act of the Parliament
of the United Kingdom to authorize the Crown to create A House can declare the seat of a member vacant if he is
High Courts in the Indian colony. Queen Victoria created absent from all its meetings for a period of sixty days without
the High Courts in Calcutta, Madras, and Bombay by its permission. In computing the period of sixty days, no
Letters Patent in 1862. account shall be taken of any period during which the House
is prorogued or adjourned for more than four consecutive
• The Indian Divorce Act, 1869, also known as the Divorce
days.
Act, 1869 was introduced to govern the divorce laws for
Christian couples in India. It deals with the dissolution Important Tips
of marriage between a man and a woman in the Christian In the following cases, a member of Parliament vacates his
community. seat.
45. Solution (b) • Double Membership: A person cannot be a member of
Exp) Option b is the correct answer. both Houses of Parliament at the same time.
Mazdoor Kisan Shakti Sangathan (MKSS) is a social • Disqualification on the grounds of defection:
movement in India that has been associated with the Right Under the provisions of the Tenth Schedule of the
to Information (RTI) Act. MKSS played a significant Constitution.

223 PYQ Workbook


INDIAN POLITY AND GOVERNANCE

• Resignation: A member may resign his seat by writing Individual Privileges


to the Chairman of Rajya Sabha or Speaker of Lok The privileges belonging to the members individually are:
Sabha, as the case may be.
• They cannot be arrested during the session of
• Absence: Discussed above Other cases: A member has Parliament and 40 days before the beginning and
to vacate his seat in the Parliament: (a) if his election is 40 days after the end of a session. This privilege is
declared void by the court; (b) if he is expelled by the available only in civil cases and not in criminal cases or
House; (c) if he is elected to the office of President or preventive detention cases.
Vice-President; and (d) if he is appointed to the office
• They have freedom of speech in Parliament. No member
of governor of a state .
is liable to any proceedings in any court for anything
49. Solution (b) said or any vote given by him in Parliament or its
Exp) Option b is the correct answer. committees. This freedom is subject to the provisions
of the Constitution and to the rules and standing orders
The quorum to constitute a meeting of the Lok Sabha is regulating the procedure of Parliament.
one-tenth of the total members of the House. This means
that there must be at least 55 members present in the Lok • They are exempted from jury service. They can refuse to
Sabha for a meeting to be valid. If the required quorum is not give evidence and appear as a witness in a case pending
met, the Speaker of the Lok Sabha may either adjourn the in a court when Parliament is in session. (Hence, this is
meeting or proceed with the business if the lack of quorum an individual privilege)
is not pointed out by a member. 51. Solution (b)
50. Solution (c) Exp) Option b is the correct answer.
Exp) Option c is the correct answer. A financial bill (II) under article 117 contains provisions
Freedom from attendance as Witness is an individual involving expenditure from the Consolidated Fund of India,
privilege (not a collective privilege). Other than that, but does not include any of the matters mentioned in Article
Freedom of debates and proceedings, the Rights to regulate 110.
the internal matters of the Parliament and the privileges Important Tips
of excluding strangers from the house are the collective The financial bills (I) and (II) have been dealt with in
privilege. Article 117 of the Constitution.
Important Tips Financial Bills (I):
Collective privilege • A financial bill (I) is a bill that contains not only any or
These privileges belonging to each House of Parliament all the matters mentioned in Article 110, but also other
collectively: matters of general legislation.

• It has the right to publish its reports, debates and proceedings • In two respects, a financial bill (I) is similar to a money
and also the right to prohibit others from publishing the same. bill–(a) both of them can be introduced only in the Lok
Sabha and not in the Rajya Sabha, and (b) both of them
• It can exclude strangers from its proceedings and hold can be introduced only on the recommendation of the
secret sittings to discuss some important matters. president.
• It can make rules to regulate its own procedure and • In all other respects, a financial bill (I) is governed
the conduct of its business and to adjudicate upon such by the same legislative procedure applicable to an
matters. ordinary bill.
• It can punish members as well as outsiders for breach of • Hence, it can be either rejected or amended by the
its privileges or its contempt by reprimand, admonition Rajya Sabha.
or imprisonment (also suspension or expulsion, in case • In case of a disagreement between the two Houses over
of members). such a bill, the president can summon a joint sitting of
• It has the right to receive immediate information of the two Houses to resolve the deadlock.
the arrest, detention, conviction, imprisonment and • When the bill is presented to the President, he can
release of a member. either give his assent to the bill or withhold his assent
• It can institute inquiries and order the attendance of to the bill or return the bill for reconsideration of the
witnesses and send for relevant papers and records. Houses.
• The courts are prohibited to inquire into the Financial bill (II):
proceedings of a House or its committees. • It is treated as an ordinary bill and in all respects. The
• No person (either a member or outsider) can be only special feature of this bill is that it cannot be passed
arrested, and no legal process (civil or criminal) can be by either House of Parliament unless the President has
served within the precincts of the House without the recommended to that House the consideration of the
permission of the presiding officer. bill.

PYQ Workbook 224


INDIAN POLITY AND GOVERNANCE

• Hence, financial bill (II) can be introduced in either Important Tips


House of Parliament Decision on questions as to disqualification on ground
• Recommendation of the President is not necessary for of defection:
its introduction. In other words, the recommendation • If any question arises as to whether a member of a
of the President is not required at the introduction House has become subject to disqualification under
stage but is required at the consideration stage. 10th Schedule, the question is referred for the decision
• It can be either rejected or amended by either House of the Chairman or, as the case may be, the Speaker of
of Parliament. such House and his decision should be final.
• In case of a disagreement between the two Houses over
54. Solution (c)
such a bill, the President can summon a joint sitting of
the two Houses to resolve the deadlock. Exp) Option c is the correct answer.

• When the bill is presented to the President, he can either Article 100 of the Constitution of India states that “The
give his assent to the bill or withhold his assent to the Speaker or Chairman, or person acting as such, shall not
bill or return the bill for reconsideration of the Houses. vote in the first instance, but shall have and exercise a casting
vote in the case of an equality of votes to break the tie. The
52. Solution (c) Speaker’s vote is called a casting vote, and it is not counted
Exp) Option c is the correct answer. as a vote in the ordinary sense.
In case the Deputy Speaker is presiding the Lok Sabha, he 55. Solution (b)
has the right to take part in its deliberations, vote on any
Exp) Option b is the correct answer.
proposition before the house as a member, and vote in the
event of equality of votes. He also has the right to preside a Article 248 of the Indian Constitution deals with the
joint sitting of the House of Parliament in case the Speaker residuary powers of legislation. The residuary subjects are
is absent. not mentioned in any of the three lists (Union, State or
Concurrent list). The power to make laws with respect to
Important Tips residuary subjects is vested in the Parliament. This residuary
Few facts about the Deputy Speaker of Lok Sabha: power of legislation includes the power to levy residuary
• The Deputy Speaker is the second-highest-ranking taxes. Vesting of residuary powers in the Centre is taken
officer in the Lok Sabha, the lower house of the from the Constitution of Canada.
Parliament of India.
56. Solution (c)
• The Deputy Speaker is elected by the members of the
Exp) Option c is the correct answer.
Lok Sabha from among themselves
Union Territories in India receive representation in both
• The Deputy Speaker presides over the Lok Sabha in the
Houses of Parliament, namely the Lok Sabha and the
absence of the Speaker.
Rajya Sabha. In the Lok Sabha, representation for Union
• The Deputy Speaker has the same powers and
Territories is achieved through the nomination of members
privileges as the Speaker, except for the power to
by the President of India. Additionally, Union Territories
decide whether a bill is a money bill or not.
have direct representation in the Rajya Sabha, where each
• The Deputy Speaker holds office for a term of five Union Territory is allocated a fixed number of seats.
years, or until the Lok Sabha is dissolved.
57. Solution (d)
• The Deputy Speaker is responsible for maintaining
order and decorum in the Lok Sabha Exp) Option d is the correct answer.
• The Deputy Speaker is responsible for conducting the The authority to prorogue the session of the House of the
business of the Lok Sabha, including the scheduling People, the Lok Sabha, lies with the President. Prorogation
of debates and the voting on bills. is the formal ending of a session. On the advice of the
• The Deputy Speaker can be removed from office by a Prime Minister or the Council of Ministers, the President
resolution of the Lok Sabha passed by a majority of has the authority to conclude the current session of the Lok
all the members of the House. Sabha. Prorogation typically occurs after the completion of
the business of a particular session.
53. Solution (a)
58. Solution (c)
Exp) Option a is the correct answer.
Exp) Option c is the correct answer.
Article 192 of the Indian Constitution states that the
Governor is the final authority to decide any question Namit Sharma v/s Union of India is related to the Right
relating to the disqualification of a Member of a House of to Information Act, 2005. Namit Sharma, a law student,
the Legislature of a State. The Governor’s decision is based filed a Right to Information (RTI) application seeking
on the advice of the Election Commission of India or the information on the appointment of judges in the Allahabad
President of India. High Court. The application was rejected by the Central

225 PYQ Workbook


INDIAN POLITY AND GOVERNANCE

Public Information Officer (CPIO) of the Supreme Court, 62. Solution (d)
citing that the Supreme Court was not a “public authority” Exp) Option d is the correct answer
under the RTI Act. Namit Sharma then approached the
The Parliamentary Committee has not been set up for
Central Information Commission (CIC), which ruled that
Welfare of Minorities. Parliamentary Committee has been
the Supreme Court was indeed a “public authority” under
set up for Public Undertakings, Government Assurances and
the RTI Act.
Estimates.
• Lily Thomas v/s Union of India is a Supreme Court 1. Financial Committees
judgment related to the disqualification of convicted
• Public Accounts Committee
lawmakers.
• Estimates Committee
• Nandini Sundar v/s State of Chhattisgarh is related to
the appointment and arming of Special Police Officers • Committee on Public Undertakings
(SPOs) by the State of Chhattisgarh which was declared 2. Committees to Scrutinize and Control
unconstitutional by the Supreme Court • Committee on Government Assurances

59. Solution (c) • Committee on Subordinate Legislation

Exp) Option c is the correct answer. • Committee on Papers Laid on the Table

Statement 1 is correct- The Rajya Sabha is a permanent • Committee on Welfare of SCs and STs
body and is not subject to dissolution. • Committee on Empowerment of Women
Statement 2 is incorrect- The Rajya Sabha has a term of six • Joint Committee3 on Offices of Profit
years.
63. Solution (b)
Statement 3 is correct- One-third of the members of the
Exp) Option b is the correct answer.
Rajya Sabha retire after every two years. This is done to
A money bill can be introduced in the state legislative
ensure that the Rajya Sabha is always representative of the
assembly only and that too on the recommendation of
states and union territories.
the governor. A money bill is a bill that deals with financial
Statement 4 is incorrect- The members of the Rajya Sabha matters like taxation, public expenditure, borrowing,
must be at least 30 years old. This is to ensure that the and custody of the state’s funds. A money bill cannot be
members of the Rajya Sabha have sufficient maturity and introduced in the legislative council, which is the upper
experience to be able to contribute effectively to the work of house in those states that have a bicameral state legislature.
the Parliament. Every money bill is considered to be a government bill and
60. Solution (d) can be introduced only by a minister.

Exp) Option d is the correct answer. 64. Solution (c)


The Speaker of Lok Sabha presides over a joint sitting of the Exp) Option c is the correct answer.
two Houses and the Deputy Speaker, in his absence. If the The preparation of electoral rolls for the Lok Sabha elections
Deputy Speaker is also absent from a joint sitting, the Deputy lies with the Election Commission of India, an autonomous
Chairman of Rajya Sabha presides. If he is also absent, such constitutional authority. They are responsible for the
other person as may be determined by the members present preparation and revision of electoral rolls, delimitation
at the joint sitting, presides over the meeting. It is clear that of constituencies, and the overall administration of the
the Chairman of Rajya Sabha does not preside over a joint electoral process.
sitting as he is not a member of either House of Parliament.
65. Solution (c)
61. Solution (b) Exp) Option c is the correct answer.
Exp) Option b is the correct answer. The Constitution refers to the budget as the ‘annual financial
When two or more States make a Request, Parliament may statement’. In other words, the term ‘budget’ has nowhere
legislate on any subject of State List. A law so enacted applies been used in the Constitution. It is the popular name for
only to those states which have passed the resolutions. the ‘annual financial statement’ that has been dealt with in
However, any other state may adopt it afterwards by passing Article 112 of the Constitution. The Consolidated Fund of
a resolution to that effect in its legislature. Such a law can be India is mentioned in the article 266. While, article 114 deals
amended or repealed only by the Parliament and not by the with the Appropriation Bill.
legislatures of the concerned states. Some examples of laws
66. Solution (a)
passed under the above provision are Prize Competition Act,
1955; Wild Life (Protection) Act, 1972; Water (Prevention Exp) Option a is the correct answer.
and Control of Pollution) Act, 1974; Urban Land (Ceiling Unlike the Lok Sabha, which is subject to dissolution, the
and Regulation) Act, 1976; and Transplantation of Human Rajya Sabha is a permanent body and is not dissolved. The
Organs Act, 1994. Rajya Sabha is a continuing chamber, and its members have

PYQ Workbook 226


INDIAN POLITY AND GOVERNANCE

a fixed term of six years. One-third of the members retire 70. Solution (d)
every two years, ensuring continuity in the functioning of Exp) Option d is the correct answer.
the house.
Goa is the only state in India to have a Common Civil Code.
67. Solution (c) Unlike the rest of India, which follows different personal
Exp) Option c is the correct answer. laws based on religious affiliations, Goa has a uniform civil
code for all its residents.
Salary and allowances of the Prime Minister of India is not
charged on the Consolidated Fund of India. It is made from Important Tips
the Consolidated Fund of India. • “Goa Civil Code,” was inherited from the Portuguese
colonial era and is applicable to all residents of the
Important Tips
state irrespective of their religion.
The budget consists of two types of expenditure–the
• It covers matters such as marriage, divorce, inher-
expenditure ‘charged’ upon the Consolidated Fund of
itance, adoption and maintenance.
India and the expenditure ‘made’ from the Consolidated
Fund of India. • It has some special provisions for different commu-
nities, such as allowing bigamy to Hindu men under
• The charged expenditure is non-votable by the
certain conditions.
Parliament, that is, it can only be discussed by the
Parliament. • Uniform Civil Code is mentioned in Article 44 of the
Indian Constitution as one of the directive principles
• The expenditure ‘made’ from the Consolidated Fund
of state policy.
of India has to be voted by the Parliament.
Charged Expenditure: Salaries and allowances of the 71. Solution (c)
President; Chairman and the Deputy Chairman of the Exp) Option c is the correct answer
Rajya Sabha and the Speaker and the Deputy Speaker of the
Members of the Rajya Sabha are not associated with the
Lok Sabha; the judges of the Supreme Court; Comptroller
Estimates Committee. It has 30 members and all these
and Auditor General of India; chairman and members of
members are from Lok Sabha (Lower House). Committees
the Union Public Service Commission, etc.
on Public Accounts and Public Undertakings consists of 22
68. Solution (b) members (15 from the Lok Sabha and 7 from the Rajya Sabha).
Exp) Option b is the correct answer. Thus, both the committee has members from Rajya Sabha.

The Anti-Defection Law, also known as the Tenth Schedule 72. Solution (a)
of the Constitution of India, was enacted in 1985 to prevent Exp) Option a is the correct answer.
the “floor crossing” of legislators. Floor crossing is the act
Article 265 of the Indian constitution deals with the taxes
of a legislator switching from one political party to another,
not to be imposed save by authority of law. It means that no
often in order to gain political advantage. The Anti-Defection
tax shall be levied or collected except by the authority of law.
Law prohibits legislators from defecting from their party
unless they have the support of two-thirds of the members 73. Solution (b)
of their party. This means that large defections in a party
Exp) Option b is the correct answer.
in various spells are not covered by the Anti-Defection Law.
During second stage, the bill receives not only the general
69. Solution (c) but also the detailed scrutiny and assumes its final shape.
Exp) Option c is the correct answer. Hence, it forms the most important stage in the enactment
of a bill.
73rd Constitutional Amendment Act, 1992, which came
into force on April 24, 1993, provides for the reservation Important Tips
of not less than one-third of the total number of seats Ordinary Bills:
for women in the Panchayats at all levels, as well as the Every ordinary bill has to pass through the following five
reservation of one-third of the offices of Chairpersons of stages in the Parliament before it finds a place on the
Panchayats at all levels for women. Statute Book
Stage 1 : First Reading
Option a is correct: Hindu Marriage Act: 1955 is an Act
• An ordinary bill can be introduced in either House of
that regulates the marriage and divorce among Hindus, Parliament. Such a bill can be introduced either by a
Buddhists, Jains and Sikhs. minister or by any other member. No discussion on
Option b is correct: Hindu Succession Act: 1956 is an Act the bill takes place at this stage.
that codifies the law relating to intestate succession among Stage 2 : Second Reading
Hindus, Buddhists, Jains and Sikhs. • During this stage, the bill receives not only the general
but also the detailed scrutiny and assumes its final
Option d is correct: Sati (Prevention) Act: 1987 is an Act shape. This stage involves three more sub-stages,
that prohibits the practice of sati, the burning or burying namely, stage of general discussion, committee stage
alive of widows, and its glorification. and consideration stage.

227 PYQ Workbook


INDIAN POLITY AND GOVERNANCE

Stage 3 : Third Reading The functions of the Public Accounts Committee are
• At this stage, the debate is confined to the acceptance to examine the appropriation accounts and the finance
or rejection of the bill as a whole and no amendments accounts of the Union government and any other accounts
are allowed.
laid before the Lok Sabha. It is Financial Committee. The
Stage 4 : Bill in the Second House
• In the second House also, the bill passes through all Committee on Petitions is an Functional Committee. Joint
the three stages, that is, first reading, second reading Committee on Stock Market Scam was an Ad hoc Committee.
and third reading. The committee recommended sweeping changes in stock
Stage 5 : Assent of the President market regulations. Departmental Committees are Standing
• Every bill after being passed by both Houses of Committee. The main objective of the standing committees
Parliament either singly or at a joint sitting, is presented
is to secure more accountability of the Executive (i.e.,
to the president for his assent. A bill becomes an act
after his assent. the Council of Ministers) to the Parliament, particularly
financial accountability.
74. Solution (d)
Important Tips
Exp) Option d is the correct answer.
• Parliamentary committees are of two kinds–Standing
Union Parliament may legislate on any subject of State List Committees and Ad Hoc Committees. The former are
in following cases; (a) To Implement International permanent (constituted every year or periodically)
Agreements; (b) By consent of State; (c) When Proclamation and work on a continuous basis, while the latter are
temporary and cease to exist on completion of the task
of Emergency is in operation; (d) In national interest and assigned to them.
after a resolution passed by Council of states with 2/3 • On the basis of the nature of functions performed by
majority. Hence, all the options are correct. them, standing committees can be classified into the
following six categories
Important Tips
1. Financial Committees
Parliamentary Legislation on subjects in State List :
2. Departmental Standing Committees (24)
The Constitution empowers the Parliament to make laws
3. Committees to Inquire
on any matter enumerated in the State List under the
following five extraordinary circumstances: 4. Committees to Scrutinise and Control

• When Rajya Sabha Passes a Resolution (Art. 249) 5. Committees Relating to the Day-to-Day Business of the
House.
• If the Rajya Sabha proposes with a two-thirds majority
of its members present and voting that it is necessary 6. House-Keeping Committees or Service Committees
or beneficial in the national interest, the Parliament (i.e., Committees concerned with the Provision of
can legislate on any of the subjects listed in the State Facilities and Services to Members):
List. Ad hoc committees can be divided into two categories, that
• During a National Emergency (Art. 250) is, Inquiry Committees and Advisory Committees.
• It provides power to Parliament to legislate with respect 1. Inquiry Committees are constituted from time to time,
to any matters in the State List if a Proclamation of either by the two Houses on a motion adopted in that
Emergency is in operation. behalf, or by the Speaker / Chairman, to inquire into
• When States Make a Request: (Art.252) and report on specific subjects. For example:
• It provides power to Parliament to legislate for two or 2. Advisory Committees include select or joint
more States, when respective State legislatures pass a committees on bills, which are appointed to consider
resolution in this regard. and report on particular bills. These committees are
• To Implement International Agreements (Art. 253) distinguishable from the other ad hoc committees
• The Parliament can make laws on any matter in the in as much as they are concerned with bills and the
State List for implementing the international treaties, procedure to be followed by them is laid down in the
agreements or conventions. This provision enables Rules of Procedure and the Directions by the Speaker /
the Central government to fulfil its international Chairman
obligations and commitments.
76. Solution (c)
• During President’s Rule (Art. 356)
• When the President’s rule is imposed in a state, the Exp) Option c is the correct answer.
Parliament becomes empowered to make laws with Madhya Pradesh has the highest number of reserved seats for
respect to any matter in the State List in relation to
the Scheduled Tribes in the Lok Sabha. There are currently
that state.
29 seats reserved for Scheduled Tribes in Madhya Pradesh.
75. Solution (c) This is followed by Gujarat with 26 seats, Maharashtra with
Exp) Option c is the correct answer 25 seats, and Odisha with 21 seats.

PYQ Workbook 228


INDIAN POLITY AND GOVERNANCE

Important Tips Statement 4 is incorrect- The power to create All India


Services, such as the Indian Administrative Service (IAS)
State Total Total Seats reserved
and the Indian Police Service (IPS), is not exclusive to the
Seats for Scheduled Tribes.
Lok Sabha. It is a shared power between the Union (Central)
Chattisgarh 11 4
Government and the State Governments.
Odisha 21 5
Madhya Pradesh 29 6 Important Tips
Jharkhand 14 5 • Money Bills are specifically defined in the Indian
Rajasthan 25 4 Constitution under Article 110 and pertain to
taxation, government expenditure, borrowing, etc.
Maharashtra 48 5
Money Bills can only be introduced in the Lok Sabha,
Gujarat 26 4 the lower house of Parliament.
• Finance Bills have a broader scope and can be
77. Solution (b)
introduced in either the Lok Sabha or the Rajya
Exp) Option b is the correct answer. Sabha (upper house), but they require the President’s
The ordinary bill passed by the State Assembly can be recommendation for introduction in the Rajya Sabha.
delayed by the Legislative Council for a maximum period of
four months. 79. Solution (a)

• If the bill is passed by the Legislative Assembly and sent Exp) Option a is the correct answer.
to the Legislative Council, the latter can either pass it, The Economic Survey is presented in both the houses of
reject it, or not take any action on it within three months. Parliament, a day before the announcement of the Union
If the Legislative Council rejects the bill or does not pass
Budget. The Economic Survey is prepared under the guidance
it within three months or passes it with amendments
not acceptable to the Legislative Assembly, the bill is of the Chief Economic Advisor.
considered again by the Legislative Assembly, which can
80. Solution (a)
pass it again with or without amendments and send it
back to the Legislative Council. Exp) Option a is the correct answer.
• The Legislative Council then has to pass the bill within Zero Hour is an informal device available to the members
one month or it is deemed to have been passed by of the Parliament to raise matters without any prior notice.
both houses in the form in which it was passed by the The zero hour starts immediately after the question hour
Legislative Assembly.
and lasts until the agenda for the day (ie, regular business
• Therefore, the final power of passing an ordinary bill lies
of the House) is taken up. In other words, the time gap
with the Legislative Assembly and the Legislative Council
between the question hour and the agenda is known as zero
can only delay it for four months at most
hour. It is an Indian innovation in the field of parliamentary
78. Solution (b) procedures and has been in existence since 1962. Unlike the
Exp) Option b is the correct answer. question hour, the zero hour is not mentioned in the Rules
Statement 1 is correct- As per Article 109 of the Indian of Procedure.
Constitution, a Money Bill can only be introduced in the
Lok Sabha and not in the Rajya Sabha. The Rajya Sabha can 81. Solution (d)
make recommendations on the Money Bill, but it cannot Exp) Option d is the correct answer.
initiate or introduce such bills.
The sitting of the House of the People (Lok Sabha) can
Statement 2 is correct- The Rajya Sabha does not have the
be terminated through adjournment, prorogation, and
power to reject or amend a Money Bill. It can only make
recommendations to the Lok Sabha, which may or may not dissolution. Adjournment is done by the Speaker of the
accept those recommendations. Ultimately, it is the Lok Lok Sabha, while prorogation and dissolution are carried
Sabha that has the final authority over the Money Bill. out by the President of India, as prescribed in Article 85(2)
Statement 3 is correct- The Lok Sabha has the exclusive of the Constitution.
power to hold the Council of Ministers accountable. Note- The adjournment of the House means the suspension
Members of the Lok Sabha can question the ministers,
of the sitting of the House till the following or some later day
participate in debates, and pass resolutions to express their
dissatisfaction or lack of confidence in the government. The or hour of time. Adjournment sine die means termination
Rajya Sabha does not have this power to hold the Council of of the sitting of the House without specifying or fixing any
Ministers directly accountable. definite date for its next sitting.

229 PYQ Workbook


INDIAN POLITY AND GOVERNANCE

Important Tips 85. Solution (c)

• Adjournment refers to the temporary suspension of Exp) Option c is the correct answer.
the sitting of the House. The House can be adjourned Contingency Fund is placed at the disposal of the president,
sine die, which means that it is adjourned for an and he can make advances out of it to meet unforeseen
indefinite period of time, or it can be adjourned to a expenditure pending its authorisation by the Parliament.
specific date. Hence, President can spend from Contingency fund
• Prorogation refers to the termination of the sitting even before authorization of Parliament. But it shall be
of the House for a fixed period of time. The House is authorised by the Parliament later on.
prorogued by the President on the advice of the Prime 86. Solution (a)
Minister.
Exp) Option a is the correct answer.
• Dissolution refers to the termination of the House
of the People and the calling of fresh elections. The In the Rajya Sabha elections, members of a state’s Legislative
House of the People can be dissolved by the President Assembly vote using the proportional representation with
on the advice of the Prime Minister the single transferable vote (STV) system. Each MLA’s vote
is counted once. To win a Rajya Sabha seat, a candidate
82. Solution (d) must obtain a required number of votes, which is calculated
Exp) Option d is the correct answer. using the formula: Required vote = Total number of votes /
(Number of Rajya Sabha seats + 1) + 1.
Consolidated Fund of India is a fund to which all receipts
are credited and all payments are debited. No money out 87. Solution (b)
of this fund can be appropriated (issued or drawn) except Exp) Option b is the correct answer
in accordance with a parliamentary law. Hence, Parliament The Committee on Subordinate Legislation was constituted
has the right to issue money from the Consolidated Fund of in 1953 examines and reports to the House whether the
India. powers to make rules, regulations, bye laws, schemes, or
other statutory instruments conferred by the Constitution or
83. Solution (a) delegated by Parliament are being properly exercised.
Exp) Option a is the correct answer.
Important Tips
There is no reservation of seats for Scheduled Tribes in the • In both the Houses, the Committee on Subordinate
Lok Sabha in the states of Kerala and Tamil Nadu. This is Legislation consists of 15 members.
because these states have a very small tribal population. The • The Chairman of the Committee shall be appointed
Scheduled Tribes constitute only 1.5% of the population of by the Chairman from amongst the members of the
Kerala and 1.2% of the population of Tamil Nadu. Committee.

84. Solution (a) • In order to constitute a meeting of the Committee, the


quorum shall be five.\
Exp) Option a is the correct answer.
• The Committee shall have the power to require the
The MISA Act, or the Maintenance of Internal Security attendance of persons or the production of papers or
Act, was a controversial law passed by the Indian parliament records if such a course is considered necessary for the
in 1971 that gave the administration of Prime Minister discharge of its duties.
Indira Gandhi and Indian law enforcement agencies very
88. Solution (e)
broad powers, including indefinite preventive detention of
Exp) Option e is the correct answer.
individuals, search and seizure of property without warrants,
and wiretapping. It was not a social act. Article 110 of the Constitution deals with the definition of
money bills. It states that a bill is deemed to be a money bill
The other options are all social acts: if it contains ‘only’ provisions dealing with all or any of the
• The Anti-Dowry Act refers to the Dowry Prohibition following matters:
Act, which was enacted in 1961 to prohibit the giving or (a) The imposition, abolition, remission, alteration or
taking of dowry. regulation of any tax; (b) The regulation of the borrowing
• The Protection of Civil Rights Act was enacted in 1955 of money by the Union government; (c) The custody of
the Consolidated Fund of India or the contingency fund
to prescribe punishment for the preaching and practice
of India, the payment of moneys into or the withdrawal of
of “Untouchability” and for the enforcement of any
money from any such fund; (d) The appropriation of money
disability arising therefrom.
out of the Consolidated Fund of India; (e) Declaration of
• The Prevention of Immoral Traffic Act refers to the any expenditure charged on the Consolidated Fund of India
Immoral Traffic (Prevention) Act, which was enacted in or increasing the amount of any such expenditure; (f ) The
1956 to provide for the prevention of immoral traffic receipt of money on account of the Consolidated Fund of

PYQ Workbook 230


INDIAN POLITY AND GOVERNANCE

India or the public account of India or the custody or issue Important Tips
of such money, or the audit of the accounts of the Union or
Minimum age required for other elections in India:
of a state. Since, more than one statement are correct. Hence,
option e is the correct answer. • President of India: 35 years
• Vice President of India: 35 years
89. Solution (a)
• Member of Rajya Sabha (Upper House): 30 years
Exp) Option a is the correct answer.
• Member of Legislative Assembly (State Assembly): 25
Article 330 of the Constitution of India provides for years
reservation of seats for Scheduled Castes in the Lok Sabha.
• Member of Legislative Council (State Legislative
The number of seats reserved for Scheduled Castes in the
Council): 30 years
Lok Sabha is proportional to the population of Scheduled
Castes in India. • Panchayat Samiti (Block/Intermediary level): 21
years
Important Tips • Gram Panchayat (Village level): 21 years
• The 104th Constitutional Amendment Act extended • Municipal Corporation: 21 years
the reservation of seats for Scheduled Castes (SCs)
and Scheduled Tribes (STs) in the Lok Sabha and state • Municipal Council/Nagar Panchayat: 21 years
assemblies. • Mayor of Municipal Corporation: 25 years
• The 104th Amendment Act removed the reserved 93. Solution (d)
seats for the Anglo-Indian community in the Lok
Exp) Option d is the correct answer.
Sabha and state assemblies.
• Article 331: President’s power to nominate two The Lok Sabha can be dissolved before completing its term
members from the Anglo-Indian community to the by the President on the recommendation of the Prime
Lok Sabha. Minister. As per Article 85 of the Constitution of India,
the Lok Sabha has a fixed term of five years, but it can be
• Article 332: Reservation of seats for Scheduled Castes
dissolved earlier if the Prime Minister advises the President
and Scheduled Tribes in the legislative assemblies of
the states. to do so. This typically occurs in situations where there is a
breakdown of the government, loss of majority, or other
• Article 333: Representation of the Anglo-Indian
circumstances that warrant the dissolution of the Lok Sabha.
community in the legislative assemblies of the states.
94. Solution (d)
90. Solution (c)
Exp) Option d is the correct answer .
Exp) Option c is the correct answer.
As per the Article 168 of the Indian Constitution, the
According to Article 80, the Rajya Sabha consists of a
legislature of a state consists of the Governor and either
maximum of 250 members. Out of these 250 members,
one or two houses, depending on the state. The one-house
238 members are elected by the elected members of the
Legislative Assemblies of the states and union territories. legislature is called the Legislative Assembly or Vidhan Sabha,
The remaining 12 members are nominated by the President and the two-house legislature consists of the Legislative
of India, based on their expertise in various fields like Assembly and the Legislative Council or Vidhan Parishad
literature, science, art, and social service. 95. Solution (d)
91. Solution (b) Exp) Option d is the correct answer.
Exp) Option b is the correct answer According to the section 34(1) of the Representation of
Public Accounts Committee submits its report to the Lok the People Act, 1951 states that “Where a candidate fails to
Sabha Speaker. Public Accounts Committee’s main function secure 1/6 of the total number of votes polled at the election,
is to examine the audit report of Comptroller and Auditor his or her security deposit shall be forfeited.”
General (CAG) and submits its report to the parliament. The
96. Solution (c)
Chairman of the committee is appointed by the speaker of its
members of Lok Sabha. Exp) Option c is the correct answer.
According to Article 179 of the Constitution of India,
92. Solution (c) a member holding office as Speaker or Deputy Speaker of
Exp) Option c is the correct answer. an Assembly may at any time by writing under his hand
The minimum age laid down for a person to seek election to addressed, if such member is the Speaker, to the Deputy
the Lok Sabha is 25 years. This is as per Article 84(b) of the Speaker, and if such member is the Deputy Speaker, to the
Constitution of India. The same minimum age also applies Speaker, resign his office. This means that the Speaker of
to candidates seeking election to the Legislative Assemblies the Legislative Assembly of a State submits his resignation
of the states to the Deputy Speaker of the same Assembly.

231 PYQ Workbook


INDIAN POLITY AND GOVERNANCE

97. Solution (a) 101. Solution (c)


Exp) Option a is the correct answer. Exp) Option c is the correct answer.
Article 170 of the Indian Constitution consists of the Under Section 20 of the Right to Information Act, both
provision of the election of Legislative Assemblies of States. the Central Information Commission and the State
It deals with the composition of the Legislative Assemblies, Information Commission have the authority to impose
the allocation of seats, and other related matters. penalties. The Information Commissions at both the central
and state levels are empowered to adjudicate on appeals and
Important Tips complaints related to the implementation of the Act.
• Article 176 deals with the special address by the
They have the power to impose penalties on public information
Governor to the State Legislature.
officers or any other person who has contravened the
• Article 178 deals with the election of the Speaker and provisions of the Act, including withholding information,
Deputy Speaker of the Legislative Assembly. providing false or misleading information, or obstructing
the flow of information. The penalties imposed by the
98. Solution (b)
Information Commissions serve as a deterrent and ensure
Exp) Option b is the correct answer. compliance with the provisions of the Right to Information
The leader of the Lok Sabha is the Prime Minister. The Act.
Prime Minister is the leader of the party that has a majority
102. Solution (c)
in the Lok Sabha. The Prime Minister is responsible for
the formation of the government and for leading the Exp) Option c is the correct answer.
government in the Lok Sabha. According to the provisions of the Scheduled Castes and
Scheduled Tribes (Prevention of Atrocities) Act, 1989, an
Important Tips
appeal can be filed to the High Court from any judgment,
• The Constitution does not provide a specific procedure sentence, or order. This appeal can be made on both factual
for the selection and appointment of the Prime grounds, involving the interpretation of evidence and
Minister. facts of the case, as well as legal grounds, involving the
• Article 75 of the Constitution states that the President interpretation and application of laws. The High Court has
shall appoint the Prime Minister. the authority to review and decide on both the factual and
• As per the conventions of the parliamentary system, legal aspects of the case while considering the appeal.
the President appoints the leader of the majority party
in the Lok Sabha as the Prime Minister. 103. Solution (d)
• If no party has a clear majority in the Lok Sabha, the Exp) Option d is the correct answer.
President may exercise personal discretion in selecting There are currently 84 seats reserved for Scheduled
and appointing the Prime Minister. Castes in the Lok Sabha. This is as per the order issued
by the Delimitation Commission in 2018. The number of
99. Solution (d)
seats reserved for Scheduled Castes is proportional to the
Exp) Option d is the correct answer. population of Scheduled Castes in India.
Provision of joint sitting is applicable to ordinary bills or
Important Tips
financial bills only. When both Houses have different opinion
• Currently, there are 41 seats reserved for Scheduled
on any ordinary bill, joint sitting can be called on for it.
Tribes in the Lok Sabha.
Ordinary bills are those bills which are outside the scope of
Article 368 and can be passed with simple majority. Whereas, • The maximum strength of the Lok Sabha, as envisioned
to make any changes in the election of the President or Vice- by the Constitution, is 552 members.
President, or to pass the Bill relating to the amendment of • The Lok Sabha consists of 543 seats, with 530 members
Constitution, a constitutional amendment bill with special representing the states through by-elections.
majority needs to be passed. But for any bill under article • Approximately 24% of all Lok Sabha seats, out of
368, there is no provision of joint sitting. the total 543, are reserved for Scheduled Castes and
Scheduled Tribes, with 84 seats reserved for Scheduled
100. Solution (b)
Castes and 47 seats reserved for Scheduled Tribes.
Exp) Option b is the correct answer.
104. Solution (b)
The representation of states in the Council of States,
also known as the Rajya Sabha, is based on the ratio of Exp) Option b is the correct answer.
their population. This is determined by the principle of The power to make rules for carrying out the purposes of the
“proportional representation” in which the number of seats Scheduled Castes and the Scheduled Tribes (Prevention of
allocated to each state is proportionate to its population. On Atrocities) Act, 1989 is vested in the Central Government.
the other hand, in USA, every state has equal representation The Scheduled Castes and Tribes (Prevention of Atrocities)
in Senate or the Upper house. Act, 1989 is an Act of the Parliament of India enacted to

PYQ Workbook 232


INDIAN POLITY AND GOVERNANCE

prohibit discrimination, prevent atrocities and hate crimes Important Tips


against scheduled castes and scheduled tribes. The Act is
popularly known as the SC/ST Act, POA, the Prevention of Some important salient features of the Scheduled castes
Atrocities Act, or simply the Atrocities Act and scheduled tribes (prevention of atrocities) Act,
1989, are following: -
105. Solution (b) • Denial of Anticipatory bail (Section 18).
Exp) Option b is the correct answer. • Grant arms licenses to STs and SCs (Rule 3v).
According to section 15 of the Protection of civil rights • Section 19 - Denial of Probation to convict.
act, 1955: Every offence punishable under this Act shall
be cognizable and every such offence, except where it • Creation of Special courts (Section 14).
is punishable with imprisonment for a minimum term • Appointment of Special Public prosecutors (Section
exceeding three months, maybe tried summarily by a 15).
Judicial Magistrate of the first class or in a metropolitan • Creation of new types of offenses not in the Protection
area by a Metropolitan Magistrate in accordance with the of Civil Rights Act 1955 or Indian Penal Code.
procedure specified in the said Code.
• Empowers the Government to impose collective fines
106. Solution (b) (Section 16).
Exp) Option b is the correct answer. • Punishment for neglect of Duties by a Public servant
The State Government is empowered for imposition and (Section 4).
realization of a collective fine under the Scheduled Castes 109. Solution (b)
and Scheduled Tribes (Prevention of Atrocities) Act, 1989
Exp) Option b is the correct answer.
In 1989, the Government of India enacted the Scheduled
Castes and Tribes (Prevention of Atrocities) Act in order to The State Government can specify a Court of Session to be a
prevent atrocities against SC/STs. The purpose of the Act was Special Court to try the offences under the Scheduled Castes
to prevent atrocities and help in social inclusion of Dalits and Scheduled Tribes (Prevention of Atrocities) Act, 1989
into the society with the concurrence of the Chief Justice of High Court
107. Solution (b) This ensures the involvement of the judiciary in designating
Exp) Option b is the correct answer. a specific court to handle cases related to atrocities against
Scheduled Castes and Scheduled Tribes.
Article 171 of the Indian Constitution provides for the
composition of the Legislative Council, also known as 110. Solution (a)
Vidhan Parishad, in the States of India. This article lays down
Exp) Option a is the correct answer.
the provisions related to the establishment, membership,
and other related matters of the Legislative Council in states The section 3 of the Protection of Civil Rights Act, 1955,
where such a council exists. The Legislative Council is the which replaced the Untouchability (Offences) Act, 1955,
upper house of the state legislature, and its members are states that a court can presume that any act constituting
partially elected and partially nominated. an offence under this Act was committed on the ground of
Important Tips “untouchability” if such act is committed in relation to a
• Article 170 of the Indian Constitution deals with the member of a Scheduled Caste. A Scheduled Caste is a group
composition of the Legislative Assemblies of the States. of historically disadvantaged people who are recognized by
• Article 172 of the Indian Constitution pertains to the the Constitution of India as eligible for special protection
duration of the State Legislative Assemblies. It states and assistance.
that the Legislative Assembly of a State shall continue
111. Solution (c)
for five years from the date of its first meeting, unless
dissolved earlier. Exp) Option c is the correct answer.
• Article 173 of the Indian Constitution deals with the The Scheduled Castes and Scheduled Tribes (Prevention of
qualifications for membership of the State Legislature. Atrocities) Act is a special provision for the advancement
It sets certain criteria that a person must meet to be of Scheduled Castes and Scheduled Tribes in India. It
eligible to be a member of the Legislative Assembly of is based on the doctrine of protective discrimination,
a State.
which recognizes the historically disadvantaged position
108. Solution (a) of these communities and seeks to provide them with
Exp) Option a is the correct answer. specific safeguards and protections against discrimination,
Under the Scheduled Castes and Scheduled Tribes atrocities, and social injustices. The Act aims to prevent and
(Prevention of Atrocities) Act, 1989, Anticipatory Bail is address atrocities committed against Scheduled Castes and
barred. This means that a person accused of committing an Scheduled Tribes and ensure their social, educational, and
offence under this Act cannot apply for anticipatory bail. economic empowerment.

233 PYQ Workbook


INDIAN POLITY AND GOVERNANCE

Important Tips union territories of India, including Jammu and Kashmir,


Goa, Daman and Diu, and the scheduled areas.
• Doctrine of Severability: The doctrine of severability
refers to the principle that if a provision of a law or Important Tips
statute is found to be unconstitutional, it can be The offences punishable under Protection of Civil
separated or “severed” from the rest of the law, allowing Rights Act are:
the remaining valid provisions to remain in effect.
• Enforcing religious disabilities, such as preventing a
• Doctrine of Preferred Position: The doctrine of person from entering a place of worship or performing
preferred position refers to the principle that certain a religious service on the ground of untouchability.
fundamental rights or constitutional provisions hold a
• Enforcing social disabilities, such as denying access
higher or preferred position and should be interpreted
to a shop, public restaurant, hotel, place of public
and protected more extensively than other rights.
entertainment, public conveyance, or any occupation,
• Doctrine of Harmonious Construction: The doctrine trade or business on the ground of untouchability.
of harmonious construction is a principle of statutory
• Refusing to admit a person to a hospital, dispensary,
interpretation that aims to reconcile and harmonize
educational institution, or any other public service on
different provisions of a law to give effect to the overall
the ground of untouchability.
legislative intent.
• Refusing to sell goods or render services to a person on
112. Solution (a) the ground of untouchability.
Exp) Option a is the correct answer. • Other offences arising out of untouchability, such as
insulting, intimidating, assaulting, or boycotting a
Ganesh Vasudev Mavalankar was the first Speaker of the
person on the ground of untouchability.
Lok Sabha. He was elected on 17 April 1952, the day the
Lok Sabha was constituted after India’s first general election. • Unlawful compulsory labour, such as forcing a person
to do any work which is an offence under the Indian
He served as Speaker until his death on 27 February 1956.
Penal Code on the ground of untouchability.
Mavalankar was a respected leader of the Indian National
Congress and a veteran of the Indian independence 115. Solution (b)
movement. He was known for his impartiality and his ability
Exp) Option b is the correct answer.
to maintain order in the Lok Sabha. He was also a strong
advocate of parliamentary democracy and the rule of law. Statement 1 is correct- According to Article 84 of the
Indian Constitution, a person must be at least 30 years of
Important Tips age to be eligible for membership in the Rajya Sabha.
First person to hold constitutional posts: Statement 2 is incorrect- The requirement of being registered
• President of India: Dr. Rajendra Prasad as a voter in the state from which the candidate is to be elected
• Vice President of India: Dr. Sarvepalli Radhakrishnan applies to the Lok Sabha (the lower house of Parliament) and
not the Rajya Sabha. Rajya Sabha members are elected by
• Prime Minister of India: Jawaharlal Nehru
the elected members of the Legislative Assembly of the
• Chief Justice of India: Harilal Jekisundas Kania state, and voter registration is not a qualification criterion
• Governor-General of India: Lord Louis Mountbatten for Rajya Sabha membership.
(prior to India becoming a republic) Statement 3 is correct- According to Article 102 of the Indian
113. Solution (b) Constitution, a person cannot be a member of Parliament if
they hold any office of profit under the government of India
Exp) Option b is the correct answer.
or the government of any state. Holding an office of profit
The Protection of Civil Rights Act, 1955 is an important law would result in disqualification from being a member of the
that aims to prevent discrimination and oppression based on
Rajya Sabha.
caste and untouchability in India. The act specifies that all
offences under it are cognizable and non-compoundable. 116. Solution (a)
This means that the police can arrest the accused without Exp) Option a is the correct answer.
a warrant and that the accused cannot be released on bail
without the permission of the court. The Governor is the authority who prorogues the Vidhan
Sabha of a State. Prorogation is the act of terminating a
The act was amended in 1976 to enhance the penalties
session of the Vidhan Sabha by an order of the Governor.
for the offences and to make them summarily triable by a
magistrate. This means that the magistrate can try the case Important Tips
quickly and without following the formal procedures of a • Prorogation ends both the sitting and the session of the
regular trial House.
114. Solution (a) • Usually, within a few days after the House is adjourned
Exp) Option a is the correct answer. sine die (without fixing a date for reassembly) by
the Speaker, the Governor issues a notification for
The Protection of Civil Rights Act extends to the whole of
India. This means that the act applies to all the states and prorogation of the session.

PYQ Workbook 234


INDIAN POLITY AND GOVERNANCE

• Prorogation does not affect the bills or motions pending 121. Solution (a)
in the House, which can be taken up in the next session Exp) Option a is the correct answer.
The Governor of the State can make advances from the
117. Solution (b)
Contingency Fund of the State, which is established
Exp) Option b is the correct answer. under Article 267 (2) of the Constitution, to meet urgent
The Speaker of the Lok Sabha can be removed by a resolution unforeseen expenditure, pending authorisation by the State
passed by a majority of all the members of the House. This Legislature. The corpus of the fund varies across states and
means that a simple majority of all the members present and the quantum is decided by the State legislatures.
voting in the Lok Sabha is required to pass the resolution for
Important Tips
the removal of the Speaker. The procedure for the removal
of the Speaker is governed by the rules and procedures of • The Contingency Fund of India is a fund that is placed
the Lok Sabha. at the disposal of the President of India, who can make
advances out of it to meet unforeseen expenditure
118. Solution (c) pending its authorisation by the Parliament.
Exp) Option c is the correct answer. • The fund is administered by the Finance Secretary
A money bill cannot be introduced in the legislative council (Department of Economic Affairs) on behalf of the
(the upper house in those states that have a bicameral state President of India and it can be operated by executive
legislature). It can be introduced in the legislative assembly action.
(the lower house) only and that too on the recommendation • The fund was established under Article 267 (1) of the
of the governor. Indian Constitution and in 1950, the Indian Parliament
enacted the Contingency Fund of India Act 1950.
119. Solution (d)
• The fund was raised from Rs. 50 crores to Rs 500 crore
Exp) Option d is the correct answer. in 2005.
The failure of the government in the Rajya Sabha does not
122. Solution (d)
have any consequences for the government. The government
can continue to function even if it does not have the support Exp) Option d is the correct answer.
of the Rajya Sabha. The government is formed by the party Provision for the inspection of the agency’s work is not a
or coalition of parties that has the majority in the Lok basic element of the citizen charter.
Sabha. A Citizens’ Charter represents the commitment of the
Important Tips Organisation towards standard, quality and time frame of
service delivery, grievance redress mechanism, transparency
The Lok Sabha can express lack of confidence in the
and accountability. The basic objective of the Citizens
government in the following ways
Charter is to empower the citizen in relation to public
• When government fails to pass No-Confidence motion. service delivery.
• When Money Bill is not passed in the Lok Sabha. • Department of Administrative Reforms and Public
• When Motion of Thanks is not passed in the Lok Sabha Grievances in Government of India (DARPG)
on the President’s inaugural address. initiated the task of coordinating, formulating and
• When Censure Motion for Adjournment Motion is operationalising Citizens’ Charters. The Charters are
passed. expected to incorporate the following elements:
• When Cut Motion is passed. • Vision and Mission Statement Details of business
• When the government is defeated on vital issue transacted by the organisation Details of clients
Description of services being provided by department
120. Solution (c) or the agency.
Exp) Option c is the correct answer. • Details of grievance redress mechanism and how to
According to Article 169 of the Constitution of India, access it Promotion of various methods to get benefit
the creation or abolition of the Legislative Council in a from the services available.
state depends on the resolution passed by the Legislative • Expectations from the clients Provide public records.
Assembly of that state by a special majority and approved
123. Solution (c)
by Parliament by a simple majority. This means that the
Parliament has the power to make or repeal a law regarding Exp) Option c is the correct answer.
the existence of the Legislative Council in a state, but only if While the Right to Information Act, 2005, aims to achieve
the state’s Legislative Assembly agrees to it by a two-thirds various objectives, including promoting transparency,
majority of its members present and voting. The law may accountability, and empowering citizens, it does not
also contain provisions for amending the Constitution as specifically mention the objective of empowering women
necessary to give effect to the changes in the state legislature. and weaker sections of society.

235 PYQ Workbook


INDIAN POLITY AND GOVERNANCE

The Act primarily focuses on ensuring transparency and Governments and their instrumentalities accountable to the
accountability in the working of the government, making governed;
democracy work for the people, and empowering citizens And whereas revelation of information in actual practice is
by providing them with the right to access information. likely to conflict with other public interests including efficient
However, it does not explicitly mention empowerment of operations of the Governments, optimum use of limited
specific social groups such as women or weaker sections. fiscal resources and the preservation of confidentiality of
sensitive information;
124. Solution (b)
And whereas it is necessary to harmonies these conflicting
Exp) Option b is the correct answer. interests while preserving the paramountcy of the
Under Article 169 of the Indian Constitution, Parliament democratic ideal;
“may by law” provide for the abolition of the Legislative Now, therefore, it is expedient to provide for furnishing
Council of a State having such a Council or for the creation certain information to citizens who desire to have it.
of such a Council in a State having no such Council, if the
Important Tips
Legislative Assembly of the State passes a resolution to that
effect by a majority of the total membership of the Assembly Right to Information (RTI):
and by a majority of not less than two-thirds of the members • It is an act of the Parliament of India which sets out
of the Assembly present and voting. the rules and procedures regarding citizens’ right to
It does not impose an obligation on the Union Government information.
to take action for initiating legislation in Parliament. • Under the provisions of the RTI Act, any citizen
of India may request information from a “public
125. Solution (d)
authority” (a body of Government or instrumentality
Exp) Option d is the correct answer. of State) which is required to reply expeditiously or
According to Article 87 of Indian Constitution, Special within thirty days.
address by the President is given in following cases: (a) At the • In case of the matter involving a petitioner’s life and
commencement of the first session after each general election liberty, the information has to be provided within 48
to the House of the People; and (b) At the commencement hours.
of the first session of each year, the President shall address • The Act also requires every public authority to
both Houses of Parliament assembled together and inform computerize their records for wide dissemination and
Parliament of the causes of its summons to proactively publish certain categories of information
126. Solution (c) so that the citizens need minimum recourse to request
information formally.
Exp) Option c is the correct answer
Financial Committees of Indian Parliament are; 128. Solution (c)
• Estimates Committee Exp) Option c is the correct answer.
• Public Account Committee The rule of procedure for Motion for Adjournment on a
matter of public importance includes:
• Committee on Public Undertakings
• A motion for an adjournment of the business of the
Important Tips House for the purpose of discussing a definite matter of
Joint Committee on Salaries and Allowance for Members urgent public importance may be made with the consent
of Parliament of the Speaker.
• The Joint Committee was constituted to frame rules • The motion should raise a matter which is definite,
under the Salaries and Allowances of Members of factual, urgent and of public importance.
Parliament Act, 1954. • The motion should not cover more than one matter,
• It consists of 10 members from Lok Sabha and 5 from and should be restricted to a specific matter of recent
Rajya Sabha. occurrence.
• The members of the Joint Committee elect its chairman. • The motion should not raise a question of privilege, nor
127. Solution (d) should it revive discussion on a matter that has been
discussed in the same session.
Exp) Option d is the correct answer.
• Not more than one such motion shall be made at the
The Preamble of the Right to Information Act, 2005 is as
same sitting.
follows:
Whereas the Constitution of India has established democratic 129. Solution (d)
Republic; Exp) Option d is the correct answer.
And whereas democracy requires an informed citizenry The group of States/Union Territories that has only one
and transparency of information which are vital to its seat in the Lok Sabha includes Chandigarh, Sikkim, and
functioning and also to contain corruption and to hold Mizoram. The number of seats in the Lok Sabha varies for

PYQ Workbook 236


INDIAN POLITY AND GOVERNANCE

different states and union territories based on factors such Jurisdiction of Lok Sabha
as population. • Committee on Social Justice and Empowerment
Important Tips • Committee on Urban Development
State with Highest Lok Sabha Seat- Uttar Pradesh (80 • Committee on Labour
seats).
The Committee on Health and Family Welfare is not under
States with only one Lok Sabha seat: the Lok Sabha Jurisdiction
• Mizoram
133. Solution (a)
• Nagaland
Exp) Option a is the correct answer.
• Sikkim
Assertion(A) is correct: Article 163 of the Constitution
Union Territories with only one Lok Sabha seat:
of India reads: The question of whether any and if so what,
• Andaman and Nicobar Islands
advice was tendered by Ministers to the Governor shall not
• Chandigarh be inquired into in any Court.
• Dadra and Nagar Haveli Reason (R) is correct: There is no provision in the Indian
• Daman and Diu Constitution for the system of legal responsibility of the
• Lakshadweep ministers in the States. Therefore, Both Assertion (A)
• Puducherry and Reason (R) are true but Reason (R) is the correct
explanation of Assertion (A)
130. Solution (a)
Exp) Option a is the correct answer. 134. Solution (c)

The objective of the Right to Information Act is to secure Exp) Option c is the correct answer.
access to information from public authorities. The Act • Constitution of Legislatures in States: Article 168 states
provides a mechanism for citizens to access information held that every state shall have a legislature consisting of the
by public authorities, ensuring transparency, accountability, Governor and either one house (Legislative Assembly)
and promoting good governance. or two houses (Legislative Council and Legislative
It empowers individuals to seek information and obtain Assembly).
records, documents, and other relevant information
• The Speaker and Deputy Speaker of the Legislative
from public authorities. The Act aims to enhance citizen
Assembly: Article 178 states that every Legislative
participation, promote informed decision-making, and hold
Assembly of a State shall choose two members of the
public authorities accountable for their actions.
Assembly to be the Speaker and Deputy Speaker, and fill
131. Solution (d) the vacancies whenever they arise.
Exp) Option d is the correct answer. • Salaries and allowances of members: Article 195 states
Statement 1 is incorrect: Provision for Joint sitting is that members of the Legislative Assembly and the
applicable for only ordinary bills and financial bills. Legislative Council of a State shall be entitled to receive
There is no provision of joint sitting for deadlock on the such salaries and allowances as may from time to time be
Constitutional amendment bills. Constitutional amendment determined by the Legislature of the State by law.
bill must be passed by each House separately.
• Definition of ‘Money Bills: Article 199 is about the
Statement 2 is incorrect: The Speaker of Lok Sabha presides
definition of Money Bills. A Money Bill is a bill that
over a joint sitting of the two Houses and the Deputy
deals with the allocation of funds, such as taxes, public
Speaker, in his absence. If the Deputy Speaker is also absent
spending, borrowing, and custody of the state’s funds.
from a joint sitting, the Deputy Chairman of Rajya Sabha
presides. If he is also absent, such other person as may be Important Tips
determined by the members present at the joint sitting, • A Money Bill can only be introduced in the Legislative
presides over the meeting. Hence, Deputy Chairman of
Assembly of a state and can only be passed or rejected
Rajya Sabha can preside over the joint sitting.
by it.
Statement 3 is incorrect: The quorum to constitute a joint
• The Legislative Council of a state can only make
sitting is one-tenth (not one-sixth) of the total number of
recommendations on a Money Bill within 14 days, but
members of the two Houses. The joint sitting is governed by
the Legislative Assembly can accept or reject them.
the Rules of Procedure of Lok Sabha and not of Rajya Sabha.
• The Speaker of the Legislative Assembly has the final
132. Solution (d) authority to decide whether a bill is a Money Bill or
Exp) Option d is the correct answer not, and his decision is final.
Lok Sabha is composed of representatives of the people • A Money Bill has to be certified by the Speaker before
chosen by direct election on the basis of adult suffrage. it is sent to the Governor for assent

237 PYQ Workbook


INDIAN POLITY AND GOVERNANCE

135. Solution (c) Statement 1 is incorrect: The tenure of the State Legislative
Exp) Option c is the correct answer. Council is not six years, but its members have a term of six
years.
The recognition of a merger of a political party with another
political party in India is subject to certain conditions as per Statement 2 is correct: Unlike the State Legislative Assembly,
the Tenth Schedule of the Indian Constitution, also known which can be dissolved by the Governor on the advice of the
as the Anti-Defection Law. According to the Anti-Defection Chief Minister, the State Legislative Council is a permanent
body. It is not subject to dissolution.
Law, a merger of a political party with another party is
recognized if not less than two-thirds of the members of Statement 3 is correct: State Legislative Council cannot be
the legislature party have agreed to such merger. dissolved. It continues to exist and functions as a legislative
body even during times of political upheaval or government
Important Tips changes.
If a member, after being elected as the presiding officer Statement 4 is incorrect: One-third (not one-sixth)
of the House, voluntarily gives up the membership of members of the House to be elected by electorates consisting
his party or rejoins it after he ceases to hold that office, of members of municipalities, district boards and such other
he would not be disqualified on the grounds of Anti- local authorities in the State as Parliament may by law specify
Defection Law. This exemption has been provided in view
Statement 5 is incorrect: One-third (not one-sixth)
of the dignity and impartiality of this office.
members to be elected by the members of the Legislative
136. Solution (a) Assembly of the State from amongst persons who are not
members of the Assembly
Exp) Option a is the correct answer.
Statement 6 is correct: As per the Constitution, one-third of
Joint sitting is an extraordinary machinery provided by the
the members of the State Legislative Council retire every
Constitution to resolve a deadlock between the two Houses
two years. This arrangement ensures a continuous rotation
over the passage of a bill. If the bill (under dispute) has
of members and prevents a complete overhaul of the council
already lapsed due to the dissolution of the Lok Sabha, no
at once.
joint sitting can be summoned. But, the joint sitting can be
held if the Lok Sabha is dissolved after the President has Statement 7 is incorrect: The State Legislative Council
notified his intention to summon such a sitting (as the bill is presided over by a chairman who is not the Deputy
does not lapse in this case). After the President notifies his Governor but is elected from among its members. The
Chairman’s position is similar to that of the Speaker in the
intention to summon a joint sitting of the two Houses, none
State Legislative Assembly.
of the Houses can proceed further with the bill.
Statement 8 is correct: The members of the State Legislative
Important Tips Council serve a term of six years, and their tenure is not
Joint Sitting: subject to dissolution during this period. Members may
• A deadlock is deemed to have taken place under any retire, be replaced, or continue for the full six-year term
one of the following three situations after a bill has depending on the rotation system.
been passed by one House and transmitted to the other
139. Solution (a)
House:
Exp) Option a is the correct answer.
• if the bill is rejected by the other House;
The tenure of the 5th Lok Sabha was about 6 years. The Lok
• if the Houses have finally disagreed as to the
Sabha is the lower house of the Parliament of India, and its
amendments to be made in the bill; or
usual term duration is 5 years. However, the 5th Lok Sabha’s
• if more than six months have elapsed from the date of tenure was extended due to the declaration of a state of
the receipt of the bill by the other House without the emergency in India from 1975 to 1977.
bill being passed by it.
140. Solution (c)
• In the above three situations, the president can
summon both the Houses to meet in a joint sitting for Exp) Option c is the correct answer.
the purpose of deliberating and voting on the bill. A minister who is a member of the Rajya Sabha is permitted to
participate in the deliberations of the Lok Sabha, as stated by Article
137. Solution (c)
88 of the Indian Constitution. This provision grants them the right
Exp) Option c is the correct answer. to speak and engage in the proceedings of the lower house.
The Speaker of the Lok Sabha is elected by the members of
141. Solution (c)
the Lok Sabha and does not need to take an oath of office.
The President, Vice-President, Prime Minister, and Members Exp) Option c is the correct answer.
of the Council of Ministers, all take the oath of office. Privilege motion is concerned with the breach of
parliamentary privileges by a minister. It is moved by a
138. Solution (c) member when he feels that a minister has committed a breach
Exp) Option c is the correct answer. of privilege of the House or one or more of its members by

PYQ Workbook 238


INDIAN POLITY AND GOVERNANCE

withholding facts of a case or by giving wrong or distorted The Lok Sabha Secretariat is responsible for the day-to-day
facts. Its purpose is to censure the concerned minister. functioning of the Lok Sabha, including the preparation of
Important Tips the agenda, the conduct of debates, and the recording of
votes.
Motions:
• No discussion on a matter of general public importance 146. Solution (b)
can take place except on a motion made with the Exp) Option b is the correct answer.
consent of the presiding officer.
State Legislative Assembly can participate in the election of
• The House expresses its decisions or opinions on
President of India, Rajya Sabha Members and Members of
various issues through the adoption or rejection of
Legislative Council of the state.
motions moved by either ministers or private members.
• The motions moved by the members to raise discussions The vice president is elected by an electoral college
on various matters fall into three principal categories: consisting of members of both Houses of Parliament, in
accordance with the system of proportional representation
• Substantive Motion: It is a self-contained independent
proposal dealing with a very important matter like by means of the single transferable vote and the voting in
impeachment of the President or removal of Chief such election is by secret ballot.
Election Commissioner. Important Tips
• Substitute Motion: It is a motion that is moved in • President of India: The President is elected by the
substitution of an original motion and proposes an members of an electoral college consisting of the elected
alternative to it. If adopted by the House, it supersedes members of both the Houses of Parliament and the
the original motion. elected members of the Legislative Assemblies of States
• Subsidiary Motion: It is a motion that, by itself, has and the Union Territories of Delhi and Pondicherry.
no meaning and cannot state the decision of the House • Rajya Sabha members: The Rajya Sabha, or Council
without reference to the original motion or proceedings of States, is the upper house of Parliament. It has
of the House. 245 members, of which 233 are elected by the elected
142. Solution (d) members of the State Legislative Assemblies using
single transferable votes through Open Ballot.
Exp) Option d is the correct answer.
• Members of Legislative Council of the State: One-
According to the exception rule of the Anti-Defection Bill,
third members of the Council are elected by members
in order to uphold the dignity and impartiality of the office,
of the Legislative Assembly from among persons who
if a member is elected as the presiding officer of the House,
are not members of either House of Parliament or
they are exempted from disqualification if they voluntarily
either House of Legislature.
relinquish their party membership or rejoin the party
after they cease to hold that office. 147. Solution (b)

143. Solution (b) Exp) Option b is the correct answer.


Exp) Option b is the correct answer. Statement 1 is incorrect- The Committee on Public
The Constitution under Article 105 expressedly mentioned Undertakings consists of members from both the Rajya
two privileges, that is, freedom of speech in Parliament and Sabha (Council of States) and the Lok Sabha (House of the
right of publication of its proceedings. People). The committee has a total of 22 members, out of
which 15 members are from the Lok Sabha, and 7 members
144. Solution (a)
are from the Rajya Sabha.
Exp) Option a is the correct answer.
Statement 2 is correct- The Chairperson of the Committee
The state-wise seat allocation in the Lok Sabha is based on is appointed by the Speaker of the Lok Sabha. The
the population of the state as per the latest census. This Chairperson plays a significant role in presiding over the
is as per Article 81(1) of the Constitution of India. The
committee’s meetings, organizing its activities, and ensuring
number of seats allotted to a state is determined by dividing
effective functioning.
the population of the state by the average population of a
parliamentary constituency in India. 148. Solution (c)
145. Solution (d) Exp) Option c is the correct answer.
Exp) Option d is the correct answer. A censure motion is a motion that expresses strong
The Lok Sabha Secretariat is the administrative body of disapproval of the actions of a minister or group of
the Lok Sabha, the lower house of the Parliament of India. ministers. Censure motions can be moved against individual
It comes under direct supervision of Lok Sabha Speaker. ministers, groups of ministers, or even the entire Council
It is headed by the Secretary-General of the Lok Sabha. of Ministers.

239 PYQ Workbook


INDIAN POLITY AND GOVERNANCE

Important Tips notification in the Official Gazette. The notification must be


issued at least 14 days before the date of election.
Some other types of motion in the Parliament:
• No-Confidence Motion: A motion moved in the 152. Solution (a)
Parliament to test the support and confidence of the Exp) Option a is the correct answer.
ruling government. If the motion is passed, it indicates
A censure motion can be moved by any member of
a lack of confidence in the government, potentially
leading to its resignation. Parliament, and it does not require a leave of the House.
The Speaker of the House decides whether the motion is in
• Adjournment Motion: A motion moved to suspend
order, and if it is, it is then debated and voted on.
the regular business of the Parliament and discuss
a matter of urgent public importance. It leads to the 153. Solution (b)
adjournment of the current proceedings to address the
Exp) Option b is the correct answer.
specific issue raised in the motion.
• Calling Attention Motion: A motion moved by a The Constitution of India provides for the division of
member of Parliament to bring attention to an urgent powers between central and state government. The seventh
matter that requires immediate government attention schedule provides for three list- State List, Central List, and
and response. It seeks to elicit a response from the Concurrent List. Out of the three list, the Concurrent list
concerned minister. includes subjects on which both the Parliament (Central
• Cut Motion: A motion moved to reduce the amount of government) and State Legislature (state governments)
a demand presented in the budget. It can be of three can legislate independently and concurrently.
types: Policy Cut, Economy Cut, and Token Cut. It
Important Tips
allows members to express their dissatisfaction or
disagreement with the proposed expenditure. Subjects under the Central List:
• Motion of Thanks: A motion moved to express • Defense and armed forces
gratitude and appreciation to the President of India • Foreign affairs and international relations
for their address to the joint session of Parliament at • Atomic energy
the beginning of a new session.
• Currency, coinage, and legal tender
149. Solution (c) • Railways
Exp) Option c is the correct answer. • Airports and airways
The Public Accounts Committee (PAC) is a parliamentary • Interstate trade and commerce
committee in India that plays a crucial role in ensuring • Banking and financial institutions
financial accountability and transparency in government • Taxes on income other than agricultural income
expenditures. It examines the audit reports of the Subjects under the State List:
Comptroller and Auditor General (CAG) and scrutinizes • Police and public order
the financial operations of the government. Members of the • Public health and sanitation
Public Accounts Committee are appointed for a term of one • Agriculture and irrigation
year. After the term expires, new members are appointed or • Fisheries
elected to the committee. • Local government
• Public works
150. Solution (a) • Public entertainment
Exp) Option a is the correct answer. • Land and land revenue
Somnath Lahiri was a member of the Constituent Assembly • Education at the state level Subjects under the
of India and a prominent leader of the Communist Party Concurrent List:
of India. He was a vocal critic of the British rule in India • Criminal law and procedure
and he believed that the Constituent Assembly was not • Marriage and divorce
truly representative of the Indian people. In a speech in • Adoption
the Constituent Assembly on 19 December 1946, Lahiri said • Bankruptcy and insolvency
that the Constituent Assembly was “British made” and that it • Education, including technical education
was “working the British plan as the British should like it to • Population control and family planning
be worked out”. He argued that the Assembly was dominated • Protection of wildlife and animals
by the Indian National Congress, which was a party that had • Trusts and trusteeship
close ties to the British government • Price control and essential commodities

151. Solution (a) 154. Solution (c)


Exp) Option a is the correct answer. Exp) Option c is the correct answer.
The President of India fixes the date of election of the Statement 1 is correct- Parliament can legislate a subject
Speaker of the Lok Sabha. This is done by issuing a under the State list in certain conditions. These conditions

PYQ Workbook 240


INDIAN POLITY AND GOVERNANCE

include during a national emergency under Article resolution is received, it is entered into the list of business
250, when a state makes a request under Article 252, to in the member’s name, and the Speaker fixes a day for its
implement international agreements under Article 253, and discussion and debate, which must be at least fourteen days
during President’s Rule under Article 356. after the receipt of the notice.
Statement 2 is incorrect- Parliament can legislate a subject
158. Solution (a)
under the State list under certain circumstances during
National Emergency and President’s rule. So, the statement Exp) Option a is the correct answer.
that Parliament cannot legislate a subject under the State list The Public Accounts Committee (PAC) is a committee of
under any circumstances is not correct. the Parliament of India that examines the audit reports of
Statement 3 is correct- As per Article 249 of the Constitution, the Comptroller and Auditor General of India (CAG). The
if the Rajya Sabha passes a resolution supported by not less PAC consists of not more than 22 members, of whom 15 are
than two-thirds of the members present and voting, stating elected by the Lok Sabha and 7 are elected by the Rajya
that it is necessary or expedient in the national interest, Sabha. The term of office of a member of the PAC is one
Parliament can make laws on any matter enumerated in the year.
State list.
Important Tips
155. Solution (d) Some other information about Public Accounts Committee
Exp) Option d is the correct answer. • The PAC was first constituted in 1921
Statement 1 is correct - Article 85(2) of the Constitution of • The PAC is one of the three financial standing
India states that on dissolution of the House of the People, committees of the Parliament of India, the other two
all seats in the House shall become vacant. This means that being the Estimates Committee and the Committee
both the Speaker and Deputy Speaker, who are members of on Public Undertakings
the Lok Sabha, will also cease to be members of the House • The Chairperson of the PAC is elected by the members
on dissolution. of the committee from amongst themselves
Statement 2 is incorrect - The Speaker does not vacate his
office on dissolution of the Lok Sabha. He continues to 159. Solution (a)
hold office until immediately before the first meeting of Exp) Option a is the correct answer.
the House after dissolution. Statement 1 is incorrect- The election of the Speaker of
Statement 3 is correct - Only Deputy Speaker vacates Lok Sabha is conducted by the members of the Lok Sabha
his office. Article 95(3) of the Constitution states that the themselves. The Election Commission of India does not have
Deputy Speaker shall vacate his office on the dissolution any role in the election of the Speaker.
of the House of the People, but the Speaker shall not vacate
Statement 2 is correct- The Speaker on assuming his office
his office until immediately before the first meeting of the
is not required to make and subscribe oath or affirmation.
House after dissolution.
This is because the Speaker is already a member of the Lok
156. Solution (b) Sabha and has already taken the oath or affirmation as a
Exp) Option b is the correct answer. member of the House
Statement 1 is incorrect- The Panel of Chairperson in Lok Statement 3 is incorrect- The Speaker does exercise a
Sabha is not restricted to members of the ruling party. casting vote in case of equality of votes. This is a special
It consists of members nominated by different political power that is given to the Speaker to break a tie in the Lok
parties. Sabha
Statement 2 is correct- The Panel of Chairperson is indeed 160. Solution (c)
nominated by different political parties, and the Speaker
Exp) Option c is the correct answer
of Lok Sabha appoints them to their respective positions.
Financial Bills are a broader category that includes Money
Statement 3 is correct- The Panel of Chairperson in Lok
Bills as well as bills that deal with any other financial
Sabha consists of 10 members, and one of them is appointed
to preside over the House in the absence of both the Speaker matters. Financial Bills are dealt with under Articles 117
and the Deputy Speaker. (1) and Article 117 (3.) Though Money Bill is a species of
the financial bill however, not all financial bills are money
157. Solution (b) bill.
Exp) Option b is the correct answer.
Important Tips
A member wishing to give notice of a resolution for the
Financial Bills (I)
removal of the Speaker or Deputy Speaker should address
it in writing to the Secretary General of Lok Sabha. The • They are bills that contain matters related to finances,
Secretary General is responsible for receiving and processing including matters beyond the scope of a Money Bill
such notices. Once the motion for leave to move the (Article 110).

241 PYQ Workbook


INDIAN POLITY AND GOVERNANCE

• They are introduced in the Lok Sabha and require the • Independently Elected Member: If any independently
President’s recommendation. elected member joins any political party.
• Unlike Money Bills, they can be rejected or amended • Nominated Member: If any nominated member joins
by the Rajya Sabha. any political party after the expiry of six months.
• In case of a deadlock, a joint sitting can be summoned.
163. Solution (b)
The President has the authority to give assent, withhold
the bill, or return it for reconsideration. Exp) Option b is the correct answer

Financial Bills (II) Pair 1 matched correctly: Public health and sanitation are
among the subjects listed in the State List.
• They solely deal with provisions involving expenditure
from the Consolidated Fund of India and do not Pair 2 matched correctly: Citizenship, naturalization, and
include any matter of a Money Bill (Article 110). aliens are among the subjects listed in the Union List.

• They are treated as ordinary bills in all respects. These Pair 3 matched correctly: Legal, medical, and other
bills cannot be passed by either house of Parliament professions are among the subjects listed in the Concurrent
unless recommended by the President. List.
• They can be rejected or amended by either House, and 164. Solution (c)
a joint sitting can be summoned in case of a deadlock. Exp) Option c is the correct answer
• The President has the power to give assent, withhold
Statement I is true: Article 80 of the Constitution of
the bill, or return it for reconsideration.
India states that not more than 238 of the 250 members of
161. Solution (b) the Rajya Sabha are elected by the elected members of the
State Legislative Assemblies. This means that a majority
Exp) Option b is the correct answer
of the members of the Rajya Sabha are elected by the State
Statement 1 is correct: Short notice questions allow Legislative Assemblies.
Members of Parliament or Legislators to seek immediate
Statement II is False: Article 84 of the Constitution of India
clarification or information from the government on
states that a person can be elected to the Rajya Sabha if he/
pressing issues. These questions can be asked for oral answer
she is a citizen of India and has completed 30 years of age.
at a notice period less than 10 days.
There is no requirement that a member of the Rajya Sabha
Statement 2 is correct: In the case of short notice questions, elected by the State Legislative Assemblies must have his/her
the concerned Minister has the discretion to accept domicile in the concerned State.
or reject the question. If the Minister agrees to answer,
the question becomes admissible and can be taken up for 165. Solution (b)
discussion during the session. Exp) Option b is the correct answer
Statement 3 is incorrect: Short notice questions are taken The Anti-Defection Law empowers the Speaker/Chairman
up after the question hour. to make decisions on disqualification and to maintain the
stability and integrity of political parties and legislative
162. Solution (d) bodies by curbing defections. Thus, it can be said that
Exp) Option d is the correct answer the 52nd Amendment has enormously strengthened the
When there is a dispute regarding the disqualification of a powers of the Speaker/Chairman of the Parliament and State
member on grounds of defection, the Speaker or Chairman Legislatures.
of the House makes the decision and not Supreme Court,
166. Solution (c)
however the decision of the Speaker or Chairman is subject
to judicial review. Exp) Option c is the correct answer
The Estimates Committee is a Parliamentary Committee
Important Tips that examines the budget estimates of the government. The
Grounds for Defection Committee is composed of 30 members, all of whom are
• Voluntary Give Up: If an elected member voluntarily members of the Lok Sabha. The Rajya Sabha does not have
gives up his membership of a political party. any representation on the Estimates Committee.
• Violation of Instructions: If he votes or abstains from 167. Solution (c)
voting in such House contrary to any direction issued
Exp) Option c is the correct answer
by his political party or anyone authorized to do so,
without obtaining prior permission. Joint Sitting of Indian parliament has been called for only 3
bills:
• As a pre-condition for his disqualification, his
abstention from voting should not be condoned by • Dowry Prohibition Bill, 1961
his party or the authorized person within 15 days of • Banking Service Commission (Repeal) Bill, 1978
such incident. • Prevention of Terrorism Bill, 2002

PYQ Workbook 242


INDIAN POLITY AND GOVERNANCE

168. Solution (d) of the legislature. This separation of powers ensures a clear
Exp) Option d is the correct answer distinction between the executive and legislative functions.
• Rule of lapse refers to the provision that unspent money from Option b is incorrect: The Presidential form of government
a budget does not carry over to the next financial year. If the does separate the legislative and executive functions,
government doesn’t utilize the allocated funds, the remaining with the President being the head of the executive branch
amount lapses and cannot be used in the following year. and the legislature having its own separate powers and
• A supplementary grant is an additional allocation for responsibilities.
unforeseen expenses, provided when the government Option c is incorrect: The principle of collective
underestimates its financial needs. responsibility is usually associated with parliamentary
• A vote on account is a temporary grant to meet systems, where the executive (Prime Minister and
government expenses until the final budget is passed, Cabinet) is collectively responsible to the legislature. In
typically granted at the start of a financial year before the a presidential system, the President is not bound by the
budget process is complete. principle of collective responsibility.

169. Solution (a) Option d is incorrect: In a presidential form of government,


the tenure of the President is typically fixed and does not
Exp) Option a is the correct answer
depend on the legislature. The President serves a fixed term
The correct sequence of different stages a budget has to go as specified by the constitution and is not subject to removal
through in the Parliament is as follows: by the legislature, except under specific circumstances
1- Presentation of the Budget outlined in the constitution.
2- Scrutiny by Departmental Committees
171. Solution (b)
4- Passing of Appropriation Bill
Exp) Option b is the correct answer.
3- Passing of Finance Bill
Article 169 of the Constitution of India contains provisions
Important Tips for the abolition and creation of Legislative Councils in
The stages of the enactment of the budget are as follows: states. It empowers the Parliament to pass a law to create or
A) Presentation: The budget is presented by the Finance abolish the Legislative Council in a state if the Legislative
Minister, with a budget speech in February, laying it Assembly of that state passes a resolution to that effect by a
before both houses. special majority.
B) General discussions: General principles of the budget Important Tips
are discussed, without motions of reduction or votes, • Article 171 of the Constitution of India pertains to the
in both houses. The Finance Minister provides a composition of the Legislative Councils in states that
concluding response. have such councils. It specifies the number of members,
C) Scrutiny by departmental committees: Each their method of election, and other related details.
departmental standing committee scrutinizes the • Article 356 of the Constitution of India deals with the
demands for grants, producing a report after several imposition of President’s rule in a state when there is
weeks of recess. a failure of the constitutional machinery in that state.
D) Voting on demands for grants: Members of the Lok • Article 182 of the Constitution of India relates to the
Sabha vote on each demand for grants, excluding power of the Governor of a state to issue ordinances
expenditure charged on the consolidated fund. CUT during recess periods of the state legislature.
Motions for grant reduction can be moved.
172. Solution (a)
E) Appropriation and Finance bills: The appropriation
bill includes voted demands for grants and expenses Exp) Option a is the correct answer.
from the consolidated fund, while the finance bill Option a is incorrect: The Speaker of Lok Sabha cannot
contains provisions for taxation. Amendments can be call a joint sitting of both Houses to pass a Constitution
proposed for the finance bill. Both bills become acts Amendment Bill, as such a bill must be passed separately by
after receiving presidential assent, enabling expenses each House with the required majority.
and taxation. Option b is correct: A Constitution Amendment Bill must be
170. Solution (a) passed separately by both the Houses of Parliament with a
special majority. This means that it should be supported by
Exp) Option a is the correct answer.
a majority of the total membership of each House, as well
In a presidential form of government, the President is as by a majority of not less than two-thirds of the members
separate from the legislative body, which means that the present and voting.
President is not a member of the legislature. The President is
Option c is correct: A Constitution Amendment Bill can be
the head of the executive branch and is elected independently
introduced in either House of Parliament by a minister or

243 PYQ Workbook


INDIAN POLITY AND GOVERNANCE

a private member, and does not require the prior approval Important Tips
of the President
• Article 3 of the Constitution of India deals with
Option d is correct: A Constitution Amendment Bill can be the formation of new States and alteration of areas,
sponsored by a private member, but it has very little chance boundaries or names of existing States. In this regard,
of being passed by Parliament. the Parliament may by law:
• form a new State by separation of territory from any
173. Solution (b)
State or by uniting two or more States or parts of States,
Exp) Option b is the correct answer. or by uniting any territory to a part of any State;
A “Cut Motion” is a parliamentary procedure in India where • increase the area of any State;
a member of the Parliament proposes to reduce the amount • diminish the area of any State;
of a demand in the budget presented by the government.
• alter the boundaries of any State;
These motions can be introduced after the presentation of
• alter the name of any State.
the Railway Budget and the General Budget. The purpose of
Cut Motions is to debate and challenge specific expenditures 175. Solution (a)
mentioned in the budget proposals. Exp) Option a is the correct answer.
Important Tips The Rajya Sabha can withhold its consent to a Money Bill for
• There are three types of Cut Motions: Policy Cut, a maximum period of 14 days.
Economy Cut and Token Cut • According to Article 109 of the Constitution of India, if
• Policy Cut is a motion that seeks to reduce the amount a Money Bill is passed by the Lok Sabha (the lower house
of demand to Re.1. It represents the disapproval of of Parliament), it is transmitted to the Rajya Sabha (the
the policy underlying the demand and suggests an upper house).
alternative policy. • The Rajya Sabha must return the Bill to the Lok Sabha
• Economy Cut is a motion that seeks to reduce the within 14 days, either with recommendations or without
amount of demand by a specified amount. It represents any recommendations.
the economy that can be effected by reducing or • The Lok Sabha can then consider these
omitting an item in the demand. recommendations, but it is not bound to accept them.
• Token Cut is a motion that seeks to reduce the amount After the expiry of the 14-day period, whether with or
of demand by Rs.100. It represents a specific grievance without recommendations, the Bill is deemed to have
that is within the responsibility of the government. been passed by both houses.

174. Solution (c) 176. Solution (a)

Exp) Option c is the correct answer. Exp) Option a is the correct answer.
Both the statements are individually true and Statement II is
According to Article 3 of the Constitution of India, the
the correct explanation of Statement I.
Parliament may by law:
Statement I is correct because the Rajya Sabha, which is
• Alter the boundaries of any State: The Parliament has
the upper house of the Parliament of India, is not subject to
the power to alter the boundaries of any State in India.
dissolution unlike the Lok Sabha, which is the lower house.
This can be done through the process of reorganization The members of the Rajya Sabha enjoy a tenure of six years,
of states, which requires the introduction and passing of after which they are either re-elected or replaced by new
a law by Parliament. members
• Increase the area of any State: The Parliament also has Statement II is correct because according to Article 83 of
the authority to increase the area of any existing state by the Constitution of India, one-third of the members of the
annexing or merging territories with it. This can be done Rajya Sabha retire every two years in accordance with the
through the enactment of a law by Parliament. provisions made by Parliament by law. This ensures that
the Rajya Sabha is a permanent and continuous body that
Statement 1 is not related to Article 3, but to Article 253,
reflects the changing composition and preferences of the
which empowers Parliament to make laws for implementing
states and the union territories.
treaties, agreements or conventions with foreign countries or
Statement II is the correct explanation of Statement
international organizations.
I because it shows how the Rajya Sabha maintains its
Statement 4 is not related to Article 3, but to Article 244A, continuity and stability without being dissolved. The
which provides for the creation of an autonomous state staggered retirement of one-third of its members every
comprising certain tribal areas in Assam and for the creation two years allows for a gradual and periodic renewal of the
of local legislature or council of ministers or both for such Rajya Sabha, while preserving its institutional memory and
state. experience

PYQ Workbook 244


INDIAN POLITY AND GOVERNANCE

177. Solution (c) of Australia, but follows the Constitution of Canada. The
Exp) Option c is the correct answer. Constitution of Australia does not vest the residuary powers
in any level of government, but leaves them to be determined
The Committee on Public Assurances is responsible for
by judicial interpretation. The Constitution of Canada, on
examining the assurances, promises, and undertakings
the other hand, vests the residuary powers in the federal
given by Ministers on the floor of the House and ensuring
government by Section 91 of its Constitution Act, 1867.
their implementation. This committee acts as a watchdog
to verify whether the commitments made by Ministers have Statement 3 is incorrect: Schedule 7 of the Constitution
been fulfilled or not. of India does not provide a list of residuary powers, but
provides three lists of subjects on which Parliament and state
Important Tips legislatures can make laws. These are List I (Union List), List
• Departmentally Related Committees are II (State List) and List III (Concurrent List). The residuary
parliamentary committees that are assigned specific powers are those that are not covered by any of these lists.
departments or ministries for detailed examination and Statement 4 is correct: Government of India Act, 1935 placed
scrutiny of their policies, budgets, and other matters. residuary powers in the hands of the Governor-General.
• The Business Advisory Committee is a committee This was done to ensure that no important subject was left
formed in the Parliament of India. Its main function is out from legislative competence and to give flexibility to deal
to allocate time for the discussions on various items of with unforeseen situations. The Governor-General could
business and to recommend the allocation of time for legislate on any matter that was not included in any of the
different stages of the legislative process. seven lists provided by this Act
• The Political Affairs Committee of the Union
180. Solution (c)
Cabinet is a high-level committee consisting of senior
ministers. It is responsible for discussing and deciding Exp) Option c is the correct answer.
various political matters and policy issues. Statement 1 is correct: The head of government is also
the head of state in the presidential system. The president
178. Solution (c)
represents the nation both internally and externally, and has
Exp) Option c is the correct answer. the authority to appoint and dismiss cabinet members, issue
Option a is correct: As per Article 110 of the Constitution of executive orders, veto legislative acts, negotiate treaties,
India, a Money Bill can only be introduced in the Lok Sabha command the armed forces, etc.
(House of the People), the lower house of Parliament. It Statement 2 is correct: Executive can veto legislative acts
cannot be introduced in the Rajya Sabha (Council of States), in the presidential system. The president has the power to
the upper house of Parliament. reject or approve bills passed by the legislature, and can also
Option b is correct: According to Article 110(3) of the propose legislation to the legislature. However, the legislature
Constitution of India, the decision of the Speaker of the can override the president’s veto by a supermajority vote in
Lok Sabha on whether a bill is a Money Bill or not is final. some cases
The Speaker’s decision is not subject to judicial review.
181. Solution (c)
Option c is incorrect: In the case of a deadlock over a
Exp) Option c is the correct answer.
Money Bill between the Lok Sabha and the Rajya Sabha,
the President does not have the power to summon a joint Money Bills in India are governed by Article 110 of the
sitting of the Parliament. The deadlock can only be resolved Constitution, which specifies that provisions related to the
by the passage of the Money Bill by the Lok Sabha. imposition, abolition, remission, alteration, or regulation
of any tax, appropriation of money from the Consolidated
Option d is correct: According to Article 117(1) of the
Fund, and custody of the Consolidated Fund or Contingency
Constitution of India, a Money Bill can only be introduced
Fund are considered Money Bills.
in the Lok Sabha with the recommendation of the
President. The President’s recommendation is necessary for The imposition of fines by a local authority for a local
the introduction of a Money Bill. purpose fall outside the scope of Money Bills.

179. Solution (c) Important Tip

Exp) Option c is the correct answer. • The President can either give assent to the bill or
withhold assent, but cannot return a money bill for
Statement 1 is correct: Residuary powers have been given
reconsideration like an ordinary bill.
to the Union Parliament by Article 248 and Entry 97 of the
Union List. These provisions give Parliament exclusive power • The Speaker of the Lok Sabha has the authority to
to make any law with respect to any matter not enumerated decide whether a bill is a money bill or not, and their
in List II or List III, including any tax not mentioned in decision is final. It cannot be questioned or challenged
either of those lists. in any court of law.

Statement 2 is incorrect: In the matter of residuary powers, • Money bills can only be introduced in the Lok Sabha
the Constitution of India does not follow the Constitution and not in the Rajya Sabha (Council of States).

245 PYQ Workbook


INDIAN POLITY AND GOVERNANCE

• The Rajya Sabha can discuss and suggest amendments • Parliament can only initiate constitutional amendment
to a money bill, but it does not have the power to reject bill.
or block it. The Lok Sabha has the final say on the • It can be introduced by either minister or private
bill. member- does not require president approval.
182. Solution (d) • The bill must be passed by each house by Special
Majority.
Exp)- Option d is the correct answer.
• The President must give his assent- Can not return or
Statement 1 is incorrect- Committee on Estimates is a
withhold assent of the constitutional amendment bill.
standing committee. The function of the committee is to
examine the estimates included in the budget and suggest • Three ways in which constitution can be amended:
‘economies’ in public expenditure. Hence, it has been • Simple Majority of the parliament.
described as a ‘continuous economy committee’. • Special Majority of the Parliament.
Statement 2 is incorrect- The Committee on Railway • Special Majority of the Parliament and the ratification
Convention is an ad hoc committee established in 1949. Its of half of the state legislatures
initial focus was determining the Rate of Dividend payable
185. Solution (c)
by Railways to General Revenues. Since 1971, it has expanded
to cover broader subjects related to Railways and Railway Exp) Option c is the correct answer
Finances. Chairperson of the Public Accounts Committee (PAC) is
not elected by its members. The Chairperson of the PAC
Important Tips
is appointed by the Speaker of the Lok Sabha from among
• The first Estimates Committee was established in 1950 its members.
based on the suggestion of John Mathai, the finance
minister at that time. 186. Solution (d)
• It has 30 members, all from Lok Sabha only. Rajya Exp) Option d is the correct answer
Sabha has no representation in it. The Rajya Sabha does have the power to discuss a Money
• The term of office for members is one year. Bill . but it has no authority to reject or amend the bill.
• The committee’s chairman is chosen by the Speaker Important Tip:
from its members, typically belonging to the ruling
Facts regarding Money Bill in India
party.
• Unlike Ordinary Bills, Money Bills are introduced only
183. Solution (b) in Lok Sabha on the President’s recommendation,
Exp) Option b is the correct answer. which is mandatory.
The Speaker of the Lok Sabha, who serves as the presiding • The bill, introduced in the Lok Sabha on the President’s
officer, typically holds office for the duration of the Lok recommendation, is referred to as a government bill.
Sabha’s term. However, if the Speaker wishes to resign • It should be noted that all government bills are
before the completion of the term, they must submit their introduced only by the Minister.
resignation in writing to the Deputy Speaker of the Lok • After the bill is passed by the Lok Sabha, it is sent to
Sabha. the Rajya Sabha, which has only limited authority. It
has no authority to reject or amend the bill.
184. Solution (b)
• There is no possibility of disagreement, hence there is
Exp) Option b is the correct answer. no provision for a joint sitting regarding money bills.
A joint sitting of both Houses of Parliament is not allowed • The Lok Sabha speaker certifies the bill as a money
in the case of a deadlock over the passage of a constitutional bill and his decision is final in this regard.
amendment bill. However, in the case of an ordinary bill,
• The President’s consent is required when the bill
there is a provision for a joint sitting if needed. has passed both houses. He can either give assent
Important Tips: or withhold assent but cannot send a bill for
reconsideration.
• The procedure for amending the Indian Constitution
was borrowed from the South Africa Constitution. 187. Solution (d)
• Article 368 in Part XX of the Constitution deals with Exp) Option d is the correct answer
the powers of Parliament to amend the Constitution
According to Article 80 of the Constitution of India, the
and its procedure.
President of India can nominate members to the Parliament
• “Parliament can amend the constitution provided that under the category of “Persons having special knowledge
Basic Structure remains intact”- Kesvananda Bharti or practical experience in respect of literature, science,
Case 1973. art, and social service.” “State Service” is not a category

PYQ Workbook 246


INDIAN POLITY AND GOVERNANCE

under which the President of India nominates Members express their lack of confidence in the Council of Ministers.
of Parliament. If such a motion is passed, the Council of Ministers has to
resign.
188. Solution (d)
Exp) Option d is the correct answer 193. Solution (a)
Option (d) is not a strength or chief benefit of bicameralism. Ex) Option a is the correct answer.
In fact, it presents a potential limitation or criticism of Statement 1 is correct- The Speaker of the House has the
bicameral systems. When the members of the second authority to permit any Member to address the House in
chamber are non-elected or indirectly elected, it can lead their mother tongue if they are unable to adequately express
to a situation where the democratic principle of direct themselves in Hindi or English. This provision ensures that
representation is undermined. Critics argue that this can Members have the freedom to communicate effectively in a
diminish the democratic legitimacy of the second chamber language they are comfortable with.
and create an imbalanced representation of the people’s will.
Statement 2 is correct- The business of the Parliamentary
189. Solution (b) Committees can be transacted in either Hindi or English. This
Exp) Option b is the correct answer allows Members to participate and contribute to committee
discussions using the language of their preference.
Statement 1 is correct: The casting vote in the Parliament
is cast by the Speaker or a person acting as the Speaker. The Statement 3 is correct- The minutes of the Parliamentary
Speaker is the presiding officer of the House and has the Committees are invariably prepared in Hindi or English.
authority to exercise the casting vote when required. This ensures that the proceedings and decisions of the
committees are documented and accessible to all members
Statement 2 is incorrect: The casting vote is not cast in
in the official languages of the Parliament.
addition to voting in the first instance. It is only used in
specific situations when there is an equality of votes on a 194. Solution (b)
particular matter.
Exp) Option b is the correct answer.
Statement 3 is correct: The casting vote is cast in the case
of equality of votes. In parliamentary proceedings, if there State No. of MP’s in the Rajya Sabha
is a tie or equal number of votes on a particular issue, the Bihar 16
Speaker or the person acting as the Speaker can exercise the
Uttar Pradesh 31
casting vote to break the tie and reach a decision.
Statement 4 is incorrect: The casting vote is not always cast Karnataka 12
to maintain the status quo. The Speaker or the person acting Rajasthan 10
as the Speaker can exercise the casting vote based on their
Andhra Pradesh 11
discretion or in accordance with parliamentary rules and
procedures. Gujarat 11
Kerala 9
190. Solution (d)
Exp) Option d is the correct answer 195. Solution (b)
Referring an Ordinary Bill to a Joint Committee of both Exp) Option b is the correct answer.
Houses of Parliament requires the concurrence of both Statement 1 is correct- The procedure for amending the
Houses. The purpose of a Joint Committee consisting of
Constitution is provided in Article 368 of the Constitution of
members from both Houses of parliament is to ensure
India. According to article 368, constitution can be amended
cooperation and consensus on the matter being examined.
in three ways- Simple Majority, Special Majority, Special
191. Solution (a) Majority of the Parliament and the ratification of half of the
Exp) Option a is the correct answer state legislatures.

The Rajya Sabha members are not elected directly by the Statement 2 is correct- A bill to amend the Constitution
people. Instead, they are elected by the elected members of can be introduced in either House of Parliament. It can be
the State Legislative Assemblies. introduced as a Constitutional Amendment Bill and must go
through the prescribed legislative process before it can be
192. Solution (b) enacted into law.
Exp) Option b is the correct answer Statement 3 is correct- The special procedure outlined in
A motion of no-confidence is a parliamentary motion that Article 368 of the Constitution vests constituent powers upon
is moved against the Council of Ministers and not against the ordinary legislation. This means that the Parliament,
an individual minister, the Prime Minister, or a political while acting in its legislative capacity, has the power to amend
party. A motion of no-confidence is a parliamentary or modify the provisions of the Constitution provided that
procedure through which members of the legislative body Basic Structure of the Indian Constitution remains intact.

247 PYQ Workbook


INDIAN POLITY AND GOVERNANCE

196. Solution (c) • Additional disqualifications specified in the


Exp) Option c is the correct answer. Representation of People Act (1951) include
Insurance is a subject that comes under the Union List convictions for election offenses, corrupt practices,
(Entry 43 and 47) of the Indian Constitution. The central imprisonment of two or more years, failure to
submit election expense accounts, and having an
government has the authority to make laws related to
interest in government contracts or corporations with
insurance.
government shareholding.
Important Tips • Disqualifications also apply to individuals dismissed
State List: from government service for corruption, convictions
• It contains subjects on which the state governments for promoting enmity between groups or bribery,
have the exclusive power to make laws and regulations. and punishments related to social crimes such as
untouchability, dowry, and sati. The President’s
• It includes matters such as public health, police, public
decision, based on the Election Commission’s opinion,
order, local government, agriculture, state taxes, land
is final in determining disqualifications.
and land reforms, education, and public libraries.
• The State List consists of a total of 59 subjects listed 198. Solution (a)
under Schedule VII of the Indian Constitution. Exp) Option a is the correct answer.
Union List: Statement 1 is correct - An overseas elector must be a citizen
• The Union List contains subjects on which the central of India to be eligible for voting in the Lok Sabha elections.
government has exclusive authority to make laws and Statement 2 is correct - The name of the overseas elector
regulations. must be included in the electoral roll of the constituency
• It includes matters such as defense, foreign affairs, they would be voting from to be eligible to vote.
atomic energy, banking, currency, railways, airways, Statement 3 is correct - Overseas Indian voters are required
communication, and inter-state trade and commerce. to be physically present in India for voting, as e-voting is not
• The Union List consists of a total of 98 subjects listed allowed. They have the options of postal ballots and proxy
under Schedule VII of the Indian Constitution. voting, with certain restrictions and eligibility criteria.
Concurrent List:
199. Solution (c)
• The Concurrent List contains subjects on which both
Exp) Option c is the correct answer
the central and state governments have the power to
make laws and regulations. The Secretary General of the Lok Sabha does not work under
the Speaker with delegated authority. Instead, the Secretary
• It includes matters such as criminal law, marriage
General works independently and performs various
and divorce, bankruptcy and insolvency, adoption,
administrative functions related to the functioning of
education, forests, trade unions, bankruptcy and
the Lok Sabha. The Secretary General is the principal
insolvency, and bankruptcy and insolvency.
administrative officer of the Lok Sabha Secretariat and is
• The Concurrent List consists of a total of 52 subjects responsible for the overall management and coordination of
listed under Schedule VII of the Indian Constitution. its activities.
197. Solution (b) Important Tips
Exp) Option b is the correct answer. • In the discharge of his constitutional and statutory
According to Article 102 of the Indian Constitution, certain responsibilities, the Speaker of the Lok Sabha is assisted
conditions can lead to the disqualification of a Member of by the secretary general of Lok Sabha, (whose pay
scale, position and status etc. is equivalent to that
Parliament. Option (b) Holding the office of the Chairperson
of the highest-ranking official in the Government of
of the National Commission for Women does not come
India i.e. Cabinet Secretary).
under the office of profit.
• The secretary general remains in office must retire
Important Tips at the age of 60. The secretary general is answerable
Factors leading to disqualification from being elected as a only to the Speaker; actions cannot be discussed or
Member of Parliament (MP) in India: criticised in or outside the Lok Sabha.
• Holding any office of profit under the Union or state • On behalf of the President, the secretary-general
government, except for exempted offices. summons each Member of Lok Sabha to attend
• Being declared of unsound mind by a court or being an session of the Parliament and authenticates bills in
undischarged insolvent. the absence of the Speaker.
• Acquiring citizenship of a foreign state voluntarily or 200. Solution (b)
having allegiance to a foreign state. Exp) Option b is the correct answer

PYQ Workbook 248


INDIAN POLITY AND GOVERNANCE

Statement 1 is correct: The privileges of Members of • In a state, the state legislative council is considered as the
Parliament (MPs) are separate from the fundamental upper house of the state which have a bicameral legislature
rights guaranteed under Article 19(1)(a). The privileges of and the state Legislative assembly is called as lower
MPs are defined by the Constitution and laws passed by house house of the state. One third of the Members of
the Parliament, and they are not subject to the limitations legislative council are elected by electorate consisting
imposed by Article 19(1)(a). of local bodies and authorities (option c is incorrect)
Statement 2 is correct: While the privileges of MPs are • At present there are 6 out of 28 states have a legislative
protected, they are subject to certain limitations. Articles Council.
20-22 of the Indian Constitution provide specific rights and
• The members of the Legislative council can’t be more
protections regarding protection against self-incrimination,
than one-third of the membership of the state legislative
protection against double jeopardy, and protection of
assembly and its composition can’t be less than 40
personal liberty. Article 32 allows individuals to approach
members.
the Supreme Court for the enforcement of their fundamental
rights. Therefore, the privileges of MPs must be read subject Important Tips
to these constitutional provisions. • India has a bicameral system of legislatures both at the
Statement 3 is incorrect: The privileges of MPs grant them centre and in some states.
immunity from civil proceedings only and not from criminal • At the state level, the equivalent of the Lok Sabha is
prosecution. MPs are not immune from criminal charges the Vidhan Sabha or Legislative Assembly and that of
and can be prosecuted for offenses committed like any other the Rajya Sabha is the Vidhan Parishad or Legislative
citizen. Council.
Statement 4 is incorrect: While MPs enjoy certain • Article 169 provides for creation and abolition of
protections for their speech and conduct within Parliament, legislative councils in states.
these privileges do not extend to their private or personal
capacity outside of parliamentary proceedings. Outside of 203. Solution (d)
Parliament, MPs are subject to the same laws and regulations Exp) Opton d is the correct answer
as any other citizen, and their freedom of speech is governed Rajya Sabha being a federal chamber - representing States/
by the general laws of the land. Union territories, enjoys certain special powers:
Important Tips a) To empower Parliament to make laws in respect of any
Individual Privileges of the Members of the Parliament matter enumerated in the State List in the national
interest by adopting a resolution to this effect (article
• No arrest of the member of the parliament can take
249), (option d is correct)
place during its session. Also, members can’t be
arrested 40 days before and after the session’s beginning b) Creation of All India Services (article 312)
and end of the session. c) Approving Proclamations (issued under article 352 or
• The members of parliament are entitled to the article 356 or article 360) if the Lok Sabha stand dissolved
freedom of speech in the houses. They are not liable or the dissolution of the Lok Sabha takes place within the
to any court proceedings for the speech given in the period allowed for the approval of the Proclamation by
parliament or its committees. However, it is regulated Parliament.
using the rules guiding such provisions of the house. d) To Initiate the proceeding for removal of Vice President
• They are exempted from jury service. They can refuse is also the exclusive jurisdiction of Rajya Sabha i.e. The
to give evidence and appear as a witness in a case resolution for removal of Vice President can only be
pending in a court when Parliament is in session. initiated in Rajya Sabha and not in Lok Sabha.
201. Solution (d) Important Tips
Exp) Option d is the correct answer The Council of States (Rajya Sabha) was First Constituted
The process of appointing Members of Parliamentary on 3rd April, 1952.
Committees involves a combination of appointment, • The Council of States as Rajya Sabha in Hindi on 23rd
election, and nomination. In the case of some committees, August, 1954.
the Members are appointed or elected through a motion • Rajya Sabha is a permanent and a continuous chamber
made and adopted by the Speaker or Chairman. In other where representatives are elected for the house.
cases, Members may be nominated directly by the Speaker • The First Chairman of Rajya Sabha was Dr. S. Radhakrishnan.
or Chairman without the need for a motion. Nominations • Dr. S. Radhakrishnan was the first Chairman of
are made based on various considerations such as party Rajya Sabha for two consecutive terms (13.5.1952 to
representation, expertise, and experience. 12.5.1962).
202. Solution (c) • The First Deputy Chairman of Rajya Sabha was Shri
Exp) Option c is the correct answer. S.V. Krishnamoorthy Rao.

249 PYQ Workbook


INDIAN POLITY AND GOVERNANCE

204. Solution (a) budget year in three separate parts viz. Consolidated
Exp) Option a is the correct answer Fund of India, Contingency Fund of India and Public
Account of India.
• To discuss state matters in general condition does not lie
under procedure and conduct of business of Parliament 208. Solution (d)
under Article 246 of the Constitution. (option a is not
Exp): Option d is the correct answer.
correct)
• Parliament has right to make laws on Union list and Third It contains the forms of oath and affirmation
Concurrent list of Schedule seven. Schedule for:
• Parliament can discuss the issue of Police action against • Union Ministers of India
the Schedule Castes and Tribes under the Scheduled • Parliament Election Candidates
Castes and Tribes (Prevention of Atrocities) Act, 1989
• Members of Parliament (MPs)
Issue of Industrial labours come under Concurrent list,
so it can be discussed in Parliament. • Supreme Court Judges
• Parliament can also discuss issues in dealing with violent • Comptroller and Auditor General
disturbances in an undertaking under the control of the • State Ministers
Union Government • State Legislature Elections’ Candidates
Important Tips • State Legislature Members
Rules of Procedure and Conduct of Business in the • High Court Judges
Council of States have been formulated under Article 118
Fourth It contains the provisions in relation to
of the Constitution wherein each House of Parliament is
Schedule the allocation of seats for States and Union
required to make rules for regulating its Procedure and
Territories in the Rajya sabha (option d is
conduct of business.
correct)
205. Solution (d) Fifth It contains provisions in relation to the
Exp) Option d is the correct answer Schedule administration and control of scheduled areas
and scheduled tribes
• The legislature of states involve Governor, Legislative
Assembly and the Legislative Council. Sixth It contains provisions in relation to the
• Under article 164 of the Indian constitution chief Schedule administration of tribal areas in the states of
minister shall be appointed by the governor. Assam, Meghalaya, Tripura and Mizoram

Important Tips 209. Solution (b)


• Part VI of the Constitution containing Articles 153 to Exp) Option b is the correct answer.
167 deals with the government in the States. • According to article 102 of the constitution of India:
• The state executive consists of the Governor, the Chief 1. A person shall be disqualified for being chosen as, and for
Minister, the Council of Minister and the Advocate being, a member of either House of Parliament:
General of the state.
(a) if he holds any office of profit under the Government of
206. Solution (d) India or the Government of any State, other than an office
Exp) Option d is the correct answer. declared by Parliament by law not to disqualify its holder

• According to article 108 a joint sitting is called by the (b) if he is of unsound mind and stands so declared by a
President and presided over by the speaker. competent court;

• A bill which requires special majority in both the houses (c) if he is an undischarged insolvent;
cannot be introduced in joint sitting of the Parliament (d) if he is not a citizen of India, or has voluntarily acquired
• Even for money bill Rajya sabha can only amended some the citizenship of a foreign State, or is under any
changes and it is Lok sabha only which passes the money acknowledgement of allegiance or adherence to a foreign
bill. Hence there is no point in introducing the money State ;
bill in the joint sitting of the Parliament (e) if he is so disqualified by or under any law made by
Parliament Explanation
207. Solution (c)
2. A person shall be disqualified for being a member of
Exp) Option c is the correct answer.
either House of Parliament if he is so disqualified under
• Annual Financial Statement is a document presented to the Tenth Schedule.
the Parliament every Financial Year as a part of Budget
process under Article 112. Important Tips

• This document comprises the receipts and expenditures Disqualification of MPs & MLAs according to
of the government of current year, previous year and representation of people act 1951:

PYQ Workbook 250


INDIAN POLITY AND GOVERNANCE

a. A member of a house belonging to any political party 214. Solution (b)


becomes disqualified for being a member of the house: Exp) Option b is the correct answer.
• if he voluntarity gives his membership of such political • It is introduced in the Lok Sabha to draw the attention
party. of the House to a definite matter of urgent public
• if he votes or abstains from voting importance.
b. He/she doesn’t become disqualified if it is expelled by • It involves an element of censure against the
the political party from which he/she had been elected government.
to the house. (statement b is incorrect) • It can be moved only in Lok Sabha
• An adjournment of the house suspends the work in a
210. Solution (b)
sitting for a specified time, which may be hours, days
Exp) Option b is the correct answer. or weeks.
• Pro tem is a latin phrase meaning “for the time being”. • The power of adjournment lies with the presiding
• The Protem Speaker is mainly an operating and officer of the House.
temporary Speaker. • The subject of the motion must have a direct or indirect
• After Every general election, usually a senior most relation to the conduct or default on the part of the
member of Lok Sabha is appointed by the President as Union Government. (option a is incorrect)
Protem Speaker
Important Tips
• The Protem Speaker has all the powers of the Speaker
Special powers of Rajya Sabha:
and presides over the first sitting of the newly-elected
Lok Sabha a) Legislation on matters of the state list
• His main duty is to administer oath to the new members. b) Proclamation of the President rule
c) Creation of All India Services
211. Solution (a)
d) Removal of vice-President.
Exp) Option a is the correct answer.
Special powers of Lok Sabha:
• As per Article 79, Parliament shall consist of the
a. To pass a motion of no-confidence against the Council
President and the two Houses, i.e. House of the People
of Ministers.
and Council of States. (Option a is correct)
b. Money bills can only be introduced in the Lok Sabha.
• The President is indirectly elected by an electoral
college consisting of the elected representatives of the c. In matters about non-financial (ordinary) bills.
Parliament of India and the legislative assemblies of 215. Solution (c)
the states of India and the union territories of Delhi
Exp) Option c is the correct answer.
and Puducherry. (Option b is incorrect)
• Ordinary Bill can be introduced without the
212. Solution (b) recommendation of the President of India.
Exp) Option b is the correct answer. • Features of an ordinary bill :
• As per Article 112 of the Indian Constitution, the a) It can be introduced either in the Lok Sabha or the Rajya
President shall in respect of every financial year cause to Sabha.
be laid before both the Houses of Parliament a statement
b) It can be introduced either by a minister or by a private
of the estimated receipts and expenditure of the
member.
Government of India for that year, in this part referred to
c) It is introduced without the recommendation of the
as the ‘Annual Financial Statement’.
President.
213. Solution (d) d) It can be amended or rejected by the Rajya Sabha.
Exp) Option d is the correct answer. e) It can be detained by the Rajya Sabha for a maximum
• A no-confidence motion is a parliamentary motion that period of six months.
is moved in the Lok Sabha against the entire council f ) It doesn’t require the certification of the speaker,when
of Ministers (not an individual minister). (option a is transmitted to the Rajya Sabha.
incorrect)
216. Solution (d)
• It means that the ministry stays in office so long as it
enjoys confidence of the majority of the members of the Exp) Option d is the correct answer.
Lok Sabha. • Departmentally Related Standing Committee (DRSC)
• According to Dr Ambedkar, “Collective responsibility evolved as a Parliamentary innovation in 1993.
can be achieved only through the instrumentality of the • It has evolved as a mini parliament with proportional
Prime Minister. representation of all parties.

251 PYQ Workbook


INDIAN POLITY AND GOVERNANCE

• There are 24 DRSCs at present with 31 members each 21 219. Solution (c)
from Lok sabha and 10 from Rajya Sabha. Exp) Option c is the correct answer.
• The term of office of these committees doesn’t exceed
• The Deputy Speaker of the Lok Sabha is the vice-
one year.
Presiding officer of the Lok Sabha.
• The Parliamentary committees are of two kinds – standing
• He/She is elected from amongst the member Lok Sabha is
or permanent committees and ad hoc committees
the lower house of the Parliament of India
Important Tips
• Deputy speaker acts as the presiding offier in case of
• The Parliamentary Committee on Empowerment of leave or absence of speaker of the Lok Sabha.
Women was constituted in 1997.
• He holds the office till either he ceases to be member of
• It has 30 members, 20 from the Lok Sabha & 10 from
the Lok Sabha or resigns himself.
the Rajya Sabha.
• The mandate of the Committee is to review and monitor the Important Tips
measures taken by the Union Government in the direction of • The institutions of Speaker & deputy speaker originated
securing for women equality, status and dignity in all matters. in India in 1921 under the provisions of GOI act of
• The functions of the Committee include, inter-alia, 1919
considering the reports submitted by the National • The Speaker is elected by the Lok Sabha from amongst
Commission for Women (NCW) and to report on
its members. (Article 93)
the measures taken by the Union Government for
improving the status/condition of women. (statement • The date of election of the Speaker is fixed by the
d is incorrect). President.
• The date of election of the deputy Speaker is fixed by
217. Solution (d)
the Speaker.
Exp) Option d is the correct answer.
• When Lok Sabha is dissolved. Speaker does not vacate
• As per Article 248, Parliament has exclusive power to
office and continues till the newly elected Lok Sabha
make any law with respect to matters not enumerated in
the Concurrent List and State List. meets.

• As per Article 252, Parliament has power to legislate • G.V. Mavalankar & Ananthasayanam Ayyangar
for two or more states by consent and adoption of such had the distinction of being the first speaker & first
legislation by any other state. deputy speaker (respectively) of the Lok Sabha.
• Article 253 gives power to Parliament to make any law for 220. Solution (c)
any part of territory of India for implementing any treaty,
agreement or convention with any other country. Exp) Option c is the correct answer.
• Public Accounts is fund accounts for flows for those
Important Tips
transactions where the government is merely acting as
• The Parliament is the legislative organ of the Union
a banker.
Government.
• The committee on Public Accounts was first constituted
• Part V of the Constitution from Article 79 to Article
in 1921 under the act of 1919.
122 provides provisions for the Parliament.
• It is constituted by Parliament each year for examination
• Article 79: The Parliament to consist (President +
council of states + House of people) of accounts.

• Powers of the Parliament: Legislative Power, Financial • Apart from the reports of the Comptroller and Auditor
power, Electoral Functions, Amendment of the General of India on appropriation accounts of the Union
Constitution, Power over Executive. Government, the committee also examines the various audit
reports of Comptroller and Auditor General on revenue
218. Solution (a)
receipts, expenditure by various ministries/ Department of
Exp) Option a is the correct answer
Government and accounts of autonomousbodies.
• Article 4 itself declares that law made for the admission
or establishment of new States under Article 2 and Important Tips
formation of new States and alteration of area,boundaries • most important financial committee of the Parliament.
or names of existing State under Article 3 are not to be • 22 members [15 (LS) + 7 (RS)]
considered as amendments of the Constitution under • Term (1 year); No membership of minister; Chairman
Article 368 This means that such laws can be passed by
appointed by Speaker;
a simple majority and by the ordinary legislative process
• Chairman from opposition (Convention)
• The amendment of the 1st and 4th Schedule is related to
• Functions: Audits the appropriation and Revenue
the federal structure of the polity with actually amending
Account of the CAG.
the Constitution. Hence, statement 3 is incorrect.

PYQ Workbook 252


INDIAN POLITY AND GOVERNANCE

221. Solution (c) 222. Solution (b)


Exp) Option c is the correct answer. Exp) Option b is the correct answer.
• The first estimates committee in the post-independence • As per Article 108 of Constitution, a Joint session of
was constituted in 1950 on the recommendation of John Parliament can be summoned in the following situation:
Mathai, Finance Minister. • To resolve deadlock when any house of the Parliament
The function of the committee are to passes a bill and when the other House rejects this bill, or
• report what economies, improvements in organisation, • The houses do not agree on the amendments made to the
efficiency or administrative reform consistent with the bill, or
policy underlying estimates can be affected. • More than six months elapsed with the bill being received
• suggest the form in which the estimates are to presented by the other House without it being passed. However, in
to Parliament. calculating the period of six months, those days are not
• suggest alternative policies in order to bring about considered when house is prorogued or adjourned for
efficiency and economy in administration. more than 4 consecutive days.
• examine whether the money is well laid out within the • According to the Indian Constitution, there are two
limits of the policy implied in the estimates exceptions when a joint sitting cannot be summoned.
• They are for the following bills: Money Bill and
Important Tips constitution amendment bill
Estimates Committee • The joint sitting of the Parliament is called by the
• 30 members (Lok Sabha) President and is presided over by the Speaker
• Members elected by Proportional representation; • Article 108 has laid down the provisions for calling a
Term (1 year); No Membership of minister; Chairman joint sitting it does not establish the supremacy of Lok
appointed by Speaker. Sabha.
• Function: Scrutinizes the Estimates Presented in the • It is not frequently resorted to establish the supremacy of
Budget the Lok sabha. (statement 4 is incorrect).

253 PYQ Workbook


INDIAN POLITY AND GOVERNANCE

INDIAN POLITY AND GOVERNANCE


LOCAL GOVERNMENT – UNION TERRITORIES – SPECIAL STATUS AREAS
*This unit consists of questions from Rural and Urban local Governments, Union Territories and
special status areas.

7.1. UPSC CSE Previous Years’ Questions (c) Ministry of Rural Development
(d) Ministry of Tribal Affairs
1. With reference to “Scheduled Areas’ in
India, consider the following statements: 4. Under which Schedule of the Constitution
of India can the transfer of tribal land to
1. Within a State, the notification of an area
private parties for mining be declared null
as Scheduled Area takes place through an
and void? [UPSC CSE Pre. 2019]
Order of President.
(a) Third Schedule
2. The largest administrative unit forming
(b) Fifth Schedule
the Scheduled Area is the District and
(c) Ninth Schedule
the lowest is the cluster of villages in the
(d) Twelfth Schedule
Block.
3. The Chief Ministers of the Concerned 5. Local self-government can be best explained
States are required to submit annual as an exercise in [UPSC CSE Pre. 2017]
reports to the union Ministry on the (a) Federalism
administration of Scheduled Areas in the (b) Democratic decentralization
States. (c) Administrative delegation
How many of the above statements are (d) Direct democracy
correct? [UPSC CSE Pre 2023] 6. Consider the following statements:
(a) Only one 1. The minimum age prescribed for any
(b) Only two person to be a member of Panchayat is 25
(c) All three years.
(d) None
2. A Panchayat reconstituted after
2. If a Particular area is brought under the premature dissolution continues only for
Fifth Schedule of the Constitution of India, the remainder period.
which one of the following statements best Which of the statements given above is/are
reflects the consequence of it? correct? [UPSC CSE Pre. 2016]
[UPSC CSE Pre. 2022] (a) 1 only
(a) This would prevent the transfer of land of (b) 2 only
tribal people to non-tribal people. (c) Both 1 and 2
(b) This would create a local self- governing (d) Neither 1 nor 2
body in that area.
(c) This would convert that area into a Union 7. The fundamental object of Panchayati
Territory. Raj system is to ensure which among the
(d) The State having such areas would be following?
declare a Special Category State. 1. People’s participation in development
2. Political accountability
3. At the national level, which ministry
is the nodal agency to ensure effective 3. Democratic decentralization
implementation of the scheduled Tribes 4. Financial mobilization
and Other Traditional Forest Dwellers Select the correct answer using the code given
(Recognition of Forest Rights) Act, 2006? below. [UPSC CSE Pre. 2015]
[UPSC CSE Pre. 2021] (a) 1, 2 and 3 only
(a) Ministry of Environment, Forest and (b) 2 and 4 only
Climate Change (c) 1 and 3 only
(b) Ministry of Panchayati Raj (d) 1, 2, 3 and 4

PYQ Workbook 254


INDIAN POLITY AND GOVERNANCE

8. Under the Scheduled Tribes and Other Select the correct answer from the code given
Traditional Forest Dwellers (Recognition below. [U.P.P.C.S. (Pre) 2022]
of Forest Rights) Act, 2006, who shall (a) (A) is true, but (R) is false
be the authority to initiate the process (b) Both (A) and (R) are true and (R) is the
for determining the nature and extent of correct explanation of (A)
individual or community forest rights or (c) (A) is false, but (R) is true
both? [UPSC CSE Pre. 2013] (d) Both (A) and (R) are true, but (R) is not
(a) State Forest Department the correct explanation of (A)
(b) District Collector/Deputy Commissioner
12. When was the Panchayats (Extension to
(c) Tahsildar/Block Development Officer/
the Scheduled Areas) Act passed by Indian
Mandal Revenue Officer
Parliament? [U.P.P.C.S. (Pre) 2022]
(d) Gram Sabha
(a) 1996
9. In the areas covered under the Panchayat (b) 1993
(Extension to the Scheduled Areas) Act, (c) 1998
1996, what is the role/power of Gram Sabha? (d) 1995
1. Gram Sabha has the power to prevent
13. Which region of the country conferred
alienation of land in the Scheduled Areas.
special status under Article 371-J of the
2. Gram Sabha has the ownership of minor Constitution of India?
forest produce.
[U.P.P.C.S. (Pre) 2020]
3. Recommendation of Gram Sabha is
(a) Nagaland
required for granting prospecting licence
(b) Hyderabad and Karnataka
or mining lease for any mineral in the
(c) Maharashtra and Gujarat
Scheduled Areas.
(d) Ladakh
Which of the statements given above is/are
correct? [UPSC CSE Pre. 2012] 14. Reservation of seats in Panchayat elections
(a) 1 only for Scheduled Castes shall not apply to the
(b) 1 and 2 only State of- [U.P.P.C.S. (Pre) 2016]
(c) 2 and 3 only (a) Uttar Pradesh
(d) 1, 2 and 3 (b) Assam
(c) Arunachal Pradesh
10. Which one of the following authorities (d) Haryana
makes recommendation to the Governor of a
State as to the principles for determining the 15. Who among the following constitutes
taxes and duties which may be appropriated ‘Finance Commission’ to review the
by the Panchayats in that particular State? financial position of Panchayats?
[UPSC CSE Pre. 2012] [U.P.P.C.S. (Pre) 2015]
(a) District Planning Committees (a) Chief Minister of the concerned State
(b) State Finance Commission (b) Finance Minister of the concerned State
(c) Finance Ministry of that State (c) The Governor of the concerned State
(d) Panchayati Raj Ministry of that State (d) Panchayati Raj Minister of the concerned
State
7.2. Other Examination Previous Years’ 16. Which of the following are Constitutional
Questions Authorities? Select the correct answer using
11. Given below are two statements, one is code given below:
labelled as Assertion (A) and the other as 1. State Election Commission
Reason (R): 2. State Finance Commission
Assertion (A): The 73rd Amendment of the 3. District Panchayat
Constitution is considered to be a watershed 4. State Electoral Officer
in the history of local self-government in Code: [U.P.P.C.S. (Pre) 2012]
India. (a) 1 and 2 Only
Reason (R): The 73rd Constitution (b) 1, 2 and 3 Only
Amendment conferred the much-coveted (c) 2, 3 and 4 Only
Constitutional status on Panchayats. (d) 1, 2, 3 and 4

255 PYQ Workbook


INDIAN POLITY AND GOVERNANCE

17. Consider the following statements and (a) Rajasthan-Telangana-Maharashtra


select the correct answer using the code (b) Andhra Pradesh-Jharkhand-Odisha
given below: (c) Assam-Meghalaya-Tamil Nadu
Assertion (A): The Union Finance (d) Himachal Pradesh-Gujarat-Chhattisgarh
Commission does not have any role in 23. Consider the following functions of State
considering measures for providing financial Election Commission and select the correct
assistance to local bodies. answer using the code given below?
Reason (R): In spite of the 73rd and 1. Prepares the electoral rolls for Panchayats
74th Constitutional Amendments, local and Municipalities elections in the State.
government continues to be a state subject in 2. Conducts elections for Panchayats and
the Seventh Schedule of the Constitution. Municipalities of the State.
Code: [U.P.P.C.S. (Mains) 2011] 3. Conducts elections for other bodies in
(a) Both (A) and (R) are true, and (R) is the addition to Panchayats and Municipalities
correct explanation of (A). in accordance with the directions given
(b) Both (A) and (R) are true, but (R) is not by the Governor.
the correct explanation of (A). 4. Prepares the electoral rolls and conducts
(c) (A) is true, but (R) is false. elections as assigned by Election
(d) (A) is false, but (R) is true. Commission of India.
18. Article 371 of the Constitution makes Code: [R.A.S./R.T.S.(Pre) 2013]
special provisions for which of the following (a) (1) and (2)
States? [U.P.P.C.S. (Mains) 2011] (b) (3) and (4)
(a) Jammu and Kashmir (c) (1), (2) and (4)
(b) Maharashtra and Gujarat (d) (4) only
(c) Nagaland 24. Which of the following bears the
(d) Andhra Pradesh responsibility of planning, execution and
19. Article 371-B of the Constitution of India implementation of MNREGA schemes?
makes special provisions for which of the [R.A.S./R.T.S.(Pre) 2012]
following State/s? (a) Gram Sabha
[U.P.P.C.S. (Mains) 2010] (b) Gram Panchayat
(c) State Government
(a) Maharashtra and Gujarat
(d) DRDA
(b) Assam
(c) Nagaland 25. Who speaks of India’s self-reliant social
(d) Manipur system ‘The rural system in India is a tiny
Republic’? [M.P.P.C.S. (Pre) 2020]
20. Which of the following States does not have
any Panchayati Raj Institution? (a) Charles Metcalfe
(b) Burke
[U.P.P.C.S. (Mains) 2005] (c) Mill
(a) Assam (d) Cunningham
(b) Kerala
(c) Nagaland 26. The Committee which recommended a
(d) Tripura three-tier Panchayati Raj System in India
was: [M.P.P.C.S. (Pre) 2014]
21. Which Article of the Indian Constitution (a) Ashok Mehta Committee
deals with special provisions with respect to (b) Balwant Rai Mehta Committee
different States? [U.P.P.C.S. (Pre) 1997] (c) G.K.V. Rao Committee
(a) Art. 369 (d) L.M. Singhvi Committee
(b) Art. 370
(c) Art. 371 27. The State Government controls over urban
(d) Art. 372 local bodies:
(A) in legislative matters
22. In which of the following group of the States,
PESA [Panchayats (Extension to Scheduled (B) in financial matters
Areas) Act], 1996 is not enforceable? (C) in personnel matters
[R.A.S./R.T.S. (Pre) 2016] (D) in respect of citizens’ grievances

PYQ Workbook 256


INDIAN POLITY AND GOVERNANCE

Select the correct answer from the codes (a) Both Assertion (A) and Reason (R)
given below. are true, and Reason (R) is correct
[Chhattisgarh P.C.S. (Pre) 2021] explanation of Assertion (A)
(a) (A) and (B) (b) Both Assertion (A) and Reason (R)
(b) (B) and (C) are true, but Reason (R) is not correct
(c) (B), (C) and (D) explanation of Assertion (A)
(d) (A), (B) and (C) (c) Assertion (A) is true, but Reason (R) is
false
28. Which of the following pairs is/are correctly (d) Assertion (A) is false, but Reason (R) is
matched? true
A. Rationalization of : V.R. Rao 31. What is correct?
Panchayat Statistics (i) In a Gram Panchayat there can be more
B. Panchayat Raj Training : A.S. Trivedi than one village.
C. Working Group : S.D. Mishra (ii) Meeting of Gram Sabha held in each
on Panchayats and village.
Cooperative (iii) Meeting of Gram Sabha held in the
D. A d m i n i s t r a t i v e : G.V.K. Rao headquarter of Gram Panchayat.
Arrangement for Rural (iv) Meeting of Gram Panchayat held in each
Development and Poverty village.
Alleviation Programmes (v) Meeting of Gram Panchayat held in the
Code: [Chhattisgarh P.C.S. (Pre) 2021] headquarter of Gram Panchayat.
(a) (A), (C) and (D) Code: [Chhattisgarh P.C.S. (Pre) 2018]
(b) Only (B) (a) (i), (iii), (iv)
(c) (A), (B) and (C) (b) (ii), (iv), (v)
(d) (A) and (B) (c) (ii), (iii), (iv)
(d) (i), (ii), (v)
29. Which of the following has been provided
in the 74th Amendment Act, 1992 of the 32. If a candidate expired before polling in
Constitution? Panchayat election, then what shall happen?
(i) Municipal Committees i. Polling shall be postponed.
(ii) Ward Committees ii. Polling shall not be postponed.
(iii) Metropolitan Planning Committee iii. If only one candidate remains, then polling
will be postponed.
(iv) District Planning
iv. If more than one candidate remains, then
Select the correct answer from the codes polling will not be postponed.
given below.
Code: [Chhattisgarh P.C.S. (Pre) 2017]
[Chhattisgarh P.C.S. (Pre) 2021]
(a) i, ii
(a) (i) and (iv) (b) ii, iii
(b) (i), (ii), (iii) and (iv) (c) iii, iv
(c) (i), (ii) and (iv) (d) ii, iv
(d) (ii), (iii) and (iv) (e) None of the above
30. In the following questions, there are two 33. What is correct about the tenure of
statements. One is labelled as Assertion (A) municipal council?
and other is labelled as Reason (R): i. The tenure of municipal council is counted
Assertion (A): Municipal corporations are from the date of declaration of election
formed in big cities having a definite area and result.
population. ii. The tenure of the municipal council is
Reason (R): Urban administrations of big counted from the date of its first meeting.
cities are very complex in nature. iii. The tenure of municipal council is 5 years.
In the context of the above two statements iv. If the municipal council is dissolved in
which of the following is correct? two years, then a new municipal council
[Chhattisgarh P.C.S. (Pre) 2021] will be organised for three years.

257 PYQ Workbook


INDIAN POLITY AND GOVERNANCE

v. If the municipal council is dissolved in two (a) To plan minor water bodies
years, then a new municipal council will be (b) To control over institutions in all social
organised for 5 years. sectors
Code: [Chhattisgarh P.C.S. (Pre) 2017] (c) To control on tribal sub plans
(a) i, iii, v (d) To exercise other functions as conferred
(b) ii, iii, iv by the State Govt.
(c) i, iii, iv (e) All of the above
(d) ii, iii, v
38. Who presides over the meeting of Gram
(e) None of the above
Sabha in Scheduled Areas?
34. What is correct about the wards committee [Chhattisgarh P.C.S. (Pre) 2015]
of Nagar Panchayat? (a) Sarpanch
i. The Wards committee is formed with some (b) Deputy Sarpanch
wards of nagar Panchayat. (c) Sarpanch or Deputy Sarpanch
ii. Councillor elected from the wards become (d) Schedule Tribe member present who is
its member. elected by Gram Sabha
iii. Two persons are nominated in the (e) None of the above
committee residing in the ward. 39. Which of the following statements about
iv. Chairman of the wards committee is the Panchayati Raj System are correct?
nominated by the President of nagar 1. All seats in a Panchayat are filled by
Panchayat. persons chosen by direct election.
Code: [Chhattisgarh P.C.S. (Pre) 2017] 2. The ‘Gram Sabha’ consists of persons
(a) i only registered in the village electoral rolls.
(b) i, ii only 3. The Chairperson of a Panchayat is elected
(c) i, ii, iii only
in accordance with a law passed by each
(d) i, ii, iii, iv
state.
(e) None of the above
4. All states in India have a three-tier system
35. Who notify the alteration in limits of Block of Panchayats.
and Zila Panchayat?
Select the correct answer using the codes
[Chhattisgarh P.C.S. (Pre) 2016] given below: [CDS 2021(I)]
(a) Governor (a) 1 and 4
(b) State Election Commission
(b) 1, 2 and 3
(c) Divisional Commissioner
(c) 2 and 3
(d) Collector and District Returning Officer
(d) 2, 3 and 4
(e) None of the above
36. If there is dispute between Panchayat and 40. Which of the following statements is/are
Cantonment Board, who will decide finally? correct?
[Chhattisgarh P.C.S. (Pre) 2016] 1. The elections to the Panchayat are
conducted by the State Election
(a) Collector, subject to approval of
Divisional Commissioner Commission.
(b) Divisional Commissioner, subject 2. The State Election Commissioner
to approval of Panchayat and Rural is appointed by the Chief Election
Development Department Commissioner of India.
(c) Panchayat and Rural Development 3. The State Legislatures have the power
Department subject to approval of State to make laws on all matters relating to
Government Panchayat elections.
(d) State Government, subject to approval of Select the correct answer using the codes
Central Government given below: [CDS 2021(I)]
(e) None of the above
(a) 2 only
37. What is correct about the powers of Zila (b) 1 and 2 only
Panchayats of Scheduled Areas? (c) 1, 2 and 3
[Chhattisgarh P.C.S. (Pre) 2016] (d) 1 and 3 only

PYQ Workbook 258


INDIAN POLITY AND GOVERNANCE

41. Which one of the following is not an objective of 1. It makes it mandatory for all states
MGNREGA?- [CDS 2020 (I)] to establish a three-tier system of
(a) Providing up to 100 days of skilled labour Government.
in a financial year 2. Representatives should be directly elected
(b) Creation of productive assets for five years.
(c) Enhancing livelihood security 3. There should be mandatory reservation
(d) Ensuring empowerment to women of one-third of all seats in all Panchayats
at all levels for women.
42. Which one of the following statements with
Select the correct answer using the codes
regard to the functioning of the Panchayats given below: [CDS Pre.2017 II]
is not correct? - [CDS 2020 (I)] (a) 1 and 2
(a) Panchayats may levy, collect and (b) 2 and 3
appropriate taxes, duties, tolls, etc. (c) 1, 2 and 3
(b) A person who has attained the age of 25 (d) Only 3
years will be eligible to be a member of a
47. Which of the following statements with
Panchayat. regard to Panchayats in India are correct?
(c) Every Panchayat shall ordinarily continue
1. Seats in a Panchayat are filled by direct
for five years from the date of its first election from the territorial constituencies
meeting. in the Panchayat area.
(d) A Panchayat reconstituted after premature 2. The Gram Sabha is the body of persons
dissolution shall continue only for the registered in the electoral rolls relating to
remainder of the full period. a village within the Panchayat area.
43. The power to legislate on all matters relating 3. The Panchayats work on the principle of
to elections to Panchayats lies with- constitutional autonomy.
[CDS 2020 (I)] 4. The State Legislature may by law endow
the Panchayats with the power and
(a) the Parliament of India
authority to enable them to function.
(b) the State Legislatures
(c) the State Election Commission Select the correct answer using the codes
given below: [CDS Pre.2017 I]
(d) the Election Commission of India
(a) 1, 2 and 3
44. The 11th Schedule of the Constitution of (b) 2, 3 and 4
India distributes powers between- (c) 1, 2 and 4
[CDS 2020 (I)] (d) 1 and 4
(a) the Union and the State Legislatures 48. The Constitution of India contains
(b) the State Legislatures and the Panchayat no provision for the Constitution of
(c) the Municipal Corporation and the municipalities in every state for
Panchayat [CAPF 2021]
(d) the Gram Sabha and the Panchayat (a) a Nagar Panchayat
(b) a Municipal Council
45. Which one of the following regarding
(c) a Municipal Corporation
the tenure of the elected members of the
(d) an Urban Panchayat
Autonomous District Council is correct
[CDS Pre.2018 II] 49. Which of the following is/are correct in
relation to the Panchayats?
(a) Five years from the date of election
1. Only the Parliament may endow
(b) Five years from the date appointed for
Panchayats with powers and authority.
the first meeting of the council after the
2. A state may devolve powers to Panchayats
election
in preparation of plans for economic
(c) Six years from the date of administration development and social justice.
of oath
3. A state may devolve powers to Panchayats
(d) Six years from the date of election in implementation of schemes for
46. Which of the following statements about economic development and social justice.
the 73rd and 74th Constitution Amendment Select the correct answer using the codes
Act is/are correct? given below: [CAPF 2020]

259 PYQ Workbook


INDIAN POLITY AND GOVERNANCE

(a) 1 only (c) 1, 2 and 3


(b) 1 and 3 (d) 2, 3 and 4
(c) 2 and 3
54. Which of the following statements is true?
(d) 3 only
[CAPF 2014]
50. Which one of the following States does (a) The Panchayat system exists in all states
not find a mention in Article 371 of the of India except Nagaland, Meghalaya
Constitution of India? [CAPF 2018] and Mizoram. It also exists in all Union
(a) Maharashtra Territories except Delhi.
(b) Madhya Pradesh (b) The Panchayat system is introduced
(c) Gujarat in India by the Constitution 74th
(d) Karnataka Amendment Act and envisages a three-
51. Which one of the following statements with tier system of Local Governance.
regard to Panchayats is not correct? (c) Panchayat system is not provided for states
[CAPF 2018] having less than 2 million population.
(d) Gram Panchayat can continue for 3 years
(a) Members of Panchayats are elected
directly by the Gram Sabha from the date of election.
(b) The elections to Panchayats are conducted 55. Which of the following is/are true regarding Urban
by the State Election Commission Local Bodies in India?
(c) The Central Government may by law 1. The Constitution 74th Amendment
authorise a Panchayat to levy taxes Act envisages three types of urban local
(d) Every Panchayat continues for five years bodies, namely Municipal Corporation,
from the date of convening of its first Municipality and City Council.
meeting 2. Municipal Corporations are established
52. Who among the following may be the Chairman of in cities with population greater than 1
the Zila Parishad? [CAPF 2016] million.
(a) Chief Minister 3. Bombay and Calcutta Corporations were
(b) District Collector the first Municipal Corporations that
(c) Member of Parliament in the District were established during British time.
(d) Elected from its own members Select the correct answer using the codes
53. Which of the following reasons were cited given below. [CAPF 2014]
by the Ashok Mehta Committee for the Codes:
decline of the Panchayati Raj Institutions (a) 1 and 2
in India? [CAPF 2015] (b) Only 2
1. Role of Bureaucracy (c) 1 and 3
2. Lack of Political Will (d) 1, 2 and 3
3. Lack of conceptual clarity 56. The Members of Panchayat Samiti are-
4. Lack of gender parity [Jharkhand P.C.S. (Pre) 2011]
Select the correct answer using the code given (a) Directly elected by the public
below. (b) Elected by the members of Panchayat
(a) 1 and 4 (c) Nominated by District Magistrate
(b) 2 and 3 (d) Recruited on the basis of open competition

PYQ Workbook 260


INDIAN POLITY AND GOVERNANCE

SOLUTIONS

and Other Traditional Forest Dwellers (Recognition Of


7.1. UPSC CSE Previous Years’ Questions Forest Rights) Act, 2006. The ministry deals with forest and
1. Solution: (b) livelihood objectives at the national level.
Exp) Option b is the correct answer. 4. Solution: (b)
Statement 1 is correct. As per Article 244(1) of the Exp) Option b is the correct answer.
Constitution’s Fifth Schedule, “Scheduled Areas” are areas
Fifth Schedule contains provisions for administration of
that the President may declare to be such by order after
schedule and tribal areas. Under the Fifth Schedule of the
consultation with the Governor of that State.
Constitution of India, the governor of a state can make
Statement 2 is correct. District is considered as the largest regulations that prohibits or restricts transfer of land by
administrative unit which forms the Scheduled Area while Schedule Tribes.
the lowest administrative unit is the cluster of villages in the
In Samatha vs. State of Andhra Pradesh (1997), the Supreme
Block.
Court declared that the transfer of tribal land to private
Statement 3 is incorrect. The Fifth Schedule to the
parties for mining was null and void under the Fifth Schedule.
Constitution of India states in para 3 that “the Governor of
The scheduled areas are treated differently from the
each State having Scheduled Areas therein shall annually,
or whenever so required by the President, make a report other areas in the country because they are inhabited by
to the President regarding the administration of the ‘aboriginals’ who are socially and economically rather
Scheduled Areas in that State and the executive power of backward, and special efforts need to be made to improve
the Union shall extend to the giving of directions to the State their condition.
as to the Administration of the said areas.” Option a is incorrect. Third Schedule contains provisions
for Oath and affirmation of Constitutional offices like
2. Solution: (a) Ministers, Member of Parliament, Supreme court etc.
Exp) Option a is the correct answer. Option c is incorrect. Ninth Schedule contains provisions
Option a is correct. The Fifth Schedule of the Constitution that legislation which aims at implementing land reforms in
deals with the administration and control of Scheduled the country shall be exempted from Judicial review. Under
Areas as well as of Scheduled Tribes residing in any State I.R. Coelho case, supreme court brought Ninth Schedule in
other than the States of Assam, Meghalaya, Tripura and the domain of Judicial review.
Mizoram. Option d is incorrect. Twelfth Schedule contains list of
Governor can make regulations for the peace and good subjects for administration by Municipalities (local bodies).
government of a scheduled area after consulting the tribes
advisory council. Such regulations may prohibit or restrict 5. Solution: (b)
the transfer of land by tribal to non tribal members Exp) Option b is the correct answer.
or among members of the scheduled tribes, regulate the Local self-government can be best explained as an exercise in
allotment of land to members of the scheduled tribes. democratic decentralization.
Option b is incorrect. According to the provisions of Democratic decentralization helps to further democratize
Paragraph 4, under Article 244(1) of Fifth Schedule of the local self-government to enable it to enjoy more authority,
Constitution of India, the Tribes Advisory Councils (TAC) shoulder greater responsibility, take more initiative and
shall be established in each State having Scheduled Areas experience greater autonomy in the management of the
therein and, if the President so directs, also in any State affairs of the local area.
having Scheduled Tribes but not Scheduled Areas. Tribal
advisory council is an advisory body, not a governing body. 6. Solution: (b)

Option c is incorrect. Bringing any particular area under the Exp) Option b is the correct answer.
Fifth Schedule of the Constitution of India does not convert Statement 1 is incorrect. No person shall be disqualified
the area into a Union Territory. on the ground that he is less than 25 years of age if he has
Option d is incorrect. Bringing any particular area under attained the age of 21 years. A candidate must be registered
the Fifth Schedule of the Constitution of India does not get voter in the electoral roll of that Gram Panchayat and should
declared as a Special Category State. have attained age of 21 years.
Statement 2 is correct. There is a five-year term of office
3. Solution: (d) to the panchayat at every level. However, it can be dissolved
Exp) Option d is the correct answer. before the completion of its term. A Panchayat reconstituted
The Union Ministry of Tribal Affairs (MoTA) is the nodal after premature dissolution shall continue for only the
agency for the implementation of the Scheduled Tribes reminder of the period.

261 PYQ Workbook


INDIAN POLITY AND GOVERNANCE

7. Solution: (c) financial position of the panchayats in a state and makes


Exp) Option c is the correct answer. recommendations to the Governor about the principles
Grass-roots democracy was established in India not to that should govern the distribution of tax proceeds – taxes,
ensure any political accountability, but to ensure citizen’s duties, levies, toll fee collected by the state between the state
participation in governance and development leading to and its Panchayati Raj Institutions at all three levels – village
democratic decentralization. So, the term “democratic level, block level and district level.
decentralization” is an appropriate replacement for the term It also recommends the following:
“political accountability”. • Taxes, levies and fees levied or appropriated by Panchayats
Moreover, political accountability already existed in the themselves.
villages by way of elections to State legislatures. • Grants-in-aid to Panchayati Raj Institutions from the
Statement 4 is incorrect. Financial mobilization was never consolidated fund of a state.
the basis for the establishment of Panchayati raj. Local
• Ways to improve the financial position of the Panchayati
financial mobilization (by way of octroi taxes etc.) is the
Raj Institutions.
outcome of the establishment of the PRIs, not its objective.
In fact, only few states have devolved financial powers to • Measures for the overall improvement of Panchayat’s
their PRIs. finances.
Under Article 243-I of the Indian Constitution, the governor
8. Solution: (d)
of a state ensures the laying of a State Finance Commission’s
Exp) Option d is the correct answer. recommendations to the table of the state legislature. It also
Under the provisions of the Scheduled Tribes and Other includes a memorandum of action taken by the government
Traditional Forest Dwellers (Recognition of Forest Rights) on the Commission’s report.
Act, 2006, the Gram Sabha shall be the authority to initiate
the process for determining the nature and extent of 7.2. Other Examination Previous Years’
individual or community forest rights. The Gram Sabha shall Questions
pass a resolution to that effect and thereafter forward a copy
11. Solution: (b)
of the same to the Sub-Divisional Level Committee.
Exp) Option b is the correct answer.
9. Solution: (b)
Both (A) and (R) are true and (R) is the correct explanation
Exp) Option b is the correct answer.
of (A).
The Panchayat (Extension of the Scheduled Areas) Act, 1996
Assertion (A) is correct: 73rd Amendment of the
or PESA, was enacted by the Centre to ensure self-governance
Constitution is considered to be a watershed in the history
through gram sabhas for people living in scheduled areas. It
of local self-government in India. The 73rd Amendment Act
legally recognizes the right of tribal communities, residents
gave constitutional status and recognition to the panchayats
of the scheduled areas, to govern themselves through their
and enabled them to function as units of self-government
own systems of self-government. It also acknowledges their
traditional rights over natural resources. Reason (R) is correct: The 73rd Amendment of the Indian
Statement 1 is correct. Section 4(m)(iii) of PESA requires Constitution, passed in 1992, granted Constitutional status
the State to enact laws for the purpose of empowering Gram to Panchayats by adding Part IX to the Constitution, which
Sabhas and Panchayats at the appropriate level “to prevent deals with the institution of Panchayati Raj. This amendment
alienation of land in the Scheduled Areas and to take marked a significant milestone in strengthening local self-
appropriate action to restore any unlawfully alienated land government in India and devolving power to the grassroots
of a Scheduled Tribe”. level.
Statement 2 is correct. Section 4(m)(ii) of PESA endows the 12. Solution: (a)
Panchayats at the appropriate level and the Gram Sabha with
Exp) Option a is the correct answer.
the ownership of minor forest produce.
Statement 3 is incorrect. According to the provisions of the The Panchayats (Extension to the Scheduled Areas) Act,
PESA Act, 1996 the recommendations of the Gram Sabha 1996 or PESA was passed by the Indian Parliament on 24th
or the Panchayats at the appropriate level shall be made December 1996 to extend the provisions of Part IX of the
mandatory prior to grant of prospecting licence or mining Constitution relating to the Panchayats to the Scheduled
lease for minor minerals (not any mineral) in the Scheduled Areas. Scheduled Areas are areas identified by the Fifth
Areas. Schedule of the Constitution, which have predominant
population of tribal communities. The PESA Act aims at
10. Solution: (b) ensuring self-governance through traditional Gram Sabhas
Exp) Option b is the correct answer. for people living in the Scheduled Areas and empowering
Under Article 243-I of the Constitution of India, the governor them to manage their natural resources and customary
of a state is required to constitute a Finance Commission practices. The PESA Act is also called “Constitution within
every five years. A State Finance Commission reviews the the Constitution”.

PYQ Workbook 262


INDIAN POLITY AND GOVERNANCE

13. Solution: (b) Important Tips


Exp) Option b is the correct answer State Finance Commission recommends principles which
Article 371-J of the Constitution of India confers special should govern:
status on the Hyderabad-Karnataka region in the State of The distribution between the State and the Panchayats of
Karnataka. It provides for the establishment of a separate the net proceeds of the taxes, duties, tolls and fees leviable
development board for the region, equitable allocation by the State, which may be divided between them under
of funds for developmental expenditure, and equitable this Part and the allocation between the Panchayats at all
opportunities and facilities in matters of public employment, levels of their respective shares of such proceeds;
education, and vocational training for the people belonging The determination of the taxes, duties, tolls and fees which
to the said region. may be assigned to, or appropriated by, the Panchayats;
The grants-in-aid to the Panchayats from the Consolidated
Important Tips Fund of the State;
Article 371A of the Indian Constitution grants special The measures needed to improve the financial position of
provisions to the State of Nagaland. the Panchayats;
• It allows the State’s Legislative Assembly to decide Any other matter referred to the Finance Commission
whether certain central laws relating to religious or by the Governor in the interests of sound finance of the
social practices, customary law, land ownership, and Panchayats.
justice will apply in Nagaland.
16. Solution: (b)
• The Governor has a special responsibility for law and
order in Nagaland. Exp) Option b is the correct answer.

• Additionally, a regional council for Tuensang district State Election Commission: It is a Constitutional Authority.
is established. It is established under Article 243K of the Constitution
of India, which states that the superintendence, direction
There is no special provision in the Constitution of India
for Ladakh under any article. However, there have been and control of the preparation of electoral rolls for, and the
demands from civil society groups in Ladakh to include conduct of, all elections to the Panchayats shall be vested in
the region under the Sixth Schedule, which provides for a State Election Commission consisting of a State Election
the formation of autonomous administrative divisions in Commissioner to be appointed by the Governor.
tribal areas. • State Finance Commission: It is a Constitutional
Authority. It is established under Article 243I of the
14. Solution: (c)
Constitution of India, which states that the Governor of
Exp) Option c is the correct answer. each State shall, as soon as may be within one year from
Reservation of seats in Panchayat elections for Scheduled the commencement of the Constitution (Seventy-third
Castes shall not apply to the State of Arunachal Pradesh Amendment) Act, 1992, and thereafter at the expiration
because: of every fifth year, constitute a Finance Commission to
• Arunachal Pradesh does not have any Scheduled Castes review the financial position of the Panchayats and to
population as per the Census of India 2011. make recommendations to the Governor as to various
matters related to their finances.
• Article 243M(4)(b) of the Constitution exempts
Arunachal Pradesh from the application of Part IX • District Panchayat: It is a Constitutional Authority. It
relating to Panchayats, except for the provisions relating is established under Article 243B of the Constitution of
to the Gram Sabha and the reservation of seats for India, which states that there shall be constituted in every
Scheduled Tribes. State, Panchayats at the village, intermediate and district
levels in accordance with the provisions of this Part. The
• The Arunachal Pradesh Panchayati Raj Act, 1997 provides
District Panchayat is not mentioned in the Constitution
for reservation of seats for Scheduled Tribes and women
as an authority, but as a level of Panchayat.
in PRIs, but not for Scheduled Castes.
State Electoral Officer is not a Constitutional Authority.
15. Solution: (c) He or she is an officer appointed by the Election Commission
Exp) Option c is the correct answer. of India under Article 324 of the Constitution of India,
Article 243I of the Constitution of India states that the which states that the superintendence, direction and control
Governor of each State shall, as soon as may be within one of the preparation of electoral rolls for, and the conduct of,
year from the commencement of the Constitution (Seventy- all elections to Parliament and to the Legislature of every
third Amendment) Act, 1992, and thereafter at the expiration State and of elections to the offices of President and Vice-
of every fifth year, constitute a Finance Commission to President held under this Constitution shall be vested in a
review the financial position of the Panchayats and to make Commission (referred to in this Constitution as the Election
recommendations to the Governor Commission).

263 PYQ Workbook


INDIAN POLITY AND GOVERNANCE

17. Solution: (d) 21. Solution: (c)


Exp) Option d is the correct answer. Exp) Option c is the correct answer
Assertion (A) is incorrect: Union Finance Commission Article 371 of the Indian Constitution deals with special
does have a role in considering measures for providing provisions with respect to different States. It provides for
financial assistance to local bodies. According to Article special provisions and safeguards for various states to protect
280 of the Constitution of India, one of the functions of the their customs, culture, and traditions.
Union Finance Commission is to make recommendations
Important Tips
to the President as to the measures needed to augment the
Consolidated Fund of a State to supplement the resources of • Article 371 – Special provision with respect to the
the Panchayats and Municipalities in the State on the basis states of Maharashtra and Gujarat
of the recommendations made by the Finance Commission • Article 371A – Special provision with respect to the
of the State. state of Nagaland
• Article 371B – Special provision (administration) with
Reason (R) is correct: Local government is still a state respect to the state of Assam
subject in the Seventh Schedule of the Constitution, • Article 371C – Special provision (administration) with
even after the 73rd and 74th Constitutional Amendments. respect to the state of Manipur
These amendments added Part IX and Part IX-A to the • Article 371D – Special provisions with respect to the
Constitution, which deal with Panchayats and Municipalities state of Andhra Pradesh or the state of Telangana
respectively. However, these parts do not take away or • Article 371E – Establishment of Central University in
abridge the powers of the State Legislatures to make laws Andhra Pradesh
with respect to any matters relating to or concerning local • Article 371F – Special provisions with respect to the
self-government. The State Legislatures have been given wide state of Sikkim
discretion and flexibility in framing laws for local bodies, • Article 371G – Special provision with respect to the
subject to certain basic features and principles laid down in state of Mizoram
these parts. • Article 371H – Special provision (law and order) with
respect to the state of Arunachal Pradesh
18. Solution: (b)
• Article 371I – Special provision with respect to the
Exp) Option b is the correct answer. state of Goa
Article 371 of the Indian Constitution provides special • Article 371J – Special provisions with respect to the
provisions for the States of Maharashtra and Gujarat. It state of Karnataka
authorizes the President to confer special responsibility on
the Governors for the establishment of separate development 22. Solution: (c)
boards, equitable allocation of funds, and providing facilities Exp) Option c is the correct answer.
for technical education, vocational training, and employment PESA Act is not enforceable in these three states (Assam-
opportunities in specific regions of these states. Meghalaya-Tamil Nadu) because:
19. Solution: (b) Assam and Meghalaya do not have any Scheduled Areas as
per the Fifth Schedule of the Constitution of India, which are
Exp) Option b is the correct answer.
the areas covered by the PESA Act.
Article 371B provides for the special provisions for
the state of Assam. Under Article 371B, the President is Tamil Nadu has only one Scheduled Area, namely
empowered to provide for the creation of a committee of the Kanyakumari district, but it is exempted from the application
Assam Legislative Assembly consisting of members elected of Part IX of the Constitution relating to the Panchayats by
from the Tribal Areas of the state and such other members Article 243M(4)(a), and hence also from the PESA Act
as he may specify. This article was added by the 22nd The states that have enacted PESA rules are Andhra
Constitutional Amendment Act of 1969. Pradesh, Telangana, Himachal Pradesh, Rajasthan, Gujarat,
Maharashtra, Madhya Pradesh and Chhattisgarh.
20. Solution: (c)
Exp) Option c is the correct answer. 23. Solution: (a)

Nagaland is the state in India where Panchayati Raj Exp) Option a is the correct answer.
Institution does not exist. The state has a unique system Statement 1 is correct: One of the functions of the State
of local self-government known as the Nagaland Village Election Commission is to prepare and revise the electoral
Councils Act, 1978. Under this act, the village councils are rolls for the elections to the rural and urban local bodies,
the primary units of local self-government in the state. as per Article 243K and Article 243ZA of the Constitution
As per Article 243M of the Constitution, the State of Statement 2 is correct: Another function of the State
Nagaland, Meghalaya, Mizoram, Sixth Schedule Areas and Election Commission is to conduct free and fair elections
hill areas of Manipur are exempted from application of Part for the rural and urban local bodies, as per Article 243K and
IX of the Constitution. Article 243ZA of the Constitution.

PYQ Workbook 264


INDIAN POLITY AND GOVERNANCE

Statement 3 is incorrect: State Election Commission is 26. Solution: (b)


mandated to Conducts elections only for Panchayats and Exp) Option b is the correct answer.
Municipalities of the State. The committee which recommended a three-tier Panchayati
Statement 4 is incorrect: This function (preparation of the Raj System in India was the Balwant Rai Mehta Committee.
electoral rolls and conducts elections) is not assigned to
Important Tips
the State Election Commission, but to the Chief Electoral
Officer of each state, who works under the supervision and The Balwant Rai Mehta Committee was appointed
in 1957 by the Government of India to examine and
control of the Election Commission of India. The Chief
suggest measures for better working of the Community
Electoral Officer is responsible for preparing and revising
Development Programme and the National Extension
the electoral rolls and conducting elections for Parliament
Service.
and state legislatures, as per Article 324 of the Constitution.
The committee submitted its report in 1958, and
24. Solution: (b) recommended the establishment of a democratic
Exp) Option b is the correct answer. decentralised local government, which came to be known
as the Panchayati Raj.
The Gram Panchayat bears the responsibility of planning,
The committee suggested a three-tier system of Panchayati
execution, and implementation of MNREGA (Mahatma
Raj, comprising the following:
Gandhi National Rural Employment Guarantee Act) schemes
at the local level. Gram Panchayat at the village level, directly elected by the
people.
Important Tips Panchayat Samiti at the block level, indirectly elected by
Features of MGNREGA: the members of the Gram Panchayats within the block.
• It is a scheme that provides legal guarantee of wage Zila Parishad at the district level, indirectly elected by the
employment to the adult members of rural households. members of the Panchayat Samitis within the district.
• Every rural household has the right to register 27. Solution: (d)
under MGNREGA, and job cards are issued to every
Exp) Option d is the correct answer.
household registered under MGNREGA within 15 days
from the date of receipt of the application for the job The state government controls over urban local bodies in
card registration. legislative, financial, and personnel matters. This means
that:
• A minimum of 10 job seekers shall apply to sanction a
• The state government can enact laws and regulations
new work under MGNREGA
regarding the functioning, powers, and finances of the
• The work shall be provided within 5 km jurisdiction urban local bodies, such as municipalities and municipal
from the village. If the work provided is beyond 5 Kms, corporations. The urban local bodies can only make rules
the job seekers shall be given 10% of the minimum or by-laws within the framework of the state laws.
wages as the additional amount
• The state government can determine the sources and
• Social Audit of MGNREGA works is mandatory, which methods of revenue generation for the urban local
lends to accountability and transparency bodies, such as taxes, fees, grants, loans, etc. The state
government can also regulate the expenditure and audit
25. Solution: (a)
of the urban local bodies, and impose financial discipline
Exp) Option a is the correct answer. and control on them.
Charles Metcalfe was an acting Governor-General of India • The state government can decide the recruitment,
from March 1835 to March 1836 and he wrote about the self- training, promotion, transfer, discipline, and service
sufficiency and vitality of the Indian village communities. conditions of the employees of the urban local bodies.
He said, “The village communities are little republics, having The state government can also prescribe the qualifications
nearly everything that they want within themselves, and and pay scales of the employees of the urban local bodies.
almost independent of any foreign relations.” • The state government does not control over urban
local bodies in respect of citizens’ grievances. This
Important Tips
means that:
Metcalfe (governor general—1835–36) repealed the
• The state government cannot interfere with the
obnoxious 1823 ordinance (using a press without license
complaints or grievances of the citizens against the
was a penal offence) and earned the epithet, “liberator of
urban local bodies or their functionaries. The urban
the Indian press”.
local bodies are responsible for addressing and redressing
The new Press Act (1835) required a printer/publisher to the grievances of the citizens within their jurisdiction.
give a precise account of premises of a publication and
• The state government can only appoint ombudsmen
cease functioning, if required by a similar declaration. or lokayuktas to investigate and redress the grievances

265 PYQ Workbook


INDIAN POLITY AND GOVERNANCE

of the citizens against the urban local bodies or their government bodies established in large urban areas to
functionaries. The ombudsmen or lokayuktas are administer and manage the complex affairs of the city.
independent and impartial authorities who act as Reason (R) is correct: Urban administrations of big cities
watchdogs over the urban local bodies. are very complex in nature. This statement is also true.
28. Solution: (a) Big cities have dense populations, multiple infrastructure
needs, various social and economic complexities, and
Exp) Option a is the correct answer.
require specialized governance structures to address these
Pair A is correctly matched: The Committee on challenges effectively.
Rationalization of Panchayat Statistics was appointed by the
Government of India in 1960 under the chairmanship of V.R. 31. Solution: (d)
Rao Exp) Option d is the correct answer.
Pair B is incorrectly matched: A.S. Trivedi was not According to the Panchayati Raj system in India, a Gram
associated with Panchayat Raj Training. S.P. Jain was Panchayat is the elected body that governs a village or a
involved in developing an action plan for training of group of villages, while a Gram Sabha is the assembly of all
Panchayati Raj functionaries adult members of a village or a group of villages
Pair C is correctly matched: The Working Group on Statement 1 is correct: In a Gram Panchayat there can be
Panchayats and Cooperatives was constituted by the more than one village. This is correct, as a Gram Panchayat
Government of India, Ministry of Community Development may consist of one or more villages, depending on the
and Cooperation in 1961 under the chairmanship of S.D. population and area
Mishra Statement 2 is correct: Meeting of Gram Sabha held in
Pair D is correctly matched: The Committee to review each village. This is correct, as the meeting of Gram Sabha
the existing Administrative Arrangements for Rural is held at least twice a year in each village within the area of
Development and Poverty Alleviation Programmes under the the Gram Panchayat
chairmanship of G.V.K. Rao was appointed by the Planning Statement 3 is incorrect: Meeting of Gram Sabha held in
Commission in 1985 the headquarter of Gram Panchayat. This is incorrect, as the
meeting of Gram Sabha is not held in the headquarter of
29. Solution: (d)
Gram Panchayat, but in each village under its jurisdiction.
Exp) Option d is the correct answer.
Statement 4 is incorrect: Meeting of Gram Panchayat held
Option I is incorrect: 74th Amendment Act does not in each village. This is incorrect, as the meeting of Gram
mention municipal committees as a type of urban local Panchayat is not held in each village, but in the headquarter
government. It only mentions three types of municipalities: of the Gram Panchayat or any other place fixed by it.
nagar panchayats, municipal councils and municipal
Statement 5 is correct: Meeting of Gram Panchayat held
corporations.
in the headquarter of Gram Panchayat. This is correct, as
Option II is correct: 74th Amendment Act provides for the the meeting of Gram Panchayat is usually held in the
constitution of ward committees at ward level or other headquarter of the Gram Panchayat or any other place fixed
level within the territorial area of a municipality as may be by it
provided in the state law.
Option III is correct: 74th Amendment Act provides for 32. Solution: (c)
the constitution of a metropolitan planning committee Exp) Option c is the correct answer.
for every metropolitan area, which is an area having Statement 1 is incorrect: Polling shall be postponed. This
a population of ten lakhs or more, to prepare a draft statement is false because polling shall not be postponed in
development plan for the area. all cases of death of a candidate before polling. It depends
Option IV is correct: 74th Amendment Act provides for on whether there are other contesting candidates in that seat
the constitution of a district planning committee for every or not.
district, to consolidate the plans prepared by the panchayats Statement 2 is incorrect: Polling shall not be postponed.
and the municipalities in the district and to prepare a draft This statement is false because polling shall be postponed in
development plan for the district as a whole some cases of death of a candidate before polling. It depends
on whether there are other contesting candidates in that seat
30. Solution: (a)
or not.
Exp) Option a is the correct answer.
Statement 3 is correct: If only one candidate remains, then
Both Assertion (A) and Reason (R) are true, and Reason polling will be postponed. This statement is true because if
(R) is correct explanation of Assertion (A). a candidate dies before the polling date and there is no other
Assertion (A)is correct: Municipal corporations are formed contesting candidate in that seat, then the election for that
in big cities having a definite area and population. This seat will be countermanded and fresh election will be held
statement is true. Municipal corporations are local self- later.

PYQ Workbook 266


INDIAN POLITY AND GOVERNANCE

Statement 4 is correct: If more than one candidate remains, accommodate changes in the demographic, administrative,
then polling will not be postponed. This statement is true or developmental requirements of the region. These changes
because if a candidate dies before the polling date and there could include the creation of new Blocks or Zila Panchayats,
are other contesting candidates in that seat, then the election the merger of existing ones, or alterations in their territorial
for that seat will not be countermanded and polling will be jurisdiction.
held as per schedule.
36. Solution: (d)
33. Solution: (b)
Exp) Option d is the correct answer.
Exp) Option b is the correct answer. In such a case, the state government may have the primary
Statement 1 is incorrect and Statement 2 is correct: responsibility to resolve the dispute, as it has administrative
The term begins from the date of the first meeting of the control over the panchayats and their areas. However, the
Municipal Corporation. state government may also need the approval of the central
Statement 3 is correct: The tenure of the Municipal Council government, as it has legislative and executive control over
is 5 years the cantonments and their boards.
Statement 4 is correct: A municipality constituted upon Important Tips
the dissolution of a municipality before the expiration of its A panchayat is a local self-government institution under
duration shall continue only for the remainder of the period the 73rd Constitutional Amendment Act, 1992. It has
for which the dissolved municipality would have continued powers and functions as per the state laws and the Eleventh
had it not been so dissolved. If the municipal council is Schedule of the Constitution.
dissolved within the first two years of its tenure, a new A cantonment board is a statutory body under the
municipal council will be organized for the remaining three Cantonments Act, 2006. It has powers and functions as per
years of its tenure. the central laws and the rules and regulations made by the
Statement 5 is incorrect: A municipality reconstituted after central government.
premature dissolution does not enjoy the full period of
37. Solution: (e)
five years but remains in office only for the remainder of
the period. Exp) Option e is the correct answer.
The Zila Panchayats of Scheduled Areas have the powers
34. Solution: (c) to:
Exp) Option c is the correct answer. (a) Plan minor water bodies: They can plan and undertake
Statement 1 is correct: The wards committee is formed projects related to minor water bodies within their
with some wards of nagar Panchayat: This statement is jurisdiction.
correct because a nagar panchayat is an urban local body that (b) Control over institutions in all social sectors: Zila
administers a town with a population of 100,000 or less, and Panchayats have the authority to oversee and manage
it is divided into wards according to population. institutions in various social sectors, including education,
Statement 2 is correct: Councillor elected from the wards health, and other welfare activities.
become its member: This statement is correct because the (c) Control on tribal sub plans: They have the responsibility
members of the wards committee are elected by direct to implement and monitor tribal sub-plans, which are
election from electoral wards in the nagar panchayat for a designed to ensure the socio-economic development of
term of five years. tribal communities.
Statement 3 is correct: Two persons are nominated in the (d) Exercise other functions as conferred by the State
committee residing in the ward: This statement is correct Government: In addition to the above-mentioned
because in addition to the elected members, there are three powers, Zila Panchayats may be entrusted with other
nominated members in the wards committee, who are functions by the State Government as per the specific
residents of the ward. requirements and policies of the state.
Statement 4 is incorrect: The Chairperson of the Wards
38. Solution: (d)
Committee is elected by the elected members thereof from
among themselves. Exp) Option d is the correct answer.
According to the Panchayats (Extension to Scheduled
35. Solution: (a)
Areas) Act, 1996 or PESA, which is a law enacted by the
Exp) Option a is the correct answer. Government of India for ensuring self-governance through
The alteration in the limits of Block and Zila Panchayats traditional Gram Sabha’s for people living in the Scheduled
is notified by the Governor of the respective state. When Areas of India, the meeting of Gram Sabha in Scheduled
there is a need to alter the boundaries or geographical Areas is presided over by a Schedule Tribe member present
limits of Block and Zila Panchayats, it is usually done to who is elected by Gram Sabha.

267 PYQ Workbook


INDIAN POLITY AND GOVERNANCE

39. Solution: (b) 42. Solution: (b)


Exp) Option b is the correct answer Exp) Option b is the correct answer.
Statement 1 is correct: In the Panchayati Raj System, all As per the 73rd Amendment to the Constitution of India,
seats in a Panchayat are filled through direct elections. The individuals must be at least 21 years of age to be eligible for
members of the Panchayat, including the Sarpanch (village Panchayat membership, contrary to the mentioned 25 years.
head) or Chairperson, are elected by the people through the This amendment lowered the age requirement to encourage
process of direct voting. youth participation in local governance.

Statement 2 is correct: The ‘Gram Sabha’ is a village-level Important Tips


assembly that consists of all the adults who are registered in Panchayati Raj was given constitutional status through
the electoral rolls of the village. The members of the Gram the 73rd Constitutional Amendment Act, 1992 (Part
Sabha have the right to participate in the decision-making IX, Schedule 11, Article 243-243 O), and it was first
process of the Panchayat and are involved in various local established in Rajasthan on October 2, 1959, followed
governance activities. by Andhra Pradesh, as recommended by the Balwant Rai
Mehta Committee.
Statement 3 is correct: The election of the Chairperson of
The 73rd Constitutional Amendment Act aims to provide
a Panchayat is governed by the respective state’s legislation.
a three-tier system of Panchayati Raj, which consists of
Each state in India has its own law that defines the process
Village-level Panchayats, Block-level Panchayats, and
and criteria for electing the Chairperson of a Panchayat. District-level Panchayats . Panchayati Raj System (PRS)
Statement 4 is incorrect: While most states in India have a was initially implemented based on the recommendations
three-tier system of Panchayats, there are a few exceptions. of the Balwant Rai Mehta Committee. Later, it was granted
Some states have a two-tier system, consisting of the Gram constitutional status through on recommendation of the
Panchayat at the village level and the Zilla Parishad at the LM Singhvi committee.
district level. States like Kerala and Goa follow a two-tier Different committees associated with Panchayati Raj (in
system, whereas states like Rajasthan, Maharashtra, and Chronological Order):
Andhra Pradesh have a three-tier system. • Balwant Rai Mehta Committee (January 1957)
• Ashok Mehta Committee (December, 1957)
40. Solution: (d)
• G.V.K. Rao Committee (1985)
Exp) Option d is the correct answer
• L.M. Singhvi Committee (1986)
Statement 1 is correct: The elections to the Panchayats
• Thungon Committee (1988)
in India are conducted by the respective State Election
Commissions (SECs). The State Election Commission is • Gadgil Committee (1988)
responsible for overseeing and conducting the electoral The Panchayati Raj was implemented under the Prime
process for Panchayat elections within the state under Ministership of P.V. Narasimha Rao.
Article 243K of the constitution. 43. Solution: (b)
Statement 2 is incorrect: The State Election Commissioner Exp) Option b is the correct answer.
is appointed by the Governor of the state.
Each Panchayat operates under its own set of regulations,
Statement 3 is correct: As per the provisions of the with the state legislature conferring powers to facilitate
Constitution of India, the State Legislatures have the power their functioning effectively. These powers encompass
to make laws on all matters relating to Panchayat elections. formulating economic development and social justice plans,
This includes the conduct of elections, the qualifications plan implementation, and handling matters specified in the
and disqualifications of candidates, the delimitation of eleventh schedule.
constituencies, and other related aspects.
44. Solution: (b)
41. Solution: (a) Exp) Option b is the correct answer.
Exp) Option a is the correct answer. The 11th Schedule of the Constitution of India distributes
While MGNREGA guarantees 100 days of wage employment, powers between the State Legislatures and the Panchayats.
These powers include areas such as rural development,
it primarily focuses on providing unskilled manual work
planning, agriculture, education, health, water supply,
rather than specifically emphasizing skilled labor.
sanitation, poverty alleviation, and social justice. The
The other objectives of MGNREGA include the creation State Legislatures are responsible for enacting laws and
of productive assets, enhancing livelihood security, and providing the necessary framework for the functioning of
ensuring empowerment to women. These objectives aim to the Panchayats, while the Panchayats have the authority to
enhance livelihood security of households in rural areas of implement and execute various developmental programs and
the country. initiatives at the grassroots level.

PYQ Workbook 268


INDIAN POLITY AND GOVERNANCE

45. Solution: (b) 47. Solution: (c)


Exp) Option b is the correct answer. Exp) Option c is the correct answer.
• The elected members of the District Council shall hold • Article 243 G mentions “subjected to the provision of
office for a term of five years from the date appointed this constitution the legislature of a state which says that
for the first meeting of the Council after the general Panchayats are under the umbrella of the Constitution.
elections to the Council. (option b is correct) • Within the constitutional framework, ‘Panchayats’ is a
• Sixth schedule to the constitution of India provides for State subject. Hence, the Panchayat does not work on
setting up of autonomous district council the principle of constitutional autonomy. (Statement 3
• The Constitution of India makes special provisions for is not correct)
the administration of the tribal dominated areas in four
48. Solution: (d)
states viz Assam, Meghalaya Tripura and Mizoram.
Exp) Option d is the correct answer.
• As per Article 244 and sixth Schedule these areas are
called “Tribal Areas The Constitution of India does not contain any provision
for the constitution of urban panchayats in every state. The
• The Sixth Schedule envisages establishment of
Autonomous District Councils (ADC) Constitution only makes provisions for the constitution
of municipalities, which are divided into three categories-
Important Tips Nagar Panchayats, Municipal Council, and Municipal
District council for each autonomous district: Corporations. The Constitution leaves it up to the states to
• It has 30 members. decide whether or not to constitute urban panchayats.
• 4 are nominated by the governor — They perform their 49. Solution: (c)
duties during the pleasure of the governor
Exp) Option c is the correct answer.
• 26 are elected using the adult franchise — Their term
Statement 1 is incorrect- According to the Article 243G
of office is five years.
of the Indian Constitution, the state legislature has the
• There is a separate regional council for each
authority to endow Panchayats with powers and authority
autonomous district
necessary for self-government. The state legislature may
• The laws related to the following can be made by the authorize a Panchayat to levy, collect, and appropriate taxes,
regional and autonomous councils with the assent duties, tolls, and fees.
of the governor: Land, Forest, Canal Water, Shifting
Statement 2 is correct- The Constitution empowers the
Cultivation, Village Administration, Inheritance of
Property, Marriage & Divorce, Social Customs. State Legislature to devolve powers to Panchayats for the
preparation of plans related to economic development and
• The central and state acts do not apply on these
social justice. This enables Panchayats to play a role in local-
autonomous and regional councils (unless modified
level planning and implementation of development schemes.
and accepted).
Statement 3 is correct- The State Legislature can also
46. Solution: (b) devolve powers to Panchayats for the implementation of
Exp: Option b is the correct answer. schemes related to economic development and social
• Part IX of the 73rd Constitutional amendment act 1992 justice. This enables Panchayats to actively participate in the
constitutionalised the Panchayati Raj Institutions (PRIs) execution of development programs at the local level.
• Article 243, 243A to 243 O under Part IX has been 50. Solution: (b)
included in the Indian constitution.
Exp) Option b is the correct answer
• The PRIs are the local self-governing bodies that
Articles 371 to 371-J in Part XXI of the Constitution of India
ensure the opportunity for people’s participation and
contain special provisions for twelve states: Maharashtra,
involvement in the formulation and implementation of
Andhra Pradesh, Telangana, Sikkim, Mizoram, Arunachal
rural development programmes.
Pradesh, Gujarat, Nagaland, Assam, Manipur, Goa and
• The main objective of Panchayat System in India is to Karnataka. Madhya Pradesh is not mentioned in Article
strengthen the base of democracy at the grass root level. 371 of the Constitution of India.
• Part IX A of the 74th Constitutional amendment act 1992
constitutionalised the The Municipality in the Indian 51. Solution: (c)
constitution. Exp) Option c is the correct answer
• Few states like Nagaland, Meghalaya, Mizoram and some In India, the power to levy taxes by Panchayats is primarily
other tribes don’t have a three tier system of government. within the domain of the State Governments. The State
The act doesn’t apply to these areas. (statement 1 is not Governments have the authority to authorize Panchayats to
correct) levy certain taxes, fees, or other local charges.

269 PYQ Workbook


INDIAN POLITY AND GOVERNANCE

52. Solution: (d) also applicable in all Union Territories except Delhi. These
Exp) Option d is the correct answer. three states have their own traditional systems of local self-
The Chairman of the Zila Parishad is chosen through an governance.
election process among the members of the Zila Parishad Option b is incorrect: The Panchayat system was introduced
itself. This ensures that a member of the Zila Parishad is in India by the Constitution 73rd Amendment Act, not the
selected to lead and represent the local government body. 74th Amendment Act. The 73rd Amendment Act deals with
Important Tips the Panchayats, while the 74th Amendment Act deals with
the Municipalities. Therefore, this statement is false.
• A Zila Parishad is the third tier of the Panchayati Raj
system and functions at the district level in all states. Option c is incorrect: The Panchayat system is provided for
• A Zila Parishad is an elected body, consisting of the all states in India. Panchayati Raj system is not provided for
representatives of the Panchayat Samitis (block-level states having less than 20 lakh population, not 2 million
panchayats) under the district, the members of the population.
state legislature and the parliament from the district, Option d is incorrect: Gram Panchayat can continue for
and some nominated members. five years from the date of election, not three years
• The chairman of the Zila Parishad is elected from
among its own members by a simple majority. 55. Solution: (a)
• The chairman, also called Adhyaksha or President, Exp) Option a is the correct answer.
heads the Zila Parishad and presides over its meetings. Statement 1 is correct: The Constitution 74th Amendment
The vice-chairman is also elected similarly. Act of 1992 does indeed envisage three types of urban local
53. Solution: (c) bodies, they are Municipal Corporation, Municipality, and
Nagar Panchayat (City Council).
Exp) Option c is the correct answer.
The Ashok Mehta Committee was a committee appointed Statement 2 is correct: Municipal Corporations are
by the Janata Government in 1977 to study the Panchayati established in cities with a population greater than
Raj institutions in India and suggest measures to revive and 1 million. The population criterion may vary slightly
strengthen them. The committee submitted its report in depending on the state-specific legislation, but generally,
1978 and made 132 recommendations on various aspects Municipal Corporations are set up in large cities.
of local governance such as structure, functions, finances, Statement 3 is incorrect: While Bombay and Calcutta (now
elections, etc. Some of the reasons cited by the Ashok Kolkata) are significant cities in India, they were not the first
Mehta Committee for the decline of the Panchayati Raj
Municipal Corporations established during British times.
institutions in India were:
The first Municipal Corporation in India was established
• Lack of political will among the state governments to in Madras (now Chennai) in 1688 during the British colonial
devolve power and resources to the local bodies
period.
• Lack of conceptual clarity among the policy makers
and administrators about the role and objectives of the 56. Solution: (a)
Panchayati Raj institutions Exp) Option a is the correct answer.
• Role of bureaucracy in dominating and interfering The Members of Panchayat Samiti are directly elected by the
with the functioning of the local bodies public. This is because Panchayat Samiti or Block Panchayat
• Lack of adequate financial resources and taxing powers is a rural local government body at the intermediate tehsil or
for the Panchayati Raj institutions block level in India, and it has some directly elected members
• Lack of effective participation of the people, especially by the voters of that area.
the weaker sections, in the decision-making process.
Important Tips
The Ashok Mehta Committee did not cite lack of gender
The other members of Panchayat Samiti are the Sarpanches
parity as a reason for the decline of the Panchayati Raj
or Chairpersons of the Gram Panchayats within the block,
institutions, though it recommended reservation of one-
the MPs, MLAs, and MLCs of that block, the elected
third seats for women in these bodies
members of Zila Parishad from that block, and some
54. Solution: (a) officers of that block.
Exp) Option a is the correct answer. The Panchayat Samiti is headed by a chairperson or
Option a is correct: The Panchayat system exists in all states president and a deputy chairperson or vice president
of India, except Nagaland, Meghalaya, and Mizoram. It is elected by the members of the Panchayat Samiti.

PYQ Workbook 270


INDIAN POLITY AND GOVERNANCE

INDIAN POLITY AND GOVERNANCE


CONSTITUTIONAL AND NON-CONSTITUTIONAL BODIES
*This unit consists of questions from Constitutional, Statutory and Other Executive Bodies

8.1. UPSC CSE Previous Years’ Questions 4. The Board has overseas office at Dubai
and Moscow.
1. Consider the following organizations/ Which of the statements given above are
bodies in India: correct? [UPSC CSE Pre. 2022]
1. The National Commission for Backward (a) 1 and 3
Classes (b) 2 and 4
2. The National Human Rights Commission (c) 3 and 4
3. The National Law Commission (d) 1 and 4
4. The National Consumer Disputes 4. With reference to the Constitution of India,
Redressal Commission prohibitions or limitations or provisions
How many of the above are constitutional contained in ordinary laws cannot act
bodies? [UPSC CSE Pre 2023] as prohibitions or limitations on the
(a) Only one constitutional powers under Article 142. It
(b) Only two could mean which one of the following?
(c) Only three [UPSC CSE Pre. 2019]
(d) All four (a) The decisions taken by the Election
2. In India, what is the role of the Coal Commission of India while discharging
Controller’s Organization (CCO)? its duties cannot be challenged in any
1. CCO is the major source of coal Statistics court of law.
in Government of India. (b) The Supreme Court of India is not
constrained in the exercise of its powers
2. It monitors progress of development of
by laws made by the Parliament.
Captive Coal/ Lignite blocks.
(c) In the event of grave financial crisis in
3. It hears any objection to the Government’s the country, the President of India can
notification relating to acquisition of declare Financial Emergency without the
coal-bearing areas. counsel from the Cabinet.
4. It ensures that coal mining companies (d) State Legislatures cannot make laws on
deliver the coal to end users in the certain matters without the concurrence
prescribed time. of Union Legislature.
Select the correct answer using the code given 5. With reference to the ‘Quality Council
below: [UPSC CSE Pre. 2022] of India (QCI)’, consider the following
(a) 1, 2 and 3 statements:
(b) 3 and 4 only 1. QCI was set up jointly by the Government
(c) 1 and 2 only of India and the Indian Industry.
(d) 1, 2 and 4 2. Chairman of QCI is appointed by the
3. With reference to the “Tea Board” in India, Prime Minister on the recommendations
consider the following statements: of the industry to the Government.
1. The Tea Board is a statutory body. Which of the above statements is/are correct?
2. It is a regulatory body attached to the [UPSC CSE Pre. 2017]
Ministry of Agriculture and Farmers (a) 1 only
Welfare. (b) 2 only
3. The Tea Board’s Head Office is situated in (c) Both 1 and 2
Bengaluru. (d) Neither 1 nor 2

271 PYQ Workbook


INDIAN POLITY AND GOVERNANCE

6. Consider the following statements: 4. The Report of the National Commission


Attorney General of India can for Scheduled Castes
1. take part in the proceedings of the Lok Select the correct answer using the codes
Sabha given below: [UPSC CSE Pre. 2012]
2. be a member of a committee of the Lok (a) 1 only
Sabha (b) 2 and 4 only
3. speak in the Lok Sabha (c) 1, 3 and 4 only
4. vote in the Lok Sabha (d) 1, 2, 3 and 4
Which of the statements given above is/are 9. With reference to the Finance Commission
correct? [UPSC CSE Pre. 2013] of India, which of the following statements
(a) 1 only is correct? [UPSC CSE Pre. 2011]
(b) 2 and 4 only (a) It encourages the inflow of foreign capital
(c) 1, 2 and 3 for infrastructure development
(d) 1 and 3 only (b) It facilitates the proper distribution
of finances among the Public Sector
7. In India, other than ensuring that public Undertakings
funds are used efficiently and for the (c) It ensures transparency in financial
intended purpose, what is the importance administration
of the office of the Comptroller and Auditor (d) None of the statements (a), (b) and (c)
General (CAG)? given above is correct in this context
1. CAG exercises exchequer control on
10. Consider the following statements:
behalf of the Parliament when the
President of India declares national The function(s) of the Finance Commission
emergency/financial emergency. is/are-
2. CAG reports on the execution of projects 1. To allow the withdrawal of the money out
or programmes by the ministries which of the Consolidated Fund of India.
are discussed by the Public Accounts 2. To allocate among the States the shares of
Committee. proceeds of taxes.
3. Information from CAG reports can be 3. To consider applications for grants-in-aid
used by investigating agencies to press from States.
charges against those who have violated 4. To supervise and report on whether the
the law while managing public finances. Union and State Governments are levying
4. While dealing with the audit and taxes in accordance with the budgetary
accounting of government companies, provisions.
CAG has certain judicial powers for Which of these statements is/are correct?
prosecuting those who violate the law. [UPSC CSE (Pre) 2003]
Which of the statements given above is/are (a) Only 1
correct? [UPSC CSE Pre. 2012] (b) 2 and 3
(a) 1, 2 and 4 only (c) 3 and 4
(b) 2 only (d) 1, 2 and 4
(c) 2 and 3 only 11. Who of the following shall causes every
(d) 1, 2, 3 and 4 recommendation made by the Finance
8. According to the Constitution of India, it is Commission to be laid before each House
the duty of the President of India to cause of the Parliament? [UPSC CSE (Pre) 2010]
to be laid before the Parliament which of (a) The President of India
the following? (b) The Speaker of Lok Sabha
1. The Recommendations of the Union (c) The Prime Minister of India
Finance Commission (d) The Union Finance Minister
2. The Report of the Public Accounts 12. The decision to disqualify the person
Committee declared convict by the Court to contest the
3. The Report of the Comptroller and elections has been made by-
Auditor General [UPSC CSE (Pre) 2009]

PYQ Workbook 272


INDIAN POLITY AND GOVERNANCE

(a) The Government of India 17. If in an election to a State Legislative


(b) The Supreme Court Assembly, the candidate who is declared
(c) The Election Commission elected loses his deposit, it means that-
(d) The Parliament [UPSC CSE (Pre) 1995]
13. Which of the following recommends the (a) The polling was very poor.
(b) The election was for a multi-member
principles for sharing of revenues among
constituency.
the Union and the States? [UPSC CSE (Pre) (c) The elected candidate’s victory over his
2002, Chhattisgarh P.C.S. (Pre) 2003] nearest rival was very marginal.
(a) Finance Commission (d) A very large number of candidates
(b) Inter-State Council contested.
(c) Union Ministry of Finance
18. Which one of the following statements
(d) Planning Commission
regarding Exit Poll is correct?
14. The Dinesh Goswami Committee [UPSC CSE (Pre) 1994]
recommended: [UPSC CSE (Pre) 1997] (a) Exit Poll is a term used to denote a post-
(a) The constitution of state-level election election survey of voters regarding the
commission candidates in whose favour they had
(b) List system of election to the Lok Sabha exercised their franchise
(c) Government funding of parliamentary (b) Exit Poll and Opinion Polls are one and
elections the same
(d) A ban on the candidature of independent (c) Exit Poll is a device through which results
candidates for the parliamentary elections of voting can be most exactly predicted
(d) Exit Poll is an administrative device
15. Proportional representation is not made recently by the Chief Election
necessary for a country where: Commissioner to prevent impersonation
[UPSC CSE (Pre) 1997] 19. Which of the election system have been
(a) There are no reserved constituencies adopted for different elections in India?
(b) a two-party system has developed 1. Direct Election System on the basis of
(c) the first-past-post system prevails Adult Suffrage.
(d) there is a fusion of Presidential and 2. System of Proportional representation by
Parliamentary forms of government a Single Transferable Vote.
16. Which one of the following is correct 3. List System of Proportional
in respect of the commencement of the Representation.
election process in India? 4. The cumulative Voting system of Indirect
[UPSC CSE (Pre) 1995] Election.
(a) The recommendation for election is made Choose the correct answer from the given
by the Government, and the notification code. [UPSC CSE (Pre) 1994]
for election is issued by the Election (a) 1 and 2
Commission. (b) 1 and 3
(b) The recommendation for election is made (c) 1, 2 and 3
by the Election Commission, and the (d) 2, 3 and 4
notification for election is issued by the 20. Which of the following are the functions of
Home Ministry at the Centre and Home the Election Commission of India?
Departments in the States. 1. To conduct elections for the post of
(c) The recommendation for election is Speaker and Deputy-speaker of Lok
made by the Election Commission, and Sabha and the Deputy Chairman of Rajya
the notification for election is issued by Sabha.
the President and Governors of the States 2. To conduct elections for the municipality
concerned. and municipal corporations.
(d) Both the exercises of making a 3. To make a decision on all the doubts and
recommendation for election and that of disputes arising from the election.
issuing a notification in respect of it are Select the correct answer from the code given
done by the Election Commission. below: [UPSC CSE (Pre) 1994]

273 PYQ Workbook


INDIAN POLITY AND GOVERNANCE

(a) 1 and 2 Select the correct answer using the codes


(b) 1 and 3 given below.
(c) 2 and 3 (a) Both (A) and (R) are true and (R) is
(d) None of the above correct explanation of (A)
21. Who recommended to end Indian (b) Both (A) and (R) are true, but (R) is not
Administrative Services and Indian Police correct explanation of (A)
Services? [UPSC CSE (Pre) 1993] (c) (A) is true, but (R) is false
(a) Dhebar Commission (d) (A) is false, but (R) is true
(b) Kalelkar Commission
26. Consider the facts given below:
(c) Kher Commission
(d) Rajamannar Commission 1. Sukumar Sen was the first Chief Election
Commissioner of India.
22. The question considered by Swarn Singh 2. Rama Devi was the first woman Chief
Committee was related to -
Election Commissioner of India.
[UPSC CSE (Pre) 1993]
Choose the correct answer from the codes
(a) More autonomy for Punjab than Jammu
and Kashmir. given below. [U.P.P.C.S. (Pre) 2021]
(b) Suitability of President oriented (a) Only 1 is correct
governance for India. (b) Only 2 is correct
(c) Priority to Directive Principles of State (c) Both 1 and 2 are correct
Policy in comparison to fundamental (d) Both 1 and 2 are incorrect
rights.
27. In the removal of which of the following
(d) Administrative reform.
officials does the Parliament NOT play any
8.2. Other Examination Previous Years’ role?
Questions 1. Judges of High Court
2. Judges of the Supreme Court
23. A Joint Public Service Commission for two
or more States can be established by- 3. Chairman of the Union Public Service
[U.P.P.C.S. (Pre) 2022] Commission
(a) Parliament, if requested by concerned 4. Comptroller and Auditor General of India
States Choose the correct answer from the codes
(b) Lok Sabha given below: [U.P.P.C.S. (Pre) 2020]
(c) Union Public Service Commission (a) 1 and 2 only
(d) Rajya Sabha (b) 3 and 4 only
24. A member of State Public Service (c) 1, 2 and 3 only
Commission may be removed by - (d) 3 only
[U.P.P.C.S. (Pre) 2021] 28. In which year was accounting separated
(a) The Governor on the basis of impeachment from auditing and the Comptroller and
in Vidhan Sabha Auditor General’s task only remained
(b) The Governor, after the enquiry confined to auditing the Government
conducted by the Supreme Court
Accounts? [U.P.P.C.S. (Pre) 2019]
(c) The President, after the enquiry
conducted by the Supreme Court (a) 1975
(d) The Governor, after the enquiry (b) 1977
conducted by the High Court (c) 1976
(d) 1981
25. Given below are two statements one is
labelled as Assertion (A) and other as 29. Which of the following is an extra
Reason (R): [U.P.P.C.S. (Pre) 2021] constitutional agency?
Assertion (A): The President of India [U.P.P.C.S.(Pre) 2018]
determines the qualifications of the Chairman (a) Union Public Service Commission
and Members of the Finance Commission. (b) Finance Commission
Reason (R): Chairman and members are (c) Election Commission
appointed by the President of India. (d) NITI Aayog

PYQ Workbook 274


INDIAN POLITY AND GOVERNANCE

30. Given below are two statements: 3. Religion, Race, Sex, etc., will have no
Assertion (A): Model code of conduct is to place in the formation of the electoral
be followed by political parties as soon as an roll.
election is announced. 4. The political parties will be free to have
their norms.
Reason (R): Model code of conduct was
enacted by Parliament. Select the correct answer from the given code:
[U.P.P.C.S. (Mains) 2011]
Choose the correct answer from the code
given below: [U.P.P.C.S. (Pre) 2017] (a) 1, 3 and 4
(b) 2, 3 and 4
(a) Both (A) and (R) are true and (R) is the
(c) 1, 2 and 3
correct explanation of (A). (d) All the four
(b) Both (A) and (R) are true and (R) is not
the correct explanation of (A). 35. Which Constitutional Amendment reduced
(c) (A) is true but (R) is false. the voting age from 21 years to 18 years?
(d) (A) is false but (R) is true. [U.P.P.C.S. (Mains) 2010]
(a) Sixty-third Amendment
31. Consider the following statements and
(b) Sixty-second Amendment
select the correct answer from the code (c) Sixty-first Amendment
given below: (d) Sixtieth Amendment
[U.P.P.C.S. (Pre) (Re. Exam) 2015]
36. Which one of the following functions is not
Assertion (A): State Finance Commission is related to the Election Commission?
a Constitutional body.
[U.P.P.C.S. (Mains) 2009]
Reason (R): Union Finance Commission (a) Direction and control of the preparation
cannot recommend financial assistance to of the electoral rolls.
Panchayats. (b) Conduct of all elections to the Parliament
Code: and Legislatures of every State.
(a) Both (A) and (R) are true and (R) is the (c) To conducts the election of the Offices of
correct explanation of (A) President and Vice-president.
(b) Both (A) and (R) are true, but (R) is not (d) To make provision with respect to
the correct explanation of (A) elections to Legislatures.
(c) (A) is true, but (R) is false 37. Who decides disputes regarding
(d) (A) is false, but (R) is true disqualification of Members of Parliament?
32. In which of the following States, the office [U.P.P.C.S. (Mains) 2009]
of Lokayukta was, first established? (a) The President
[U.P.P.C.S. (Pre) (Re-Exam) 2015] (b) The Concerned House
(c) The Election Commission
(a) Maharashtra
(d) The President in consultation with the
(b) Uttar Pradesh Election Commission
(c) Bihar
(d) Odisha 38. Which Act established Public Service
Commission in India for the first time?
33. The concept of Human Rights primarily [U.P.P.C.S. (Pre) 2008]
emphasizes on [U.P.P.C.S. (Mains) 2014]
(a) Indian Council Act, 1892
(a) Right to Property (b) Council Act, 1909
(b) Right to equality (c) Government of India Act, 1919
(c) Right to Religion (d) Government of India Act, 1935
(d) Dignity of Man as a Human Being
39. Who is called the Guardian of Public Purse?
34. The Principles for election to the State [U.P.P.C.S. (Mains) 2008]
Legislatures include: (a) President
1. It will be on the basis of adult suffrage. (b) Comptroller and Auditor General
2. There will be only one electoral roll for (c) Parliament
every territorial constituency. (d) Council of Ministers

275 PYQ Workbook


INDIAN POLITY AND GOVERNANCE

40. Given below are two statements: 44. The Chief Election Commissioner can be
Assertion (A): Powers for conducting removed from office by-
elections to the Parliament and State [U.P.P.C.S. (Mains) 2002]
Legislatures in a free and fair manner have (a) Both Houses of Parliament by two-thirds
been given to an independent body i.e., the majority in each House.
Election Commission. (b) The same procedure applies in the case of
Reason (R): Power of removal of Election removal of Supreme Court Judges.
Commissioners is with the executive. (c) The President of India in consultation
with the Chief Justice of India.
In the context of above, which one of the
(d) The President on the advice of the Council
following is correct? [U.P.P.C.S. (Pre) 2006]
of Ministers.
(a) Both (A) and (R) are true, and (R) is the
correct explanation of (A). 45. Given below are two statements, in which
(b) Both (A) and (R) are true, but (R) is not a one is Assertion and second is Reason:
correct explanation of (A). Assertion (A): The definition of term
(c) (A) is true but, (R) is false. ‘Minority’ is not given in the Constitution of
(d) (A) is false but, (R) is true. India.
41. Which one of the following duties is not Reason (R): Minority Commission is not a
performed by Comptroller and Auditor Constitutional body.
General of India? [U.P.P.C.S. (Mains) 2004, Which of the following is correct in regards to
UPSC CSE (Pre) 2001] the statements above? [U.P.P.C.S. (Pre) 1991]
(a) To audit and report on all expenditure (a) Both (A) and (R) are true, and (R)
from the Consolidated Fund of India explains (A) correctly.
(b) To audit and report on all expenditure (b) Both (A) and (R) are true, but (R) does
from the Contingency Funds and Public not explain (A) correctly.
Accounts (c) (A) is true, but (R) is false.
(c) To audit and report on all trading, (d) (A) is false, but (R) is true.
manufacturing, profit and loss accounts
(d) To control the receipt and issue of public 46. Which one of the following is not correct
money, and to ensure that the public about NITI Aayog?
revenue is lodged in the Exchequer. [U.P. Lower Sub. (Pre) 2015]
(a) It was set up to replace the Planning
42. Accounts of the States are controlled by the: Commission
[U.P.P.C.S. (Mains) 2003] (b) It has a full-time Chairman
(a) Governor (c) It was set up in January 2015
(b) Chief Minister (d) It is based on the principle of Cooperative
(c) State Finance Secretary Federalism
(d) None of these
47. Given below are two statements, one
43. Consider the following statements regarding labelled as Assertion (A) and the other as
appointment of National Commission for Reason (R):
reviewing the Constitution- Assertion (A): The system of proportional
1. The report will be of recommendatory representation may solve the problem of
nature. minority representation to some extent.
2. The appointment was approved by Reason (R): The system of proportional
Parliament. representation enables due representation to
3. It is presided over by Chief Justice M.N. all types of groups based on ethnicity, gender,
Venkatchelaiya. interests, and ideologies.
4. It will focus on socio-economic needs of Choose the correct answer from the code
the country. given below: [U.P. Lower Sub. (Mains) 2013]
Which of these statements are correct? (a) Both (A) and (R) are true, and (R) is the
[U.P.P.C.S. (Pre) 2003] correct explanation of (A).
(a) 1 and 2 (b) Both (A) and (R) are true, but (R) is not a
(b) 1 and 3 correct explanation of (A).
(c) 1, 2 and 3 (c) (A) is true, but (R) is false.
(d) 2, 3 and 4 (d) (A) is false, but (R) is true.

PYQ Workbook 276


INDIAN POLITY AND GOVERNANCE

48. The system of proportional representation (a) Gorwala Report


as an electoral mechanism ensures- (b) Kriplani Committee
[U.P. Lower Sub. (Pre) 2013, U.P.P.C.S. (Pre) (c) Santhanam Committee
2013] (d) Administrative Reforms Commission of
(a) Majority rule India
(b) Stability in Government 54. Choose the correct answer: The appointment
(c) Common political thinking of Lok Ayukta at the state level was first
(d) Representation of minorities recommended by-
49. The Chief Election Commissioner of India [R.A.S./R.T.S. (Pre) 2016]
is appointed by- (a) Administrative Reforms Commission of
[Uttarakhand P.C.S. (Pre) 2010] India (1966-70)
(b) Santhanam Committee
(a) Lok Sabha
(c) Rajasthan State Administrative Reforms
(b) Prime Minister
Committee
(c) President (d) Second Administrative Reforms
(d) Chief Justice Commission
50. When was the Universal Declaration of 55. The Chairman of a Joint Public Service
Human Rights adopted by the United Commission fomore States is appointed by-
Nations? [Jharkhand P.C.S. (Pre) 2021]
[R.A.S./R.T.S. (Pre) 2016]
(a) 10 January, 1920 (a) The President of India
(b) 4 April, 1949 (b) The President of India on the
(c) 24 October, 1945 recommendation of the Governors
(d) 10 December, 1948 concerned
51. Union Public Service Commission may (c) The Governor of the largest State
agree to serve the needs of the state- (d) A Committee of Governors of all the
[Jharkhand P.C.S. (Pre) 2021] States concerned
(a) On request of the Chief Minister with 56. Which one of the following is not the
approval of the Prime Minister. function of the State Human Rights
(b) On request of the election Commission Commission? [R.A.S./R.T.S.(Pre) 2013]
with approval of the President. (a) Enquire suo-moto the violation of Human
(c) On request of the Governor with approval Rights
of the Prime Minister. (b) Visit any Jail
(d) On request of the Governor with approval (c) Review the protection of Human Rights
of the President. (d) To punish for the violation of Human
Rights
52. Consider the following statements and
choose the correct answer from the code 57. In which of the following matters is the State
given below: [R.A.S./R.T.S. (Pre) 2021] Public Service Commission not consulted?
Assertion (A): The Duty of Comptroller and [R.A.S./R.T.S.(Pre) 2013]
Auditor General is not merely to ensure the (a) On the method of recruitment of Civil
legality of expenditure but also its propriety. Services.
Reason (R): He has to uphold the Constitution (b) On the principles to be followed in
and the Laws of Parliament in the field of making appointments to Civil Services.
(c) On the principles to be followed in
financial administration.
making promotions in Civil Services and
Code: transfers from one service to another.
(a) (A) is false but (R) is true (d) On making transfers of Civil Servants.
(b) Both (A) and (R) are true and (R) is the
correct explanation of (A) 58. The Comptroller and Auditor-General of
India can be removed from his office in like
(c) (A) is true but (R) is false
manner as- [R.A.S/R.T.S. (Pre) 2018]
(d) Both (A) and (R) are true but (R) is not a
correct explanation of (A) (a) Speaker of Lok Sabha
(b) Attorney General of India
53. Central Vigilance Commission was set up (c) Judge of the Supreme Court
on the recommendation of - (d) Chairman of Union Public Service
[R.A.S./R.T.S. (Pre) 2018] Commission

277 PYQ Workbook


INDIAN POLITY AND GOVERNANCE

59. What is/are true in reference to the Finance 63. Vohra Committee was formed to study
Commission? [Chhattisgarh P.C.S. (Pre) 2003]
[Chhattisgarh P.C.S. (Pre) 2021] (a) Police Reform
(i) There are total 5 members in Commission (b) Financial Reform
(c) Nexus of Politicians and Criminals
(ii) At least one member of Commission must (d) Appointment of governors
be Judge of High Court or Supreme Court
64. Comptroller and Auditor General of India
(iii) The Chairman of Commission cannot be submits its reports relating to accounts of
reappointed the Union to- [M.P.P.C.S. (Pre), 2021]
(iv) First Chairman of Commission was K. (a) Chief Justice of India
Santhanam (b) Prime Minister
Code: (c) Vice President
(d) None of the above
(a) (i), (ii) and (iv)
(b) (i), (ii) and (iii) 65. Chairperson and Member of the State
(c) (i) and (iii) Public Service Commission are appointed-
(d) Only (i) [M.P.P.C.S. (Pre), 2021]
(a) by Governor
60. Read the following statements in relation (b) by Chief Minister
to reservation of seats in Lok Sabha and (c) by President
Legislative Assembly, and choose the (d) by Chairman Union Public Service
correct option: Commission
Statement I: Out of the 543 elected seats in 66. The power to grant or refuse leave to the
the Lok Sabha, 84 are reserved for Scheduled Comptroller and Auditor General shall vest
Castes and 44 are reserved for Scheduled in the- [M.P.P.C.S. (Pre), 2021]
Tribes. (a) Finance Minister
Statement II: Decision, on which constituency (b) President
is to be reserved, is taken by the Delimitation (c) Prime Minister
Commission. (d) Chief Justice of India
Statement III: The Delimitation Commission 67. Consider the following statements:
is appointed by the Lok Sabha Speaker. (i) It shall be the duty of the Comptroller
Code: [Chhattisgarh P.C.S. (Pre) 2020] and Auditor-General to audit all receipts
(a) Statement I, II and III all are true. which are payable into the Consolidated
Fund of India.
(b) Statement I is true, but Statement II and
III are false. (ii) The Comptroller and Auditor General
shall have authority to audit and report
(c) Statement I and III are false, but Statement
on the accounts of stores or stock kept in
II is true. any office or department of the Union or
(d) Statement I, II and III are all false. a State.
61. In which article of the constitution is Adult Select the correct answer using the codes
Suffrage recognized? given below. [M.P.P.C.S. (Pre) 2020]
[Chhattisgarh P.C.S. (Pre) 2018] (a) Both (i) and (ii) are incorrect
(a) Article 324 (b) Both (i) and (ii) are correct
(b) Article 325 (c) (i) is incorrect but (ii) is correct
(d) (i) is correct but (ii) is incorrect
(c) Article 326
(d) Article 327 68. Who is not included in the Committee to
recommend name for the appointment of
62. Which of the following Article of Indian Chairperson and members of State Human
Constitution mentions the ‘Doctrine of Rights Commission?
Pleasure’? [Chhattisgarh P.C.S. (Pre) 2016] [M.P.P.C.S. (Pre) 2020]
(a) Article 200 (a) Chief Minister
(b) Article 301 (b) Speaker of the Legislative Assembly
(c) Article 310 (c) Incharge of the Department of Home
(d) Article 311 Affairs
(e) None of the above (d) Governor

PYQ Workbook 278


INDIAN POLITY AND GOVERNANCE

69. Which Article of the Constitution prescribes (a) Governor


the duties of the comptroller and Auditor (b) State Government
General of India? [M.P.P.C.S. (Pre) 2019] (c) Chief Justice of the High Court
(a) Article 146 (d) Chief Justice of India
(b) Article 147
75. Every member of the Commission, Sate
(c) Article 148
Commission and every officer appointed
(d) Article 149
or authorized by the Commission or State
70. Who is not a member of the committee Commission to exercise functions under
for the appointment of Chairperson and the Protection of Human Rights Act is to be
members of the National Human Rights deemed- [M.P.P.C.S. (Pre) 2018]
Commission? [M.P.P.C.S. (Pre) 2019]
(a) Public Officer
(a) Speaker of the House of People (b) Public Servant
(b) Chairman of the Council of States (c) Officer of Commission
(c) Leader of opposition in the House of
(d) None of the above
People
(d) Leader of opposition in the Council of 76. In relation to offences described under
States Section 175, Section 178, Section 179,
71. Chairperson and Members of the State Section 180 or Section 228 of the Indian
Human Rights Commission are appointed Penal Code, the Human Rights Commission
by- [M.P.P.C.S. (Pre) 2019] is to be deemed- [M.P.P.C.S. (Pre) 2018]
(a) Governor (a) Criminal Court
(b) President (b) Civil Court
(c) Chief Justice of the High Court of the (c) Revenue Court
concerned State (d) None of the above
(d) Chief Justice of India 77. While dealing with complaints of violation
72. Who is not the ex-officio member of the of Human Rights by the members of the
National Human Rights Commission? armed forces, the Commission either on its
[M.P.P.C.S. (Pre) 2018] own motion or on receipt of a petition will?
(a) The Chairperson of the National [M.P.P.C.S. (Pre) 2018]
Commission for Minorities (a) Enquire itself
(b) The Chairperson of the Law Commission (b) give directions to the concerned police
of India officers to enquire
(c) The Chairperson of the National (c) seek a report from the Central
Commission for the Scheduled Castes Government
and Scheduled Tribes (d) None of the above
(d) The Chairperson of the National
Commission for Women 78. What was the objective of the Protection of
Human Rights Act, 1993?
73. A Sitting Judge of the High Court or a
Sitting District Judge can be appointed [M.P.P.C.S. (Pre) 2015]
as a member of the State Human Rights (a) Better protection of Human Rights
Commission after consultation with? (b) To constitute Human Rights Protection
[M.P.P.C.S. (Pre) 2018] Commission
(a) the Governor (c) To constitute Human Rights Protection
(b) the Chief Justice of the High Court of the Commission in the State
concerned State (d) All of the above
(c) the Chief Justice of the Supreme Court of 79. Which one of the following is the ‘National
India Voters Day’? [M.P.P.C.S. (Pre) 2012]
(d) the President (a) 5th June
74. The State Human Commission submits its (b) 1st November
annual report to the - (c) 25th January
[M.P.P.C.S. (Pre) 2018] (d) 8th March

279 PYQ Workbook


INDIAN POLITY AND GOVERNANCE

80. In India, which one of the following is 4. Undertaking and promoting research in
responsible for the implementation of the the field of human rights.
Government of India’s policy in all matters Select the correct answer using the codes given
concerning telecommunications? below : [CDS Pre.2018 I]
[CDS 2022 (II)] (a) 1 and 2
(a) Digital Communications Commission (b) 2, 3 and 4
(b) Telecom Regulatory Authority of India (c) 1, 3 and 4
(c) Telecom Advisory Committee
(d) All of these
(d) Telecommunications Consultants India
Limited 85. The National Commission for Women was
81. Which one of the following Commissions is created by [CDS Pre.2017 II]
related to Article 338A? [CDS 2020 (II)] (a) an amendment in the Constitution of
(a) The National Commission for Scheduled India
Castes (b) a decision of the Union Cabinet
(b) The National Commission for Scheduled (c) an Act passed by the Parliament
Tribes (d) an order of the President of India
(c) The National Commission for Backward 86. Which one of the following statements with
Classes regard to the National Commission for
(d) The National Commission for Women
Scheduled Tribes is not correct
82. Which one of the following is not correct [CDS Pre.2016 II]
about Administrative Tribunals? (a) The Union and every State Government
[CDS Pre. 2019 I] shall consult the commission on all major
(a) The Parliament may Constitute policy matters affecting Scheduled Tribes
Administrative Tribunals both at the (b) All the reports of the commission and
Union and State levels its recommendations shall be laid only
(b) Tribunals may look into complaints with before Lok Sabha
respect to recruitment and conditions of (c) The commission, while investigating any
service of persons appointed to public matter, has all the powers of a Civil Court
services
(d) The commission has the power to regulate
(c) Tribunals established by a law of the
Parliament can exclude the jurisdiction its own procedures.
of all courts to allow to special leave to 87. Which of the following are the essential
appeals. requirements for a person to be appointed
(d) The law establishing the Tribunals may as a member of the Finance Commission?
provide for procedures including rules of 1. A Supreme Court judge or one qualified
evidence to be followed. to be appointed as such.
83. The Prime Minister’s National Relief Fund 2. A person having wide experience in
is operated by which one of the following financial matters and administration.
bodies? [CDS Pre. 2019 I] 3. A person having special knowledge of
(a) The Prime Minister’s Office (PMO) economics.
(b) The National Disaster Management
Authority Select the correct answer using the code given
(c) The Ministry of Finance be low: [CAPF 2019]
(d) The National Development Council (a) 1, 2 and 3
(NDC) (b) 1 and 2 only
(c) 2 and 3 only
84. Which of the following are the functions of (d) 1 and 3 only
the National Human Rights Commission
(NHRC)? 88. Who among the following is not associated
1. Inquiry at its own initiative on the to the Governing Council of NITI Aayog?
violation of human rights. [CAPF 2018]
2. Inquiry on a petition presented to it by a (a) The Prime Minister
victim. (b) The President
3. Visit to jails to study the condition of the (c) The Chief Ministers of States
inmates. (d) The Chief Ministers of Union Territories

PYQ Workbook 280


INDIAN POLITY AND GOVERNANCE

89. If ‘One Nation One Election’ is to be realized 1. The Finance Commission is a statutory
in India, which Article of the Indian body
Constitution will require an amendment? 2. The finance Commission was set up
[67th B.P.S.C. (Pre) (Re-Exam), 2022] under Article 280 of the Constitution
(a) Article 83 3. The recommendations made by the
(b) Article 172 Finance Commission are only advisory in
(c) Article 356 nature
(d) Article 246 4. The First Finance Commission was set up
(e) None of the above/More than one of the in 1950
above Which of the above statements are correct?
(a) 1 and 4 only
90. With reference to the Finance Commission (b) 3 and 4 only
of India, consider the following statements: (c) 2 and 3 only
[67th B.P.S.C. (Pre), 2021] (d) 2 and 4 only

281 PYQ Workbook


INDIAN POLITY AND GOVERNANCE

SOLUTIONS

opening and reopening of coal mines... has been entrusted


8.1. UPSC CSE Previous Years’ Questions to CCO.
1. Solution: (a) Statement 3 is correct. Under Coal Bearing Area
Exp) Option a is the correct answer (Acquisition and Development) Act, 1957-
Option 1 is correct: Although the National Commission for Coal Controller is the competent authority under this act to
Backward Classes (NCBC) was originally constituted as the hear any objection to the Central Government’s Notification
Statutory body by establishing it under the provisions of relating to acquisition of coal bearing land and to furnish his
the National Commission for Backward Classes Act, 1993. reports to Central Govt.
Recently the NCBC was accorded constitutional status by Statement 4 is incorrect. Ensuring that coal mining
the 102nd Constitutional Amendment Act (CAA) which companies deliver the coal to end users in the prescribed
inserted Article 338B, forming a National Commission for time is not the function of Coal Controller’s Organization
Backward Classes. Hence NCBC is a constitutional body. (CCO).
Option 2 is incorrect: The National Human Rights
3. Solution: (d)
Commission (NHRC) of India is a statutory body constituted
in 1993 under the Protection of Human Rights Act, 1993. Exp) Option d is the correct answer.
Hence NHRC is not a constitutional body. Statement 1 is correct: The Tea Board of India is a statutory
Option 3 is incorrect: The National Law Commission is body created under the Tea Act, 1953 and it was established
neither a constitutional body nor a statutory body, it was for the purposes of regulating the Indian tea industry and
constituted by a notification of the Government of India, protecting the interests of tea producers in India.
Ministry of Law & Justice, Department of Legal Affairs. Its Statement 2 is incorrect: It is functioning as a statutory
purpose is to carry out research in the field of law and the body of the Central Government under the Ministry of
Commission makes recommendations to the Government Commerce.
and provides excellent thought provoking and vital review Statement 3 is incorrect: Tea Board of India’s Head Office is
of the laws in India. situated in Kolkata.
Option 4 is incorrect: The National Consumer Disputes Statement 4 is correct: The Tea Board of India has
Redressal Commission of India is a statutory body overseas offices in Moscow, Dubai, Hamburg, London
established under the Consumer Protection Act, 1986. To and New York. Moscow office. The Moscow office of
provide inexpensive, speedy redressal of consumer disputes, the Tea Board of India operates under the Embassy of India,
the act established quasi-judicial bodies have been set up and its area of activity includes Russia and the CIS countries,
in each District and State and at the National level, called which comprise 50% of Indian tea exports.
the District Consumer Disputes Redressal Commissions,
the State Consumer Disputes Redressal Commissions and 4. Solution: (b)
the National Consumer Disputes Redressal Commission Exp) Option b is the correct answer.
respec tively. Article 142 of Constitution of India deals with Enforcement
of decrees and orders of the Supreme Court. It states that the
2. Solution: (a) Apex Court in the exercise of its jurisdiction may pass such
Exp) Option a is the correct answer. decree or make such order as is necessary for doing “complete
The Coal Controller’s Organisation (CCO) is a subordinate justice” in any case pending before it. Such orders of the
office of the ministry of coal, having its headquarters at Supreme Court are enforceable throughout the territory of
Kolkata and field offices at Dhanbad, Ranchi, Bilaspur, India as prescribed by any law made by Parliament or order
Nagpur, Sambalpur, Kothagudem and Asansol. It collects of the President of India. Article 142 enables superseding
and maintains coal production data of all private and public the executive and the legislative. Thus, the Supreme Court
sector coal mines in the country. The information is collected of India is not constrained in the exercise of its powers by
on a monthly basis. laws made by the Parliament. From Article 142, the Supreme
Court derives overarching powers to perform the functions
Statement 1 is correct. Under Collection of Statistics Act,
of Executive and legislative in order to bring about complete
2008 Coal Controller has been made the statistical authority
justice.
with respect to coal and lignite statistics. Entrusted the
responsibility of carrying out Annual Coal & Lignite survey 5. Solution: (c)
and publishing of Provisional Coal Statistics and Coal Exp) Option c is the correct answer.
Directory of India. Statement 1 is correct. The Quality Council of India, an
Statement 2 is correct. It is entrusted with the task of autonomous body set up by the Ministry of Commerce and
monitoring captive mines. Work such as permission for Industry, Government of India jointly with the Indian

PYQ Workbook 282


INDIAN POLITY AND GOVERNANCE

Industry, represented by the three premier industry 2. The Report of the Public Accounts Committee
associations i.e., Associated Chambers of Commerce and 3. The Report of the Comptroller and Auditor General
Industry of India (ASSOCHAM), Confederation of Indian 4. The Report of the National Commission for Scheduled
Industry (CII) and Federation of Indian Chambers of Castes
Commerce and Industry (FICCI). It aims to establish and 5. The Report of the National Commission for Scheduled
operate national accreditation structure and promote quality Tribes
through National Quality Campaign.
6. Reports of the Comptroller and Auditor General
Statement 2 is correct. Chairman of QCI is a non-executive
7. The Report of the Union Public Service Commission
post. He is nominated by the Prime Minister of India on the
8. The Report of the National Commission of Backward
recommendations of the industry.
Classes
6. Solution: (c) The President does not lay the reports of Public Accounts
Exp) Option c is the correct answer. Committee before the Parliament. In fact, the function of
The Attorney General of India (AGI) is the only person under the committee is to examine the annual audit reports of the
the Indian system who can take part in the proceedings of Comptroller and Auditor General of India (CAG), which are
the Parliament or any parliamentary committee, but cannot laid before the Parliament by the President.
vote. 9. Solution: (d)
Statements 1, 2 and 3 are correct. He has the right to speak Exp) Option d is the correct answer.
and to take part in the proceedings of both the Houses of
Set up under Article 280 of the Constitution, the Finance
Parliament or their joint sittings and in any committee of
Commission is a Constitutionally mandated body that
the Parliament of which he may be named a member, but
is at the centre of fiscal federalism. Its core responsibility
without the right to vote. is to evaluate the state of finances of the Union and State
7. Solution: (c) Governments, recommend the sharing of taxes between
Exp) Option c is the correct answer. them, lay down the principles determining the distribution
of these taxes among States. Thus, none of the statements
The Comptroller and Auditor General (CAG) of India is the given in above options are correct with reference to
constitutional authority, established under Article 148 of the Finance Commission.
Constitution of India.
The Finance Commission is required to make
He is empowered to Audit all receipts and expenditure of the recommendations to the President of India on the following
Government of India and the State Governments, including matters:
those of autonomous bodies and corporations substantially 1. The distribution of the net proceeds of taxes to be shared
financed by the Government. between the Centre and the states, and the allocation
The CAG is also the statutory auditor of Government- between the states of the respective shares of such
owned corporations and conducts supplementary audit of proceeds.
government companies in which the Government has an
2. The principles that should govern the grants-in-aid to
equity share of at least 51 per cent or subsidiary companies the states by the Centre (i.e., out of the consolidated fund
of existing government companies. of India).
Statements 2 and 3 are correct. The reports of the CAG are
3. The measures needed to augment the consolidated fund
laid before the Parliament/Legislatures and are being taken
of a state to supplement the resources of the panchayats
up for discussion by the Public Accounts Committees (PACs)
and the municipalities in the state on the basis of the
and Committees on Public Undertakings (COPUs), which
recommendations made by the state finance commission.
are special committees in the Parliament of India and the
state legislatures. Also, information from CAG reports can be 4. Any other matter referred to it by the president in the
used by investigating agencies to press charges against those interests of sound finance.
who have violated the law while managing public finances. 10. Solution: (b)
This was seen during 2G and Commonwealth Games scams.
Exp) Option b is the correct answer
Statements 1 and 4 are incorrect. CAG does not exercise
The Function of the Finance Commission to make
exchequer control on behalf of the Parliament when the
recommendations to the President as to:
President of India declares national emergency/financial
emergency. The CAG does not have any judicial powers. The distribution between the Union and the States of the
net proceeds of taxes which are to be, or maybe, divided
8. Solution: (c) between them and the allocation between the States of the
Exp) Option c is the correct answer. respective shares of such proceeds. Hence statement 2 is
According to the Constitution of India, it is the duty of the President correct.
of India to cause to be laid before the Parliament following reports - The principles which should govern the grants-in-aid of
1. The Recommendations of the Union Finance the revenues of the States out of the Consolidated Fund of
Commission India; Hence statement 3 is correct.

283 PYQ Workbook


INDIAN POLITY AND GOVERNANCE

Important Tips would help to reduce the influence of money in elections


and make it easier for independent candidates to contest
Finance Commission
elections
• The Finance Commission is a Constitutionally
mandated body that is at the centre of fiscal federalism. Important Tips

• It was set up under Article 280 of the Constitution. Other Important Committees/Reports related to Electoral
Reforms:
• The first Finance Commission was set up in 1951
• Tarkunde Committee (1974)
and there have been fifteen Finance commission so
far. • Jaya Prakash Narayan Committee (1974)
• Goswami Committee on Electoral Reforms (1990)
11. Solution: (a)
• Vohra Committee Report (1993)
Exp) Option a is the correct answer
• Indrajit Gupta Committee on State Funding of
The President shall cause every recommendation made Elections (1998)
by the Finance Commission under the provisions of
• Law Commission Report on Reform of the Electoral
Constitution together with an explanatory memorandum as
Laws (1999)
to the action taken thereon to be laid before each House of
Parliament. • National Commission to Review the Working of the
Constitution (2001)
12. Solution: (d) • Election Commission of India - Proposed Electoral
Exp) Option d is the correct answer. Reforms (2004)
Section 8 of the Representation of the People Act, 1951 • Jeevan Reddy Committee (2004)
provides that a person who has been convicted of a crime • The Second Administrative Reforms Commission
and sentenced to imprisonment for two years or more is (2008)
disqualified from contesting elections to Parliament or the
State Legislatures. It is a law made by Parliament. Article 15. Solution: (b)
102(1)(e) of the Constitution of India provides that a person Exp) Option b is the correct answer.
shall be disqualified for being chosen as, and for being, a Proportional representation is a system of voting in which
member of either House of Parliament if he is so disqualified the number of seats a party wins is proportional to the
by or under any law made by Parliament. number of votes it receives. This system is often used in
13. Solution: (a) countries with a multi-party system, as it ensures that all
shades of opinion are represented in the legislature. A two-
Exp) Option a is the correct answer
party system is a system in which there are only two major
Article 280 of the Constitution of India provides that, political parties. In a two-party system, the party that wins
every five years, the President shall constitute a Finance the most votes usually wins a majority of the seats in the
Commission, which shall recommend assistance/grants legislature. This means that there is no need for proportional
from the Consolidated Fund of India. representation, as the majority party will already be
It is the duty of the Commission to make recommendations represented in the legislature.
to the President as to:
16. Solution: (c)
• The distribution between the Union and the States
Exp) Option c is the correct answer.
of the net proceeds of taxes which are to be, or maybe,
divided between them and the allocation between the The Election Commission decides the schedule for elections
States of the respective shares of such proceeds. to the Parliament or the State Legislature, taking into account
the recommendations of the respective government. While
• The principles which should govern the grants-in-aid of
the Election Commission announces the schedule, the actual
the revenues of the States out of the Consolidated Fund
election process commences only after the President, or the
of India;
Governor issues the official notification.
• The measures needed to augment the Consolidated Fund
of a State to supplement the resources of the Panchayats Important Tips
in the State on the basis of the recommendations made by Role of the President in Elections:
the Finance Commission of the State. • As the head of the state, the President plays a crucial
14. Solution: (c) role in the electoral process of the country.

Exp) Option c is the correct answer. • The President’s power to issue the election notification
marks the beginning of the electoral process.
The Dinesh Goswami Committee was constituted
in 1990 to review the electoral system in India. The • The appointment of the Chief Election Commissioner
Committee recommended that the government should fund and other Election Commissioners is made by the
parliamentary elections. The Committee argued that this President.

PYQ Workbook 284


INDIAN POLITY AND GOVERNANCE

• The President ensures that the elections are conducted for a specific candidate. The party then presents a list of
with fairness and impartiality. candidates, and the seats in the legislature are allocated to
the parties in proportion to the number of votes they receive.
• Role of the Election Commission of India:
This system is not used in India.
• The Election Commission of India (ECI) operates as
Statement 4 is incorrect- The Cumulative System is an
an independent constitutional body with the primary
indirect election system in which voters vote for a group of
responsibility of conducting transparent elections in
candidates, not for a single candidate. This system is used
the country.
in the USA for election to the members of Senate.
• The ECI proposes the dates for the elections to the
President of India. 20. Solution: (d)
• It lays down the rules and regulations for conducting Exp) Option d is the correct answer.
elections and ensures compliance by all involved Statement 1 is incorrect- The elections for the post of
parties. Speaker and Deputy-Speaker of Lok Sabha and the Deputy
• The ECI conducts elections for the Lok Sabha, Rajya Chairman of Rajya Sabha are conducted by the respective
Sabha, and State Legislative Assemblies. Houses of Parliament (Lok Sabha and Rajya Sabha)
themselves.
17. Solution: (d)
Statement 2 is incorrect- Local body elections, including
Exp) Option d is the correct answer. elections for municipalities and municipal corporations, are
If a candidate fails to secure the minimum percentage of conducted by the respective State Election Commissions in
votes required in an election to a State Legislative Assembly, each state.
it results in the forfeiture of the candidate’s deposit. The Statement 3 is incorrect- The power to make a decision
minimum percentage of votes needed to retain the deposit is on all the doubts and disputes arising from the election to
1/6th of the total votes polled. This outcome signifies that the President and Vice President is vested in the Supreme
the candidate was unable to gather sufficient support from Court and for election to Parliament and State Legislature
the voters during the election. The reason for low support is vested in High Court.
could be that a very large number of candidates contested,
leading to the division of votes among them. This makes it Important Tips
difficult for any one candidate to obtain a significant number Some important functions of the Election Commission of
of votes. India:
• Delimitation of Electoral Constituencies: The
18. Solution: (a) Election Commission is responsible for determining
Exp) Option a is the correct answer. the territorial areas of electoral constituencies across
the country, ensuring a fair and equitable distribution of
Exit poll is a survey of voters conducted after they have constituencies based on the Delimitation Commission
cast their ballots. The purpose of an exit poll is to gauge Act.
the voting preferences of the electorate and to predict the • Electoral Roll Preparation and Revision: The
outcome of an election. Opinion polls, on the other hand, Election Commission plays a crucial role in preparing
are surveys of voters conducted before they have cast their and periodically updating electoral rolls, ensuring
ballots. The purpose of an opinion poll is to gauge the voting the registration of all eligible voters and maintaining
preferences of the electorate and to predict the outcome of accurate voter lists.
an election. • Conducting Elections: The Election Commission
is tasked with notifying the dates and schedules
19. Solution: (a) of elections, scrutinizing nomination papers, and
overseeing the entire electoral process to ensure fair
Exp) Option a is the correct answer.
and transparent elections.
Statement 1 is correct- Direct Election System on the basis • Recognition of Political Parties: The Election
of Adult Suffrage is used for elections to the Lok Sabha Commission grants recognition to political parties
(House of the People) and State Legislative Assemblies. It and allocates election symbols to them, which is a
allows citizens above the age of 18 years (adult suffrage) to significant aspect of the democratic electoral process.
vote directly for their representatives. • Dispute Settlement: The Election Commission acts
as a court for settling disputes related to granting
Statement 2 is correct- The System of Proportional
recognition to political parties, allotment of election
representation by a Single Transferable Vote is used for the symbols, and inquiring into disputes regarding
elections to the President and Vice-President. It is a form of electoral arrangements.
proportional representation where voters rank candidates
in order of preference, and candidates need to reach a 21. Solution: (d)
certain quota to be elected. Exp) Option d is the correct answer
Statement 3 is incorrect- The List System is a proportional The Rajamannar Commission, Rajamannar Commission
representation system in which voters vote for a party, not on Centre State relation was set up by the then Government

285 PYQ Workbook


INDIAN POLITY AND GOVERNANCE

of Tamil NAdu in 1960 under under the Chairmanship of • For a brief while after the establishment of Haryana
Dr. P.V. Rajamanar. It was appointed to examine all aspects from Punjab in 1966, the two states of Punjab and
of Centre-State relations and to investigate and identify Haryana had a JSPSC.
the causes of unitary tendencies in Central-State relations.
• The chairman and members of a JSPSC are appointed
It proposed the abolition of IAS and IPS due to their by the President.
excessive centralization, which was seen as a hindrance to
• They hold the office for a period of six years or until
local self-government.
they reach the age of 62, whichever comes first.
22. Solution: (c) • They can be fired or suspended by the President. They
Exp) Option c is the correct answer can also send the president their resignation letters at
any time.
The Swaran Singh Committee, formed in 1976, was tasked
with examining the relationship between the DPSP and • The president determines the number of JSPSC
fundamental rights. The committee explored ways to ensure members and their service conditions.
greater effectiveness and implementation of DPSP and to • A Joint State Public Service Commission presents
harmonize their objectives with fundamental rights. each of the concerned state governors with an annual
performance report. The report is presented to the
Important Tips state legislature by the governor.
Swaran Singh Committee Recommendations:
24. Solution: (c)
1. A separate chapter under the name Fundamental
Duties was recommended for inclusion in the Indian Exp) Option c is the correct answer
Constitution. Article 317 of the Constitution of India provides that a
2. The committee proposed eight Fundamental Duties, member of a State Public Service Commission (SPSC)
but the 42nd amendment included ten. Not every may be removed by the President on the ground of proved
recommendation was approved out of the whole set of misbehaviour or incapacity after the enquiry conducted by
recommendations. the Supreme Court.
3. Certain recommendations of the Committee were Important Tips
not accepted by the Congress Party and hence, not
Article 317(3): Conditions for Removal
incorporated in the Constitution. These include:
• The Parliament may provide for the imposition of The Chairman or any other member of UPSC may be
such penalty or punishment as may be considered removed if he/she:
appropriate for any non-compliance with or refusal to • is adjudged an insolvent.
observe any of the duties.
• engages during his/her term of office in any paid
• No law imposing such penalty or punishment shall
be called in question in any court on the ground of employment outside the duties of his/her office.
infringement of any of Fundamental Rights or on the • is, in the opinion of the President, unfit to continue
ground of repugnancy to any other provision of the in office by reason of infirmity of mind or body.
Constitution.
• Duty to pay taxes should also be a Fundamental Duty 25. Solution: (d)
of the citizens. Exp) Option d is the correct answer
Article 280 of the Constitution of India provides for a Finance
8.2. Other Examination Previous Years’ Commission as a quasi-judicial body. It is constituted by
Questions the President of India every fifth year or at such an earlier
time as he considers necessary. The Finance Commission
23. Solution: (a)
consists of a chairman and four other members to be
Exp) Option a is the correct answer
appointed by the president. They hold office for such a
According to Article 315(2) of the Indian Constitution, period as specified by the president in his order. They are
two or more States may pass a resolution in their respective eligible for reappointment. The Constitution authorises the
legislatures, agreeing to establish a Joint State Public Service Parliament to determine the qualifications of members of
Commission. Upon such agreement, Parliament can enact the commission and the manner in which they should be
a law for the appointment of the Joint Public Service selected.
Commission.
26. Solution: (c)
Important Tips
Exp) Option c is the correct answer.
Joint State Public Service Commission
Statement 1 is correct- Sukumar Sen was the first Chief
• A JSPSC is a statutory body. It does not have the Election Commissioner of India. He served from 21 March
status of a constitutional body. 1950 to 19 December 1958.

PYQ Workbook 286


INDIAN POLITY AND GOVERNANCE

Statement 2 is correct- Rama Devi was the first woman Important Tips
Chief Election Commissioner of India. She served from 26
• Extra- Constitutional Bodies are those which are not
January 1975 to 31 December 1977. defined in the Constitution of India.
27. Solution: (d) • Extra- Constitutional Bodies are of further two
types:
Exp) Option d is the correct answer
• Statutory Bodies: These are those bodies that are
According to Article 317, the removal of the Chairman of
formed by passing a law of either of the parliament of
the Union Public Service Commission (UPSC) can only India or by the respective state legislature(s).
be done by the President based on the ground of proved
• Some examples are: SEBI, National Human Rights
misbehavior or incapacity. Parliament is not involved in Commission, National Green Tribunal, etc.
the process. The removal process involves a reference made
• Executive Bodies: These are those bodies that are
by the President to the Supreme Court, which conducts an formed by the executive order of the government.
inquiry following the procedure prescribed under Article There is no need for parliament or state legislatures
145. The Supreme Court’s report determines whether the approval to form these bodies.
Chairman should be removed from their office. In the • For example: Niti Aayog, National Development
removal of Judges of High Court, Supreme Court and Council, Law Commission, etc.
CAG, Parliament is involved. • Constitutional Bodies are those which are defined
Important Tips in the constitution of India and derives their powers,
limitations and other features directly from the
• A member of the higher judiciary, which means the
Constitution of India. These are those bodies which are
Judges and Chief Justices of the Supreme Court of
clearly mentioned in some article of the Constitution
India and the state High Courts, can be removed from
itself.
service only through the process of impeachment by
Parliament under Article 124 (4) of the Constitution • For example- UPSC, State Public Service Commissions,
and Judges Inquiry Act 1968. CAG, Election Commission, Finance Commission and
many others are constitutional Bodies.
• The Comptroller and Auditor General of India shall be
appointed by the President by a warrant under his hand 30. Solution: (c)
and seal and shall only be removed from office by an
Exp) Option c is the correct answer.
order of President passed after an address by each
House of parliament supported by a majority of the Assertion (A) is true- The MCC (Model Code of Conduct)
total membership of that house and by a majority of consists of guidelines issued by the Election Commission
not less than two-thirds of the members of that house to govern the behavior of political parties and candidates
present and voting has been presented to the President before elections. While it is not a legally binding document,
in the same session for such removal on the ground of
Political Parties, Candidates, and Polling Agents are expected
proved misbehavior or incapacity.
to adhere to these norms. The Model Code of Conduct
28. Solution: (c) (MCC) is in effect from the date the Election Commission
Exp) Option c is the correct answer. announces the election schedule until the date the results are
The accounting function was separated from the auditing announced.
function in 1976, when the Department of Expenditure Reason (R) is false- The Election Commission of India issues
was bifurcated into two separate departments, namely, the Election Model Code of Conduct (MCC), which serves
Department of Expenditure and Department of Accounts and as a set of guidelines for ensuring free and fair elections in
Audit. The latter was renamed as Department of Expenditure the country.
and Controller General of Accounts in 1985.
31. Solution: (c)
Since then, the CAG’s duty is only confined to auditing
Exp) Option c is the correct answer
the government accounts, while the accounting function is
performed by the various departments themselves with the The State Finance Commission is a Constitutional Body, as
help of the Indian Civil Accounts Service. it is formed under the 73rd Constitutional Amendment Act
1992. According to article 280 (3) (bb) - It shall be the duty of
29. Solution: (d) the Commission to make recommendations to the President
Exp) Option d is the correct answer. as to - the measures needed to augment the Consolidated
NITI Aayog is an extra constitutional agency as it is not Fund of a State to supplement the resources of the Panchayats
mentioned in the Constitution. It is created by Executive in the State on the basis of the recommendations made by the
orders of the Union Cabinet. Finance Commission of the State.

287 PYQ Workbook


INDIAN POLITY AND GOVERNANCE

Important Tips 34. Solution: (d)

• Under Article 280, the State Finance Commission was Exp) Option d is the correct answer.
established in all the states of India from 1993 on the Statement 1 is correct- The election to the State Legislature
lines of the Finance Commission of the Center. is on the basis of adult suffrage. This means that all citizens
• According to Article 243 I of the Indian Constitution, above 18 years of age have the right to vote in elections.
the State Finance Commission is appointed by the Statement 2 is correct- According to Section 15 of
Governor for a period of 5 years.
Representation of People Act, 1950, each territorial
• The Governor also appoints the other members constituency will have only one electoral roll containing the
(maximum 4) for the commission. names of eligible voters.
32. Solution: (a) Statement 3 is correct- According to the Article 325 of the
Exp) Option a is the correct answer Indian Constitution, the electoral roll is prepared without
any consideration of factors like religion, race, sex, etc.,
Maharashtra was the first state to introduce Lokayukta.
ensuring impartiality and non-discrimination.
The Maharashtra Lokayukta and Upa-Lokayuktas Act, 1971
was passed by the Maharashtra Legislative Assembly in 1971, Statement 4 is correct- According to the Representation of
and it came into effect on October 25, 1972. The Lokayukta People Act, 1951-Political parties are allowed to have their
is an independent statutory authority that investigates internal rules and regulations as long as they abide by the
allegations of corruption against public officials in the state laws and regulations set by the Election Commission.
of Maharashtra.
35. Solution: (c)
33. Solution: (d) Exp) Option c is the correct answer.
Exp) Option d is the correct answer The Sixty-first Amendment of the Constitution of India,
The concept of human rights is based on the belief that passed in 1988, reduced the voting age from 21 years to 18
all human beings are born free and equal in dignity and years. The amendment was passed to give citizens of India
rights. This means that all human beings have certain above 18 years of age, the right to vote.
fundamental rights and freedoms, simply because they are
Important Tips
human beings. The dignity of man as a human being is the
foundation of the concept of human rights. It is the belief • Sixty-third Amendment- Punjab was brought at
that all human beings are worthy of respect and that they par with the other states in respect of emergency
should be treated with dignity. provisions.
• Sixty-second Amendment- Extended the reservation
Important Tips of seats for the SCs and STs and special representation
Natural Rights: for the Anglo-Indians in the Lok Sabha and the state
• Fundamental rights inherent to every individual as legislative assemblies for the further period of ten
human beings. years (i.e., up to 2000).

• Not granted by any government or authority, considered • Sixtieth Amendment- Increased the ceiling of taxes
universal and inalienable. on professions, trades, callings and employments
from ‘250 per annum to ‘2,500 per annum.
• Examples: The right to life, the right to liberty, the
right to possess property, the right to make a living 36. Solution: (d)
Constitutional Rights: Exp) Option d is the correct answer.
• Specifically protected and guaranteed by a country’s The Election Commission is an independent body that is
constitution. responsible for conducting elections to the Parliament and
• Ensured by the constitution to provide legal protection State Legislatures in a free and fair manner. The Parliament
to citizens. (and not the Election Commission) has the power to make
• Examples: Freedoms of speech, religion, assembly, etc provision with respect to elections to Legislatures.
Legal Rights: 37. Solution: (d)
• Recognized and enforced by the legal system of a Exp) Option d is the correct answer.
country.
Under Article 103 of the Constitution of India, disputes
• Encompass natural and constitutional rights, as well as
regarding the disqualification of Members of Parliament are
other rights created by laws and regulations.
decided by the President in consultation with the Election
• Examples: Right to education, right to a fair trial, rights Commission. The advice tendered by Election Commission
related to employment. in this regard is binding on the President of India.

PYQ Workbook 288


INDIAN POLITY AND GOVERNANCE

38. Solution: (c) Important Tips


Exp) Option c is the correct answer The duties of CAG include:
The Government of India Act, 1919, established the Public To audit and report on all expenditure from the
Service Commission in India for the first time. Hence Consolidated Fund of India, Consolidated Fund of each
Central Public service commission was set up in 1926 for State, Consolidated Fund of each Union Territory having
recruiting Civil servants. The Public Service Commission a Legislative Assembly, Contingency Funds and Public
was set up to oversee the recruitment and appointments to Accounts of India, and Contingency Funds and Public
the civil services of the British India government. Accounts of each State or Union Territory having a
Legislative Assembly under Article 149.
39. Solution: (b)
• To audit and report on all trading, manufacturing,
Exp) Option b is the correct answer. profit and loss accounts, balance sheets, and other
The Comptroller and Auditor General (CAG) of India is subsidiary accounts kept by any department of the
often referred to as the “Guardian of Public Purse.” This Union or a State.
title is attributed to the CAG because of their role in auditing • To perform such duties and exercise such powers in
and ensuring accountability in the financial administration relation to the accounts of any other authority or body
of the government. The CAG is responsible for examining as may be prescribed by or under any law made by
and reporting on the collection and expenditure of public Parliament under Article 149
funds, ensuring that they are utilized in a lawful and efficient • To submit his/her audit reports to the President or
manner. Governor.
He ensures that all expenditure made from the
42. Solution: (d)
Consolidated Fund of India and each State and Union
Exp) Option d is the correct answer.
Territory is done as per law and that the executive (i.e., the
council of ministers) is accountable to Parliament and State The accounts of the States are controlled by the Comptroller
Legislatures and through them to the people in the area of and Auditor General of India (CAG), who is the supreme
financial administration. audit authority of the country and audits the accounts of the
Union and State governments.
40. Solution: (b) The CAG is responsible for compiling the accounts of all
Exp) Option b is the correct answer. States, as per Section 11 of the Comptroller and Auditor
General’s (Duties, Powers and Conditions of Service) Act,
Assertion (A) is true- The Election Commission is an
1971. The accounts of the States are kept in such forms as the
independent body that is responsible for conducting
President may, on the advice of the CAG, prescribe under
elections to the Parliament and State Legislatures in a free Article 150 of the Constitution. The CAG submits his/her
and fair manner. The Election Commission has been given audit reports on the accounts of the States to the Governor,
a number of powers to ensure that elections are conducted who shall cause them to be laid before the Legislature of the
fairly, including the power to register political parties, to State under Article 151 (2) of the Constitution.
derecognize political parties, to conduct elections, and to
43. Solution: (b)
disqualify candidates.
Exp) Option b is the correct answer
Reason (R) is true- The power of removal of Election
Commissioners is with the President of India, who is the Statement 1 is correct: The report of the commission was of
a recommendatory nature, and any proposed amendments
head of the executive. However, the President can only
would need to be approved and adopted by Parliament.
remove an Election Commissioner on the grounds of proved
misbehavior or incapacity. However, Reason (R) is not a Statement 2 is incorrect: The National Commission for
Revising the Constitution was appointed by the Government
correct explanation of Assertion (A).
of India in 2000.
41. Solution: (d) Statement 3 is correct: The Commission was headed by
Exp) Option d is the correct answer. Justice M.N. Venkatchelaiah, a former Chief Justice of India.
The duties and powers of CAG are mainly derived from Statement 4 is incorrect: The primary task of the National
Articles 149 to 151 of the Constitution and the Comptroller Commission for reviewing the Constitution was to examine
its provisions and propose necessary amendments. The
and Auditor General’s (Duties, Powers and Conditions of
commission’s scope did not specifically include addressing
Service) Act, 1971.
the socio-economic needs of the country.
The power to control the receipt and issue of public
money, and to ensure that the public revenue is lodged in 44. Solution: (b)
the Exchequer is not a duty performed by CAG. This power Exp) Option b is the correct answer.
belongs to the executive branch of the government under The Chief Election Commissioner can be removed from
Article 266 to 268A of the Constitution. office by the same procedure which applies in case of removal

289 PYQ Workbook


INDIAN POLITY AND GOVERNANCE

of Supreme Court Judges. This means that the Chief Election minorities are represented in the legislature. This is because
Commissioner can be removed from office only on the even if a minority party does not win a majority of the votes,
grounds of proved misbehavior or incapacity and he can be it may still be able to win a few seats in the legislature. This
removed by the president only when a motion is passed by gives the minority party a voice in the legislature and helps
both Houses of Parliament with a two-thirds majority. to ensure that their interests are represented.

45. Solution: (b) 49. Solution: (c)


Exp) Option b is the correct answer Exp) Option c is the correct answer.
Assertion (A) is true: Constitution of India does not define The Chief Election Commissioner of India is appointed
the term “minority”. The term is used in several articles of by the President of India. The appointment is made in
the Constitution, but it is not defined anywhere. consultation with the Prime Minister, the Chief Justice of
Reason (R) is true: The Minority Commission, also known as India, and the Leader of the Opposition in the Lok Sabha.
the National Commission for Minorities, is a statutory body The Chief Election Commissioner is the head of the Election
established by an Act of Parliament (National Commission Commission of India, which is responsible for conducting
for Minorities Act, 1992). elections to the Lok Sabha, the Rajya Sabha, and the State
Both (A) and (R) are true, but (R) does not explain (A) Legislative Assemblies. He is also responsible for ensuring
correctly: The two statements are independent of each other, that elections are conducted in a free and fair manner.
and there is no causal relationship between both statements.
Important Tips
46. Solution: (b) Structure of Election Commission of India:
Exp) Option b is the correct answer. • Since its inception in 1950 and till 1989, the Election
Option a and c are correct: On 1 January 2015, the Commission functioned as a single member body
Government of India has decided to set up NITI Aayog consisting of the Chief Election Commissioner only.
(National Institution for Transforming India), in place of But the Election Commissioner Amendment Act of
the erstwhile Planning Commission, as a means to better 1989 transformed it into a multi-member body.
serve the needs and aspirations of the people of India. • As per Article 324 of the Indian Constitution, the
Election Commission consist of the Chief Election
Option b is incorrect: The Prime Minister is the Chairperson
Commissioner and such number of other Election
of NITI Aayog. He/She shall be the Chairman as long as he/
Commissioners as the President from time-to-time
she is the Prime Minister. So NITI Aayog does not have a
fix. At present, it includes the CEC and two other
full-time Chairman.
Election Commissioners.
Option d is correct: NITI Aayog has been constituted to
• At the state level, the Chief Electoral Officer, an UPSC
actualise the important goal of cooperative federalism and
CSE rank Officer, assists the Election Commission.
to enable good governance in India.
50. Solution: (d)
47. Solution: (a)
Exp) Option d is the correct answer
Exp) Option a is the correct answer.
The Universal Declaration of Human Rights (UDHR)
Assertion (A) is true- Proportional representation
was adopted by the UN General Assembly in Paris on 10
guarantees the representation of diverse opinions in the
December 1948 during its 183rd plenary meeting. The
legislature, including minority perspectives. This is possible
UDHR sets out the fundamental human rights and freedoms
because even if a minority party doesn’t secure a majority
of votes, it can still win a few seats in the legislature. This that everyone is entitled to, regardless of their race, sex,
gives the minority party a voice in the legislature and helps nationality, ethnicity, language, religion, or any other status.
to ensure that their interests are represented. 51. Solution: (d)
Reason (R) is true- The system of proportional representation
Exp) Option d is the correct answer
enables due representation to all types of groups based on
ethnicity, gender, interests, and ideologies. This is because Article 315(4) of the Indian Constitution allows the Union
the number of seats a party wins is proportional to the Public Service Commission (UPSC) to serve a state’s needs
number of votes it receives. This means that even if a small upon the Governor’s request with the President’s approval.
group of people have a particular interest, they can still be The UPSC can conduct exams or advise the state government
represented in the legislature if they vote for a party that on recruitment and promotions, but it’s not obliged to do
represents their interests. so. The President approves the request if it aligns with the
public interest.
48. Solution: (d)
Exp) Option d is the correct answer. 52. Solution: (b)

Proportional representation is a voting system where a Exp) Option b is the correct answer.
party’s seat allocation is directly proportional to the votes Assertion (A) is correct: The duty of CAG is not merely to
it garners. Proportional representation helps to ensure that ensure the legality of expenditure but also its propriety.

PYQ Workbook 290


INDIAN POLITY AND GOVERNANCE

This means that he/she has to examine whether the 56. Solution: (d)
expenditure is in accordance with the rules and regulations, Exp) Option d is the correct answer
as well as whether it is prudent, economical, and efficient.
The SHRC does not have the power to punish for the
Reason (R) is correct: He/she has to uphold the Constitution violation of human rights. This power is vested with the
and the laws of Parliament in the field of financial courts. The SHRC can only recommend measures to the
administration. This means that he/she has to ensure that state government to improve the protection of human rights
the executive (i.e., the council of ministers) is accountable to in the state.
Parliament in the area of financial administration. The CAG’s
57. Solution: (d)
role is not limited to verifying the legality of expenditures but
also extends to upholding the constitutional and legislative Exp) Option d is the correct answer
provisions related to financial administration. The State Public Service Commission is not consulted on
making transfers of Civil Servants because transfers are
53. Solution: (c) considered to be administrative decisions falling under
Exp) Option c is the correct answer the purview of the executive branch of the government.
The State Public Service Commission’s role is primarily
The Central Vigilance Commission (CVC) was established
limited to matters related to recruitment, appointments,
in 1964 on the recommendations of the Santhanam
promotions, and disciplinary matters.
Committee, which was set up in 1962 to review the working
of the central government’s anti-corruption machinery. It 58. Solution: (c)
had suggested a high-powered central agency to investigate Exp) Option c is the correct answer.
corruption cases involving government employees and
The CAG can be removed from his/her office in like manner
recommend disciplinary action against corrupt officials.
as a judge of the Supreme Court, who can also be removed
Important Tips by the President only on an address from both Houses of
Parliament on the grounds of proved misbehaviour or
The Gorwala Committee (1951) recommendations to
incapacity under Article 124 (4) of the Constitution.
enhance India’s public administration:
• The committee proposed changes in the recruitment 59. Solution: (d)
process for administrators to ensure efficiency and Exp) Option d is the correct answer
competence in the civil services. The Finance Commission consists of a chairman and four
• Emphasis was placed on improving training to equip other members to be appointed by the president. Hence, there
administrators with necessary skills for effective are total 5 members in commission. According to Article
governance. 280 (2) of the constitution, Parliament may by law determine
the qualifications which shall be requisite for appointment as
• Mechanisms were suggested to strengthen the
members of the Commission and the manner in which they
connection between administrators and legislative
shall be selected. Hence, it is not necessary that at least one
bodies for improved coordination and governance. member of the Commission must be a Judge of the High
• Recommendations aimed at establishing a framework Court or Supreme Court. As per Section 6 in The Finance
for administrators to work closely with the Planning Commission (Miscellaneous Provisions) Act, 1951, every
Commission for planned economic development. member of the Commission shall hold office for such period
• Setting up of Organisation & Method (O & M) as may be specified in the order of the President appointing
him, but shall be eligible for reappointment: Provided that
divisions at various government levels was suggested
he may, by letter addressed to the President, resign his office.
to enhance organizational efficiency.
The First Chairman of the Commission was K. C. Neogy
54. Solution: (a) and Fifteenth Chairman is N. K. Singh.
Exp) Option a is the correct answer 60. Solution: (c)
Administrative Reforms Commission of India (1966-70) Exp) Option c is the correct answer.
was headed by Morarji Desai, and it submitted a report that Statement I is incorrect- Out of the 543 elected seats in the
recommended the establishment of Lokpal and Lokayukta Lok Sabha, 84 are reserved for Scheduled Castes and 47
at the central and state levels. The commission argued that are reserved for Scheduled Tribes. The allocation of seats
these institutions would help to improve the standards of to them is determined based on their percentage of the total
public administration and to prevent corruption. population in the country. This ensures that the reservation
system does not exclude representation for other social
55. Solution: (a)
groups in the country.
Exp) Option a is the correct answer
Statement II is correct- The Delimitation Commission
According to Article 316(1) of the Indian Constitution, is responsible for deciding which constituencies are to be
the Chairman and members of the Union or Joint Public reserved for Scheduled Castes (SCs) and Scheduled Tribes
Service Commission are appointed by the President. (STs).

291 PYQ Workbook


INDIAN POLITY AND GOVERNANCE

Statement III is incorrect- The Delimitation Commission • Prosecution of Offenders: Ensure swift prosecution of
is appointed by the President of India and operates in
individuals involved in criminal activities, especially
coordination with the Election Commission of India.
those connected to politicians and government
Important Tips officials.
In India, Delimitation Commissions have been constituted • Strengthen Criminal Justice System: Enhance legal
on four occasions: in 1952 under the Delimitation provisions to effectively combat economic offenses and
Commission Act, 1952; in 1963 under the Delimitation organized crime.
Commission Act, 1962; in 1973 under the Delimitation Act,
1972; and in 2002 under the Delimitation Act, 2002. The • Focus on Criminal Gangs: Direct law enforcement
Delimitation Commission consists of a retired Supreme to target not only individual crimes but also criminal
Court judge, the Chief Election Commissioner, and the gangs, armed groups, drug mafias, and smuggling
respective State Election Commissioners. The primary networks.
functions of the Delimitation Commission are as follows:
• End Political Patronage: Sever ties between
• Determining the boundaries of territorial politicians and criminal elements, breaking the cycle
constituencies in the country to account for changes
of criminalization of politics.
in population.
• Identifying seats reserved for Scheduled Castes and 64. Solution: (d)
Scheduled Tribes in areas where their population is
Exp) Option d is the correct answer.
relatively high.
The CAG submits his/her reports relating to the accounts
61. Solution: (c) of the Union to the President, who shall cause them to be
Exp) Option c is the correct answer. laid before each House of Parliament under Article 151 (1) of
Article 326 of the Constitution of India recognizes adult the Constitution. Similarly, the CAG submits his/her reports
suffrage. It states that every citizen of India who is 18 years relating to the accounts of a State to the Governor, who shall
of age or above on the qualifying date is entitled to be cause them to be laid before the Legislature of the State
registered as a voter at any election to the Lok Sabha and under Article 151 (2) of the Constitution.
the State Legislative Assemblies.
Important Tips
Important Tips
• The Comptroller and Auditor General of India (CAG)
Article 324- Superintendence, direction and control of
is the supreme audit authority of the country and
elections to be vested in an Election Commission.
he/she audits the accounts of the Union and State
Article 325- No person to be ineligible for inclusion in, governments.
or to claim to be included in a special, electoral roll on
• He/she is appointed by the President of India under
grounds of religion, race, caste or sex.
Article 148 of the Constitution and holds office for a
Article 327- Power of Parliament to make provision with term of six years or until he/she attains the age of 65
respect to elections to legislatures. years, whichever is earlier.
62. Solution: (c) • He/she can be removed by the President only on an
Exp) Option c is the correct answer address from both Houses of Parliament on the grounds
of proved misbehaviour or incapacity
The doctrine of pleasure in Article 310 of the Indian
Constitution allows the government to terminate the 65. Solution: (a)
employment of public servants at will. It grants the
Exp) Option a is the correct answer
government the right to appoint and dismiss public servants.
According to Article 316(1) of the Indian Constitution, the
63. Solution: (c)
Chairman and other members of a State Public Service
Exp) Option c is the correct answer Commission are appointed by the Governor of the
The Vohra (Committee) Report was submitted by the former respective state.
Indian Home Secretary, N. N. Vohra, in October 1993. It
studied the problem of the criminalization of politics and 66. Solution: (b)
of the nexus among criminals, politicians and bureaucrats Exp) Option b is the correct answer.
in India.
The power to grant or refuse leave to the CAG and to
Important Tip revoke or curtail leave granted to him/her, shall vest
Key Recommendations of Vohra Committee: in the President, as per Section 5 (1) of the Comptroller
• Establishment of a Nodal Point: Create a central hub and Auditor General’s (Duties, Powers and Conditions of
for intelligence and enforcement agencies to promptly Service) Act, 1971.
share crime syndicate-related information, improving CAG is appointed by the President of India under Article 148
coordination. of the Constitution and holds office for a term of six years or

PYQ Workbook 292


INDIAN POLITY AND GOVERNANCE

until he/she attains the age of 65 years, whichever is earlier. 70. Solution: (b)
He/she can be removed by the President only on an address Exp) Option b is the correct answer
from both Houses of Parliament on the grounds of proved
The Chairman of the Council of States is not a member
misbehavior or incapacity. of the committee. Section 2, 3 and 4 of The Protection of
67. Solution: (b) Human Rights Act, 1993 lay down the rules for appointment
to the NHRC. The Chairperson and members of the
Exp) Option b is the correct answer.
NHRC are appointed by the President of India, on the
Statement 1 is correct: It is the duty of the Comptroller and recommendation of a committee consisting of:
Auditor-General (CAG) to audit all receipts that are payable
• The Prime Minister (Chairperson)
into the Consolidated Fund of India. The CAG ensures that
the revenue received by the government is properly audited • The Home Minister
and accounted for. • The Leader of the Opposition in the Lok Sabha (Lower
Statement 2 is correct: The CAG has the authority to audit House)
and report on the accounts of stores or stock kept in any • The Leader of the Opposition in the Rajya Sabha (Upper
office or department of the Union or a State. This authority House)
allows the CAG to examine the management and utilization • The Speaker of the Lok Sabha (Lower House)
of stores or stock maintained by government offices or
• The Deputy Chairman of the Rajya Sabha (Upper House)
departments.
Important Tips
68. Solution: (d)
The Protection of Human Rights (Amendment) Act,
Exp) Option d is the correct answer 2019
Governor is not member of the committee to recommend • NHRC Composition: The Act allowed a former Chief
name for the appointment of Chairperson and members of Justice or Judge of the Supreme Court to become
State Human Rights Commission. Instead, the Chairperson NHRC’s chairperson. The number of members
and members of the SHRC are appointed by the Governor, increased from two to three, with at least one member
on the recommendation of a committee consisting of: being a woman. Chairpersons of commissions for
• The Chief Minister (chairperson) Backward Classes, Child Rights, and Disabilities would
• The Home Minister also join NHRC as members.

• The Leader of the Opposition in the legislative council • SHRC Chairperson: The Bill allowed a former Chief
Justice or Judge of a High Court to become the
• The Leader of the Opposition in the legislative assembly chairperson of SHRC.
• The Speaker of the legislative assembly • Term of Office: The term of office for NHRC and
• The Chairman of the legislative council SHRC chairpersons and members was reduced to
three years or until seventy years of age. The limit on
69. Solution: (d)
reappointment for members was removed.
Exp) Option d is the correct answer.
• Powers of Secretary-General: The Secretary-General
Article 149 of the Indian Constitution prescribes the duties and Secretary of NHRC and SHRCs received broader
and powers of the Comptroller and Auditor General administrative and financial powers, excluding judicial
(CAG) of India. This article outlines the role, functions, and functions, under the respective chairperson’s control.
responsibilities of the CAG in auditing and reporting on the • Union Territories: The central government could
accounts of the Union and the States. It provides the CAG assign human rights functions of Union Territories to
with the authority to examine and audit the accounts related SHRCs, while Delhi’s human rights functions would
to the receipt and expenditure of public funds. remain under NHRC’s jurisdiction.
Important Tips 71. Solution: (a)
Article 146 deals with the officers and servants and the Exp) Option a is the correct answer
expenses of the Supreme Court. It states that the Chief
Governor of the state appoints the chairperson and other
Justice of India can appoint officers and servants of the
members of the State Human Rights Commission.
Supreme Court in consultation with the President.
Article 147 covers the interpretation of the constitution Important Tips
along with that of the government of India act, 1935. State Human Rights Commission:
Article 148 states that there shall be a CAG who shall be • Composition: The Commission is made up of three
appointed by the President by warrant under his hand and members: a chairperson and two members.
seal and shall only be removed from office in like manner • Chairperson: The chairperson shall be a retired Chief
and on like grounds as a Judge of the Supreme Court. Justice of a High Court or a High Court Judge

293 PYQ Workbook


INDIAN POLITY AND GOVERNANCE

• Members: Members should be a serving or retired High Section 175, Section 178, Section 179, Section 180 or Section
Court judge or a District Judge. With a minimum of 228 of the Indian Penal Code, the Human Rights Commission
seven year experience as a District Judge, as well as is to be deemed a civil court. This means that the Human
someone with knowledge or practical experience in Rights Commission has the same powers as a civil court in
human rights. respect of these offences. For example, the Human Rights
• Tenure: The chairperson and members are elected for Commission can issue summons, compel the attendance of
a three-year term or until they reach the age of 70, witnesses, and take evidence on oath.
whichever comes first. Important Tips
• Removal: The governor appoints the chairperson Sections of IPC mentioned in the question:
and members of a State Human Rights Commission,
• Section 175:- Omission to produce document to public
however, they can only be removed by the President
servant by person legally bound to produce it.
72. Solution: (b) • Section 178:- Refusing oath or affirmation when duly
Exp) Option b is the correct answer required by public servant to make it.
The Chairperson of the Law Commission of India is not • Section 179:- Refusing to answer public servant
the ex-officio member of the National Human Rights authorized to question.
Commission. • Section 180:- Refusing to sign statement.
Ex-officio members of NHRC: • Section 228:- Intentional insult or interruption to
• Chairperson, National Commission for Scheduled Castes public servant sitting in judicial proceeding.
• Chairperson, National Commission for Scheduled Tribes 77. Solution: (c)
• Chairperson, National Commission for Minorities Exp) Option c is the correct answer
• Chairperson, National Commission for Women
The Protection of Human Rights Act, 1993 (PHRA) provides
• Chairperson, National Commission for Backward Classes that while dealing with complaints of violation of Human
• Chairperson National Commission for Protection of Rights by the members of the armed forces, the Commission
Child Rights either on its own motion or on receipt of a petition, shall
• Chief Commissioner for Persons with Disabilities seek a report from the Central Government. The Central
Government shall, within a period of three months, or
73. Solution: (b)
such longer period as may be specified by the Commission,
Exp) Option b is the correct answer submit a report to the Commission. The Commission shall,
A Sitting Judge of the High Court or a Sitting District Judge after considering the report of the Central Government,
can be appointed as a member of the State Human Rights take such action as it deems fit.
Commission after consultation with the Chief Justice of
the High Court of the concerned State. 78. Solution: (d)
Exp) Option d is the correct answer
74. Solution: (b)
The objective of the Protection of Human Rights Act, 1993,
Exp) Option b is the correct answer
was to achieve all of the following:
The State Human Rights Commission (SHRC) submits its
annual or special reports to the state government. These • The primary aim of the Act was to ensure improved
reports are laid before the state legislature, along with a protection and promotion of human rights in India.
memorandum of action taken on the recommendations of • The Act established the National Human Rights
the Commission and the reasons for nonacceptance of any of Commission (NHRC) at the national level to inquire into
such recommendations. human rights violations and recommend measures for
their prevention and redressal.
75. Solution: (b)
Exp) Option b is the correct answer • Apart from the NHRC, the Act also allowed for the
creation of State Human Rights Commissions (SHRCs)
Section 39 of the Protection of Human Rights Act, 1993 says
in each state to address human rights issues at the state
that Every Member of the Commission, State Commission
level.
and every officer appointed or authorized by the Commission
or the State Commission to exercise functions under this Act Important Tips
shall be deemed to be a public servant.
key provisions of the Protection of Human Rights Act,
76. Solution: (b) 1993:
Exp) Option b is the correct answer • The establishment of the NHRC and SHRCs
As per the section 13 (4) of the national human rights • The investigation of allegations of human rights
protection act, 1993, In relation to offences described under violations

PYQ Workbook 294


INDIAN POLITY AND GOVERNANCE

• The taking of appropriate action against those • It consists of only two articles: article 323 A dealing
responsible for human rights violations with administrative tribunals and article 323 B
• The establishment of Human Rights Courts dealing with tribunals for other matters.

• The training of law enforcement officials on human • Article 323 A empowers the parliament to establish
the administrative tribunals in 1985.
rights
• This article empowers the Central government to
• The promotion of human rights education
establish one central administrative tribunal and the
79. Solution: (c) state administrative tribunals.
Exp) Option c is the correct answer. • Article 323 B empowers the parliament and state
legislature to establish tribunal for different disputes
National Voters Day is celebrated every year on 25 January
in India to mark the foundation of the Election Commission 83. Solution: (a)
of India. The Election Commission of India was established Exp) Option c is the correct answer.
on 25 January 1950.
• Established in: 1948
Important Tip • Used for: immediate relief to families of those killed in
The main objective of celebrating National Voters Day is to natural calamities like floods, cyclones and earthquakes,
encourage people to participate in the electoral process and etc. and to the victims of the major accidents and riots
to make them aware of their rights and responsibilities • Headed by: Prime Minister
as voters. On this day, various awareness programmes • Administration: Administered on an Honorary basis by
are organized by the Election Commission of India and Joint Secretary to the Prime Minister as Secretary of the
other organizations to promote voter education and fund.
participation. • PMNRF operates from the Prime Minister’s Office
(PMO) (option a is correct)
80. Solution: (b)
• Features:
Exp) Option b is the correct answer
(a) Consists entirely of public contributions and does not get
The Telecom Regulatory Authority of India (TRAI)
any budgetary support.
is responsible for the implementation of the
(b) Recognized as a Trust under the Income Tax Act
Government of India’s policy in all matters concerning
telecommunications in India. TRAI is an independent (c) Exempt under the Income Tax Act, 1961 under Section
regulatory body established under the Telecom Regulatory 10 and 139 for return purposes.
Authority of India Act, 1997. It functions as the regulator for 84. Solution: (d)
the telecommunications sector in India and has the authority
Exp) Option d is the correct answer.
to make regulations, issue guidelines, and ensure compliance
with the government’s policies in the telecom sector. The Commission shall, perform all or any of the following
functions, namely:-
81. Solution: (b) • Inquire at its own initiative on the violation of human
Exp) Option b is the correct answer rights
Article 338A of the Indian Constitution deals specifically • Inquiry on a petition presented to it by a victim
with the establishment of the National Commission for • Visit to jails to study the condition of the inmates
Scheduled Tribes (NCST). This article was inserted into • Undertaking and promoting research in the field of
the Constitution through the 89th Amendment Act of 2003. human rights
The NCST is a constitutional body tasked with safeguarding • encourage the efforts of NGOs and institutions working
the rights and interests of the Scheduled Tribes in India. in the field of human rights
82. Solution: (c) • review the factors, including acts of terrorism that inhibit
the enjoyment of human rights
Exp) Option c is the correct answer.
• such other functions as it may consider necessary for the
Tribunals established by a law of the Parliament can exclude
promotion of human rights
the jurisdiction of all courts (except the jurisdiction of
Supreme Court under Article 136) to allow to special leave Important Tips
to appeal. • It is formed as per the statutory provisions of
Protection of Human Rights Act, 1993.
Important Tips
• It is a multi-member body consist of Chairman and
• The dictionary meaning of the word ‘Tribunal’ is seat four members. President appoints the Chairman
of the Judge. and members of the NHRC on the recommendation
• The 42nd amendment act of 1976 added a new part of the high-powered committee headed by the Prime
XIV-A entitled as Tribunals. Minister.

295 PYQ Workbook


INDIAN POLITY AND GOVERNANCE

85. Solution: (c) Aayog comprises the Hon’ble Prime Minister of India;
Exp) Option c is the correct answer. Chief Ministers of all the States and Union Territories with
legislature; Lt Governors of other UTs; Ex-Officio Members;
• The National Commission for Women was set up as
Vice Chairman, NITI Aayog; Full-Time Members, NITI
statutory body in January 1992 under
Aayog; and Special Invitees.
• the National Commission for Women Act, 1990.
• The First Commission was constituted on 31st January 89. Solution: (e)
1992 with Mrs. Jayanti Patnaik as the Chairperson. Exp) Option e is the correct answer.
• The Functions of National commission for women: The Constitution of India does not provide for simultaneous
a) It reviews the Constitutional and legal safeguards for elections to the Lok Sabha and the state legislative assemblies.
women; Therefore, if ‘One Nation One Election’ is to be realized in
b) Recommend remedial legislative measures, India, then Articles 83, 172, 356, and 246 would require an
amendment.
c) Facilitate redressal of grievances and advise the
Government on all policy matters affecting women. Important Tips

86. Solution: (b) Article 83: This Article specifies the term of the Lok
Sabha. The term of the Lok Sabha is currently five years. If
Exp) Option b is the correct answer.
‘One Nation One Election’ is to be realized, then the term
• The commission presents an annual report to the of the Lok Sabha may have to be changed.
President of India.
Article 172: This Article specifies the term of the state
• The president places all such reports before the legislative assemblies. The term of the state legislative
Parliament, along with a explaining the action taken on assemblies is currently five years. If ‘One Nation One
the recommendations made by the commission. Election’ is to be realized, then the term of the state
• The president also forwards any report of the commission legislative assemblies may have to be changed.
pertaining to a state government to the state governor Article 356: This Article empowers the President of India
and the governor turns places it before state legislature to declare President’s Rule in a state. If ‘One Nation One
Election’ is to be realized, then the provisions of Article
Important Tip
356 may have to be amended to ensure that President’s Rule
• National Commission for scheduled Tribes is an Indian cannot be imposed in a state during the period leading up
constitutional body, established through constitution to or during the simultaneous elections.
act, 2003.
Article 246: This Article specifies the legislative powers
• Its duty is to investigate and monitor all matters of the Union and the states. If ‘One Nation One Election’
relating to the safeguards provided for the STs. is to be realized, then the provisions of Article 246 may
• The commission require to include at least one woman have to be amended to ensure that the Union and the states
member. have concurrent powers to legislate on matters relating to
elections.
87. Solution: (c)
Exp) Option c is the correct answer 90. Solution: (c)
The chairman of a finance commission is selected from Exp) Option c is the correct answer
people with experience of public affairs. The other four The finance commission is a constitutional body mandated
members are selected from people who: by article 280 of the constitution. The President of India
• Are, or have been, or are qualified, as judges of a high constitutes the Finance Commission every 5 years. The
court, recommendations made by the Finance Commission are of
• Have knowledge of government finances or accounts, an advisory nature only and therefore, not binding upon the
or government. The first Finance Commission was constituted
• Have had experience in administration and financial in 1951.
expertise; or Important Tips
• Have special knowledge of economics • The eligibility of members of the Finance Commission
88. Solution: (b) is determined by the Parliament of India.

Exp) Option b is the correct answer • Composition of Finance Commission: Chairman and
4 members.
The President of India is not a member of the Governing
Council of NITI Aayog. The Governing Council of NITI • It submits its report to the President of India

PYQ Workbook 296


INDIAN POLITY AND GOVERNANCE

INDIAN POLITY AND GOVERNANCE


MISCELLANEOUS
*This unit consists of questions from General and State Elections, Political Parties and other
Constitutional dimensions.

9.1. UPSC CSE Previous Years’ Questions 3. Consider the following statements in respect
of the National Flag of India according to
1. With reference to Home Guards, consider the Flag Code of India, 2002:
the following statements: Statement-I:
1. Home Guards are raised under the Home One of the standard sizes of the National Flag
Guards Act and Rules of the Central of India is 600 mm x 400 mm.
Government. Statement-II:
2. The role of the Home Guards is to serve The ratio of the length to the height (width)
as an auxiliary force to the police in of the Flag shall be 3:2.
maintenance of internal security.
Which one of the following is correct in
3. To prevent infiltration on the international respect of the above statements?
border/coastal areas, the Border Wing [UPSC CSE Pre 2023]
Home Guards Battalions have been raised
(a) Both Statement-I and Statement-II are
in some States. correct and Statement-II is the correct
How many of the above statements are explanation for Statement-l
correct? [UPSC CSE Pre 2023] (b) Both Statement-I and Statement-II are
(a) Only one correct and Statement-II is not the correct
(b) Only two explanation for Statement-I
(c) All three (c) Statement-l is correct but Statement-II is
(d) None incorrect is incorrect
(d) Statement-l is incorrect Statement-II is
2. With reference to India, consider the correct
following pairs:
4. With reference to anti-defection law in
Action The Act India, consider the following statements:
under which [UPSC CSE Pre. 2022]
it is covered 1. The law specifies that a nominated
1. Unauthorized wearing The Official legislator cannot join any political party
of police or military Secrets Act, within six months of being appointed to
uniforms 1923 the House.
2. Knowingly misleading: The Indian 2. The law does not provide any time-frame
or otherwise interfering Evidence Act, within which the presiding officer has to
with a police officer or 1872 decide a defection case.
military officer when Which of the statements given above is/are
engaged in their duties correct?
3. Celebratory gunfire The Arms (a) 1 only
which can endanger (Amendment) (b) 2 only
the personal safety of Act, 2019 (c) Both 1 and 2
others (d) Neither 1 nor 2
How many of the above pairs are correctly 5. With reference to the “Tea Board” in India,
matched? [UPSC CSE Pre 2023] consider the following statements:
(a) Only one 1. The Tea Board is a statutory body.
(b) Only two 2. It is a regulatory body attached to the
(c) All three Ministry of Agriculture and Farmers
(d) None Welfare.

297 PYQ Workbook


INDIAN POLITY AND GOVERNANCE

3. The Tea Board’s Head Office is situated in 3. Aadhaar is mandatory for obtaining
Bengaluru. insurance products.
4. The Board has overseas office at Dubai 4. Aadhaar is mandatory for getting benefits
and Moscow. funded out of the Consolidated Fund of
Which of the statements given above are India.
correct? [UPSC CSE Pre 2022] Which of the statements given above is/are
(a) 1 and 3 correct?
(b) 2 and 4 (a) 1 and 4 only
(c) 3 and 4 (b) 2 and 4 only
(d) 1 and 4 (c) 3 only
(d) 1, 2 and 3 only
6. Consider the following statements:
9. Along with the Budget, the Finance Minister
1. ‘’Right to the city’’ is an agreed human
also places other documents before the
right and the UN-Habitat monitors the Parliament which include “The Macro
Commitments made by each country in Economic Framework Statement’. The
this regard. aforesaid document is presented because
2. “Right to the City” gives every occupant of this is mandated by [UPSC CSE Pre. 2020]
the city the right to reclaim public spaces (a) Long standing parliamentary convention
and public participation in the city. (b) Article 112 and Article 110(1) of the
3. “Right to the City” means that the State Constitution of India
cannot deny any public service or facility (c) Article 113 of the Constitution of India
to the unauthorized colonies in the city. (d) Provisions of the Fiscal Responsibility
Which of the statements given above is/are and Budget Management Act, 2003
correct? [UPSC CSE Pre 2021] 10. One common agreement between Gandhism
(a) 1 only and Marxism is: [UPSC CSE Pre. 2020]
(b) 3 only (a) the final goal of a stateless society
(c) 1 and 2 (b) class struggle
(d) 2 and 3 (c) abolition of private property
7. Consider the following statements in (d) economic determinism
respect of Bharat Ratna and Padma Awards: 11. Consider the following statements:
[UPSC CSE Pre. 2021] [UPSC CSE Pre. 2019]
1. Bharat Ratna and Padma awards are 1. The Parliament (Prevention of
titles under the Article 18 (1) of the Disqualification) Act, 1959 exempts
Constitution of India. several posts from disqualification on the
2. Padma Awards, which were instituted in grounds of ‘Office of Profit’.
the year 1954, were suspended only once. 2. The above-mentioned Act was amended
3. The number of Bharat Ratna Awards five times.
is restricted to a maximum of five in a 3. The term ‘Office of Profit’ is well-defined
particular year. in the Constitution of India.
Which of the above statements are not Which of the statements given above is/are
correct? correct?
(a) 1 and 2 only (a) 1 and 2 only
(b) 3 only
(b) 2 and 3 only
(c) 2 and 3 only
(c) 1 and 3 only
(d) 1, 2 and 3
(d) 1, 2 and 3
12. For election to the Lok Sabha, a nomination
8. Consider the following statements: paper can be filed by: [UPSC CSE Pre. 2017]
[UPSC CSE Pre. 2020] (a) anyone residing in India.
1. Aadhaar metadata cannot be stored for (b) a resident of the constituency from which
more than three months. the election is to be contested.
2. State cannot enter into any contract (c) any citizen of India whose name appears
with private corporations for sharing of in the electoral roll of a constituency.
Aadhaar data. (d) any citizen of India.

PYQ Workbook 298


INDIAN POLITY AND GOVERNANCE

13. Consider the following statements: 16. With reference to the Delimitation
[UPSC CSE Pre. 2017] Commission, consider the following
1. The Election Commission of India is a statements: [UPSC CSE Pre. 2012]
five-member body. 1. The orders of the Delimitation
2. Union Ministry of Home Affairs decides Commission cannot be challenged in a
the election schedule for the conduct of Court of Law.
both general elections and bye-elections. 2. When the orders of the Delimitation
3. Election Commission resolves the Commission are laid before the Lok Sabha
or State Legislative Assembly, they cannot
disputes relating to splits/mergers of
effect any modifications in the orders.
recognized political parties.
Which of the statements given above is/are
Which of the statements given above is/are
correct?
correct?
(a) 1 only
(a) 1 and 2 only (b) 2 only
(b) 2 only (c) Both 1 and 2
(c) 2 and 3 only (d) Neither 1 nor 2
(d) 3 only
17. Consider the following statements:
14. The National Green Tribunal Act, 2010 was [UPSC CSE Pre. 2011]
enacted in consonance with which of the
1. Central Administrative Tribunal (CAT)
following provisions of the Constitution of
was set up during the Prime Ministership
India? [UPSC CSE Pre. 2012] of Lal Bahadur Shastri.
1. Right to healthy environment, construed 2. The Members of CAT are drawn from
as a part of Right to life under Article 21 both judicial and administrative streams.
2. Provision of grants for raising the level
Which of the statements given above is/are
of administration in the Scheduled Areas correct?
for the welfare of Scheduled Tribes under
(a) 1 only
Article 275(1)
(b) 2 only
3. Powers and functions of Gram Sabha as (c) Both 1 and 2
mentioned under Article 243(A) (d) Neither 1 nor 2
Select the correct answer using the codes
18. Consider the following statements:
given below:
1. The Commonwealth has no charter,
(a) 1 only
treaty or constitution.
(b) 2 and 3 only
2. All the territories/countries once under
(c) 1 and 3 only
the British Empire (jurisdiction/rule/
(d) 1, 2 and 3
mandate) automatically joined the
15. Consider the following statements: Commonwealth as its members.
[UPSC CSE Pre. 2012] Which of the statements given above is/are
1. Union Territories are not represented in correct? [UPSC CSE Pre 2010]
the Rajya Sabha. (a) 1 only
2. It is within the purview of the Chief (b) 2 only
Election Commissioner to adjudicate the (c) Both 1 and 2
election disputes. (d) Neither 1 nor 2
3. According to the Constitution of India, 19. Which Article of the Constitution provides
the Parliament consists of the Lok Sabha that every state shall endeavour to provide an
and the Rajya Sabha only. adequate facility for instruction in mother-
Which of the statements given above is/are tongue at primary stage of education?
correct? [UPSC CSE Pre 2001]
(a) 1 only (a) Article 349
(b) 2 and 3 only (b) Article 350
(c) 3 only (c) Article 350-A
(d) None (d) Article 351

299 PYQ Workbook


INDIAN POLITY AND GOVERNANCE

20. Given below are two statements, one 23. Given below are two statements, one
labelled as Assertion (A) and the other labelled as Assertion (A) and the other
labelled as Reason (R): labelled as Reason (R):
Assertion (A): The reservation of thirty-three Assertion (A): In India, the political parties
percent of seats for women in Parliament which formed the governments represented
and State Legislature does not require a the majority of seats secured in the elections
Constitutional amendment. to the House of the People at the Centre and
the Legislative Assemblies in the States but
Reason (R): Political parties contesting not the majority of votes.
elections can allocate thirty-three percent
of seats they contest to women candidates Reason (R): The result of elections based on
the majority-vote-system is decided on the
without any Constitutional amendment.
basis of the relative majority of votes secured.
In the context of the above two statements In the context of the above two statements
which one of the following is correct? which one of the following is correct?
[UPSC CSE Pre 1997] [UPSC CSE Pre 1994]
(a) Both (A) and (R) are true, and (R) is the (a) Both (A) and (R) are true, and (R) is the
correct explanation of (A). correct explanation of (A).
(b) Both (A) and (R) are true, but (R) is not a (b) Both (A) and (R) are true, but (R) is not a
correct explanation of (A). correct explanation of (A).
(c) (A) is true but, (R) is false. (c) (A) is true but, (R) is false.
(d) (A) is false hut, (R) is true. (d) (A) is false but, (R) is true.
21. Which one of the following is not a principle 24. According to the United Nations Charter
of Panchsheel? [UPSC CSE Pre 1997] which of the following statements is not
(a) Non-alignment true regarding exercise of veto power in the
(b) Peaceful Co-existence Security Council? [UPSC CSE Pre 1994]
(c) Mutual respect for each other’s territorial (a) For the Security Council decision on
integrity and sovereignty every procedural matter there should
(d) Mutual non-interference in each other’s be 9 affirmative votes of its members
internal affairs including the 5 votes of five permanent
members.
22. Match list-I with list-II and select the (b) Every permanent member of the Security
correct answer using the codes given below Council may by use of Veto Power,
the lists: withhold acceptance on any decision.
(c) The word ‘Veto’ has been used under
List-I List-II (Oaths of Article 27 of the UN Charter so that any
(Functionaries) affirmations) permanent member may stop a resolution
A. President of 1. Secrecy of going to be passed with a majority.
India Information (d) Any member of the Security Council may
B. Judges of 2. Faithful discharge of use Veto Power to stop a resolution going
Supreme Duties to be passed with a majority.
Court
9.2. Other Examination Previous Years’
C. Members of 3. Faith and Allegiance Questions
Parliament to the Constitution
of India 25. With reference to the National Calendar,
which of the statements is/are correct?
D. Ministers for 4. Upholding the 1. The National Calendar of India is based
the Union sanctity of the on Vikram Era.
Constitution and
2. The National Calendar is adopted from
Law
January 26, 1950.
Code: [UPSC CSE Pre 1997] Select the correct answer using the codes
A B C D given below: [U.P.P.C.S. (Pre) 2019]
(a) 3 4 1 2 (a) 1 only
(b) 4 3 2 1 (b) 2 only
(c) 3 4 2 1 (c) Both 1 and 2
(d) 4 3 1 2 (d) Neither 1 nor 2

PYQ Workbook 300


INDIAN POLITY AND GOVERNANCE

26. Which one of the following is correct (c) 2 and 3


about the languages mentioned in the (d) 2 only
Constitution of India?
31. Which is the correct chronological order of
[U.P.P.C.S. (Mains) 2016] the following?
(a) State can compel the linguistic minorities 1. Shimla Agreement
to choose their mother tongue as a
2. Indo-Soviet Treaty
medium of instruction in primary
schools. 3. Farakka Accord
(b) ‘Angika’ is now included in the 8th 4. Tashkent Agreement
Schedule of the Constitution. Select the correct answer using the code given
(c) Hindi is National language of India. below: [U.P.P.C.S. (Mains) 2002]
(d) Bodo is a recognized language in the 8th (a) 4, 2, 1, 3
Schedule of the Constitution. (b) 3, 2, 1, 4
27. Study the following statements and select (c) 2, 3, 4, 1
the correct answer from the code given (d) 1, 2, 3, 4
below: 32. Meaning of Extra-Constitutional Right-
Assertion (A): India has a written [U.P. P.C.S. (Pre) 2002]
Constitution. (a) That power which violates the limits of
Reason (R): Growth of strong regional parties Constitution.
is an indicator of regional aspirations. (b) A person who has got exceptional powers
Code: [U.P.P.C.S. (Pre) (Re-Exam) 2015] under the Constitution.
(a) Both (A) and (R) are true, and (R) is the (c) That power which is not in accordance
correct explanation of (A). with the provisions of the Constitution.
(b) Both (A) and (R) are true, but (R) is not (d) That official who has all the powers of
the correct explanation of (A). the State during the proclamation of
(c) (A) is true, but (R) is false. emergency.
(d) (A) is false, but (R) is true. 33. Under Article 344 of Indian Constitution,
28. The Indian Foreign Policy of Non- the first ‘Official Language Commission’
Alignment was initiated by- was established- [U.P.P.C.S. (Pre) 1998]
[U.P.P.C.S. (Mains) 2014] (a) In 1950 under the Chairmanship of K.M.
Munshi
(a) Jawaharlal Nehru
(b) In 1955 Under the Chairmanship of B.G.
(b) Mrs. Indira Gandhi
Kher
(c) Lal Bahadur Shastri
(c) In 1960 under the Chairmanship of M.C.
(d) Morarji Desai
Chagala
29. Which of the following is a reason of (d) In 1965 under the Chairmanship of
emergence of regionalism in India? Humayun Kabir
[U.P.P.C.S. (Spl) (Mains) 2008] 34. Inner-Party Democracy stands for-
(a) Uneven development [U.P. Lower Sub. (Spl) (Pre) 2004]
(b) Fear of losing cultural identity
(c) Political prominence (a) A Government formed by several parties
(d) All of the above with a common programme
(b) Party closest to the party or parties
30. Consider the following statements: forming the Government
[U.P.P.C.S. (Mains) 2004] (c) Periodical elections within the party to
1. The design of the National Flag was elect the office bearers of the party
adopted by the Constituent Assembly of (d) Secrets of Government are shared by
India on 22nd July, 1947 parties in power
2. The wheel in National Flag has 21 spokes 35. The Press Council of India is a-
3. The ratio of the width of the National [U.P. Lower Sub. (Spl) (Pre) 2004]
Flag to its length is 3 : 4 (a) Statutory Body
Which of the above statements is/are correct? (b) Statutory Quasi-judicial Body
(a) 1 and 2 (c) Judicial Body
(b) 1 only (d) Advisory Body

301 PYQ Workbook


INDIAN POLITY AND GOVERNANCE

36. When the Hindi version of National 41. Who was among the following to receive the
Anthem Jana-Gana-Mana was adopted by first Bharat Ratna?
the Constitution Assembly of India? [66th B.P.S.C. (Pre) (Re-Exam), 2020]
[Uttarakhand PCS (Pre) 2021] (a) Rajendra Prasad
(a) 26 November, 1949 (b) Jawahar Lal Nehru
(b) 24 January, 1950 (c) Lal Bahadur Shastri
(c) 17 November, 1947 (d) S. Radhakrishnan
(d) 9 December, 1946 (e) None of the above / More than one of the
above
37. Who was the Prime Minister of India
42. Consider the following statements about
when the basic principle of India’s Nuclear
Electoral Bond Scheme 2018:
Doctrine of ‘No First Use’ was adopted?
(A) The aim of this scheme is to bring about
[Uttarakhand P.C.S. (Pre) 2021] transparency in the funding process of
(a) I.K. Gujaral political parties.
(b) Atal Bihari Vajpayee (B) Only the political parties recognized by
(c) Manmohan Singh the Election Commission which secured
(d) Indira Gandhi not less than two percent of the votes
38. Who recognizes the political parties in polled in the last general election to
India? [Uttarakhand P.C.S. (Pre) 2016] the House of People or the Legislative
Assembly of the State shall be eligible to
(a) President of India
receive the Electoral Bonds.
(b) Election Commission of India
(c) Ministry of Law and Justice (C) Electoral Bonds shall be valid for fifteen
(d) Speaker of Lok Sabha calendar days from the date of issue.
(D) The Electoral Bond deposited by an
39. What does the wheel on the ‘Indian National eligible political party in its account shall
Flag’ symbolize? be credited on the same day.
[Uttarakhand P.C.S. (Pre) 2005] Which of the above statements are correct?
(a) Freedom [R.A.S./R.T.S. (Pre) 2018]
(b) Justice (a) Only (A) and (B)
(c) Equality (b) (A), (B), (C) and (D)
(d) Brotherhood (c) Only (B), (C) and (D)
40. With reference to the UN peacekeeping (d) Only (A), (C) and (D)
operations, consider the following 43. Who has authored ‘Gandhian Constitution
statements: for Free India’? [R.A.S./R.T.S. (Pre) 2018]
1. UN peacekeepers can use force only for (a) Aruna Asaf Ali
their self-defense. (b) Achyut Patwardhan
2. Peacekeeping operations can only be (c) Shriman Narayan Agarwal
withdrawn by a resolution by the UN (d) Humayun Kabir
Security Council. 44. The powers of District Collector as a
3. USA is the largest budget contributor to District Magistrate are:
the UN peacekeeping operations. (A) To maintain law and order
Which of the statements given above is/are (B) Control over police
correct? (C) To check passports of foreigners
[67th B.P.S.C. (Pre) (Re-Exam), 2022] (D) To control land revenue
(a) 1 and 2 only Select the correct answer using the code given
(b) 2 and 3 only below: [R.A.S./R.T.S. (Pre) 2016]
(c) 3 only (a) A, C, D
(d) 1, 2 and 3 (b) A, B, D
(e) None of the above/More than one of the (c) A, B, C
above (d) B, C, D

PYQ Workbook 302


INDIAN POLITY AND GOVERNANCE

45. What is true about gallantry award ‘Param (b) Basis of depriving a person of his/her
Vir Chakra’? personal liberty must be in accordance
[Chhattisgarh P.C.S. (Pre) 2021] with procedures established by law.
(a) This award can be given to an individual (c) Personal liberty can be secured by the
only once in his/her lifetime judicial writ of Habeas Corpus.
(b) On its medal, four replicas of ‘Indra Vajra’ (d) The majority view of the Supreme Court
are embossed in A.K. Gopalan v/s. State of Madras case
(c) Till 31st December 2021, 25 people have invented ‘due process of law’.
been awarded with Param Vir Chakra 52. Which one of the following is NOT true of
(d) This award was instituted on 26th January socialism? [CAPF 2020]
1952 (a) Socialists analyse society in terms of
46. What is not true in reference to Chief of income distribution
Defence Staff? (b) It emphasises the centrality of community
[Chhattisgarh P.C.S. (Pre) 2021] (c) Socialists believe that class divisions
(a) Maximum age limit for this post is 65 in society are a fact and therefore
years irremediable
(b) He is permanent Chairman of Chief of (d) It believes that material benefits should
Staff Committee be distributed on the basis of need
(c) He is Chairman of Defence Acquisition 53. Match List I with List II and select the
Council correct answer using the codes given below
(d) He is member of Defence Planning the lists:
Committee
List I List II
47. Which of the following may be considered a
Pressure Group? [M.P.P.C.S. (Pre) 2017] (Concept) (Explanation/Aim)
(a) Members of the Lok Sabha A. Pluralist 1. Power is always
(b) Members of a Panchayat democracy exercised by a
(c) Members of the Cabinet privileged few
(d) Members of a Trade Union B. People’s 2. Highest and
48. Scientific socialism is attributed to democracy harmonious
[M.P.P.C.S. (Pre) 2016] development
(a) Karl Marx of indi- vidual
(b) Adam Smith capabilities
(c) J. M. Keynes C. Developmental 3. Social equality
(d) Thomas Mann democracy through common
ownership of
49. The state emblem was adopted by Govt. of
wealth
India on [M.P.P.C.S. (Pre) 1999]
th
(a) 15 August, 1948 D. Elitist 4. Capacity of
(b) 2nd October, 1947 democracy groups to ensure
(c) 26th January, 1948 Governmental
(d) 26th January, 1950 responsiveness
Codes [CAPF 2020]
50. A complete or formal rendition of national
anthem takes- [Jharkhand P.C.S. (Pre) 2011] A B C D
(a) 50 seconds (a) 4 3 2 1
(b) 45 seconds (b) 4 2 3 1
(c) 52 seconds (c) 1 2 3 4
(d) 55 seconds (d) 1 3 2 4
51. Which one of the following statements 54. The idea of social justice requires that
about ‘personal liberty’ is not correct? 1. equal treatment be supplemented with
[CAPF 2021] principle of proportionality.
(a) State does not have the authority to 2. due and equal consideration is given to
deprive any person within the Territory of all individuals
India of his/her personal liberty without 3. equality of treatment be transcended to
any rational basis. recognise special needs

303 PYQ Workbook


INDIAN POLITY AND GOVERNANCE

Select the correct answer using the codes (c) 2, 3 and 4 only
given below: [CAPF 2020] (d) 1, 2, 3 and 4
(a) 1, 2 and 3 59. Which one of the following is not a source
(b) 1 and 3 only of political legitimacy? [CAPF 2016]
(c) 2 only
(a) Consent
(d) 2 and 3 only
(b) Rational prudence
55. Which one among the following is NOT the (c) Political will of the ruler
central feature of Polyarchy? [CAPF 2019] (d) Public reason
(a) Government is in the hands of elected 60. Parochial political culture is generally
officials found in [CAPF 2016]
(b) Right to run for office is restricted (a) developing societies
(c) Practically, all adults have the right to (b) societies having multi-party system
vote (c) developed societies
(d) There is free expression and a right to (d) monarchical societies
criticise and protest
61. What is meant by Social Equality?
56. Which of the following are the defining
[CAPF 2016]
features of liberal democracy?
(a) Each person should perform the functions
1. Constitutional government based on
according to his status
formal, usually legal, rules
(b) No effort should be made to modify the
2. Guarantees of civil liberties and individual existing social system
rights (c) No person should be made to suffer a
3. It invests political authority with position of relative inferiority in relation
potentially unlimited power to other men due to caste, race or religion
Select the correct answer using the code given (d) Special effort to improve the lot of weaker
below: [CAPF 2019] section
(a) 1 and 3 only 62. Which one of the following statements
(b) 1 and 2 only regarding Human Rights is not correct?
(c) 2 and 3 only
[CAPF 2016]
(d) 1, 2 and 3
(a) Human Rights derive their inspiration
57. Who among the following leaders is from moral principles
associated with the concept of party less (b) They are applicable subject to their
democracy? [CAPF 2017] adoption by States
(a) JB Kripalani (c) The International Covenant on Civil
(b) Jay Prakash Narayan and Political Rights was adopted by the
(c) Acharya Narendra Dev United Nations in the year 1948
(d) Vinoba Bhave (d) The universalism of Human challenged
58. Which of the following most suitably by the cultural relativists Rights is
captures the meaning and essence of challenged by the cultural relativists
‘globalization”? 63. Who among the following divided
1. It is about conducting external relations governments into ‘Republican’,
between two or more States of the world. ‘Monarchical’ and ‘Despotic”? [CAPF 2016]
2. It is a process of coming together of (a) Aristotle
world-wide social relations bypassing (b) Hobbes
territories. (c) Montesquieu
3. It refers to thinking trans territorially (d) St. Augustine
about global risks affecting people. 64. Which one of the following is under the
4. It has a wishful vision of suprastate control of the Ministry of Defence?
governance [CAPF 2015]
Select the correct answer using the code given (a) Indo-Tibetan Border Police
below: [CAPF 2017] (b) Rashtriya Rifles
(a) 1 and 4 only (c) National Security Guard
(b) 2 and 3 only (d) Sashastra Seema Bal

PYQ Workbook 304


INDIAN POLITY AND GOVERNANCE

65. The provision under Article 350A of the (c) The National Authority Chemical
Constitution of India relates to the Weapons Convention
[CAPF 2015] (d) The Department of Jammu, Kashmir and
(a) right of any section of the citizens to Ladakh Affairs
preserve its distinct language and culture 70. National Disaster Management Authority is
(b) right of the Sikh community to carrying headed by [CDS 2021(I)]
and wearing of kirpans (a) the Prime Minister
(c) provision for providing facilities for (b) the Home Minister
instruction in mother-tongue at primary (c) the President
stage (d) the Health Minister
(d) freedom of Minority-managed educational 71. ‘Legal Positivism’ theory was developed by
institutions from discrimination in the [CDS 2021(I)]
matter of receiving aid from the State
(a) T.H. Green
66. Which of the following committees (b) Thomas Hobbes
submitted reports on Electoral reforms? (c) John Austin
1. Dinesh Goswami Committee (d) Patrick Devlin
2. Tarkunde Committee 72. Who among the following said that
3. Indrajit Gupta Committee Democracy means a system of ‘Government
4. Balwantrai Mehta Committee by Consent’? [CDS 2021(I)]
Select the correct answer using the code given (a) John Locke
(b) J.S. Mill
below. [CAPF 2015]
(c) Jeremy Bentham
(a) 1, 3 and 4 (d) J.J. Rousseau
(b) 2 and 4
(c) 1, 2 and 3 73. Which one of the following is not an Indian
(d) 3 and 4 form of Socialism? - [CDS 2022 (I)]
(a) Democratic Socialism
67. Under Section 10(26) of the Income Tax (b) Radical Socialism
Act, which of the following category of (c) Communitarian Socialism
people are exempted from income tax? (d) Nehruvian Socialism
[CAPF 2015]
74. Which one of the following statements is
(a) Members of the Scheduled Tribes in not correct about “Totalitarianism”?
Chhattisgarh [CDS 2021 (II)]
(b) Members of the Scheduled Tribes and
(a) It is not akin to autocracy and
the Scheduled Castes in Special Category authoritarianism.
States (b) It usurps political freedom of the
(c) Members from the Economically individuals, but it doesn’t usurp personal
Backward Class in All States freedoms.
(d) Members of the Scheduled Tribes in (c) It implies abolition of civil society.
States of Manipur, Mizoram, Nagaland (d) It is usually identified with a one-party
and Tripura state.
68. The major cleavage in global human rights 75. Which of the following statements is
discourse has been between: [CAPF 2014] not correct regarding the Members of
(a) religious rights vs. civil rights Parliament Local Area Development
(b) national rights vs. minority rights Scheme (MPLADS)? - [CDS 2020 (I)]
(c) gender rights vs. equal rights (a) Members of the Parliament (MPs)
(d) civil and political rights vs. economic, sanction, execute and complete works
social and cultural rights under the scheme
(b) Nominated Members of the Parliament
69. Which one among among the following can recommend works for implementation
Departments/Authorities is not under the anywhere in the country
Ministry of Home Affairs? [CDS 2023 (I)] (c) The scheme is fully funded by the
(a) The Department of States Government of India
(b) The Department of Language Official (d) The annual entitlement per MP is 5 crores

305 PYQ Workbook


INDIAN POLITY AND GOVERNANCE

76. Overseas Indians can exercise franchise in (a) The Prime Minister’s Office (PMO)
an election to the Lok Sabha under which of (b) The National Disaster Management
the following conditions? Authority
1. They must be citizens of India. (c) The Ministry of Finance
(d) The National Development Council
2. Their names must figure in the electoral
(NDC)
roll.
3. They must be present in India to vote. 82. Which one of the following is not an
International Human Rights Treaty?
Select the correct answer using the codes
[CDS Pre.2018 I]
given below. [CDS 2020 (I)]
(a) International Covenant on Civil and
(a) 1, 2 and 3 Political Rights
(b) 2 and 3 (b) Convention of the Elimination of All
(c) 1 and 2 Forms of Discrimination against Women
(d) Only 1 (c) Convention on the Rights of Persons with
77. The First Delimitation Commission in Disabilities
India was constituted in- [CDS 2020 (I)] (d) Declaration on the Right to Development
(a) 1949 83. According to the Election Commission
(b) 1950 of India, in order to be recognised as a
(c) 1951 ‘National Party’, a political party must be as
(d) 1952 a recognised as a state party in how many
states? [CDS Pre.2018 I]
78. The formulation of policy in respect to (a) At least two states
Intellectual Property Rights (IPRs) is the (b) At least three states
responsibility of- [CDS 2020 (I)] (c) At least four states
(a) the Ministry of Law and Justice (d) At least five states
(b) the Department of Science and 84. Which one of the following intellectual
Technology property rights is protected without making
(c) the Department for Promotion of any registration? [CDS Pre. 2017(I)]
Industry and Internal Trade (a) Copyright
(d) the Ministry of Human Resource (b) Patent
Development (c) Industrial design
79. Which one of the following is not a (d) Trademark
classified category of political parties as 85. Which among the following Acts were
outlined by the Election Commission of repealed by Article-395 of the Constitution
India? [CDS 2020 (II)] of India?
(a) National Parties 1. The Government of India Act. 1935
(b) State Recognised Parties 2. The Indian Independence Act. 1947
(c) Regional Parties 3. The Abolition of Privy Council
(d) Registered Unrecognised Parties Jurisdiction Act, 1949
4. The Government of India Act. 1919
80. The ‘Gujral Doctrine’ relates to which one
of the following Issues? [CDS Pre. 2019 I] Which of the statement(s) given above is/are
correct? [CDS Pre. 2016 II]
(a) Build trust between India and its
(a) 1 and 2
neighbours (b) 1 and 3
(b) initiate dialogue with all insurgent groups (c) 1, 2 and 3
in India (d) All of these
(c) Undertake development activities in
Naxal-dominated areas 86. Which one of the following statements
(d) Ensure food security about electoral government in India is not
correct? [CDS Pre.2015 II]
81. The Prime Minister’s National Relief Fund (a) The superintendence direction and
is operated by which one of the following control of elections are vested in the
bodies? [CDS Pre. 2019 I] Election Commission of India

PYQ Workbook 306


INDIAN POLITY AND GOVERNANCE

(b) There is one general electoral roll for decolonisation would be able to develop
every territorial constituency a new and more principled approach to
(c) The Parliament has the power to make international relations.
laws relating to the delimitation of
constituencies 3. The first formal codification in treaty
(d) The Supreme Court of India has the form was done in an agreement between
authority to scrutinise the validity of a law China and India.
relating to delimitation of constituencies.
Select the correct answer using the codes
87. Which of the following about the principles given below: [CDS Pre.2014 II]
of Panchsheel are correct?
(a) 1 and 2
1. These are a set of five principles governing
relations between States. (b) 2 and 3
2. The assumption of Panchsheel was (c) 1, 2 and 3
that newly independent States after (d) 1 and 3

307 PYQ Workbook


INDIAN POLITY AND GOVERNANCE

SOLUTIONS

3. Solution: (d)
9.1. UPSC CSE Previous Years’ Questions
Exp) Option d is the correct answer
1. Solution: (b)
The use, display and hoisting of the National Flag in the
Exp) Option b is the correct answer. country is guided by an overarching set of instructions called
‘Home Guards’ is a voluntary force, first raised in India the ‘Flag Code of India 2002’. The code brings together all
in December 1946, to assist the police in controlling civil laws, conventions, practices, and instructions for the display
disturbance and communal riots. of the National Flag.
Statement 1 is incorrect: Home Guards are raised under Statement 1 is incorrect: As per the Flag code of India
the Home Guards Act and Rules of the States/Union 2002, the National Flag cannot be in the dimension of
Territories (not central Government). They are recruited 600mm*400mm. The code states that the National flag can
from amongst all classes of people and walks of life, who give be of nine standard dimensions as follows:
their spare time to the organisation for betterment of the
community. Flag Size No. Dimensions
Statement 2 is correct: The role of Home Guards is to serve as 1 6300 × 4200
an auxiliary Force to the Police in maintenance of internal 2 3600 × 2400
security situations, help the community in any kind of 3 2700 × 1800
emergency such as an air-raid, fire, cyclone, earthquake,
4 1800 × 1200
epidemic etc., help in maintenance of essential services,
promote communal harmony and assist the administration 5 1350 × 900
in protecting weaker sections, participate in socio-economic 6 900 × 600
and welfare activities and perform Civil Defence duties. 7 450 × 300
Statement 3 is correct: Fifteen Border Wing Home Guards 8 225 × 150
(BWHG) Battalions have been raised in the border States
9 150 × 100
viz. Punjab (6 Bns.), Rajasthan ( 4 Bns.), Gujarat (2 Bns.)
and one each Battalion for Meghalaya, Tripura and West Statement 2 is correct: The National Flag shall be rectangular
Bengal to serve as an auxiliary to Border Security Force for in shape and the ratio of the length to the height (width) of
preventing infiltration on the international border/coastal the Flag shall be 3:2. Hence Statement 2 is correct.
areas, guarding of VA/VPs and lines of communication in
4. Solution: (b)
vulnerable area at the time of external aggression.
Exp) Option b is the correct answer.
Therefore, only two statements are correct.
Statement 1 is incorrect: A nominated member of a House
2. Solution: (b) shall be disqualified for being a member of the House if
Exp) Option b is the correct answer. he joins any political party after the expiry of six months
Pair 1 is correctly matched: The Official Secrets Act, 1923 from the date on which he takes his seat. Thus he/she is free
prohibits person from using or wearing, without lawful to join any political party within six months and would not
authority, any naval, military, air force, police or other be disqualified for the same.
official uniform, or any uniform so nearly resembling the Statement 2 is correct: The law does not specify a
same as to be calculated to deceive, or falsely represents time period for the Presiding Officer to decide on a
himself to be a person who is or has been entitled to use or
disqualification plea under the Tenth Schedule.
wear any such uniform
Pair 2 is incorrectly matched: It is under The Official 5. Solution: (d)
Secrets Act, 1923, that no person in the vicinity of any Exp) Option d is the correct answer.
prohibited place shall obstruct, knowingly mislead or
Statement 1 is correct: The Tea Board of India is a statutory
otherwise interfere with or impede, any police officer, or
body created under the Tea Act, 1953 and it was established
any member of 21 [the Armed Forces of the Union] engaged
for the purposes of regulating the Indian tea industry and
on guard, sentry, patrol or other similar duty in relation to
protecting the interests of tea producers in India.
the prohibited place.
Statement 2 is incorrect: It is functioning as a statutory
Pair 3 is correctly matched: The Arms Act Amendment
body of the Central Government under the Ministry of
of 2019 adds news offences such as forcefully taking a
firearm from police or armed forces and using firearms in a Commerce.
celebratory gunfire which endangers human life or personal Statement 3 is incorrect: Tea Board of India’s Head Office is
safety of others. situated in Kolkata.

PYQ Workbook 308


INDIAN POLITY AND GOVERNANCE

Statement 4 is correct: The Tea Board of India has overseas Statement 2 is incorrect. Padma Awards were instituted in
offices in Moscow, Dubai, Hamburg, London and New 1954 to be awarded to citizens of India in recognition of their
York. Moscow office. The Moscow office of the Tea Board distinguished contribution in various spheres of activity.
of India operates under the Embassy of India, and its area The government suspended the practice of granting the
of activity includes Russia and the CIS countries, which Padma awards for two years in 1977. It was again suspended
comprise 50% of Indian tea exports. during mid-1992 when 2 PILs were filed in the High Courts
of India
6. Solution: (d)
They are given in three categories: Padma Vibhushan (for
Exp) Option d is the correct answer. exceptional and distinguished service), Padma Bhushan
The “right to the city” is individual and collective right of (distinguished service of higher-order) and Padma Shri
all residents of a city/town/village to an equal share of its (distinguished service).
benefits and to participate in its development. It is not to be Statement 3 is incorrect. A maximum of 3 people can be
viewed as a new legalistic right, but merely as an articulation awarded the Bharat Ratna.
to consolidate the demand for the realisation of multiple The total number of Padma awards to be conferred each
human rights. It developed as a movement for demand of year is limited to 120. But the count excludes posthumous
basic rights to city dwellers by Global NGO’s. awards and any non-resident Indian or Overseas Citizen of
Statement 1 is Incorrect : Right to the city has developed as India or foreign-based winners.
a response of social groups and civil society organizations to 8. Solution: (b)
ensure better access to and opportunities for everyone living Exp) Option b is the correct answer.
in cities, especially the most marginalized and deprived
Statement 1 is incorrect. A Constitution Bench of the
sections. Social movements and organizations from across
Supreme Court ruled that the Aadhaar metadata cannot
the world worked together to develop a World Charter on be stored beyond six months. The Aadhar Act had allowed
the Right to the City that is also supported by UNESCO and storage of such data for five years.
UN Habitat, among other agencies. Hence UN HABITAT
Statement 2 is correct. Supreme Court has struck down
only supports and doesnot monitor the commitments made Section 57 that allowed sharing of data with private
by each country in this regard. entities. But has allowed the unique number to be used for
Statements 2 and 3 are correct : The Right to city government schemes and subsidies.
demand is based on basic human rights principles of- non- Statement 3 is incorrect. Aadhaar is not mandatory for
discrimination; indivisibility of human rights; gender obtaining insurance products. Insurance regulator IRDAI
equality; progressive realisation; non-retrogression, has advised insurers not to mandatorily seek Aadhaar and
subsidiarity; solidarity; and, cooperation. It emphasises PAN/Form 60 from customers, existing or new, for KYC
priority attention to persons and groups in situations of (Know Your Customer) purpose. The insurers can accept
vulnerability, including the homeless/landless, children, Aadhaar as one of the documents for KYC, only when the
persons with disabilities, religious and sexual minorities, same is offered voluntarily by the proposer/policy-holder.
older persons, informal sector workers, Dalits/members of Statement 4 is correct. The amendment made to Section
Scheduled Castes, indigenous and tribal peoples, persons 7 of the Aadhaar (Targeted delivery of financial and other
from economically weaker sections, single women, and subsidies, benefits and services) Act 2016, make Aadhar
women-headed households, among others. It also stresses mandatory for beneficiaries for receipt of a subsidy, benefit
responsibility and sustainability as core principles. or service for which the expenditure is incurred from the
The concept of the right to the city is a means to combat consolidated fund.
the exclusionary development, selective benefit sharing, 9. Solution: (d)
marginalisation, and discrimination rampant in cities Exp) Option d is the correct answer.
today.
The Fiscal Responsibility and Budget Management Act,
7. Solution: (d) 2003 mandates that Central Government shall lay in each
financial year before both Houses of Parliament the following
Exp) Option d is the correct answer.
statements (along with the annual financial statement):
Statement 1 is incorrect. National awards such as Bharat
(a) the Medium-term Fiscal Policy Statement;
Ratna, Padma Vibhushan, Padma Bhushan and Padma Shri
do not amount to titles within the meaning of Article 18(1) (b) the Fiscal Policy Strategy Statement;
of the Constitution and thus are not to be used as prefixes or (c) the Macro-economic Framework Statement;
suffixes to the name of the recipient in any manner. (d) the Medium-term Expenditure Framework Statement.
Article 18(1) abolishes all titles. It prohibits the State to Macro-economic Framework Statement provides an
confer titles on anybody whether a citizen or a non-citizen. overview of the economy containing an assessment regarding
Military and academic distinctions are, however, exempted the fiscal balance of the Central Government, Price related
from the prohibition. Thus, a university can give title or fluctuations, GDP growth rate, and the external sector
honor on a man of merit. balance of the economy.

309 PYQ Workbook


INDIAN POLITY AND GOVERNANCE

10. Solution: (a) 2) He must make and subscribe to an oath or affirmation


Exp) Option a is the correct answer. before the person authorised by the election commission
for this purpose. In his oath or affirmation, he swears
One common agreement between Gandhism and Marxism
is the final goal of stateless and classless society, whereas (a) To bear true faith and allegiance to the Constitution of
means to achieve these final goals differ. India
For Marx, State is an instrument of oppression and an organ (b) To uphold the sovereignty and integrity of India
of the bourgeoisie that only works for maintaining the class 3) He must be not less than 30 years of age in the case of the
dominance. Therefore, he strives for class struggle and Rajya Sabha and not less than 25 years of age in the case
revolution which will ultimately result in a stateless society. of the Lok Sabha.
For Gandhi also, in an ideal state, there is no State. In his 4) He must possess other qualifications prescribed by
words “political power means capacity to regulate national Parliament.
life through national representatives. If national life becomes The Parliament has laid down the following additional
so perfect as to become self-regulated, no representation qualifications in the Representation of People Act (1951).
becomes necessary. There is then a state of enlightened 1) He must be registered as an elector for a parliamentary
anarchy. In such a state everyone is his own ruler. He rules constituency. This is same in the case of both, the Rajya
himself in such a manner that is never a hindrance to his Sabha and the Lok Sabha.
neighbour. In the ideal state, therefore, there is no political
2) He must be a member of a scheduled caste or scheduled
power because there is no State.”
tribe in any state or union territory, if he wants to
11. Solution: (a) contest a seat reserved for them. However, a member of
scheduled castes or scheduled tribes can also contest a
Exp) Option a is the correct answer.
seat not reserved for them.
Statement 1 is correct. Section 3 of the Parliament
(Prevention of Disqualification) Act, 1959 lists certain 13. Solution: (d)
offices of profit under the central and state governments, Exp) Option d is the correct answer.
which do not disqualify the holders from being an MP. It Statement 1 is incorrect. Election Commission consists of a
exempts posts like: Chief Election Commissioner and such number of Election
(i) Ministers of State and Deputy Ministers Commissioners as the President may deem it necessary
(ii) Parliamentary Secretaries and Parliamentary Under subject to the law made by Parliament in this regard.
Secretaries, Currently, it consists of Chief Election Commissioner and
two Election Commissioners.
(iii) Deputy Chief Whips in Parliament,
Statement 2 is incorrect. Election Commission (not Ministry
(iv) Vice-Chancellors of Universities, of Home Affairs) decides election schedule for the conduct
(v) Officers in the National Cadet Corps, and the Territorial of both general elections and bye-elections.
Army, and Statement 3 is correct. Election Commission resolves the
(vi) the Chairman and members of Advisory Committees set disputes relating to splits/mergers of recognized political
up by the Government when they are not entitled to any parties.
fee or remuneration other than compensatory Etc. 14. Solution: (a)
Statement 2 is correct. The Parliament (Prevention of Exp) Option a is the correct answer.
Disqualification) Act, 1959 has been amended 5 times since
The National Green Tribunal Act, 2010 was enacted to
its formulation (in 1960, 1992, 1993, 2006 and 2013).
create the National Green Tribunal for the facilitation of the
Statement 3 is incorrect. Article 102 of the Constitution of judicial administration.
India provides that a person shall be disqualified from his
The National Green Tribunal Act, 2010 specifically aims
membership of parliament if he holds any office of profit at the creation of a National Green Tribunal which would
under the Government of India or the Government of any help in the reduction of the burden of cases from the higher
State. However, the ‘Office of Profit’ is not defined in the judiciary and effectively and efficiently dispose of issues
Constitution of India. relating to the environment, including basic rights of the
12. Solution: (c) individual for free and safe environment, protection of the
flora and fauna, etc.
Exp) Option c is the correct answer.
This act was created in consonance with the need to effectively
According to section 4 (d) of the Representation of People
manage and divert the increasing chunk of pending cases
Act, 1951, for election to Lok Sabha, a nomination paper
from the other organs of the judiciary. This tribunal was also
can be filed by any citizen whose name appears in the created so that the basic right of the individuals to a safe
electoral rolls of any parliamentary constituency. environment can be safeguarded as part of Article 21 of the
The Constitution lays down the following qualifications for a Constitution. As the aims of the National Green Tribunal
person to be chosen a member of the Parliament: suggest, there is a huge importance attached to Article 21 of
1) He must be a citizen of India. Constitution concerning environmental matters.

PYQ Workbook 310


INDIAN POLITY AND GOVERNANCE

The Act was not enacted in consonance with the provisions in connection with the affairs of the Union or other local
of grants for raising the level of administration in the authorities within the territory of India or under the control
Scheduled Areas for the welfare of Scheduled Tribes under of Government of India. CAT was set up in 1985 during
Article 275(1) and powers and functions of Gram Sabha as Rajiv Gandhi’s tenure. This was done in pursuance of the
mentioned under Article 243(A) amendment of Constitution of India by Articles 323A.
15. Solution: (d) Statement 2 is correct. The Chairman of CAT who has been
a sitting or retired Judge of a High Court heads the Central
Exp) Option d is the correct answer.
Administrative Tribunal. At present, there are 66 Hon’ble
Statement 1 is incorrect. Union Territories does have Members in various Benches of the Tribunal out of which 33
representation in the Rajya Sabha. Although, out of the are Judicial Members, including the Hon’ble Chairman and
nine union territories, only three (Delhi, Puducherry and 33 are Administrative Members. Subject to other provisions
Jammu & Kashmir) have representation in Rajya Sabha. The of the Act, a Bench consists of one Judicial Member and one
populations of other six union territories are too small to Administrative Member.
have any representative in the Rajya Sabha. The maximum
membership of Rajya Sabha as laid down in the Constitution 18. Solution: (a)
is 250. The present strength, however, is 245 members of Exp) Option a is the correct answer
whom 233 are representatives of the States and Union Statement 1 is correct: The Commonwealth is a voluntary
territories and 12 are nominated by the President. The association of member States and has no charter, treaty or
allocation of seats to be filled by representatives of the States constitution.
and Union territories as presently laid down in the Fourth
Statement 2 is incorrect: For becoming a member of the
Schedule.
Commonwealth, a country needs to apply for its membership.
Statement 2 is incorrect. Chief Election Commissioner Further, not all the former British colonies are the member
does not adjudicate the election disputes. Special Election of Commonwealth like the USA.
Tribunals are to be constituted at the regional level under
Article 329B of the Constitution to ensure speedy disposal 19. Solution: (c)
of election petitions and disputes within a stipulated period Exp) Option c is the correct answer
of six months. Article 350A facilitates instruction in the mother tongue at
However, currently no such tribunal has been established by primary stage of education. The article 350A says that it shall
the parliament. So, at present, election petitions are filed in be the endeavor of every State and of every local authority
the High Court of the particular state in which the election within the State to provide adequate facilities for instruction
was conducted. in the mother tongue at the primary stage of education to
Statement 3 is incorrect. According to the Constitution, children belonging to linguistic minority groups. It also says
Parliament consists of the President and the two Houses— that the President may issue such directions to any State as
Rajya Sabha and Lok Sabha. he considers necessary or proper for securing the provision
of such facilities.
16. Solution: (c)
Exp) Option c is the correct answer. Important Tips
The Delimitation Commission or Boundary Commission of Some important articles of the Indian Constitution are
India is a commission established by the Government of India following:
under the provisions of the Delimitation Commission Act. • Article 343 - Official Languages of the Union.
The main task of the commission is redrawing the boundaries
• Article 345 - Official Languages or languages of states.
of the various assembly and Lok Sabha constituencies based
on a recent census. The representation from each State is not • Article 348 - Languages to be used in the Supreme
changed during this exercise. However, the number of SC court and in the High Courts.
and ST seats in a state are changed in accordance with the • Article 351 - Directive for the Development of the
census. Hindi language
Both Statements 1 and 2 are correct. The Commission is
20. Solution: (d)
a powerful and independent body whose orders cannot be
challenged in a court of law. The orders are laid before the Exp) Option d is the correct answer.
Lok Sabha and the respective State Legislative Assemblies. Assertion (A) is false- The reservation of 33% seats for
However, modifications are not permitted. women in Parliament and State Legislature requires a
constitutional amendment. This is because the Constitution
17. Solution: (b)
of India does not explicitly provide for reservation of seats for
Exp) Option b is the correct answer. women. The Constitution (108th Amendment) Bill, 2008
Statement 1 is incorrect. The Central Administrative proposes to reserve one-third of all seats in the Lok Sabha
Tribunal (CAT) has been established for adjudication of and state legislative assemblies for women. The allocation of
disputes with respect to recruitment and conditions of reserved seats will be determined by an authority prescribed
service of persons appointed to public services and posts by Parliament.

311 PYQ Workbook


INDIAN POLITY AND GOVERNANCE

Reason (R) is true- Political parties can allocate 33% of seats with the most votes wins. However, this does not mean that
they contest to women candidates without any constitutional the party with the most votes will also have the majority of
amendment. However, this does not mean that women will be seats. This is because the number of seats a party wins is
guaranteed 33% of seats in Parliament and State Legislature. based on the number of votes it wins in each constituency.
This is because the number of seats that women actually win So, it is possible for a party to win the most votes overall, but
will depend on the number of women candidates that are not have the majority of seats.
fielded by political parties and the number of votes that they
win. 24. Solution: (d)
Exp) Option d is the correct answer
21. Solution: (a)
The veto power is only available to the five permanent
Exp) Option a is the correct answer
members of the UN Security Council. Any other member of
The Panchsheel Agreement is a pact of “Five Principles
the Security Council cannot use veto power.
of Peaceful Coexistence”. The first formal declaration of
these concepts was an Agreement on Trade and Commerce
between India and China’s Tibet region, signed on 29th
9.2. Other Examination Previous Years’
April 1954. It includes principles of mutual respect for Questions
territorial integrity, non-aggression, non-interference in 25. Solution: (d)
internal affairs, equality & mutual benefit, and peaceful
Exp) Option d is the correct answer
coexistence. Non-alignment, a distinct policy, involves
maintaining neutrality without aligning with major power The national calendar based on the Saka Era, with Chaitra
blocs during the Cold War era and is not part of Panchsheel. as its first month and a normal year of 365 days was adopted
from 22 March 1957 along with the Gregorian calendar for
Important Tips
the official purposes:
• The 5 principles were emphasized by Prime Minister
Jawaharlal Nehru and Premier Zhou Enlai in a Important Tips
broadcast speech made at the time of Asian Prime • The Vikrama era (57 BC) is said in the Jain book
Ministers Conference in Colombo, Sri Lanka after Kālakācāryakathā to have been founded after a victory
signing the Sino-Indian Agreement in Beijing, 1954. of King Vikramaditya over the Śaka.
• The five principles were subsequently modified in a • The Saka era was started by Kanishka I from 78 CE.
form of a statement of ten principles issued in April
• The Hijri year started from 622 AD.
1955 at the historic Asian-African Conference in
Bandung, Indonesia. 26. Solution: (d)
• The conference itself would lead to the foundation of Exp) Option d is the correct answer
the Non-Aligned Movement which gave shape to the
The Eighth Schedule of the Indian Constitution includes 22
idea that the post-colonial nations had something to
offer to the bipolar world of the Cold War. languages. Originally, it had 14 languages but presently there
are 22 languages. The 22 languages are (1) Assamese, (2)
22. Solution: (c) Bengali, (3) Gujarati, (4) Hindi, (5) Kannada, (6) Kashmiri,
Exp) Option c is the correct answer (7) Urdu (8) Malayalam, (9) Telugu, (10) Marathi, (11)
A. President of India: 3. Faith and Allegiance to the Tamil,(12) Oriya, (13) Punjabi, (14) Sanskrit, (15) Sindhi,
Constitution of India (16) Nepali,(17) Manipuri, (18) Konkani,(19) Bodo, (20)
B. Judges of Supreme Court: 4. Upholding the sanctity of Santhali, (21) Maithili and (22) Dogri.
the Constitution and Law Important Tips
C. Members of Parliament: 2. Faithful discharge of Duties Eighth Schedule:
D. Ministers for the Union: 1. Secrecy of Information • The Eighth Schedule to the Constitution of India lists
23. Solution: (a) the official languages of India. The Eighth Schedule
recognises a total of 22 languages.
Exp) Option a is the correct answer.
• However, it can be noted that there are no fixed criteria
Assertion (A) is true- In India, the political parties that form
for any language to be considered for inclusion in the
the government do not always have the majority of votes.
Eighth Schedule.
For example, in the 2014 Lok Sabha elections, the Bharatiya
• Part XVII of the Indian constitution deals with the o
Janata Party (BJP) won 282 seats, which was only 31.1% of
fficial languages in Articles 343 to 351.
the total votes cast. However, the BJP was able to form the
government because it had the most seats. • The Constitutional provisions relating to the Eighth
Schedule occur in Articles 344(1) and 351 of the
Reason (R) is true and (R) is the correct explanation of
Constitution.
(A)- The majority-vote-system is a system where the party

PYQ Workbook 312


INDIAN POLITY AND GOVERNANCE

27. Solution: (b) Important Tips


Exp) Option b is the correct answer. • The top saffron colour indicates the strength and
courage of the country.
Assertion (A) is true- India does have a written constitution.
• The white middle band indicates peace and truth with
The Constitution of India was adopted on 26 January 1950, Dharma Chakra.
and it is the supreme law of the land. • The green shows the fertility, growth and auspiciousness
Reason (R) is true, but (R) is not the correct explanation of the land.
of (A)- The growth of strong regional parties is an indicator 31. Solution: (a)
of regional aspirations. This means that people in different Exp) Option a is the correct answer
regions of India have different needs and interests, and they
• Tashkent Agreement (1966): India-Pakistan accord after
want to be represented by political parties that will fight for
the 1965 war, called for a ceasefire and troop withdrawal
their interests. However, The fact that India has a written from the border.
constitution does not necessarily mean that there will be
• Indo-Soviet Treaty (1971): Friendship and cooperation
strong regional parties. There are many other factors that
treaty with the Soviet Union.
can contribute to the growth of regional parties, such as
• Shimla Agreement (1972): India-Pakistan pact after the
economic inequality, linguistic diversity, and historical
Bangladesh Liberation War, establishing the LOC and
factors.
promoting peaceful dispute resolution.
28. Solution: (a) • Farakka Accord (1977): India-Bangladesh agreement aimed to
Exp) Option a is the correct answer resolve the water-sharing dispute related to the Ganges River.

The policy of non-alignment was first articulated by 32. Solution: (c)


Jawaharlal Nehru, India’s first Prime Minister, in the early Exp) Option c is the correct answer
years of India’s independence. Nehru believed that India Extra-constitutional right refers to a power that is not
should not align itself with either the Western bloc or the explicitly granted by the constitution. Such powers may be
Eastern bloc, but should instead pursue an independent derived from custom, tradition, or other sources of law.
foreign policy based on the principles of peace,
33. Solution: (b)
cooperation, and non-violence.
Exp) Option b is the correct answer
29. Solution: (d)
Part XVII of the Constitution deals with the official
Exp) Option d is the correct answer language in Articles 343 to 351. The first Official Language
Causes of regionalism in India: Commission was formed in 1955 under the chairmanship
of B.G. Kher. The commission submitted its report to the
• Uneven development: Disparities in economic, social,
President in 1956. The report was examined by a committee
and infrastructure development between regions lead to
of Parliament constituted in 1957 under the chairmanship of
resentment and frustration. Gobind Ballabh Pant.
• Fear of losing cultural identity: Globalization and
homogenization create a fear of losing diverse cultural Important Tips
identity, sparking regionalism to protect it. • Article 344(1) provides for the constitution of a
Commission by the President on expiration of five
• Political prominence: Rise of regional parties in
years from the commencement of the Constitution
Indian politics empowers leaders to advocate for their and thereafter at the expiration of ten years from such
constituents’ concerns and demand more resources for commencement. This commission shall consist of a
their regions. Chairman and such other members representing the
different languages specified in the Eighth Schedule
30. Solution: (b) to make recommendations to the President for the
Exp) Option b is the correct answer progressive use of Hindi for official purposes of the
Union.
The National Flag of India “Tiranga” i.e., having three colors
• The Official Language Commission (as envisaged by
in a rectangular shape. Saffron in the top, White with Navy
the Constitution) was not appointed in 1960.
blue wheel having 24 spokes in Center and Green in the
bottom. It was adopted by the Constituent Assembly on 34. Solution: (c)
22 July 1947 and became official National Flag when India Exp) Option c is the correct answer.
became Republic. The ratio of the width of the National Flag Inner-party democracy is a system of governance within
to its length is 2: 3. a political party that allows for the participation of its

313 PYQ Workbook


INDIAN POLITY AND GOVERNANCE

members in the decision-making process. This includes the Important Tips


right to elect party leaders, to participate in party debates,
As per the ECI’s Political Parties and Election Symbols,
and to hold the party leadership accountable. Periodical
2019 handbook, a political party can be recognized as a
elections within the party to elect the office bearers of the national party if it meets any of the following conditions:
party is one of the key features of inner-party democracy.
• It is recognized in four or more states.
This ensures that the party leadership is accountable to the
party members and that the party is responsive to the needs • Its candidates have secured at least 6% of the total
valid votes in a minimum of 4 states during the latest
of the people.
Lok Sabha or Assembly elections, and the party has at
35. Solution: (b) least 4 Members of Parliament (MPs) from the last
Lok Sabha polls.
Exp) Option b is the correct answer
• It has won at least 2% of the total seats in the Lok
The Press Council of India (PCI) is a statutory body
Sabha from a minimum of 3 states.
established under the Press Council Act, 1978, serving as a
• Currently, Seven Political Parties in India are
self-regulatory body for India’s print media. It investigates
recognized as National Party by the Election
complaints, issues directions for corrections and apologies,
Commission of India
and recommends actions against journalists. Though not
a judicial body, it holds quasi-judicial powers, conducting 39. Solution: (b)
hearings and issuing legally enforceable orders. Exp) Option b is the correct answer
36. Solution: (b) The wheel at the centre of our National Flag is known as
Ashoka Chakra. Our National Flag was composed by Pingali
Exp) Option b is the correct answer.
Venkayya. The blue colour denotes the colour of the sky and
The National Anthem of India Jana-gana-mana, composed ocean. The wheel in the national flag represent Motion.
originally in Bengali by Rabindranath Tagore, was adopted This wheel is also known as wheel of the law of dharma/
in its Hindi version by the Constituent Assembly as the Justice. Truth or satya, dharma or virtue ought to be the
National Anthem of India on 24 January 1950. controlling principle of those who work under this flag.
Important Tips
40. Solution: (b)
National Anthem:
Exp) Option b is the correct answer
• It was first sung on 27 December 1911 at the Kolkata
Statement 1 is incorrect: UN peacekeepers are not only
Session of the Indian National Congress.
authorized to use force in self-defense or in defense of
• Rabindranath Tagore had published it in ‘Tatvabodhini’ the mandate given to them by the UNSC, but in certain
in 1912 with the title ‘Bharat Bhagya Vidhata’ and volatile situations, the UNSC has given UN peacekeeping
translated it into English in 1919 with the title ‘Morning operations “robust” mandates authorizing them to “use all
song of India’. necessary means” to deter forceful attempts to disrupt the
• The credit of composing the present tune (music) political process, protect civilians under imminent threat
of our national anthem goes to Captain Ram Singh of physical attack, and/or assist the national authorities in
Thakur (an I.N.A. sepoy). maintaining law and order.
Statement 2 is correct: The Security Council is responsible
37. Solution: (b) for authorizing and overseeing peacekeeping missions, and
Exp) Option b is the correct answer its decisions determine the continuation or termination of
such operations.
The ‘No First Use’ policy means that India will not be the first
to use nuclear weapons in any conflict or war. This policy was Statement 3 is correct: The USA is the largest budget
contributor to UN peacekeeping operations, providing the
officially declared by India in 1999 under the leadership
highest percentage (28% in 2021) of assessed contributions
of Prime Minister Atal Bihari Vajpayee. It reflects India’s
among all member states.
commitment to using nuclear weapons solely as a deterrent
against aggression and not for initiating hostilities. 41. Solution: (d)
Exp) Option d is the correct answer
38. Solution: (b)
The Bharat Ratna is the highest civilian award of the Republic of
Exp) Option b is the correct answer.
India. Instituted on 2 January 1954, the award is conferred in
The Election Commission of India (ECI) is an autonomous recognition of “exceptional service/performance of the highest
constitutional body responsible for conducting elections to order”, without distinction of race, occupation, position, or sex.
the Lok Sabha, Rajya Sabha, state legislative assemblies, The First Recipient of the Bharat Ratna was Dr. Sarvepalli
and local bodies in India. The ECI recognizes political Radhakrishnan, who received this esteemed award in 1954 for his
parties in India based on certain objective criteria. distinguished services as a philosopher, statesman, and academic.

PYQ Workbook 314


INDIAN POLITY AND GOVERNANCE

Important Tips The District Magistrate is a judicial officer, while the District
Collector is an executive officer. In the role of a magistrate
Additional facts on Bharat Ratna Awards
DC has the powers to maintain law and order, control the
• On the Prime Minister’s suggestion to the President of police, and check the passports of foreigners. He/she also has
India, a maximum of three Bharat Ratna awards can be the power to control land revenue, but this is under the role
presented each year.
of Collector, not as the Magistrate.
• The recipient of the award receives a Sanad (certificate)
signed by the President as well as a medallion. The Important Tips
prize does not come with a monetary grant. Other functions of DC as a Magistrate:
• The awardees are not allowed to use Bharat Ratna as • As District Magistrate, the powers and responsibilities
a prefix or suffix to their names, as stated in Article include issuance of adoption orders under the Juvenile
18(1) of the Indian Constitution. Justice Act, granting arms and ammunition licenses,
• There is no formal requirement that the Bharat Ratna and cinema licenses, all subject to appeal to divisional
be granted solely to Indian nationals. This prestigious commissioners.
prize was given to a naturalized Indian citizen, Agnes • They conduct criminal courts as an executive
Gonxha Bojaxhiu, popularly known as Mother Teresa magistrate, and supervise subordinate executive
(1980), as well as non-Indians, Khan Abdul Ghaffar magistracy.
Khan (1987) and Nelson Mandela (1990). • Additionally, they handle cases under the preventive
• Posthumous honours were not permitted under the section of the Criminal Procedure Code, oversee jails,
1954 legislation, but this was changed in the January and certify capital sentence executions.
1955 act, and Lal Bahadur Shastri became the first • Their duties involve inspecting police stations,
winner to be honoured posthumously in 1966. prisons, and juvenile homes, authorizing parole orders,
preparing panels for appointing public prosecutors,
42. Solution: (b)
and managing crises during riots, and issues related to
Exp) Option b is the correct answer. child labor and bonded labor.
Statement A is correct- The Electoral Bond Scheme was
introduced by the Government of India in 2018. The aim of 45. Solution: (b)
the scheme is to bring about transparency in the funding Exp) Option b is the correct answer
process of political parties. The Param Vir Chakra, highest gallantry award in India,
Statement B is correct- Only political parties recognized by features a circular bronze disc 35 millimetres in diameter.
the Election Commission which secured not less than two The obverse side displays the National Emblem of India
percent of the votes polled in the last general election to the at the center, surrounded by four copies of the vajra, the
House of People or the Legislative Assembly of the State shall weapon of Indra, the ancient Vedic king of the gods. This
be eligible to receive the Electoral Bonds. motif symbolizes the sacrifice of Rishi Dadhichi, who gave
Statement C is correct- The scheme allows individuals and his bones to make the vajra to kill the demon Vritra.
companies to donate money to political parties through
Important Tips
Electoral Bonds. Electoral Bonds are bonds that are issued
by the government and can be purchased by individuals and Gallantry Awards in India:
companies. The bonds are valid for fifteen calendar days • Param Vir Chakra is the highest wartime award for
from the date of issue. exceptional valor, followed by Maha Vir Chakra and
Vir Chakra.
Statement D is correct- The Electoral Bond deposited by an
eligible political party in its account shall be credited on the • Ashoka Chakra tops peacetime awards for bravery
same day. or sacrifice, followed by Kirti Chakra and Shaurya
Chakra.
43. Solution: (c)
• Additional awards like Sena Medal, Nao Sena Medal,
Exp) Option c is the correct answer and Vayu Sena Medal are bestowed for acts of courage
The Gandhian Constitution for Free India was a draft or devotion to duty in their respective services.
constitution that was prepared by Shriman Narayan • The first recipient was Major Somnath Sharma for his
Agarwal in the early 1940s. The constitution was based on actions in the 1947-48 India-Pakistan War and the last
the principles of Gandhian economics and social justice. It recipient was Captain Vikram Batra for actions in the
was never adopted by the Indian government, but it remains Kargil War in 1999.
an important document in the history of Indian political • Till now it has been granted to 21 armed forces
thought. personnel.
44. Solution: (c) • It was designed by Savitri Khanolkar, a Swiss national
Exp) Option c is the correct answer whose real name was Eve Yvonne Maday de Maros.

315 PYQ Workbook


INDIAN POLITY AND GOVERNANCE

46. Solution: (c) • Some of the key tenets of scientific socialism include:
Exp) Option c is the correct answer • The materialist conception of history: This is the
The DAC is the highest decision-making body in the Defence belief that the material conditions of life, such as the
Ministry for deciding on new policies and capital acquisitions way people produce and distribute goods and services,
for the three services (Army, Navy and Air Force) and the determine the social, political, and intellectual
Indian Coast Guard. The defence minister is the chairman development of society.
of DAC. Its members include the Chief of Defence Staff • The class struggle: This is the belief that society is
(CDS) and Chiefs of the Army, Navy, and Air Force. divided into two main classes, the bourgeoisie and
Important Tips the proletariat. The bourgeoisie are the owners of the
means of production, while the proletariat are the
• The CDS holds the highest military rank in India
workers who sell their labor to the bourgeoisie.
and provides single-point military advice to the
government. • The dictatorship of the proletariat: This is the belief
• Appointed by the President on the advice of the Prime that the working class will eventually overthrow the
Minister, the CDS is a four-star general and serves capitalist system and establish a socialist society. The
as the permanent Chairman of the Chiefs of Staff socialist society will be a classless society in which the
Committee (COSC). means of production are owned and controlled by the
working class.
• The CDS is a member of the Defence Planning
Committee (DPC) responsible for formulating long- 49. Solution: (d)
term defence plans.
Exp) Option d is the correct answer.
• As the head of the Department of Military Affairs
(DMA) in the Ministry of Defence, the CDS acts as National Emblem of India is a representative seal of the
the single-point adviser to the Defence Minister. Republic of India that is adapted from the Lion Capital
of Ashoka Pillar (based in Sarnath, Uttar Pradesh.) India
• In the Department of Defence (DoD), the CDS holds
adopted it as the State Emblem on 26th January 1950. The
the rank of Secretary and exercises powers limited to
the revenue budget. Additionally, the CDS performs motto of the Indian National Emblem is ‘Satyamev Jayate’ or
an advisory role in the Nuclear Command Authority ‘Truth Alone Triumphs.
(NCA). Important Tips
47. Solution: (d) • The National Emblem is the symbol of authority and
is present in all the official communications of the
Exp) Option d is the correct answer.
government.
A pressure group is an organized group of people who
• There are Four Asiatic Lions and one each of Elephant,
try to influence government policy. They do not contest
Horse, and a Bull on national emblem.
elections, but they try to influence the government by
lobbying, protesting, and other forms of advocacy. • Dinanath Bhargava sketched and illuminated India’s
Members of a trade union are a pressure group because they National Emblem
try to influence government policy on issues such as wages,
50. Solution: (c)
working conditions, and benefits. They do this by lobbying
government officials, organizing strikes, and other forms of Exp) Option c is the correct answer.
advocacy. The formal rendition of ‘Jana Gana Mana’, which is written
in Bengali Sanskrit, takes 52 seconds, a shorter version takes
48. Solution: (a) approximately 20 seconds. The shorter version includes the
Exp) Option a is the correct answer first and last lines of the National Anthem and is played on
Scientific socialism is attributed to Karl Marx. He, along certain occasions. Composed by Rabindranath Tagore, ‘Jana
with Friedrich Engels, developed the concept of scientific Gana Mana’ was officially declared India’s National Anthem
socialism, which forms the theoretical foundation of on January 24, 1950.
Marxism. It emphasizes the scientific analysis of society,
51. Solution: (d)
historical materialism, and the inevitable transition from
capitalism to communism. Exp) Option d is the correct answer.
“Due process of law” was already a part of the Indian legal
Important Tips
system, inherited from the British common law tradition.
• Scientific socialism employs the scientific method to The majority view in the A.K. Gopalan case held that
analyze historical trends, predict social and economic the right to “due process of law” was not a fundamental
outcomes, and understand material phenomena. It right under the Constitution of India, allowing the state to
differs from utopian socialism and classical liberalism, deprive a person of their personal liberty without following
focusing on materialist explanations rather than established legal procedures, as long as the deprivation was
metaphysical or moral notions. not arbitrary or mala fide. However, the minority view in

PYQ Workbook 316


INDIAN POLITY AND GOVERNANCE

the same case argued that “due process of law” was indeed regardless of their background, identity, or social status. It
a fundamental right, necessitating adherence to established emphasizes fairness and equal opportunities for all.
legal procedures even if the deprivation was not arbitrary or Statement 3 is correct- Social justice recognizes that
mala fide. Subsequently, the Supreme Court overruled its equality of treatment alone may not address the diverse
decision in the A.K. Gopalan case in the Maneka Gandhi needs and challenges faced by individuals or groups. It
case, holding that the right to “due process of law” is emphasizes the need to go beyond equal treatment and take
indeed a fundamental right. Consequently, the state can into account special needs, circumstances, and disadvantages
only deprive a person of their personal liberty if it adheres to to ensure fair outcomes.
the procedures established by law.
55. Solution: (b)
52. Solution: (b) Exp) Option b is the correct answer
Exp) Option b is the correct answer. One of the key features of polyarchy is that the right to
While community and cooperation are valued in socialism, run for office is not restricted. In other words, anyone who
they are not the central focus or defining characteristic of the meets the legal requirements should be able to run for office,
ideology. Socialism primarily centers around the economic regardless of their race, gender, religion, or social class.
and political aspects of society, with a focus on collective
Important Tips
ownership of the means of production, economic equality,
and social justice. While community plays a role, it is not the In his 1989 book Democracy and Its Critics, Dahl gives
the following characteristics of a polyarchy:
sole emphasis of socialism. The ideology primarily seeks to
transform the economic system and redistribute resources • Control over governmental decisions about policy is
and wealth to create a fairer society for all. constitutionally vested in elected officials.
• Elected officials are chosen and peacefully removed
53. Solution: (a)
in relatively frequent, fair and free elections in which
Exp) Option a is the correct answer. coercion is quite limited.
A. Pluralist democracy - (4) Capacity of groups to ensure • Practically all adults have the right to vote in these
Governmental responsiveness. Pluralist democracy refers elections.
to a system where power is dispersed among different • Most adults also have the right to run for the public
interest groups, and these groups have the ability to offices for which candidates run in these elections.
influence government decisions and policies.
• Citizens have an effectively enforced right to freedom
B. People’s democracy - (3) Social equality through of expression, particularly political expression,
common ownership of wealth. People’s democracy including criticism of the officials, the conduct of the
aims for social equality by advocating for common government, the prevailing political, economic, and
ownership of wealth and resources, ensuring equitable social system, and the dominant ideology.
distribution among all members of society.
• They also have access to alternative sources of
C. Developmental democracy - (2) Highest and information that are not monopolized by the
harmonious development of individual capabilities. government or any other single group.
Developmental democracy focuses on promoting the
holistic development of individual capabilities, 56. Solution: (b)
allowing individuals to reach their full potential in a Exp) Option b is the correct answer
harmonious society. Statement 1 is correct: Liberal democracies have a system of
D. Elitist democracy - (1) Power is always exercised by a government that is based on a constitution, and government
privileged few. Elitist democracy refers to a system where operates within the framework of formal rules, which provide
power is concentrated in the hands of a small, privileged a check on the exercise of power.
group, often excluding the majority of the population Statement 2 is correct: Liberal democracies place a strong
from decision-making processes. emphasis on protecting the civil liberties and individual
rights of their citizens. The government is obligated to
54. Solution: (d)
respect and protect these rights, and individuals have the
Exp) Option d is the correct answer. freedom to exercise them without undue interference.
Statement 1 is incorrect- The idea of social justice does Statement 3 is incorrect: Liberal democracy is characterized
not necessarily require the principle of proportionality. by a limitation on political authority and the existence of
The principle of proportionality means that people should checks and balances.
be treated differently if there is a good reason to do so.
For example, a person with a disability may need to be given 57. Solution: (b)
special treatment in order to be able to participate in society. Exp) Option b is the correct answer
Statement 2 is correct- The idea of social justice entails Jay Prakash Narayan (JP) was an Indian socialist leader and
giving due and equal consideration to all individuals, advocate of partyless democracy. He believed that political

317 PYQ Workbook


INDIAN POLITY AND GOVERNANCE

parties were corrupt and inefficient, and proposed a system that are still developing and transitioning from traditional
without parties for greater transparency, accountability, and or agrarian structures to modern political systems.
people’s participation.
61. Solution: (c)
58. Solution: (c) Exp) Option c is the correct answer.
Exp) Option c is the correct answer Social equality refers to the principle that all individuals
Statement 1 is incorrect: Globalization is not just about in a society should have equal rights, opportunities, and
conducting external relations between two or more States of treatment without discrimination based on factors such
the world. It is a much broader process that encompasses as caste, race, religion, gender, or socioeconomic status. It
the free flow of goods, services, capital, and people across emphasizes the elimination of social hierarchies and the
national borders. absence of any form of relative inferiority or subjugation.
Statement 2 is correct: Globalization involves the integration Option (c) correctly captures the essence of social equality
and interconnectedness of social, economic, cultural, and by stating that no person should be subjected to a
political relations on a global scale, often transcending position of relative inferiority in comparison to others
national boundaries and territories. due to factors like caste, race, or religion. This means
Statement 3 is correct: Globalization entails recognizing that individuals should be treated with dignity and respect
and addressing global risks and challenges that affect regardless of their background, and they should have equal
people worldwide, emphasizing the need for a transnational access to opportunities, resources, and social benefits.
perspective and cooperation. 62. Solution: (c)
Statement 4 is correct: There is no single global government Exp) Option c is the correct answer.
that is responsible for managing the world. However, there
Option a is correct: Human rights are based on fundamental
are a number of international organizations that play a role
moral principles and values that recognize the inherent
in managing global issues. These organizations can help to
dignity and worth of every individual. They encompass
coordinate our efforts and to ensure that we are working
principles such as equality, freedom, justice, and respect for
together to address global challenges.
human dignity.
59. Solution: (c) Option b is correct: Human rights are recognized
Exp) Option c is the correct answer. and protected through international human rights
instruments, such as treaties and conventions, which
Statement a is correct: Consent refers to the voluntary
are adopted by states. States have the responsibility to
agreement of the governed to be governed and to accept
incorporate these rights into their domestic laws and ensure
the authority of the government. When people willingly
their implementation and enforcement.
consent to be governed, it contributes to the legitimacy of
the political system. Option c is incorrect: International Covenant on Civil and
Political Rights (ICCPR) was adopted by the United Nations
Statement b is correct: Rational prudence involves the use
General Assembly on December 16, 1966, and it entered
of reason and prudence in decision-making, taking into
into force on March 23, 1976. The Universal Declaration of
account the best interests of the society. It is a source of
Human Rights (UDHR) was adopted by the United Nations
political legitimacy.
General Assembly on December 10, 1948, and it served as
Statement c is incorrect: It implies that Government has an inspiration for the development of various human rights
authority because of its own power or interests, which does instruments, including the ICCPR.
not match any of the sources mentioned above. In fact, this
Option d is correct: The concept of cultural relativism
option could be seen as a form of illegitimate rule, especially
challenges the universality of human rights. Cultural
if it contradicts the other sources of legitimacy. relativists argue that human rights should be understood and
Statement d is correct: Public reason refers to the use of interpreted within the context of specific cultural, social,
impartial and justifiable reasoning in political decision- and historical circumstances, and that different cultures may
making. It involves considering the common good and the have different values and norms regarding rights.
interests of all citizens when making policy choices. Public
reason can enhance political legitimacy by ensuring that 63. Solution: (c)
decisions are fair, transparent, and grounded in rationality. Exp) Option c is the correct answer.
Montesquieu, a French political philosopher of the
60. Solution: (a)
Enlightenment era, categorized governments into three
Exp) Option a is the correct answer. types: Republican, Monarchical, and Despotic. He
Parochial political culture is generally found in developing discussed these forms of government in his influential work
societies. In parochial political cultures, people tend to have “The Spirit of the Laws” (1748). Montesquieu’s ideas on
limited political awareness, low political participation, and the separation of powers and the importance of checks and
a strong attachment to traditional and local loyalties. This balances in government greatly influenced political thought
type of political culture is often associated with societies and constitutional design.

PYQ Workbook 318


INDIAN POLITY AND GOVERNANCE

Important Tips • Dinesh Goswami Committee (1990): This committee


made several recommendations on issues such as the
• Aristotle was an ancient Greek philosopher and
appointment of the Chief Election Commissioner, the
polymath. He is known for his contributions to various
anti-defection law, the voting system, the model code of
fields, including philosophy, politics, ethics, and more.
Aristotle’s works, such as “Politics,” explored different conduct, the security deposit, etc.
forms of government and discussed the ideal forms of • Tarkunde Committee (1975): This committee was
governance. headed by Justice V.M. Tarkunde and consisted of
• Thomas Hobbes was an English philosopher who is eminent citizens such as Jaya Prakash Narayan, A.G.
best known for his book “Leviathan.” In his work, Noorani, R.D. Desai, etc. The committee suggested
Hobbes discussed social contract theory and the need changes in the procedure of appointment of the Chief
for a strong central authority to maintain order and Election Commissioner, the reduction of voting age to 18
prevent the “state of nature.” years, the use of electronic voting machines, etc.
• St. Augustine, also known as Augustine of Hippo, was • Indrajit Gupta Committee (1998): This committee
an early Christian theologian and philosopher. While was headed by former Home Minister Indrajit Gupta
he made significant contributions to Christian theology and dealt with the issue of state funding of elections.
and philosophy, his writings do not specifically The committee recommended partial state funding of
focus on categorizing governments into Republican, elections in kind such as petrol, paper, vehicles, etc. to
Monarchical, and Despotic forms. recognized political parties and candidates.
64. Solution: (b) The Balwantrai Mehta Committee (1957) was not related to
Exp) Option b is the correct answer. electoral reforms, but to panchayati raj institutions.
Rashtriya Rifles is under the control of the Ministry of 67. Solution: (d)
Defence. The Rashtriya Rifles is a counter-insurgency force Exp) Option d is the correct answer.
of the Indian Army that operates in the state of Jammu
Section 10(26) of the Income Tax Act provides specific
and Kashmir. It was established in 1990 with the objective
exemptions from income tax for certain categories of
of conducting anti-insurgency operations and maintaining
individuals. In this case, the exemption applies to members
peace in the region.
of the Scheduled Tribes in the states of Manipur, Mizoram,
Important Tips Nagaland, and Tripura.
• The Indo-Tibetan Border Police (ITBP) is under the
68. Solution: (d)
control of the Ministry of Home Affairs. It is primarily
responsible for guarding the Indo-Tibetan border. Exp) Option d is the correct answer.
• The National Security Guard (NSG) is also under The major cleavage in global human rights discourse is often
the control of the Ministry of Home Affairs. It is a seen between civil and political rights on one hand, and
specialized counter-terrorism force that deals with economic, social, and cultural rights on the other hand.
specific security situations. There is often a debate and tension between these two
• The Sashastra Seema Bal (SSB) is also under the categories of rights, as they require different approaches
control of the Ministry of Home Affairs. It is primarily and resource allocation. Some argue that civil and political
responsible for guarding the borders with Nepal and rights are fundamental and must be prioritized, while others
Bhutan. emphasize the importance of economic, social, and cultural
rights for achieving social justice and equality.
65. Solution: (c)
Exp) Option c is the correct answer. 69. Solution: (c)

Article 350A of the Constitution of India states that it Exp) Option c is the correct answer
shall be the endeavor of every State and of every local The National Authority for Chemical Weapons Convention
authority within the State to provide adequate facilities (NACWC) has been set up as an office of the Cabinet
for instruction in the mother tongue at the primary stage Secretariat, Government of India in 1997 to fulfill, on
of education to children belonging to linguistic minority behalf of the Government of India, the obligations under the
groups. This provision aims to protect and promote the Chemical Weapons Convention and to act as the national
linguistic rights of minority communities by ensuring that focal point for effective liaison with the Organization for the
children have the opportunity to receive education in their Prohibition of Chemical Weapons (OPCW) and other State
mother tongue. Parties on matters relating to the Convention.

66. Solution: (c) 70. Solution: (a)


Exp) Option c is the correct answer. Exp) Option a is the correct answer
The following committees submitted reports on electoral The National Disaster Management Authority (NDMA) is
reforms in India: the apex statutory body for disaster management in India

319 PYQ Workbook


INDIAN POLITY AND GOVERNANCE

and is headed by the Prime Minister. As the head of the governs. He believed that individuals enter into a social
NDMA, the Prime Minister oversees the overall functioning contract with the government, willingly giving up some of
of the organization and provides strategic direction for their freedoms in exchange for the protection of their rights
disaster management initiatives. and the common good.
Important Tips Important Tips
• Mandate: Its primary purpose is to coordinate response • Views of J.S Mill on Democracy: Mill believed in
to natural or man-made disasters and for capacity- the importance of democracy as a means to protect
building in disaster resiliency and crisis response. It individual liberties and promote social progress. He
is also the apex body to lay down policies, plans and argued that democracy serves as a safeguard against
guidelines for Disaster Management to ensure timely the potential tyranny of the majority by providing
and effective response to disasters. mechanisms for the protection of individual rights and
• Vision: To build a safer and disaster resilient India by freedoms.
a holistic, proactive, technology driven and sustainable • Views of Jeremy Bentham on Democracy: He saw
development strategy that involves all stakeholders democracy as a means to achieve the greatest happiness for
and fosters a culture of prevention, preparedness and the greatest number of people and promote the public interest.
mitigation. Bentham favored representative democracy, where elected
• Divisions: NDMA has 5 major divisions viz. Policy & representatives make decisions on behalf of the people.
Plans, Mitigation, Operations & Communications • Views of J.J Rousseau on Democracy: According to
& Information & Technology, Administration and him, a true democracy operates when the general will
Finance. guides decision-making processes. Rousseau favored
direct democracy, where citizens actively participate
71. Solution: (c)
in decision-making rather than relying on elected
Exp) Option c is the correct answer representatives.
John Austin is widely credited with developing the theory
of Legal Positivism. According to Legal Positivism, the law 73. Solution: (b)
derives its authority from the recognition and acceptance Exp) Option b is the correct answer.
by the legal system rather than from any inherent moral or While Democratic Socialism, Communitarian Socialism, and
natural principles. Nehruvian Socialism represent different forms of socialism
Important Tips in India, Radical Socialism is not recognized as an Indian
Here are few features of legal positivism as explained by form of socialism.
John Austin: Democratic Socialism refers to a system that combines
• Separation of Law and Morality: According to legal democracy with socialist principles, Communitarian
positivism, the existence and validity of a law are not Socialism emphasizes community ownership and
dependent on its moral or ethical qualities. participation, and Nehruvian Socialism refers to the
• Command Theory of Law: Austin’s legal positivism is socialist policies and ideals advocated by India’s first Prime
based on the command theory of law, which suggests Minister, Jawaharlal Nehru.
that laws are essentially commands issued by a
sovereign to its subjects. 74. Solution: (b)

• Legal Positivism and Legal Certainty: By separating Exp) Option b is the correct answer.
law from morality, he aimed to create a clear and concise Totalitarianism seeks absolute control over all aspects
legal system based on explicit rules and commands, of public and private life. It aims to eliminate dissent and
enabling individuals to know their legal obligations. extend authority into personal domains. This ideology
• Criticism of Natural Law: He criticized natural law suppresses individual autonomy and freedom of expression.
concepts for being vague, subjective, and susceptible Totalitarian rule spans political, social, cultural, economic,
to varying interpretations. In contrast, legal positivism and ideological spheres.
sought to provide a more concrete and objective
foundation for the understanding and application of 75. Solution: (a)
law. Exp) Option a is the correct answer.
MPs do not personally sanction, execute, and complete works
72. Solution: (a) under the Members of Parliament Local Area Development
Exp) Option a is the correct answer Scheme (MPLADS). Their role is to recommend development
According to John Locke, democracy is a system of works to the District Authorities or District Collectors, who
government by consent because the authority of the are responsible for the implementation and execution of the
government is derived from the consent of the people it projects.

PYQ Workbook 320


INDIAN POLITY AND GOVERNANCE

Important Tips • The Delimitation Commission’s recommendations are


• MPLADS is a Central Sector Scheme introduced in binding and come into effect after they are published
December 1993. in the official gazette of the concerned state or
• MPs receive Rs. 5 crores annually in two instalments union territory. These recommendations determine
for MPLADS, and the funds are non-lapsable. the allocation of seats and help maintain equitable
representation in democratic elections.
• Lok Sabha MPs are required to recommend projects
to the district authorities within their respective Lok 78. Solution: (c)
Sabha constituencies, whereas Rajya Sabha MPs are Exp) Option c is the correct answer.
expected to allocate funds within the state that elected
them to the House. The Department for Promotion of Industry and Internal
Trade (DPIIT) is a department under the Ministry of
• Nominated Members of both the Rajya Sabha and
Commerce and Industry in the Government of India.
Lok Sabha have the authority to recommend works
anywhere in the country. It is responsible for formulating policies and promoting
industrial development, foreign investments, and internal
• MPLADS funds can be used for various schemes such
trade in the country. One of its key areas of focus is the
as Swachh Bharat Abhiyan, Accessible India Campaign,
protection and promotion of Intellectual Property Rights
water conservation, and Sansad Aadarsh Gram Yojana.
(IPRs).
The district authority must inspect a minimum of 10%
of works under implementation each year. 79. Solution: (c)
76. Solution: (a) Exp) Option c is the correct answer
Exp) Option a is the correct answer. While there are numerous political parties in India that
Statement 1 is correct - An overseas elector must be a citizen are based on regional interests and have a strong presence
of India to be eligible for voting in the Lok Sabha elections. in specific states or regions, the Election Commission
does not specifically classify them as “Regional Parties.”
Statement 2 is correct - The name of the overseas elector
The classification system mainly focuses on recognizing
must be included in the electoral roll of the constituency
parties at the national and state levels based on their
they would be voting from to be eligible to vote.
electoral performance and adherence to certain criteria.
Statement 3 is correct - Overseas Indian voters are required
to be physically present in India for voting, as e-voting is not 80. Solution: (a)
allowed. They have the options of postal ballots and proxy Exp) Option a is the correct answer
voting, with certain restrictions and eligibility criteria.
• This doctrine is considered as epitome of India’s foreign
77. Solution: (d) policy
Exp) Option d is the correct answer. • It was propounded by IK GUJRAL, the 12 th PM of India.
The First Delimitation Commission in India was constituted • The doctrine advocates that India, being the biggest
in 1952. The Delimitation Commission is responsible for country in South Asia, should extend unilateral
redrawing the boundaries of parliamentary and assembly concessions to the smaller neighbours.
constituencies based on population changes and demographic • In other words, the doctrine is formulated on India’s
shifts. accommodating approach towards its smaller neighbours
Important Tips on the basis of the principle of non-reciprocity.

• The Delimitation Commission is a statutory body in • It recognises the supreme importance of friendly and
India responsible for redrawing the boundaries of cordial relations with India’s neighbours.
parliamentary and assembly constituencies. 81. Solution: (a)
• The first Delimitation Commission was established in Exp) Option a is the correct answer
1952, and subsequent commissions have been formed
• Established in: 1948
when needed.
• Used for: immediate relief to families of those killed in
• The purpose of the Delimitation Commission is to
natural calamities like floods, cyclones and earthquakes,
ensure fair representation by redefining constituency
etc. and to the victims of the major accidents and riots
boundaries based on changes in population,
geographical factors, and other relevant considerations. • Headed by: Prime Minister
• The Delimitation Commission is appointed by the • Administration: Administered on an Honorary basis by
President of India and consists of a Chief Election Joint Secretary to the Prime Minister as Secretary of the
Commissioner or Election Commissioner as the fund.
ex-officio chairperson, along with other members • PMNRF operates from the Prime Minister’s Office
nominated by the President. (PMO) (option a is correct)

321 PYQ Workbook


INDIAN POLITY AND GOVERNANCE

Features: • Industrial design: An industrial design right is an


(a) Consists entirely of public contributions and does not get intellectual property right that protects the visual design
any budgetary support. of objects that are purely utilitarian.
(b) Recognized as a Trust under the Income Tax Act
• Trademark: A trademark is a type of intellectual property
(c) Exempt under the Income Tax Act, 1961 under Section
consisting of a recognizable sign, design, or expression
10 and 139 for return purposes.
that identifies products or services from a particular
82. Solution: (d) source and distinguishes them from others.
Exp) Option d is the correct answer.
85. Solution: (a)
International Human Rights Treaty consists of 9 core
treaties that are responsible for the international system for Exp) Option a is the correct answer.
the promotion and protection of human rights.
As per Art 395 of the constitution of India- The Indian
Important Tips Independence Act, 1947, and the Government of India
There are seven core international human rights treaties: Act, 1935, together with all enactments amending or
Treaty Date supplementing the latter Act, but not including the Abolition
International Convention on the Elimination 1965 of Privy Council Jurisdiction Act, 1949, are hereby repealed.
of All Forms of Racial Discrimination (ICERD)
86. Solution: (d)
International Covenant on Civil and Political 1966
Rights (ICCPR) Exp) Option d is the correct answer.
International Covenant on Economic, Social 1966 • Article 324 provides for the superintendence, direction
and Cultural Rights (ICESCR) and control of elections are vested in the Election
Convention on the Elimination of All Forms of 1979 Commission of India (ECI) which prepares and
Discrimination against Women (CEDAW) periodically revised one general electoral roll for every
Convention against Torture and Other 1984 territorial constituency.
Cruel, Inhuman or Degrading Treatment or
• It is based on the delimitation act of Parliament. The
Punishment (CAT)
Supreme Court has no authority to scrutinise the validity
Convention on the Rights of the Child (CRC) 1989
of this act. (statement d is incorrect)
International Convention on Protection of the 2003
Rights of All Migrant Workers and Members of Important Tips
Their Families (ICMRW) • The Election Commission of India (ECI) is a
constitutional body.
83. Solution: (c)
• It was established by the Constitution of India to
Exp) Option c is the correct answer. conduct and regulate elections in the country.
As per the ECI’s Political Parties and Election Symbols, 2019 • Article 324 of the Constitution provides that the power
handbook, a political party would be considered a national of superintendence, direction, and control of elections
party if : to parliament, state legislatures, the office of the
i. it is ‘recognised’ in four or more states; or president of India, and the office of vice-president of
ii. if its candidates polled at least 6% of total valid votes in India shall be vested in the election commission.
any four or more states in the last Lok Sabha or
87. Solution: (c)
iii. Assembly elections and has at least four MPs in the last
Lok Sabha polls; or Exp) Option c is the correct answer.
iv. If it has won at least 2% of the total seats in the Lok Sabha • The Panchsheel agreement is a set of principles that
from not less than three states. govern relation between states. It is also known as five
84. Solution: (a) principles of co-existence
Exp) Option a is the correct answer. • Their first formal codification in treaty form was in an
• Copyrights: A copyright exists from the moment the agreement between China and India on 29th April, 1954 .
work get created, so registration is voluntary hence it is
• An underlying assumption of the Five Principles was that
protected without making any registration
newly independent States after decolonisation would be
• Patent: A patent is used to prevent an invention from
being created sold or used by another party without able to develop a new and more principled approach to
permission International relations.

PYQ Workbook 322


INDIAN POLITY AND GOVERNANCE

Important Tips
The Panchsheel or ‘Five Principles of Peaceful Existence’ was a joint statement issued during Chinese premier Zhou Enlai’s
visits to India in 1954.
It was the period marking end of colonialism and emergence of new nations in Asia and Africa.
These five principles were as follows:
• Mutual respect for sovereignty and territorial integrity
• Mutual non-aggression
• Mutual non-interference in internal matters
• Equality and mutual benefit
• Peaceful co-existence

323 PYQ Workbook

You might also like